SP7 - General Insurance

Download as pdf or txt
Download as pdf or txt
You are on page 1of 1346

General Insurance Reserving

and Capital Modelling


Specialist Principles
Combined Materials Pack
for exams in 2019

The Actuarial Education Company


on behalf of the Institute and Faculty of Actuaries
All study material produced by ActEd is copyright and is sold for
the exclusive use of the purchaser. The copyright is owned by
Institute and Faculty Education Limited, a subsidiary of the
Institute and Faculty of Actuaries.

Unless prior authority is granted by ActEd, you may not hire out,
lend, give out, sell, store or transmit electronically or photocopy
any part of the study material.

You must take care of your study material to ensure that it is


not used or copied by anybody else.

Legal action will be taken if these terms are infringed. In


addition, we may seek to take disciplinary action through the
profession or through your employer.

These conditions remain in force after you have finished using


the course.

© IFE: 2019 Examinations The Actuarial Education Company


SP7: Study Guide Page 1

Subject SP7
2019 Study Guide
Introduction
This Study Guide has been created to help guide you through Subject SP7. It contains all the
information that you will need before starting to study Subject SP7 for the 2019 exams and you
may also find it useful to refer to throughout your Subject SP7 journey.

The guide is split into two parts:


 Part 1 contains general information about the Specialist Principles subjects
 Part 2 contains specific information about Subject SP7.

Please read this Study Guide carefully before reading the Course Notes, even if you have studied
for some actuarial exams before.

Contents

Part 1 Section 1 Before you start Page 2


Section 2 Core study material Page 3
Section 3 ActEd study support Page 5
Section 4 Study skills Page 10
Section 5 The examination Page 15
Section 6 Queries and feedback Page 16
Part 2 Section 1 Subject SP7 – background Page 17
Section 2 Subject SP7 – Syllabus and Core Reading Page 18
Section 3 Subject SP7 – the course structure Page 24
Section 4 Subject SP7 – summary of ActEd products Page 26
Section 5 Subject SP7 – skills and assessment Page 27
Section 6 Subject SP7 – frequently asked questions Page 28

The Actuarial Education Company © IFE: 2019 Examinations


Page 2 SP7: Study Guide

1.1 Before you start


When studying for the UK actuarial exams, you will need:
 a copy of the Formulae and Tables for Examinations of the Faculty of Actuaries and the
Institute of Actuaries, 2nd Edition (2002) – these are often referred to as simply the
Yellow Tables or the Tables
 a ‘permitted’ scientific calculator – you will find the list of permitted calculators on the
profession’s website. Please check the list carefully, since it is reviewed each year.

These are both available from the Institute and Faculty of Actuaries’ eShop. Please visit
www.actuaries.org.uk.

© IFE: 2019 Examinations The Actuarial Education Company


SP7: Study Guide Page 3

1.2 Core study material


This section explains the role of the Syllabus, Core Reading and supplementary ActEd text. It also
gives guidance on how to use these materials most effectively in order to pass the exam.

Some of the information below is also contained in the introduction to the Core Reading
produced by the Institute and Faculty of Actuaries.

Syllabus
The Syllabus for Subject SP7 has been produced by the Institute and Faculty of Actuaries. The
relevant individual Syllabus Objectives are included at the start of each course chapter and a
complete copy of the Syllabus is included in Section 2.2 of this Study Guide. We recommend that
you use the Syllabus as an important part of your study.

Core Reading
The Core Reading has been produced by the Institute and Faculty of Actuaries. The purpose of
the Core Reading is to ensure that tutors, students and examiners understand the requirements
of the syllabus for the qualification examinations for Fellowship of the Institute and Faculty of
Actuaries.

The Core Reading supports coverage of the Syllabus in helping to ensure that both depth and
breadth are re-enforced. It is therefore important that students have a good understanding of
the concepts covered by the Core Reading.

The examinations require students to demonstrate their understanding of the concepts given in
the syllabus and described in the Core Reading; this will be based on the legislation, professional
guidance etc that are in force when the Core Reading is published, ie on 31 May in the year
preceding the examinations.

Therefore the exams in April and September 2019 will be based on the Syllabus and Core Reading
as at 31 May 2018. We recommend that you always use the up-to-date Core Reading to prepare
for the exams.

Examiners will have this Core Reading when setting the papers. In preparing for examinations,
students are advised to work through past examination questions and may find additional tuition
helpful. The Core Reading will be updated each year to reflect changes in the syllabus and current
practice, and in the interest of clarity.

Accreditation

The Institute and Faculty of Actuaries would like to thank the numerous people who have helped
in the development of the material contained in this Core Reading.

The Actuarial Education Company © IFE: 2019 Examinations


Page 4 SP7: Study Guide

ActEd text
Core Reading deals with each syllabus objective and covers what is needed to pass the exam.
However, the tuition material that has been written by ActEd enhances it by giving examples and
further explanation of key points. Here is an excerpt from some ActEd Course Notes to show you
how to identify Core Reading and the ActEd material. Core Reading is shown in this bold font.

Note that in the example given above, the index will fall if the actual share price goes below the
theoretical ex-rights share price. Again, this is consistent with what would happen to an
underlying portfolio.
This is
After allowing for chain-linking, the formula for the investment index then becomes: ActEd
text
 Ni ,t Pi ,t
I (t )  i This is Core
B(t ) Reading

where N i ,t is the number of shares issued for the ith constituent at time t;

B(t ) is the base value, or divisor, at time t.

Copyright

All study material produced by ActEd is copyright and is sold for the exclusive use of the
purchaser. The copyright is owned by Institute and Faculty Education Limited, a subsidiary of the
Institute and Faculty of Actuaries. Unless prior authority is granted by ActEd, you may not hire
out, lend, give out, sell, store or transmit electronically or photocopy any part of the study
material. You must take care of your study material to ensure that it is not used or copied by
anybody else.

Legal action will be taken if these terms are infringed. In addition, we may seek to take
disciplinary action through the Institute and Faculty of Actuaries or through your employer.

These conditions remain in force after you have finished using the course.

© IFE: 2019 Examinations The Actuarial Education Company


SP7: Study Guide Page 5

1.3 ActEd study support


This section gives a description of the products offered by ActEd.

Successful students tend to undertake three main study activities:


1. Learning – initial study and understanding of subject material
2. Revision – learning subject material and preparing to tackle exam-style questions
3. Rehearsal – answering exam-style questions, culminating in answering questions at exam
speed without notes.

Different approaches suit different people. For example, you may like to learn material gradually
over the months running up to the exams or you may do your revision in a shorter period just
before the exams. Also, these three activities will almost certainly overlap.

We offer a flexible range of products to suit you and let you control your own learning and exam
preparation. The following table shows the products that we produce. Note that not all products
are available for all subjects.

LEARNING LEARNING & REVISION REVISION & REHEARSAL


REVISION REHEARSAL
Course Notes X Assignments Flashcards Revision Notes Mock Exam

Combined ASET Mock Exam


Materials Pack Marking
(CMP)

X Assignment
Marking

Tutorials

The products and services are described in more detail below.

The Actuarial Education Company © IFE: 2019 Examinations


Page 6 SP7: Study Guide

‘Learning’ products
Course Notes

The Course Notes will help you develop the basic knowledge and understanding of principles
needed to pass the exam. They incorporate the complete Core Reading and include full
explanation of all the syllabus objectives, with worked examples and questions (including some
past exam questions) to test your understanding.

Each chapter includes:


 the relevant syllabus objectives
 a chapter summary
 practice questions with full solutions.

‘Learning & revision’ products


X Assignments

The Series X Assignments are written assessments that cover the material in each part of the
course in turn. They can be used to both develop and test your understanding of the material.

Combined Materials Pack (CMP)

The Combined Materials Pack (CMP) comprises the Course Notes and the Series X Assignments.

The CMP is available in eBook format for viewing on a range of electronic devices. eBooks can be
ordered separately or as an addition to paper products. Visit www.ActEd.co.uk for full details
about the eBooks that are available, compatibility with different devices, software requirements
and printing restrictions.

X Assignment Marking

We are happy to mark your attempts at the X assignments. Marking is not included with the
Assignments or the CMP and you need to order it separately. You should submit your script as a
PDF attached to an email. Your script will be marked electronically and you will be able to download
your marked script via a secure link on the internet.

Don’t underestimate the benefits of doing and submitting assignments:


 Question practice during this phase of your study gives an early focus on the end goal of
answering exam-style questions.
 You’re incentivised to keep up with your study plan and get a regular, realistic assessment
of your progress.
 Objective, personalised feedback from a high quality marker will highlight areas on which
to work and help with exam technique.

In a recent study, we found that students who attempt more than half the assignments have
significantly higher pass rates.

© IFE: 2019 Examinations The Actuarial Education Company


SP7: Study Guide Page 7

There are two different types of marking product: Series Marking and Marking Vouchers.

Series Marking

Series Marking applies to a specified subject, session and student. If you purchase Series Marking,
you will not be able to defer the marking to a future exam sitting or transfer it to a different subject
or student.

We typically send out full solutions with the Series X Assignments. However, if you order Series
Marking at the same time as you order the Series X Assignments, you can choose whether or not
to receive a copy of the solutions in advance. If you choose not to receive them with the study
material, you will be able to download the solutions via a secure link on the internet when your
marked script is returned (or following the final deadline date if you do not submit a script).

If you are having your attempts at the assignments marked by ActEd, you should submit your scripts
regularly throughout the session, in accordance with the schedule of recommended dates set out in
information provided with the assignments. This will help you to pace your study throughout the
session and leave an adequate amount of time for revision and question practice.

The recommended submission dates are realistic targets for the majority of students. Your scripts
will be returned more quickly if you submit them well before the final deadline dates.

Any script submitted after the relevant final deadline date will not be marked. It is your
responsibility to ensure that we receive scripts in good time.

Marking Vouchers

Marking Vouchers give the holder the right to submit a script for marking at any time, irrespective of
the individual assignment deadlines, study session, subject or person.

Marking Vouchers can be used for any assignment. They are valid for four years from the date of
purchase and can be refunded at any time up to the expiry date.

Although you may submit your script with a Marking Voucher at any time, you will need to adhere
to the explicit Marking Voucher deadline dates to ensure that your script is returned before the date
of the exam. The deadline dates are provided with the assignments.

Tutorials

Our tutorials are specifically designed to develop the knowledge that you will acquire from the
course material into the higher-level understanding that is needed to pass the exam.

We run a range of different tutorials including face-to-face tutorials at various locations, and Live
Online tutorials. Full details are set out in our Tuition Bulletin, which is available on our website at
www.ActEd.co.uk.

Regular and Block Tutorials

In preparation for these tutorials, we expect you to have read the relevant part(s) of the Course
Notes before attending the tutorial so that the group can spend time on exam questions and
discussion to develop understanding rather than basic bookwork.

The Actuarial Education Company © IFE: 2019 Examinations


Page 8 SP7: Study Guide

You can choose one of the following types of tutorial:


 Regular Tutorials spread over the session.
 A Block Tutorial held two to eight weeks before the exam.

‘Revision’ products
For most subjects, there is a lot of material to revise. Finding a way to fit revision into your
routine as painlessly as possible has got to be a good strategy. Flashcards are an inexpensive
option that can provide a massive boost. They can also provide a variation in activities during a
study day, and so help you to maintain concentration and effectiveness.

Flashcards

Flashcards are a set of A6-sized cards that cover the key points of the subject that most students
want to commit to memory. Each flashcard has questions on one side and the answers on the
reverse. We recommend that you use the cards actively and test yourself as you go.

Flashcards are available in eBook format for viewing on a range of electronic devices. eBooks can
be ordered separately or as an addition to paper products. Visit www.ActEd.co.uk for full details
about the eBooks that are available, compatibility with different devices, software requirements
and printing restrictions.

The following questions and comments might help you to decide if flashcards are suitable for you:

Flashcards

 Do you have a regular train or bus journey?


Flashcards are ideal for regular bursts of revision on the move.

 Do you want to fit more study into your routine?


Flashcards are a good option for ‘dead time’, eg using flashcards on your phone or sticking
them on the wall in your study.

 Do you find yourself cramming for exams (even if that’s not your original plan)?
Flashcards are an extremely efficient way to do your pre-exam memorising.

If you are retaking a subject, then you might consider using flashcards if you didn’t use them on a
previous attempt.

‘Revision & rehearsal’ products


Revision Notes

Our Revision Notes have been designed with input from students to help you revise efficiently.
They are suitable for first-time sitters who have worked through the ActEd Course Notes or for
retakers (who should find them much more useful and challenging than simply reading through
the course again).

© IFE: 2019 Examinations The Actuarial Education Company


SP7: Study Guide Page 9

The Revision Notes are a set of A5 booklets – perfect for revising on the train or tube to work.
Each booklet covers one main theme or a set of related topics from the course and includes:
 Core Reading with a set of integrated short questions to develop your bookwork
knowledge
 relevant past exam questions with concise solutions from the last ten years
 other useful revision aids.

ActEd Solutions with Exam Technique (ASET)

The ActEd Solutions with Exam Technique (ASET) contains our solutions to eight past exam
papers, plus comment and explanation. In particular, it highlights how questions might have been
analysed and interpreted so as to produce a good solution with a wide range of relevant points.
This will be valuable in approaching questions in subsequent examinations.

‘Rehearsal’ products
Mock Exam

The Mock Exam is a 100-mark mock exam paper that provides a realistic test of your exam
preparation.

Mock Marking

We are happy to mark your attempts at the mock exam. The same general principles apply as for
the X Assignment Marking. In particular:
 Mock Exam Marking is available for the Mock Exam and it applies to a specified subject,
session and student
 Marking Vouchers can be used for the Mock Exam.

Recall that:
 marking is not included with the products themselves and you need to order it separately
 you should submit your script as a PDF attached to an email
 your script will be marked electronically and you will be able to download your marked
script via a secure link on the internet.

The Actuarial Education Company © IFE: 2019 Examinations


Page 10 SP7: Study Guide

1.4 Skills

The Subject SP exams


It is important to recognise that the SP subject exams are very different from the Core Principles
subject exams in both the nature of the material covered and the skills being examined.

Both the Core Reading and the exam papers themselves are generally much less numerical and
more ‘wordy’ than the Core Principles subjects. The exam will primarily require you to explain a
particular point in words and sentences, rather than to manipulate formulae or do calculations.
Numerical questions typically account for only a small part of each exam paper. If you haven’t sat
this type of exam for some time, you need to start practising again now. Many students find that
it takes time to adjust to the different style of the SP subject exam questions. As ever, practice is
the key to success.

The aim of the exams is to test your ability to apply your knowledge and understanding of the key
principles described in the Core Reading to specific situations presented to you in the form of
exam questions. Therefore your aim should be to identify and understand the key principles, and
then to practise applying them. You will also need a good knowledge of the Core Reading to score
well and quickly on any bookwork questions.

Study skills
Overall study plan

We suggest that you develop a realistic study plan, building in time for relaxation and allowing
some time for contingencies. Be aware of busy times at work, when you may not be able to take
as much study leave as you would like. Once you have set your plan, be determined to stick to it.
You don’t have to be too prescriptive at this stage about what precisely you do on each study day.
The main thing is to be clear that you will cover all the important activities in an appropriate
manner and leave plenty of time for revision and question practice.

Aim to manage your study so as to allow plenty of time for the concepts you meet in this course
to ‘bed down’ in your mind. Most successful students will probably aim to complete the course at
least six weeks before the exam, thereby leaving a sufficient amount of time for revision. By
finishing the course as quickly as possible, you will have a much clearer view of the big picture. It
will also allow you to structure your revision so that you can concentrate on the important and
difficult areas of the course.

You can also try looking at our discussion forum on the internet, which can be accessed at
www.ActEd.co.uk/forums (or use the link from our home page at www.ActEd.co.uk). There are
some good suggestions from students on how to study.

Study sessions

Only do activities that will increase your chance of passing. Try to avoid including activities for the
sake of it and don’t spend time reviewing material that you already understand. You will only
improve your chances of passing the exam by getting on top of the material that you currently
find difficult.

© IFE: 2019 Examinations The Actuarial Education Company


SP7: Study Guide Page 11

In particular, you may already be familiar with the content of some of the chapters (from the Core
Principles (CS, CM or CB subjects), Subject CP1 or other SP subjects). Try to cover these chapters
quickly to give yourself more time on the material with which you are less comfortable. Where
chapters refer back to material from the Core Principles subjects, you don’t have to follow these
links up unless you are feeling curious or clueless.

Ideally, each study session should have a specific purpose and be based on a specific task,
eg ’Finish reading Chapter 3 and attempt Practice Questions 3.4, 3.7 and 3.12 ’, as opposed to a
specific amount of time, eg ‘Three hours studying the material in Chapter 3’.

Try to study somewhere quiet and free from distractions (eg a library or a desk at home dedicated
to study). Find out when you operate at your peak, and endeavour to study at those times of the
day. This might be between 8am and 10am or could be in the evening. Take short breaks during
your study to remain focused – it’s definitely time for a short break if you find that your brain is
tired and that your concentration has started to drift from the information in front of you.

Order of study

We suggest that you work through each of the chapters in turn. To get the maximum benefit from
each chapter you should proceed in the following order:
1. Read the Syllabus Objectives. These are set out in the box at the start of each chapter.
2. Read the Chapter Summary at the end of each chapter. This will give you a useful overview
of the material that you are about to study and help you to appreciate the context of the
ideas that you meet.
3. Study the Course Notes in detail, annotating them and possibly making your own notes. Try
the self-assessment questions as you come to them. As you study, pay particular attention
to the listing of the Syllabus Objectives and to the Core Reading.

4. Read the Chapter Summary again carefully. If there are any ideas that you can’t
remember covering in the Course Notes, read the relevant section of the notes again to
refresh your memory.

5. Attempt (at least some of) the Practice Questions that appear at the end of the chapter.

It’s a fact that people are more likely to remember something if they review it several times. So,
do look over the chapters you have studied so far from time to time. It is useful to re-read the
Chapter Summaries or to try the Practice Questions again a few days after reading the chapter
itself. It’s a good idea to annotate the questions with details of when you attempted each one. This
makes it easier to ensure that you try all of the questions as part of your revision without repeating
any that you got right first time.

Once you’ve read the relevant part of the notes and tried a selection of questions from the
Practice Questions (and attended a tutorial, if appropriate) you should attempt the corresponding
assignment. If you submit your assignment for marking, spend some time looking through it
carefully when it is returned. It can seem a bit depressing to analyse the errors you made, but
you will increase your chances of passing the exam by learning from your mistakes. The markers
will try their best to provide practical comments to help you to improve.

The Actuarial Education Company © IFE: 2019 Examinations


Page 12 SP7: Study Guide

To be really prepared for the exam, you should not only know and understand the Core Reading but
also be aware of what the examiners will expect. Your revision programme should include plenty of
question practice so that you are aware of the typical style, content and marking structure of exam
questions. You should attempt as many past exam questions as you can.

Active study

Here are some techniques that may help you to study actively.
1. Don’t believe everything you read. Good students tend to question everything that they
read. They will ask ‘why, how, what for, when?’ when confronted with a new concept,
and they will apply their own judgement. This contrasts with those who unquestioningly
believe what they are told, learn it thoroughly, and reproduce it (unquestioningly?) in
response to exam questions.
2. Another useful technique as you read the Course Notes is to think of possible questions
that the examiners could ask. This will help you to understand the examiners’ point of
view and should mean that there are fewer nasty surprises in the exam room. Use the
Syllabus to help you make up questions.
3. Annotate your notes with your own ideas and questions. This will make you study more
actively and will help when you come to review and revise the material. Do not simply
copy out the notes without thinking about the issues.
4. As you study each chapter, condense the key points (not whole chunks of text) on to a
double side of A4 or less. This is essential as otherwise, when you come to revision, you
will end up having to re-read the whole course again, and there won’t be time.

5. Try to use memory aids, such as mind maps and acronyms, to help remember the material
when you come back to it later.

6. Attempt the questions in the notes as you work through the course. Write down your
answer before you refer to the solution.

7. Attempt other questions and assignments on a similar basis, ie write down your answer
before looking at the solution provided. Attempting the assignments under exam
conditions has some particular benefits:
 It forces you to think and act in a way that is similar to how you will behave in the
exam.
 When you have your assignments marked it is much more useful if the marker’s
comments can show you how to improve your performance under exam conditions
than your performance when you have access to the notes and are under no time
pressure.
 The knowledge that you are going to do an assignment under exam conditions and
then submit it (however good or bad) for marking can act as a powerful incentive to
make you study each part as well as possible.
 It is also quicker than trying to write perfect answers.

© IFE: 2019 Examinations The Actuarial Education Company


SP7: Study Guide Page 13

8. Sit a mock exam four to six weeks before the real exam to identify your weaknesses and
work to improve them. You could use a mock exam written by ActEd or a past exam
paper.

You can find further information on how to study in the profession’s Student Handbook, which
you can download from their website at:

www.actuaries.org.uk/studying

Revision and exam skills


Revision skills

You will have sat many exams before and will have mastered the exam and revision techniques
that suit you. However it is important to note that due to the high volume of work involved in
Subject SP7, it is not possible to leave all your revision to the last minute. Students who prepare
well in advance have a better chance of passing the exam on the first sitting.

We recommend that you prepare for the exam by practising a large number of exam-style
questions under exam conditions. This will:
 help you to develop the necessary knowledge and understanding of the key principles
described in the Core Reading
 highlight exactly which are the key principles that crop up time and time again in many
different contexts and questions
 help you to practise the specific skills that you will need to pass the exam.

There are many sources of exam-style questions. You can use past exam papers, the Practice
Questions at the end of each chapter (which include many past exam questions), assignments,
mock exams, the Revision Notes and ASET.

Exam question skill levels

Exam questions are not designed to be of similar difficulty. The Institute and Faculty of Actuaries
specifies different skill levels that questions may be set with reference to.

Questions may be set at any skill level:


 Knowledge – demonstration of a detailed knowledge and understanding of the topic
 Application – demonstration of an ability to apply the principles underlying the topic
within a given context
 Higher Order – demonstration of an ability to perform deeper analysis and assessment of
situations, including forming judgements, taking into account different points of view,
comparing and contrasting situations, suggesting possible solutions and actions, and
making recommendations.

The Actuarial Education Company © IFE: 2019 Examinations


Page 14 SP7: Study Guide

Command verbs

The Institute and Faculty of Actuaries use command verbs (such as ‘Define’, ‘Discuss’ and
‘Explain’) to help students to identify what the question requires. The profession has produced a
document, ‘Command verbs used in the Associate and Fellowship written examinations’, to help
students to understand what each command verb is asking them to do.

It also gives the following advice:


 The use of a specific command verb within a syllabus objective does not indicate that this
is the only form of question which can be asked on the topic covered by that objective.
 The Examiners may ask a question on any syllabus topic using any of the agreed command
verbs, as are defined in the document.

You can find the relevant document on the profession’s website at:

https://www.actuaries.org.uk/studying/prepare-your-exams

© IFE: 2019 Examinations The Actuarial Education Company


SP7: Study Guide Page 15

1.5 The examination

What to take to the exam


IMPORTANT NOTE: The following information was correct at the time of printing, however it is
important to keep up-to-date with any changes. See the profession’s website for the latest
guidance.

For the written exam, the examination room will be equipped with:
 the question paper
 an answer booklet
 rough paper
 a copy of the Yellow Tables.

Remember to take with you:


 black pens
 a permitted scientific calculator – please refer to www.actuaries.org.uk for the latest
advice.

Past exam papers


You can download some past exam papers and Examiners’ Reports from the profession’s website
at www.actuaries.org.uk. However, please be aware that these exam papers are for the
pre-2019 syllabus and not all questions will be relevant.

The Actuarial Education Company © IFE: 2019 Examinations


Page 16 SP7: Study Guide

1.6 Queries and feedback

Questions and queries


From time to time you may come across something in the study material that is unclear to you.
The easiest way to solve such problems is often through discussion with friends, colleagues and
peers – they will probably have had similar experiences whilst studying. If there’s no-one at work
to talk to then use our discussion forum at www.ActEd.co.uk/forums (or use the link from our
home page at www.ActEd.co.uk).

Our online forum is dedicated to actuarial students so that you can get help from fellow students
on any aspect of your studies from technical issues to study advice. You could also use it to get
ideas for revision or for further reading around the subject that you are studying. ActEd tutors
will visit the site from time to time to ensure that you are not being led astray and we also post
other frequently asked questions from students on the forum as they arise.

If you are still stuck, then you can send queries by email to the relevant subject email address (see
Section 2.6), but we recommend that you try the forum first. We will endeavour to contact you as
soon as possible after receiving your query but you should be aware that it may take some time to
reply to queries, particularly when tutors are away from the office running tutorials. At the
busiest teaching times of year, it may take us more than a week to get back to you.

If you have many queries on the course material, you should raise them at a tutorial or book a
personal tuition session with an ActEd tutor. Information about personal tuition is set out in our
current brochure. Please email ActEd@bpp.com for more details.

Feedback
If you find an error in the course, please check the corrections page of our website
(www.ActEd.co.uk/paper_corrections.html) to see if the correction has already been dealt with.
Otherwise please send details via email to the relevant subject email address (see Section 2.6).

Each year our tutors work hard to improve the quality of the study material and to ensure that
the courses are as clear as possible and free from errors. We are always happy to receive
feedback from students, particularly details concerning any errors, contradictions or unclear
statements in the courses. If you have any comments on this course please email them to the
relevant subject email address (see Section 2.6).

Our tutors also work with the profession to suggest developments and improvements to the
Syllabus and Core Reading. If you have any comments or concerns about the Syllabus or Core
Reading, these can be passed on via ActEd. Alternatively, you can send them directly to the
Institute and Faculty of Actuaries’ Examination Team by email to
education.services@actuaries.org.uk.

© IFE: 2019 Examinations The Actuarial Education Company


SP7: Study Guide Page 17

2.1 Subject SP7 – background

History
The Specialist Principles subjects are new subjects in the Institute and Faculty of Actuaries 2019
Curriculum.

Subject SP7 is General Insurance Reserving and Capital Modelling Principles.

Predecessors
The Specialist Principles subjects cover content that was previously in the equivalent Specialist
Technical subjects. So:
 Subject SP7 replaces Subject ST7.

Exemptions
You will need to have passed or been granted an exemption from Subject ST7 to be eligible for a
pass in Subject SP7 during the transfer process.

Links to other subjects


 Subject CS2 – Risk Modelling and Survival Analysis.
 Subject CP1 – Actuarial Practice.
 Subject SP8 – General Insurance: Pricing Principles.
 Subject SA3 – General Insurance Advanced.

The Actuarial Education Company © IFE: 2019 Examinations


Page 18 SP7: Study Guide

2.2 Subject SP7 – Syllabus and Core Reading

Syllabus
The Syllabus for Subject SP7 is given here. To the right of each objective are the chapter numbers
in which the objective is covered in the ActEd course.

Aim

The aim of this General Insurance Reserving and Capital Modelling Principles subject is to instil in
successful candidates the ability to apply, in simple reserving and capital modelling situations, the
mathematical and economic techniques and the principles of actuarial planning and control
needed for the operation on sound financial lines of general insurers.

Competences

On successful completion of this subject, a student will be able to:


1. understand the main principles and techniques of reserving and technical modelling that
are relevant to general insurance
2. apply these principles to simple situations within the context of general insurance
3. analyse hypothetical scenarios, including using judgement to assess the implications of
possible actions and to develop appropriate proposals or recommendations relating to
reserving and capital modelling in general insurance business.

Syllabus topics

1. General insurance products and general business environment (20%)

2. Risk, uncertainty and regulation (15%)

3. Reserving (30%)

4. Capital modelling (17.5%)

5. Data, investigations, reinsurance and accounting (17.5%)

The weightings are indicative of the approximate balance of the assessment of this subject
between the main syllabus topics, averaged over a number of examination sessions.

The weightings also have a correspondence with the amount of learning material underlying each
syllabus topic. However, this will also reflect aspects such as:
 the relative complexity of each topic, and hence the amount of explanation and support
required for it
 the need to provide thorough foundation understanding on which to build the other
objectives
 the extent of prior knowledge which is expected
 the degree to which each topic area is more knowledge or application based.

© IFE: 2019 Examinations The Actuarial Education Company


SP7: Study Guide Page 19

Detailed syllabus objectives

Introduction

Define the principal terms in use in general insurance. (Glossary)

1. General insurance products and general business environment (20%)

1.1 Describe the main types of general insurance product in terms


of: (Chapters 2 and 3)

 the needs of customers


 the financial and other risks they pose for the general insurer including their
capital requirements and possible effect on solvency.

1.2 Describe the main types of general reinsurance products and the purposes for
which they may be used. (Chapters 5 and 6)

1.3 Describe the implications of the general business environment in terms of:
(Chapters 7 and 9)

 the main features of the general insurance market


 the effect of different marketing strategies
 the effect of fiscal regimes
 the effect of inflation and economic factors
 the effect of legal, political and social factors
 the effect of the climate and environmental factors
 the general effect of professional guidance
 the impact of technological change.

1.4 Outline the key features of the Lloyd’s market. (Chapter 10)

2. Risk, uncertainty and regulation (15%)

2.1 Describe the major areas of risk and uncertainty in general insurance business
with respect to reserving and capital modelling, in particular those that might
threaten profitability or solvency. (Chapter 11)

2.2 Discuss the purposes of regulating general insurance business. (Chapter 8)

2.3 Outline possible methods by which general insurers can be regulated, including
advantages and drawbacks of each. (Chapter 8)

3 Reserving (30%)

3.1 With regard to reserving work using triangulations: (Chapter 15)

 Analyse the range of general issues that can affect reserving work using
triangulations.
 Identify how to deal with these general issues in reserving work.
 Evaluate the main triangulation methods in use – namely the chain ladder
method, the Bornhuetter-Ferguson method and the Average Cost per Claim
method.

The Actuarial Education Company © IFE: 2019 Examinations


Page 20 SP7: Study Guide

3.2 Suggest appropriate reserving bases for general insurance business, having regard
to: (Chapter 14)

 the different reasons for calculating reserves


 the assumptions that might be appropriate in each case
 why the assumptions may differ from a rating exercise
 the allowance for future inflation
 whether or not to discount for investment income
 the approach for additional unexpired risk reserve
 communication of the reserving basis.

3.3 Evaluate stochastic reserving processes. (Chapter 16)

3.3.1 Describe the uses of stochastic reserving methods.

3.3.2 Describe the likely sources of reserving uncertainty.

3.3.3 Describe the following types of stochastic reserving methods:

 analytic methods
 simulation-based methods.

3.3.4 Describe the Mack and Bootstrapping approaches to reserving.

3.3.5 Describe the issues, advantages and disadvantages of each of the methods.

3.3.6 Describe the approach to aggregating the results of stochastic reserving across
multiple lines of business, and discuss methods of correlation.

3.4 Evaluate reserving result analyses. (Chapter 17)

3.4.1 Describe the factors an actuary should consider in assessing the reasonableness of
the results of a reserving exercise.

3.4.2 Describe typical diagnostics that are commonly used to assess the reasonableness
of the results of a reserving exercise.

3.4.3 Describe the factors an actuary should consider in assessing the reasonableness of
changes in results of a reserving exercise over time.

3.4.4 Describe how an analysis of experience might be carried out in the context of a
reserving exercise.

3.4.5 Describe how alternative results of reserving exercises can arise and highlight
some of the professional issues in resolving them.

3.5 Assess uncertainty and its communication in reserving. (Chapter 18)

3.5.1 Discuss what is meant by a ‘best estimate’ reserve.

© IFE: 2019 Examinations The Actuarial Education Company


SP7: Study Guide Page 21

3.5.2 Describe the following approaches to estimating ranges of reserves:

 stochastic models
 scenario tests
 use of alternative sets of assumptions.

3.5.3 Discuss the uses, advantages and disadvantages of each of these methods.

3.5.4 Discuss the issues to be considered when communicating reserve ranges and
uncertainties.

4. Capital modelling (17.5%)

4.1 Evaluate the key considerations in deriving and applying capital modelling
techniques. (Chapter 20)

4.2 Evaluate the following approaches to capital modelling: (Chapter 20)

 deterministic models
 stochastic models.

4.3 Discuss the following issues with regard to parameterisation of capital models:
(Chapter 20)

 developing assumptions
 validation.

4.4 Describe approaches to the assessment of capital requirements for the following
risk types: (Chapter 21)

 insurance risk
 market risk
 credit risk
 operational risk
 liquidity risk
 group risk.

4.5 Explain some of the areas to consider when approaching a capital modelling
exercise. (Chapter 22)

4.6 Describe the practical considerations which should be borne in mind when
undertaking capital modelling. (Chapter 23)

5. Data, investigations, reinsurance and accounting (17.5%)

5.1 With regard to the use of data in reserving and capital modelling: (Chapter 12)

 describe the types of data that are used


 describe the main uses of data
 describe the requirements for a good information system
 outline the possible causes of data errors
 analyse the effects of inadequate data.

The Actuarial Education Company © IFE: 2019 Examinations


Page 22 SP7: Study Guide

5.2 Outline the major actuarial investigations and analyses of experience undertaken
with regard to reserving and capital modelling for general insurers. (Chapter 13)

5.3 Describe the factors influencing the choice of an appropriate reinsurance


programme for a general insurer. (Chapter 24)

5.4 Describe how to test the appropriateness of alternative reinsurance structures for
a general insurer. (Chapter 24)

5.5 Describe how reinsurance purchasing decisions might be impacted by capital


management considerations. (Chapter 24)

5.6 Describe the following approaches to reserving for outwards reinsurance:


(Chapter 25)

 gross less net


 application of standard techniques to reinsurance data
 use of appropriate factors
 application of detailed contract terms.

5.7 Compare the advantages and disadvantages of each of the above methods and
the appropriate circumstances in which to use them. (Chapter 25)

5.8 Discuss suitable approaches to reserving for inwards reinsurance. (Chapter 25)

5.9 Describe, within the context of investment and ALM: (Chapter 19)

 the principles of investment


 the asset-liability matching requirements of a general insurer
 how projection models might be used to develop an appropriate investment
strategy.

5.10 Describe the methods and principles of accounting for general insurance business
and interpret the accounts of a general insurer. (Chapters 26 and 27)

5.11 Describe the changes to accounting methods expected under IFRS.


(Chapters 26 and 27)

6. Solving problems (Chapter 4)

6.1 Analyse hypothetical examples and scenarios in relation to the financial


management of general insurance companies.

6.1.1 Propose solutions and actions that are appropriate to the given context, with
justification where required.

6.1.2 Suggest possible reasons why certain actions have been chosen.

6.1.3 Assess the implications of actions within a given scenario.

6.1.4 Discuss the advantages and disadvantages of suggested actions, taking into
account different perspectives.

© IFE: 2019 Examinations The Actuarial Education Company


SP7: Study Guide Page 23

Core Reading
The Subject SP7 Course Notes include the Core Reading in full, integrated throughout the course.

The Actuarial Education Company © IFE: 2019 Examinations


Page 24 SP7: Study Guide

2.3 Subject SP7 – the course structure


There are six parts to the Subject SP7 course. The parts cover related topics and have broadly
equal marks in the exam. The parts are broken down into chapters.

The following table shows how the parts, the chapters and the syllabus items relate to each other.
The end columns show how the chapters relate to the days of the regular tutorials. We have also
given you a broad indication of the length of each chapter. This table should help you plan your
progress across the study session.

© IFE: 2019 Examinations The Actuarial Education Company


SP7: Study Guide Page 25

No of Syllabus 3 full
Part Chapter Title
pages objectives days
1 Insurance companies 42
2 Insurance products – background 40 1.1
3 Insurance products – types 68 1.1
4 Problem solving 20 6
5 Reinsurance products – background 29 1.2
1
6 Reinsurance products – types 68 1.2
7 General insurance markets 32 1.3 1
8 Regulation 27 2.2, 2.3
9 External environment 52 1.3
10 The Lloyd’s market 26 1.4
11 Risk and uncertainty 39 2.1
2 12 Data 59 5.1
13 Actuarial investigations 28 5.2
14 Reserving bases 25 3.2
3
15 Triangulation methods 119 3.1
16 Stochastic reserving models 44 3.3
2
17 Assessment of reserving results 33 3.4
4
18 Reserving – use of ranges and best estimates 31 3.5
19 Investment principles and ALM 48 5.9
20 Capital modelling – methodologies 50 4.1–4.3
21 Capital modelling – assessment of capital for 56 4.4
various risk types
5
22 Capital modelling – allowance for 23 4.5
diversification
23 Capital modelling – practical considerations 21 4.6 3
24 Determining appropriate reinsurance 35 5.3–5.5
25 Reinsurance reserving 29 5.6–5.8
6 26 Accounting methods 40 5.10, 5.11
27 Interpreting accounts 48 5.10, 5.11
Glossary 42 Introduction

The Actuarial Education Company © IFE: 2019 Examinations


Page 26 SP7: Study Guide

2.4 Subject SP7 – summary of ActEd products


The following products are available for Subject SP7:
 Course Notes
 X Assignments – six assignments:
– X1-X3: 80-mark tests (you are allowed 2¾ hours to complete these)
– X4-X6: 100-mark tests (you are allowed 3¼ hours to complete these)
and X Assignment marking (Series Marking and Marking Vouchers)
 Flashcards
 Revision Notes – seven A5 booklets

 ASET – four years’ exam papers, ie eight papers, covering the period April 2014 to
September 2017
 Mock Exam and marking (Series Marking and Marking Vouchers).

We will endeavour to release as much material as possible but unfortunately some revision
products may not be available until the September 2019 or even April 2020 exam sessions.
Please check the ActEd website or email ActEd@bpp.com for more information.

The following tutorials are typically available for Subject SP7:


 regular tutorials (three days)
 block tutorials (three days).

Full details are set out in our Tuition Bulletin, which is available on our website at
www.ActEd.co.uk.

© IFE: 2019 Examinations The Actuarial Education Company


SP7: Study Guide Page 27

2.5 Subject SP7 – skills and assessment

Exam skills
Exam question skill levels

In the SP subjects, the approximate split of assessment across the three skill types is:
 Knowledge – 25%
 Application – 50%
 Higher Order skills – 25%.

Assessment
The Specialist Principles examinations are in the form of 3¼-hour paper-based examinations.

The Actuarial Education Company © IFE: 2019 Examinations


Page 28 SP7: Study Guide

2.6 Subject SP7 – frequently asked questions


Q: What knowledge of earlier subjects should I have?

A: The Course Notes have been written assuming that you have already studied, or been
exempted from, the Core Principles subjects (the CS, CM and CB subjects) or the equivalent
CT subjects.

The key material that you will need in studying Subject SP7 is that in Subjects CM2, CS1 and
CS2 (or the previous Subject CT6).

Q: What is your advice if I am simultaneously studying Subject SP8?

A: There is a fair amount of duplication between Subjects SP7 and SP8, particularly in the
early part of the course. We suggest you review the material in both subjects
simultaneously.

Q: What is your advice if I am simultaneously studying Subject SA3?

A: Subject SA3 builds on the common principles developed in Subjects SP7 and SP8, but
requires a much greater depth of knowledge and understanding. Consequently, there is a
degree of overlap between the subjects – both in the Core Reading and also possibly in
the types of questions that are likely to appear on the exam papers. It is therefore
important to assimilate the key ideas presented in Subjects SP7 and SP8 before tackling
the same ground in Subject SA3.

We suggest that you aim to cover the Subject SP7 and SP8 courses as quickly as possible,
so as to get a general feel for the principles underlying general insurance, together with
an overview of the course content. It also makes sense to quickly review the relevant
Subject SP7 and SP8 material prior to working through each chapter in Subject SA3.

From time to time over the study session, and particularly at the revision stage, it might
also be a good idea to review the Subjects SP7, SP8 & SA3 Course Notes at the same time,
along with the Practice Questions. In particular, it is always worth thinking about how
each idea or principle is presented in each of SP7, SP8 and SA3 and hence how it might
consequently be examined any exam.

Q: What should I do if I discover an error in the course?

A: If you find an error in the course, please check our website at:

www.ActEd.co.uk/paper_corrections.html

to see if the correction has already been dealt with. Otherwise please send details via
email to SP7@bpp.com.

© IFE: 2019 Examinations The Actuarial Education Company


SP7: Study Guide Page 29

Q: Who should I send feedback to?

A: We are always happy to receive feedback from students, particularly details concerning
any errors, contradictions or unclear statements in the courses.

If you have any comments on this course in general, please email to SP7@bpp.com.

If you have any comments or concerns about the Syllabus or Core Reading, these can be
passed on to the profession via ActEd. Alternatively, you can send them directly to the
Institute and Faculty of Actuaries’ Examination Team by email to
education.services@actuaries.org.uk.

The Actuarial Education Company © IFE: 2019 Examinations


All study material produced by ActEd is copyright and is sold
for the exclusive use of the purchaser. The copyright is
owned by Institute and Faculty Education Limited, a
subsidiary of the Institute and Faculty of Actuaries.

Unless prior authority is granted by ActEd, you may not hire


out, lend, give out, sell, store or transmit electronically or
photocopy any part of the study material.

You must take care of your study material to ensure that it


is not used or copied by anybody else.

Legal action will be taken if these terms are infringed. In


addition, we may seek to take disciplinary action through
the profession or through your employer.

These conditions remain in force after you have finished


using the course.

The Actuarial Education Company © IFE: 2019 Examinations


SP7-01: Insurance companies Page 1

Insurance companies
Syllabus objectives
There are no syllabus objectives specifically covered in this introductory chapter.

The Actuarial Education Company © IFE: 2019 Examinations


Page 2 SP7-01: Insurance companies

0 Introduction to Subject SP7


This chapter aims to give you a broad understanding of many of the key topics in general
insurance. Subsequent study of individual topics in the later chapters should then be clearer
because you will understand the importance of the topic in the context of the whole subject.
Because we cover so many fundamental topics in this chapter, we recommend that you work
slowly and carefully. If you are simultaneously studying Subject SP8, most of the material in this
chapter is covered there too.

The sections in this chapter are as follows:

Section 1: Introduction to general insurance

Section 2: A general insurer’s balance sheet

Section 3: Technical reserves

Section 4: Free reserves

Section 5: Premium rating

Section 6: Capital modelling

Section 7: Investments

Section 8: Profitability and cashflow

Section 9: Reinsurance

Section 10: Glossary items

© IFE: 2019 Examinations The Actuarial Education Company


SP7-01: Insurance companies Page 3

1 Introduction to general insurance

1.1 Why do general insurers exist?

To meet a need
Centuries ago merchants were encouraged to hazard great journeys by the existence of
insurance: if they took the risk and disaster struck, then they would not be ruined if they were
insured. The same social advantage is still there today. The exciting ventures have changed
somewhat, but the ability to insure against various perils still enables individuals and companies
to take on risks that they would not otherwise undertake.

The ability to make a known, small outlay to insure against the risk of a potentially large loss is
justified by economists’ utility theory, empirical evidence and common sense. People will pay
more for insurance than the expected recovery from insurance (ie ‘the risk premium’), because
they are risk-averse and prefer the more certain outcome.

To make money
Most general insurance companies exist primarily to make money. Get this firmly in your mind at
this early stage. A moneymaking opportunity exists for Insurer A, because of the last sentence of
the previous section. The constraints on A’s profitability are how much the customer is prepared
to pay, any statutory controls on insurers and the competition from other insurance companies.

These concepts apply to most industries, not just general insurance. All companies weigh up the
potential risks of their business and invest their capital so as to maximise return to shareholders.
For most of Subject SP7 (and SP8 and SA3) you need very commercial, profit-driven thinking.

Question

Outline why some insurance companies might quote much higher premiums than other insurance
companies for car insurance for the same individual and period of cover. (Try to give several
different ideas.)

Solution

There are lots of possible ideas, including:


 different estimates of the expected claim amount
 different levels of expenses, commission, reinsurance costs, investment returns and profit
 some insurers don’t want the business, so quote too high
 others may undercharge, hoping the policyholder will stay for years and that they will
make money later

The Actuarial Education Company © IFE: 2019 Examinations


Page 4 SP7-01: Insurance companies

 different forms of cover may have been requested, eg:


– comprehensive or third party fire and theft only
– as the only named driver, or possibly with family / friends as drivers too
– different excess levels
 quotations could have related to driving in different countries etc, etc, etc.

Welcome to Subject SP7. This sort of careful question reading plus common sense / lateral
thinking and the generation of a lot of relevant ideas is often just what is needed.

1.2 What is general insurance?


In simple terms, general insurance is any type of insurance that is not life insurance.

General insurance therefore encompasses a wide range of types of insurance. In most cases a
general insurance policy is a contract of indemnity, eg if a loss occurs of an insured article, the
insurer will reimburse the value of the insured article. Other policies might pay specified amounts
on specified contingencies only (eg £10,000 if you lose the use of an eye) or if the loss is unclear,
the amount might be determined by a court of law.

1.3 Risks, uncertainty and general insurance


Policyholders reduce their uncertainty by passing risks to insurance companies. It is not
surprising, therefore, that insurance companies themselves are subject to risk and uncertainty.
How to quantify and control this uncertainty is the underlying theme throughout this course. As
you work through the different areas of the course think about how product design, capital
requirements, pricing, reserving, reinsurance and accounts are influenced by the different risk and
uncertainties involved in writing insurance.

Most of the major uncertainties centre around how many claims will occur and how much the
insurer will have to pay to settle them. These uncertainties have a big influence on how much the
insurer will charge for the protection provided (covered in Subject SP8) and how much the insurer
needs to reserve for future claims payments (covered in Subject SP7). Other risks to the insurer
include:
 failure to recover fixed expenses
 failure of other parties (eg brokers or reinsurers)
 falls in asset values
 the insurance cycle (also known as the underwriting cycle).

The size of the free reserves will influence the ability of the insurer to cope with these risks, as will
the insurer’s reinsurance cover and investment policy.

© IFE: 2019 Examinations The Actuarial Education Company


SP7-01: Insurance companies Page 5

1.4 Actuaries and general insurance


That’s all very well, but what about actuaries?

It’s clear to us as actuaries that we should have lots to offer this industry, but the industry was
originally a little slow to recognise that. Actuaries have only become involved in general
insurance within the last 50 years

The main actuarial roles have traditionally been in reserving and setting premiums. More
recently, some actuaries have moved into much wider areas within general insurance. For
example:
 capital allocation
 risk assessment, eg modelling catastrophic events
 strategic management of the business
 determining a suitable investment strategy
 assessing reinsurance requirements
 expense allocation
 assessing the effectiveness of marketing campaigns
 assisting with the early settlement of liabilities in the event of a wind-up.

Many of the topics that we study in general insurance are, in fact, areas where actuarial
involvement is increasing.

The Actuarial Education Company © IFE: 2019 Examinations


Page 6 SP7-01: Insurance companies

2 A general insurer’s balance sheet


The balance sheet of any organisation gives a snapshot of the finances of that organisation as at
the date of the balance sheet. The balance sheet of a general insurer is therefore a summary
(strictly an estimate) of the financial status of the company at a particular moment. The balance
sheet is summarised in the diagram shown below.

The right-hand side is a statement of everything the insurer owns, and the left-hand side is a
summary of what it owes. (Free reserves are ‘owed’ by the insurer to the owner(s).)

LIABILITIES ASSETS

Free reserves

Investments

Technical reserves

Fixed assets
Net current assets

Free reserves: The balancing item equal to the excess of assets over insurance (or
technical) liabilities. Discussed in Section 4.

Technical reserves: These are the amounts set aside in respect of expected claim payments to
or on behalf of policyholders, plus related expenses. Discussed in
Section 3.

Investments: These might be bonds, equities, property etc. Discussed in Section 7.

Fixed assets: For example, the office building and equipment. Not discussed much in
this course, but you should be aware that they exist.

Net current assets: Excess of current assets over current liabilities, eg money due from
brokers.

© IFE: 2019 Examinations The Actuarial Education Company


SP7-01: Insurance companies Page 7

2.1 Different types of balance sheet


The values placed on each of the items in the balance sheet are not always uniquely defined.
Firstly, there may be an element of judgement used in deciding the values, so that a balance sheet
based on a very prudent approach could be very different from a balance sheet produced with
less conservative assumptions. Secondly, balance sheets produced for different purposes will be
assessed using different approaches. For example:
 Statutory accounts, if required, may be in a prescribed format or there may be certain
principles that must be applied.
 Internal management accounts may be produced to assist internal decision-making.
These are likely to be on an on-going, realistic basis, although a variety of ‘what-if’
scenarios are also likely to be produced.

Question

Suggest reasons why the latest balance sheet of an insurance company (as given in the company
accounts) might not give a true indication of the financial strength of the company.

Solution

Some judgement is required in setting values for assets and liabilities. So, for example, when
assessing the financial strength of the company a prudent (not realistic) basis may be used.

Also, the balance sheet is a snapshot at a given moment. Circumstances may have changed since
the date of the balance sheet.

The Actuarial Education Company © IFE: 2019 Examinations


Page 8 SP7-01: Insurance companies

3 Technical reserves
The technical reserves are held to cover the liabilities relating to policies already written. The
technical reserves might also be called insurance reserves or insurance provisions because they
relate to the liabilities arising from writing insurance business.

These liabilities can be split into two main categories:

(a) Past: liabilities (claims plus expenses) in respect of accidents or losses from events that
have occurred prior to the accounting date. Liabilities for outstanding claims are
discussed in Section 3.2.

(b) Future: liabilities (claims plus expenses) in respect of future insurance cover from policies
for which premiums have already been received. Unexpired risks are discussed in
Section 3.3.

First, in Section 3.1, we explain how the claim characteristics of the major insurance classes
influence the degree of uncertainty in the reserving process.

3.1 Major claim characteristics


Claim characteristics refer to the ways in which, and speed with which, claims originate, are
reported, are settled and are, on occasion, reopened.

Imagine the rather extreme scenario in which insurance companies make claim payments as soon
as a claim event occurs. For example, the instant that a policyholder has a car accident, there is
an immediate transfer to the accounts of the affected parties. In this scenario, what is the
insurance company’s liability at any one time for outstanding claims?

The answer is zero. If all claims are settled the instant the event occurs, then the company never
has any outstanding claims liability. In reality, insurance companies do have outstanding claims
liabilities because there are delays between claims occurring and being settled. Hence insurance
companies will hold reserves for outstanding claims. Before looking at reserves, let’s consider the
delays more fully.

There are two main types of delays: reporting delays and settlement delays.

Reporting delays
The reporting delay is the time from when the event occurs through to the time that the
insurance company is notified of the event. Sometimes policyholders may be slow in getting
round to advising the insurer – possibly because the amount involved is quite small. Other times
the policyholders do not submit claims because they do not realise there is cause for claiming.
For example, in the case of a number of industrial diseases (eg asbestos-related diseases,
industrial deafness) it may be many years before the condition emerges. In these cases, reporting
delays are often considerable.

© IFE: 2019 Examinations The Actuarial Education Company


SP7-01: Insurance companies Page 9

The part of the delay that relates to the period between when the insured event happens
(eg original exposure to asbestos) and when the policyholder realises the event has happened
(eg the policyholder starts to develop signs of illness) is referred to as the event delay. In many
cases the event delay is minimal (eg car accidents). In practice, the term ‘event delay’ is not often
used. Many people simply use the term ‘reporting delay’ to mean ‘reporting delay plus event
delay’.

Settlement delays
The settlement delay is the period between notification to the insurance company and the
payment of the claim. These delays are due to:
(a) initial administrative processing
(b) establishing whether the insurer is liable
(c) waiting for a condition to stabilise (eg will the injured party recover, or is the disability
permanent?)
(d) establishing how much should be paid.

In a few cases where the insurer and the claimant cannot agree, the case may go to court.

In general (but certainly not always):


 property damage claims are settled more quickly than claims in respect of bodily injury
 large claims take longer to settle than small claims.

Short tail and long tail


Different classes of business are referred to as ‘short tail’ or ‘long tail’, where:
 short tail means that claims are generally reported quickly and settled quickly by the
insurer
 long tail means that there is a sizeable proportion of total claim payments that take a long
time for the insurer to settle.

When looking at an individual class of business, or type of claim, you should make a point of
noting whether it is short tail or long tail (or in-between). This is important in developing your
understanding of an insurance class, and may influence your answer to a question in the exam.
The claim characteristics of the main insurance classes are detailed later in the course.

The Actuarial Education Company © IFE: 2019 Examinations


Page 10 SP7-01: Insurance companies

3.2 Reserves for outstanding claims


The outstanding claims reserve is the first of the two main components of the technical reserves.
It may be given as a total figure or, alternatively, it may be split into anything up to five separate
components:
(a) Reserve for outstanding reported claims: this is the estimated reserve needed to settle
the claims that the company knows about at the accounting date.
(b) Reserve for incurred but not reported (IBNR) claims: the IBNR reserve is needed to cover
the claim payments for incidents which have happened, but have not been reported to
the insurance company on or before the accounting date.
(c) Reserve for incurred but not enough reported (IBNER): the IBNER reserve is needed to
cover expected increases (or decreases) in estimates for reported claims.
(d) Reserve for re-opened claims: this is an additional reserve which may be explicitly shown
to allow for claims that the insurance company treats as being fully settled, but which
might one day require further payments. In practice, insurers differ significantly over
when they ‘close’ a claim.
(e) Reserve for claims handling expenses: in settling claims in each of the above categories,
the company will incur some additional expenses (eg legal fees). The reserve for these
expenses may be held separately.

Even if the reserves are not shown split into these categories, an insurance company should still
hold reserves to cover all of these items. For example, if the reserves are shown split into the first
two components only, then the reserve for re-opened claims might be within the outstanding
reported claims reserve, the reserve for IBNER might be within the reserve for IBNR and the
reserve for claims handling expenses would be split between the two.

Question

There are two main reasons for needing outstanding claims reserves: reporting delays and
settlement delays. Suggest which type of claims delay can affect each component of the
outstanding claims reserves.

Solution

Outstanding reported claims reserve is in respect of settlement delays.

IBNR reserve is in respect of reporting delays.

The IBNER reserve exists to cover expected increases (or decreases) in outstanding reported
amounts. Such changes could result from either reporting or settlement delays.

Re-opened claims are caused by premature closure of a claims file. The cause of the closure is
needed to determine whether a reporting or settlement delay is the reason.

Reserves for claims’ handling expenses can be in respect of both types of delay.

© IFE: 2019 Examinations The Actuarial Education Company


SP7-01: Insurance companies Page 11

Estimating outstanding claims reserves


There is great uncertainty about the payments an insurer will need to make in respect of
outstanding claims. The amounts which will be paid are not known, so the insurer must use
estimates when deciding how much to set aside in respect of these liabilities. Two distinct
approaches are:
(a) making estimates of the liability for each individual outstanding claim (‘case estimates’, or
‘case by case estimation’)
(b) using statistical techniques to estimate the total outstanding payments for the portfolio.

In practice, insurers use a combination of the two. We study this topic in detail in the reserving
chapters later in the course. In the meantime, the following observations should be apparent:
 individual estimates cannot be used for IBNR because the insurer does not yet know
about the claim
 statistical techniques are more useful for classes of insurance where there are lots of
claims (eg private motor), and where there is stability in the numbers and amounts of
claims.

One important consequence of the uncertainty about the liability for outstanding claims is that
any aspect of the insurer which relies upon estimates for outstanding claims (eg profitability) is, as
a result, also subject to uncertainty. This is an important point that you should be aware of at all
times when working with figures that rely upon technical reserves.

The degree of this uncertainty will vary from class to class. Generally, there is much more
uncertainty with long-tail classes, where the reserves for outstanding claims are larger in relation
to premium income.

Question

An insurance company splits its outstanding claims reserves into two components: reported
claims and IBNR.

The company writes two classes of business. For each class, the outstanding claims reserve is split
as follows:

Class 1 Class 2

Reserve for reported claims 90% 50%

Reserve for IBNR 10% 50%

The reserves for Class 2 are a much bigger proportion of premium income than are the reserves
for Class 1.

Suggest two common classes of insurance this company might write.

The Actuarial Education Company © IFE: 2019 Examinations


Page 12 SP7-01: Insurance companies

Solution

Class 1 must be a short-tail class of business (because total claims reserves are relatively low),
with little in the way of reporting delays, eg household contents.

Class 2 is a long-tail class (because the total claims reserves are a much bigger proportion of
premiums), with extensive reporting delays. A class such as employers’ liability is possible, where
some illnesses may not emerge for many years which will make the IBNR significant.

3.3 Reserves for unexpired policies


We have discussed reserves in respect of claim events that have already happened. We now turn
to the other main component of the technical reserves, the liabilities in respect of existing policies
with some unexpired exposure, ie future claim events from policies with future periods of cover
remaining at the accounting date.

Unearned premium reserves


One basis for determining the reserves in respect of the unexpired exposure is to hold a portion of
premiums in respect of that exposure. For example, on a straight averaging basis, for a policy
with half of its term still to run, it might be reasonable to hold a reserve of half of the premium
that was charged. For an annual policy with one month to run, a reserve of one twelfth of the
premium might be held.

The name given to the portion of premiums held in respect of unexpired exposure is the unearned
premium reserve (UPR). The title is quite logical: the UPR is simply the premiums that have been
received which have not yet been earned. This is a retrospective assessment of the reserve.

The straight averaging approach used above has a number of fundamental weaknesses in
practice:
 it ignores the fact that the risk from the policy may not be spread evenly over the period
of cover, eg seasonal effects
 it ignores the fact that the expenses of setting up and servicing the policy may not be
incurred evenly over the period of cover.

For the purpose of this discussion, we will assume that risk is even over the period of cover.
However, a similar assumption about expenses would not be appropriate because there is clearly
a large element of expense that is generally incurred at the commencement of a policy,
eg commission paid to the sales outlet.

The expenses that are incurred by the insurer at the start of a policy are called acquisition costs.
Commission is generally the major component of acquisition costs. But how should these
acquisition costs be allowed for when setting the reserves in respect of unexpired exposure? The
best way to see this is through a simple example.

© IFE: 2019 Examinations The Actuarial Education Company


SP7-01: Insurance companies Page 13

Example
Suppose a policy has acquisition costs of 20% of the premium. Then 80% of the premium is
available to meet claims, on-going expenses and profit. If we assume that the risk and on-going
expenses are spread evenly over the period of cover, then the 80% of premium (ie after deduction
of acquisition costs) could also be spread evenly. If the policy were half-way through its life at the
accounting date, the UPR would be 40% of the premium (ie half of the 80% which is to be spread
over the life of the policy).

This approach to establishing the UPR can be extended to a more general formula:

UPR  proportion of risk unexpired  (premium  acquisition costs)

The UPR calculated in this way is the net UPR. The gross UPR doesn’t allow for the acquisition
expenses.

Question

An insurance company calculates its UPR using an individual, policy by policy approach. Calculate
the UPR as at 31 December 2018 for the following annual policies:
(a) Premium £1,000. Commission £120. Started 1 October 2018.
Risk assumed to be uniform over the year.
(b) Premium £3,500. Commission 10%. Started 1 July 2018.
Risk starts at zero, and increases daily by a constant linear amount over the policy year.

(c) Premium £2,200. Commission £200. Started at 11.59pm on 31 December 2018


Risk fluctuates throughout the policy year.
(d) Premium £10,000. No acquisition costs. Started 1 November 2018.
All risk assumed to be in April.

Solution

(a) £660, ie 75% of (1,000  120)


(b) £2,362.50 (75% of risk is outstanding at year-end, so the UPR is 75% of 90% of £3,500)
(c) £2,000, ie 100% of (2,200  200)
(d) £10,000, ie 100% of 10,000

Unexpired risk reserve


So far, our assessment of the reserve to be held in respect of unexpired exposure has been
retrospective by looking at the premiums we have received in the past. We have set up a reserve
equal to the unearned premium net of acquisition costs, ie the net UPR. A prospective approach
should also be considered by thinking about possible future claim events. What reserve do we
think we need to hold to cover the unexpired risks?

The Actuarial Education Company © IFE: 2019 Examinations


Page 14 SP7-01: Insurance companies

The unexpired risk reserve (URR) is the name for this prospective assessment. Again, the title is
quite logical. Remember that such a reserve would need to cover all the claims and all the
expenses that are expected to be incurred in the future from the unexpired portion of existing
policies.

We would normally expect the unearned premium reserve to be bigger than the unexpired risk
reserve. This is effectively the same as saying we expect the premiums to be big enough to cover
the claims and expenses – which is what a profit-seeking insurer would generally want. In the
cases where the UPR is greater than the URR, there is no need for the insurer to keep reserves
greater than the UPR for unexpired policies. Because of the accounting accruals principle we
would generally hold at least the full UPR. Here, holding a reserve equal to the UPR, we would
expect some profit to emerge over the coming months from these policies.

However, for cases where URR is greater than UPR, the calculations become more complex.
These cases imply that the company expects to make a loss on the unexpired policies because it
expects to pay out more in claims and expenses than the amount of premium held back for the
unexpired period. Therefore, the UPR will be insufficient to meet the expected payments, and the
insurer should set up additional reserves to meet this strain.

These additional reserves are known (again, quite logically) as the additional unexpired risk
reserves (AURR), or the additional reserves (or provision) for unexpired risks.

It should not surprise you that:

AURR = URR  UPR (minimum of zero)

You will need to be particularly careful with the expression unexpired risk reserve. In some
contexts practitioners use it to mean the total URR (as defined above), and others may use it to
mean the AURR. So:
 stick to the definitions given in this chapter
 be careful when reading unexpired risk reserve, and check which is meant in the context
 be clear in your own work as to whether you mean total unexpired risk or additional
unexpired risk.

Question

State in layman’s terms the key difference between the UPR and the URR. Justify which of the
two calculations, UPR or URR, is open to most uncertainty.

Solution

Whereas UPR is the portion of premium set aside for unexpired risks, the URR is our estimate of
how much we need to cover the claims and expenses from unexpired risks.

The URR is probably open to more uncertainty. We know what premiums we charged, but we
don’t know what the claims experience will be next year.

© IFE: 2019 Examinations The Actuarial Education Company


SP7-01: Insurance companies Page 15

Note that since Solvency II came into force in the UK (and other EU countries), insurance
companies need to estimate their reserves for unexpired exposures using a prospective approach
(ie unexpired risk reserve) as opposed to a retrospective approach (ie unearned premium
reserves).

3.4 Other types of technical reserve


So far in this section we have looked at reserves in respect of outstanding claims and reserves in
respect of unexpired policies. We now look briefly at two further types of technical reserve which
do not fall easily into either category: claims equalisation reserves and catastrophe reserves.

Claims equalisation reserve


Because insurance business is volatile, the year-to-year profits of an insurance company can be
very volatile.

One way to reduce the volatility of profits over time is to hold a claims equalisation reserve. This
is a reserve that is used to smooth the profits from one year to another. In a good year when
profits are large, money is transferred to the claims equalisation reserve, thereby reducing the
initial assessment of profit. In a bad year, money is transferred from the equalisation reserve,
thereby increasing the initial assessment of profit.

Since Solvency II came into force, UK (and other EU) insurance companies no longer need to hold
claims equalisation reserves. However many other countries worldwide still hold them.

Catastrophe reserve
In the context of general insurance, a catastrophe is a single event that gives rise to an
exceptionally large aggregation of losses. Examples of catastrophic events range from natural
catastrophes such as floods, windstorms earthquakes, to man-made catastrophes such as aircraft
crashes, explosions or oil spillages.

An insurance company may choose to set aside an additional reserve to cover the losses that
might arise from a catastrophe. Whereas an insurer would expect to use other types of reserve
for, say, outstanding claims, an insurer would not expect to have to pay out from the catastrophe
reserve. It is genuinely a contingency reserve that would be held just in case something awful
were to happen.

If an insurer did hold a large explicit catastrophe reserve, there would be less need for the insurer
to hold extensive free reserves (ie the excess of assets over liabilities). Conversely, the free
reserves for a company that does not hold a catastrophe reserve need to be sufficiently big to
cover the possibility of a catastrophe (or two).

Question

A non-EU insurance company that recognises that it has been writing business unprofitably for
the last six months shows just three different types of technical reserve in its accounts. Suggest
what they might be. Explain how your answer would change if the question had said very
profitably instead.

The Actuarial Education Company © IFE: 2019 Examinations


Page 16 SP7-01: Insurance companies

Solution

 Outstanding claims reserves (includes IBNR etc)


 UPR
 AURR (needed because the URR is expected to exceed UPR, since business is unprofitable)

If the question had said that the company was writing business very profitably, then we would not
have needed an AURR. So the answer would have been:
 Outstanding reported claims reserves
 IBNR
 UPR

© IFE: 2019 Examinations The Actuarial Education Company


SP7-01: Insurance companies Page 17

4 Free reserves
The free reserves are the excess of the assets over the technical reserves (as shown in the
diagram of the balance sheet on page 6).

You need to be particularly careful here because lots of different expressions are used by
different practitioners in different circumstances to refer to the excess of assets over technical
liabilities. The following expressions are all commonly used as alternatives to free reserves:
 free assets
 solvency margin
 shareholders’ funds
 capital employed.

The word ‘solvency’ has several different possible interpretations for a general insurance
company. The most common is the concept that the assets exceed the liabilities. Hence the
excess of assets over liabilities may be called the solvency margin. Where a solvency ratio is
discussed in general insurance, it is the solvency margin divided by the written premiums, not the
solvency margin divided by total assets. This is another area where terminology sometimes
differs. Some practitioners use the term solvency margin when referring to the ratio defined
above.

4.1 Significance of free reserves


Without free reserves, an insurance company would not be able to operate.

Firstly, you may regard the free reserves as the pool of funds being used to provide the backing
for insurance risks. If the insurer did not have an adequate level of free reserves, policyholders
would have no reason to believe that the insurer would be able to meet claims in the event of
disaster. Meeting claims after adverse events is the whole purpose of insurance.

Secondly, there may be a legal requirement for an insurance company’s free reserves to exceed a
statutory minimum level. In the UK, this minimum capital amount is often called the Minimum
Capital Requirement (MCR), or Required Minimum Margin (RMM).

The extent of the free reserves is very important for the management of the insurance company.
For example, it is closely linked to the following:
 The maximum amount of business the company should write: free reserves are required
to provide a cushion against unexpected adverse results. The more business that is
written, the bigger the required cushion. Conversely, there is a maximum amount of
business that a given level of free reserves should support.
 The classes of business written: some classes of business have more variable claims
experience than others and some classes involve bigger risks. A higher level of free
reserves can support more variable and larger risks.

The Actuarial Education Company © IFE: 2019 Examinations


Page 18 SP7-01: Insurance companies

Question

Other aspects of the management of a general insurer are also influenced by the size of the free
reserves. Suggest what you think the effect of higher free reserves will be on an insurer’s:
 reinsurance programme
 investment strategy
 pricing policy.

Solution

Higher free reserves result in:


 less need for reinsurance
 greater investment freedom, ie scope to mismatch
 greater scope for competitive pricing.

© IFE: 2019 Examinations The Actuarial Education Company


SP7-01: Insurance companies Page 19

5 Premium rating

5.1 Introduction
The process of setting appropriate premium rates is key to the operation of a successful insurance
company, and is a major area of actuarial involvement.

Although premium rating is not covered in great depth in Subject SP7, it forms a core part of the
syllabus for Subject SP8. Some of the topics covered here will be familiar to you from your study
of earlier subjects.

5.2 Technical methods revisited


Throughout all areas of general insurance work, be it reserving, pricing or capital modelling,
actuaries will be interested in the frequency and severity distributions of claims. In particular, the
estimation of claim distributions will allow a pricing actuary to calculate the pure ‘risk premium’,
ie the amount of premium that is required to exactly cover the expected cost of claims alone.
Actuaries will therefore attempt to model aggregate claim distributions, or find reasonable
approximations where analytical methods are impractical.

The pricing actuary will also wish to ensure that different types of policyholders are charged the
most appropriate premium given their own risk characteristics (subject to any legislative
requirements). For example, older drivers are considered on average to be lower risk and
therefore can expect to be charged a lower motor insurance premium than young drivers. The
actuary will therefore want to identify those characteristics of a policyholder that have the
greatest impact on the amount of risk taken on. Generalised Linear Models (GLMs) and
multivariate analyses are key tools in the identification of these risk factors.

5.3 Other pricing techniques


A ‘burning cost’ approach is a simple method often used as a starting point to price certain types
of insurance and reinsurance. It calculates the risk premium as actual cost of claims during a past
time period, expressed as an annual rate, per unit of exposure. You might therefore like to think
of it as just an average claim cost per year, per unit of exposure.

The ‘frequency-severity’ approach to pricing calculates the risk premium as the expected average
claim cost multiplied by the expected average number of claims in the period.

Original loss curves are often used in general insurance to derive premium rates where past
claims data is too sparse to derive a credible price using more traditional techniques. They are
often used to price certain types of reinsurance arrangements. However, a key problem of
original loss curves is the difficulty of estimating and/or selecting appropriate curves, since the
resulting expected claim cost is often very sensitive to the specific model selected.

The Actuarial Education Company © IFE: 2019 Examinations


Page 20 SP7-01: Insurance companies

5.4 Pricing methodology


A pricing actuary is likely to estimate the expected cost of claims based on past data. However, it
would be naïve to assume that past claims experience will always be an accurate predictor of
claims experience on a future policy. Adjustment may have to be made to allow for:

 unusually heavy / light experience in the past data

 large or exceptional claims  the actuary may exclude these from the past data and then
make an explicit loading in the risk premium later for catastrophes and large claims

 trends in claim experience which mean the past data will be out of date by the time the
new rates are in service

 inflation between the date of the data and the date the policy will be in force

 changes in the risk and/or cover provided

 changes in environmental factors, eg legislative / technological changes

 incomplete past data arising from the fact that these claims might not yet be fully
developed.

A corresponding analysis and projection of exposure data will also need to be made.

The premium actually charged to a policyholder will not only be based on the expected cost of
claims on that policy, however. Further adjustments are made, to arrive at the ‘office premium’
ie the actual market price for a policy. These adjustments include:

 expected reinsurance costs and recoveries

 expense loadings such as commission, policy administration costs, claim handling costs,
overheads, levies etc

 loadings for profit

 allowance for investment income.

Further adjustments will also be made for practical considerations such as:

 the insurer’s business objectives eg desire to build up market share

 competitive pressures

 the level of brand or customer loyalty amongst the insurer’s target market

 the state of the insurance cycle

 market acceptability, ie the level of cross-subsidy between policies, for example between
new business and renewals business, and the likely reaction of policyholders to these
differences

 the use of no-claims discounts.

© IFE: 2019 Examinations The Actuarial Education Company


SP7-01: Insurance companies Page 21

6 Capital modelling

6.1 Introduction
Capital modelling is a relatively new field in general insurance. Recent changes in regulation, and
the increasing focus on risk modelling in all sectors of the financial services industry has meant
that this is a growing area of actuarial work.

Any resources held by the company in excess of the value of its technical liabilities are called its
capital, and we saw in Section 4 that a general insurer will hold capital well in excess of these
liabilities. In fact, it will hold enough capital to be sure of meeting all its obligations to
policyholders with a certain degree of confidence. The process by which this confidence level is
obtained is called ‘capital modelling’. A capital model will also be used to help allocate capital
between classes, products and individual policies.

6.2 Structure of a capital model


A capital model will most often take the form of a cashflow model, and will project all aspects of a
general insurer’s business into the future, to assess the company’s ability to meet its financial
obligations. A range of assumptions will therefore be required, including:
 future written premium income
 future claims, on current and future business, and the timing of these claims
 future reserving basis
 future expenses
 investment return
 reinsurance arrangements, costs and recoveries, potential reinsurance exhaustion
 economic variables such as future inflation and interest rates
 likelihood / cost of catastrophes
 the insurance cycle
 operational losses
 tax
 dividends.

It may help when developing ideas on this in the exam, to think about the components of a
general insurer’s income statement.

A capital model will usually analyse the general insurer’s business in a considerable level of detail.
We would expect at least that each class of business will be projected separately, but projections
may also include separate analyses for different sources of business, locations, claim types, asset
types, asset categories etc. The level of detail, or granularity, built in to the capital model will
depend on the uses to which the model is put.

The Actuarial Education Company © IFE: 2019 Examinations


Page 22 SP7-01: Insurance companies

Given the degree of volatility in general insurance business, a range of methods will be used,
including stress testing and scenario testing. It is also likely that stochastic models will be used
since the tail end of an insurer’s claims distributions will be of particular interest in determining
confidence levels.

6.3 Allowance for risk in a capital model


The modern approach to assessing the level of capital that general insurers are required to hold is
to allow for the inherent riskiness of the insurer’s activities. The greater the risk taken on by the
insurer, the greater the level of capital that it is required to hold. Conversely, if an insurer is
considered to be less risky, this can be reflected in the capital model, and a lower capital level will
be required. Thus, companies can be rewarded for maintaining a sound risk management policy.

The types of risk that may be taken into account in the capital modelling process include:
 insurance risks  the uncertainty arising from the amount and timing of claims, expenses
and premiums
 market risk  risks relating to changes in investment market values or investment income
 credit risk  the risk of failure of third parties to repay debts, including the failure of
reinsurers
 operational risk  the risk of loss due to failures of people, processes and systems,
eg fraud or mismanagement within the general insurer itself
 liquidity risk  the risk that the company is unable to meet its obligations as they fall due
as a consequence of a timing mismatch between its assets and liabilities
 group risk  the risk that the company experiences as a result of being part of a group
eg from being a subsidiary of a parent company, as opposed to a stand-alone entity
 other risks such as strategic risks, political risks and enterprise risk management risks
(ERM). You may have come across ERM from your earlier studies, where it might have
been called integrated risk management. ERM is discussed further later in the course.

Insurance risks can be further sub-divided into:


 reserving risk  the risk that the level of technical reserves held in respect of risks already
written / earned is inappropriate
 underwriting or premium risk – relating to risks yet to be written / earned.

An insurer’s required level of capital can be adjusted for the fact that some of the risks outlined
above will be negatively correlated. A good capital model will make allowance for this
diversification effect. Conversely however, should the insurer be exposed to accumulations of
risk, the level of capital will need to be increased.

© IFE: 2019 Examinations The Actuarial Education Company


SP7-01: Insurance companies Page 23

7 Investments
As with any investing institution, a general insurance company will want to achieve the maximum
possible return from its investments, without exposing itself to an undesirable level of risk.

Here’s a quick summary of the major factors influencing the company’s investment strategy.

7.1 Nature of liabilities


Some of the liabilities will be in fixed monetary amounts. Investments producing fixed monetary
returns are appropriate for these liabilities.

However, many of the liabilities will need to be settled in prices applicable at the time of
settlement. This means that there is an element of inflation underlying most of the liabilities. The
type of inflation the liabilities are exposed to varies by class and peril. This becomes important for
those classes of insurance where there are considerable reporting and settlement delays.
Investments that tend to maintain their real value are desirable for such liabilities.

7.2 Term of liabilities


Investments should have similar terms to those of the liabilities. The appropriate term of
investments is very dependent upon the classes of business written. Compared with life
assurance and pension funds, many liabilities tend to be short term, and the investments tend to
reflect this. For long-tail classes of business (eg employers’ liability), some medium-term and
even some long-term investments are appropriate.

7.3 Currency of liabilities


In some classes of insurance, for example, marine, travel, product liability, insurers may have
liabilities in several currencies. Assets should be held in appropriate currencies to match these
liabilities.

7.4 Uncertainty of liabilities


In general insurance there is often a great deal of uncertainty about the amounts that will be
necessary to settle claims and the timing of the claim payments. The proportion of the assets
held in a form that is liquid must be sufficient to cover this uncertainty.

7.5 Size of free reserves


The bigger the free reserves (relative to the size of the company), the greater the extent to which
the investment strategy can be aimed at maximising returns.

7.6 Legislative influences


The need to maintain the free reserves above a particular minimum solvency margin means that
insurers may be very wary of holding too many investments that have volatile market values. The
extent to which this is a concern depends on the size of the free reserves relative to any statutory
minimum solvency margin that may exist.

The Actuarial Education Company © IFE: 2019 Examinations


Page 24 SP7-01: Insurance companies

7.7 Taxation
Insurers will want to maximise their post-tax investment returns, therefore the taxation basis for
insurers is relevant to the choice of investment strategy.

Question

Suggest which two aspects of an insurance company’s operations will have the most impact on
the company’s investment strategy.

Solution

 classes of business written, hence nature / term / currency and uncertainty of liabilities
 size of free reserves

© IFE: 2019 Examinations The Actuarial Education Company


SP7-01: Insurance companies Page 25

8 Profitability and cashflow

8.1 Basic measure of profits


In this section we will look at the items that make up profits.

Intuitively, it should be clear that:


post-tax profits  premiums  claims  expenses  investment return  tax

(In case you are wondering why reserves are not explicitly shown in this equation, all is explained
below.)

With the premiums, claims and expense items, we need to be careful that the figures we use are
sensible and consistent. Suppose we are trying to calculate the profit earned in 2018. Then, if we
write £10,000 of new business on 30 December 2018, does this mean that the 2018 profits will be
bigger by £10,000? If we follow usual accounting principles, the answer is NO.

The accounting principle that matters here is accruals. Income and expenditure should accrue
over the period to which they relate.

Let’s consider various measures of premiums and claims and then select the measures that are
consistent with the accruals principle.

Written premiums
This is the total amount of premium income written in the year. So for policies starting in the
year, the whole premium will be included within written premiums (also expressed as premiums
written).

Earned premiums
This is the amount of premium income relating to insurance cover provided during the year (also
expressed as premiums earned). For example, if a new annual policy is started on
1 December 2018 for a premium of £1,200, the earned premium in 2018 from this policy would
be £100 (assuming that the risk and expenses are even over the policy year). This policy would
then contribute £1,100 to earned premiums in 2019.

So, which of these two measures of premiums is consistent with the accruals concept?

Hopefully, you will agree that it is earned premiums because this tells us how much premium has
accrued during the year.

Paid claims
This is the total amount of claim payments made by the insurer during the year (also expressed as
claims paid).

The Actuarial Education Company © IFE: 2019 Examinations


Page 26 SP7-01: Insurance companies

Incurred claims
This is the amount of claims paid (as above) plus the increase in the total reserve for outstanding
claims (also expressed as claims incurred). For example, suppose that a claim for £1,000 is
reported on 20 December 2018, but payment is delayed until 2 January 2019. The effect of this
delay is to decrease the 2018 claims paid by £1,000 but the 2018 claims incurred is unchanged
(assuming that the reserve for outstanding claims as at 31 December 2018 is increased by
£1,000).

It’s claims incurred that is consistent with the accruals principle.

Question

An insurance company (which writes remarkably little business) writes six-monthly policies, each
for a fixed premium of £1,200. Policies commencing in 2017 and 2018 were written on the
following dates:

1/3/17, 1/4/17, 1/8/17, 1/10/17, 1/12/17, 1/1/18, 1/3/18, 1/5/18, 1/6/18, 1/8/18, 1/11/18.

At the end of 2017, the reserve for outstanding claims was £6,000. By the end of 2018, the
reserve for outstanding claims was £7,300. A total of £5,500 was paid in claim settlements in
2018.

Calculate (for the 2018 accounting year):


 premiums written
 premiums earned
 claims paid
 claims incurred.

Solution

 premiums written  6  1,200  £7,200

 premiums earned  1,200 6  (0  0  1  3  5  6  6  6  6  5  2)  £8,000

 claims paid = £5,500 (given)


 claims incurred  7,300  6,000  5,500  £6,800

Expenses incurred
When policies are written the insurer pays commission and other initial expenses. The acquisition
costs already paid for unexpired policies at the accounting date are known as Deferred Acquisition
Costs (DAC).

To be consistent with the treatment of premiums and claims above, the expenses item will also
need to be based on an incurred basis rather than just showing expenses paid.

© IFE: 2019 Examinations The Actuarial Education Company


SP7-01: Insurance companies Page 27

Underwriting result
The underwriting result (or underwriting profit) is the term given to the excess of premiums over
claims and expenses:

Earned premiums
– Claims incurred
– Expenses incurred
= Underwriting result

Note that the whole of this account is based on the accruals concept. The underwriting result
shown in the revenue account of a general insurance company is analogous to the operating
profit of non-insurance companies. We discuss the issues covered in this section in more detail
later in the course.

Question

The following data is available for a general insurance company for an accounting year (in £
million):
premiums receivable: 165
unearned premium brought forward at 1 January: 75
unearned premium carried forward at 31 December: 83
claims paid: 103
outstanding claims reserve at 1 January: 124
outstanding claims reserve at 31 December: 133
expenses incurred: 31

Prepare the revenue account showing the underwriting profit (or loss) for the year.

Solution

Earned premiums 157 (  165  75  83 or 165  (83  75) )

Claims incurred 112 (  103  133  124 or 103  (133  124) )

Expenses incurred 31

Underwriting profit 14

The Actuarial Education Company © IFE: 2019 Examinations


Page 28 SP7-01: Insurance companies

8.2 Cashflow diagram


When considering the mechanics of an insurance operation, it is often useful to consider all the
various cashflows that may take place. A simple diagram summarising the cashflows is:

PREMIUMS


TAX 

TECHNICAL INVESTMENTS

EXPENSES  RESERVES Equities

and Bonds
COMMISSION  FREE Property

REINSURANCE  RESERVES IL bonds

Cash

SHAREHOLDERS 

CLAIMS

Most of the expressions in this diagram should now be familiar to you, and the mechanics should
be self-evident. However, the following points are worth bringing out:

Reinsurance
This is the insurance company’s own insurance. That is, the insurance company pays out
reinsurance premiums to a reinsurance company and the reinsurer covers part of the risk the
insurer has taken on. The insurance company will, as a result of this insurance, sometimes make
reinsurance recoveries from the reinsurer.

Investments
The reason for the double arrow is to highlight that money may be lost on investments as well as
being gained. Hopefully the investment income and investment gains will far outweigh the losses.

© IFE: 2019 Examinations The Actuarial Education Company


SP7-01: Insurance companies Page 29

Shareholders
Shareholders will hope to receive dividends (hence the ‘out’ arrow). From time to time
shareholders may be asked to put more money into the company through rights issues.

Finally, thinking of the cashflow diagram and the parties involved in each cashflow is often useful
in helping to generate ideas in the exam.

Question

Determine what would happen to the post-tax profits of a general insurer if the company decided
to reduce the reserve it had been holding for IBNR by £20 million. Assume tax is charged at a rate
of 20%. Suggest (ie use some imagination and make something up) a single event which might
cause this to happen.

Solution

This year’s pre-tax profits would be increased by £20 million, so assuming that this goes straight
into the assessment of tax, we would pay an extra £4 million tax. Post-tax profits would be
increased by £16 million.

IBNR might be revised downwards if it included a big allowance for claims that might come
through from a particular cause, eg employees claiming against their employers for back-ache
from sitting at desks. If a court case has just gone through in favour of the employer, then the
insurer will give a big sigh of relief and reduce its IBNR.

The Actuarial Education Company © IFE: 2019 Examinations


Page 30 SP7-01: Insurance companies

9 Reinsurance
The final topic that we wish to introduce here is reinsurance. Individuals and companies take out
insurance when they perceive a need for it, so as to reduce risk to themselves. Insurance
companies do precisely the same.

Reinsurance is the name for the insurance of insurance companies.

We study reinsurance in some detail in later chapters, but it is helpful early on to look at a couple
of situations in which it may be useful.

9.1 Enormous risks


Insurers like to do business, but there are some risks, eg large industrial or commercial properties,
that are so large that no one insurer could possibly take them on, without endangering their own
solvency position. A way for them to do some business without taking on undue risk is to reinsure
the portion of the risk that they cannot cope with.

Proportional reinsurance or coinsurance can help with this situation.

9.2 Accumulations of risk


Accumulations of risk occur when the insurer has an unbalanced portfolio of risks. This imbalance
may be due to the nature of the classes of business written (eg too much exposure to liability
claims), the geographical areas covered or the types of policyholder attracted. The unbalanced
portfolio exacerbates the problems of non-independent risks. An example would be the severe
flooding in the North of England in late December 2015, leading to many domestic property
insurance claims.

The risk of accumulations can be mitigated, in certain circumstances, by the insurer arranging
some reinsurance to enable them to cope with the possibility of a catastrophic event occurring.

Question

Imagine you work for an insurance company specialising in selling travel insurance, through travel
agents, to parties of over 50’s going on skiing holidays in the US.

List some examples to explain why your company is likely to want to take out some reinsurance.

© IFE: 2019 Examinations The Actuarial Education Company


SP7-01: Insurance companies Page 31

Solution

It depends critically on how wide the cover provided is, but examples of some potentially scary
things, for which reinsurance may help the insurer sleep at night, are:
 One or more whole parties could be affected by the same claim event, eg:
– delayed departure from airport
– no snow on arrival, or other sub-standard holiday features
– plane / coach crash
– avalanche.
 Similarly, if one or more travel agents became bankrupt the insurer might, conceivably,
suffer a large loss from many claims.
 One or more of the skiers may suffer injury, with potentially enormous medical expense
claims, especially with older skiers (complications?) and particularly in the US.

The Actuarial Education Company © IFE: 2019 Examinations


Page 32 SP7-01: Insurance companies

10 Glossary items
At the end of most chapters we will include a section like this one which lists the glossary items
that have either been introduced in the chapter or are related to the material in the chapter. You
will not have met all of the terms given here in the chapter, but it is a good idea to become
familiar with these terms at this early stage. To study actively you should attempt to explain each
of the terms that you have come across and then check your definition against the Glossary.

You should now read the definitions of the following Glossary items:
 Accumulation of risk
 Acquisition costs
 Additional reserve for unexpired risk
 Burning cost
 Case by case estimation
 Casualty insurance
 Catastrophe
 Catastrophe reserve
 Claim
 Claim size distribution
 Claims incurred
 Coinsurance
 Cover note
 Deep pocket syndrome
 Deferred acquisition costs (DAC)
 Earned premiums
 Event
 Experience account
 Franchise
 Free reserves
 Incurred but not reported (IBNR) reserve
 Incurred claims
 Indemnity, principle of
 Insurance certificate
 Insured
 Knock-for-knock agreement

© IFE: 2019 Examinations The Actuarial Education Company


SP7-01: Insurance companies Page 33

 Long-tailed business
 Loss
 No-claim discount (NCD)
 Office premium
 Outstanding claims reserve (OCR)
 Period of unexpired risk
 Points rating system
 Rating
 Rating factor
 Recoveries
 Reinsurance
 Reinsurer
 Re-opened claim
 Risk factor
 Risk premium
 Short-tailed business
 Solvency margin
 Solvency ratio
 Technical reserves
 Unearned premiums
 Unearned premium reserve (UPR)
 Unexpired risks reserve (URR)
 Written premiums.

The Actuarial Education Company © IFE: 2019 Examinations


Page 34 SP7-01: Insurance companies

The chapter summary starts on the next page so that you can
keep all the chapter summaries together for revision purposes.

© IFE: 2019 Examinations The Actuarial Education Company


SP7-01: Insurance companies Page 35

Chapter 1 Summary
The existence of general insurance is good for society as a whole and for individuals.

On one side of the balance sheet are free reserves and technical reserves. On the other we
have all the assets.

Technical reserves (or insurance reserves) might be split into:

Past events: – outstanding reported claims


– IBNR and IBNER
– re-opened claims
– claims handling expenses

Future events: – UPR


– AURR

Insurers may also hold catastrophe reserves or claims equalisation reserves.

The claims reserves might be shown as one amount for outstanding claims and one for
unexpired risks.

Claims reserves occur because there are reporting delays, settlement delays and premature
closure of claims files. Claims reserves are generally larger for long-tail classes of business.

Estimates for outstanding claims reserves are carried out by estimates of individual
outstanding claims or by using statistical methods for the totals. Individual estimates can’t
be used for IBNR.

Claims reserves are estimates. Therefore any work which is based on claims reserves should
recognise the uncertainty underlying the estimates. This uncertainty is generally greater for
long-tail classes.

UPR is the portion of premiums set aside to cover the claims and expenses for future
accounting periods for which premiums have already been received.

URR is a prospective assessment of the amount required as at the accounting date to cover
the claims and expenses from the unexpired risks.

AURR is the excess of URR over UPR, subject to a minimum of zero.

Free reserves are the excess of assets over technical reserves. They may also be referred to
as free assets, the solvency margin, shareholders’ funds or capital employed.

The Actuarial Education Company © IFE: 2019 Examinations


Page 36 SP7-01: Insurance companies

Chapter 1 Summary continued


The size of free reserves is an important determinant of:
 the amount of business the company can reasonably write
 the size of risks written
 the amount of risk within the investment strategy
 the need for reinsurance.

Premium rating can involve techniques such as:


 a burning cost approach
 a frequency severity approach
 modelling aggregate claims distributions
 identifying appropriate rating factors eg using GLMs and other multivariate analyses
 original loss curves, often used to price reinsurance contracts where past data is too
sparse to be credible.

Adjustments will be made to past data to ensure it remains a good predictor of future
experience. Further adjustments will be made to arrive at an ‘office premium’. For example:
 allowance for reinsurance and investment income
 loadings for profit and expenses
 commercially-driven adjustments eg no-claims discounts, allowance for the
insurance cycle, policyholders’ reactions, the company’s strategic objectives and the
state of the insurance cycle.

Capital modelling is the process by which general insurers ensure they hold enough capital to
meet all their obligations subject to a given degree of confidence, and allocate this capital to
different areas of the business.

A good capital model will include assumptions on all aspects of the business which will affect
its future financial strength, and can be a considerably detailed exercise. It may include
stress testing, scenario testing and stochastic modelling.

The level of capital held by a general insurer will depend on the riskiness of the insurer’s
activities. Allowance should be made for the level of correlation between different risks.

The main influences on the investment strategy will be the currency, term, nature and level
of uncertainty of the liabilities (these factors are determined mainly by the classes of
business written), the size of the free reserves, and legislative factors.

The profit of an insurer is the excess of premiums and investment returns over claims and
expenses.

© IFE: 2019 Examinations The Actuarial Education Company


SP7-01: Insurance companies Page 37

Chapter 1 Summary continued


Earned premiums, rather than written premiums, should be used to determine underwriting
profit. Similarly, we should use claims incurred rather than claims paid.

Claims incurred is claims paid plus the increase in outstanding claims reserves.

Underwriting profit equals earned premiums less claims incurred less expenses incurred.

The cashflow diagram is a useful way to study the mechanics of a general insurer. The
diagram includes all the main monetary flows:
 premiums
 claims
 expenses (including commission)
 investment (in & out)
 reinsurance (in & out)
 dividends (ie out to shareholders)
 rights issues (ie in from shareholders)
 tax.

Reinsurance can protect insurance companies from various risks that may otherwise be too
large for them to bear.

The Actuarial Education Company © IFE: 2019 Examinations


Page 38 SP7-01: Insurance companies

The practice questions start on the next page so that you can
keep the chapter summaries together for revision purposes.

© IFE: 2019 Examinations The Actuarial Education Company


SP7-01: Insurance companies Page 39

Chapter 1 Practice Questions


1.1 List seven different types of reserve which, when added together, make up the total technical
reserves. Explain which of these is most likely to be zero.

1.2 Explain in layman’s terms what is meant by UPR.

1.3 State the two main approaches to estimating outstanding claims. Identify which of the two
should be used for estimating IBNR and which is more likely to benefit from actuarial input.

1.4 An insurance company produces provisional accounts, as at 31 December four days later on
4 January. These accounts do not include an AURR for its domestic household account.

When the draft accounts are drawn up two months later, the balance sheet does include an AURR
for household business. Assuming that there was no change in the reserving or accounting basis,
suggest reasons why the accounts might have been modified.

1.5 Insurance companies are described as ‘capital intensive’. Explain why they need so much capital.

1.6 Complete the following statements by filling in the blanks:

AURR is the _______ of _______ over _____, subject to a _______ of ________.

UPR is the portion of __________ that relates to __________ cover.

Claims incurred is defined as the __________ __________ plus the __________ in outstanding
claims reserves.

The main determinants of the investment strategy will be:


 the __________, __________, __________ and __________ of the __________
 the relative size of the __________ __________.

A reserve used to smooth profits from year to year is called a claims _______ _______.

The excess of __________ premiums over __________ claims and __________ is called the
________ __________.

1.7 State the reasons why insurance companies hold free reserves. [5]
Exam style

The Actuarial Education Company © IFE: 2019 Examinations


Page 40 SP7-01: Insurance companies

The solutions start on the next page so that you can


separate the questions and solutions.

© IFE: 2019 Examinations The Actuarial Education Company


SP7-01: Insurance companies Page 41

Chapter 1 Solutions
1.1 The seven types of reserves are:
 Outstanding reported claims
 IBNR
 IBNER
 Re-opened claims
 Claim handling expenses
 Unearned premium reserve
 Additional reserve for unexpired risks.

The AURR is most likely to be zero. If it is not zero, it indicates that we think that we have recently
been writing business on unprofitable terms.

1.2 UPR is the portion of premiums due or received in respect of policies already taken out that is set
aside for future accounting periods.

1.3 The two main approaches are:


 Estimates of individual outstanding reported claims.
 Statistical estimation of totals.

Use statistical methods for IBNR.

Actuarial input is more likely to be used in statistical estimation of totals.

1.4 Perhaps the accounts were modified because the company revised its view on the unexpired
risks. After two months of the new year, the company would know more. Perhaps there was a
run of large claims in January or February.

1.5 This is the same as asking why they need large free reserves:
 it may be a legal requirement
 to meet normal fluctuations in claims experience
 to protect against unexpected adverse experience (catastrophes, large claims etc)
 to support and attract new business
 to protect against failure of a third party, eg a reinsurer.

We discuss these ideas in more detail later in the course.

The Actuarial Education Company © IFE: 2019 Examinations


Page 42 SP7-01: Insurance companies

1.6 AURR is the excess of URR over UPR, subject to a minimum of zero.

UPR is the portion of premiums that relates to unexpired cover.

Claims incurred is defined as the claims paid plus the increase in outstanding claims reserves.

The main determinants of the investment strategy will be:


 the nature, term, currency and uncertainty of the liabilities
 the relative size of the free reserves.

A reserve used to smooth profits from year to year is called a claims equalisation reserve.

The excess of earned premiums over incurred claims and expenses is called the underwriting
result.

1.7 Why hold free reserves:


 it’s imprudent not to [½]
 it may be a statutory requirement [½]
 to prevent ruin from random claims fluctuation [½]
 to protect against bad claims experience, eg: [½]
– large individual claims [½]
– catastrophes [½]
– poor experience [½]
 to provide a cushion against other unexpected events, eg:
– investment failure [½]
– reinsurer failure [½]
– failure of other third parties [½]
– expense inflation [½]
– fraud [½]
 to increase freedom to: [½]
– use less reinsurance [½]
– invest more freely [½]
– accept bigger risks [½]
 to provide capital for expansion. [½]
[Maximum 5]

© IFE: 2019 Examinations The Actuarial Education Company


SP7-02: Insurance products – background Page 1

Insurance products –
background
Syllabus objectives

1.1 Describe the main types of general insurance product in terms of:
 the needs of customers
 the financial and other risks they pose for the general insurer including their
capital requirements and possible effect on solvency.

Covered in part in this chapter.

The Actuarial Education Company © IFE: 2019 Examinations


Page 2 SP7-02: Insurance products – background

0 Introduction
This chapter and the following chapter give an overview of the essential features that
distinguish the main types of general insurance products and so help to identify the crucial
aspects that influence the nature and extent of the risk to be covered by the insurance.
Reinsurance products are described in Chapters 5 and 6 and health products are covered
elsewhere in the syllabus (ie in other subjects).

In this chapter we look at general insurance principles and introduce the broad categories of
insurance products. In the next chapter we look at the different general insurance products in
detail.

© IFE: 2019 Examinations The Actuarial Education Company


SP7-02: Insurance products – background Page 3

1 General insurance principles


Before considering the main types of insurance and reinsurance, it is worth noting some
overriding features of all types of insurance.

You might recognise some of the following material from your earlier studies, although the
material here might be slightly different.

1.1 Insurable risk


For a risk to be insurable:

 the policyholder must have an interest in the risk being insured, to distinguish
between insurance and gambling

 a risk must be of a financial and reasonably quantifiable nature

 the amount payable by the insurance policy in the event of a claim must bear some
relationship to the financial loss incurred.

Ideally risk events also need to meet the following criteria if they are to be insurable:

 Individual risk events should be independent of each other.


However, as we have seen in Chapter 1, reinsurance is available to help cope with
situations where risks are not independent. In practice we won’t often get strict
independence but a low correlation is desirable.
 The probability of the event should be relatively small. In other words, an event that
is nearly certain to occur is not conducive to insurance.
For example, a house would not be insured if it stood on the edge of a crumbling cliff.
The premium to cover such a risk would be exorbitantly high.
 Large numbers of similar risks should be pooled to reduce the variance (of the
average claim size) and hence achieve more certainty.
With a large enough number of similar risks, we can appeal to the law of large numbers,
and see that the insurer will benefit from more predictable claims experience than the
policyholders. The similar risks should still be independent.
 There should be an overall limit on the liability undertaken by the insurer.
This would help the risk event meet the above criteria that it must be of a reasonably
quantifiable nature.
 Moral hazards should be eliminated as far as possible because these are difficult to
quantify, result in selection against the insurer and lead to unfairness in treatment
between one policyholder and another.

The Actuarial Education Company © IFE: 2019 Examinations


Page 4 SP7-02: Insurance products – background

Question

Define moral hazard.

Solution

Moral hazard is defined as the action of a party who behaves differently from the way that they
would behave if they were fully exposed to the circumstances of that action. The party behaves
inappropriately or less carefully than they would otherwise, leaving the organisation to bear some
of the consequences of the action. For example, a policyholder may start to leave spare keys
under the doormat after taking out household contents insurance because they feel less
concerned about possible adverse consequences.

See also ‘uberrima fides’ below.


 There should be sufficient existing statistical data / information to enable the insurer
to estimate the extent of the risk and its likelihood of occurrence.
We will see later in the course that there are a number of sources of data that might be
used if the insurer doesn’t ‘own’ any relevant data itself.

However, the fact that these ideal criteria are not always met in practice does not
necessarily mean that insurance cannot be found. Insurers may be prepared to underwrite
simply to generate income, to build a relationship or develop a new market
entrepreneurially. All risks were once insured for the first time.

1.2 Other terms


Other important general insurance terms you will come across are given in this section. Many
others are given in the Glossary – remember you will need to know the Glossary definitions for
the exam.

Uberrima fides
Latin for ‘utmost good faith’. This honesty principle is assumed to be observed by the
parties to an insurance, or reinsurance, contract. An alternative form is uberrimae fidei: ‘of
the utmost good faith’.

The principle of honesty underlies all insurance business. For example, misrepresentation or
non-disclosure of any material fact in the proposal can make the policy void. Each renewal of a
general insurance policy actually constitutes a new proposal, and the insured should disclose to
the insurer any material changes during the period covered by an insurance policy.

The insurer faces a variety of moral hazards because insureds may not abide by this legal principle
and, if they do not, it may be hard for the insurer to detect.

Multiple claims
Unlike life insurance policies, many general insurance policies allow the insured to claim as many
times as necessary during the period of cover, usually a year, provided by the policy.

© IFE: 2019 Examinations The Actuarial Education Company


SP7-02: Insurance products – background Page 5

Nil claim (or zero claim)


A claim that results in no payment by the insurer, because, for example:

 the claim is found not to be valid

 the amount of the loss turns out to be no greater than the excess

 the policyholder has reported a claim in order to comply with the conditions of the
policy, but has elected to meet the cost in order to preserve any entitlement to
no-claim discount.

Again, unlike simple life insurance policies, for which the claim event, eg death, is fairly clear cut,
a significant number of general insurance policies generate claims which are subsequently settled
by the insurer with no payment to the insured. This may be, for example, because the insurer is
not liable (eg the policy wording excludes that type of claim). However, nil claims still invariably
result in administrative expenses for the insurer.

Underinsurance
If the contents of your home are worth £30,000 then you should take out a home contents policy
with a sum insured of £30,000. However, you may choose a policy with a sum insured of only
£20,000 because you have under-estimated the value of your belongings, or because you think
the chance of losing more than £20,000 is very small. This is known as underinsurance.

Question

Explain why underinsurance is a hazard to the insurer.

Solution

An insurer will base its premium rates on the expected claim amounts. This will take into account
the expected frequency of claims and the expected size of the claims.

The higher the sum insured the higher the expected size of claims. Therefore if a policyholder
quotes a sum insured lower than the actual value of the contents, there is a risk that the
premiums will be inadequate.

Average
In order to prevent underinsurance, some property insurance policies, where premium rates are
based on the sum insured, contain an average clause. This provides that, if the sum insured is less
than the full value of the property at the time of a loss, the insurance payment will only be a
proportion of the value of the loss – the same proportion as the sum insured bears to the full
value. This is known as the principle of average.

For example, if you insured the contents of your house for £20,000 but the actual value of the
contents was £30,000 then the principle of average may apply if you make a claim. A claim for
£600 may result in a payment of only £400 because the claim is scaled down by 20,000/30,000.

The Actuarial Education Company © IFE: 2019 Examinations


Page 6 SP7-02: Insurance products – background

However, a word of warning; the term ‘average’ has different meanings in different contexts
within the general insurance world.

In non-marine insurance, the term relates to the practice of reducing the amount of a claim
in proportion to the extent of underinsurance.

In marine insurance, the term is generally used to describe damage or loss.

The word average derives from a Latin word havaria which broadly means ‘loss’. There are many
uses of this word within marine insurance. For example, a general average loss is a loss resulting
from a sacrifice or expenditure made by an individual for the benefit of others at a time of peril,
eg throwing cargo overboard from a boat to stop it sinking, thereby saving the remaining cargo
and the vessel.

Subrogation
The substitution of one party for another as creditor, with a transfer of rights and
responsibilities. It applies within insurance when an insurer accepts a claim by an insured,
thus assuming the responsibility for any liabilities or recoveries relating to the claim. For
example, the insurer will be responsible for defending legal disputes and will be entitled to
the proceeds from the sale of damaged or recovered property.

Subrogation means that the insurer replaces the policyholder in law and acquires all rights and
responsibilities in legal matters regarding the loss suffered, be it before or after the claim has
been settled.

For example, if you receive a payment from an insurer for replacement of your boat, following
serious damage or loss, then the original boat becomes the insurer’s property. The insurer may
then be able to recover a salvage value, for its own benefit.

Discovery period
A time limit, usually defined in the policy wording or through legislative precedent, placed
on the period within which claims must be reported. It generally applies to classes of
business where several years may elapse between the occurrence of the event or the
awareness of the condition that may give rise to a claim and the reporting of the claim to the
insurer: for example employers’ liability or professional indemnity.

Employers’ liability and professional indemnity insurance will be described in the next chapter.

The discovery period prevents claims being made to insurance companies many years after the
event that caused the claim. In principle it allows insurance companies to write off IBNR liabilities
from a contract once the discovery period has elapsed, although this has sometimes been
overridden by the courts.

The discovery period is often defined in the sunset clause. Both terms relate to the same topic.
The discovery period is the actual time limit; the sunset clause is a clause that defines the time
limit.

© IFE: 2019 Examinations The Actuarial Education Company


SP7-02: Insurance products – background Page 7

Underwriting
The process of consideration of an insurance risk. This includes assessing whether the
risk is acceptable and, if so, the appropriate premium, together with the terms and
conditions of the cover. It may also include assessing the risk in the context of the other
risks in the portfolio. The more individual the risk (for example most commercial lines), the
more detailed the consideration. The term is also used to denote the acceptance of
reinsurance and, by extension, the transacting of insurance business.

Underwriting is the process of assessing the risk for individual policies. As well as setting premium
rates, underwriters will specify excesses or exclusions to cover or possibly required improvements
to the risk before cover is provided. For small and standard homogeneous risks, insurers will
often provide insurance automatically, without referring the individual risks to the underwriters.

1.3 The policy document


The policy document is important. It sets out the terms and conditions under which an
insurer is liable to pay insurance claims in specific circumstances and must, therefore, be
carefully worded to cover all possible circumstances under which payment will (and will
not) be made.

The relationship between the insured and the insurer is governed by the policy document. It is a
legal and binding contract and hence subject to contract law.

Policy forms are normally standard for all personal lines business and small commercial
policies, in the sense that an insurer will issue the same wording to all policyholders. Items
that vary between policyholders will be included in a schedule.

Examples of common items in a schedule are:

 details of vehicle / property / people covered

 excess applied

 any limits to the cover

 exclusions

 time limits

 whether or not any optional covers have been taken

 details of insurance premium paid.

Larger commercial and London Market risks tend to have policies that are individually made
for the particular policyholder, possibly assembled from a library of standard clauses.

The Actuarial Education Company © IFE: 2019 Examinations


Page 8 SP7-02: Insurance products – background

1.4 Exclusions
Exclusions are clauses in a policy that limit the circumstances in which a claim may be
made.

Some examples of common exclusions are:


 suicide or self-inflicted injury
 dangerous pastimes
 losses resulting from illegal activity by the policyholder
 war, terrorism, civil riot.

Question

Suggest some more examples of common exclusions.

Solution

 nuclear or radio-active risks


 earthquakes
 unseaworthiness of vessels
 loss of money and documents

Exclusions can apply to certain perils (eg terrorism) or particular types of loss (eg cash, whatever
the peril).

Question

List the exclusions that are likely to be applied on a private motor policy.

Solution

 certain specified uses, eg for business (unless explicitly allowed), racing


 depreciation, wear and tear, or damage to car tyres
 where there was an element of illegality, eg the driver did not hold a driving licence or
was under the influence of alcohol or drugs
 where the insured did not take due care, eg left the keys in the ignition
 insured’s personal accident benefits where the insured is very old
 losses arising in consequence of earthquakes, war, riot or civil commotion

© IFE: 2019 Examinations The Actuarial Education Company


SP7-02: Insurance products – background Page 9

Exclusions are used to avoid payment by the insurer in situations where the:

 policyholder is at an advantage through possessing greater personal information


about the likelihood of a claim

 claim event is largely under the control of the policyholder

 claim event would be very difficult to verify or

 loss occurs as part of the normal course of events and could be considered to be
depreciation

 probability of a claim is very high

 risk cannot be reasonably estimated.

Remember from Section 1.1 that one of the criteria for an event to be insurable is for the
probability of the event to be relatively small.

Exclusions are also used where the risk is covered by a third party such as the government.
Terrorism is an example of such a risk in the UK.

Exclusions are also used to limit the scope of the policy to make it more appropriate for a
particular target market or to reduce the premium for competitive reasons.

Exclusions are sometimes used to reduce the risk of moral hazard and fraud. For example, theft
of a vehicle when the keys have been left in the ignition might be excluded from a motor policy.
This reduces the moral hazard of policyholders’ carelessness, which might result from the
existence of insurance cover.

Indemnity for loss of cash might be excluded from a household contents policy. This reduces the
risk of fraudulent exaggeration of loss amounts, as it would be very easy for a policyholder to
claim, but difficult to verify, that large sums of cash had been stolen when a burglary has taken
place.

The Actuarial Education Company © IFE: 2019 Examinations


Page 10 SP7-02: Insurance products – background

2 Types of product
The types of insurance cover provided by general insurance products can be classified
under four main headings:

 liability

 property damage

 financial loss

 fixed benefits.

This section gives a brief overview of these categories of insurance product. They are described in
detail in the next chapter.

2.1 Liability
The essential characteristic of liability insurance is providing indemnity where the insured is
legally liable to pay compensation to a third party.

Examples of liability insurance include:


 employers’ liability – where the insurance indemnifies the employer against
compensation payable to employees for losses that they suffer as a result of negligence of
the employer
 motor third party liability – where the insurance indemnifies the owner of a motor vehicle
against compensation payable to third parties for personal injury or damage to their
property (eg their vehicle)
 product liability – where the insurance indemnifies a manufacturer against compensation
to a third party for losses that they suffer as a result of a product fault.

2.2 Property damage


The main characteristic of property damage insurance is providing indemnity to the insured for
loss of, or damage to, the policyholder’s own property.

Examples of property damage insurance include:


 motor insurance
 buildings insurance, which includes both residential buildings (houses) and commercial
buildings, such as offices, shops and industrial buildings
 contents insurance.

© IFE: 2019 Examinations The Actuarial Education Company


SP7-02: Insurance products – background Page 11

2.3 Financial loss


Financial loss indemnifies the insured against financial losses arising from certain causes.

Examples of financial loss insurance include:


 creditor insurance (also called Payment Protection Insurance, or PPI) – where the policy
will make regular loan repayments if the policyholder becomes disabled (so that they
cannot work) or otherwise unemployed
 business interruption cover (also called consequential loss insurance) – where the policy
will pay out to compensate the policyholder for not being able to conduct their business,
eg as a result of a fire in the building.

2.4 Fixed benefit


The defining characteristic of fixed benefit insurance is that the benefits are usually specified,
fixed amounts, payable on certain losses occurring.

An example is personal accident insurance, where the insured receives a fixed payment on
suffering a specified injury, eg the loss of a limb.

2.5 Combining categories of insurance


Insurance policies may comprise elements of one or more of these types of cover.

For example, a typical comprehensive motor insurance policy will provide cover for:

 compensation for personal injury to third parties and damage to their property

 compensation for loss of or damage to the insured’s vehicle

 fixed benefits in the event of defined categories of personal accident to the insured.

Therefore, a typical motor policy may comprise elements of liability, property damage and fixed
benefit cover.

Similarly, a typical policy covering household contents will provide cover for the financial
loss, property damage and liability of the insured (and any member of the insured’s family
living at the same address) to third parties.

Therefore a household contents policy may comprise elements of property damage and liability
cover.

Question

Suggest examples of how, under a household contents policy, the insured may be liable to a third
party.

The Actuarial Education Company © IFE: 2019 Examinations


Page 12 SP7-02: Insurance products – background

Solution

Cover may be provided in respect of visitors’ belongings that may for example be damaged in a
fire.

The insured may also be indemnified for personal liability arising out of accidents to members of
the public somehow caused by the property of the insured, eg a garden wall falling on a
pedestrian.

Policies that are likely to be combined as a single product are described further in Chapter 3.

2.6 The customer

Personal lines and commercial lines business


General insurance may be sold to both individuals and businesses.

Insurance products sold to individuals are known as personal lines business. They include private
motor, domestic household, personal accident and travel insurance.

Insurance products sold to businesses are known as commercial lines or group business.

Package policies
Policies for businesses often include all types of cover that the business needs (apart from
motor, in the case of small businesses); appropriately, they are called ‘package policies’.

Question

List the types of cover a small retailer might require as part of a ‘package policy’.

Solution

Other than motor, which is not usually included in ‘package policies’, a small retailer might
require cover to:
 compensate employees for accidents occurring due to negligence of the employer
(employers’ liability and personal accident)
 compensate third parties, eg customers, for accidents occurring in the shop / on the shop
premises (public liability)
 protect against damage to the shop (commercial buildings cover)
 compensate for lost revenues should the shop be unable to trade (business interruption
cover)
 protect against loss of or damage to stocks while in the shop (moveable property
(contents) cover)
 protect against loss of or damage to stocks while in transit (goods in transit cover)

© IFE: 2019 Examinations The Actuarial Education Company


SP7-02: Insurance products – background Page 13

 protect against bad debts by suppliers of goods / materials (credit insurance)


 protect against bad debt by customers buying goods on trade credit (credit insurance)
 protect against financial losses due to dishonest actions of employees (fidelity guarantee
insurance)
 pay any legal expenses as a result of legal proceeding being initiated against the shop or
the shop needing to initiate legal proceedings against another party (legal expenses
cover).

Don’t worry if you didn’t get all (or even nearly all) of these points. Hopefully by the time you’ve
finished reading the next chapter, it won’t seem like such a tall order.

The Actuarial Education Company © IFE: 2019 Examinations


Page 14 SP7-02: Insurance products – background

3 Cover provided
For each of the four main types of cover, the features will be discussed in the following
sections under six headings:

 benefits

 insured perils

 basis for cover


 measures of exposure to which premiums are related

 claim characteristics

 risk factors and rating factors.

In the next chapter, the products are discussed under the headings of the four main types of
cover that they provide.

Any unusual words or phrases, for example risk factors used in the above paragraph, will be
explained fully later in this chapter.

The following general comments relate to these six headings.

3.1 Benefits
The benefits provided by an insurance policy will vary between types of insurance and
between insurers. Typically, the intention is to provide the insured with money to cover his
or her financial loss as a result of an insured event, although policies that provide benefits
in kind are also possible.

The provision of a courtesy car under a motor policy (while your car is being repaired) is an
example of a benefit in kind provided by an insurance policy.

Question

Describe the benefits you would expect to receive for each of the following claims:
 loss of luggage whilst on holiday
 loss of a finger
 loss (to your vehicle and injury to yourself) as a result of a car accident.

Solution

Loss of luggage whilst on holiday

Your insurance is likely to cover you for the cost of the luggage, subject to any limits that may
apply. This is an example of the benefit being money to cover financial loss as a result of an
insured event.

© IFE: 2019 Examinations The Actuarial Education Company


SP7-02: Insurance products – background Page 15

Loss of a finger

Your insurance is likely to provide a fixed benefit that is intended to go at least some way towards
compensating you for the loss. It is usually impossible to quantify exactly how much the payment
should be.

Loss (to vehicle and self) as a result of a car accident

Your insurance is likely to cover you for the cost of damage to your vehicle and will also
compensate you for your discomfort or inconvenience. The former should be fairly
straightforward to determine (as it should cover the financial loss incurred). The latter may be
more complicated: the insurance policy may specify the amount of the payment, which would
depend upon the type of injury, or alternatively the payment may have to be determined by the
courts.

In some territories, certain insurance cover is compulsory.

3.2 Insured perils


A peril is a type of event that may cause losses, eg theft or flood.

In virtually all types of insurance, it would be difficult to list all possible perils against which
a policyholder might wish to be protected. Claims can result from a very large number of
perils; not all of them will be currently apparent and not all of them will be standard to every
insurance product on the market.

The perils are also likely to vary by country. For example, in some countries it is more likely (or
necessary) for perils such as volcanic activity or stampeding animals to be included in the cover.

The precise form of the cover provided in respect of any insured peril may vary between
insurers, and the benefits provided may be defined in any one of a number of ways.

Exam tip
The examiners will expect candidates to apply some common sense to extend the examples
or groupings given for the products mentioned in this chapter and the following chapter.

3.3 Basis for cover


This section describes the basis on which different types of cover may be written. Insurance
policies may be written as:
 losses-occurring policies
 claims-made policies.

Losses-occurring policy
A losses-occurring policy is a policy providing cover for losses occurring in the defined period, no
matter when they are reported. A losses-occurring policy may also be referred to as a
‘claims-occurring policy’.

The Actuarial Education Company © IFE: 2019 Examinations


Page 16 SP7-02: Insurance products – background

Claims-made policy
A claims-made policy covers all claims reported to an insurer within the policy period, irrespective
of when they occurred. A claims-made policy may also be referred to as a ‘claims-reported
policy’.

Question

An individual purchases a one-year motor insurance policy from Cars ‘R’ Us on 1 January 2018.
The following year, they buy another one-year policy from Cars 4 You.

On 27 December 2018, the individual has an accident, which causes significant damage to their
car. They decide to wait until the New Year to report the claim.

Identify which insurer the individual should contact if the policies are written on a:
(a) losses-occurring basis
(b) claims-made basis.

Justify which basis is more common for this type of insurance.

Solution

(a) The accident occurred when the individual was insured by Cars ‘R’ Us. Therefore for a
losses-occurring basis, the individual should contact Cars ‘R’ Us to report the claim.

(b) The individual is reporting the claim while the insurance cover is being provided by Cars 4
You. Therefore for a claims-made basis, the individual should contact Cars 4 You to report
the claim.

A losses-occurring basis seems more sensible, as this is not affected by when the individual
chooses to report the claim. In other words, under this basis, the individual cannot choose which
insurer should pay the claim.

3.4 Measures of exposure to which premiums are related


This section discusses the underlying principles involved in choosing exposure measures. More
specific comments on the various measures used for different insurance classes are given later in
the chapter. As you read the Core Reading think actively about how the practical choices made
have been influenced by the principles.

One of the objectives of the insurer when setting premium rates is to charge a premium which
accurately reflects the amount of risk. The pure risk premium is the premium required to cover
the expected claim amount only. No allowance is made for expenses or profit. For example, if a
particular policy gives rise to a claim of £1,000 with a probability of 10% and no claim otherwise,
the pure risk premium would be £100. The actual premium charged would then need to cover
this level of risk and also include an allowance for expenses, profit and investment income and
any other loadings.

© IFE: 2019 Examinations The Actuarial Education Company


SP7-02: Insurance products – background Page 17

Selection, anti-selection
If an insurer does not charge premiums that accurately reflect the amount of risk, the insurer may
suffer from selection (strictly called anti-selection in this context). If in the example above the
insurer charged a premium based on a risk premium of £120, they may not get the business if
other more accurate insurers used a risk premium of £100. On the other hand, if they allowed for
only £80 they might get the business but make a loss.

In general, insurers who fail to charge premiums that reflect the amount of risk run the danger of
getting little business (premiums too high) or lots of loss-making business (premiums too low).
Assessing the amount of risk involved with a particular policy is a critical aspect of an insurer’s
work.

Question

Explain whether the dangers of selection still exist if all insurers charge the same premiums.

Solution

Yes. If insurance is optional, policyholders may select against the insurance market. Those least
likely to claim may choose not to purchase insurance and vice versa.

Exposure
In practice, insurance proposals do not come with neat little labels stating the probability of claim
and the expected cost per claim. The amount of risk underlying an individual policy is often
largely unknown. In fact, the amount of risk is never known exactly.

For the purpose of setting premiums, insurance companies try to determine measures which give
an indication of how much risk there is within each policy. These measures are called measures of
exposure.

In practice, the chosen measure of exposure should meet two key criteria:
(a) It should be a good measure of the amount of risk, allowing for both the expected
frequency of claim and the expected severity of claim (ie the average claim amount). In
other words, the total expected claim amount should be proportional to the exposure.
(b) It should be practical. This criterion embraces several aspects. The measure should be
objectively measurable and should be easily obtainable, verifiable and not open to
manipulation.

There are rarely perfect measures of exposure which completely define the amount of risk
underlying each policy. For most classes in fact, the exposure measure used is a basic principal
indicator of risk. Given a measure of exposure, risk can be further classified by rating factors.
These are discussed further in Section 3.6.

The Actuarial Education Company © IFE: 2019 Examinations


Page 18 SP7-02: Insurance products – background

In some types of insurance there is a degree of choice as to the measure of exposure, as it


will not be immediately obvious which reliable and measurable factor bears the closest
relationship to the expected claim amounts. Examples of measures used in some classes
are given in the next chapter.

An exposure measure isn’t just a factor that is used to calculate a premium. It is a measure, which
when added over all policies, gives an indicator of the ‘volume of business’ or the total amount of
risk. As such, the expected total claim amount should be proportional to the exposure. Clearly
the best measure of exposure is the expected claim amount on each policy, but exposure should
also be a simple measure that is verifiable etc. The number of policies always has this property
but there may be another quantifiable factor on each policy that works better.

Question

Suggest another possible exposure measure, other than the number of policies, that would give a
reasonable indicator of the total level of risk on any portfolio of business.

Solution

Premiums. The total premium should be a good indicator of the total risk on a portfolio of
business.

3.5 Claim characteristics


This topic was introduced in Chapter 1.

As well as the amount that becomes payable, the claim characteristics refer to the ways in
which and speed with which the claims:

 originate

 are notified

 are settled and paid

 are, on occasion, reopened.

Claim characteristics also refer to the frequency with which claims are made. For some lines of
business, such as private motor insurance, claims occur with a far higher frequency than other
lines, such as various liability classes.

When assessing claim characteristics, it is therefore necessary to consider:


 delays (event, reporting, settlement etc) and hence whether claims are short-tail or
long-tail claims
 claim frequency
 claim severity / amount (allowing for accumulations, catastrophes etc).

Claim frequencies and claim costs are often discussed in terms of a claim frequency / claim cost
distribution. This is just a statistical distribution for claim frequency / individual claim cost.

© IFE: 2019 Examinations The Actuarial Education Company


SP7-02: Insurance products – background Page 19

All these features have implications for the assessment of risk borne by the insurer.

3.6 Risk factors and rating factors


The fact that exposure measures are never perfect measures of the amount of risk means that
there is scope for refinement of the premium even after applying the exposure measure. An
example will help to illustrate the point.

Example
Consider a special class of insurance whereby policyholders who bang their heads on doorframes
or low ceilings of homes are entitled to an insurance payment. The worse the injury, the more
the payment.

What exposure measure should we use to determine premiums? There are several candidates,
but let’s suppose that we decide to use the policyholder’s height as the basis for setting
premiums. Our measure of exposure would then be centimetres. Exposure measures often
incorporate time units to reflect the fact that policies for two years should be charged twice the
premium of one-year policies. Hence our exposure measure is centimetre-years. If the premium
is £0.10 per centimetre-year, a policyholder of height 175cm would be charged an annual
premium of £17.50.

In actual fact, centimetre-year is probably not the best exposure measure to use. The reasons
why include:
 The risk does not increase linearly with the exposure measure. For example, a child
whose height is 110cm is no more likely to hit their head than one whose height is 90cm
(and possibly less likely, if they are a couple of years older).
 It is not very practical. It would not be easy to verify (cheaply) and will be continuously
increasing for the young.

A better measure of exposure is probably person-year. It is much more practical and when all
other factors are constant, it will be proportional to the expected claim amount.

In this example, our insurance company may still be exposed to the possibility of selection. The
premium rating structure has done nothing to incorporate allowances for the following factors:
 the non-linearity of the relationship between height and risk
 short-sighted people may hit their heads more often
 clumsy people may hit their heads more often
 people who move around quickly will hit their heads harder
 people living in homes with low doorframes and ceilings will hurt themselves more often.

These further considerations of risk are called risk factors. Note that the exposure measure itself
is also a risk factor. Risk factors are any factors that have a bearing on the amount of risk.

The Actuarial Education Company © IFE: 2019 Examinations


Page 20 SP7-02: Insurance products – background

To prevent selection, the insurer must try to incorporate these factors into the premium rating
process. If our 175cm policyholder turned out to be short-sighted, clumsy, fast and the inhabitant
of a 16th Century cottage, the premium might be increased from £17.50 to, say, £40 each year.

Risk factors will depend on precisely the cover provided, but the factors applicable in most
cases are given in the next chapter.

Sometimes direct use of the risk factors in the rating process is not practical (eg the risk factors
may not be easily measured). In these cases, other factors that are more easily identified may be
used as proxies for the underlying risk factors. The expression for the factors actually used in the
premium rating process is rating factors.

Rating factors will be either objectively measurable risk factors or other factors that can be
used as reliable proxies for the risk factors. Where credible exposure and claims data exist,
experience rating can be used to take account of residual risk factors.

This is saying that we can use the actual claims experience of the insured in the past to help set an
appropriate premium for the future. For example, the number of claims you have made on your
motor policy is likely to affect your insurance premium. Experience rating is discussed in more
detail in Subject SP8.

Note that in some countries, certain rating factors are not allowed to be used. For example, the
European Court of Justice ruled that, with effect from 21 December 2012, a person’s gender can
no longer be used to calculate insurance premiums.

You may hear the term underwriting factors used by some practitioners. There are some
differences in how this term is used with some taking it as synonymous with rating factors.
However most people (and importantly for Subject SP7, the Core Reading), take it to mean rating
factors plus subjective factors that, although they cannot be measured, the underwriter takes into
account in setting premiums or policy conditions. A subjective factor in our head-banging
example might be how accident-prone the person appeared to be.

3.7 Combining exposure measures, risk factors and rating factors


The following statements follow on from the discussion above:
(a) The more heterogeneous the class of insurance and the types of risk covered, the greater
the number of risk factors needed to identify or define the amount of risk.
If, for example, we were dealing with a class of business where all the risks were identical,
there would be a single premium rate that applied to all policyholders. In this case there
would be no additional rating factors.
At the other extreme is a class of insurance like commercial property insurance where the
risks are very heterogeneous. The amount of risk will vary from one insured to another
according to a number of risk factors. We will see later that for commercial property
insurance, many rating factors are used to calculate the premium.
(b) The better the measure of exposure in identifying the amount of risk, the lesser the
importance of other rating factors.

© IFE: 2019 Examinations The Actuarial Education Company


SP7-02: Insurance products – background Page 21

(c) The choice of rating factors will depend on the choice of exposure measure.

Sometimes an exposure measure used in practice does not satisfy the ideal property of
proportionality exactly. The exposure measure can be used as a rating factor as well. A real-life
example that you will see is domestic property insurance in which the size of the property (ie the
sum insured) is taken as the exposure measure.

For contents insurance the main peril is theft, but the sum insured includes the value of items that
are rarely stolen, such as carpets, curtains, the toilet brush etc. If the value of the goods likely to
be stolen, as a proportion of the total sum insured, tends to decrease as the total sum insured
increases then the expected loss increases less than proportionately with the sum insured.
Therefore an insurer might use sum insured as a rating factor as well as an exposure measure.

It may do this by charging different rates for different bands of sum insured. An example is shown
in the following table.

Sum insured band Premium rate per 1,000 sum insured


0 – 10,000 18.6
10,001 – 20,000 17.5
20,001 – 30,000 16.9
> 30,000 16.7

In one sense, the exposure measure is always a ‘rating factor’ as it always affects the premium.
However, ‘rating factor’ is usually used to mean a factor that affects the premium rate, hence
these comments.

The rating factors used in practice will vary between different insurers as they attempt to
find a competitive edge. The more common rating factors, some of which have been
justified by statistical analysis, are given in the next chapter.

Question

Suggest circumstances when an insurer’s underwriters would not look at the risk and rating factor
details for new policy proposals.

Solution

 the potential claim amount is very small


 risks are very homogeneous, so standard book rates apply to all risks

The Actuarial Education Company © IFE: 2019 Examinations


Page 22 SP7-02: Insurance products – background

4 How does the insurer’s risk vary by class?


Some classes of business cause insurers more risk and uncertainty than others because of
the nature of the risks involved and the claims that can arise from those risks.

4.1 Homogeneity of risks


Where there is a lack of homogeneity of risks within a class of business, there is greater risk to the
insurer.

Even within a given rating category, exposures can be variable and dissimilar.

In commercial buildings insurance, for instance, properties can vary considerably by size,
construction and value. Moreover, you may have a mixture of properties, ranging from small
shops to large chemical factories.

You can therefore have policyholders with very different risk potential within the same rating
category. This will be reflected in the subsequent claims experience, and its inherent variability.
This will be particularly true for some of the liability classes.

For certain other classes, the risks within each rating category tend to be more homogeneous,
and the experience will therefore tend to be more predictable from year to year. Private motor is
an example of this, due to the large number of rating factors used to categorise risks. Some
insurers also insure vast numbers of (independent) cars, which reduces the relative variability of
the experience, and so also increases the predictability.

4.2 Non-independence of exposures


The variability is increased where exposures are not independent, as this can lead to an
accumulation of risk. For example, if the majority of policyholders in a household insurance
portfolio live in a certain area of the country, there will be a disproportionate claim cost if
there is a local catastrophe. If the book has exposures spread across the country (so
exposure is more independent geographically), the claim cost will not be affected so greatly
by a local catastrophe.

Some classes will lend themselves more to independence than others. For example, a
personal motor insurance portfolio should have a reasonable spread of exposures, whereas
creditor insurance will be heavily linked to the state of the economy and unemployment.

The policyholders of a motor insurer are not generally concentrated in one geographical region
because business tends to be sold nationally.

Creditor insurance provides cover to insureds who are subject to obligations to repay credit
advances or debt. It is described in detail in the next chapter.

4.3 Changing risk


Another feature leading to risk uncertainty is that the nature of the risk may change, both
during a particular policy year and in succeeding years.

© IFE: 2019 Examinations The Actuarial Education Company


SP7-02: Insurance products – background Page 23

In some cases a change in the underlying risk should be notified to the insurer by the
policyholder. For example, a motor policyholder should inform the insurer if he or she
moves house; this is a rating factor.

In extreme cases, failure to notify the insurer could make the cover void.

Examples where changes should be notified include:

 fire precautions within a building

 drivers or location under a motor policy

 economic conditions under a mortgage indemnity policy.

In the last bullet point the insured would be required to notify the insurer if there was a change to
their own financial position, eg a change in their income or employment status. The insurer will
be aware of the wider economic climate.

In each example above, the insurer’s risk portfolio will change over time from that originally
written, leading to difficulties in pricing and managing that portfolio.

Changes in background conditions, such as economic conditions, would not normally need
to be notified.

Some classes of risk vary more than others in this respect. Employers’ liability can also fluctuate
markedly in certain years, due to the turnover of employees, or through business acquisitions.
Other classes of risk can be fairly stable in the short term.

Question

Suggest how stable the risk would be for a typical simple industrial buildings and contents policy
(with no business interruption cover) for a manufacturing company.

Solution

Manufacturing companies are unlikely to change the nature of their business markedly during any
given year, although in the longer term they may do so. The value of the stocks and output may
vary, however, depending on the economic climate and the time of year.

Overall, the risk will be fairly stable unless stock levels are significant and variable.

4.4 How do claim characteristics affect risk?

Numbers of claims
In general insurance, as distinct from life assurance, there is often no limit to the number of
claims that can arise from a policy while it is in force.

Some classes, such as motor and household contents, have a relatively high claim
frequency, with sometimes 15% or more of policies having a claim each year. Other types
of cover can have much lower claims frequency, particularly commercial classes with high
excesses.

The Actuarial Education Company © IFE: 2019 Examinations


Page 24 SP7-02: Insurance products – background

Examples of such commercial classes include public and product liability and commercial
buildings.

Claim cost
The cost of a claim from any given policy cannot be predetermined except in a very small
number of classes where there is a fixed benefit (such as personal accident) and is often
very variable.

This applies when the basis of cover is indemnity rather than fixed benefit. Personal accident is a
case where cover is generally on a fixed benefit basis.

While there will often be a maximum sum insured stated (or implied) in the policy, relatively
few claims will be settled for that maximum sum. The usual principle of general insurance
is to indemnify the insured for any losses or claims made upon them. Most claims will
therefore be for only a portion of the maximum cover, depending on the circumstances of
the incident.

For most classes, a large proportion of claims will be for small amounts (the attritional
claims) and there will be only a small number of large claims. The precise distribution of
claim amounts will, however, vary greatly by class, in particular between property and
bodily injury claims, and also year by year.

When a probability distribution is used to represent the distribution of the sizes of claims in
a class of business it is conventional to use highly skewed distributions with no theoretical
upper limit, such as lognormal or Pareto.

Exposures are not all identical, even within the same class of business, and each exposure
has its own claim cost distribution. Exposures are often too small to yield useful
information individually, so their experience is aggregated to derive parameters of the claim
cost distributions when the risks are homogenous enough and attritional. The shape of
these claim cost distributions depends on risk characteristics demonstrated by different
classes of business and the insured’s risk profile, among other factors.

Claim inflation
Where a class is exposed significantly to the risk of inflation, any unexpected change in
inflation, for whatever reason, will affect the risk profile of that business.

© IFE: 2019 Examinations The Actuarial Education Company


SP7-02: Insurance products – background Page 25

Different classes of business are affected by inflation in different ways:

 Property insurance responds mostly to the cost of property, and claims will tend to
increase in line with general inflation, although repair costs can be linked to
earnings.

 A large proportion of motor claims cost is for the repair of vehicles and will be
affected by the level of earnings since these determine labour costs.

 Liability classes are often subject to higher levels of inflation, especially on personal
injury claims, as there is a trend to more generous compensation in many markets.
However, this may arise in steps rather than as a continuous process of inflation, as
landmark legal judgements are handed down or legal reform comes into effect.

Delay patterns
An insurer’s ability to manage a general insurance account is further complicated, and
hence the degree of risk increased, by the length of time that it takes for claims to emerge,
to be reported and to be settled.

Claim delays can arise for various reasons, for example:

 a delay between the incident occurring and the policyholder becoming aware of it,
eg the time between a burglary occurring in a property and the policyholder returning
home to discover it
 a delay between the insured becoming aware of the loss and reporting it, eg the
policyholder may be slow to report a claim if it is quite small
 a delay before sufficient details of the incident can be gathered to assess the value
of the claim

 a delay until an injured party’s condition stabilises to the extent that assessment of
damages is appropriate, eg to assess whether the injured party will recover or is now
permanently disabled
 delay in agreeing the actual value at which the claim is to be settled, and the
payment of this amount to the insured.

The typical extent of such delays will differ according to the class of business.

Bodily injury cases tend to have the longest delay tails, owing to the contentious issues of
many of the claims involved, often with the need for legal proceedings. This may be
worsened by the greater likelihood of latent claims or claims for industrial disease where
the delay from event to reporting can be considerable.

For example, some liability claims relating to disease and pollution have taken decades to emerge.
In many cases, the insurers were largely ignorant of the potential risks at the time that cover was
given. Such claims are known as latent claims. The possibility of latent claims increases the level
of risk for insurers.

By contrast, property damage classes have a much shorter delay tail, and hence in this
respect a lower degree of risk, since the losses are more immediately apparent and can
usually be valued reasonably accurately by a competent assessor.

The Actuarial Education Company © IFE: 2019 Examinations


Page 26 SP7-02: Insurance products – background

Until all claims have been settled from a given exposure period, the insurer is uncertain both as to
the number of such claims and, more importantly, their cost. This will have consequences for
setting reserves when drawing up accounts, and for evaluating future rating needs.

Also, while claims are outstanding they will be subject to increases in cost due to inflation, sudden
jumps in court awards, changing legislation and indirect taxes. This further complicates the
insurer’s ability to estimate future claim settlement costs.

Question

One of your student colleagues says that for motor third party cover, liability claims are long
tailed and property claims are short tailed. Explain whether you agree with the student.

Solution

Not quite. Many liability claims involve property damage only (for example, if you accidentally
scrape against somebody else’s car in a car park). These claims will also be relatively short tailed
(although there may be some delay whilst you argue whose fault it was). The student has
therefore confused the term liability with bodily injury, a common mistake. This will be discussed
in more detail in Chapter 3.

Variability of experience
Depending on the class involved, the numbers of claims can vary according to such
features as unusually bad weather, the economic situation and catastrophes.

Some classes are more susceptible than others to individual claims that are large enough to
affect the results of the whole class.

The pattern of claims arising within a year for any given class will rarely, if ever, equate with the
theoretical claims cost distribution for that class. One or more very large claims can easily swamp
the normal cost.

Accumulations
The insurer can be exposed to accumulations of risk if the portfolio is unbalanced. There is a
possibility of many claims arising from a single event or a single cause.

Some classes of business are (particularly) subject to accumulations of risk.

Property classes are prone to catastrophes: external events that affect a large number of
policies at the same time, possibly causing mainly small claims, but in aggregate giving rise
to a very large total loss. The most important examples are extreme weather, earthquakes
and civil disturbance.

Such accumulations tend to be based on single events affecting a number of risks in the same
geographic region.

© IFE: 2019 Examinations The Actuarial Education Company


SP7-02: Insurance products – background Page 27

However, such accumulations need not be single incidents; for example, a very dry summer
may cause a large number of subsidence claims, although there is no single event to link
them.

Other forms of accumulation are not necessarily based on geographical concentration, eg for
creditor insurance, there may be many claims triggered by high unemployment.

Liability insurance is less susceptible to large single incident accumulation losses, but a
single cause may give rise to a large number of claims. The most obvious example is
exposure to asbestos, which has given rise to claims under liability policies that are
expected to exceed any single event catastrophe.

Fraudulent claims
Certain classes are more exposed than others to the risk that the insured will make false or
invalid claims, or exaggerate the amount claimed following a loss. Often, it will be difficult
or uneconomic for the insurer to check whether the claims are genuine or not. At the
extreme, such false claims could include arson and embezzlement. The rate of fraudulent
claims has been observed to increase in times of economic stringency.

Examples under personal lines cover can include:


 Motor: a policyholder, with non-comprehensive cover, suffers non-claimable damage to
the insured vehicle. The policyholder then drives the vehicle to a remote spot, destroys it
by fire and claims under the fire and theft sections of the policy.
 Household contents: claims are made for loss of fictitious articles, or the values are
exaggerated.
 Buildings: claims are made for defective roofs, which are really due to natural wear and
tear.
 Buildings: after a genuine insured loss, a builder may offer to add the amount of the
insurance excess to their invoice. The insurer then unwittingly pays the full amount of the
actual repair cost.

Under commercial lines of business, examples can include:


 Buildings: the insured deliberately sets fire to the insured premises because the business
is losing money, and the insured stock is actually of little value.
 Liability: malicious or extravagant claims are made by third parties because they have
seen people with valid claims receive substantial awards.
 Financial loss: embezzlement, fraud or falsification of accounts is carried out by the
insured, in order to exaggerate a loss.

In all cases, these risks tend to be greatest during periods of economic downturn and depression.
The insured sees the insurer as an illegal means of recovering losses that may threaten the
insured’s own financial position.

The Actuarial Education Company © IFE: 2019 Examinations


Page 28 SP7-02: Insurance products – background

Question

If you were the owner and manager of a general insurance company, suggest what you would do
to try to reduce the fraudulent claims against you.

Solution

Things that insurers have tried include:


 making the policy wording as tight as possible, and reviewing it regularly in the light of
market and judicial changes
 devising contracts which minimise the fraud risks, eg minimum and indexed sums insured
for household contents policies
 working together with other insurers and industry bodies to try to identify and punish
persistent offenders
 random spot checks on claims, even smaller ones
 having repairs done by a small number of approved firms (rather than at the choice of the
claimant)
 insisting on the police being involved before paying out on a theft claim
 publicity to advise against it, eg ‘It’s a crime to …’ and ‘Look what happened to this
fraudster ...’.

© IFE: 2019 Examinations The Actuarial Education Company


SP7-02: Insurance products – background Page 29

5 Capital requirements and impact on solvency


General insurers require resources beyond those needed to cover their technical liabilities
in respect of the business they have written. The excess of the value of the assets over the
value of the liabilities is known as the capital or solvency margin of the company.

You may hear it referred to as the free reserves of the company.

The amount of capital held is subject to minimum requirements if a company is to be


allowed to continue to trade. These requirements are laid down in legislation that varies
from one country to another, although all countries in the EU have a common minimum
solvency regime. This solvency margin will act as a safety margin against the uncertainty
surrounding the ultimate future cost of liabilities and the value of the assets supporting
them.

If the number or cost of claims is greater than the reserves held to meet the claims, then the
solvency margin (free reserves) will be needed to pay the claims. Unexpected claims may result in
the need to realise assets to pay those claims. Free reserves will guard against the danger of
volatile asset values.

Question

Suggest other reasons why a general insurer needs to hold free reserves.

Solution

 it may have to by law


 to guard against catastrophes / accumulations / latent claims
 to guard against lower than expected investment return
 to give more investment freedom
 to guard against higher than expected expenses or expense inflation
 to guard against failure of a third party, for example a reinsurer or broker
 to reduce the need for reinsurance
 to attract new business
 to demonstrate financial strength, eg to regulators, shareholders, analysts

Determining the capital that a general insurer ought to hold to cater adequately for the risks
associated with the business it transacts is a complex issue. The longer the tail of the
business written the greater the uncertainty and hence, other things being equal, the more
capital will be required. In setting its capital requirements, beyond those specified by law,
the general insurer will need to take into account the uncertainty and variability of the
business it writes.

The Actuarial Education Company © IFE: 2019 Examinations


Page 30 SP7-02: Insurance products – background

Generally, for two classes where the same amount of business – measured by premium
income – has been written, the capital requirements should be larger for the class with the
greater uncertainty and variability in its future claims experience and in the run-off of
reserves.

Question

Write down the following classes in decreasing order of uncertainty and variability of future
claims experience:
 employers’ liability
 household contents
 motor liability.

Solution

employers’ liability, motor liability, household contents

Motor liability has a longer tail than household contents because of the occasional bodily injury
claim.

When considering its capital requirements a general insurer will need to consider each
class of business individually. However, its overall capital requirement will be more
important. An insurer that writes a variety of classes of business with a good spread of
risks is likely to be exposed to less overall uncertainty than one that writes limited classes
of business in a limited market.

You will probably already be familiar with the principle of diversification of investments. Holding
equities, fixed interest bonds and property is usually considered less risky than just holding
property. This principle can also be applied to the liabilities too. An insurer that writes lots of
different classes of business is exposed to less risk compared with an insurer that concentrates on
one class.

An insurer will often be allowed to take credit for this diversification when determining its capital
requirements. However, the extent of this diversification benefit will be sensitive to the
groupings used by the insurer in its capital model.

Diversification in the context of capital modelling is discussed in detail later in the course.

An example of how classes may be subdivided into lines of business for solvency purposes
arises from the EU’s approach within Solvency II:

Accident

This line of business includes obligations caused by accident or misadventure but excludes
obligations considered as workers’ compensation insurance.

Sickness

This line of business includes obligations caused by illness, but excludes obligations
considered as workers’ compensation insurance.

© IFE: 2019 Examinations The Actuarial Education Company


SP7-02: Insurance products – background Page 31

Workers’ compensation

This line of business includes obligations covered with workers’ compensation insurance
which insures accidents at work, industrial injury and occupational diseases.

Motor vehicle liability – Motor third party liability

This line of business includes obligations which cover all liabilities arising out of the use of
motor vehicles operating on the land including carrier’s liability.

Motor, other classes

This line of business includes obligations which cover all damage to or loss of land motor
vehicles, land vehicles other than motor vehicles and railway rolling stock.

Marine, aviation and transport

This line of business includes obligations which cover all damage or loss to river, canal,
lake and sea vessels, aircraft, and damage to or loss of goods in transit or baggage
irrespective of the form of transport. This line of business also includes all liabilities arising
out of use of aircraft, ships, vessels or boats on the sea, lakes, rivers or canals including
carrier’s liability irrespective of the form of transport.

Fire and other damage

This line of business includes obligations which cover all damage to or loss of property
other than motor, marine aviation and transport due to fire, explosion, natural forces
including storm, hail or frost, nuclear energy, land subsidence and any event such as theft.

General liability – third party liability

This line of business includes obligations which cover all liabilities other than those
included in motor vehicle liability and marine, aviation and transport.

Credit and suretyship

This line of business includes obligations which cover insolvency, export credit, instalment
credit, mortgages, agricultural credit and direct and indirect suretyship.

Legal expenses

This line of business includes obligations which cover legal expenses and cost of litigation.

Assistance

This line of business includes obligations which cover assistance for persons who get into
difficulties while travelling, while away from home or while away from their habitual
residence.

Miscellaneous non-life insurance

This line of business includes obligations which cover employment risk, insufficiency of
income, bad weather, loss of benefits, continuing general expenses, unforeseen trading
expenses, loss of market value, loss of rent or revenue, indirect trading losses other than
those mentioned before, other financial loss (not trading) as well as any other risk of
non-life insurance business not covered by the lines of business mentioned before.

The Actuarial Education Company © IFE: 2019 Examinations


Page 32 SP7-02: Insurance products – background

Note that the terminology and groupings given above are not always consistent with those used
later in the Course Notes. The types of cover provided by general insurers are vast and varied,
and no single set of definitions or groupings exist. It is much more important to understand the
cover provided, than to label it with any particular sub-heading.

Types of general insurance products are covered in detail in the next chapter.

© IFE: 2019 Examinations The Actuarial Education Company


SP7-02: Insurance products – background Page 33

6 Glossary items
Having studied this chapter you should now read the following Glossary items:

 Anti-selection  Nil claim


 Average  No-claim discount (NCD)
 Claim amount distribution  Peril
 Claim cost inflation  Personal lines
 Claim frequency  Rating factor
 Claim frequency distribution  Recoveries
 Claim size distribution  Retroactive date
 Claims made policy  Risk factor
 Claims reported  Risk premium
 Commercial lines  Salvage
 Discovery period  Subrogation
 Excess  Sunset clause
 Exclusion  Uberrima fides
 Expiry date  Underinsurance
 Exposure  Underwriter
 Exposure unit / measure  Underwriting
 Inception date  Underwriting factor
 Liability insurance  Zero claim.
 Moral hazard

The Actuarial Education Company © IFE: 2019 Examinations


Page 34 SP7-02: Insurance products – background

The chapter summary starts on the next page so that you can
keep all the chapter summaries together for revision purposes.

© IFE: 2019 Examinations The Actuarial Education Company


SP7-02: Insurance products – background Page 35

Chapter 2 Summary
General insurance principles
For a risk to be insurable the policyholder should have an interest in the risk, it should be
quantifiable and the claim amount payable should be commensurate with the size of the
financial loss.

Ideally, risk events should:


 be independent
 have low probability of occurring
 be pooled with similar risks
 have an ultimate liability
 avoid moral hazards.

In addition, there should be sufficient data to enable the insurer to estimate the size of the
risk and likelihood of occurrence.

Principles underpinning an insurance contract include assessing the insurable risk, the details
of the proposal form and contract itself, identifying an insurable interest and uberrima fides.

Exclusions are used to avoid payments by the insurer for a variety of reasons.

Types of product
The generic types of general insurance cover:
 liability to third parties
 property damage
 financial loss
 fixed benefits.

Cover provided
The benefits usually aim to indemnify the insured for any financial losses suffered as a result
of an insured event, although fixed benefits may sometimes be provided. A peril is a type of
event that may cause losses, eg theft, flood. Policies may be written on a losses-occurring or
a claims-made basis.

The exposure measure is the principal measure of risk for an individual policy. Premiums will
be set according to the measure of exposure.

The claims characteristics refer to the ways in which and speed with which claims originate,
are notified, are settled and paid and are, on occasion, reopened. Claim frequency and
amount are also relevant. Claims characteristics vary by class.

The Actuarial Education Company © IFE: 2019 Examinations


Page 36 SP7-02: Insurance products – background

Chapter 2 Summary continued


Risk factors are any factors that have a bearing on the amount of risk. Rating factors are
factors that are actually used in the premium rating process. Rating factors are either
measurable risk factors or proxies for the underlying risk factors (ie where it is not practical
to use the true risk factor).

Underwriting factors are rating factors, together with subjective factors that cannot be
measured, but will still be taken into account when setting premiums / policy conditions.

Selection against the insurer may occur where an insurer’s premium rating structure does
not reflect the underlying risks, especially if premiums differ from those offered by the rest
of the market.

How risk varies by class


Some classes of business cause insurers more risk and uncertainty than others because of
the nature of the risks involved and the claims that can arise from those risks. Factors that
affect the level of risk and uncertainty include:
 homogeneity of risks
 non-independence of risks
 changing risks
 numbers of claims
 claim cost
 claims inflation
 delay patterns
 variability of experience
 accumulations
 fraudulent claims.

Capital requirements and solvency


General insurers are required to hold capital over and above their technical reserves. This
capital is known as solvency capital.

Generally, the capital requirements will be larger for a class with a greater uncertainty and
variability in its future claims experience.

© IFE: 2019 Examinations The Actuarial Education Company


SP7-02: Insurance products – background Page 37

Chapter 2 Practice Questions


2.1 List the reasons for applying exclusions to an insurance policy.

2.2 Suggest claims characteristics that might make a claims-made basis appropriate.

2.3 Explain what the key problem is for an employer in switching insurance from an insurer providing
cover on a claims-made basis to an insurer providing cover on a losses-occurring basis.

2.4 Explain:
(a) the main criteria used in the choice of an exposure measure
(b) the difference between a risk factor and a rating factor
(c) why rating factors are needed
(d) the difference between a rating factor and an underwriting factor.

The Actuarial Education Company © IFE: 2019 Examinations


Page 38 SP7-02: Insurance products – background

The solutions start on the next page so that you can


separate the questions and solutions.

© IFE: 2019 Examinations The Actuarial Education Company


SP7-02: Insurance products – background Page 39

Chapter 2 Solutions
2.1 Exclusions might be used:
 to avoid payment by the insurer in situations where:
– the policyholder is at an advantage through possessing greater personal
information about the likelihood of a claim
– the claim event is largely under the control of the policyholder
– the claim event would be very difficult to verify
– the loss occurs as part of the normal course of events, and could be considered to
be depreciation
 where the risk cannot be reliably estimated by the insurer, regardless of whether or not
the policyholder has better information
 when the probability of loss is very high
 the risk is covered by a third party such as the government
 to limit the scope of the policy to make it more appropriate for a particular target market
 to reduce the premium for competitive reasons
 to reduce the risk of moral hazard and fraud.

2.2 A claims-made basis may be appropriate when it is not clear when the loss actually occurred. This
might be true for certain types of liability classes, where the loss emerges gradually over time,
eg deafness caused by continual exposure to loud noises at work under an employers’ liability
product.

2.3 There is the potential for a gap in the cover. For example, suppose, as an employer, we switch on
1/1/19 from a policy on a ‘claims-made’ basis to a policy on a ‘losses-occurring’ basis. Then on
2/1/19 we discover that our foreman, Bert, who suffered an injury at work last year which
appeared to have completely healed, has had a major relapse and is instituting proceedings for
compensation.

We cannot claim for this under our current policy, since the event occurred last year. We cannot
claim for it under our previous policy either, because the claim was not reported last year.

2.4 (a) An exposure measure should be:


 a good measure of the amount of risk (eg doubling the exposure measure should
double the amount of risk)
 measurable
 easily obtainable and objectively verifiable
 not capable of manipulation.

The Actuarial Education Company © IFE: 2019 Examinations


Page 40 SP7-02: Insurance products – background

(b) A risk factor is a factor that affects the level of risk for a particular policy.

A rating factor is a factor used in the rating process, either because it is a measurable risk
factor or because it is a proxy for a risk factor.

(c) Rating factors are needed because different policies have different levels of risk and
because the exposure measure is rarely good enough by itself to gauge the level of risk.

(d) An underwriting factor is one that is used to determine the premium, terms and
conditions for a policy. It may be a rating factor or some other risk factor that is
accounted for in a subjective manner by the underwriter. Remember that rating factors
must be measurable, verifiable etc.

© IFE: 2019 Examinations The Actuarial Education Company


SP7-03: Insurance products – types Page 1

Insurance products – types


Syllabus objectives
1.1 Describe the main types of general insurance product in terms of:
 the needs of customers
 the financial and other risks they pose for the general insurer including their
capital requirements and possible effect on solvency.

Covered in part in this chapter.

The Actuarial Education Company © IFE: 2019 Examinations


Page 2 SP7-03: Insurance products – types

0 Introduction
The range of general insurance products is very wide and continually changing and
therefore it is difficult to set out the features of all types of product.

The sections below provide a general indication and examples of the knowledge that
examiners would expect a candidate to have in relation to the features of the major types of
general insurance product. However, the examiners will also expect candidates to apply
this knowledge to any other products that may exist.

This chapter is not intended to be an exhaustive description of all the main products available.
However, it should give you:
 a sound introduction to the main product types
 a good understanding of the framework within which any other product could be
analysed.

In this chapter, the products are discussed under the headings of the four main types of
cover that they provide. The candidate will need to bear in mind the possible effects of
combining different types of cover within a single policy. For example, the risk factors
relevant to the liability cover will not necessarily be the same as those that are relevant to
the cover for damage to property.

This chapter concludes with a discussion of how products may be combined or ‘packaged’ up.

As well as being a sound introduction, this chapter will be very useful to you in your revision.

This chapter is very long. You may therefore wish to use two or three study sessions to work
through it.

The examiners do not only examine the ‘basic’ products such as motor and household.
Sometimes, more unusual products are examined. If you would like to know more about some of
the more unusual products, you may want to have a look at the following book:
Insurance Law Handbook (4th edition)
ISBN: 9781847660930
By Barlow Lyde and Gilbert LLP
To order, call Tottel Publishing on +44 (0)1444 416119

This textbook is part of the recommended reading for the Chartered Insurance Institute. It gives a
comprehensive factual introduction to the legal and coverage sides of many classes of insurance,
and so would be useful for students who wish to find out more about the more ‘unusual’ classes
of business. It also covers other issues such as insurance markets (eg Lloyd’s) and regulation.
However, the book is quite UK-specific and is not part of the Core Reading for this subject.

© IFE: 2019 Examinations The Actuarial Education Company


SP7-03: Insurance products – types Page 3

1 Liability
The essential characteristic of liability insurance (also called ‘casualty insurance’ in North
America) is providing indemnity where the insured, owing to some form of tort (private or
civil wrong, such as negligence), is legally liable to pay compensation to a third party.

The main types of liability insurance are:

 employers’ liability / workers’ compensation

 motor third party liability

 marine and aviation liability

 public liability – often linked to other types of insurance such as property and
marine

 product liability

 professional indemnity and errors and omissions (E&O) liability

 Directors’ and Officers’ (D&O) liability

 environmental liability and pollution liability.

(Motor third party liability is a particular type of public liability insurance but is treated
separately because of its importance in personal lines business.)

Other forms of liability cover are variations of one or more of these types.

Question

Suggest an example of the sort of negligence that you think may result in a claim under a
professional indemnity contract.

Solution

Incorrect actuarial advice.

Question

Suggest what Directors’ and Officers’ insurance might cover.

Solution

Directors and Officers cover is a form of liability insurance which will indemnify directors or senior
managers of a company against them, or their company, being sued for acts that they have
committed. For example, they may have ‘unfairly’ dismissed an employee.

The extent of any legal liability may depend on the prevailing legislation. For marine and
aviation liability, international law prevails, depending on which jurisdiction applies and
how the contract is worded. For classes such as motor and employers’ liability, national
laws are likely to apply.

The Actuarial Education Company © IFE: 2019 Examinations


Page 4 SP7-03: Insurance products – types

1.1 Benefits
The basic benefit provided by any liability insurance is an amount to indemnify the
policyholder fully against a financial loss. However, subject to any statutory requirements,
this benefit may be restricted by:

 a maximum indemnity per claim or per event (this may involve more than one claim),
or an aggregate maximum per year

 an excess.

Subject to the details of any reinstatement clause, payment of any benefits may result in a
cancellation of cover or the need for a further premium.

Question

Explain what the aims are of having an excess.

Solution

An excess:
 reduces the amount of each claim (by the excess)
 reduces the number of claims (all claims less than the excess are eliminated)
 in particular eliminates the small claims just above the excess, where the policyholder
may feel it’s not worth claiming and thus results in expense savings for the insurer
 arguably encourages policyholders to be more careful and so helps prevent claims
 may allow the company to reduce premiums and so make them appear more competitive.

Any legal expenses relating to such liability are usually also covered. (Note that an illegal
act of negligence will often invalidate the cover.)

For example, an insurer is unlikely to pay out compensation if the insured caused a crash whilst
drink driving or whilst driving a stolen car. However, the insurer would pay out if the crash was
due to a punctured tyre or just a complete accident.

Employers’ liability
This insurance indemnifies the insured against legal liability to compensate an employee or
his or her estate for bodily injury, disease or death suffered, owing to negligence of the
employer, in the course of employment. Loss of or damage to employees’ property is
usually also covered.

Employers are liable if they are negligent in providing their employees with safe working
conditions. Generally speaking the employer would be liable if failing to:
 provide a safe working place with proper equipment in which the employee can work
 properly maintain the working place, as well as tools and equipment
 create and enforce proper working procedures and methods.

© IFE: 2019 Examinations The Actuarial Education Company


SP7-03: Insurance products – types Page 5

The benefit can be in the form of regular payments to compensate for disabilities that
reduce the employee’s ability to work, lump sum payments to compensate for permanent
injuries to the employee and benefits under the legal framework.

Legal costs will also be covered. Other costs such as care costs can also be included.

Compulsory cover

In many countries cover is compulsory.

For example, the Employers’ Liability (Compulsory Insurance) Act 1969 requires that every
employer in the UK (except some bodies such as local authorities) must maintain, with an
authorised insurer, an employers’ liability insurance policy covering all its employees. The
policy must have a minimum level of indemnity of £5m per event but will usually provide a
higher level of cover. Most authorised insurers will provide cover of at least £10m.

Most countries have similar requirements to the UK, although the details vary from country
to country.

The most important distinction is between countries that have a system of employers’
liability (in which losses must arise from the employer’s negligence if they are to form the
basis of compensation, eg in the UK), and workers’ compensation (in which losses merely
have to be suffered in the course of employment, eg in the US).

In some countries, for example Australia, employers’ liability insurance is more frequently
referred to as workers’ compensation insurance.

Motor third party liability


This insurance indemnifies the owner of a motor vehicle against compensation payable to
third parties for personal injury or damage to their property.

The benefits will include compensation for loss of earnings, hospital costs and damage to
property costs, and can be paid in a lump sum or periodically to the injured party.

Compulsory cover

In most countries such cover is compulsory, although precise rules vary, for example in the
amount of cover required. The cover provided may or may not be limited to that required by
legislation.

For example, third-party liability cover is required for all motor vehicles in the UK, under the
Road Traffic Act 1972. The cover must be unlimited in relation to personal injury claims but
is required only up to £250,000 for damage to a third party’s property.

In fact, the Core Reading is many years out of date here. The minimum cover for third party
property damage is now £1.2m, although many UK insurers provide unlimited cover.

Since motor liability insurance is a legal requirement for vehicle owners in most countries,
policies that provide liability cover only are widely available.

In South Africa, motor liability insurance is not compulsory unless the car has been bought on
credit, in which case comprehensive insurance is compulsory.

The Actuarial Education Company © IFE: 2019 Examinations


Page 6 SP7-03: Insurance products – types

Marine and aviation liability


The insured is indemnified against the legal liability to compensate a third party for bodily
injury, death or damage to property arising out of operation of the vessel or aircraft. The
third parties include, but are not limited to, passengers.

A large proportion of marine liability insurance is provided not by commercial insurance


companies but through Protection and Indemnity Clubs. These are mutual associations of
ship owners that provide cover for their members on a pooled basis.

Protection and Indemnity Clubs are described later in the course.

Public liability
The insured is indemnified against legal liability for the death of or bodily injury to a third
party or for damage to property belonging to a third party, other than those liabilities
covered by other liability insurance.

For example, suppose you are a recently qualified actuary, married with young children, walking
past a construction site underneath the covered pedestrian pavement. Despite the overhead
protection, a brick falls through and knocks you on the head, causing instant death. Your
dependants could sue the building contractor for their loss of future support because of your
death as both spouse and parent. The contractor would then rely on public liability insurance to
meet the claim.

Compulsory cover

In some countries – and for certain individuals or institutions – public liability is compulsory.

For example, statute requires public liability policies to be held by horse-riding


establishments and nuclear installations in the UK. However, there is no general
requirement in the UK for individuals or businesses to hold public liability insurance.

There are two main types of cover, namely:

 the risk at insured’s own premises (eg commercial premises such as a warehouse)

 the risk when work is carried out by the insured away from their own premises
(eg on the site where builders are working, away from their official place of work).

Product liability
This insurance indemnifies the insured against legal liability for the death of or bodily injury
to a third party, or for damage to property belonging to a third party, that results from a
product fault.

The policy will usually also cover legal costs. Some policies will include the costs of
recalling faulty products that have not actually caused damage.

Professional indemnity
This insurance indemnifies the insured against legal liability for losses resulting from
negligence in the provision of a service, for example unsatisfactory medical treatment or
incorrect advice from an actuary or solicitor. The insured will be a professional person or a
professional firm.

© IFE: 2019 Examinations The Actuarial Education Company


SP7-03: Insurance products – types Page 7

You might often hear the term ‘E&O’ being used, which stands for ‘errors and omissions’ cover.
This is another term for ‘professional indemnity’, the two are used interchangeably.

Like companies selling products, enterprises offering a service may also be concerned to protect
themselves against possible liability claims.

In a company take-over, for example, considerable weight may be given to professional reports
from accountants, merchant bankers and even, where relevant, actuaries. Negligence by one of
the advisors could cause their client to suffer large losses.

There are several types of professional indemnity insurance. The fundamental cover is the same –
the difference is due to the parties that buy the cover. For example, the US is notorious for the
high cost of professional indemnity cover for medical doctors, because they can be sued for
millions of dollars by patients who are not satisfied with treatment. This has given rise to medical
malpractice insurance (which is often abbreviated to ‘med mal’).

Compulsory cover

Holding professional indemnity insurance is often a legal or regulatory condition of being


allowed to practise a profession or may be imposed as a condition by a professional body.

Directors’ and Officers’ liability


This is another important type of liability cover. E&O is primarily concerned with performance
failures and negligence with respect to products and services, whereas D&O is concerned with the
performance and duties of management.

D&O insurance indemnifies the insured against the legal liability to compensate third
parties owing to any wrongful act of the insured in his or her capacity as a director or officer
of a company. The insurance is personal to the director or officer, but is usually bought for
him or her by the company.

Deliberate fraud by directors and officers will typically be excluded.

Environmental liability
The insured is indemnified against the legal liability to compensate third parties as a result
of bodily injury, death and damage to property as a result of unintentional pollution for
which the insured is deemed responsible.

The costs of cleaning up the pollution and regulatory fines may also be covered. Gradual
and sudden environmental pollution will generally both be covered.

The Actuarial Education Company © IFE: 2019 Examinations


Page 8 SP7-03: Insurance products – types

1.2 Insured perils

Employers’ liability
The perils can be largely grouped into the following:

 accidents caused by the negligence of the employer or by other employees

 exposure to harmful substances (eg chemicals, coal dust, asbestos)

 exposure to harmful working conditions (eg loud noises or repetitive strain).

At any given time there is often a particular cause which gives rise to a large number of claims,
eg asbestos-related diseases, industrial deafness, repetitive strain injury (RSI) and stress have
each been common.

Motor third party liability


The perils for motor third party can be grouped into the following categories:
 loss or damage to the property of third parties caused by the insured vehicle
 bodily injury and death of third parties caused by the insured vehicle.

Marine and aviation liability


The perils can be grouped into the following:

 loss of, or damage to, passengers’ property, including luggage

 bodily injury and death of passengers either while on board the vessel or aircraft or
when boarding or leaving the aircraft

 bodily injury caused by the vessel or aircraft

 damage to property caused by the vessel or aircraft.

There are likely to be a number of exclusions on aircraft liability insurance, including perils
such as terrorism, war and illegal activities, although some of these may be covered under
extensions to cover and may be a legal requirement in some countries.

In December 2007, US law was changed to provide a transparent system of compensation for
insured losses resulting from acts of terrorism. Under the law, compensation would be shared
between the public and private sector.

The insurance market in the US offers terrorism insurance coverage to the majority of large
companies that request it. Depending on the required level of coverage and the location of the
company, the cover may be expensive.

In France, it is mandatory for insurers to include terrorism coverage when insuring commercial
risks.

© IFE: 2019 Examinations The Actuarial Education Company


SP7-03: Insurance products – types Page 9

Public liability
As this type of insurance forms part of many types of insurance policy, the insured perils
will relate to the type of policy. For example, compensation for a dog bite may be covered
by a household policy, while compensation for injury from a falling object may be covered
by a commercial policy held by a builder.

In general, the policy will not be restricted to named perils, although some perils may be
excluded.

Product liability
Here the perils depend greatly on the nature of the product, but include:

 faulty design

 faulty manufacture

 faulty packaging

 incorrect or misleading instructions.

Examples could include:


 An electrical appliance with faulty wiring which could injure people using it.
 A pharmaceutical company concerned about future unwanted side effects emerging in
those taking a new drug. Two real life examples are:
– Thalidomide – in the early 1960s this sedative, known for its apparent lack of
toxicity was found to relieve the symptoms of morning sickness, and so was given
to pregnant women. The drug was later found to have caused the arrested
development of arms and legs in thousands of babies.
– Opren (also known as Benoxaprophen) was prescribed in the early 1980s to
relieve the symptoms of rheumatoid arthritis. Dozens of elderly patients died as a
result of its unpleasant side effects.

Professional indemnity
The perils here depend on the profession of the insured. Examples include:

 wrong medical diagnosis

 error in medical operation

 error in an actuarial report.

Directors’ and Officers’ liability


The perils include the following:

 allowing a company to continue operating in circumstances when it should have


been declared insolvent

 any act resulting in the insured being declared unfit for his or her role

 allowing false financial statements to be published.

The Actuarial Education Company © IFE: 2019 Examinations


Page 10 SP7-03: Insurance products – types

Environmental liability
The insured perils can be regarded as any incident causing gradual or sudden
environmental pollution.

1.3 Basis for cover

Employers’ liability
Most classes of business are written on the basis of when the loss was incurred, ie cover relates to
the date of the accident rather than the date of reporting the accident. So if a policy provided
cover for 2018, say, the insurer would be liable for claims resulting from accidents in 2018.

With employers’ liability, this gives rise to a few problems:


 Where an industrial disease results from prolonged exposure, it is hard to define the
‘accident date’. In practice, where the company has changed insurer over the period, the
claim is split between the insurers in proportion to the time span each provided cover
over the relevant period of exposure.
 Because many diseases take a very long time to develop fully, there have been extensive
reporting delays with employers’ liability insurance. This makes it more difficult for the
insurer to manage the business.

To overcome these difficulties, several insurers devised policies on a claims-made basis whereby
the insurer was liable for claims reported in the period of insurance cover. This approach to
insurance has led to a number of difficulties (eg overlaps in insurance cover), so many policies are
still (currently) operating on the original losses-occurring basis.

In the UK, employers’ liability insurance cannot currently be written on a claims-made basis.

Motor third party liability


Cover is usually provided on a losses-occurring basis.

Marine and aviation liability


Policies are usually written on a losses-occurring basis.

Public liability
This is likely to depend on the exact cover being provided, although is likely to be on a
losses-occurring basis.

Public liability insurance may be combined with other insurances, for example residential
buildings and contents products may include public liability cover. In such cases, the basis for
cover would be the same as for the other insurance cover provided.

Product liability
Policies are likely to be written on a claims-made basis.

© IFE: 2019 Examinations The Actuarial Education Company


SP7-03: Insurance products – types Page 11

Question

Explain why a claims-made basis might be an appropriate way of writing product liability cover.

Solution

Customers may not always remember exactly when they noticed a product fault, or when they
bought the product, and health problems caused by product faults (eg pharmaceuticals) may take
a long time to arise. In these cases it is likely to be easier for this business to be written on a
claims-made basis.

If a claims-made basis is used for certain products, such as motor insurance, the insured might
strategically choose when to report a claim, for example to retain the next year’s no-claims
discount. However, for product liability, it is the customer who will initiate the claim, but it is
actually the company that is the insured. The customer therefore has no personal interest in the
exact time at which the claim is reported.

Alternatively, an insurer could give cover relating to sales in a given year.

Professional indemnity
Professional indemnity is usually written on a claims-made basis.

Directors’ and Officers’ liability


D&O liability is usually written on a claims-made basis.

Environmental liability
Policies are likely to be written on a claims-made basis.

1.4 Measures of exposure to which premiums are related


Most policies that cover the various types of commercial liability will be underwritten on the
basis of the perceived risk with the premium adjusted in the light of the insured’s turnover.

Employers’ liability
Person-hours worked could be the best exposure measure when assessing claim frequency.
However, claim settlements are often related to loss of earnings so, in practice:

The main measure of exposure for employers’ liability is payroll, or total wage and salary
costs.

Payroll is used as the exposure measure for employers’ liability because the compensation
payable to employees who claim is often related to salary. For example, if an employee is unable
to work again due to injury, the main component of compensation would be loss of earnings.

The Actuarial Education Company © IFE: 2019 Examinations


Page 12 SP7-03: Insurance products – types

The premium rate for an employer will be agreed when a policy is taken out and a premium
will be paid on the basis of the estimated payroll for the year. At the end of the year an
adjustment premium will be paid or refunded on the basis of the actual total payroll for the
year.

This is because the annual payroll cannot usually be determined accurately in advance.

Motor third party liability


For motor third party liability the measure of exposure is the vehicle-year, ie an agreed
monetary premium is charged for the insurance of a single car for a year.

Marine and aviation liability


For marine and aviation liability insurance alternative measures of exposure can include:

 passenger kilometres

 passenger voyages

 in-service seats

 in-service vessels / aircraft.

However, turnover is still the most commonly used measure, although it is not an accurate
measure of the risk.

For example, a manufacturer could have zero turnover in a policy year and yet still have
significant exposure as coverage is based on accident year (ie on a losses-occurring basis),
irrespective of when the craft was made or sold. Therefore, previous years’ turnover can
give rise to claims on the current year’s policy.

Additionally for marine insurance, sum insured, number of feet drilled and turnover are
used where suitable. (Feet drilled might be an appropriate exposure measure for an offshore oil
rig.)

Some underwriters see past production as a key exposure measure. A better measure
might be rolling average turnover. For example, if a craft or vessel spends an average of 20
years in service, the average turnover in the last 20 years might be used as an exposure
measure.

Or the historical annual turnover could be weighted by a risk profile. This could be derived
from the age of craft or vessels in accidents.

The main problem with these exposure measures is their lack of uniformity. For example, a
jet engine as one of four on a wide-bodied airliner represents a very different risk to a
turboprop engine on a cargo aircraft. All of the exposure measures used need large
adjustments derived from rating factors before they can be of any use for rating.

Public liability
The most commonly used measure of exposure is turnover.

An alternative is payroll. The argument for this is that if you are a furniture maker paying
£200,000 pa in wages you would present a far greater public liability risk than if you paid wages of
only £50,000 pa because you will probably have a larger workforce.

© IFE: 2019 Examinations The Actuarial Education Company


SP7-03: Insurance products – types Page 13

Turnover is sometimes considered a better measure since the relative cost of materials and
equipment used is also indicative of the public liability risk.

For specific types of policies, such as for hospitals’ public liability, the number of beds may be a
more appropriate measure.

Product liability
The most commonly used measure of exposure is turnover.

Despite the problems with heterogeneity, annual turnover of the company is normally used
because no better alternative exists.

Professional indemnity
The most commonly used measure of exposure is turnover. However, alternatives can be
used in some situations such as amount of funds under management.

This would be appropriate if the insured is a fund manager.

Directors’ and Officers’ liability


As a type of professional indemnity insurance, the exposure measures will be similar, eg turnover.
Another measure that may be used is net assets and liabilities.

Environmental liability
The measure of exposure will depend heavily on the nature of the industry carried out by
the insured. For example, the number of power stations or the amount of energy generated
could be used for a power company.

Alternatively, a measure related to the value of the land may be used.

1.5 Claim characteristics


Three key questions must be answered before a liability can be settled:
1. Has there been a loss?
2. Is the insured liable?
3. What is the quantum (amount) of the loss?

The loss may also be reduced if there is contributory negligence on the part of the victim.

Reporting delays
Reporting delays might arise because, say, an injury gets worse over time whereas the sufferer
had expected the problem to be only temporary, or because the seriousness of the condition only
comes to light in a later routine check-up.

The Actuarial Education Company © IFE: 2019 Examinations


Page 14 SP7-03: Insurance products – types

Some liability claims remain undetected for many years and are often referred to as ‘latent
claims’. This means that a loss may not become evident until many years after it was
caused. An example of this is mesothelioma – a disease that arises from exposure to
asbestos – where the first symptoms are usually not evident for at least twenty years after
exposure and sometimes not for fifty years or more. This exacerbates the long-tailed nature
of the associated liability insurance if it was written on a losses-occurring basis.

Question

Explain whether or not a claims-made basis would exacerbate the long-tailed nature of the
business.

Solution

No – if the business was written on a claims-made basis, then the reporting delay would be
irrelevant, as the claim is effectively deemed to occur when the claim is made.

A further complication is that claims may arise over a long period; for example a storage
tank may leak contents into the ground, causing pollution, for some years before the
damage is discovered, and typically, those with asbestos-related diseases were exposed to
asbestos for many years before their condition became manifest. This means that it may
not be clear which insurance should respond to the loss, and often some form of allocation
is necessary.

Settlement delays
Settlement delays may be due to the delays caused by establishing liability, or the time to
establish the extent of injuries and to assess the speed of recovery.

If the loss is personal injury, it may take some years before the victim’s condition has
stabilised and can be assessed for damages. Claim settlement will involve litigation for any
claim of significant size, even though most cases are resolved out of court before they go to
trial.

Settlement for motor liability is often faster than for other types of liability claims, although
claims arising from serious bodily injury do typically take several years to settle. Even so,
motor liability claims, and particularly bodily injury claims, are significantly longer tailed
than other motor claims. In some jurisdictions, bodily injury claims can be settled as
regular payments for life to the victim, rather than lump sum amounts.

These are often called Periodical Payment Orders (PPOs), or structured settlements.

The legal environment has a significant effect on the settlement of claims, and this varies
from country to country. Therefore, claims development patterns for similar classes of
business may vary significantly between countries.

These characteristics (both reporting and settlement delays) mean that most liability classes
of business are long-tailed, whether the cover is written on a claims-made or a
claims-occurring basis.

© IFE: 2019 Examinations The Actuarial Education Company


SP7-03: Insurance products – types Page 15

Claim frequency
Claim frequencies on liability classes of business tend to be low.

Motor liability claims tend to be more frequent and usually smaller than claims on other
liability policies, both for property damage and for bodily injury to third parties.

Amount of the claim


Claim cost distributions tend to be more widely spread than for property classes, and there
can be some extremely large individual claims that take many years to settle.

Accumulations of risk arise in many liability classes as a result of an unbalanced portfolio. For
example, almost by definition, employers’ liability insurance produces a concentration of risk.
Because one policy will provide cover for an employer, there will be groups of employees
together in the same office or factory so a single event (such as the Piper Alpha oil rig explosion)
can lead to many claims.

Accumulations can arise in other ways too:


 If there is a court award in favour of a claimant, it may trigger a large number of similar
claims from individuals with the same complaint against their employer.
 An insurer who provides cover for several employers in the same industry is exposed to
the possibility that a large number of claims will emerge from a common cause. Previous
examples of this are asbestos-related diseases, industrial deafness and repetitive strain
injury.

1.6 Risk factors and rating factors

Employers’ liability and public liability


For employers’ liability and public liability, the main factor influencing the risk is the type of
industry or the occupation(s) of the insured.

Another important factor is the extent to which the employer implements safety measures.

The underwriting factors are as follows:

 type of industry or occupation

 exposure and claims experience

 location of the workforce

 frequency of visitors to the site

 the materials handled

 the processes involved

 safety precautions in place, for example sprinklers in a factory or office

 turnover

 size of deductible

The Actuarial Education Company © IFE: 2019 Examinations


Page 16 SP7-03: Insurance products – types

A deductible is a portion of a loss that is paid by the policyholder. It may be an amount or


a percentage. For example, a 5% deductible will restrict the size of any claim payment to
95% of the loss.
 payroll
 level of staff training (linked to quality of management)
 provision of first aid facilities.

Question

Demonstrate by means of a simple numerical example, the difference between the operation of
an excess and a deductible on an insurance policy.

Solution

Suppose we have two policies. Policy A has an excess of $50 and policy B has a deductible of $50.
The policies are identical in all other ways. The policies have a sum insured of $1,000.

For any claim less than $1,000 then both policies pay the same amount to the insured, ie the
amount of the claim less $50.

However, the amount paid by the insurer on the two policies is different if the claim exceeds the
sum insured. Suppose the claim is $1,500.

Policy A pays $1,000. Policy B pays $950 (ie the amount of the claim paid by the insurer is
reduced by the deductible).

These are factors that underwriters will take into account, some quantitatively, some
qualitatively in assessing an appropriate premium. In addition, underwriters should vary
the premium according to a general assessment of the risk management standards of the
insured.

For very large companies, the previous claims experience will also be relevant.

Motor third party liability


Since motor third party cover is usually provided in the same policy as cover for damage to
the insured vehicle, the risk factors and rating factors are discussed in the section below on
property damage.

In some European countries motor liability insurance and motor own-damage insurance are
sold as separate policies, but the rating factors are likely to be similar.

This is the case in Hungary, for example.

Marine and aviation liability


A wide range of different rating factors are used by insurers. Some form the basis of
actuarial rating models whereas other, more subjective, rating factors are used by
underwriters as part of the rating process.

© IFE: 2019 Examinations The Actuarial Education Company


SP7-03: Insurance products – types Page 17

Some of these are:

 loss history

 type of craft or vessel (as accident rates and types / sizes of claims will vary), for
example, jets, turboprop and rotor wing in the case of aircraft

 commercial category (commercial, private, military)

 satellites and missiles also attract separate rating structures

 use of craft or vessel (passenger / cargo / leisure / business)

 geographic region (jurisdiction of litigation).

Product liability
The nature of the products produced by the insured will be an important factor in assessing
the level of premium. Other risk factors include:

 the distribution channel of the product

 how much US-exposure the product has – in general, products sold in the US show
a different (higher) claims distribution (since there tends to be a different attitude to
claiming), and more claims are likely to be made
 its usage

 the general trade of policyholder


 any potentially dangerous components within the product, and how quickly they
deteriorate.

However, each policy will be underwritten individually, and the underwriter will need to
assess the risks arising from a policy subjectively.

Professional indemnity
The nature of the profession is an important factor in assessing the level of premium.
However, each policy will be underwritten individually, and the underwriter will need to
assess the risks arising from a policy subjectively.

The risks may also vary between size of professional firm, with sole practitioners presenting
a very different risk profile from large professional services firms.

Directors’ and Officers’ liability


The nature of the company will be an important factor in assessing the level of premium.
However, to some extent each policy will be underwritten individually, and the underwriter
will need to assess the risks arising from a policy subjectively.

Other than the type of business, a typical risk factor would be the past experience of the
company.

Environmental liability
Each policy will be underwritten individually, and the underwriter will need to assess the
risks arising from a policy subjectively.

The Actuarial Education Company © IFE: 2019 Examinations


Page 18 SP7-03: Insurance products – types

This will take into account:

 the processes carried out by the insured

 the likely effects of any accident

 the likely cost of clean-up

 a general assessment of the risk management practices of the insured.

Question

You propose to market a new type of policy to small shopkeepers selling general groceries and
other goods in suburban neighbourhoods. You want to develop a package that will cover all of
their insurance needs. Suggest what types of peril your package would cover.

Solution

Assuming that the shopkeeper does not own the shop itself, the shopkeeper is likely to require
some or all of the following:
 protection for the stock and fixtures and fittings against theft, damage from fire, flood,
lightning, hold up or malicious persons
 consequential loss of profits insurance in case the shop suffers a major fire
 plate glass insurance for the shop window
 employers’ liability insurance if anyone else is employed in the business
 protection for frozen food in freezers against spoilage as a result of freezer breakdown
 product and/or public liability insurance to protect against claims from the public in
respect of faulty goods sold
 insurance against theft of cash
 protection for goods in transit.

In practice, most insurance companies would be able to offer the small trader a ‘package policy’
incorporating all of these benefits, together with a number of other ‘optional extras’.

© IFE: 2019 Examinations The Actuarial Education Company


SP7-03: Insurance products – types Page 19

2 Property damage
The main characteristic of property damage insurance is indemnifying the policyholder.
However, here the indemnity is against loss of or damage to policyholders’ own material
property.

The main types of property that are subject to such damage are:

 residential buildings (for example, houses)

 commercial and industrial buildings (for example, offices, shops and factories)

 moveable property (for example, the contents of a home or of commercial premises)

 land vehicles (for example, cars, buses, taxis)

 marine craft

 aircraft

 goods in transit

 property under construction


 engineering plant and machinery

 goods insured under an extended warranty policy


 crops.

Land vehicles can be further divided into:


 private motor
 commercial vehicle
 motorcycle
 motor fleet.

Policies are commonly available under these sub-headings.

A motor fleet policy provides cover for a number of different vehicles belonging to an individual
owner, eg a company. A small fleet might have five vehicles in it, a large fleet could have 500 or
more.

A motor fleet may extend beyond cars to include other types of commercial vehicle (eg lorries and
vans). In these cases, the fleets may be very heterogeneous.

Marine, aviation and goods in transit are often labelled as ‘MAT’: Marine, Aviation and
Transport.

The Actuarial Education Company © IFE: 2019 Examinations


Page 20 SP7-03: Insurance products – types

2.1 Benefits
The benefit is usually the amount needed to indemnify the insured against the value of the
loss or damage, subject to any limits or excesses.

Household buildings property


For buildings insurance, the benefit, by the principle of indemnity, will be the amount required to
fully reinstate the property, rather than the market value of the property.

The amount paid under the policy is usually the amount needed to restore it to its previous
condition, subject to any excess or deductible. Ancillary costs, such as the provision of
alternative accommodation while the insured property is uninhabitable, may also be
covered, but are not standard.

The policy will define precisely what is included within the definition of buildings. For example,
the fixtures and fittings, patio, drive, swimming pool, walls, garages, garden sheds, fences and
gates may all be included with the building covered by a household policy.

Household policies normally include insurance for liabilities arising from the insured
property.

For example, household insurance may also contain public liability cover in respect of the house.

Commercial buildings property


The cover is likely to be for the full value of the property, however, alternatively, it may be on a
first loss basis.

First loss is a form of insurance cover for which the chosen sum insured is restricted, with the
insurer’s agreement, to a figure less than the full reinstatement-as-new value of the property.
The insured therefore has to bear any loss in excess of the sum insured.

This may be appropriate in circumstances where:


 the insured considers that a loss in excess of the sum insured is extremely unlikely
 the item is effectively priceless, eg a stately home, for which there may be no possibility
of the building being reinstated
 the insurance is against water damage (where only the ground floor areas may be
affected).

Moveable property
For buildings, the sum insured is based on the total cost of rebuilding the property. For contents,
it is based on the value of the contents.

The policy will define precisely what moveable property is covered by the insurance. For
example, under a household contents policy, the insured’s household goods and personal
possessions plus visitors’ personal effects may be covered.

© IFE: 2019 Examinations The Actuarial Education Company


SP7-03: Insurance products – types Page 21

Household policies may have a number of extensions of cover, which may be optional or
may be used by companies to distinguish their own products; examples of these would
include food in a freezer spoiled during a power cut, pet insurance (which is also widely
available as a separate product), bicycles and the contents of sheds.

The amount paid on a claim can be:

 the replacement value, which is the cost of a new item reduced to allow for the
depreciation on the lost item, or

 on a ‘new for old’ basis, under which the cost of an equivalent new item is provided.

The insurer may retain the right to provide a replacement item rather than provide monetary
compensation.

New for old cover would not be normal in commercial policies but is common in household
policies.

For commercial contents the cover is usually for pure indemnity, ie the full depreciated value of
the contents is covered. As with household insurance, the insurer may apply the principle of
average to scale down claim amounts in cases of underinsurance.

The effect of inflation

Insurers believe that, during times of high inflation, many policyholders become underinsured
because they fail to increase the total sum insured sufficiently from year to year. To help prevent
this, many insurers have introduced policies with automatic indexation of the sum insured. Each
year the level of cover is automatically increased in line with an appropriate inflation index. The
inflation index should reflect the increased cost in claims (eg an index of goods for contents and
an index of building cost inflation for buildings insurance). The indexation facility is detailed in the
escalation clause.

Motor property
The three main levels, or types, of cover usually available are:
(a) third party
(b) third party, fire and theft
(c) comprehensive.

However, in some countries there may be slightly differing levels of cover to this.

The most basic level is third party insurance described in Section 1. This basic level of cover is not
that commonly used.

The next level of cover is third party, fire and theft (TPF&T). This provides the same cover as third
party insurance plus:
 losses or damage caused to the insured’s own vehicle from fire and/or theft.

The highest level of cover is comprehensive. This gives the cover provided by third party, fire and
theft, and additionally:
 accidental or malicious damage to the insured’s own car.

The Actuarial Education Company © IFE: 2019 Examinations


Page 22 SP7-03: Insurance products – types

The maximum benefit is the depreciated value of the vehicle. In most cases of damage, the
insurer will pay to have the vehicle repaired.

The vehicle is insured for its replacement value. However, if repair costs exceed the market value
of the vehicle at the time of an incident, it will be declared a complete ‘write-off’ and only the
second-hand market value at the time of the accident will be paid to the insured.

Comprehensive policies will also usually include cover for:


 specific injuries to the insured and/or spouse usually a defined payment (eg £2,500)
 personal medical expenses (up to a defined amount, eg £150 per person)
 the insured’s legal expenses in relation to any claim
 loss or damage to personal effects that were in the car (up to a defined amount, eg £100)
 replacement of locks after theft of the car
 replacement / repair of broken windscreens.

Marine and aviation property


There is a wide variety of types of cover available for marine insurance risks. However, the
principal forms of insurance likely to be needed by a ship-owner are as follows:
 loss of or damage to the craft (often called ‘hull insurance’)
 loss of or damage to cargo (‘cargo insurance’ is for the actual contents, ‘freight’ covers the
money payable for shipment of the cargo).

In addition, other types of insurance cover could be included, such as:


 compensation for loss of use of the craft
 insurance against damage to a vessel still under construction.

In marine property insurance, definitions are used to define the extent of the loss. In particular,
the Marine Insurance Act defines actual total loss and constructive total loss.

Actual total loss is deemed to occur in one of three ways:


(1) The insured item is totally destroyed.
(2) The item is so damaged that it can no longer be classed as the type of object
originally insured.
(3) The insured is irretrievably deprived of the insured item.

Constructive total loss is where the insured abandons the insured item because an ‘actual
total loss’ is unavoidable or because the costs of preventing a total loss exceed the value
saved.

© IFE: 2019 Examinations The Actuarial Education Company


SP7-03: Insurance products – types Page 23

Goods in transit
This is a commercial insurance cover against loss of or damage to goods whilst being transported
in vehicles specified in the policy (eg the company’s vehicles or perhaps also by a carrier). The
periods of loading and unloading are usually covered in addition to the journey. Losses due to
business interruption are not usually included and a special extension is usually required if
livestock are to be covered. The sum insured is likely to be the value of the goods.

Construction and engineering


Large construction and engineering projects can take up to five years (or longer), and the
associated policies will last until the end of the project. Since projects often overrun their
schedules the policies are usually extendible. A policy for a large project may continue for
some years after its completion to cover the late discovery of construction faults.

Since claims can arise over a number of years, and the value of the property under construction is
likely to vary significantly over the period, this is likely to have a material impact on claims
experience. CAR covers are adjustable for changes in the value of property and material, in
order to ensure that the cover continues to meet the insured’s needs.

Extended warranty
Extended warranty insurance covers losses arising from the need to replace or repair faulty
parts in a product (usually electrical goods, furniture or motor vehicles) beyond the
manufacturer’s normal warranty period. Policies may have a term of several years.

Crop insurance
This will indemnify the insured (eg a farmer or orchard owner) against losses to the crop due to
specified perils.

2.2 Insured perils

Household buildings property


In respect of buildings, fire is the principal peril insured against but policies can cover many
other perils such as explosion, lightning, theft, storm and flood.

Question

Explain why theft is listed as an insured peril on a buildings policy despite it being very unlikely
that someone will steal your house.

Solution

It refers to the damage done to buildings in the course of forced entry by thieves, eg windows
broken.

Subsidence is a peril that is commonly covered under household insurance.

The Actuarial Education Company © IFE: 2019 Examinations


Page 24 SP7-03: Insurance products – types

Subsidence has become a fairly big problem in the UK.

Damage to the insured property caused by measures taken to put out a fire is also covered.

More examples of perils covered are:


 earthquake (not so important in the UK, but vital in many countries)
 burst pipes (eg damage caused by water or oil leaking from pipes)
 vandalism
 impact (eg from falling branches, aircraft, cars, stampeding animals).

Commercial buildings property


Commercial buildings will be subject to the same sorts of perils as household buildings, eg fire,
explosion, lightning, theft, storm and flood.

Commercial property insurance is often referred to as ‘fire insurance’, but conventionally


covers other perils as well, such as weather-related losses and malicious damage. A
reference to ‘fire’ as an insurance class should not be taken to mean that the policy is
limited to fire risks unless the context clearly indicates that it is, but rather it should be
assumed to include other property damage perils.

‘All-risks’ policies are also common, especially for household policies; these cover all
causes of damage that are not explicitly excluded.

Moveable property
As with buildings insurance, the policy will set out the perils covered.

The most important perils covered under property insurance are fire and theft. Malicious
damage and damage arising from weather events are usually covered. Accidental damage
is often provided under household policies, but often as an optional extra.

Motor property
The perils include accidental or malicious damage to the insured vehicle, and fire or theft of
that vehicle.

Marine and aviation property


The following perils relate specifically to marine hull cover, but similar perils are covered for
marine cargo and aviation insurance:

 perils of the seas (or other navigable waters)

 fire

 explosion

 jettison

 piracy

© IFE: 2019 Examinations The Actuarial Education Company


SP7-03: Insurance products – types Page 25

 sinking

 damage etc.

Other sections included under marine property include:

 energy (offshore, onshore and construction)

 liability (third party property)

 specie (more valuable items in transit)

 war.

It should be noted that the traditional definition of marine insurance is anything that a
seagull can fly over.

Goods in transit
Damage, loss and theft are the main perils for goods in transit.

Construction
The main insured perils are damage, destruction, design defect, faulty parts and failure to
finish the construction project.

Engineering
Machinery breakdown, explosion and electronic failure are the main insured perils.

Extended warranty
The main peril is faulty manufacture.

Crop insurance
The main perils include disease, fire and weather-related perils (such as storms or drought).

2.3 Basis for cover


Property damage insurance is likely to be written on a losses-occurring basis.

2.4 Measure of exposure to which premiums are related

Household property (buildings and contents)


For household property the exposure is frequently measured in sum insured years: the sum
for which a property is insured multiplied by the period at risk.

The sum insured should be the cost of rebuilding the property should it be destroyed,
including the clearing away of debris. This is usually less than the market value of the
property, since the value of the land itself is not normally impaired by the destruction of the
building and the loss of the land, for example through coastal erosion, is not normally an
insured risk.

The Actuarial Education Company © IFE: 2019 Examinations


Page 26 SP7-03: Insurance products – types

It is generally accepted that the bigger the property, the greater the total amount of risk.
Exposure measures are therefore linked to the size of the property. The most commonly used
exposure measure, sum insured per year, can be used for both buildings and contents.

Other measures of the size of the risk have been used. For example, some insurers have used
number of bedrooms for both buildings and contents. The logic behind this is that both the
rebuilding costs and the value of contents are a function of the number of bedrooms.

Question

List the perils that might be covered by a home contents insurance policy.

Solution

 fire, lightning, explosion, earthquake


 storm, flood
 burst pipes
 theft, vandalism, civil commotion
 subsidence or land heave
 impact (eg from falling trees, aircraft)
 damage caused by measures to put out a fire
 accidental damage

Commercial property (buildings and contents)


For commercial property as with household property, the measure of exposure is usually
the sum insured year.

The sum insured should cover the full value of the property, unless first loss cover is
intended.

The sum insured for buildings should reflect the cost of rebuilding, not the market value.

However, the sum insured year is complicated for two reasons:


1. The amounts of stocks held may vary considerably over the period of the insurance.
Hence the stock may be covered on a declaration basis, determined retrospectively
with an adjustment premium.
2. There is no standard way of allowing for inflation in the policy. Hence policies with
different types of inflation treatment need to be considered separately to determine
the exposure.

© IFE: 2019 Examinations The Actuarial Education Company


SP7-03: Insurance products – types Page 27

However, the premium for large commercial property policies will often be determined by
reference to the maximum amount that the underwriter believes could feasibly be lost in a
single incident, either because complete destruction is an event so remote that it need not
be provided against or else because the policy covers a number of properties, sufficiently
far apart that the possibility of damage to more than one in a single incident is remote. This
amount is known variously as the estimated maximum loss (EML) or probable maximum
loss (PML).

Virtually every commercial property is unique, at least in respect of location, use, size and
construction. It is therefore almost impossible to group them in a homogeneous manner.

Motor property
For motor insurance, vehicle-miles would probably be the best measure of exposure for
damage claims, as the chance of an accident depends on the extent to which the vehicle will
be used. However, it will be difficult to verify miles travelled as stated by the proposer.
Therefore, the measure usually used is the vehicle-year, which has the advantage of ease of
calculation.

A motor policy that insures one vehicle for one year is exposed for one vehicle-year.

For a motor fleet, the amount of exposure will probably not be available in advance, since the
number of vehicles in the fleet may change during the year. At the end of the year an adjustment
will be made to the premium for the year just gone to reflect the number of vehicles actually
insured.

For example, several motor insurers in the UK have launched a ‘Pay as you drive’ motor insurance
product under which policyholders have a ‘black box’ installed in their vehicle, which logs the
details of their journeys – in particular, the distance driven and the time of day.

Policyholders are charged a fixed basic monthly premium, based on the standard motor insurance
rating factors (such as age and vehicle make and model) plus a variable monthly premium based
on information provided by the black box. Lower premiums are charged to those with low
mileage clocked up during daylight hours.

Using actual mileage as a measure of exposure (for the variable part of the premium) has led to
significant premium reductions for many customers.

Marine and aviation property


For marine property, the insured value of the hull or the value of the cargo might be used as
exposure measures.

Similarly, for aviation, the insured value of the aircraft or the value of the goods carried
might be used, as well as the number of take-offs and landings because this is when most
losses occur.

Goods in transit
The consignment value is most commonly used. Specific types of cover can exist, for
example, relating to specie (high-value items), fine art, jewellery and so on.

The Actuarial Education Company © IFE: 2019 Examinations


Page 28 SP7-03: Insurance products – types

Construction
The value of the contract may be used as an exposure measure.

Engineering
The sum insured value or value of the contract may be used as an exposure measure.

For both engineering and construction, the risk is not uniform along the timeline of the
project. For example, if the site is destroyed by a storm at the start of the project, little may
be lost. Whereas, just before completion, the value of the loss will be much higher (tending
towards the rebuild value). During the rating process, the risk profile will be taken into
account and a percentage rate will be loaded for this, before it is applied to the exposure
measure.

Extended warranty
The number of appliances, or appliance-years (number of appliances times number of
years), could be used as an exposure measure.

Crop insurance
Possible exposure measures include the value of crops, processed food or the sum insured.

2.5 Claim characteristics


Property claims are generally reported and paid quickly compared to liability claims.
Losses arise from incidents that are observed at the time and the value of the loss is usually
straightforward to establish. There are exceptions, and the reinstatement of a major
industrial plant after a major incident can be a long project, but generally the property
damage classes are among the shorter-tailed classes of insurance. Comments are included
below where there are specific features to note.

Household and commercial property


The event giving rise to a claim for damage to buildings or contents usually occurs
suddenly (as for fire or burglary) and the cause is easily determinable.

Notification is then also made promptly, and a reasonably good estimate of the claim
amount can be made. (Subsidence claims are an exception to this.)

Settlement is in many cases by a single payment but larger claims can take longer and may
be settled with intermediate payments as the building project to repair or replace the
building proceeds. Delays may be greater, however, where it is necessary to verify the
value of stock held in a commercial property.

Domestic property claims will tend to be fairly consistent in size and distribution, with a
small number of larger, total loss claims and occasional liability losses. Commercial
property claims have similar features, although they tend to have a more scattered cost
distribution, owing to the singular nature of the properties insured.

These classes are the ones that are most exposed to moral hazard.

© IFE: 2019 Examinations The Actuarial Education Company


SP7-03: Insurance products – types Page 29

Question

Explain the difference between ‘indemnity’ and ‘replacement’ cover. Justify which one would
expose the insurer to the greater risk of fraudulent claims.

Solution

Indemnity cover the insurer compensates the insured for the value of
items as at the date of loss.

Replacement cover the insurer pays for new replacement items.

The risk of fraudulent claims will be greater with replacement cover. Having shelled out their
(admittedly higher) premium, policyholders with replacement cover have more to gain from such
claims, eg it may save them going shopping to buy a new TV to replace their 20 year old set.

There is also the risk of exposure to catastrophes, for example after severe storms or
floods.

Question

List three other examples of how a household insurance portfolio could be exposed to
accumulations.

Solution

Subsidence, fire, explosions.

Motor property

Claims for damage to the insured’s vehicle are usually reported and settled quickly.

Claims for damage to the property of third parties may take slightly longer to settle than claims
for damage to the property of the insured.

There may be some delay in settling property claims while the liability for settlement is
established. Generally, delays for settlement of property claims should be in weeks or months
rather than years.

As you might know from bitter personal experience, motor claims have a relatively high
frequency. The average claim frequency for different insurers will depend very much on the type
of business they write. As a rough indicator, many insurers will have claim frequencies of about
25% on a comprehensive portfolio, ie 1 claim for each 4 vehicles each year. Claim frequencies for
a non-comprehensive portfolio may typically be about 15%.

The Actuarial Education Company © IFE: 2019 Examinations


Page 30 SP7-03: Insurance products – types

In general, private motor insurance does not give rise to extreme accumulations. Because
business is sold nationally, an insurer is unlikely to have a concentration of risks in one
geographical area.

Question

Suggest differences between the claims characteristics of a motor fleet portfolio compared with a
private motor portfolio.

Solution

The motor fleet claim frequency will be higher. Claim frequencies of 40% or more (per
vehicle-year) are not unusual on fleet policies. Fleet cars tend to be driven long distances by a
variety of drivers, some of whom may not be totally familiar with the vehicles they are driving.
Some people with company cars might also think ‘it’s only a company car so …’.

Many motor fleets will be subject to greater accumulations of risk than exists for private motor
insurance, eg vehicles may regularly be parked in the same place.

Marine and aviation property


Among the claim characteristics for this business, the following are important:

 Claims are usually reported as soon as the vessel reaches a major port (or indeed
the event might take place in port) and reporting delays may be very long after initial
advice of the incident.
The insured is likely to notify the insurer of an incident having taken place fairly quickly.
However repairs may be deferred until the vessel is scheduled for its next refit, when the
full extent of the damage and the likely costs incurred will be reported to the insurer. This
could be months, or even years later, so the delay before all information is received could
be significant.
 Settlement delays might be long if there is a dispute over legal liability or the amount
that should be paid.

 Minor damage is often repaired when the vessel goes into dock for routine
maintenance.

 Claim amounts can vary from relatively small amounts for minor hull damage to
small vessels, to very large amounts in respect of the complete loss of a large
vessel and its cargo.

Accumulations of risk are quite possible with marine insurance:


 Geographical concentration would make an insurer exposed to the possibility of lots of
claims arising from one incident. Storms or tidal waves could affect many ships in one
harbour. A good example of this would be where an insurer sells lots of policies to a
particular yacht club and a severe storm then damages all the yachts.
 If a ship spills hazardous material in a populated coastal area, there may be many liability
claims.

© IFE: 2019 Examinations The Actuarial Education Company


SP7-03: Insurance products – types Page 31

Goods in transit
There may be reporting delays if claims are not reported until vehicles or vessels reach their
destination. However, these generally amount to months rather than years.

Construction and engineering


Property claims can generally be reported and settled quickly, although as with other
classes serious damage may take a relatively long time to repair. Liability claims will take
longer to settle as liability needs to be established before amounts of loss can be
determined. However, the claims from a single underwriting year can take many years to
emerge because of the long-term nature of the policies and the fact that policies often cover
a multi-year contract period.

It may be hard to determine the date of loss for a construction policy.

Extended warranty
Claim costs are fairly uniform by product, as they are related to the cost of the original
product. Where repairs are covered, there is a risk of multiple repairs being needed, which
may increase the claim cost above the original price of the goods.

Claims can arise because a proportion of goods sold turns out to have been faulty, which
may be stable, or there could be a large number of claims caused by a defective product
design. This would add some volatility to the reporting patterns.

Crop insurance
Most claims will be reported at the end of the growing season, although some may be notified
during the season. Most claims will be settled quickly, ie very soon after the growing season.

Claims size will be related to the size of farms insured. The claim frequency is likely to be low in
most years.

There will be significant accumulations of claims from adverse weather conditions / pest
epidemics. Accumulations might occur from the same geographic region or type of crop.

2.6 Risk factors and rating factors

Household property (buildings and contents)


A measure of the scale of the risk (for example the amount of sum insured or number of
rooms) is the key risk factor. The remaining principal risk factor is location because such
risks as theft, subsidence and flooding vary by location.

In this class many risk factors have been used as rating factors. Regression analysis has
shown their suitability to differentiate between levels of risk.

Common rating factors include:

 sum insured

 number of rooms

 location

The Actuarial Education Company © IFE: 2019 Examinations


Page 32 SP7-03: Insurance products – types

 the voluntary or compulsory use of excesses

 whether there is any business use of the property

 whether the policyholder owns or rents the property

 if the property is normally unoccupied during the day

 whether it is a house or flat or some other construction

 type and standard of construction

 age of the building

 type of locks and/or burglar alarms fitted.

This might seem like enough but there are many more that are frequently used, for example:
 whether smoke alarms have been fitted
 high risk contents
 type of cover (‘new for old’ or indemnity)
 family composition
 smoker / non-smoker
 type of heating
 age of policyholder.

By using a no-claim discount or other experience-rating system the insurer may be able to
make an adjustment for any other influences on the level of risk. This is, however,
comparatively rare in household insurance.

It is important to realise that the rating factors used may vary by country. For example, in
countries where the risk of burglary is relatively high, the insurer may ask prospective
policyholders if the windows of the property are barred, or if the homeowner is a member of an
armed response unit. (This is an armed unit that would come to the property in an emergency to
try to prevent the burglary and ensure that the residents were safe.)

Question

Consider the rating factor ‘whether the property is normally unoccupied during the day’. Justify
whether this affects the claim frequency or the claim severity or both.

Solution

It affects the claim frequency because a house that is unoccupied during the day is more likely to
be burgled than an occupied house. Also, a fire is less likely to be noticed at an early stage and be
put out to prevent a claim being made.

It also affects the claim severity. Burglars will have more time to steal more items, increasing the
claim size. A fire may have more time to spread and cause more damage. Also a burst water pipe
will cause more damage if not noticed and stopped immediately.

© IFE: 2019 Examinations The Actuarial Education Company


SP7-03: Insurance products – types Page 33

There may also be an impact on perils other than theft and fire.

Commercial property
Apart from the monetary value of the property and the surveyor’s report of the property, the
trade or business is the key risk factor.

Regression analysis has shown that the following rating factors might also be taken into
account:

 estimated maximum loss

 age of building

 fire protection equipment

 construction type

 excesses

 location of building

 hazardous building materials.

The estimated maximum loss is the largest loss that is reasonably expected to arise from a single
event. This may well be less than either the market value or the replacement value of the insured
property and is used as an exposure measure in rating certain classes of business.

For larger risks, the underwriters will take into account further factors:
 qualitative impressions: the nature of adjacent buildings, how well the company is run
(there will be less risk in a tightly-managed factory with disciplined procedures)
 previous claims experience, especially with smaller claims (large claims are so rare that
past experience of them is unlikely to be credible).

Motor property
Motor insurance is a good example of a class of insurance where possible risk factors can
be identified, but it is generally difficult to regard them as meeting the criteria needed to use
them as rating factors.

For example the following are all risk factors:

 number of miles driven

 density of traffic where the car is driven

 ability of the driver

 speed at which the vehicle is usually driven and its general level of performance

 ease with which the vehicle can be damaged and the cost of repairing it

 theft risk

 weight of the vehicle


 fire risk.

The Actuarial Education Company © IFE: 2019 Examinations


Page 34 SP7-03: Insurance products – types

However, the insurer cannot depend on information on these risks received from the
policyholder as there is considerable scope for the policyholder to stretch the truth in his or
her favour or be too subjective about his or her own skills. These are generally not all
measurable or quantifiable statistics.

However, there is in many countries a gradual increase in the use of telematics, whereby the
driving behaviour and other factors can be monitored through the use of a black box
installed in the insured vehicle. This makes some of the above factors measurable, and the
results can be used to help price the policy.

The term ‘black box’ here doesn’t refer to a box that is necessarily black (although it might be). As
explained earlier, it is a device or system that is placed in a motor vehicle to monitor the way in
which the vehicle is driven. It can measure speed, acceleration, braking, etc as well as monitoring
exactly when and where the vehicle is driven. It’s called a black box because we will generally
know very little about its inner workings.

Question

Suggest some other risk factors for motor insurance that can also be used as rating factors.

Solution

 type of cover
 excess
 value of the car and its contents (eg car stereo)

Type of cover is the most important rating factor, as varying the type of cover can exclude
an entire class of claims.

For example, third party cover will not include any claims for accidental damage to the insured’s
own vehicle.

Policy excess is also an important rating factor as it too will affect claim sizes.

Many small claims may be eliminated altogether leading to a reduction in claim frequency and
expense savings.

An excess may be compulsory (for example for a young driver) or optional to secure a
reduction in the premium. Typically insurers will offer proposers a choice of excess levels.

Other rating factors are proxies for those risk factors for which direct information is
unreliable. These include:

 the use to which the vehicle is put (eg for business use)

 age of the vehicle

 occupation of the policyholder and other drivers

 whether there are additional drivers of the vehicle as well as the policyholder

 gender of main driver (note that current European regulations ban gender as a rating
factor)

© IFE: 2019 Examinations The Actuarial Education Company


SP7-03: Insurance products – types Page 35

 age of policyholder and other drivers

 whether or not driving is restricted to certain named drivers

 make and model of vehicle

 the extent of any modification to the engine or body

 location of policyholder (eg postal code)

 where the vehicle is kept overnight: on the road / on a driveway / in a garage etc

 whether or not the driver has any driving convictions


 past experience.

Question

An insurer uses the rating factors ‘age of policyholder’ and ‘address of policyholder’. Suggest
which risk factors these are proxies for.

Solution

Age of policyholder:
 the ability of the driver (very young drivers tend to be more accident prone)
 the number of miles driven (some age groups tend to drive more than others)

Address of the policyholder:


 the density of the traffic where the car is driven
 the number of miles driven
 the theft risk

For fleet motor, the rating factors detailed above, other than those that apply to individual
drivers, are relevant. The important factors will be:
 types of vehicles
 type of cover
 level of excess
 types of use and goods carried.

Age of driver, used for private motor, would not be practical for anything other than the smallest
fleets. Insurers make use of the fleet’s own claims experience. In simple terms, the larger the
fleet, the more weight the insurer will put on the fleet’s own claims experience when setting
premiums.

For many larger fleets, experience rating will take the form of a profit-sharing arrangement,
whereby the insured receives a refund, if claims experience is good, or pays a further premium, if
claims experience is poor, at the year end.

The Actuarial Education Company © IFE: 2019 Examinations


Page 36 SP7-03: Insurance products – types

Marine and aviation property


The main risk factors for marine and aviation property are the size, type and age of the craft
or vessel and the nature of the cargo.

There are no definitive rating factors. Factors used might well depend on the rating factors
deemed appropriate for any associated marine liability cover.

Important factors will usually include:


 the type and value of the craft
 the scope of the voyages
 the areas covered or the destination
 the number and experience of the crew
 previous claims experience.

Goods in transit
The main rating factors are:

 mode of transport

 nature of goods

 type of storage used (for example refrigerated)

 time period of transit and number of stages

 length of time spent at warehouse.

Construction and engineering


The main rating factors are:

 type of project (for example, block of flats, airport, tunnel)

 term of project

 contracting firm

 materials and technologies used

 location of project.

Extended warranty
The premium will depend on the:

 make and model of the item being covered

 the length of the manufacturer’s guarantee

 the term of the warranty.

© IFE: 2019 Examinations The Actuarial Education Company


SP7-03: Insurance products – types Page 37

Crop insurance
Rating factors used may include:
 type and mix of crops grown
 geographical region
 size of the farms
 availability / method of irrigation
 pest control techniques used
 level of excess
 claims history
 sum insured.

The Actuarial Education Company © IFE: 2019 Examinations


Page 38 SP7-03: Insurance products – types

3 Financial loss
Financial loss insurance can be categorised as follows:

 fidelity guarantee

 credit insurance

 creditor insurance

 business interruption cover, also known as consequential loss

 legal expenses cover.

3.1 Benefits
The benefit provided is indemnity against financial losses arising from a peril covered by
the policy.

Fidelity guarantee insurance


Fidelity guarantee covers the insured against financial losses caused by dishonest actions
of its employees (fraud or embezzlement). These will include loss of money or goods
owned by the insured or for which the insured is responsible, and reasonable fees incurred
in establishing the size of the loss (paid to auditors or accountants, for example).

Fraud is wrongful or criminal deception intended to result in financial or personal gain.

To embezzle is to steal or misappropriate money or other assets placed in one’s trust or under
one’s control.

Credit insurance
Credit insurance covers a creditor against the risk that debtors will not pay their
obligations. The principal types are:

 trade credit

 mortgage indemnity.

Trade credit may cover uncollectible debts and be sold on an annual basis; cover may also
be for the length of a project, for example a ship built for a customer who does not pay for it
at the end of construction.

Mortgage indemnity covers the lender (mortgagee) in a mortgage loan against the risk of the
borrower (mortgagor) defaulting and the value of the property on which the loan is secured
not being sufficient to repay the loan. These policies may last for the duration of the
mortgage and have terms of many years.

The perils leading to default are not specified under credit insurance, so non-payment for any
reason is covered. Claim payments are usually single lump sums.

© IFE: 2019 Examinations The Actuarial Education Company


SP7-03: Insurance products – types Page 39

When a lender (eg a building society) provides a mortgage to an individual for house purchase,
the lender will be worried that:
 the borrower may default on the interest payments
 in taking possession of the property, the proceeds from the sale of the property may be
insufficient to cover the amount of the mortgage and outstanding interest.

Mortgage indemnity insurance protects the mortgage lender against the risk that the borrower
defaults on the loan and a loss is made by the lender.

Where the amount of mortgage is low in relation to the value of the property, the lender should
be quite relaxed because the sale proceeds will almost certainly cover the mortgage and interest.
However, where the mortgage is a large percentage of the property value, then the lender will
want a mortgage guarantee insurance policy to protect against the risk described above.

Typically, lenders require this insurance when the mortgage exceeds 75% of the valuation of the
property at purchase. The lender takes out the policy and then sometimes makes the borrower
pay the insurance premiums. With most insurance policies, the beneficiary of the policy cover
pays the premiums. Mortgage guarantee is unusual in this respect because sometimes the
borrower pays the premiums but the lender is the beneficiary.

Question

Explain the unusual features of mortgage indemnity insurance compared to other insurance
classes.

Solution

The policy is usually single premium but the cover may last for say 25 years. The policy is of no
direct benefit to the person often paying the premium, ie the borrower. In other words the
beneficiary of any claim is the lender.

The risk of a claim is closely linked to the economic cycle, with its impact on interest rates and
employment levels.

The claim amount is the difference between the resale value of the property and the amount of
the loan outstanding, including any interest due. This should reduce over time, although this also
depends heavily on economic conditions (eg house prices).

Creditor insurance
Creditor insurance provides cover to insureds who are subject to obligations to repay credit
advances or debt. Most policies are made to individuals to cover personal loans, mortgage
loans or credit card debts.

The cover is usually against disability and unemployment, on the basis that these perils
may prevent the insured getting an income. The policy will pay the regular loan payments
until the borrower has recovered, obtains new work or until the loan is fully repaid or a
maximum number of payments made.

The Actuarial Education Company © IFE: 2019 Examinations


Page 40 SP7-03: Insurance products – types

Creditor insurance for personal loans and credit cards is normally sold alongside an
associated life assurance policy that will repay the whole outstanding balance of the loan if
the insured borrower dies.

Creditor insurance is also known as payment protection insurance (PPI).

Business interruption cover


The financial consequences of a fire to a company can be much more significant than the cost of
repairs to premises. If the company’s production lines are hit, income from customers will be
much reduced until alternative production arrangements can be made. If this income stream was
being used to pay off loans from a bank, the accumulating interest charges can put further
financial strain on the company.

Business interruption cover indemnifies the insured against losses made as a result of not
being able to conduct business.

Methods used to achieve this need to take account of the following factors:
 Turnover will drop off dramatically after a major fire, but will (hopefully) start to build up
again as soon as production facilities can be restarted.
 When the policy is taken out, the proposer will need to assess how long it would take to
rebuild the business in the event of a major disaster. This period will then be specified in
the policy, and is known as the indemnity period. The insured will then be indemnified for
loss of profit over this period.
 Profit will depend on the proportions of fixed and variable costs incurred by the business.
 Additional temporary costs may also be incurred as a result of the fire.
 Assessing what the turnover figure would have been without the fire will also have to take
account of:
– any inherent seasonal variation in the business
– any organic growth (or shrinkage) in the business itself.

A fixed sum insured per day will normally be specified in the policy and the insured will
receive this until the property can be occupied again or until the term specified in the policy
has expired. There is usually a minimum period before a claim can be made. This cover is
also known as consequential loss and loss of profits.

Question

Define suretyship.

© IFE: 2019 Examinations The Actuarial Education Company


SP7-03: Insurance products – types Page 41

Solution

The Glossary defines suretyship to be a product ‘that provides a guarantee of performance or for
the financial commitments of the insured’.

It is a specialised class of insurance where one party (known as the surety) guarantees the
performance of an obligation by another party (the insured). There are three parties to the
agreement:
1) the party that undertakes the obligation
2) the surety that guarantees that the obligation will be fulfilled
3) the obligee, who receives the benefit of the surety bond.

It is therefore a form of financial guarantee, although, unlike ‘normal’ insurance the party that
undertakes the obligation still retains the risk, because it’s the obligee that gets the benefit.

This product is not covered in detail by the Core Reading, however the examiners frequently use
uncommon or specialised classes of business as a basis for exam questions. Therefore it’s a good
idea to read a little about unusual classes whenever you come across them. For more information
about suretyship, you can visit:

http://smallbusiness.chron.com/definition-surety-insurance-43269.html .

Legal expenses cover


Legal expenses cover indemnifies the insured against legal expenses incurred as a result
of:

 legal proceedings being initiated against the insured

 the need for the insured to initiate proceedings.

Legal expenses policies will normally cover the payments to legal representatives.

Personal legal expenses insurance is often sold as an optional add-on to a household policy.
Commercial legal expenses insurance can be sold as a stand-alone policy or as an add-on to a
policy such as employers’ liability or commercial property.

Such policies might cover legal expenses associated with a neighbourly dispute (personal) or
defending against a consumer complaint in respect of goods and services provided (commercial).

The Actuarial Education Company © IFE: 2019 Examinations


Page 42 SP7-03: Insurance products – types

3.2 Insured perils


For financial risks the insured perils will depend on the precise cover. The perils might
include:

 dishonest actions by employees (fidelity guarantee)

 failure of third parties specified in the policy (credit)

 accident or sickness resulting in loss of income with which to repay debt (creditor)

 fire at the insured’s own property (business interruption cover)

 fire at neighbouring premises causing loss of access to own property (business


interruption cover)

 legal proceedings being brought against the insured (legal expenses cover).

Business interruption cover is normally sold in conjunction with property insurance and will
cover the same perils: fire, weather losses and so on, and the business interruption claim
will follow a property damage claim.

3.3 Basis for cover


Financial loss insurance is likely to be written on a losses-occurring basis.

Legal expenses insurance is sometimes written on a basis combining losses-occurring and


claims-made. Claims must both occur and be reported within the policy period.

3.4 Measure of exposure to which premiums are related

Fidelity guarantee and credit insurance


For fidelity guarantee and credit insurance, the measure of exposure will depend on the
precise cover provided.

A possible, though imperfect, measure of exposure for mortgage indemnity guarantee


insurance is the excess of the amount of the loan over a certain percentage of the value of
the property.

This defined percentage is often referred to as the ‘normal advance’.

For example, if premium rates were £5 per £100 above the normal advance, a borrower had a
mortgage for £132,000 and the normal advance was £114,000 (75% of the property valuation of
£152,000), then the premium would be £900.

Creditor insurance
The exposure measure for creditor insurance on personal loans is normally the amount of
the loan or the total amount repayable. On a mortgage it will be the insured monthly benefit.
On a credit card it will normally be the outstanding balance at the latest monthly statement
date.

Business interruption cover


The exposure measure for consequential loss is the annual profit or turnover.

© IFE: 2019 Examinations The Actuarial Education Company


SP7-03: Insurance products – types Page 43

Obviously this cannot be known in advance so adjustment premiums are common.

The amount payable in the event of complete cessation of the business will normally be set
out in the policy. This should be set so that the insured has an incentive to get the
business running again as soon as possible.

Legal expenses cover


The number of policyholders or policyholder-years are possible exposure measures for
legal expenses insurance.

3.5 Claim characteristics


Financial loss insurance covers a wide variety of risks that can give rise to a financial loss.
Frequency and claim cost distributions tend to vary by class, but in general these risks tend
to be short tailed, even if the policies concerned have relatively long terms for general
insurance.

Fidelity guarantee insurance


It may take time to discover that there has been fraudulent behaviour by employees, so there
may be reporting delays for fidelity guarantee insurance. Settlement delays could be significant,
as it may take time to establish the size of the loss (including the resulting fees, eg to auditors).

The potential size of fidelity guarantee insurance claims could be large (especially for large
companies), however claim sizes may also depend on the level of regulation in the territory.

Credit insurance
The claims experience on mortgage indemnity guarantee insurance depends heavily on
economic factors, as was illustrated in the UK in the early 1990s when some insurers
incurred severe losses on this type of business.

Question

Suggest which economic factors are likely to affect claims experience, and what effects these
factors could have.

Solution

Claims experience is likely to depend on:


 interest rates – higher interest rates are more likely to lead to default on mortgage
payments (for variable rate mortgages)
 house prices – falling house prices might lead to negative equity; if the insurer is forced to
take possession of the property, then the property value may be insufficient to cover the
amount of the mortgage and any outstanding interest
 rising unemployment – individuals may be unable to afford mortgage payments.

The Actuarial Education Company © IFE: 2019 Examinations


Page 44 SP7-03: Insurance products – types

Creditor insurance
Creditor insurance claims will normally be a series of payments made until the insured
recovers or the limit on payments is reached:

 The payments on personal loan policies will be the monthly repayment specified in
the loan agreement; these loans are normally made on a rate of interest that does
not vary after the loan is taken out.

 The monthly benefit on mortgage policies is normally a set amount selected by the
insured when he or she takes the policy out and linked to the monthly repayment,
although it may sometimes be varied if interest rates change.

 The monthly benefit on a credit card policy is usually the minimum monthly payment
on the balance as at the monthly statement date preceding the claim.

The frequency of claims depends on factors such as the rate of unemployment (higher if
there is a recession) and the likelihood of policyholders becoming sick or having accidents.

Business interruption cover


For business interruption cover claims, the reporting delays are directly linked to any
associated property claim. Settlement, however, is likely to be slower than for property
claims, owing to the greater need for verification, although once a loss is established
payments can be made regularly.

3.6 Risk factors and rating factors


The risk factors and rating factors for financial loss depend on the cover provided and the
nature of the policy.

Fidelity guarantee and credit insurance


Fidelity guarantee and trade credit are commercial policies that will be individually
underwritten, and the nature of the business and the size of the sums at risk will be taken
into account.

Mortgage indemnity insurance premium is usually based on the amount of loan in excess of
a certain proportion of the value of the property and will take into account the quality of loan
underwriting by the lender.

With mortgage indemnity insurance, in practice insurers do not incorporate the circumstances of
the individual borrower into the rating structure. They rely upon the lender to be prudent in
granting mortgages.

Also, the effect of house price falls will be factored into the premium rates on a global basis rather
than different rates for different houses.

The term of the mortgage might be used for rating. Otherwise, standard rates will apply for all
policies issued through particular mortgage lenders.

© IFE: 2019 Examinations The Actuarial Education Company


SP7-03: Insurance products – types Page 45

Creditor insurance
Creditor insurance cover for personal loans is usually provided at a fixed price percentage
of the amount lent or the total amount repayable. Premium rates will vary with the term of
the personal loan. The premium will either be paid as a single amount at the start of the
loan (in which case it is usually financed as an addition to the loan) or as a monthly addition
to the repayment, in which case it will also be covered by the insurance.

The premium for mortgage cover is a proportion of the insured monthly benefit, payable
each month.

The monthly premium for credit card cover will be a proportion of the outstanding balance.

No allowance is made for the profile of the individual being covered.

In reality, the risk factors include:

 age

 gender (note that current European regulations ban gender as a rating factor)

 employment status

 occupation

 state of health

 amount of monthly benefit.

When setting the premium rates for a lender’s business the underwriter will consider the
composition of the lender‘s book of business in relation to these variables.

Business interruption cover


The risk and rating factors are likely to be similar to those for commercial property damage
insurance. Depending on the exposure measure, annual profits and/or turnover may also be
used.

The insurer may also want to incorporate the insured company’s dependence on the economic
cycle into the rating process, since companies that are dependent on the economy will face
significantly larger (smaller) claim sizes during a boom (recession).

Legal expenses cover


For legal expenses, the rating is normally based on the sum insured.

The Actuarial Education Company © IFE: 2019 Examinations


Page 46 SP7-03: Insurance products – types

4 Fixed benefits
Fixed benefit claims arise under personal accident insurance.

A personal accident contract may specify an amount to pay in the event of the insured suffering
the loss of a limb, £50,000 for losing an arm, for instance. It is very hard to quantify the value of
the loss of a limb and so a fixed benefit is provided.

There are also other fixed benefits insurance products, examples of which are described below.

4.1 Benefits

Personal accident cover


Benefits are usually specified fixed amounts in the event that an insured party (this may
include the policyholder’s family as well as the policyholder) suffers the loss of one or more
limbs or other specified injury, or accidental death. This is not indemnity insurance
because it is not possible to quantify the value of the loss, for instance, of an arm.

Cover may be offered on a group basis by an employer to all employees. Alternatively (or
additionally), it might be combined with a sickness policy.

An individual is usually assumed to have an unlimited financial interest in his or her own
life, and this may be extended to personal injury, although an insurer should take care to
avoid moral hazard by granting large sums insured for relatively minor injuries.

For example, an individual is fairly unlikely to risk their life just because they are safe in the
knowledge that death will lead to a life insurance payout. Similarly, an individual is unlikely to
take less care in a particular activity and risk losing a limb. However, loss of a finger or toe might
be considered to be minor and so an individual may not take as much care not to lose one of
these, especially given that it might lead to an insurance payout of several thousand pounds.

Often a policyholder will be able to select his or her level of cover, which will be paid in full
if he or she suffers the complete loss of or loss of use of a major limb or permanent total
disablement. Often there is a reducing scale of benefits, known as a continental scale, for
lesser injuries, in which each specified injury is worth a set proportion of the full sum
insured.

Policies such as this are often sold as units of sum insured; for example a unit might be
£25,000 and the policyholder may select a number of units of cover, up to a maximum.

Other fixed benefits cover


Fixed benefits may also be granted for periods of disability through accident or injury or for
periods of hospitalisation. In this case the benefit is usually a fixed amount per day, week
or month until recovery, possibly after a waiting period for benefit.

© IFE: 2019 Examinations The Actuarial Education Company


SP7-03: Insurance products – types Page 47

4.2 Insured perils


Here the perils are any form of accident that results in the loss of limbs or other specified
injury.

For example, some motor policies cover fixed personal accident benefits to the policyholder in
respect of injuries arising from the peril of motor accidents caused by the policyholder.

4.3 Basis for cover


Personal accident insurance will typically be written on a losses-occurring basis.

4.4 Measure of exposure to which premiums are related


For personal accident insurance the true measure of exposure is the person-year multiplied
by the sum insured.

This is because the cover is defined in relation to the level of cover normally required by one
person. However, in many cases, the member-year or employee-year may be all that is available.

In other words, if family members are covered under the group or individual plan then ideally we
would need more details of these members to calculate the exposure measure. However, in
practice this is often not available and so member-year is used instead.

The insurance may cover the whole family at a standard rate but the insurer may or may not be
made aware of the number of family members. The insurer charges a rate assuming an average
family unit size and, hopefully, gets it right on average.

Question

Describe the risk to the insurer of using a standard rate to cover the whole family.

Solution

The insurer will be exposed to the risk of selection. The policy might only be attractive to large
families. This would particularly be the case if other insurers do use the actual family details in
the rating process. The insurer would usually market the policy very carefully to minimise the
selection risk.

For group policies the number of people covered by a policy may need to be adjusted at the
year-end and the premium adjusted in line with their risk characteristics.

Under personal accident insurance if policyholders can select more than one unit of sum insured,
this too will need to be reflected in the exposure.

For group schemes, where the scale of benefits is linked to salaries, the exposure measure is often
the sum insured or total salaries.

The Actuarial Education Company © IFE: 2019 Examinations


Page 48 SP7-03: Insurance products – types

4.5 Claim characteristics


Claims are usually reported quickly. The incidence of an event is usually very clear (so
reducing reporting delays), although with accidental death claims the insured’s dependants
may not always know the policy exists and may discover their entitlement after an extended
period.

The claims may be settled quickly, although if a claim is for permanent total disability it may
be necessary to wait some years for a claimant’s condition to stabilise. As the claim cost is
known for these claims the settlement delays are reduced.

As well as paying out on specified injuries, many personal accident policies will make a payment if
the insured is ‘permanently and totally disabled’. It may, however, take several months or years
before establishing that this is the case for these claims.

The claim frequency tends to be reasonably stable.

Claims can be large: cover of several hundred thousand pounds per person is not
uncommon.

4.6 Risk factors and rating factors


Apart from sum insured, the prime factor affecting the risk is usually occupation. Those
employed in more dangerous occupations such as demolition work will certainly need to be
distinguished from those in safer occupations such as clerical work.

Other factors may be:


 age – age is not usually a major determinant of risk until old age is reached, and this
cover is not normally sold to the elderly

 gender – gender may be a factor: women tend to be less prone to accidents than
men (note that current European regulations ban gender as a rating factor)

 health condition may also be a determinant of risk– those in worse health may be higher
risk than average
 dangerous pastimes – those following hazardous hobbies may also be higher risk
than average.

The rating factors are the same as the risk factors, because they can all be measured.

© IFE: 2019 Examinations The Actuarial Education Company


SP7-03: Insurance products – types Page 49

5 How policies may be combined

5.1 Motor insurance policies


Recall that motor insurance cover may be for:
 third party liability, or
 property damage.

However, it is common for motor liability cover and cover for damage to the vehicle to be
provided in a single policy.

For example, in the UK, a motor policy that provides cover for liability and damage to the
vehicle from all perils including accident is known as ‘comprehensive’. In some other
European countries, it is common for motor liability and damage policies to be provided
separately.

Recall that the main perils for motor insurance are:


 accidental or malicious damage to the insured vehicle
 fire or theft to the insured vehicle.

In many countries, including the UK, this cover is typically provided together with motor
third party cover within a single policy, whilst in other countries it may be provided in a
separate policy.

For example, in some states in Australia, the three main types of policy are broadly similar to the
UK:
 third party property damage
 third party fire and theft
 comprehensive.

However, none of the options include third party bodily injury claims. This is split into a separate
category and is sometimes insured by the State with the premiums forming part of the annual
registration fee.

Motor policies will usually also include public liability cover. A motor policy may also include
personal accident cover.

5.2 Marine and aviation policies


Aviation policies will usually also include public liability cover.

Commercial marine policies, however, usually exclude public liability cover, which is provided by
Protection and Indemnity Clubs.

Marine and aviation are often considered separately from other forms of insurance because of
their wide range of cover, the other peculiarities they demonstrate and because historically,
market practice has been to deal with them separately. However, the types of cover are
essentially the same as for other forms of transport such as land vehicles.

The Actuarial Education Company © IFE: 2019 Examinations


Page 50 SP7-03: Insurance products – types

5.3 Buildings and contents insurance policies

Household property insurance


Household policies are often sold as a package, covering both buildings and contents.

One advantage of this is that the separation between the two is sometimes not entirely
clear: which fixed items are part of the fabric of the property and which are contents?
However, tenants are not normally responsible for arranging buildings insurance and will
need only contents insurance.

Public liability cover may also be included in such policies. Buildings and contents insurance are
also provided as separate contracts.

Commercial property insurance


Small businesses are conventionally covered by ‘package’ policies, which include
buildings, contents and liability insurance. They may also include business interruption
(see Section 3, Financial loss).

5.4 Contractors’ All Risks (CAR) policies


Construction risks are typically insured under a contractors’ all risks (CAR) policy. This will
include the contractor’s liability for losses caused to third parties.

The principle of ‘all risks’ policies is to cover all perils not specifically excluded. Examples of
possible exclusions for a CAR policy include:
 liability to employees working on the project (as this would often be covered by a
separate employers’ liability policy)
 damage to the work-in-progress resulting from faulty workmanship
 damage to any property which existed on the contract site prior to the work commencing
(eg if an extension to an existing building was being constructed)
 damage due to wear and tear.

© IFE: 2019 Examinations The Actuarial Education Company


SP7-03: Insurance products – types Page 51

6 Emerging risks
Many industries have been facing a time of rapid change as technology changes what
people and companies are able to do, and industry disrupters have become a hot topic.
These changes can bring about new opportunities for the insurance market but also new
risks.

Industry disruptors are innovators that disrupt the marketplace. For example, aggregator
websites at one time disrupted the traditional personal lines insurance market.

6.1 Cyber risk


Cyber attacks are, at the time of writing, deemed to be one of the world’s top technological risks.
The topic is covered in more detail in Subject SA3.

Cyber insurance, one of the newer areas in insurance, presents the opportunity to sell an
additional product to cover a new need by policyholders that may be excluded from a
standard policy.

The insurer will indemnify the insured against losses incurred as a result of a specified
cyber or data loss event. This may include legal expenses.

For example, legal expenses could arise due to the need for a company to:
 take legal action against hackers
 defend its position against claimants, eg users of its services who suffered as a result of a
cyber attack.

Cyber and data risk insurance is designed to support and protect a business if it
experiences a data breach or is subject to an attack by a malicious hacker that affects its
computer systems.

Cover includes both liability and intellectual property losses for businesses that hold
sensitive customer data, use computer systems to conduct their business, or have a
website. Policies are likely to be individually underwritten to reflect the different needs of
the policyholders.

The main measures of exposures include:

 company revenue for business interruption related cover

 the value of IT equipment at risk.

Another possible exposure measure might be the volume of customer records held.

The Actuarial Education Company © IFE: 2019 Examinations


Page 52 SP7-03: Insurance products – types

The premium rate will depend on a number of factors, including the:

 IT maturity of the insured

 dependence of the revenue on IT

 industry sector

 volume and nature of data held by the insured


 risk management of the insured, eg data security measures and back-up procedures
 privacy policy of the insured.

Given the fast pace at which the cyber risk landscape is evolving, coupled with the lack of
ability to assess the IT maturity of each insured at the point of underwriting, underwriters
are increasingly using cyber security firms’ data and software to assess the potential for a
cyber-attack on an insured.

Loss events can occur from a wide range of perils. Common perils include:

 virus damage, eg due to an employee opening an infected email attachment

 hacker attack
A recent example of such an event is the attack on the telecommunications company
TalkTalk of 2015 in which thousands of customers’ details were stolen, including bank
account numbers and dates of birth.
 infringement of intellectual property, eg a film studio may seek compensation if hackers
steal a digital copy of an unreleased movie and leak it online
 slander or libel, eg if an employee unintentionally libels a third party in an email

 online identity fraud, eg this could be covered by a personal cyber insurance policy and
may include the costs of restoring the policyholder’s credit record as well as
compensation for direct financial losses
 personal data or privacy issues (eg loss of or disclosure of private data)

 theft of corporate information and data


 losses relating to breaches or actions by suppliers or business partners, eg an
insurer itself may need cover against the risk of a data breach at one of its brokers
 ransomware (a type of malicious software that threatens to publish the victim's data or
block all access to it unless a ransom is paid)
A notable recent example of a ransomware event is the ‘WannaCry’ worldwide attack in
2017.
 business interruption, eg due to a distributed denial of service (DDOS) attack
 physical damage, eg if hackers seize control of infrastructure forming part of an electricity
grid and damage it.

© IFE: 2019 Examinations The Actuarial Education Company


SP7-03: Insurance products – types Page 53

7 Glossary items
Having studied this chapter you should now read the following Glossary items:
 Adjustment premium
 All risks
 Business interruption insurance
 Cancellation
 Consequential loss insurance
 Deductible
 Deposit premium
 Escalation clause
 Estimated maximum loss (EML)
 First loss
 Fleet
 Latent claims
 Loss of profits
 New for old
 Probable maximum loss (PML)
 Replacement
 Suretyship.

The Actuarial Education Company © IFE: 2019 Examinations


Page 54 SP7-03: Insurance products – types

The chapter summary starts on the next page so that you can
keep all the chapter summaries together for revision purposes.

© IFE: 2019 Examinations The Actuarial Education Company


SP7-03: Insurance products – types Page 55

Chapter 3 Summary
Liability
The essential characteristic of liability insurance is to provide indemnity – subject to
maximum levels of benefit and/or excesses – where the insured, owing to some form of
negligence, is legally liable to pay compensation to a third party.

The extent of any legal liability may depend on the prevailing legislation. Legal expenses
relating to the liability are usually also covered.

Significant reporting and settlement delays mean that most liability classes are long tailed.
Claims tend to be relatively infrequent and claim cost distributions tend to be widely spread.

The main types of liability insurance are:


 employers’ liability / workers’ compensation
 motor third party liability
 marine and aviation liability
 public liability
 product liability
 professional indemnity and errors and omissions (E&O) liability
 Directors’ and Officers’ (D&O) liability
 environmental liability and pollution liability.

Employers’ liability indemnifies the insured (the employer) against legal liability to
compensate an employee / their estate for bodily injury, disease or death suffered, owing to
negligence of the employer. The benefit may be regular payments or a lump sum. The perils
include accidents and exposure to harmful substances or working conditions. The main
measure of exposure is payroll. The main risk factor is the type of industry.

Workers’ compensation is another name for employers’ liability. In some countries,


however, workers’ compensation may refer to losses suffered by employees during the
course of employment regardless of whether the employer has been negligent.

Motor third party liability indemnifies the owner of a motor vehicle against compensation to
third parties for personal injury or damage to their property. Cover is usually on a
losses-occurring basis. The measure of exposure is the vehicle-year. The risk and rating
factors are as for motor property damage.

The Actuarial Education Company © IFE: 2019 Examinations


Page 56 SP7-03: Insurance products – types

Chapter 3 Summary continued


Marine and aviation liability indemnify the insured against the legal liability to compensate a
third party for bodily injury, death or damage to property arising out of operation of the
vessel or aircraft. Exclusions are likely to include terrorism, war and illegal activities. Cover
is usually on a losses-occurring basis. Possible measures of exposure include passenger
kilometres / voyages and in-service seats. A wide range of rating factors is used in practice.

Public liability indemnifies the insured against legal liability for the death of or bodily injury
to a third party or for damage to their property from any other cause. The perils will depend
on the type of policy. The most common measure of exposure is turnover.

Product liability indemnifies the insured against legal liability for the death of or bodily injury
to a third party or for damage to their property that results from a product fault. Cover is
likely to be on a claims-made basis. Perils include faulty design, faulty manufacture, faulty
packaging and incorrect or misleading instructions. The most common measure of exposure
is turnover. The nature of the product is an important risk factor, however each policy will
be individually underwritten.

Professional indemnity indemnifies the insured against legal liability for losses resulting from
negligence in the provision of a service. Cover is usually on a claims-made basis. Perils may
include wrong medical diagnosis, and errors in a medical operation or an actuarial report.
The most common measure of exposure is turnover. The nature of the profession is an
important risk factor, however each policy will be individually underwritten.

D&O liability is a type of professional indemnity cover that indemnifies the insured against
the legal liability to compensate third parties owing to any wrongful act of the insured
director. Cover is usually on a claims-made basis. Perils include wrongfully allowing a
company to continue operating, acts resulting in the insured being declared unfit for the role
and allowing the publication of false financial statements. The nature of the profession is an
important risk factor, however each policy will be individually underwritten.

Environmental liability indemnifies the insured against the legal liability to compensate third
parties for the death of or bodily injury to a third party or for damage to their property as a
result of unintentional pollution for which the insured is deemed responsible. Cover is
usually on a claims-made basis. Perils are any incident causing environmental pollution.
Each policy will be individually underwritten.

© IFE: 2019 Examinations The Actuarial Education Company


SP7-03: Insurance products – types Page 57

Chapter 3 Summary continued


Property damage
The main characteristic of property insurance is to indemnify policyholders against loss of or
damage to their own material property.

The main types of property that are subject to such damage are:
 residential, commercial and industrial buildings
 moveable property (contents)
 land vehicles
 marine and aircraft
 goods in transit
 property under construction
 engineering plant and machinery
 goods insured under an extended warranty policy.

Property claims are usually reported and paid quickly. They are often more predictable in
frequency and size than liability claims.

Residential, commercial and industrial buildings insurance usually pays the amount needed
to restore the property to its previous condition, subject to any excess / deductible. The
main perils are fire, explosion, lightning, theft, storm, flood, subsidence and damage caused
in putting out fires. The exposure measure is usually the sum-insured year. The sum insured
should be the cost of rebuilding / restoring the property. Subsidence claims may suffer
reporting delays. Many rating factors are likely to be used.

Moveable property can pay an amount equal to the replacement value of the item, or may
be on a ‘new for old’ basis. The main perils are fire and theft. Malicious and accidental
damage may also be covered. The exposure measure is usually the sum-insured year,
although this may be less appropriate for commercial covers, where the level of stocks is
very variable. Determining levels of stocks may lead to settlement delays. Many rating
factors are likely to be used.

Land vehicle insurance provides a benefit to repair the vehicle, subject to a maximum of the
replacement value of the vehicle. The exposure measure is the vehicle-year. Many rating
factors are likely to be used, including the level of excess.

The Actuarial Education Company © IFE: 2019 Examinations


Page 58 SP7-03: Insurance products – types

Chapter 3 Summary continued


Marine and aircraft may cover loss of or damage to the craft and the cargo. Perils include
fire and explosion. The exposure measure may be the insured value of the hull or aircraft
and may allow for cargo. Claims may not be reported until the vessel reaches a port. There
are no definitive rating factors.

Goods in transit is sometimes sold alongside marine and aviation insurance. The main perils
are damage, loss and theft. The exposure measure is typically the consignment value.
Claims may not be reported until the end of the journey. The main rating factors are mode
of transport, nature of goods, type of storage, time period / number of stages of journey and
length of time spent at warehouse.

Construction insurance may be provided for projects over the course of their lifetime (and
sometimes beyond), and so may last for longer than a year. The main perils are damage,
destruction, design defects, faulty parts and failure to finish the construction project. The
exposure measure may be the value of the contract, which is unlikely to be uniform over the
lifespan of the project, so allowance must be made for this. The main rating factors are type
and term of project, contracting firm, materials and technology used and location of project.

Engineering is similar in nature to construction. The main perils are machinery breakdown,
explosion and electronic failure. The exposure measure may be the sum insured value of the
contract, which is unlikely to be uniform over the lifespan of the project, so allowance must
be made for this. Claim costs are likely to be fairly uniform, although there may be
accumulations. Rating factors are similar to those for construction.

Extended warranty covers losses arising from the need to replace / repair faulty parts in a
product beyond the manufacturer’s normal warranty period. Such policies may have a term
of several years. The exposure measure is usually the number of appliances or
appliance-years. The main rating factors include make / model of item, length of
manufacturer’s guarantee and term of warranty.

© IFE: 2019 Examinations The Actuarial Education Company


SP7-03: Insurance products – types Page 59

Chapter 3 Summary continued


Financial loss
Financial loss insurance can be categorised as:
 fidelity guarantee
 credit insurance (trade credit and mortgage indemnity guarantee)
 creditor insurance
 business interruption cover (also known as consequential loss)
 legal expenses cover.

The benefit provided is indemnity against financial losses arising from a peril covered by the
policy. Claims tend to be relatively short tailed.

Fidelity guarantee covers the insured against financial losses caused by dishonest actions of
employees. Contracts will be individually underwritten – the nature of the business and the
size of the sums at risk will be taken into account.

Trade credit covers uncollectible debts. The term of cover may be annual or may depend on
the length of a project. As above, contracts will be individually underwritten.

Mortgage indemnity guarantee covers the lender against the borrower defaulting on the
loan. The exposure measure may be the excess of the amount of the loan over a certain
percentage of the value of the property. Claims experience depends heavily on economic
factors. The premium will be affected by the quality of the lender’s loan underwriting.

Creditor insurance generally covers individuals who cannot make loan repayments due to
disability or unemployment. The loans may be personal loans, mortgages or credit card
debts. Benefits are usually limited to a maximum number of payments. The exposure
measure is usually the amount of the loan / the total amount repayable. Claim frequency
depends on the rate of unemployment and the likelihood of policyholders becoming sick /
having accidents. The premium will vary with the term of the loan. No allowance is made
for the profile of the individual being covered.

Business interruption cover indemnifies the insured against losses made as a result of not
being able to conduct business. Perils are similar to those for property damage. The
exposure measure is usually annual profit or turnover. Settlement may be slower than for
the associated property damage claim.

Legal expenses cover indemnifies the insured against legal expenses from court proceedings
being brought against / by the insured. The exposure measure may be the number of
policyholders. Premiums will depend on the sum insured.

The Actuarial Education Company © IFE: 2019 Examinations


Page 60 SP7-03: Insurance products – types

Chapter 3 Summary continued


Fixed benefits
Fixed benefit claims arise under personal accident insurance.

Personal accident cover usually provides fixed benefits in the event that the insured party (or
parties) suffers one of a specified set of injuries / losses or death. The individual may be able
to select the level of cover. The exposure measure is the person-year multiplied by the sum
insured. Claims tend to be reported and settled quickly. The main rating factor tends to be
occupation, although age, sex (if permitted as a rating factor) and hobbies may also be used.

Packaged products
Different types of general insurance products may be combined under a single policy. For
example:
 motor third party liability and motor property
 residential buildings and contents (with some public liability cover)
 commercial buildings, contents and business interruption (with some public liability
cover).

© IFE: 2019 Examinations The Actuarial Education Company


SP7-03: Insurance products – types Page 61

Chapter 3 Practice Questions


3.1 (i) List the different types of commercial liability insurance.

(ii) State which types of liability cover may be written as personal lines business.

3.2 State whether the following risk factors are likely to affect the frequency of motor insurance
claims, the size of motor insurance claims, or both:
(i) where the car is driven
(ii) how expensive the car is to replace / repair
(iii) how fast the car is driven.

3.3 (i) Explain why mileage is not used as the exposure measure in private motor insurance.

(ii) State which of the primary rating factors may potentially act as proxies for mileage.

3.4 List the main measures of exposure used in practice for the following types of insurance:
(i) employers’ liability
(ii) household contents
(iii) motor liability.

3.5 List the main risk factors for a comprehensive private motor policy.

3.6 Describe the features of extended warranty insurance under the following headings:
 cover available and perils insured
 common exclusions
 measure of exposure
 risk factors
 rating factors used in practice
 usual forms of underwriting
 major characteristics of claims.

3.7 State the main perils and risk factors associated with marine liability insurance.

3.8 The insurance company for which you work specialises in employers’ liability business. Discuss
Exam style
the points you would include in a short report summarising the nature of the business for a new
director whose previous work experience has been in insurance. [13]

3.9 Describe the main drivers of claims experience for a general insurer selling product liability
Exam style
insurance. [8]

The Actuarial Education Company © IFE: 2019 Examinations


Page 62 SP7-03: Insurance products – types

The solutions start on the next page so that you can


separate the questions and solutions.

© IFE: 2019 Examinations The Actuarial Education Company


SP7-03: Insurance products – types Page 63

Chapter 3 Solutions
3.1 (i) Commercial liability
 employers’ liability
 motor third party liability (individual vehicles or motor fleets)
 marine and aviation liability
 public liability
 product liability
 professional indemnity and E&O (errors and omissions) liability
 D&O liability
 environmental liability and pollution liability

(ii) Personal lines liability


 motor third party liability
 public liability (this is likely to be part of another type of insurance)
 environmental liability (this may be part of another type of insurance,
eg household buildings)

3.2 (i) Where the car is driven will have a big impact on the claim frequency.

(ii) How expensive the car is to replace / repair will have most effect on the claim size.

(iii) How fast the car is driven will probably influence both the frequency and the size of the
claims.

3.3 (i) It is administratively less convenient (eg there would be year-end adjustments). Verifying
mileage is deemed to be impractical.

(ii) Most of these factors might be acting as a proxy for mileage in one way or another:
 drivers with ‘business use’ will probably do more miles than drivers with ‘social
and domestic use’
 cars with very low rating groups tend to do fewer miles per year
 new cars are usually driven further than older cars
 young drivers may drive further than older drivers
 drivers in rural locations may drive further
 drivers with high mileage may have more claims and have worse NCD ratings.

The Actuarial Education Company © IFE: 2019 Examinations


Page 64 SP7-03: Insurance products – types

3.4 (i) annual payroll

(ii) sum insured years

(iii) vehicle-years

3.5 Main risk factors for comprehensive private motor:


 the ability of the driver
 how long on risk (usually a year)
 the number of miles driven
 how much time the car is used
 how many passengers carried
 the density of the traffic where the car is driven
 the time of day the car is driven
 the speed at which the vehicle is usually driven and its general level of performance
 the ease with which the vehicle can be damaged and the cost of repairing it
 where the car is left when not used (the theft risk)
 the safety and security features of the car (eg ABS braking system, alarms etc)
 size and weight of the vehicle
 fire risk.

3.6 Cover available and perils insured

As the name suggests, extended warranty insurance extends the warranty on an item beyond the
manufacturer’s own warranty, eg from one to five years.

This form of policy is often taken out by purchasers of electrical goods, eg laptops, TVs, iPads,
washing machines, etc.

The article purchased is insured against defects arising from faulty manufacture for the extended
period of warranty.

Cover might be for cost of replacement parts and/or labour charges for repair.

Note that these policies have the unusual feature that cover is not for one year. Also, the policy is
normally not renewable at the end of the period.

Common exclusions

The policy might cover damage incurred as the result of an accident. Not surprisingly, deliberate
acts of vandalism will be excluded.

Normal wear and tear is not covered.

© IFE: 2019 Examinations The Actuarial Education Company


SP7-03: Insurance products – types Page 65

Measure of exposure

The number of appliances or appliance-years.

Risk factors

The value, robustness, quality of manufacture, type and complexity of the appliance and term of
the policy are important determinants of risk.

The amount of use of the appliance by the policyholder will also be relevant.

Some ‘moral hazard’ exists from claims that should not really be covered by the policy. With most
claims being for relatively small amounts, it may be difficult for the insurer to spot all the invalid
claims. The honesty of the policyholder is therefore a risk factor.

Rating factors used in practice

Although the individual policyholder will influence the amount of risk, no rating factors are used
to reflect this. (It would be totally impractical. It would also cause resentment from
policyholders.)

The make and model of the appliance are the key rating factors. In practice, appliances are sorted
into categories, and a premium set for each category, eg TV, laptops. Some categories are also
subdivided by original purchase price, eg TVs up to £500 in value.

Other rating factors are:


 the length of the manufacturer’s guarantee
 the term of the warranty.

Usual forms of underwriting

Little or none.

Major characteristics of claims

Delays

This is a short-tail class. Claims are almost always reported very quickly. Settlement is usually
very quick. Delay could occur if there is some dispute over whether a claim is actually covered
under the individual’s policy.

Claim frequency

Claim frequency can vary a lot between appliances and manufacturers. Whereas old technology
(eg fridges) might go on for years without a fault, some newer appliances (eg laptops) can be far
less reliable.

Claim severity

Claim costs are fairly uniform by product, as they are related to the cost of the original product.
Where repairs are covered, there is a risk of multiple repairs being needed, which may increase
the claim cost above the original price of the goods.

The Actuarial Education Company © IFE: 2019 Examinations


Page 66 SP7-03: Insurance products – types

Accumulations

A new product type or a batch of an existing product could be defective, leading to many claims.
This would not usually be obvious when the appliances were sold.

3.7 Perils

 storm
 fire
 explosion
 collision
 pollution

Risk factors

 size of vessel
 type of vessel
 age of vessel
 condition and maintenance of vessel
 miles travelled
 routes travelled
 crew training / ability
 management ability
 nature of the cargo

3.8 Cover is provided for employers against claims from employees who are injured or become ill as a
result of their employment, provided the employer is negligent. [1]

Claims can arise from:


 accidents resulting in loss of limbs or other injury, or [½]
 exposure to harmful substances resulting in disease (eg lung disease caused by asbestos
or coal dust) [½]
 exposure to noisy working conditions resulting in deafness. [½]

Cover is usually based on occurrence of accident ... [½]

... but this is difficult to establish for illness claims where the problem occurs over a period of
time. [½]

© IFE: 2019 Examinations The Actuarial Education Company


SP7-03: Insurance products – types Page 67

Premiums are based on:


 size of company (usually by annual payroll)
 occupations of employees
 processes and materials handled
 company’s experience with liability claims (large companies). [½ each]

Other underwriting factors are:


 location
 medical facilities
 safety procedures
 training and management procedures. [½ each]

Premiums are not determined precisely until well after the year end due to changes in exposure
during the year. [½]

Most claims, by number, are for relatively minor injuries, mostly reported and settled quickly. [½]

There is large variation in the size of claims, with a few very large claims. [½]

However, the large claims make up a large proportion of the total claim amount. [½]

There are potential problems with slow emerging diseases (eg industrial deafness), leading to
outstanding liabilities for claims yet to be reported. [½]

There can sometimes be extensive claim settlement delays (eg from establishing liability,
establishing amount). [½]

There can often be high legal costs associated with a claim. [½]

Wage inflation, court award inflation and legal cost inflation can be significant. [½]

Due to the length of the tail, accounts may show an underwriting loss but investment return may
be sufficient to make business profitable. [½]

Many accumulations of risk arise, eg many employees are on one site, many may handle a
particular material, also a ‘benchmark’ court award may lead to many other claims. [1]

Business is usually sold through brokers. [½]

Cover is sometimes packaged with other types of cover for small businesses. [½]
[plus up to bonus 1 mark for relevant additional points, maximum 13]

The Actuarial Education Company © IFE: 2019 Examinations


Page 68 SP7-03: Insurance products – types

3.9 Product reliability is the key driver. [½]

Technological improvements should make products safer … [½]

… but as technology advances, there are more things to go wrong due to newer, perhaps
untested, components. [½]

Competitive pressures might lead to manufacturers cutting costs, which may increase the chances
of problems occurring. [½]

Greater use of outsourcing for manufacturing (eg to overseas countries) may lead to companies
having less control over production, also leading to problems. [½]

The extent of quality control procedures will have a big influence. [½]

There is an increased consumer awareness of rights and obligations. Attitudes to claiming are
such that society is more litigious. [1]

Perhaps as a result of this, legislation puts ever-increasing demands on safety procedures and
testing. [1]

There are also ever-increasing penalties for non-compliance. [½]

Policy wording is another influence. Although many claims are tested through the judicial
process, many will be contested due to unclear terms and conditions. [1]

Brand and reputation is also a driver of experience. For example, there have been cases of
product recalls not due to faulty products but because it has been discovered that child labour
has been used in their manufacture. [1]

Inflation is also a driver. Judicial decisions will have the heaviest influence on this, but also the
inflation of earnings. [1]
[Maximum 8]

© IFE: 2019 Examinations The Actuarial Education Company


SP7-04: Problem solving Page 1

Problem solving
Syllabus objectives
6.1 Analyse hypothetical examples and scenarios in relation to the financial
management of general insurance companies.
6.1.1 Propose solutions and actions that are appropriate to the given context, with
justification where required.
6.1.2 Suggest possible reasons why certain actions have been chosen.
6.1.3 Assess the implications of actions within a given scenario.
6.1.4 Discuss the advantages and disadvantages of suggested actions, taking into
account different perspectives.

The Actuarial Education Company © IFE: 2019 Examinations


Page 2 SP7-04: Problem solving

0 What is expected of candidates?


The ability to look at a problem, solve it and then communicate the results clearly is a very
important skill that actuaries need in their day to day work, whichever field they work in. The
examiners will test your ability to solve such problems in the exam.

There is no explicit Core Reading for this objective.

Well, actually there is – but it’s not technical like it is in the other chapters.

All the material that is covered in the rest of the Core Reading for this subject also covers
this objective. The examiners will expect candidates to be able to apply the knowledge and
understanding that they have developed through the study of this Core Reading to produce
coherent advice and recommendations for the overall management of a general insurer.

Candidates should recognise that the exams will contain scenarios and situations that have
not been covered in the Core Reading, in order to test that candidates have understood the
material sufficiently to apply their understanding to a new situation. The Subject SP7
exams, however, do not require any knowledge beyond that contained in the Core Reading
(although there are often additional marks available for candidates with a broader
understanding). In so far as an exam question requires underlying knowledge that is not
contained in the Core Reading, this will be provided as part of the question.

Where candidates are preparing for the exam using broader educational materials (for
example, as provided by ActEd) which contain both the Core Reading and additional
content, we recommend that candidates aim for a greater level of familiarity with the Core
Reading elements. Only the Core Reading itself is directly considered by the exam setting
team and a higher quality of answer is generally required, particularly on bookwork
questions that are directly based on Core Reading. However, the additional content should
be of great value in interpreting the Core Reading and understanding ways to apply it.

Candidates will also be expected to solve problems that may be set in novel or unfamiliar
circumstances or for which there are no generally recognised solutions by:

 drawing upon knowledge and understanding of similar products, situations and


current issues

 applying actuarial techniques and concepts

 exercising judgement

 utilising the information provided in the question.

In Section 3, we illustrate how to put these techniques into practice by considering an unusual
product which is unlike any of the products you will have seen in Chapters 2 and 3.

Although there is significant overlap in underlying content between this exam and
Subject SA3, the level of understanding expected of candidates is different. Candidates are
expected to have a solid grasp of the technical aspects of general insurance work and some
understanding of the broader commercial, regulatory and operational context; the
non-technical elements are the greater focus of Subject SA3.

Broadly, we would expect a passing Subject SP7 candidate to be a mid-senior ranking


member of a function team (eg reserving, pricing, capital), depending on the size of the
organisation, who is able to operate with limited supervision for routine analytical work, but
would draw upon more senior expertise for complex or contentious situations.

© IFE: 2019 Examinations The Actuarial Education Company


SP7-04: Problem solving Page 3

1 Examples
The problems candidates are expected to tackle may be of a similar level of complexity to:

 applying standard actuarial techniques to a situation, eg triangle projection methods


or compiling a balance sheet

 working with incomplete, distorted or unreliable historical data and understanding


the challenges and implications of this

 managing necessary data adjustments for an analysis, eg allowing for inflation,


exposure, earning distortions or time period issues

 applying actuarial insight to a review of data provided in a question to identify any


features pertinent to the question, or any anomalies or errors

 understanding the different claims characteristics of particular product lines and


how they might affect a particular situation

 assessing the implications for insurance markets of regulatory changes or


proposals

 understanding the structure of a capital model and key considerations in its


operation and use

 evaluating the risks and benefits associated with a proposed venture or strategy

 designing a suitable reinsurance structure given an insurer’s risk profile and


appetite or calculating outcomes for a reinsurance programme.

These are only some general examples, and past exam papers provide a good indicator of
the type of questions that might appear on an exam and how some of the generic questions
above might be examined in practice.

The Actuarial Education Company © IFE: 2019 Examinations


Page 4 SP7-04: Problem solving

2 Exam techniques
Stronger candidates (and indeed the exam setting team itself in generating the marking
schedule) will have a set of lists for each element of the course to remind themselves of the
points that are worth considering on a particular topic. These lists can be compiled from
the Core Reading, past exam papers and sample questions. Candidates can refine the lists
every time there is a relevant point that their lists did not pick up.

Where possible, lists should be supplemented by a general toolkit for idea generation that
can be applied flexibly to a variety of situations to generate a range of potential points.

The strongest candidates also accompany this with a good understanding of the situation
and the underlying concepts, which helps to identify points that are particularly salient, or
that are not relevant given the specifics of the way the question is constructed. Although
there are no negative marks given for providing incorrect or irrelevant answers, candidates
will create unnecessary time pressures for themselves if they do not filter their thoughts to
address the questions as set.

Below we provide some suggestions for idea generation or for compilation of useful
revision lists:

 Consider the stakeholders in any situation, for example:


– regulators
– reinsurers
– rating agencies
– customers or claimants
– management
– investors and/or shareholders
– brokers
– underwriters
– competitors.
 Consider the different functions within an insurance company which form a
necessary part of the infrastructure for any product or company to be effective, for
example:
– claims handling and reserving
– finance
– underwriting and pricing
– sales and marketing
– risk
– compliance
– IT and Operations
– data and management information
– capital modelling
– reinsurance purchasing.

© IFE: 2019 Examinations The Actuarial Education Company


SP7-04: Problem solving Page 5

 Visualise what is actually going on, for example:


– what a policyholder actually does
– what they are buying a policy for
– what the real world event is that triggers a claim
– how a claim is assessed and resolved etc.
Doing this as part of revision will help with the generation of examples.

 Think what is likely to go wrong in practice; some of the questions you may ask
yourself are:
– Is there data?
– Is it of adequate quality?
– Is it representative?
– Can you get it out of the IT systems or process it?
– Are other stakeholders or colleagues being honest with you (or with
themselves)?
– Are there political constraints that will mean the analysis isn’t considered
fairly?
– Will your competitors or the regulators change the environment before you
can respond?

 Use your knowledge of the Core Reading and any professional guidance.
 Consider each of the areas of the course or use a ‘mega-list’ of all the key words in the
course.
 Consider the steps involved in a process.
 Consider the actuarial control cycle.
 Use the clues in the question, and do a quick brainstorm of each detail.
 Use ideas from your own experience (but avoid the temptation to go over the top if you
know a lot of detail about a particular subject).
 Use ideas from similar questions (but make sure that you tailor them to the question being
asked).
 Ask yourself questions such as: why, when, who, how, what for, etc.

The above is by no means exhaustive nor necessarily the best framework for all candidates,
but it is helpful to develop some form of toolkit for idea generation. Candidates can also
evolve and enhance that toolkit by working through practice questions, and for any points
on the marking schedule that they didn’t think of, trying to identify a transferable question
they could have asked themselves which would have led to that point. This is most
effective if attempting the question under exam conditions, as many points which seem
obvious on reading a solution may not have been generated under exam conditions.

Candidates should also try to avoid thinking of questions as being confined to one topic
area, such as reserving, or capital, as this may lead to a lack of confidence in answering
questions they perceive to be outside their area of professional experience. These
traditional practice areas are just purposes to which an actuary’s understanding and
judgment can be applied.

The Actuarial Education Company © IFE: 2019 Examinations


Page 6 SP7-04: Problem solving

If a candidate has a good understanding of the underlying product and of the practical
operation of an insurance company, they should be able to apply this to a variety of
purposes, and there is in any case significant overlap. Data issues, for example, will create
challenges in every area, and the same risk and uncertainty characteristics that a pricing
model would seek to capture with rating factors can lead to large loss or late development
characteristics that create reserving challenges, or to volatility and uncertainty issues that
need to be reflected in a capital model.

It is always useful, however, for candidates to engage with colleagues working in a different
practice area to understand their perspectives on the same underlying portfolios, for their
own professional effectiveness as well as for exam preparation.

The Examiners’ Reports also contain a significant level of recurring advice for each subject
(most notably a strong recommendation to make sure that you read the question properly).
For each paper, they also highlight the mistakes commonly made by candidates on each
question, which should be of use in avoiding similar errors when sitting the exam in future.
We recommend that candidates read this.

© IFE: 2019 Examinations The Actuarial Education Company


SP7-04: Problem solving Page 7

3 Tackling an unusual product

3.1 Unusual classes of business


The Subject ST7 exam, the predecessor to Subject SP7, quite frequently covered unusual or
specialist classes of business, many of which only get a passing mention in the Core Reading, if at
all. Some notable examples include:
 sports injury insurance
 crop and hail insurance
 livestock insurance
 cyber-risk insurance
 insurance for a school fundraising game
 utility price increase insurance
 negative equity insurance
 gap motor insurance etc.

It would take you months to read up on every conceivable type of insurance cover, and we don’t
suggest you try. When very unusual or innovative products are examined, the cover provided by
the product is usually described in the question.

You can find even more examples of questions on unusual general insurance products by looking
through past exam questions for Subjects ST3, ST8 and SA3. These questions could easily be
adapted to make future Subject SP7 exam questions.

The purpose of this section is to enable you to:


 apply the principles covered in Chapters 2 and 3 to think of perils, risk factors and claims
characteristics for an unusual product
 recognise when these claims characteristics are relevant for reserving and/or capital
modelling.

Section 3.2 below covers a (very) unusual product by way of a question. To get the most from this
section you should attempt the question yourself before looking at our ideas.

3.2 Question
The National Football League is about to be rocked by a breakaway of the biggest 22 clubs. They
are going to form their own Ultimate League, with substantial extra finance promised to them by
a consortium of cable and satellite TV companies. At the end of each season the bottom three
clubs in the Ultimate League will be demoted to the top division of the remaining Football League
and replaced by the three teams who have finished at the top of that division.

The Actuarial Education Company © IFE: 2019 Examinations


Page 8 SP7-04: Problem solving

The chief executives of the 22 prospective breakaway clubs have approached the large general
insurance company that you work for. They are seeking to negotiate a new form of insurance for
the clubs against the risk of demotion from the Ultimate League to the (impoverished) Football
League, in which their share of the money from television fees is likely to be minimal.

A student actuary who is very keen on football has provided you with the following background
information:
‘There is a reasonably clear hierarchy of football clubs. The big ones (like Bigclub United, AC
Winsagain and Yeovil Town) have been winning trophies for years, while others of the breakaway
clubs (like Notsogood United, Real Trouble and Manchester City) are expected to struggle in the
new regime. Football is a funny old game though, and these expectations may not be fulfilled. At
the end of the day it’s just eleven players against eleven players, after all.’

You have been asked to prepare a report for the directors of your company on this proposed new
form of insurance.

Read the question


Reading the question carefully is an important skill that you must develop. As you read, you may
find it helpful to underline key words and annotate the question paper, or rough paper, with your
ideas. (If this seems a touch patronising we apologise, but a slightly different style is required for
the later exams and for some students this could be their first attempt at one of the later
subjects.) Having read it quickly a few moments ago, go back to the question again now before
reading our thoughts on the question, which are given below.

Thoughts on the information given


‘The National Football League is about to be rocked by a breakaway of the biggest 22 clubs. They
are going to form their own Ultimate League, with substantial extra finance promised to them by
a consortium of cable and satellite TV companies. At the end of each season the bottom three
clubs in the Ultimate League will be demoted to the top division of the remaining Football League
and replaced by the three teams who have finished at the top of that division.’
 Note that 3 teams from 22 are demoted each season. All other things being equal this
would imply a claim probability of 3/22.
 ‘Substantial extra finance’ means that any club demoted will lose significant revenue for
the following year.
 Can these TV fees be predicted at the start of the season?
 Is there a schedule of who will be on TV and when, or would we need an end of year
adjustment to reflect actual exposure?
 Do all clubs get an equal share of the fees or do some get more than others?

‘The chief executives of the 22 prospective breakaway clubs have approached the large general
insurance company that you work for. They are seeking to negotiate a new form of insurance for
the clubs against the risk of demotion from the Ultimate League to the (impoverished) Football
League, in which their share of the money from television fees is likely to be minimal.’

© IFE: 2019 Examinations The Actuarial Education Company


SP7-04: Problem solving Page 9

 Note that your company is large. This may mean that you have the resources available to
consider such a one-off arrangement. On the other hand, it may mean that you are not
interested in odd one-off arrangements that will presumably imply considerable per
policy expenses. Conversely, there may be prestige and free publicity in being involved
with the project.
 Note this is a new form of insurance. This means there is no direct historical data to use.
However we could consider league positions in the previous season as some indication of
the likelihood of demotion this season.
 We need to decide what form the new insurance will take.
 This is a niche market with a maximum of 22 policies per annum. We would want to know
how many clubs will take out the cover.

‘There is a reasonably clear hierarchy of football clubs. The big ones (like Bigclub United, AC
Winsagain and Yeovil Town) have been winning trophies for years, while others of the breakaway
clubs (like Notsogood City, Real Trouble and Manchester City) are expected to struggle in the new
regime. Football is a funny old game though, and these expectations may not be fulfilled. At the
end of the day it’s just eleven players against eleven players, after all.’
 There is clearly a difference in the likelihood of demotion between the big clubs and some
of the others likely to struggle. This would imply that we cannot just treat all clubs as
equals.
 A probable assumption, in the absence of better information, is that the big clubs appear
on TV more often and get a higher share of the money than the smaller clubs.

Now try some structured thinking


So far, all you have done is read the question, think about it and annotate the question paper.
But note how many initial points have been raised. The next stage is to do some more structured
thinking about the question. There are a number of valid approaches. The order in which you
apply them is not important.

Ask yourself as many basic generic questions as you can sensibly devise about the product
described. Then try to answer them.

(a) Questions about the cover to be provided


 Who is covered?
 What perils are to be covered?
 When does the cover apply, and when does it cease?
 Why should the club purchase cover?
 Are there any exclusions that should be applied?
 What is the sum insured?
 Are we providing indemnity cover?
 Do we want to impose a maximum amount of cover?

The Actuarial Education Company © IFE: 2019 Examinations


Page 10 SP7-04: Problem solving

Comments

Football clubs are to be indemnified against potential loss of TV fees following demotion.

The cover should be purchased before the start of the season, so a cut-off date may be
appropriate. The sum insured should be limited to a percentage, perhaps 100%, of the current
year’s fees.

The insurer needs to decide whether the claim period should be for one year only or if claims
payments should be made until the club regains promotion. The former is likely to be the only
option acceptable to the insurer. The latter option is too uncertain and open-ended. The risk to
the insurer is likely to be perceived as too large. There would also be the concern that the club
would have no incentive to regain promotion to the Ultimate League. This is an example of
potential moral hazard.

It may well be desirable to place a monetary maximum on the amount of cover provided. This
could provide protection against one of the big clubs, with a large share of the fees, being
demoted. This is an example of how the insurer could limit its exposure to large claims and
control the risks it writes.

(b) Questions about the method of sale


 What is our distribution method and target market?
 What commission is payable?
 What likely business volumes can we expect?
 Can we provide a bundled package of cover for other insurance too?
 What implications are there for administration and computer systems?

Comments

The target market is clearly defined here as the 22 clubs of the Ultimate League. There is no
guarantee that all clubs will take out the cover, or that we are the only insurer that has been
approached to provide cover. Hence, our potential sales volume ranges from 0 to 22 policies.

(c) Questions on modelling the business


 What factors will most influence claim frequency / severity?
 Can these be incorporated into a reserving model or will we have to use more aggregation
within our groupings?
 What data should we collect?
 What are the key sources of claims uncertainty?
 Will there be correlations with other classes of business, or with other risk types?
 When will claims occur?
 How will claims be assessed?
 Is there potential for disputes?

© IFE: 2019 Examinations The Actuarial Education Company


SP7-04: Problem solving Page 11

Comments

The size of TV fees will be a key factor to model. The exact amount of TV fees lost due to
demotion may not be known until the end of the following season, so the insurer will need to
estimate the amount of TV exposure as well any inflation in TV fees. The initial estimate could be
based on last season’s share of the TV money.

With only 22 clubs in the breakaway league, there will be insufficient data for a detailed statistical
analysis, so very few groupings will be used.

Data from other leagues could be collected though, eg to estimate transition rates from one
league to another.

Alternatively a more detailed (and probably more subjective) analysis could incorporate sources
of demotion risk, eg:
 poor performance
 injuries
 lack of finance
 poor management
 the sale of good players.

Note that the last item is an example of possible moral hazard against the insurer, which will:
 lead to increased risk of disputes
 be particularly difficult to model.

Claim amounts will also probably depend on:


 the share of TV fees  this will be commercially sensitive, but clubs are likely to be willing
to provide it under confidentiality agreements
 the number of TV appearances in the Football League compared to the Ultimate League
 this information is unlikely to be available at first but the insurer may be able to collect
useful industry data over time
 the number of seasons the demoted team remains in the Football League (if cover is
provided until the club regains promotion)  so the insurer should consider:
– an individual club’s past experience, eg its position last year, as this affects the
likelihood of a claim
– the results of a Markov chain model to estimate the likelihood of being promoted,
based on industry data.

Premiums will all be earned on the same date, since clubs will be demoted at the end of the
season, so:
 reserves should not be released until before the end of the season
 there will be increased correlations between insurance risk and liquidity risk, since several
clubs may claim at the same time.

The Actuarial Education Company © IFE: 2019 Examinations


Page 12 SP7-04: Problem solving

(d) Questions about the possible claim profile


 What is the likely claim frequency and average amount?
 What sort of reporting and settlement delays do we expect?
 Is this a long- or short-tailed class of business?
 Are claims fixed or unlimited?
 What potential is there for large claims or accumulations?
 Do we need to purchase any reinsurance protection?
 What investigations are we able to carry out?
 What potential is there for selection against the insurer?
 Is there any potential for moral hazard?

Comments

Depending on the number of policies sold, the claim frequency suffered by the insurer could be
anything between 0% and 100%.

Claim amounts, assuming we limit indemnity to only one season’s loss of fees, will be no higher
than that of the largest club. We could reduce this amount further by imposing a monetary
maximum.

Delays should be short. Teams will soon know when they have been demoted. Indeed, we may
well be aware that this is likely several matches before the end of the season. Settlement delays
should also be short. Hence this will be a short-tailed insurance product.

There is potential for very poor experience. We may only sell three policies, and all three clubs
may be demoted. The insurer could use some form of non-proportional reinsurance to protect
itself from this risk. There is a risk that only the clubs likely to be demoted take out cover; the
insurer should consider how it can avoid being selected against.

Question

The chief executive designate of the Ultimate League Executives is also, coincidentally, a director
of your company. They have unofficially proffered the following additional information:
(a) Taking out this insurance might be made compulsory for all clubs as a condition of joining
the Ultimate League.
(b) Given the poor international record of the National Football team, there is a growing
opinion amongst some of the players, managers and tabloid press in favour of a smaller
league. There may, as a result, be a gradual reduction in the size of the Ultimate league
from 22 to, say, 18 clubs over four seasons.

Comment on the implications for the insurance company of these options in turn, assuming that
all other details of the new form of insurance are agreed between the company and the chief
executives.

© IFE: 2019 Examinations The Actuarial Education Company


SP7-04: Problem solving Page 13

Solution

(a) The big concern about selection is much reduced if all clubs have to take out this
insurance. However, we do not know if the cover is compulsory with this insurer, or
whether it has competition. If we have competition we may still be faced with the
situation of insuring three clubs and seeing them all relegated. Alternatively, if we are
insuring all 22 clubs we know that the claim frequency will be 3/22. The only uncertainty
for the insurer will be the claim amounts.

(b) We will need to know how this is to be accomplished, eg it could be by demoting three
and promoting two clubs, or demoting four clubs and promoting three for a period of
years. We should assess whether the expected claim frequency would be 3/18 if we
insured all clubs. At first glance it might be assumed that there is no effect on potential
claim amounts. However, if the same pot of money is to be shared amongst 18 rather
than 22 clubs then average claim amounts could increase.

Other valid approaches you could use to generate ideas here are:
 Working from a checklist of ideas, to check that you have not missed a possible different
angle.
 Role playing. Imagine you are trying to set the reserves for this business. Think about the
questions you would want answers to, and ways in which the insurer could be selected
against. Alternatively, imagine you are the football club and think about your insurance
needs or things you might do for your own benefit.

What we have set out here is far more than you might expect to generate on the basis of what
you have seen so far. However, it does highlight the sort of thought processes required for the
exam.

The task, then, is to present the ideas generated in the time allowed, and in a suitable form for
the examiner. This is a skill that you need to practise on Assignments, Exam questions and past
paper questions. Try to devise new forms of insurance, then think about how you would make
them work, what the perils are, how you would model them, etc …

The Actuarial Education Company © IFE: 2019 Examinations


Page 14 SP7-04: Problem solving

4 Suggested reading
We do not make any specific recommendations for additional reading for Subject SP7, and
as noted at the beginning of this chapter there is no need for any additional knowledge
beyond that contained in the Core Reading. There is a significant volume of information
available online for any candidates who wish to carry out further reading but we would
always recommend that candidates prioritise doing extensive practice on past exam papers
and sample questions, ideally under exam conditions as far as is practical.

© IFE: 2019 Examinations The Actuarial Education Company


SP7-04: Problem solving Page 15

Chapter 4 Practice Questions


4.1 You are an actuary working for a large general insurance company who writes a large range of
Exam style
personal lines products. The company is looking to move into the pet insurance market writing a
product that covers domestic cats and dogs. You have been asked by the personal lines director,
who knows very little about this product, to write a report on the product features of pet
insurance, and the considerations to be made in designing the product.

Outline the points you would make in your report. [9]

4.2 A general insurance company writes commercial property insurance in a small country. Up until
Exam style
now, the sale of alcohol in this country has been strictly controlled by the State. As a result, all of
the country’s pubs and bars were State-owned, with any risks being borne by the State (ie they
were self-insured).

Regulation is about to change, and so private individuals (known as landlords) will be allowed to
run pubs and bars. Your company is considering insuring these landlords.

(i) Describe the particular risks to the insurer of writing this type of business. [9]

(ii) Outline the restrictions and exclusions the insurer might place on the cover. [4]
[Total 13]

4.3 Describe the characteristics of the liabilities arising for an insurer writing travel insurance
business.

The Actuarial Education Company © IFE: 2019 Examinations


Page 16 SP7-04: Problem solving

The solutions start on the next page so that you can


separate the questions and solutions.

© IFE: 2019 Examinations The Actuarial Education Company


SP7-04: Problem solving Page 17

Chapter 4 Solutions
4.1 Consideration should be given to the benefit provided, any maximum levels of benefits, and any
indexation of these benefit levels. [1]

Benefits might include:


 vets fees, which may be capped at a maximum amount per year, per illness or per
treatment [½]
 hospital benefits, which may be capped at a maximum total amount, or a maximum stay
in hospital [½]
 death benefit (for death by accident or illness up to a certain age), which should cover the
value of the pet, up to a maximum value [½]
 loss benefit, which should cover the value of the pet, up to a maximum value [½]
 advertisement costs if the animal goes missing [½]
 kennel fees if the owner is hospitalised [½]
 quarantine costs [½]
 liability cover if the pet attacks a third party [½]
 liability cover if the pet damages property. [½]

There may be different bands of benefit. [½]

Bearing in mind the benefits provided, it would be necessary to consider the perils that are likely
to be insured. [½]

The period of cover of the contract may be one year (so that it is an annually renewable product)
or a single premium contract covering the pet over its lifetime. [1]

Terms and conditions would need to be clearly defined. [½]

The terms and conditions of the product may include:


 the level of excess (if any), which may be a fixed amount, or a percentage of the claim [½]
 exclusions, eg:
– certain routine treatments and vaccinations [½]
– any pre-existing conditions [½]
– certain breeds [½]
– certain uses of pet [½]
– claims resulting from incidents outside of certain geographical limits [½]
– war / nuclear risks [½]
 eligibility criteria, such as minimum and maximum age at entry [½]
 any cancellation terms under the policy. [½]

The Actuarial Education Company © IFE: 2019 Examinations


Page 18 SP7-04: Problem solving

In determining benefit levels, terms and conditions, customer needs would need to be
considered. [½]

It may be necessary to offer different products through different distribution channels. [½]
[Maximum 9]

4.2 (i) Particular risks

There is likely to be a lack of claims data available with which to accurately price the product.
Even if the State could provide statistics on claims this is unlikely to be sufficiently detailed or
relevant for this purpose, particularly given this is a small country. [1]

Volumes of business will be uncertain. It will be difficult to predict how many people will own
pubs, and hence require this type of insurance. Competition from other insurers will also be a
very relevant factor here. [1]

It will be difficult to predict the mix of business – eg by type of pub or location – the insurer will
be undertaking. [½]

The insurer will have little experience in knowing what underwriting and rating factors to ask for
and use (and to what extent), which may leave it open to undesirable risks and anti-selection. [1]

As there have not been any similar products in the market it will be difficult to establish clear and
appropriate policy conditions (including exclusions). This may result in payment of claims that are
not intended (or may lead to low sales volumes). [1]

Claim sizes at any one time could be very variable – for example, some liability claims (eg as a
result of a serious injury) could be large. [½]

Also, claims may be very volatile over time – for example, there may be the occasional serious fire
leading to total damage to the property (and many liability claims). [½]

The volatility and variability of claims is made worse because the country is small, and so
experience will be limited. [½]

There will be uncertainty over the public’s attitude to claiming. For example, there may be an
increasing number of members of the public suing the pubs for accidents occurring on the
premises, leading to an increase in liability claims. [1]

Crime rates will be uncertain. This will, for example, affect claims for arson and theft. [½]

The attitude of landlords to claiming is uncertain. For example, fraudulent claims may be an issue
(eg a new pub fails to make money, and so the landlord makes false or exaggerated insurance
claims to make up the loss). [1]

The impact of legislation may be uncertain. The government is likely to enforce controls on these
new pubs (eg restrictions on opening hours), which are likely to change as the situation
develops. [1]

© IFE: 2019 Examinations The Actuarial Education Company


SP7-04: Problem solving Page 19

Uncertainty over economic conditions is another source of risk. This may affect sales volumes
(and renewals), as people are less likely to want to run pubs (and afford insurance premiums) in
times of economic depression. [½]

Possible accumulations of risk may arise, eg if the pubs insured are all in a similar location (it is a
small country). [½]

Reinsurance may not be available or may not be offered at a suitable price (eg liability cover may
be scarce or expensive). [½]
[Maximum 9]

(ii) Restrictions and exclusions

The insurer might decide not to cover certain properties, if these represent an unacceptable level
of risk for the insurer. [½]

For example:
 buildings with flammable materials, eg pubs with thatched roofs [½]
 buildings in high-risk locations, such as on river banks (flooding risks). [½]

The following exclusions might also apply:


 any deliberate loss, such as arson initiated by the landlord
 situations where the policyholder has not taken reasonable steps to prevent the loss (eg a
break-in where security measures have not been activated)
 liability which is the fault of a third party
 loss arising as a result of war, terrorism and similar risks
 loss arising from illegal or criminal activity
 loss or theft where the police have not been informed
 loss or theft of cash (and other items that would be difficult to verify)
 costs below a specified excess level or above a specified policy limit. [½ each]
[Maximum 4]

The Actuarial Education Company © IFE: 2019 Examinations


Page 20 SP7-04: Problem solving

4.3 This is a short-tail class consisting of property damage claims (eg for lost luggage) and medical
expense claims.

The claims are mainly small in size, although some medical expense claims can be substantial.

Property damage claims tend to be linked to price inflation. Medical expense claims are linked to
the increase in medical costs over time (which tends to be higher than price inflation).

The majority of property damage claims are likely to be settled in the domestic currency after the
policyholder returns from the destination, but some property damage claims and the medical
expense claims will need to be settled in an overseas currency.

Note that travel insurance is not an unusual product in practice. However, since the Core Reading
does not discuss this class of business in detail, this is an ideal opportunity to practice the skills
discussed in this chapter.

© IFE: 2019 Examinations The Actuarial Education Company


SP7-05: Reinsurance products – background Page 1

Reinsurance products –
background
Syllabus objectives
1.2 Describe the main types of general reinsurance products and the purposes for which
they may be used.

Covered in part in this chapter.

The Actuarial Education Company © IFE: 2019 Examinations


Page 2 SP7-05: Reinsurance products - background

0 Introduction

0.1 What is reinsurance?


Insurance companies provide insurance to individuals and corporations against many risks
(hurricanes, earthquakes, lawsuits, collisions, sickness and death, and so on).

Reinsurers, in turn, provide reinsurance to insurance companies and reinsurance companies


(retrocession).

Chapter structure
In this chapter, we discuss reinsurance in general terms. In particular:
Section 1: Insurance for insurers and reinsurers
Section 2: Other participants in the reinsurance market
Section 3: Reasons for purchasing reinsurance
Section 4: Ways of writing reinsurance business
Section 5: Traditional reinsurance – an overview
Section 6: Other types of reinsurance and alternatives to reinsurance – an overview

In the next chapter, we will consider the different reinsurance products in detail, as well as some
alternatives to traditional reinsurance.

0.2 Reinsurance language


There is quite a lot of specialist language used in reinsurance. Although you will soon get used to
it, you will find it useful to be familiar with the following terms immediately:

cede = ‘pass on’ or ‘give away’, as in ‘cede some risk to a reinsurer’

facultative = ‘individual’, as in an individually negotiated arrangement

treaty = covers a group of policies – reinsurance that the reinsurer is obliged to


accept, subject to certain conditions set out in a treaty

direct writer = the insurer with a direct contract with the insured (as opposed to a
reinsurer, who has a contract with the direct writer), also called the
primary insurer or cedant.

© IFE: 2019 Examinations The Actuarial Education Company


SP7-05: Reinsurance products – background Page 3

1 Insurance for insurers and reinsurers

1.1 Reinsurance
Reinsurance is a form of insurance. It is a means by which an insurance company obtains
from other insurance companies (reinsurers) protection against the risk of losses.

Reinsurance is very important, both in practice and as part of Subject SP7. It is important that you
take your time over this chapter and the next chapter, to make sure you are comfortable with
what reinsurance is and why an insurer may purchase reinsurance, as well as the different types
of reinsurance product.

1.2 Retrocession
Retrocession is also a form of insurance. However, rather than being bought by individuals
(insurance) or insurers (reinsurance), it is bought by reinsurers.

Retrocession is reinsurance of reinsurance and is a very specialist area of the reinsurance


market. A retrocession protection is necessary where a reinsurer that accepts substantial
amounts of reinsurance business requires reinsurance protection itself.

The ceding reinsurer in a retrocession is called the retrocedant and the assuming reinsurer
is called the retrocessionnaire.

The Actuarial Education Company © IFE: 2019 Examinations


Page 4 SP7-05: Reinsurance products - background

2 Other participants in the reinsurance market

2.1 Brokers
Brokers primarily fulfil a sales role. Their purpose is to act for their client – the insurer – to obtain
reinsurance on its behalf.

Reinsurance brokers may specialise in the reinsurance markets only, or else deal with insurance,
reinsurance, retrocession and other financial products.

2.2 Direct reinsurance placements


Reinsurance need not only be handled by reinsurance brokers. In some international
markets, professional reinsurers often deal directly with insurers in placing their
reinsurance. These relationships could have come about historically based on business
culture, customs, practices or language.

Therefore, it is not uncommon for reinsurance intermediaries (ie brokers) to compete with
professional reinsurers who offer the service of a broker directly to the client.

2.3 Fronting
Fronting occurs when an insurer, acting only as a conduit, underwrites a risk and cedes all
(or nearly all) of the risk to another insurer (which is technically acting as a reinsurer). The
ceding or ‘fronting’ insurer will typically receive a fee for its involvement to cover its
expenses and profit.

Despite the additional fee, there are a number of reasons why the assuming insurer may use
a fronting arrangement rather than underwrite the risk directly. For example:

 It may not be licensed to write a specific line of business in a particular country or


state of the US.
In some territories, it is a legal requirement for all classes or for certain classes of
insurance to be written by a local insurer. A fronting arrangement effectively allows
insurers outside the territory to cover risks in such a territory.
Sometimes, a single insured may have insurance coverage needs in many different states
and the insurer may not be licensed in all of them. Fronting arrangements could then be
used to ‘fill in the gaps’ in the states where the insurer was not licensed.
 Its credit rating may be inadequate to satisfy the insured’s minimum requirements;
for example because the insurer suffered a downgrade just prior to renewal.
If the fronting insurer has a higher credit rating than the assuming insurer, then the
assuming insurer will be able to attract and retain more business.
 There may be tax advantages in issuing the policy via the fronting insurer.
This could be the case if the fronting insurer operates in a different tax jurisdiction,
eg Bermuda (which has a favourable tax environment).

© IFE: 2019 Examinations The Actuarial Education Company


SP7-05: Reinsurance products – background Page 5

Although the fronting insurer is not generally concerned with the profitability of the
underlying business it does remain legally liable to the original insured. If the assuming
insurer fails to meet its obligations as part of the reinsurance agreement, then the fronting
insurer would still be liable for any claims incurring to the policy.

The fronting insurer should assess the credit risk of the assuming insurer before entering
into the fronting agreement. The fronting company may require the risk-bearing party to
provide some form of collateral to help mitigate the credit risk; for example a letter of credit.

Question

Suggest other factors the fronting insurer might consider when assessing credit risk.

Solution

A fronting insurer might also consider the following factors relating to the assuming insurer:
 size
 solvency level
 attitude / strength of management
 existence of a parent company
 types of business written
 level of expertise / experience with different classes of business.

The Glossary defines a letter of credit as:


A financial guarantee issued by a bank that permits the party to which it is issued to draw
funds from the bank in the event of a valid unpaid claim against another party.

In some jurisdictions, regulators do not look favourably on fronting agreements. Prevailing


regulation should be considered before entering into a fronting agreement.

Question

Explain how fronting differs from selling insurance business through an intermediary.

Solution

A fronting insurer remains legally liable to the original insured, however an intermediary does not.
An intermediary may also provide expertise to the seller of reinsurance.

The Actuarial Education Company © IFE: 2019 Examinations


Page 6 SP7-05: Reinsurance products - background

2.4 Captives
A captive insurance company may be established to self-insure insurance risk. Captive
insurance companies are generally allowed more flexibility than traditional insurers in the
creation of their reinsurance arrangements, taking advantage of local regulatory or tax
regimes.

Captives will be described in detail later in the course.

© IFE: 2019 Examinations The Actuarial Education Company


SP7-05: Reinsurance products – background Page 7

3 Reasons for purchasing reinsurance


We introduced reinsurance in Chapter 1 and mentioned a couple of reasons why insurers might
use reinsurance: to enable them to write large risks and to protect against accumulations. In this
section we will revisit these reasons and consider some additional reasons for using reinsurance.
Many of the ideas introduced in this chapter will be revisited later in the course in the chapter on
determining appropriate reinsurance.

This section mentions the different types of reinsurance, including facultative, proportional,
excess of loss (XL) and stop loss. You may have met some types of reinsurance (eg excess of loss
reinsurance) in your earlier studies, but if not, all will be revealed later in this chapter and in the
next chapter.

3.1 Why purchase reinsurance?


This section considers the following reasons for using reinsurance:
 limitation of exposure to risk or spreading of risk
 avoidance of large single losses
 smoothing of results
 increasing profitability
 improving solvency margin
 increasing capacity to accept risk
 financial assistance
 availability of expertise.

You may find it useful to come back and re-read this section after you have completed this
chapter and the next one.

3.2 Limitation of exposure to risk or spreading of risk (single risks, aggregations


of single risks, accumulations, multi-class losses)
An insurer with a portfolio of business faces an uncertain future. Even with the best
advanced statistical analysis and experienced underwriters, the claims outgo from any
period of risk cannot be predicted to any degree of accuracy. This is the reason for
insurance: to cover the unknown financial consequences of possible future perils.

In general, where an insurer fears a loss or combination of losses that would materially
(adversely) affect the results within its portfolio, it will consider reinsurance.

This adverse effect may be caused by a single event or from related events across
portfolios. It may arise, amongst other things, because an insurer is exposed to a particular
peril or has a concentration of risk in a particular territory.

These are the accumulations of risk described earlier.

The Actuarial Education Company © IFE: 2019 Examinations


Page 8 SP7-05: Reinsurance products - background

Accumulations may be:


 geographic, eg as a result of floods, hurricanes, earthquakes, volcanic activity
 by peril, eg asbestos-related claims from employers’ liability business.

Thus, the geographic location of a risk or group of risks is one important consideration in
reinsurance purchases. An insurer must control its aggregations of many individual risks
in one specific area or zone, especially if those risks are subject to natural peril events.

An insurer might also accept business from different sources.

Different sources might include fronting and reciprocal arrangements, which are discussed in the
next chapter.

Therefore, an insurer could have an accumulation of different interests on the same risk and
reinsurance offers protection against these eventualities.

The above considerations could apply to any line of business.

Factors affecting an insurer’s appetite to limit risk


An insurer’s appetite for offsetting its risks through reinsurance is influenced by the:

 size of the insurer

 insurer’s experience in the marketplace

Question

Describe how these factors are likely to affect the amount of reinsurance a general insurer uses.

Solution

Size of insurer

Larger insurance companies will generally have larger free reserves and a relatively more
diversified portfolio of business and hence will be less exposed to random fluctuation of claims
experience. As a result they may require less reinsurance.

However, it could be argued that larger companies are more likely to write types of business that
‘need’ reinsurance – ie those with large and/or unusual risks. Also, large companies may be more
prone to accumulations.

Experience in the marketplace

If a company has a lot of experience in the marketplace then it will hopefully have sufficient
credible data to be able to make an objective estimate of the expected claims outgo when setting
premium rates and reserves. It should also be able to set sensible policy conditions to help
control the risk. As a result the company will be less likely to use reinsurance to help with these
areas as well as generally limit the claims exposure.

© IFE: 2019 Examinations The Actuarial Education Company


SP7-05: Reinsurance products – background Page 9

 insurer’s available free assets


The larger a company’s available free assets, the bigger cushion it has to absorb claims
costs that are higher than expected. Hence the higher the free reserves, the less the need
for reinsurance.
 size of the insurer’s portfolio
Generally, everything else being equal, the larger the portfolio of business, the more
credible the past claims experience will be and the more predictable the business
outcome will be. Hence, the less the need for reinsurance.
 range within which the business outcome (or profit) can be forecast with confidence.

Be aware that we have made some very sweeping statements here. There are many factors
affecting the amount and type of reinsurance used and they should all be considered together.
An overriding factor in one case may be a minor factor in a different case. For example, a
company with relatively large free reserves may use more reinsurance than a company with
relatively low free reserves because it writes business with very volatile claims experience.

The factors affecting the amount of reinsurance used are covered in detail later in the course.

Cost and availability of reinsurance


Against the above factors, the cost and availability of reinsurance must be considered. For
some risks at certain phases of the insurance cycle or after particularly large losses (for
example, the terrorist attacks on New York and Washington on 11 September 2001),
reinsurance cover may be restricted or only available at unattractive terms. Reinsurers may
be reluctant to offer cover to an insurer because of its particular approach to underwriting
and claims containment.

Just like insurance companies, reinsurers are in the business to make money, usually for their
shareholders. They generally do not wish to write loss-making business. If there is great
uncertainty about the future claims cost then reinsurers may be inclined not to offer reinsurance
cover or at least charge much higher premiums than had previously been the case.

Question

Suggest when reinsurers might knowingly write loss-making business.

Solution

A reinsurance company may write loss-making business if it expects either to obtain


compensating higher future profits or profits from other connected business (from the same
cedant, or perhaps the same reinsurance broker).

At the bottom of the (re)insurance cycle, premiums across the whole market will be low and so in
order to retain market share reinsurance companies will be forced into accepting loss-making
business. Reinsurers will hope that premiums will soon start to increase and the business
becomes profitable once more.

The Actuarial Education Company © IFE: 2019 Examinations


Page 10 SP7-05: Reinsurance products - background

Also, it may write some products as a loss leader knowing that it will also be able to sell other
more profitable business on the back of the initial sales.

3.3 Avoidance of large single losses (for example, liability claim)


The size of a particular risk is very important. Reinsurance can help an insurer to reduce its
involvement in very large risks to containable levels.

What is large to an insurer will depend on the size of the free assets available. Many risks in
insurance have very high payout limits; some may even offer unlimited cover. Many small-
to medium-sized insurance companies will cede a top layer of potentially large payouts to
reinsurers as cover against this eventuality. This is especially true in liability lines of
business where excess of loss reinsurance is commonplace.

We will introduce excess of loss reinsurance in Section 5.2. It is described in more detail in the
next chapter.

A simple example of such use of reinsurance is for bodily injury claims on motor insurance.
Liability claims can be very large, running to millions of pounds, euros, dollars etc. Insurance
companies may want to limit or cap their exposure to such claims to help control the impact on
the free reserves or profits.

3.4 Smoothing of results


By reinsuring the larger risks or accumulation of smaller risks above certain limits, the
development of financial results (or profit) can be smoother year-on-year, especially when
the portfolio is relatively immature.

For a portfolio that is relatively immature, experience will be uncertain and potentially volatile.
Even for classes where claims are fairly stable and predictable, an immature portfolio may be
relatively small and so may lack diversification.

Reinsurance provides stability by reducing the potential for fluctuations or variations from
the planned result when losses are subject to variations in number and/or size. Purchasing
reinsurance will usually reduce profits, but it might also protect an insurer against severe
losses.

The insurer pays a premium to mitigate these fluctuations, and its net result is more
predictable. This predictability may be more acceptable to shareholders and regulators.
Stop loss is a form of reinsurance that could be used for these purposes.

Stop loss will be described in more detail in the next chapter, but as its name suggests it reduces
extreme losses directly and hence helps to smooth profits. However, stop loss isn’t the only type
of reinsurance that will smooth results. Nearly all types of reinsurance will smooth profits to
some degree. This might be achieved by simply limiting claim amounts (as mentioned above)
using excess of loss, or by enabling the insurer to write a larger number of smaller risks and hence
achieve a greater diversification of risk.

3.5 Increasing profitability


Reinsurance can increase the opportunity for an insurer to make a profit and plan its
business more accurately.

© IFE: 2019 Examinations The Actuarial Education Company


SP7-05: Reinsurance products – background Page 11

In the long term, reinsurance premiums are likely to be higher than the reinsurance recoveries
(payments from the reinsurer to the insurer in respect of claims) – the difference covering the
reinsurer’s expenses, profit margin etc. However, in any one year, the insurer’s profits might be
higher as a result of having reinsurance, eg in years where claims recoveries are high.

As discussed in the previous section, reinsurance should smooth results year-on-year by reducing
claim fluctuations. Increased certainty should enable the insurer to plan more effectively, which
may increase profits in the future.

For example, increased certainty may reduce the amount of capital the insurer needs to hold, and
so enable it to write a greater volume of profitable business. Similarly, increased certainty might
enable the insurer to adopt a riskier investment strategy, and increase the expected return on
investments.

3.6 Improving solvency margins


Solvency depends on the relationship between an insurer’s solvency margin and the total
premium income that insurers generate from all the lines of insurance business written.
Governments everywhere use financial regulation to protect consumers. Calculating an
insurer’s solvency margin is an important part of financial regulation and allowance is often
made for reinsurance in this calculation.

Reinsurance is an accepted management tool to improve an insurer’s solvency margin. It


helps an insurer to plan its operations more effectively and strengthen its balance sheet.

If we interpret ‘solvency margin’ as the excess of the value of the assets over the value of the
liabilities, then we can see that in order to improve the solvency position of the insurer, it is
necessary to either:
 increase the value of the assets
 decrease the value of the liabilities
 decrease the regulatory minimum difference between the assets and liabilities
 some (or all) of the above.

Reinsurance can help to achieve this.

Increasing the value of the assets


We shall see later that by obtaining certain types of reinsurance, the insurer may be able to
increase the value of the assets. This is described in Section 3.8.

Decreasing the value of the liabilities


By reinsuring the business, the insurer is reducing the value of its liabilities (as some if its liabilities
are ceded to the reinsurer). Therefore, as a result of the reinsurance, it will hold smaller reserves,
and so the solvency position of the insurer will improve, although of course assets will be reduced
by the size of the reinsurance premiums paid.

The Actuarial Education Company © IFE: 2019 Examinations


Page 12 SP7-05: Reinsurance products - background

Decreasing the regulatory minimum difference


The required solvency level is often calculated with reference to the proportion of business
reinsured. In other words, more reinsurance means a lower solvency requirement, and therefore
a stronger solvency position. However, this reduction may be subject to a limit, since there may
be a legal requirement for an insurance company’s free reserves to exceed a Required Minimum
Margin (RMM). In other words, you can only reduce the RMM so far and there will be a minimum
level that you still need to hold.

Question

Explain why the allowance for reinsurance might be limited.

Solution

The limit might exist to protect against the risk of reinsurer default. If the reinsurer was unable to
pay its claims then the insurance company would still be liable to pay the original claims to its
policyholders. Therefore, even if an insurance company reinsured all of its business, it would still
need to hold some free reserves in case things went wrong.

3.7 Increasing capacity to accept risk (that is, underwriting capacity: singly or
cumulatively)
Owing to insufficient capital backing, an insurer may be reluctant to accept, or be incapable
of accepting, particular risks by sector or by volume.

For example, an insurer may be unable to accept a risk that is larger than its financial
strength allows.

An insurer may also be reluctant to accept a particular risk if it would be exposed to an


accumulation of risk as a result.

In order to accept new business, the insurer must have adequate capital to set aside to cover the
risk. Larger risks will require more capital to be set aside. Therefore very large risks may require
more capital than an insurer has available, in which case the insurer would be unable to accept
the risk.

Reinsurance cover can prevent this situation from occurring. The solvency requirements for a
particular line of business are normally reduced in line with the proportion ceded, although this
may be subject to a limit.

Reinsurance can increase the size of an insurer’s underwriting capacity, allowing it to


compete more effectively in the market.

© IFE: 2019 Examinations The Actuarial Education Company


SP7-05: Reinsurance products – background Page 13

3.8 Financial assistance (new business strain, bolstering free assets,


merger / acquisition)
Reinsurance funds can assist financially with particular business propositions. Where a
particular distribution strategy would involve substantially more cash outflow in the initial
stages than premium income, reinsurance commission may be available in return for future
surplus streams. In effect, the reinsurer lends now (by paying commission) against the
predicted future flows of premiums less expenses and claims.

Proportional reinsurance allows this reimbursement. As companies grow and become


stronger, they tend to use more excess of loss and less proportional reinsurance.

Proportional reinsurance and excess of loss reinsurance are described later in this chapter. We
will also see that there are other types of reinsurance that can be used to improve the free asset
position that involve little or no risk transfer from the insurer to the reinsurer. Such contracts are
referred to as financial reinsurance or finite risk reinsurance and are described in the next
chapter.

An insurer could use a reinsurer as a partner to support its new operations. It can share
developmental and operational costs with the other party.

New business strain / financing projects


Here, the reinsurer is paying some money to the insurance company to improve its cash balance,
for example, to:
 enable it to write more business (ie to help meet new business strain)
 set up a new distribution channel or computer system (or both if using the internet).

This payment is effectively a loan but is ‘disguised’ as a commission payment for the reinsurance.
In return, the insurer will repay the loan out of the profits it makes on the underlying business.

This type of arrangement may be more attractive to the insurer than, say, a simple bank loan,
because the repayments will only be made if the company does well and profits are made. There
is also an accounting advantage, since the repayments are contingent on profits arising so that
such reinsurance arrangements do not appear as debt on the balance sheet, whereas a bank loan
does.

Bolstering free assets


Where a block of renewing business which is producing regular profits is identified, capital
can be found for the insurer to bolster the free asset position by reinsuring this portfolio of
profitable policies. The reinsurer would pay an initial commission in return for which the
reinsurer would be entitled to the future surplus of premiums over claims for as long as the
arrangement remained in place.

The arrangement mentioned here is identical to that described above except that it’s for a
different reason. The reinsurer still loans some money to the insurer and the loan is repaid out of
future profits. Because the insurer has no liability to repay the loan unless a surplus has been
made, it does not have to reserve for the future payments. So it has increased its assets by the
amount of the loan but not increased its liabilities and hence has improved its free asset position.

The Actuarial Education Company © IFE: 2019 Examinations


Page 14 SP7-05: Reinsurance products - background

However, the extent to which this is possible will depend on the precise requirements of the
supervisory regime concerned.

Merger / acquisition
In a similar way, reinsurance can be used to facilitate the acquisition of an insurance
company. In this case, a profitable subset of business (or all policies) is identified within
the company being acquired so that the reinsurer is prepared to advance funds in
anticipation of future surplus.

In these last two cases, quota share reinsurance is the norm, though any form of
proportional reinsurance might be used.

3.9 Availability of expertise to develop new markets and products


International reinsurers often have considerable knowledge and can provide technical
services and expertise in many insurance products in different countries around the world.

Sometimes an insurer wishes to enter a new insurance market at home or overseas. It


could use a reinsurer’s knowledge in the development and operation of any new insurance
products.

In addition, if an insurer is considering underwriting new risks, unusual risks, or risks in


new territories, the reinsurance broker can provide assistance.

Often it is the broker who is the first port of call for reinsurance expertise.

For example, when an insurer adopts a strategy that will take it into new risk areas where it
has little previous experience, the reinsurance broker can sometimes help with rating,
underwriting and claims management.

When monitoring the policy content, pricing, marketing, sales, sources of acquisition and
results of any new insurance product, it can use valuable knowledge to help future
development of its business.

In each of these cases, quota share reinsurance is usual, though any form of proportional
reinsurance might be used.

3.10 Reasons for retrocession

Question

Without reading on, suggest which two of the reasons for purchasing reinsurance are most
relevant in the context of retrocession.

Solution

The most relevant reasons are:


 limitation of exposure to risk (single risks, aggregations of single risks, accumulations,
multi-class losses)
 increasing capacity to accept risk (singly or cumulatively).

© IFE: 2019 Examinations The Actuarial Education Company


SP7-05: Reinsurance products – background Page 15

Since reinsurers tend to be larger than insurers:


 they have less need to avoid large single losses completely
 their business is likely to be more diversified, leading to smoother results
 their solvency margin may already be adequate – in fact, it may be subject to different
regulations, so that solvency requirements are not as onerous
 they are less likely to need financial assistance.

They are also likely to have a fair amount of their own expertise.

The retrocedant purchases the retrocession to gain additional capacity or to contain or


reduce its risk of loss on a specific or aggregate basis. In particular, a reinsurer would
require a retrocession protection if it has limited capital and is heavily exposed to specific
geographical regions and/or classes of business.

Typically, retrocession protections are written on an excess of loss basis. Retrocession


protections can protect the retrocedant’s facultative, proportional or excess of loss
business, or a combination of the three, which may lead to spiral of cover (where a chain of
retrocession agreements leads to the original retrocedant covering itself).

A spiral of cover can occur when reinsurers write retrocession business on risks that they already
have exposure to through other reinsurance arrangements. They can therefore (sometimes
unknowingly) end up reinsuring themselves.

Reinsurance spirals will be discussed further later in the course.

The Actuarial Education Company © IFE: 2019 Examinations


Page 16 SP7-05: Reinsurance products - background

4 Ways of writing reinsurance business


How are insured risks reinsured?

 individually, (that is, facultative reinsurance)

 groups of similar risks, (that is, treaty reinsurance).

Insurers cede risks either facultatively or by treaty according to how they weigh up the
advantages and disadvantages of each arrangement.

4.1 Facultative reinsurance


Facultative reinsurance is the reinsurance of a single risk.

When each individual risk on which reinsurance is required is offered separately to a


reinsurer, the risk is said to be offered facultatively. There is no obligation for the ceding
company to offer the business, nor is the reinsurer obliged to accept it. Each case is
considered on its own merits and the reinsurer is free to quote whatever terms and
conditions it sees fit to impose for that risk.

For example, the direct writer may be asked to provide cover for a large risk. While negotiating
terms with the potential policyholder, the direct writer will seek out a willing reinsurer that is
prepared to accept a portion of the risk on the best terms.

Advantages and disadvantages


The main advantage of facultative reinsurance is the flexibility that both parties have within the
process. For example, the direct writer is under no obligation to use a particular reinsurer. The
direct writer can approach several reinsurers in search of the best terms for each risk individually.
Similarly, the reinsurer is under no obligation to accept risks.

The main disadvantages to the insurer of facultative reinsurance include:

 it is a time-consuming and costly exercise to place such risks

 there is no certainty that the required cover will be available when needed

 even if cover is available, the price and terms may be unacceptable

 the primary insurer may be unable to accept a large risk until it has been able to find
the required reinsurance cover. This means the insurer cannot accept business
automatically when it is offered, and consequently its standing in the market may be
reduced.

This form of reinsurance placing is therefore usually limited to large one-off risks, or to
risks that fall outside the scope of any treaty reinsurance arrangements available to the
primary insurer.

© IFE: 2019 Examinations The Actuarial Education Company


SP7-05: Reinsurance products – background Page 17

4.2 Treaty reinsurance


Arranging reinsurance for each individual risk is administratively messy. Therefore direct writers
may instead set up treaties with reinsurers. This allows them to place business with the reinsurer
automatically. The terms and conditions of the treaty are carefully laid down so that both parties
know exactly where they stand. An insurer will also wish to use reinsurance to control its
solvency and growth requirements. This can be done more readily by treaty than by facultative
reinsurance.

Treaty reinsurance is the reinsurance of a group of similar risks under one reinsurance
agreement.

Treaties are usually arranged so that the ceding insurer is obliged under the terms of the
treaty to pass on some of the risk in a defined manner and the reinsurer is obliged to accept
it.

This is known as an obligatory / obligatory (oblig / oblig) basis: in other words, the insurer is
obliged to pass the risk on and the reinsurer is obliged to accept it. Such an arrangement is
common in quota share treaties, which are covered in the next chapter.

Occasionally, treaties may be written on a facultative / obligatory basis. Under such an


arrangement, for each risk, the insurer has the choice of whether to include it in the treaty,
but the reinsurer is obliged to accept all the requested risks. Such treaties are normally
associated with reciprocal arrangements, whereby each insurer reinsures a block of
business with the other.

Here, the direct writer is under no obligation to use a particular reinsurer to cover the risk, but the
reinsurer has to accept the risk if offered it, subject to the terms of the treaty. Potentially, with
fac / oblig treaties there is scope for a direct writer to select against a reinsurer because the direct
writer can pick and choose which risks to keep and which to reinsure. Such an arrangement is
common in surplus treaties, which are covered in the next chapter.

Question

Explain what is likely to happen if the direct writer manages to select against the reinsurer so that
the reinsurer’s experience is much worse than the direct writer’s experience.

Solution

The reinsurer is likely to increase premiums or change the terms of the treaty in favour of the
reinsurer at the next renewal date of the treaty.

The reinsurer may even decide not to reinsure the direct writer again.

Advantages and disadvantages


A treaty does not have the disadvantages mentioned above for facultative reinsurance and
insurers therefore tend to place the bulk of reinsurance cover on a treaty basis.

The Actuarial Education Company © IFE: 2019 Examinations


Page 18 SP7-05: Reinsurance products - background

The features of treaties are very much the reverse of the features of facultative reinsurance:

Efficient: Risks are reinsured automatically. This is administratively quicker and cheaper.

Certain: With a treaty, the direct writer knows that reinsurance is available (if the risk falls
within the limits of the treaty) and on what terms.

Inflexible: Once the treaty is set up, then both parties must operate within the terms of the
treaty (so each should ensure it is happy with the terms before it signs the treaty).

Terms within the treaty


It is essential for both the direct writer and the reinsurer that the treaty is absolutely precise in its
definition of:
 what is and what is not covered
 the financial arrangements (ie premiums, commissions, timing of payments)
 the obligations of both parties.

The treaty document sets out all the relevant details and obligations under the arrangement,
though the wording differs for different types of reinsurance. There will also be wide
differences in wording between treaties in any one category.

Question

List 20 items that might be included in a reinsurance treaty.

Solution

Depending on the type of reinsurance contract, the details set out in the treaty will include some
of the following:
 names of the parties to the treaty
 period of cover
 territorial limits
 class(es) of business covered
 exclusions to the cover
 definition of loss occurrence
 retention of the ceding company
 cover granted automatically by the reinsurer
 reinstatement provisions
 a stability clause
 premium rate
 premium payment arrangements

© IFE: 2019 Examinations The Actuarial Education Company


SP7-05: Reinsurance products – background Page 19

 ceding commissions payable


 profit commission payable to the direct writer and the method of calculation
 brokerage payable to reinsurance brokers
 claim notification arrangements
 claim payment arrangements, including special arrangements for large claims
 rendering and settlement of accounts
 a currency clause (if more than one currency is involved)
 access by the reinsurer to risk details
 terms for termination of the treaty (period of notice, etc)
 sunset clause (specifies the discovery period)
 an arbitration clause, in case of disagreements arising.

Check the Glossary if you are not familiar with any of the terms above.

The Actuarial Education Company © IFE: 2019 Examinations


Page 20 SP7-05: Reinsurance products - background

5 Traditional reinsurance – an overview


So far we have discussed facultative and treaty methods of reinsurance, ie the methods by which
reinsurers agree to accept risks. We have not discussed the mechanisms by which the premium
and claims payments under a reinsurance contract are passed to the reinsurer. These are now
discussed below.

There are two main methods of reinsurance:

 proportional – whereby the direct writer cedes a proportion of the risk, passing on a
proportion of the premium, and the reinsurer pays that proportion of the claims
 non-proportional – whereby the direct writer pays the reinsurer an agreed premium, and
in return the reinsurer pays out for claims between specified limits.

In each case, the reinsurer will usually obtain business through specialist brokers who will
receive commission for placing the business.

This commission is paid by the reinsurer and is usually called brokerage.

5.1 Proportional reinsurance


Under proportional reinsurance, the reinsurer covers an agreed proportion of each risk and
the reinsurance premium is proportional to this risk ceded.

The proportion may be constant for all risks covered (that is, quota share) or may, to some
extent, be at the discretion of the ceding insurer (that is, surplus).

Quota share and surplus reinsurance are discussed in detail in the next chapter.

Proportional reinsurance reduces the size of the ceding insurer’s net account, and so it can
be used as a means of accepting a larger size of risk than would otherwise be possible.

By purchasing proportional reinsurance, the insurer can retain a smaller share of a larger
number of risks and hence increase the diversification within its portfolio, reducing the
volatility of the overall result.

5.2 Non-proportional reinsurance


Although proportional reinsurance can be used to spread risk and to reduce pro rata the
size of risk retained, it does not cap the cost of very large claims that occur.

Under non-proportional reinsurance, the reinsurer covers the loss suffered by the insurer
that exceeds a certain amount, called the attachment point, retention or priority up to the
policy limit.

Under excess of loss (XL) reinsurance, the cost to an insurer of a large claim is capped with
the liability above a certain level being passed to a reinsurer. However, if the claim amount
exceeds the upper limit of the reinsurance, the excess will revert back to the insurer.
Variations of this form of reinsurance cover exist to limit an insurer’s loss from a single
event or over a given period.

Non-proportional types of reinsurance include risk excess of loss, stop loss or aggregate
excess of loss reinsurance.

© IFE: 2019 Examinations The Actuarial Education Company


SP7-05: Reinsurance products – background Page 21

With non-proportional reinsurance:


 The limit might operate on individual claims (eg to cover against large individual risks), or
on aggregations of claims (eg to cover against widespread damage from one event).
 There might also be an upper limit, above which the reinsurer’s liability ends.
 The reinsurer might pay for all claims within the limits or perhaps a proportion of the
claims within the limits (eg 90%). This could also be expressed as having a deductible of
10%.
 The limits might be linked to inflation.

Question

Suggest classes of business for which non-proportional reinsurance is likely to be particularly


useful.

Solution

Classes where claims are potentially very large or even unlimited. Claims from liability classes of
business fall into this category.

5.3 Different reinsurance bases


The natural basis for writing proportional reinsurance policies is to provide cover for those
policies written by the primary insurer during the term of the reinsurance contract (that is, a
policies-incepting basis). Claims-made policies and losses-occurring policies may also be
written, although the latter in particular are more commonly used for non-proportional
reinsurance.

Claims-made and losses-occurring policies were discussed in terms of insurance bases in


Chapter 2.

Policies-incepting basis
With this type of arrangement the reinsurer provides cover to the direct writer for the claims
arising from all policies written under the treaty over a period, ie corresponding to an
‘underwriting-period’ cohort. This is the natural arrangement with all proportional types of
reinsurance: all policies written during the same period will be subject to the same proportion
ceded.

This is also known as a risks-attaching basis, or a risks attaching during (RAD) basis.

The Actuarial Education Company © IFE: 2019 Examinations


Page 22 SP7-05: Reinsurance products - background

Question

Explain why proportional reinsurance is likely to be arranged on a policies-incepting basis.

Solution

With proportional reinsurance the direct writer cedes a specified portion of a risk to the reinsurer.
It is natural that they provide cover in the defined proportions throughout the duration of the
risk. That way both the division of cover into different accounting periods and the dates when
any claims are reported become irrelevant.

Losses-occurring basis
This basis provides the direct writer with cover for any claim incident(s) under the treaty occurring
within a defined period, ie corresponding to an ‘accident-period’ cohort. This is commonly used
with non-proportional types of reinsurance: any claims occurring within a certain period will be
subject to the same retention level and upper limit.

Claims-made basis
This basis would provide the direct writer with cover for any claims under the treaty reported to
them within a defined period, ie corresponding to a ‘reporting-period’ cohort.

Question

Suggest the main issue that the reinsurer should consider before providing cover on a
claims-made basis.

Solution

Unexpectedly large numbers or amounts of claims may be reported in a given period. This could
be as a result of latent types of claim, eg asbestos-related claims and industrial deafness claims,
which have come to light some time after the claim incidents occurred.

© IFE: 2019 Examinations The Actuarial Education Company


SP7-05: Reinsurance products – background Page 23

6 Other types and alternatives to reinsurance – an overview


Other reinsurance products include:
 finite risk reinsurance
 run-off reinsurance.

Capital market products may also be available as an alternative to reinsurance.

These are discussed in detail in the next chapter.

The Actuarial Education Company © IFE: 2019 Examinations


Page 24 SP7-05: Reinsurance products - background

7 Glossary items
Having studied this chapter you should now read the following Glossary items:
 Broker
 Captive
 Ceding company (cedant)
 Co-reinsurance
 Direct business
 Excess of loss (XL or XOL) reinsurance
 Facultative-obligatory reinsurance
 Facultative reinsurance
 Fronting
 Inwards reinsurance
 Letter of credit
 Net premium
 Non-proportional reinsurance
 Outwards reinsurance
 Primary insurer
 Proportional reinsurance
 Reciprocity
 Retrocession
 Retrocessionaire
 Treaty reinsurance.

© IFE: 2019 Examinations The Actuarial Education Company


SP7-05: Reinsurance products – background Page 25

Chapter 5 Summary
Reinsurance
Reinsurance is a means by which an insurance company can protect itself against the risk of
losses by ceding the risk to other companies (reinsurers).

Retrocession is a means by which a reinsurance company can protect itself against the risk of
losses by ceding risks to other reinsurance companies (retrocessionaires).

Participants in the reinsurance market


Reinsurance business is often sold by reinsurance intermediaries, ie brokers. Their role is
primarily a sales role, using their specialist knowledge and their contacts in order to get the
best reinsurance price for their clients. They may also provide expertise.

An alternative to reinsurance intermediaries is professional reinsurers dealing directly with


buyers.

Fronting occurs when an insurer underwrites a risk and cedes it all (or nearly all) to another
insurer. The fronting insurer will receive a fee.

A captive insurance company may be established to self-insure risk. This often has
regulatory and tax advantages.

Reasons for reinsurance


The main reasons for reinsurance are:
 limitation of exposure to risk or spreading of risk – the need for this will depend on
the size of the insurer, its portfolio, its level of free assets, its experience and the
type(s) of business written
 avoidance of large single losses (eg liability claims) – what is ‘large’ will depend on
the size of the insurer and the free assets available
 smoothing of results – reducing claim fluctuations
 increasing profitability – reinsurance can increase the opportunity for an insurer to
make a profit and plan its business more accurately
 improving solvency margin – reinsurance helps an insurer to plan its operations more
effectively and strengthen its balance sheet
 increasing capacity to accept risk (singly or cumulatively) – this can enable the
insurer to compete more effectively in the market
 financial assistance (new business strain, bolstering free assets, merger / acquisition)
– the reinsurer can lend the insurer cash now, which will be repaid out of the
insurer’s future profits
 availability of expertise to develop new markets and products – expertise may be
needed if the insurer is entering a new market or writing new risks, unusual risks, or
in new territories.

The Actuarial Education Company © IFE: 2019 Examinations


Page 26 SP7-05: Reinsurance products - background

Chapter 5 Summary continued


The benefits provided by reinsurance must be weighed up against the cost and availability of
the reinsurance.

The reasons for retrocession are to gain additional capacity or to contain or reduce risk of
loss (singly or cumulatively).

Ways of writing reinsurance business


Facultative reinsurance is the reinsurance of a single risk. There is no obligation for the
ceding company to offer the business and no obligation for the reinsurer to accept it. It is a
flexible way of writing reinsurance business, but is time consuming and uncertain in terms of
the availability of reinsurance and price.

Treaty reinsurance is the reinsurance of a group of similar risks under one reinsurance
arrangement. The reinsurer is obliged to accept risks that fall under the terms of the treaty.
The treaty document sets out all relevant details and obligations.

Traditional reinsurance
Reinsurance may be:
 proportional – the reinsurer covers an agreed proportion of each risk and the
reinsurance premium is proportional to the risk ceded
 non-proportional – the reinsurer covers the loss suffered above the insurer’s
retention, possibly subject to an upper limit.

Proportional reinsurance is usually written on a policies-incepting basis. Non-proportional


reinsurance is often written on a losses-occurring basis. A third option is a claims-made
basis.

© IFE: 2019 Examinations The Actuarial Education Company


SP7-05: Reinsurance products – background Page 27

Chapter 5 Practice Questions


5.1 List the reasons why a general insurance company might want to use reinsurance. [12]
Exam style

The Actuarial Education Company © IFE: 2019 Examinations


Page 28 SP7-05: Reinsurance products - background

The solutions start on the next page so that you can


separate the questions and solutions.

© IFE: 2019 Examinations The Actuarial Education Company


SP7-05: Reinsurance products – background Page 29

Chapter 5 Solutions
5.1 Reinsurance may be used by an insurer:
 to limit its exposure to risk (or spread risk) in respect of: [½]
– single risks [½]
– aggregations of single risks [½]
– accumulations [½]
– multi-class losses [½]
 to obtain additional business through reciprocity [½]
 to avoid single large losses in respect of: [½]
– single large claims [½]
– catastrophes [½]
 to smooth its results [½]
 to increase profitability by: [½]
– increasing the stability of results (and so the ability to plan) [½]
– taking advantage of cheap reinsurance [½]
 to enable it to declare profits from outstanding liabilities more quickly [½]
 to improve the solvency margin and hence reduce the risk of insolvency [½]
 to increase the capacity to accept risk, either: [½]
– singly, ie to enable it to write larger risks, or [½]
– cumulatively, ie to enable it to write more business [½]
 to obtain financial assistance to help with: [½]
– new business strain [½]
– bolstering free assets [½]
– a merger / acquisition [½]
– any other short-term cashflow needs [½]
 to get technical assistance on: [½]
– new risks [½]
– unusual risks [½]
– risks in new territories [½]
 as a supervisory condition. [½]
[Maximum 12]

The Actuarial Education Company © IFE: 2019 Examinations


All study material produced by ActEd is copyright and is sold
for the exclusive use of the purchaser. The copyright is
owned by Institute and Faculty Education Limited, a
subsidiary of the Institute and Faculty of Actuaries.

Unless prior authority is granted by ActEd, you may not hire


out, lend, give out, sell, store or transmit electronically or
photocopy any part of the study material.

You must take care of your study material to ensure that it


is not used or copied by anybody else.

Legal action will be taken if these terms are infringed. In


addition, we may seek to take disciplinary action through
the profession or through your employer.

These conditions remain in force after you have finished


using the course.

The Actuarial Education Company © IFE: 2019 Examinations


SP7-06: Reinsurance products – types Page 1

Reinsurance products – types


Syllabus objectives
1.2 Describe the main types of general reinsurance products and the purposes for which
they may be used.

Covered in part in this chapter.

The Actuarial Education Company © IFE: 2019 Examinations


Page 2 SP7-06: Reinsurance products – types

0 Introduction
In this chapter, we look in detail at the different types of reinsurance product.

We start by looking at traditional reinsurance products. Proportional and non-proportional


reinsurance were introduced in the last chapter.

Section 1: Quota share reinsurance (proportional)

Section 2: Surplus reinsurance (proportional)

Section 3: Excess of loss reinsurance (non-proportional)

Section 4: Stop loss (non-proportional)

We then go on to discuss other types of reinsurance products, which have been developed to
meet specific needs of insurers.

Section 5: Finite risk (or financial) reinsurance

Section 6: Run-off reinsurance

Reinsurers may not have the capacity to meet all an insurer’s needs in terms of limiting exposure
to risk and providing financial assistance, therefore the insurer may have to seek assistance
elsewhere. We conclude the chapter by looking at some alternatives to reinsurance, available
from the capital markets.

Section 7: Capital market products

© IFE: 2019 Examinations The Actuarial Education Company


SP7-06: Reinsurance products – types Page 3

1 Quota share reinsurance

1.1 How quota share operates


Quota share reinsurance is a form of proportional reinsurance where the apportionment of
claims and premiums between the insurer and reinsurer is constant for all risks covered by
the treaty.

Suppose the direct writer decides that for a particular class it does not want to retain all the
business it writes. So the direct writer enters into a quota share treaty with a reinsurer. Then a
constant proportion of each and every risk within the scope of the treaty is automatically passed
to the reinsurer. The treaty will specify the proportion to be ceded to the reinsurer, R% say. This
is often referred to as an R% quota share treaty.

However, in some markets, there is extensive use of quota share reinsurance to cover
single risks on a fully proportional but facultative basis.

In other words, quota share can either be written by treaty covering a number of risks or on a
facultative basis covering a single risk.

Premiums
The direct writer passes to the reinsurer R% of the policyholders’ premiums for risks covered
within the treaty. (In general insurance, under proportional reinsurance everything is shared with
the reinsurer in the specified proportion. Note that this is not always the case for life and health
products, where the premium may not be split in the same proportion to the claims.)

Claims
The direct writer recovers from the reinsurer R% of the claims from risks covered by the treaty.

Commission
The direct writer (the cedant in the reinsurance arrangement) will have suffered expenses in
order to sell the original policies (for example, commission to brokers). The reinsurer will
reimburse the direct writer with some percentage of the premium to help cover the acquisition
expenses. This payment is often called return commission.

For example, consider a policy with an original insurance premium of £100. If a 40% quota share
treaty is in place with a return commission of 4%, then the return commission (paid from the
reinsurer to the insurer) is £1.60 (4% of 40% of £100).

To compensate the direct writer for the extra work that it will have carried out in attracting and
administering the business, the reinsurer may pay a further commission payment to the direct
writer. The commission paid over and above the return commission is sometimes called override
commission. The sum of the return and override commission is often called the ceding
commission (although sometimes the return commission alone is called the ceding commission).

To add to the confusion, the Glossary states that sometimes the return commission is called the
override commission. Whichever terminology you use in the exam, be sure to define it.

The Actuarial Education Company © IFE: 2019 Examinations


Page 4 SP7-06: Reinsurance products – types

In practice, the ceding commission is rarely split up between return and override commission but
hopefully the total will be sufficient recompense for the expenses incurred by the insurer.
However, market forces (eg competition from other reinsurers) and the profitability of the
reinsurance contract will be the most important factors that influence the total amount of
commission that is paid.

There might also be a form of profit commission, which the reinsurer pays the direct writer as a
reward for passing on good business. This would be calculated (on a pre-agreed basis) at the end
of the year or end of the treaty when the claims experience is known.

In addition, brokerage would usually be paid out by the reinsurer if the reinsurance has been
arranged through a reinsurance broker.

Question

A direct writer has a 50% quota share treaty with a reinsurer.

For a risk with a gross premium of $10,000, the direct writer pays commission of 15% and receives
15% return commission from the reinsurer. The reinsurer also pays 5% override commission.

Calculate how much premium is received by the direct writer and the reinsurer after allowing for
commissions. Comment on whether these relative levels of premium look fair.

If there is subsequently a claim for $50,000, calculate how much the direct writer will be able to
reclaim from the reinsurer.

Solution

The first split of the premium is $5,000 to each of the direct writer and the quota share reinsurer.
But the direct writer pays commission of $1,500 (ie 15% of $10,000) and receives $1,000
(ie 20% of $5,000) from the reinsurer.

This leaves the net of commission premiums as $4,500 for the direct writer and $4,000 for the
quota share reinsurer.

This split doesn’t look unfair. You would expect the direct writer to have a bigger share to
compensate for the initial work.

The direct writer would be able to reclaim $25,000 from the reinsurer on a claim of $50,000.

Practical considerations
The administration of quota share is very straightforward.

Because each risk has the same proportion reinsured, there is no need for anything more than a
list of all the risks with total premiums and total claims. The premiums and claims payable
between the direct writer and the reinsurer are a straight proportion of the totals.

© IFE: 2019 Examinations The Actuarial Education Company


SP7-06: Reinsurance products – types Page 5

1.2 Cedant’s claims experience


The cedant’s loss ratio for the account will be approximately the same before and after
reinsurance, since the premiums and claims are ceded in the same proportion, depending
on the treatment of commissions.

Note that the commissions paid by the reinsurer to the insurer can vary depending on, for
example, market practice and the relationship between the reinsurer and insurer. Therefore, the
reinsurer’s loss ratio can be different from the insurer’s loss ratio. However, it is common
practice to ignore commission (reinsurance or otherwise) when calculating loss ratios, in which
case the loss ratios would be identical.

1.3 Reinsurer’s claims experience


The cedant and reinsurer will have proportionately the same overall underwriting
experience on the business included in the treaty, apart from differences in expenses and
commissions. The reinsurer will therefore be concerned at the outset to establish:

 the nature of the business being offered

 the cedant’s attitude to underwriting and claims settlement

 any previous experience of this business.

If the reinsurer is unhappy with any of these aspects, it can refuse to participate in the treaty
or ensure that the level of commission written into the treaty is low in expectation of poor
experience. On occasions, however the reinsurer may reserve the right in the treaty to be
involved in the approval and settlement process for claims above a certain size. The
reinsurer might also negotiate for part of the commission payment to the cedant to be a
profit commission that is payable only if the business ceded meets specified profitability
criteria.

Since the capacity of the reinsurer to write such business may be limited, the treaty may
specify a limit on the amount of business that may be ceded to the treaty. This limit will
normally be expressed in terms of the original gross premium income of the cedant for that
business.

1.4 Advantages of quota share


The main advantages of quota share to the direct writer are that:
 It spreads risk, enabling insurers to write larger portfolios of risk and encourage
reciprocal business.
At one level, the principle is very simple. An insurer’s risks are better spread by retaining
50% of 1,000 risks than 100% of 500 risks.
Quota share can also help spread risks through reciprocity. This is where two insurers
agree to swap some business on a reciprocating basis.
For example, Company A may have a 40% quota share treaty with Company B for business
it writes directly, while Company B has a similar quota share treaty with Company A.
Provided the two portfolios are not perfectly correlated, both companies will have
achieved a better spread of risks. An example of this would be two household insurers,
covering different geographic areas, spreading risk to limit the impact of storms.

The Actuarial Education Company © IFE: 2019 Examinations


Page 6 SP7-06: Reinsurance products – types

A large, established insurer will be far less dependent (if at all) on quota share as a
means of spreading risk than a small, new insurer. However, large insurers often
employ quota share extensively for marine and some aviation business to spread
the risk or reciprocate business in smaller classes.

 It directly improves the solvency ratio and helps the insurer to satisfy the statutory
solvency requirement.
The solvency ratio is normally defined as free assets divided by net written premiums.
This will be looked at in more detail later in the course. Using quota share will reduce net
written premiums and hence increase the solvency ratio in the short term.
Many statutory solvency requirements are based on net written premiums. The
legislation may require that the company has a solvency margin at least as great as a
defined percentage of net written premium. Quota share can be used to directly reduce
this type of solvency margin requirement.

In addition:
 it is administratively simple

 the commission may help with cashflow.

1.5 Disadvantages of quota share


The main disadvantages to an insurer of ceding business by quota share are that:
 It cedes the same proportion of low-variance and high-variance risks.
It is often totally inflexible, since a set percentage of all risks must be passed to the
reinsurer, whether or not the direct writer wants to cede the particular risk.
 It cedes the same proportion of each risk, irrespective of size. The insurer may,
however, wish to cede a greater proportion of the larger risks than the smaller ones,
owing to their greater loss potential.
It is not a good way of protecting against catastrophes or very large claims. Note that it
does not cap very large claims, since insurers are still liable for a percentage of that very
large claim.
 It passes a share of any profit to the reinsurer.

© IFE: 2019 Examinations The Actuarial Education Company


SP7-06: Reinsurance products – types Page 7

2 Surplus reinsurance

2.1 How surplus operates


Surplus reinsurance is a form of proportional reinsurance where the proportions are
determined by the cedant for each individual risk covered by the treaty, subject to
restrictions defined in the treaty.

The mechanics for operating surplus can at first be a bit confusing. However, if you take a
common sense approach, then it becomes much clearer. In the following explanation, the
principles are described without using the usual terminology for surplus reinsurance. We will
introduce these afterwards.

‘Common sense’ approach


Let’s start by imagining that there is a direct writer who specialises in industrial fire risks. The
company’s size and free reserves are such that risks greater than £200,000 are too big. A problem
faced by the company is that the number of buildings the company can insure is severely limited.
Without any form of reinsurance, the company would have to decline all risks over £200,000.

So it sets up a surplus reinsurance treaty which allows it to cede to a reinsurer the share of the
risk that it doesn’t want to keep.

Suppose that it insures a property with a maximum loss of £400,000. The direct writer might
choose to keep only 50% (so that the retained risk is limited to £200,000) and 50% of the risk is
ceded to the reinsurer. For this particular risk, the direct writer passes on 50% of the premium.
The reinsurer is then obliged to cover 50% of all claims from this risk.

The next property has a maximum loss of £1,000,000. The direct writer passes on 80% of the risk
to the surplus reinsurer, retaining 20%. In this way, the direct writer has effectively engineered
another property risk of size £200,000.

Note that the insurer is not paying all claims up to £200,000 and passing the excess to the
reinsurer  since that would be excess of loss reinsurance. It is instead using the £200,000 to
determine the proportion of risk it wishes to keep. So in the second example, the reinsurer would
pay 80% of all claims, whatever their size.

For some risks, the insurer may choose to keep less than the maximum retention. With the
maximum loss of £1,000,000, it might choose to retain only 10% of claims and engineer the risk
down to £100,000. The retention does not have to be the same for all risks  which differs from
the surplus reinsurance that you may have met in life reinsurance.

The Actuarial Education Company © IFE: 2019 Examinations


Page 8 SP7-06: Reinsurance products – types

Question

A direct writer has a limit of £50,000 exposure to any one risk. Assuming there is surplus
reinsurance that will absorb all the risks not wanted by the direct writer, and that the direct writer
retains the maximum for each risk, calculate the premiums due to and the claims due from the
reinsurer for the following risks:

Risk Size Gross premium Claim information

1 £300,000 £4,500 no claims

2 £450,000 £6,000 one claim for £80,000

3 £60,000 £1,500 no claims

4 £40,000 £700 no claims

5 £1,000,000 £12,000 no claims

6 £500,000 £7,200 one claim for £50,000

Solution

Direct Writer Reinsurer


Size Premium Claim Premium Claim
£300,000 £750 – £3,750 –
£450,000 £667 £8,889 £5,333 £71,111
£60,000 £1,250 – £250 –
£40,000 £700 – – –
£1,000,000 £600 – £11,400 –
£500,000 £720 £5,000 £6,480 £45,000

If you got the right answer to the last question, you probably understand the principles of surplus
reinsurance. Let’s now define surplus reinsurance using the correct terminology.

Measuring the risk: estimated / expected maximum loss (EML)


We have used the expression ‘size’ in the previous examples when describing different risks. But
how would size be measured?

It could be sum insured. However, this is not necessarily the basis a company would use when
deciding on whether a risk is too large. Consider two different risks:
Risk 1: 30 individual buildings spread around a site. Total sum insured of £15m.
Risk 2: One large office block. Total sum insured of £10m.

© IFE: 2019 Examinations The Actuarial Education Company


SP7-06: Reinsurance products – types Page 9

A single insurer could conceivably insure all 30 buildings, but decline the office block on the
grounds that it was too big. So how can £10m be ‘bigger’ than £15m? The office block is bigger in
that there is a much more concentrated risk. The possible loss from one incident with Risk 2 may
be much greater than the possible loss from one incident involving Risk 1, which might typically
involve a loss of about £½m.

For the purpose of determining the proportion of each risk to be retained or ceded, the
ceding insurer needs to have a satisfactory measure of risk. For most classes this can
simply be the sum insured specified in the original policy. However, for certain classes
(and in particular for commercial fire), the full sum insured is very unlikely to be paid out in
the event of a claim under a large risk, especially if the sum insured is spread over several
sites. For such classes the EML is used as the measure of the risk.

Definition – Estimated maximum loss (EML)

The ‘estimated maximum loss’ is the largest loss that is reasonably expected to arise from a
single risk.

The EML will be based on the cedant’s view as to the maximum loss that could arise from a single
event (eg a fire or explosion). The EML could be considerably less than the sum insured for
policies that cover several buildings. Even for a single tower block, the EML might be based on
the value of several floors rather than the cost of rebuilding the whole block. This will depend on
the capability of the fire precautions, such as sprinkler systems.

Defining the limits


The treaty will specify the limits for any risk to be included in the treaty. In particular, it might
specify the:
 insurer’s minimum retention
 insurer’s maximum retention
 reinsurer’s maximum level of cover.

Question

Explain why a reinsurer might require an insurer to have a:


 minimum retention level
 maximum retention level.

Solution

The reinsurer might require a minimum retention level in order to prevent the insurer from
having too little interest in the risk. Having too little interest in the risk could lead to poor
underwriting or claims management with respect to that risk.

The reinsurer might require a maximum retention level (in percentage terms) in order to prevent
very small proportions of individual risks being passed on, as these are likely to be
administratively expensive (compared to the size of the risk).

The Actuarial Education Company © IFE: 2019 Examinations


Page 10 SP7-06: Reinsurance products – types

Minimum and maximum retentions also help the reinsurer to avoid anti-selection by the cedant.
Without them, the cedant could retain very high proportions of the risks it believes to be ‘good’
and very low proportions of those it believes to be ‘bad’.

Definition – Minimum retention

This is the minimum level of retention the reinsurer requires to prevent the insurer from
having too little interest in the risk. This requires the insurer to retain all risks that fall
below the minimum retention.

This is the minimum level of retention for the insurer.

Definition – Maximum retention (R)

This is the maximum level of retention for any risk to be included in the treaty and will be
specified in the treaty.

This is the maximum level of retention for the insurer.

Definition – Number of lines of cover (L)

This is specified in the contract and is used to calculate the maximum cover available from
the reinsurer. The maximum cover available from the reinsurer is calculated as L multiplied
by R.

This defines the maximum level of cover provided by the reinsurer. Reinsurers are typically much
larger than insurers and can usually withstand higher levels of risk. Therefore the reinsurer is
likely to be able to offer cover at a higher level than the insurer. This is why the reinsurer’s
maximum cover may be expressed as a multiple of the insurer’s maximum cover.

The maximum number of lines is the biggest multiple of the direct writer’s chosen retention that
the surplus reinsurer would accept, ie effectively this defines the cedant’s minimum proportion as
1/(1  L) and the reinsurer’s maximum proportion as L /(1  L) .

The insurer’s maximum retention (R) plus the reinsurer’s maximum level of cover ( L  R ) will
determine the maximum size of risk that can be placed on the treaty.

The maximum size of risk that can be placed on the treaty (including the cedant’s retention)
will be:
(1  L)  R

It may help to visualise what’s going on here. The diagram on the following page shows the limits
that the treaty will specify.

© IFE: 2019 Examinations The Actuarial Education Company


SP7-06: Reinsurance products – types Page 11

$m
Maximum level
of cover = (1+L)R

LR

Reinsurer's maximum
cover
3R

2R

Maximum
retention = R

Minimum Insurer's maximum


retention retention

Determination of proportion ceded


The treaty lays down a framework within which the insurer can select, for each individual
risk, the amount that will be ceded to the reinsurer and the amount that it wishes to retain.

So far, we have just looked at what the treaty will specify – we have not considered how much of
each risk the insurer will actually choose to retain, and how this will be specified in terms of lines.
Let’s look at this now.

Given any risk, E, normally measured by the EML, up to a maximum size of (1  L)  R , the
insurer will analyse the nature of the risk and select a retention to keep (call this r).
Note: r  R and r is not less than any treaty-stipulated minimum.

Question

Draw on the diagram above:


 an EML (E) which satisfies the condition in the previous paragraph
 the region in which r may be chosen.

The Actuarial Education Company © IFE: 2019 Examinations


Page 12 SP7-06: Reinsurance products – types

Solution

$m
Maximum level
of cover = (1+L)R

LR

3R

2R

Maximum
retention = R
r
Minimum
retention

Note that if E exceeds (1+L)R then the insurer will probably have to arrange further reinsurance,
eg a second surplus treaty.

Also, retentions will vary from case to case and indeed a number of risks, though covered
by the scope of the treaty, may not be ceded under the arrangement.

Question

Suggest two examples of risks that may not be ceded to the reinsurer.

Solution

Risks may not be ceded to the reinsurer if:


 the EML (E) falls below the insurer’s minimum retention
 the EML (E) is above the maximum size of a risk, ie (1  L)  R (where R is the insurer’s
maximum retention). Therefore, the insurer will probably be unable to cede this risk
unless additional arrangements are made (eg a second surplus treaty).

© IFE: 2019 Examinations The Actuarial Education Company


SP7-06: Reinsurance products – types Page 13

Where the risk does not exceed the capacity of the treaty, the balance of cover (E  r ) will
be ceded to the treaty (ie the reinsurer). This will require (E  r ) / r lines of cover, with each
whole line being of the same value (r) as that retained by the insurer. Call this number of
lines k (where k  L ). The original premium and all claims for that risk will then be shared
between the insurer and the reinsurer in the proportion 1:k.

It’s often useful to remember that for a specific risk:

E  r   k 1

and also note that k does not have to be an integer.

Where, for example a treaty specifies: R  $2m , L  5 lines, the maximum that could be
written (retention plus full coverage provided by the treaty) would be (1  5)  $2m  $12m .
If a particular risk has an EML of only $6m, the insurer has a number of options available:
(a) retain the maximum of $2m and place the remainder by two lines of cover (each
$2m)
(b) use the maximum lines of cover, five, each $1m, and retain only $1m, provided that
this is not less than the minimum retention
(c) adopt any intermediate approach between the two extremes.

Question

With reference to the question on page 8, calculate the following:


(a) the direct writer’s retention
(b) how many lines of surplus reinsurance were used for each risk
(c) the ‘size’ of the largest risk that is covered by this surplus reinsurance treaty if the
maximum number of lines the reinsurer will accept is 12.

Solution

(a) The direct writer’s retention is: £50,000


(b) The number of lines used was (in the order given): 5, 8, 1/5, 0, 19, 9
(c) The largest allowable risk would be: £650,000 if 12 lines were available

Premiums
Surplus is proportional. (You must hang on to that fact, even if ‘EMLs’ and ‘lines’ worry you.)
Therefore, once a risk has been accepted under the treaty, the premiums for that individual risk
are shared in the specified proportion, ie 1:k, where k is the number of lines used for the risk in
question.

Unlike quota share, you will not be sure at the time the treaty is set up how the total premiums
will be split between the two parties. It will depend on what risks are actually written and what
proportion of each risk is ceded.

The Actuarial Education Company © IFE: 2019 Examinations


Page 14 SP7-06: Reinsurance products – types

Claims
For the individual risk, the claims are split in the same proportion as the premiums, irrespective of
the size of claim. Even if there was a disaster and the total loss was greater than the EML, the
claim would still be split in the specified proportions.

Because the proportions vary from risk to risk, there is no way of telling what the split of total
claims will be between the two companies. It is possible for the overall experience of the direct
writer and the reinsurer to be quite different.

Question

Explain what the main difference is between quota share and surplus reinsurance.

Solution

Whereas quota share has the same proportion of every risk ceded to the reinsurer, the proportion
ceded will vary from risk to risk with surplus reinsurance.

Commission
There may be return commission, override commission and profit commission in the same way as
with quota share reinsurance (and brokerage if the reinsurance was arranged by a broker).

Question

Explain what the insurer could do if the risk exceeds the capacity of the treaty.

Solution

If the risk exceeds the capacity of the treaty, then the insurer may be able to find a ‘second
surplus’ treaty to cover risks over and above the maximum covered under the first surplus treaty.

Other options include:


 not to cede the risk at all
 purchase non-proportional reinsurance on a facultative basis for amounts above the EML
 purchase inuring facultative reinsurance to reduce the size of the EML
 retain the excess of the EML itself (provided the terms of the treaty allow this).

© IFE: 2019 Examinations The Actuarial Education Company


SP7-06: Reinsurance products – types Page 15

Second surplus treaty


Some of the larger risks may exceed the capacity of the first surplus treaty, and therefore
need another reinsurance facility to protect them. It is common, therefore, to find a second
surplus treaty in place with another reinsurer, in order to accommodate automatically that
part of any risk exceeding the limit of the first surplus. There may also be a third, or even
fourth, surplus treaty for some of the larger accounts.

The second surplus acts concurrently with the first surplus for those risks that involve both
treaties. The size of line used under each treaty will be the same, both being based on the
cedant’s retention.

Therefore even if a claim does not exceed the capacity of the first surplus treaty, the risks will still
be shared in the pre-defined proportions between the cedant and the two reinsurers.

The usual practice would be to use the whole of the first surplus capacity for a risk, before
placing the balance on the second surplus treaty. Some treaties may require this, but in
other cases the treaty may stipulate that at least as much risk would have to be placed to
the first surplus treaty as to the second, to avoid selection against the second surplus
reinsurer.

Another option would be for the cedant to purchase facultative reinsurance for the risk in order
to reduce the size of the EML. Alternatively it may be able to cover the rest of the risk itself.

For example, consider a direct writer that has a 6 line surplus reinsurance treaty and has decided
to use the maximum retention of £100,000 for all risks ceded under the treaty.

The insurer writes a risk with an EML of £1,000,000. It can’t automatically cede this risk to the
surplus reinsurer because this would require 9 lines to be ceded, (ie £1m  £100,000  (1  9) )
which exceeds the maximum number of lines available. The insurer must therefore choose one of
the following four options:
1. Arrange reinsurance to operate before the surplus reinsurance, to reduce the size of the
EML eg a facultative excess of loss arrangement could cap losses for this risk at £700,000.
2. Arrange a second surplus treaty of 3 lines (or more) to operate in parallel with the first
surplus treaty.
3. Cede the maximum number of lines into the surplus treaty and pay the remaining share of
any claims itself. (This option may not be available, depending on the terms of the
treaty.)
4. Decline the risk.

Question

For each of the first three options in the example above, calculate the amounts paid by the direct
writer and the reinsurer(s) in the event of a claim for £350,000.

The Actuarial Education Company © IFE: 2019 Examinations


Page 16 SP7-06: Reinsurance products – types

Solution

Option 1
With the EML now reduced to £700,000, the number of lines k must satisfy:
£700,000  £100,000  (1  k) .

It follows that k  6. Therefore all claims on the policy will be shared in the ratio 1 : 6.

For a claim of £350,000, the cedant will pay:


1
350,000   £50,000
7

and the reinsurer will pay:


6
350,000   £300,000 .
7

Option 2
With a second surplus treaty in place, the direct writer can now cede the 9 lines it requires.
Therefore all claims on the policy will be shared in the ratio 1 : 6 : 3 , between the direct writer and
the first and second reinsurers respectively.

For a claim of £350,000, the cedant will pay:


1
350,000   £35,000 .
10

The first surplus reinsurer will pay:


6
350,000   £210,000
10

and the second surplus reinsurer will pay:


3
350,000   £105,000 .
10

Option 3
If the direct writer is allowed to take this option, the surplus reinsurer will pay the same as for
Option 2, and the remainder will be paid by the direct writer. Therefore the shared proportions
1 : 6 : 3 will become 4 : 6. In other words, the direct writer will pay 40% of every claim and the
reinsurer will pay the rest.

For a claim of £350,000, the cedant will pay:


4
350,000   £140,000
10
and the reinsurer will pay £210,000.

© IFE: 2019 Examinations The Actuarial Education Company


SP7-06: Reinsurance products – types Page 17

2.2 The cedant’s claims experience


With surplus reinsurance the cedant can choose which individual risks it wishes to cede
and in what proportion. The cedant may retain varying proportions of each risk.

Therefore, depending on the claims outcome the cedant and reinsurer will have different
outcomes.

Question

An insurance company has a 10-line surplus reinsurance treaty.

Describe how the overall nature of the portfolio of retained risks will differ from the portfolio of
ceded risks.

Solution

The smallest risks may be retained in full. A large proportion of each of the risks in the next size
up will be retained. Only a small portion of the largest risks will be retained.

In contrast, the portfolio of ceded risks will have no exposure to the smallest risks and a much
higher proportionate exposure to the larger risks (eg it could have 10 times the direct writer’s
exposure for the biggest risks).

2.3 The reinsurer’s claims experience


Smaller risks may not reach the reinsurer because they are within the retention. The cedant
and reinsurer will not therefore share the same portfolio of risks. The risks that are ceded
to the reinsurer will vary with the proportion of risk reinsured. Hence the underwriting
experience of the risks to which the treaty applies will vary between the two parties. As with
quota share, there will also be differences in expenses and in interest earnings between the
two on their respective portions of the total portfolio.

2.4 Advantages of surplus


The main advantages to the direct writer are that:
 It enables an insurer to write larger risks, which might otherwise be beyond its
writing capacity.
Surplus reinsurance is particularly suitable for large property risks, ie where the direct
writer feels that some risks are too ‘big’ to be retained fully, and that it is better to share
the risk with a reinsurer. The risks will be big enough to merit the additional individual
attention given to each risk.
 It enables the insurer to choose, within limits, the size of risks that it will retain.
It allows the direct writer to increase its exposure by a chosen amount (within limits),
helping control overall business volumes, fine-tune its exposure and control potential
accumulations of risk.

The Actuarial Education Company © IFE: 2019 Examinations


Page 18 SP7-06: Reinsurance products – types

 It is useful for those classes where a wide variation can occur in the size of risks.
Insurers use surplus reinsurance, rather than quota share, extensively for those classes
where a wide variation can occur in the size of risks. For example, it is used with
commercial fire risks. It is also suitable for some marine and aviation insurance
(ie property cover, like hull and cargo losses, rather than liability cover).
 It helps to spread the risks.
It helps the direct writer spread its exposure (ie diversify). Retaining a portion of lots of
risks is better than retaining all of a few risks.
 The commission may help with cashflow.

2.5 Disadvantages of surplus


The main disadvantages to the direct writer are that:
 The administration is more complicated than for quota share, owing to the need to
assess and record separately for each risk the amount to be ceded.

 It is unsuitable for:
– unlimited covers, eg motor liability
– personal lines cover where potential losses are small compared to the
insurer’s resources.

In addition, the treaty terms may not be flexible enough, so that it may not cover the largest risks
without the need for extra negotiation.

© IFE: 2019 Examinations The Actuarial Education Company


SP7-06: Reinsurance products – types Page 19

3 Excess of loss reinsurance

3.1 How excess of loss (XL) operates


With XL reinsurance, the reinsurer agrees to indemnify the cedant for the amount of any
loss above a stated excess point. More usually, the reinsurer will give cover up to a stated
upper limit, with the insurer purchasing further layers of XL cover – which stack on top of
the primary layer – from different reinsurers. The higher layer cover(s) come into operation
on any particular loss only when the lower layer cover has been fully used (or ‘burnt
through’).

The top layer of excess of loss reinsurance might be unlimited (ie have no upper limit).

The layers of reinsurance should be arranged so that there are no gaps, ie the lower limit of the
second layer of reinsurance starts at the upper limit of the first excess of loss reinsurance.

The expression generally used to describe the cover provided under an excess of loss reinsurance
treaty is:
Amount of layer in excess of lower limit

So a treaty that provides cover for claim amounts between an excess point of £50,000 and an
upper limit of £200,000 would be described as:
£150,000 in excess of £50,000, or simply as £150,000 xs £50,000.

Question

A direct writer has three excess of loss treaties covering its employers’ liability portfolio:
£140,000 in excess of £60,000
£300,000 in excess of £200,000
£2m in excess of £700,000

Explain what is silly about this reinsurance programme.

Calculate how much the direct writer will be able to recover in respect each of the following
claims.

(a) £80,000 (b) £280,000 (c) £2,400,000 (d) £4,000,000


(e) £680,000 (f) £50,000 (g) £500,000

Solution

There is a ‘hole’ in the cover between £500,000 and £700,000. The layers of reinsurance ought
generally to be arranged so that there are no gaps.

The Actuarial Education Company © IFE: 2019 Examinations


Page 20 SP7-06: Reinsurance products – types

Recoveries from the treaties are:

Claim 1st layer 2nd layer 3rd layer Total


(a) £20,000 – – £20,000
(b) £140,000 £80,000 – £220,000
(c) £140,000 £300,000 £1,700,000 £2,140,000
(d) £140,000 £300,000 £2,000,000 £2,440,000
(e) £140,000 £300,000 – £440,000
(f) – – – –
(g) £140,000 £300,000 – £440,000

Retention / deductible
This is the loss amount that is retained and paid by the cedant.

Working layer
Working layers refer to the layers above the cedant’s retention where moderate to heavy
loss activity is expected by the cedant and reinsurer. Working layer reinsurance
agreements often include adjustable features (for example, premiums that increase to some
extent with adverse claims experience) to reflect actual underwriting results.

An adjustable feature is one that changes with the experience of the contract. Profit commission
is an example.

Indexed limits
Where inflation has a significant effect on the cost of claims, a stability clause may be
applied to the excess point. This is so that the reinsurer does not receive a higher
proportion of the risks purely because of inflation. The cedant will normally be required to
pay an extra premium to compensate the reinsurer for the added risk if the excess point is
not indexed.

The upper limit (where one exists) may similarly be indexed to preserve the original real
value of the cover.

The basis for indexation should be a reliable inflation index that bears some relation to the
inflationary effects on the claim sizes.

© IFE: 2019 Examinations The Actuarial Education Company


SP7-06: Reinsurance products – types Page 21

Question

A direct writer has two layers of risk XL cover. The first is 100,000 in excess of 100,000, the
second is 300,000 in excess of 200,000. All limits are indexed, and the chosen index starts the
treaty at a value of 100.

Two separate claims are made: one for 350,000 when the index was 105 and the other for
600,000. The index on the dates the claim amounts were agreed was 105 and 110 respectively.

Assuming that the treaties cover these claims, calculate how much the insurer and each reinsurer
pay in respect of each claim.

Solution

Claim of 350,000 (when index was 105)

When the index is 105, the layers of reinsurance cover become 105,000 in excess of 105,000 and
315,000 in excess of 210,000. Therefore:
Direct writer pays the first 105,000
First XL reinsurer pays the next 105,000
Second XL reinsurer pays the remaining 140,000.

Claim of 600,000 (when index was 110)

When the index is 110, the layers of reinsurance cover become 110,000 in excess of 110,000 and
330,000 in excess of 220,000. Therefore:
Direct writer pays the first 110,000
First XL reinsurer pays the next 110,000
Second XL reinsurer pays the next 330,000.
Direct writer is left with the remaining 50,000, ie paying 160,000 in total.

Note that in practice, the actual operation of stability clauses might vary, depending on the exact
policy wording, so any reasonable approach should earn you marks.

Commission
Return commission and override commission are not normally relevant to excess of loss
reinsurance. This is because the reinsurer charges the insurer a premium to cover the risk and so
a commission payment back to the insurer would be equivalent to simply charging a lower
premium.

Profit commission is possible, for example, for working layer reinsurance. However, profit
commission would be totally inappropriate for the very high layer excess of loss treaties, where
the reinsurer expects to pay out only in exceptional circumstances.

The Actuarial Education Company © IFE: 2019 Examinations


Page 22 SP7-06: Reinsurance products – types

Brokerage is very likely to be paid, since this type of reinsurance is usually arranged through
brokers.

Reinstatements
Under excess of loss reinsurance arrangements, the treaty will often include
reinstatements; that is, after a number of separate events have collectively exhausted the
XL treaty limit, the reinsurer will allow one or more reinstatements of the coverage. Often,
further premiums (reinstatement premiums), which may be more or less than the original
premium and may be scaled down for the unexpired risk term, are payable.

A reinstatement is the restoration of full cover following a claim. After a reinsurance claim is
made, the cedant may be required to pay an additional premium for the insurance cover to
continue. The reinstatement premium and the number of reinstatements allowed will be set out
in the treaty document  once agreed, both parties are bound by the arrangement and there are
no options.

Limited reinstatements usually apply to higher layers of reinsurance cover. For lower layers of
cover, there may be a limited number of free reinstatements before an additional premium is
required. For very low working layers, there may be unlimited free reinstatements.

In practice, reinstatement premiums are often deducted from the claim recoveries, as this
simplifies the administration.

The precise way that reinstatements work in practice depends on the wording in the treaty. A
common approach would be to charge a proportionate premium according to the proportion of
the layer that has been burnt through (often known as pro-rata as to amount). The period of
cover remaining may also affect the premium (often known as pro-rata as to time).

For example, if a claim is made which uses half of the original layer and half the original period of
cover is remaining, a premium of a quarter of the full reinstatement premium may be deducted
from the claim recovery in order that cover can be restored.

Question

A $1.5m xs $1m excess of loss reinsurance treaty has the following terms and loss history in the
year it was written:
 up front premium of $300,000
 1 reinstatement at 140% additional premium.

In chronological order the only losses to potentially impact this treaty are:
1. $2.0m
2. $5m
3. $1.8m
4. $10.2m.

© IFE: 2019 Examinations The Actuarial Education Company


SP7-06: Reinsurance products – types Page 23

Ignoring the issue of the period of cover remaining, calculate how much of each loss is
recoverable and the reinstatement premiums generated by each loss.

Also, calculate the ‘rate on line’ of this contract.

(Hint: You need to know your Glossary definitions to pass this subject)

Solution

Assume that the reinstatement premium will be calculated according to the proportion of cover
burnt through. We are told to ignore the issue of the period of cover remaining, although in
practice it could be a significant issue.

First claim: recover $1m (ie $2.0m less $1m excess) from the reinsurer. Note that $0.5m of the
reinsurance cover, ie a third, is yet to be used. Pay a reinstatement premium of $0.28m (ie two
thirds of 140%  300,000). In practice this is likely to be offset from the reinsurance recovery.

Second claim: recover $1.5m from the reinsurer, and pay a further reinstatement premium of
$0.14m (the remainder of the reinstatement premium). Again this is likely to be offset from the
reinsurance recovery. This gives cover remaining of one-third of the layer, ie $0.5m in excess of
$1m.

Third claim: recover $0.5m (the remaining cover). No further reinstatement premium can be
paid.

Fourth claim: No reinsurance recoveries and no further reinstatement premium.

The rate on line is defined, for non-proportional reinsurance, as the total premium charged for
the reinsurance divided by the width of the layer covered. Note by total premium we are only
referring to the initial premium, the reinstatement premium should be excluded.

0.3
Hence the rate on line is , ie 20%.
1.5

Increased limit factors


These are factors that define the expected loss cost for a limit L as a multiple of the loss
cost for a loss with the basic limit B. For example, for a basic limit of B the factor is 1.0, for
a limit of 2B the factor may be 1.6 (must be ≤ 2) and for a 3B limit the factor may be 2.0
(must be ≤ 3). Thus, the expected loss costs are 60% more and 100% more than the cost of
a basic limit respectively.

The development of increased limit factors (ILFs) needs a little explanation. (Note that Subject
SP8 goes into considerably more detail on this topic.)

The Actuarial Education Company © IFE: 2019 Examinations


Page 24 SP7-06: Reinsurance products – types

Suppose we have a line of business where claims are limited to B. We are considering raising the
limit to C and want to know what the expected severity would now be.

We will need to work out the ILF which will be:


Expected loss cost at C
ILF 
Expected loss cost at B

If we assume that that the expected frequency at both limits is the same, we can then simplify
this to:
Expected severity at C
ILF 
Expected severity at B

Each element of this ratio can be expressed as a limited expected value, ie the average size of loss
where all losses are limited to the particular limit given.

For example, assume that for a particular line of business, losses are as follows:

Original loss (£) Limited at £1,000 Limited at £2,000


100 100 100
500 500 500
1,500 1,000 1,500
3,000 1,000 2,000

The ILF taking the basic limit of £1,000 to the limit of £2,000 would be calculated as:
100  500  1,500  2,000
ILF   1.6
100  500  1,000  1,000

In more complicated cases, where we have a distribution function f  x  for the claim severity, we
could work out the ILF between limits B and C as the ratio of two limited expected severities:
C 
 xf  x    Cf  x 
ILF  0 C
B 
 xf  x    Bf  x 
0 B

Question

Explain why as the limit increases you would normally expect to see the ILF increase, but at a
decreasing rate.

© IFE: 2019 Examinations The Actuarial Education Company


SP7-06: Reinsurance products – types Page 25

Solution

Claim severity distributions are normally positively skewed as smaller claims are more common
than larger ones. Expected values therefore increase, but at an ever-decreasing rate, as fewer
and fewer claims are falling above the increasing limit.

Proportional within non-proportional


For all forms of excess of loss, it is possible that the reinsurer will cover only a proportion of the
claims within the layer, by applying a deductible. For example, the cover might be set as 90% of
£150,000 in excess of £50,000. In this case, which has a 10% deductible, a claim of £120,000
would generate a recovery of £63,000 (ie 90% of £70,000).

The reason for this type of arrangement is to give the direct writer more incentive to keep the
claim settlements low. Otherwise the reinsurer may feel exposed to the ‘moral hazard’ of the
direct writer having sloppy settlement procedures for large claims.

Question

Explain why in practice the direct writer would be keen to handle claims above the excess point
prudently, even if 100% of the claim above the excess was being paid by the reinsurer.

Solution

The insurer will manage claims above the excess in order to:
 reduce the risk of them exhausting the reinsurance layer and reverting back to the insurer
 maintain a good long-term relationship with the reinsurer
 benefit from any profit commission
 reduce future reinsurance premiums (which will be related to past claims experience).

3.2 Cedant’s claims experience


The cedant’s experience will differ from the reinsurer’s experience and will be better or
worse depending on the distribution of large losses.

This is only to be expected, since (unlike proportional reinsurance) it is only once claims reach a
certain size that the reinsurer has to pay anything.

For an unlimited XL cover (although this is rarely available), the reinsurer is likely to suffer a
higher claims ratio than the cedant when more large losses occur than expected, and a
lower claims ratio when the account experiences a high frequency of low severity losses
that are less than the excess point. Note: for a limited XL cover (the more usual scenario),
the claim amount exceeding the upper limit of the reinsurance will revert back to the cedant.

The Actuarial Education Company © IFE: 2019 Examinations


Page 26 SP7-06: Reinsurance products – types

3.3 Reinsurer’s claims experience


Reinsurance claims can differ markedly in shape and size from those of the original writing office.
Typically on smaller claims, below the treaty limit, the reinsurer will not be liable at all. On larger
claims, the reinsurer may have to provide the majority of the loss, depending on the limits in the
arrangement’s wording.

The degree to which reinsurers can influence the underwriting and claims management will
depend on the wording of the treaty and the nature of the relationship between the parties.

3.4 Types of excess of loss (XL) reinsurance


There are three main types of excess of loss reinsurance:

 risk XL

 aggregate XL

 catastrophe XL.

In practice the boundaries between different forms of XL reinsurance are not distinct. In any
given case the treaty will spell out exactly what is covered.

3.5 Risk (or individual) XL


This is a type of XL reinsurance that protects against large individual losses.

The reinsurer indemnifies an insurance company for the amount of an individual loss in
excess of the excess point.

Risk XL should protect an insurer adequately against losses that affect only one insured
risk, provided that the upper limit is sufficiently high.

The direct writer pays a premium to the reinsurer in return for protection against individual large
claims (eg a large liability claim on an individual motor policy).

Risk XL can be written facultatively or by treaty.

3.6 Aggregate XL
Aggregate XL is a very simple extension of risk XL. However, rather than operating on individual
large claims, the excess point and the upper limit apply to the aggregation of multiple claims. As
you will see, there are several different ways that the claims might be aggregated.

The reinsurer indemnifies an insurance company for a cumulative, that is, aggregate
amount of losses in excess of a specified aggregate amount.

As with risk XL, there will normally be an upper limit to this cover. This and the excess
point will usually be at a higher level than for the working layer. There will also be a limit on
the aggregate amount payable in the year from all events, usually by offering a strictly
limited number of reinstatements. The reinsurer will usually also require payment of a
reinstatement premium each time.

© IFE: 2019 Examinations The Actuarial Education Company


SP7-06: Reinsurance products – types Page 27

There are several ways it can be written:

 excess of a monetary amount (for example, $500,000 in the aggregate excess of


$750,000 in the aggregate)

 excess of a percentage (loss ratio) amount (for example, 50 loss ratio points excess
of 75 loss ratio points)

 with an interior deductible; that is, to apply only to losses excess of a stated dollar
amount (for example, $500,000 in the aggregate excess of $750,000 in the aggregate
applying only to losses greater than $50,000 per loss).

This form of contract would usually protect an account that would not normally be exposed
to major ‘event’ losses but could be subject to major attritional losses (that is, a large
number of small losses), for example, a medical expenses account.

Aggregation by event
Events can occur that cause losses to several insured risks at the same time. Depending
on the insurer’s risk portfolio, that event could lead to an aggregation of claims.
Individually, each claim might not be of an exceptional size, but collectively the aggregate
cost might be damaging to the insurer’s gross account.

In this situation, the conventional risk XL treaty, by treating each claim as a separate loss,
will fail to protect the cedant adequately against the aggregate cost of such losses, thus the
need for aggregate XL reinsurance.

In this case, all the claims arising from that event might be added together. If the total amount
exceeds the lower limit, then the direct writer can make a recovery from the reinsurer.

Usually an event covered by aggregate XL will be sudden and obviously identifiable.

For example, the event might be a motorway pile-up caused by a single driver. The insurer of this
driver might then be faced with dozens of individual injury claims, none of which in isolation is too
worrying, but which in total could damage the company.

A common form of reinsurance for motor insurers is ‘claim and event XL’, under which cover is
provided for a single large claim or all claims from a single event.

However, in some cases, the time span of the event may be less easily identifiable and
therefore require careful wording in the treaty definition.

For example, the event could be a major storm. In this instance, the event would need to be
defined very carefully so that there is no doubt as to whether particular claims are to be included.
This form of aggregation by event is generally known as catastrophe excess of loss. Catastrophe
excess of loss is described in the next section.

The boundary between aggregate and catastrophe excess of loss is not distinct.

Aggregation by peril or cause


Alternatively, cover may be defined in terms of a common cause or peril, rather than a
single event, over a particular period of time.

The Actuarial Education Company © IFE: 2019 Examinations


Page 28 SP7-06: Reinsurance products – types

For example, a direct writer may seek aggregate excess of loss reinsurance for all claims from
asbestos-related claims affecting its employers’ liability account.

Aggregation by class
The direct writer may obtain aggregate excess of loss reinsurance to cover all claims from a
particular class of business. This form of aggregate excess of loss is called stop loss. It is described
in Section 4.

3.7 Catastrophe XL
In its most extreme form, an event may be of catastrophic proportions, involving losses to
many hundreds, or even thousands, of different insured risks with a potential cost falling
beyond the normal capacity and intention of aggregate treaties and possibly even beyond
the finances of a large insurer.

The scope of aggregate XL therefore needs to be extended, to catastrophe XL, to cope with such
disasters.

Catastrophe XL is a form of excess of loss reinsurance which, subject to a specified limit,


indemnifies the ceding company against the accumulated loss, in excess of a specified
retention, resulting from a catastrophic event or series of catastrophic events.

Definition – Hours clause


A clause within a catastrophe reinsurance treaty that specifies the limited period during
which claims can be aggregated for the purpose of one claim on the reinsurance contract.
Common examples are 24 or 72 hours.

How catastrophe excess of loss operates


The main difference between catastrophe XL and aggregate XL is that catastrophe XL
operates at a much higher level of aggregate cover. A catastrophic event also needs to be
defined carefully because the cause may not always be instantaneous, and it may be spread
over a wide geographical area. A severe windstorm, for instance, may last for several days;
it may even abate only to return to cause further waves of damage.

For this reason, the hours clause of a treaty usually limits windstorm claims to all losses
occurring within a consecutive period of 72 hours, whilst freeze claims are usually limited to
any period of 96 hours. The cedant is allowed to choose the starting point of the period to
which the reinsurance claim will apply.

The purpose of the hours clause is to prevent any disputes occurring in cases where, for example,
two different storms occur four days apart (in which case the two storms are treated as separate
events). The starting point will be chosen by the insurer in order to maximise the reinsurance
recoveries.

© IFE: 2019 Examinations The Actuarial Education Company


SP7-06: Reinsurance products – types Page 29

Question

Apart from brokerage, suggest what types of reinsurance commission might sensibly apply for a
catastrophe excess of loss reinsurance treaty.

Solution

None. There’s no need for return or override commission because the reinsurer charges a
premium, so the commission would simply act to reduce this premium. Profit commission would
be silly because the reinsurer would expect claims rarely.

3.8 Advantages of excess of loss


The main advantages of excess of loss to the direct writer are that:
 It allows an insurer to accept risks that could lead to large claims. ‘Large’ here
means large relative to the insurer’s solvency margin, annual premiums, expected
profits, or the results of the impacted classes.

 It reduces the risk of insolvency from a catastrophe, a large claim or an aggregation


of claims.

 It stabilises the technical results of the insurer by reducing claim fluctuations.


By reducing claim fluctuations (and hence the variance of the claims outgo), the annual
profitability of the direct writer will be less volatile. (Shareholders are not known for their
love of profit volatility.)
 It helps make more efficient use of the capital by reducing the variance of the claim
payments.
High volatility of outgo means that the company must hold large free reserves. If the
volatility is reduced, lower free reserves are required. This means that the company can
write the same amount of business with less capital by using excess of loss reinsurance.

3.9 Disadvantages of excess of loss


The insurer pays a premium to the reinsurer that in the long run, if priced accurately, will be
greater than the expected recoveries under the treaty.

This is because the excess of loss reinsurer’s premium will load the expected claims for expenses,
profit and contingency margins.

From time to time, excess of loss premiums may be considerably greater than the pure risk
premium for the cover. For example, after reinsurers have had a few years of poor results, the
supply of reinsurance falls and premiums rise, as reinsurers attempt to restore their solvency
positions.

Excess of loss cover may be hard for the insurer and reinsurer to price.

The Actuarial Education Company © IFE: 2019 Examinations


Page 30 SP7-06: Reinsurance products – types

4 Stop loss
As the name suggests, stop loss reinsurance is designed to stop losses. In fact, it doesn’t actually
stop losses altogether, but it can help make bad losses a bit less dire. With stop loss reinsurance,
the aggregation applies to …
… all claims arising …
… in a defined account(s) (eg a specified class) …
… during a defined period (eg a year).

Stop loss is a form of XL reinsurance that indemnifies the ceding company against the
amount by which its losses incurred during a specific period (usually 12 months) exceed
either:

 a predetermined monetary amount or

 a percentage of the company's subject premiums (loss ratio) for the specific period.
Subject premiums are the premiums for the account that is being reinsured.

Sometimes a particular class of business gives rise to large variations in the levels of total
claims payable in any one year. Aggregate XL only provides cover for either one cause over
the year or one event. An insurer may therefore wish to protect the class of business by a
form of reinsurance that extends cover to all causes or all events during the year. Stop loss
does this by covering the total losses for the whole account, above an agreed limit, for a
12-month period. The whole account can be one or several classes of insurance.

The excess point and upper limit for stop loss are often expressed as a percentage of the
cedant’s premium income for that account. Cover might typically be given from an excess
point of a 110% claims ratio up to an upper limit of 130% or 140%.

Question

Explain why stop loss cover is often quoted in terms of claim ratios rather than monetary
amounts.

Solution

If it was not, then the direct writer could, after taking out the cover, write loads more business, by
cutting premiums, and trigger the stop loss limits that way.

By using claim ratios, the limits (and the premium charged for the cover) rise in proportion to the
amount of business written by the direct writer.

© IFE: 2019 Examinations The Actuarial Education Company


SP7-06: Reinsurance products – types Page 31

For example, consider a company that writes just one class of business and has the following stop
loss reinsurance treaty in place for business written in a given year:
 lower limit of 105% of earned premium
 upper limit of 125% of earned premium
 reinsurer covers 80% of the claims in the layer.

Suppose that the earned premiums for the year in question were £292m and the total incurred
claims were £333m (ie a loss ratio of 114%).

The reinsurance recovery would be 0.8  (333  1.05 292)  £21.12m.

Question

(a) Explain why reinsurers are often not prepared to provide stop loss cover.

(b) If the reinsurer does provide stop loss cover, suggest what conditions they are likely to
impose on the business covered.

Solution

(a) Reinsurers are often not prepared to provide stop loss cover because:
 the reinsurer has only limited control over initial underwriting and claim
payments made
 historically some stop losses have been loss making.

(b) Conditions the reinsurer may impose before providing stop loss cover are:
 impose a deductible so that the insurer retains a proportion of the risk
 maintain some control over underwriting, premium rates and claims.

The Actuarial Education Company © IFE: 2019 Examinations


Page 32 SP7-06: Reinsurance products – types

5 Finite risk (or financial) reinsurance

5.1 Purpose of finite risk (or financial) reinsurance


Traditional risk transfer involves standard reinsurance contracts (proportional and
non-proportional). Risk financing moves away from this, however, the risk may still be transferred
to a reinsurer. Alternatively it may be transferred to the capital markets. This section and the
next one describe products that involve a reinsurer. Section 7 describes products that involve the
capital markets.

The aim of finite risk reinsurance tends to be risk financing rather than traditional risk
transfer. This is typically a multi-year contract aimed at reducing the cedant’s cost of
capital by means of earnings smoothing. The contract reduces year-to-year earnings
volatility but it provides limited risk transfer over the whole contract period.

A wide variety of financial reinsurance contracts exist, although all have been devised
primarily as a means of improving the apparent accounting position of the cedant.

5.2 Features of finite risk (or financial) reinsurance


The main feature of a financial reinsurance contract is that it involves only a small element,
if any, of transfer of insurance risk from the cedant to the reinsurer. The contract, however,
does involve investment type risks. Many forms of financial reinsurance are, in fact, often
viewed as being more similar to investment than to reinsurance. Usually the effective
‘return’ that the contracts provide is low in comparison to conventional investments.

Typical features might be:

 limited assumption of risk by the reinsurer

 multi-year contract term

 explicit inclusion of investment income in the contract

 sharing of the results with the cedant

 risk transfer and risk financing are combined; for example, time and distance deals.

Time and distance deals are described in Section 5.4.

5.3 Types of finite risk (or financial) reinsurance


There are two types of finite risk reinsurance:
 Pre-funded arrangements, whereby the insurer pays premiums into a fund held by the
reinsurer (which earns interest), and claims are paid from the fund.
 Post-funded arrangements, whereby the reinsurer pays the losses and the insurer pays
back the losses over time.

© IFE: 2019 Examinations The Actuarial Education Company


SP7-06: Reinsurance products – types Page 33

Question

Explain what the main difference is between these two arrangements as far as managing risk is
concerned.

Solution

With pre-funded arrangements, the insured bears the risk of the reinsurer’s default, and vice
versa for the post-funded arrangement.

Note that while the two of these may involve reinsurers, they may also be classed as capital
market products.

Specific types of finite risk (or financial reinsurance) include:


 time and distance deals
 spread loss covers
 financial quota share
 structured finance
 industry loss warranties.

5.4 Time and distance deals


Time and distance deals are designed to discount technical reserves for the time value of money.

An insurer pays a single premium in return for a fixed schedule of future payments matched
to the estimated dates and amounts of the insurer’s claim outgo. The purpose of such
contracts was to achieve the effect of discounting in arriving at the reserves for outstanding
claims.

For example, for a payment of £10m on 1 January 2018 the financial reinsurer may agree to pay
to the direct writer £1.2m on 1 January from 2019 – 2028 inclusive. The reinsurance premium is
£10m and the reinsurance recoveries form an annuity of £1.2m pa, payable annually in arrears for
ten years.

This does not look at all like a reinsurance policy. The insurer is effectively purchasing an annuity
from the reinsurer. The policy is more similar to an investment than a reinsurance policy.
However, this type of policy has been used in various parts of the world in order to improve the
apparent solvency position of the insurance company. You may be asking how.

The Actuarial Education Company © IFE: 2019 Examinations


Page 34 SP7-06: Reinsurance products – types

If the relevant authorities can be convinced that the arrangement constitutes reinsurance then
the annuity payments may be treated as reinsurance recoveries (ie negative claim payments)
rather than investment proceeds. If future claim payments and hence reinsurance recoveries can
be shown at face value in the balance sheet, ie the payments don’t have to be discounted (and
the present value shown), then in this example the payments would be taken into account at face
value, ie £12m. Hence, the company has swapped a cash asset of £10m (the reinsurance
premium) for an asset of £12m (the reinsurance recoveries) and its disclosed solvency position
appears to increase by £2m.

Please note that we have deliberately used the words ‘if’ and ‘may’ in the above paragraph.
There is no guarantee that the local supervisory authority will allow such contracts to be valued at
face value. If the insurer is obliged to calculate the present value of the future annuity payments
then there might not be any benefit to the insurer at all, depending on the discount rate used.

They were useful in the past to insurers who were not permitted to discount their reserves
(eg Lloyd’s syndicates).

Similarly, these contracts do not count as insurance contracts for the purpose of accounting in the
US, so they no longer achieve the desired effect on balance sheets.

Since Lloyd’s changed its rules so that the credit allowed for time and distance policies in a
syndicate’s accounts was limited to the present value, such policies have become less
popular.

However, some companies in certain countries still have these policies on their books.

A finite risk reinsurance contract has to have a reasonable level of risk transfer if it is to be
treated as reinsurance under most systems of accounting principles; otherwise it is treated
as an investment and may thus lose its appeal.

Few financial reinsurance policies are as simple as the one described above. Often they are
‘disguised’ to look like normal reinsurance contracts. If you come across a reinsurance policy that
seems to be transferring very little claims risk from the insurer to the reinsurer then it is probably
a financial reinsurance policy.

For example, assume Insurer A has the following aggregate excess of loss arrangement with
reinsurer B:

The reinsurance covers all claims due to storm and flood damage on any household property
policies written by A during the year 2019 in excess of £0.8m, with an upper limit of £2m
(ie ‘£1.2m xs £0.8m’).

At the start of 2019, a reinsurance premium of £1.2m is paid by A. Return commission of


£200,000 is paid by the reinsurer – giving a net reinsurance premium of £1m.

A 98% profit share is payable to A at the end of 2022. This is calculated as 98% of profit, where
profit is calculated as the original premium of £1.2m less any recoveries made under the
reinsurance treaty.

At the end of 2022, the arrangement is then terminated. At this time, all reinsurance recoveries
and profit share must be paid to insurer A.

© IFE: 2019 Examinations The Actuarial Education Company


SP7-06: Reinsurance products – types Page 35

What type of reinsurance arrangement is this?

In order to answer this question, we need to consider the cashflows that insurer A will experience.

In return for a net premium of £1m paid on 1/1/19, insurer A receives (on 31/12/22):
 £1.176m (ie 98% of 1.2m), if no recoveries are made
 £1.2m as recoveries, if recoveries exceed £1.2m (hence the profit is zero), or
 between £1.176m and £1.2m, if the layer is partially burnt through.

You can see that claims experience has very little effect on insurer A’s finances, so there is very
little transfer of claims risk between the two parties. It is therefore actually a financial
reinsurance contract.

Question

Discuss the reasons why insurer A and the reinsurer may wish to take out this particular
arrangement.

Solution

The insurer may wish to take out this arrangement:


 To improve its published solvency margin. Depending on local accounting regulations, it
may be able to show the expected return of (at least) £1.176m as an asset and this could
improve its published solvency margin by £0.176m.
 As an investment. It will make between 17.6% and 20% return on the reinsurance
premium over 4 years, and this may be considered to be a good return.

The insurer will also need to consider whether or not four years is a good match to the term of
the liabilities covered by the arrangement.

The reinsurer will also want to benefit from the deal. Benefits for the reinsurer might include:
 an improvement in the published solvency position
 a good investment (if it can make a better return than it is giving the insurer)
 tax efficiency
 a good relationship with the cedant, giving the opportunity for cross-selling.

Note that here, the initial payment is from the insurer to the reinsurer. In the majority of other
financial reinsurance arrangements, the initial reinsurance commission is from the reinsurer to
the insurer.

The Actuarial Education Company © IFE: 2019 Examinations


Page 36 SP7-06: Reinsurance products – types

5.5 Spread loss covers


Spread loss covers involve the insurer paying annual or single premiums to the reinsurer
for coverage of specified claims. These accumulate with interest (contractually agreed) in
an experience account, the balance of which is settled at the end of the multi-year period.

These might be useful where the insurer is exposed to a potentially large risk that may occur from
time to time, for example an earthquake. A spread loss cover would help to spread the effect of a
possible big loss over several years.

For example, an insurer covering an earthquake risk may expect that in nine years out of ten there
will be no claims, but in the tenth year, there will be a very large claim. In that year (whenever it
turns out to be), there may be a severe adverse effect on the profits of the company. To deal
with this, the insurer may pay a series of premiums to a reinsurer, and in return the reinsurer will
pay a claim when it is needed.

The definition of the claim will be such that it cannot be much greater than the sum of the
premiums, while if low or no claims occur, the premium can be refunded. In this way the
reinsurer is only taking limited timing risk, and so can keep their charges down.

However, the reinsurer bears the credit risk of the insurer, if the balance on the experience
account turns negative.

These types of contracts involve very limited underwriting risk (limited practical risk
transfer) but provide the insurer with the liquidity and security of the reinsurer.

Again, there often has to be sufficient (but minimal) risk transfer in order for this contract to
count as ‘reinsurance’.

As well as liquidity and security, this arrangement helps reduce the volatility of the insurer’s
reported results, which may be important at least for presentation purposes.

For example, suppose that when a claim in respect of this earthquake risk occurs (in one year in
ten) it is for $1m. Instead of reporting results that reflect claims outgo of $0m in nine years and
$1m in one year, an insurer may prefer its reported results to reflect claims outgo of $100,000
every year.

At a more strategic level, reduction in the volatility of reported results should reduce the
requirement for capital allocation and therefore improve return on the capital employed.

Less volatile results will reduce the buffer capital that needs to be held, thereby freeing up this
capital for other uses, eg in development of the business, that will earn better returns than would
be achieved by having to keep it invested as solvency capital.

5.6 Financial quota share


Financial quota share is an arrangement that exists for commissions to be paid for
financing purposes, rather than the transfer of risk.

© IFE: 2019 Examinations The Actuarial Education Company


SP7-06: Reinsurance products – types Page 37

This is a traditional quota share arrangement, but written for the primary purpose of a financial
arrangement involving the commission payment. Financing is achieved by overcompensating,
(ie paying more than a normal reinsurance commission), in the initial period and
undercompensating, (ie paying less than a normal reinsurance commission), over a period
thereafter.

It is therefore a type of financial reinsurance contract.

5.7 Structured finance


Structured finance is a broad term used to describe a sector of finance that was created to help
transfer risk using complex legal and corporate entities.

Reinsurers became involved in structured finance through their finite reinsurance business
and the increasing need of financial guarantee insurers and investment banks for additional
capacity.

The typical financing solution provided by the reinsurer is a credit enhancement in which
the reinsurer provides a financial guarantee or credit insurance wrap to the institution
borrowing from the capital market.

Credit enhancements involve insurance companies insuring loan portfolios or providing credit
protection to companies to improve the creditworthiness of debt instruments. These solutions
use derivative products available in the capital markets, in addition to variations on traditional
credit insurance.

Credit insurance wraps are insured or guaranteed by a third party. The third party may provide a
promise to reimburse losses up to a specified amount. Deals can also include agreements to
advance principal and interest or to buy back any defaulted loans. The third-party guarantees are
typically provided by AAA-rated financial guarantors.

Credit enhancement or financial guarantees lower the cost of borrowing.

5.8 Industry loss warranties (ILWs)


Industry loss warranties (ILWs) are a type of reinsurance where the basis of cover is not
indemnity, ie repayment of actual losses suffered.

Here one party will purchase protection based on the total loss arising from an event to the
entire insurance industry rather than their own losses.

The original size of the industry loss is used as a trigger for a recovery.

The contract pays a specified fixed amount to the insured if:


 there has been an insured loss of a particular type, eg a hurricane, to the insurance
industry of a particular size, and
 a second indemnity-based trigger is breached, based on the value of the losses incurred
by the insured.

The Actuarial Education Company © IFE: 2019 Examinations


Page 38 SP7-06: Reinsurance products – types

The ‘industry loss’ size referred to in the first bullet point is usually determined using a recognised
standard of some sort. For example, for US property events the common source is PCS (Property
Claim Services), which is an organisation in the US that collates and publishes industry
catastrophe loss information for US catastrophe events, such as tornadoes, hailstorms and floods
as well as hurricanes and earthquakes. The loss amount published by PCS is often used to
determine whether an event has breached the industry loss trigger for an ILW. In other regions a
cat model might be used, which calculates a model industry loss size based on wind speed,
earthquake magnitude etc.

The second condition is necessary to ensure that the insured has an insurable interest in the
cover.

The payout to the insured may be fixed, so there is a potential mismatch that works in favour of –
or against – the insured.

Typically, reinsurer payment should be quite quick once the insurer makes a claim.

Note: the name Original Loss Warranty (OLW) is sometimes used for the same concept.

© IFE: 2019 Examinations The Actuarial Education Company


SP7-06: Reinsurance products – types Page 39

6 Run-off reinsurance

6.1 Run-off solutions


Run-off reinsurance is not a term generally used in the market place and so the meaning would be
described in any question. It could in practice involve any treatment or processing of any closed
book of business. The reinsurance of a closed tranche is just one possible run-off solution.

The aim of run-off reinsurance is the transfer of reserve development risks. It provides
cover against the insurer’s earnings volatility arising from past activities. It may be sought
in circumstances such as:

 corporate restructuring

 mergers and acquisitions

 closing lines of business

 economic changes in the value of the liability

 regulatory, accounting or tax changes

 legal developments, for example court decisions.

The ‘book’ is sold to the reinsurer who assumes all remaining premiums and all of the risk.
The claims reserves are also transferred from the insurer to the reinsurer.

For example, there have been many run-off solutions applied to accounts with exposure to
US asbestos-related claims, in view of the uncertainty (and deterioration) of that claims
experience.

There are two main types of run-off reinsurance:

 adverse development cover


 loss portfolio transfers.

6.2 Adverse development cover


Adverse development cover is a reinsurance arrangement whereby a reinsurer agrees, in
return for a premium, to cover the ultimate settled amount of a specified block of business
above a certain pre-agreed amount.

It protects the cedant from significant reserve deterioration on run-off business. This caps the
liability and protects the balance sheet from any further development on existing losses, and from
future losses in respect of old business.

The premium that is payable for the cover will depend on the risk appetite of the market.

Usually it is only possible to reinsure a layer above a specified amount. This specified
amount may be in excess of the current level of reserves. There could be an upper limit. If
the ultimate cost of losses is in excess of this, the insurer is liable for the excess. The
reinsurer may also insist that the insurer has a small participation in the layer.

The Actuarial Education Company © IFE: 2019 Examinations


Page 40 SP7-06: Reinsurance products – types

Claims are usually still handled by the insurer and hence there are the associated expenses.
Reserves are maintained by the insurer and it receives all investment income generated
from the investments backing these reserves.

There is no transfer of reserves from the insurer to the reinsurer. The insurer simply pays a
premium for the reinsurer to take on responsibility for the development of reserves beyond a
specified position.

Question

Identify what sort of traditional reinsurance arrangement would achieve this.

Solution

A non-proportional arrangement, eg a stop loss or excess of loss contract.

The insurer is exposed to the credit risk of the reinsurer. Legally, the insurer remains liable
to the insured parties for all claims within the block reinsured. Hence, some but not all of
the risk from adverse run-off of the reserves is removed.

6.3 Loss portfolio transfers (LPTs)


Although not a form of reinsurance, a loss portfolio transfer is an alternative to an adverse
development cover when considering run-off solutions and so is included here for
completeness.

LPTs are an arrangement whereby the liability for a specified book of business is passed in
its entirety from one insurer to another. Policyholders will be informed of this ‘novation’
and the deal may need to be approved by a court. This enables the original insurer to
concentrate on any remaining book of business or exit the market.

The Glossary defines a ‘novation’ as ‘the transfer of the rights and obligations under a contract
from one party to another‘. Note that this definition is strictly only part of the Subject SA3 Core
Reading.

Novation is not strictly reinsurance since the new insurer is responsible for the liabilities in
total from the date of the transfer.

The original insurer will transfer the reserves and the remaining exposure to the new
insurer. It is likely that there will be a premium in addition to the existing reserves. This
would normally include a claims handling service.

All adverse claims risks and the investment income will be passed to the new insurer.

Advantages of LPTs
 They can improve the credit rating of the original insurer.

 The new insurer will gain diversification if it’s not already in this area and achieve a
larger client database. There are specialist players in the market that can possibly
run-off such portfolios more profitably than the original insurer.

© IFE: 2019 Examinations The Actuarial Education Company


SP7-06: Reinsurance products – types Page 41

 The original insurer will no longer have any remaining exposure to the book of business,
including any subsequent reserve deteriorations.
 The deal may be good value for money for one of the companies. For example, the
reserves transferred, plus any additional premium payable and investment income
earned, may be more than sufficient to pay the remaining claims, so the new insurer may
end up making a profit.
 The original insurer’s capital requirements will be lower and capital will be freed up for
other purposes.
 The new insurer may gain access to historical data for the class of business.
 An LPT is a quick method for exiting a line of business (and a quick way for the new
insurer to acquire a book of business).
 The original insurer may no longer need expensive specialist resources, eg claims
handlers, to manage the liabilities.
 Unlike with some forms of reinsurance arrangement (eg adverse development cover), if
the new insurer defaults on its liabilities they will not fall back on the original insurer.

Disadvantages of LPTs
 Assets may need to be realised to pass across the value of the reserves to the
accepting insurer, which is particularly important if there is mismatching or if tax
gains / losses would be crystallised.

 If the new insurer defaults, this could damage the reputation of the original insurer.

 The transfer may require the buy-in of reinsurers where there are existing
reinsurance arrangements covering the portfolio.

 There will be an associated cost to the original insurer of the risk transfer, which will
depend on the current risk appetite of the market. This cost would be any premium
payable plus the ‘lost’ investment income.

 Any required court approval may be time-consuming and expensive, and may not
necessarily be obtained.
 The new insurer may be exposed to the future emergence of new latent claims on the
portfolio which may not have been anticipated / allowed for in the LPT calculations.

The ‘premium payable’ referred to above is an amount to compensate the new insurer both for
taking on the risk and for expenses associated with the transfer. This would be paid on top of the
value of the reserves. The ‘associated cost’ referred to is therefore this premium plus the value of
any investment income effectively lost if the transferred value of the reserves uses a discount rate
which turns out to be too low.

For example, suppose a general insurer has a block of business and the discounted value of
expected future claims in respect of this business is $100m. If the regulations prohibited the
discounting of future claim reserves, the regulatory provisions would be somewhat higher, say
$150m.

The Actuarial Education Company © IFE: 2019 Examinations


Page 42 SP7-06: Reinsurance products – types

The general insurer could seek to reinsure this block of business. It would pay the reinsurer a
premium. This should be sufficient to meet expected claims (ie $100m in this case) plus the
reinsurer’s fee (say $10m). So, the insurer’s assets would decrease by $110m. However, its
liabilities would decrease by the amount of the provisions now passed to the reinsurer, ie $150m.
Therefore, the general insurer’s reported financial position has been improved.

If the actual claims experience is such that the $100m is more than is required to meet the actual
claims, the excess could be returned as a profit commission. If the $100m is less than the actual
claims, then (depending on the terms of the arrangement) the insurer might be required to pay
an additional amount to the reinsurer.

© IFE: 2019 Examinations The Actuarial Education Company


SP7-06: Reinsurance products – types Page 43

7 Capital market products

7.1 Types of capital market products


In this section we discuss products where risk is transferred to the capital markets rather than
insurance markets. We will discuss the following products:
 committed (or contingent) capital
 securitisation
– insurance-linked securities
– credit securitisation
– motor securitisation
 weather derivatives.

7.2 Committed (or contingent) capital


Committed capital or contingent capital is based on a contractual commitment to provide
capital to an insurer after a specific adverse event occurs that causes financial distress.
The insurer purchases an option to issue its securities at a predetermined price in the case
that the defined situation occurs, on the understanding that the price would be much higher
after such an event.

If the defined event occurs, leading to financial distress of the insurer, then the price of the
insurer’s securities will fall (ie it will be more expensive to raise capital by issuing securities). The
option will allow the insurer to sell its securities after the adverse event at a higher price than
their market price.

For example, if the securities might have a current market value of $100, then the insurer might
fix the predetermined price (ie the strike price of the option) at $100. Following the adverse
event, the market value of the securities might fall to $80, however, the insurer will still be able to
issue such securities at the higher price of $100.

There may be one or more triggers that have to occur before the option can be exercised, in order
to avoid moral hazard.

Contingent capital provides a mechanism of ensuring that, should a particular risk event happen,
capital will be provided. As such, it is a cost-effective method of protecting the capital base of an
insurance company. Under such an arrangement, capital would be provided as it was required
following a deterioration of experience (ie it is provided when it is needed).

The Actuarial Education Company © IFE: 2019 Examinations


Page 44 SP7-06: Reinsurance products – types

7.3 Securitisation
You may have met securitisation in an earlier subject.

Purpose of securitisation
Securitisation has two main purposes:
 Risk management – to transfer insurance risk to the banking and capital markets.
It is often used for managing risks associated with catastrophes, as the financial markets
are large and capable of absorbing catastrophe risk.
It involves turning a risk into a financial security, eg as in a catastrophe bond.
 Capital management – to convert illiquid, inadmissible assets into liquid admissible assets,
hence improving the balance sheet.
Almost any assets that generate a reasonably predictable income stream can in theory be
used as the basis of a securitisation. Examples of illiquid assets that could be securitised
are:
– future profits, eg on a block of in-force insurance policies
– mortgages (and other loans).
Each of these could be securitised into tradeable instruments (eg bonds), in order to raise
capital. The owner of the assets issues bonds to investors (eg pension funds, insurance
companies and banks) and the future cashflow stream generated by the secured assets is
then used to meet the interest and capital payments on the bonds.
There is typically risk transfer as the repayments on the bonds are made only if, for
example, the future profits emerge or mortgage repayments are made.

For example, a portfolio of mortgage loans owned by a bank could be pooled together and the
cashflows from these mortgages used to service the interest and capital payments on a bond.
Securitisation of this type that had been backed by sub-prime mortgages in the US, was the focus
of much attention during the sub-prime crisis and credit crunch.

Operation of securitisation
In simple terms, a securitisation works as follows:
1. An investor purchases a bond from the insurance company and therefore provides a sum
of money to the insurer.
2. The repayment of capital (and possibly of interest) is contingent on:
– a specified event not happening, eg an earthquake measuring 6.5 on the Richter
scale not happening, or
– the portfolio of insurance business (upon which the bond is securitised) producing
adequate profits.

© IFE: 2019 Examinations The Actuarial Education Company


SP7-06: Reinsurance products – types Page 45

3. If the event does happen (eg the aforementioned earthquake occurs), or inadequate
profits are made from the securitised business, the insurer may default on the interest
and capital payments due under the bond:
– in the case of securitising a particular risk, the insurer can use the sum of money
provided from the investor (in purchasing the bond) to cover the cost of claims
arising from the earthquake
– in the case of securitising a block of business, the poor experience of the business
has been passed directly to the investor.
4. If the event does not occur or the business makes adequate profits, the investor gets their
interest and capital back in the normal way.

In practice, the direct link between the investor and the issuer is broken by a special purpose
vehicle (SPV), which is a separate legal entity that sits between the parties. Where it is a portfolio
of business that is being securitised, the securitised assets are transferred into this vehicle.

Question

Explain why a special purpose vehicle is used in practice.

Solution

The existence of a separate vehicle with separate ownership of the securitised assets provides
better security and greater transparency for investors in the securitisation.

This may seem like a particularly high-risk investment. It is. However, as long as the expected
return on the investment is commensurate with the investor’s required (risk-adjusted) rate of
return, then a market for such an investment will exist.

The rationale is that insurance catastrophe risk or the risk of underperformance of the securitised
business, is not correlated with investment market risks and so there is a benefit to the capital
market in the diversification of risk achieved in purchasing such investments.

The banking and capital markets are used because of capacity issues and because the risks
involved are ones with which the banking and capital markets are comfortable.

A key point to note about securitisation is that it is making insurance products look much more
like banking products. The reverse, often called insuritisation, is making banking products look
more like traditional insurance.

The Actuarial Education Company © IFE: 2019 Examinations


Page 46 SP7-06: Reinsurance products – types

We now look at three specific types of securitisation:


 insurance-linked securities
 credit securitisation
 motor securitisation.

Insurance-linked securities (ILS)


Insurance-linked securities (ILS) are an innovative way of increasing (re)insurance capacity.
The valuation and performance of these financial instruments are driven by the occurrence
(or lack of occurrence) of insurance loss events. ILS offer acquirers (such as institutional
investors and pension funds) an opportunity to invest in instruments, the return from
which are largely uncorrelated with other financial assets and macroeconomic movements
and allow them to exclude surrounding risks (such as the market risk in share prices) of
investing in reinsurance companies. For purchasers, who are typically insurers or
reinsurers, ILS provide an additional source of protection and insurance risk mitigation
instruments.

From the launch of the first securitisation in the 1990s, the ILS market has grown and
cemented its place as a complementary alternative to reinsurance, notably in the property
catastrophe reinsurance market. It has developed into what is now a reasonably liquid
catastrophe (cat) bond market.

Cat bonds developed primarily in response to the hard market (ie high premiums) of traditional
catastrophe reinsurance in the 1990s.

These cat bonds allow (re)insurers to transfer high severity low probability catastrophic
risks to the capital market and spread them among many investors: if the specified
catastrophic risk is triggered, the bondholders typically forfeit the interest and principal on
the bond to the (re)insurer. If there is no catastrophic event, or trigger event before the
maturity date of the contract, investors receive their principal investment at maturity on top
of the interest payments they have received.

This is an example of the first type of securitisation, ie as a risk management tool.

There can be many variations, and many types of trigger event.

The basic advantages of ILS are that they:


 increase insurance capacity by transferring risk to the capital markets
 mitigate counterparty risk as funds are held in a secure independent vehicle.

Credit securitisation
Although not usually involving reinsurance, insurance companies have been active in the
credit securitisation markets.

Their main roles have been:


 enhancing the creditworthiness of debt instruments

 providing capital relief to banks by insuring loan portfolios

 providing credit protection to companies.

© IFE: 2019 Examinations The Actuarial Education Company


SP7-06: Reinsurance products – types Page 47

Enhancing the creditworthiness of debt instruments / providing capital relief to banks

Consider a bank securitising some of its loan portfolios. The interest and capital repayments
under the loans will be securitised and used to pay the interest and capital repayments under the
debt instruments (ie bonds).

Investors will require a return on the bonds that is adequate to compensate them for the risk of
default. The bank may want to keep the return on the bond as low as possible, therefore it must
try to ensure that the bond is relatively secure. In order to do this, it must either securitise its
best quality loans (ie the loans with the lowest risk of default), or it must securitise a large number
of loans relative to the number of bonds issued (in which case, even if the loans default, there will
still be an adequate number of bonds left with which to make payments on the bonds).

The first of these options may not be available if the bank does not have (or has already
securitised) a portfolio of ‘safe’ loans. The second option may be undesirable, because
effectively, the bank is using up a lot of its business in the securitisation, which will reduce the
profits it receives from the loans that it does not securitise.

A third option is to use insurance to reduce the credit risk of the bonds. The bank insures the
bonds so that their return is guaranteed (as long as the insurer does not default). If the payments
under the loan portfolio are not sufficient to meet the interest and capital payments under the
bond, then the insurance will kick in and make the payments to the investors.

Having insurance as an underlying guarantee will enhance the creditworthiness of the debt. This
will help to ensure that the bank does not need to pay a very high rate of return on the bonds, or,
equivalently, does not need to sell them cheap. It should therefore be able to sell the bonds at a
relatively high price, thus maximising the capital relief provided by the securitisation. This needs
to be weighed up against the cost of insuring the bad debt.

This type of arrangement falls into the category of capital management, as described above.

Providing credit protection to companies

There are numerous types of credit securitisation arrangements, although the basic contract is a
credit default swap, which is essentially an agreement to compensate the ‘insured’ (ie the buyer
of the swap) if a specified credit event occurs (eg bankruptcy or loan default of another
company).

Note that for each of these arrangements, the insurer is not usually one of the two parties
involved in the securitisation itself. Instead, it is a third party providing insurance against the risk
of default by another party.

These alternative risk transfer (ART) solutions use derivative products available in the
capital markets, in addition to variations on traditional trade credit insurance.

The Actuarial Education Company © IFE: 2019 Examinations


Page 48 SP7-06: Reinsurance products – types

Motor securitisation
Another capital market product is motor securitisation (where certain aspects of a motor
insurer’s portfolio risks are passed to the investment market).

The insurer issues a bond where the coupon payments depend on the claims experience of the
insurer’s motor portfolio. If the insurer experiences poor claims experience, it may forego some
or all of its repayments. Thus, the insurance risk is transferred to the capital markets instead of to
the reinsurance market.

As with other debt issues, these bonds are tradable financial instruments.

7.4 Weather derivatives


Weather derivatives are another example of the insurance industry’s response to a hard and
uncertain insurance market. Strictly speaking, weather derivatives are insurance rather than
reinsurance.

This is where standard derivatives techniques, such as put and call options and swaps, are then
used to make a derivative contract based on the weather.

For example, energy companies’ earnings are very dependent on the weather, and the companies
are likely to want to reduce the resulting volatility of their profits.

This can be achieved by using weather derivatives. The solution can be based on any
weather-related peril, but perhaps temperature is the most common.

Here, the payment is based on heating degree days, or cooling degree days. A heating degree
day, for example, is the number of degrees by which the day’s average temperature falls below
some reference temperature. Payments from weather derivatives are based on the accumulated
value of degree days over a period of time.

The advantages of using a derivative approach as opposed to traditional insurance are:


 there is no need for an insurable interest
 there is no need to understand the underlying business for which cover is being
purchased
 there is no need to prove the extent of the loss to a claims handler.

© IFE: 2019 Examinations The Actuarial Education Company


SP7-06: Reinsurance products – types Page 49

8 Glossary items
Having studied this chapter you should now read the following Glossary items:
 Adverse development cover
 Aggregate excess of loss reinsurance
 Balance of a reinsurance treaty
 Catastrophe reinsurance
 Excess and surplus lines insurance
 Expected maximum loss (EML)
 Financial engineering
 Financial risk reinsurance, finite risk insurance or reinsurance
 Hours clause
 Increased limit factors
 Line
 Loss portfolio transfer
 Original gross premium income (OGPI)
 Over-riding commission
 Possible maximum loss (PML)
 Profit commission
 Quota share (QS) reinsurance
 Rate on line
 Reinstatement
 Retention
 Return commission
 Risk excess of loss reinsurance
 Stability clause
 Stop loss reinsurance
 Surplus lines insurance
 Surplus reinsurance
 Time and distance reinsurance
 Working layer.

The Actuarial Education Company © IFE: 2019 Examinations


Page 50 SP7-06: Reinsurance products – types

The chapter summary starts on the next page so that you can
keep all the chapter summaries together for revision purposes.

© IFE: 2019 Examinations The Actuarial Education Company


SP7-06: Reinsurance products – types Page 51

Chapter 6 Summary
Quota share
Quota share is proportional reinsurance whereby the premiums and claims for all risks
covered by the treaty are split in a fixed proportion. The reinsurer pays return and override
commission to the insurer. Profit commission may also be payable.

The cedant’s experience (in terms of loss ratios) will be the same before and after
reinsurance. The reinsurer will have proportionately the same underwriting experience as
the cedant.

Quota share:
+ spreads risk, increasing capacity and encouraging reciprocal business
+ directly improves the solvency ratio (without losing market share)
+ is administratively simple
+ may provide commission that helps with cashflow
– cedes the same proportion of low and high variance risks
– cedes the same proportion of risks, irrespective of size
– passes a share of any profit to the reinsurer
– is unsuitable for unlimited covers.

Surplus
Surplus is proportional treaty reinsurance whereby the proportion of risk covered varies
from risk to risk depending on the size and type of risk.

The EML for a risk is used in assessing the proportion of the risk to reinsure, defined in terms
of ‘lines’. If k lines are used for a risk then premiums and claims are split in the proportion
1:k.

The width of one line represents the amount the insurer would pay if a claim equal to the
EML occurred. This amount is called the retention (r). Therefore: EML  (1  k)  r .

A surplus treaty will usually specify a maximum number of lines and a minimum and
maximum retention. Higher levels of cover can be obtained by purchasing a second (and
third, and fourth) surplus treaty.

The cedant and reinsurer will have different experience: smaller risks may be retained in full
by the cedant, whereas larger risks may be covered primarily by the reinsurer.

The Actuarial Education Company © IFE: 2019 Examinations


Page 52 SP7-06: Reinsurance products – types

Chapter 6 Summary continued


Surplus:
+ enables the insurer to fine-tune its exposure
+ enables the insurer to write larger risks
+ is useful for classes where wide variation can occur in the size of risks
+ helps to spread risks
+ may provide commission that helps with cashflow
– requires more complex administration
– is unsuitable for unlimited covers and personal lines cover.

Excess of loss
The reinsurer covers the risk (or a proportion of it) between defined layers, the limits of
which are often indexed for inflation (using a stability clause). The insurer may choose to
have a number of layers of cover with different reinsurers.

Once the layer of cover has been ‘burnt through’, it will need to be reinstated, which might
require a further reinsurance premium to be paid.

The cedant’s and reinsurer’s experience will be different and will depend on the distribution
of large losses.

There are three main types of excess of loss reinsurance:


 risk XL – this relates to individual losses and is usually written by treaty
 aggregate XL – this relates to cumulative losses, where the aggregation may be by
event, by peril or by class
 catastrophe XL – this is a form of aggregate XL covering severe losses within the
hours clause that result from a specified event.

Excess of loss:
+ allows the insurer to accept risks that could lead to large claims
+ reduces the risk of insolvency from a large claim, an aggregation of claims or a
catastrophe
+ reduces claim fluctuations (and so smooths results)
+ helps to make more efficient use of capital.

Stop loss
Stop loss is a specific type of aggregate XL, which covers against very bad experience across a
whole account over a defined time period. The limits are usually defined as loss ratios (ie as
percentages of premiums).

© IFE: 2019 Examinations The Actuarial Education Company


SP7-06: Reinsurance products – types Page 53

Chapter 6 Summary continued


Finite risk (or financial) reinsurance
The main feature of a financial reinsurance contract is that it involves only a small element, if
any, of transfer of insurance risk from the cedant to the reinsurer. Financial reinsurance was
devised primarily as a means of improving the apparent accounting position of the cedant.

The following are examples of finite risk reinsurance products:


 time and distance policies – the insurer pays the reinsurer a premium and in return,
the reinsurer pays an agreed schedule of claim payments; this has the effect of
discounting the reserves of the insurer for the time value of money
 spread loss covers – the insurer pays an annual or single premium to the reinsurer
for the coverage of specified claims; these may be used to provide liquidity and
security to the insurer and may be used for catastrophes
 financial quota share – this is quota share (as described above) purchased in order to
obtain reinsurance commissions for financing assistance
 structured finance – these often provide the insurer with a credit enhancement,
which lowers the cost of borrowing
 industry loss warranties – these are a type of reinsurance that pay out based on
industry losses rather than losses to individual insurers.

Run-off reinsurance
Run-off reinsurance solutions focus on the full-scale risk transfer of reserve development
risks.

Adverse development covers involve the purchase of reinsurance cover for the ultimate
settled amount of a block of business above a certain pre-agreed amount. Reserves are
maintained by the insurer.

Loss portfolio transfers are not a form of reinsurance. They involve the transfer of liability
for a specified book of business from one insurer to another. Reserves are transferred to the
new insurer along with all remaining exposure to the business.

The Actuarial Education Company © IFE: 2019 Examinations


Page 54 SP7-06: Reinsurance products – types

Chapter 6 Summary continued


Capital market products
Committed (or contingent) capital is a contractual commitment to provide capital to an
insurer should a specific adverse event occur that causes the insurer financial distress.

Securitisation may be used to manage risk or capital. Examples of securitisation include:


 insurance-linked securities (ILSs) – often in the form of catastrophe bonds, where the
issuer can default on the interest and capital payments on the bond if the catastrophe
occurs
 credit securitisation – these may enhance the creditworthiness of debt instruments
and provide capital relief to banks / credit protection to companies
 motor securitisation – certain aspects of a motor insurer’s portfolio risks are passed to
the investment market.

© IFE: 2019 Examinations The Actuarial Education Company


SP7-06: Reinsurance products – types Page 55

Chapter 6 Practice Questions


6.1 Define, briefly, quota share reinsurance.

6.2 A direct writer has a surplus reinsurance treaty with a maximum retention of £80,000 (there is no
minimum retention level) and a maximum of 8 lines to be passed to the reinsurer. The following
risks all gave rise to claims. In each case, calculate the range of possible amounts paid by the
reinsurer to the direct writer:
Risk EML Claim
1 £240,000 £12,000
2 £480,000 £12,000
3 £100,000 £80,000
4 £720,000 £600,000
5 £80,000 £100,000
6 £160,000 £200,000

6.3 (a) List several risks relating to the direct writer’s operations that a reinsurer would face if
they made stop loss cover available.

(b) Given these, explain why the direct writer would probably not want stop loss in practice.

(c) Suggest two broad responses to the risks in (a) that the direct writer may well find
preferable in practice.

6.4 If available, explain why stop loss reinsurance is normally arranged on a losses-occurring basis.

The Actuarial Education Company © IFE: 2019 Examinations


Page 56 SP7-06: Reinsurance products – types

6.5 A company has the following reinsurance treaties for its industrial fire business written in a
certain year:
A 10 line first surplus treaty, with a fixed retention of £250,000
B 10 line second surplus treaty
C £200,000 XS of £200,000 on individual claims on net retention after A and B
D £400,000 XS of £400,000 on individual claims on net retention after A and B
E Catastrophe XL of £50m XS of £25m.

(Note: the size of each line under the second surplus treaty is the same as that under the first
surplus treaty.)

There is a stability clause operating on both limits of treaties C and D. The index in the year the
reinsurance treaties were written was 130. The treaties operate in the order given. There were
10 claims from policies covered by these treaties (amounts in £000) as follows:

Claim EML Claim Index


1 5,000 2,000 132
2 1,000 500 135
3 4,000 10,000 137
4 500 230 138
5 1,500 100 140
6 300 300 143
7 750 3,000 145
8 5,250 101 147
9 100 50 150
10 250 250 153

(i) State the circumstances under which D would need to make a payment.

(ii) Suggest two examples of events that might give rise to E having to make a payment.

(iii) State the main benefit to the company of treaties A and B.

(iv) Explain the proportion of premiums, ignoring commission, that gets paid to B.

(v) List four different (plausible) ways the company might react if asked to provide cover for a
risk with an EML of £8m.

(vi) Calculate how much each company must pay for each claim shown.

(vii) Recalculate how much each company must pay in the following circumstances:
(a) The retention under the surplus treaty is not fixed and the company chose to cede
the maximum amount for the risk that led to claim 3.
(b) The index at the time of claim 10 is 163.

© IFE: 2019 Examinations The Actuarial Education Company


SP7-06: Reinsurance products – types Page 57

6.6 List items that should be included in a catastrophe excess of loss reinsurance treaty.

6.7 List the main features of stop loss reinsurance.

6.8 A motor insurer has the following reinsurance cover:


 A 20% quota share (QS) treaty with reinsurer X.
 An excess of loss (XOL) reinsurance treaty whereby Y meets claims amounts falling
between £0.5m and £1.25m on the insurer’s net account.
 A further XOL reinsurance treaty whereby Z meets claims amounts exceeding Y’s upper
limit on the insurer's net account.

(i) Calculate the amounts paid by each of the four parties following a gross claim of £2m.

(ii) If the motor insurer's situation in (i) is repeated except that the XOL reinsurance treaties
now act on the original gross claim (before the operation of the quota share treaty),
calculate how much each party will pay.

(iii) The situation in (ii) is now repeated except that the two XOL treaties have been drawn up
to include a stability clause which has resulted in all the limits increasing by 8% pa
compound to the date the claim is paid 9 years after the treaty was established. Calculate
how much each party will now pay if the claim amount paid is again £2m.

6.9 Explain how financial quota share can benefit an insurer.

6.10 Describe:
(i) spread loss covers
(ii) structured finance arrangements
(iii) industry loss warranties (ILWs)
(iv) committed capital arrangements.

6.11 A €2m xs €1m excess of loss reinsurance treaty has the following terms and loss history in the
Exam style
year it was written:
 up front premium of €200,000
 2 reinstatements, the first at 120% additional premium and the second at 150% additional
premium.

In chronological order the only losses (from ground up) to potentially impact this treaty are:
1. €2.5m
2. €5m
3. €2m
4. €8m

The Actuarial Education Company © IFE: 2019 Examinations


Page 58 SP7-06: Reinsurance products – types

(i) Calculate how much of each loss is recoverable and the reinstatement premiums
generated by each loss. [4]

(ii) Calculate the ‘rate on line’ of this contract. [2]


[Total 6]

6.12 A medium-sized London Market insurance company has specialised for many years in industrial
Exam style
and commercial fire business. It has been generally successful, but its solvency ratio has reduced
significantly, because of expansion rather than losses. A new chief executive and finance director
have recently been appointed, neither of whom has an insurance background.

You attend a board meeting and at the meeting, the new chief executive director says that they
have heard that time and distance reinsurance policies are helpful for London Market insurers.
The finance director suggests stop loss protection.

Outline the points you would make to the directors on the suitability and practicality of these
reinsurance policies. [7]

6.13 You are an actuary working for a general insurance company that sells a small volume of high
Exam style
layer product liability insurance.

One of the directors is worried about the reserves deteriorating so their value falls below that of
the actual unpaid claims. They have suggested issuing an insurance-linked security (ILS) to the
capital markets that will provide funds in the event of a deterioration of reserves.

Discuss the suitability of issuing such an ILS to protect against reserve deterioration and suggest
any possible alternatives. [5]

6.14 Define the following terms and phrases used in general insurance:
Exam style (i) working layer
(ii) reinstatements
(iii) increased limit factors
(iv) financial risk reinsurance. [4]

6.15 A general insurance company has a four-line surplus treaty with a maximum retention of
Exam style
£100,000.

(i) State the maximum size of risk that can be placed under this treaty. [1]

(ii) For an individual risk with an EML of £300,000, the company retains £80,000, placing the
rest of the risk under the treaty. During the year a claim for £270,000 is made in respect
of this risk. Calculate the recovery made from the reinsurer. [1]
[Total 2]

© IFE: 2019 Examinations The Actuarial Education Company


SP7-06: Reinsurance products – types Page 59

Chapter 6 Solutions
6.1 Quota share is a form of treaty reinsurance whereby all the premiums and claims (for risks within
the terms of the treaty) are split in a fixed proportion.

6.2 Assume (a) that the direct writer kept £80,000 of risk in each case, and then (b) that they ceded
the largest possible proportion. If they cede the largest possible proportion then they must use
8
all 8 lines. In which case the percentage ceded will be  88.9%.
9

The amount recovered could then be anywhere between these limits:


1. (a) £8,000 (b) £10,667
2. (a) £10,000 (b) £10,667
3. (a) £16,000 (b) £71,111
4. (a) £533,333 (b) £533,333
5. (a) zero (b) £88,889
6. (a) £100,000 (b) £177,778

6.3 (a) Risks to a stop loss reinsurer


 the direct writer’s premiums may be under-priced (eg a competitive market)
 poor underwriting by the direct writer
 poor premium rating structure leading to adverse selection
 poor claims experience (eg catastrophes and/or large claims)
 generally adverse claims experience (ie random event)
 poor claims control

(b) So why not take stop loss in practice

The reinsurer would need to charge the insurer a very high premium to cover themselves against
the risks.

(c) Better moves for the direct writer

 Set up tight internal controls to ensure good premium rating, underwriting and claims
control.
 Buy particular types of reinsurance to guard against specific events (ie buy individual
excess of loss and catastrophe excess of loss as needed).

The Actuarial Education Company © IFE: 2019 Examinations


Page 60 SP7-06: Reinsurance products – types

6.4 Stop loss cover protects the direct writer against many claims arising over an accounting period.
For a company using normal accident-year accounting the corresponding exposure period is the
accident year. Hence a losses-occurring basis would be used (except when using a funded
accounting basis, eg at Lloyd’s, when a policies-incepting basis would be used).

The concepts behind accident-year and funded accounting will be described in detail later in the
course.

6.5 (i) When D makes a payment

D would need to make a payment when the gross claim is much bigger than the EML.

This is because the company has 20 lines on the surplus treaties, it does not expect to retain a
liability for more than the retention of £250,000 after treaties A and B have applied (ie the
maximum expected claim would be £5,250,000). D pays out only if the amount retained after A
and B exceeds £400,000 (plus indexation). This can happen only if a claim exceeds the EML by a
factor of at least:
400  Index/250  130 = 1.6  Index  130, ie by at least 60%

This means that the EML must have been underestimated and that the sum insured (ie the
maximum possible payout) must also have been at least 60% higher than the EML in order to
cover the full claim.

Mathematically, D’s payment is:

min max retained claim after A and B  400  index / 130,0  ,400  index / 130 

A second circumstance would be if either or both of A and B defaulted on recoveries to the


company. (Although it would depend on how this eventuality had been dealt with in the treaties.)

A and B would probably be worried about the existence of the cover with D as this might imply
that the direct writer is not confident about calculating reliable EMLs.

(ii) Events where E makes a payment

 An extremely bad storm causing damage to many properties (if cover extends to storm
damage).
 An enormous explosion that does far more damage to more buildings than could have
been anticipated (and the company has a concentration of risk in the area).

In reality, the answer is almost certain to only be true if either or both of A and B default.
Otherwise, it would require far more than 100 large claims from one incident for the combined
retentions to exceed the excess point of the catastrophe cover (and this is pretty much impossible).

(iii) Main benefit of treaties A and B

The company can write insurance for any property up to an EML of £5.25m without having to
involve another party at the underwriting stage (otherwise the company would be limited to
£0.25m).

© IFE: 2019 Examinations The Actuarial Education Company


SP7-06: Reinsurance products – types Page 61

(iv) Proportion of premiums paid to B

We cannot tell on a global basis, as the proportion varies from risk to risk. For a policy with EML
£3m, the proportion would be 1/12th, whereas with EML of £5m, the proportion is 9/20ths.

(v) Reaction to a risk with an EML of £8m

For a risk with EML of £8m, the company could:


 decline the risk
 find coinsurer(s) for £2.75m
 find facultative proportional reinsurance for 34.4% of the risk (ie 2.75/8), operating before
the treaty with A
 have XL cover for amounts above £5.25m (which operates first, ie on gross claims).

Note that it might instead be possible to keep the residual risk after ceding the maximum number
of lines to both surplus reinsurers. However this will depend on the terms of the surplus treaties.
(C and D would not be too happy either.)

(vi) Amount each company must pay (in £000):

No. EML Claim Index Co. A B C D


1 5,000 2,000 132 100 1,000 900 0 0
2 1,000 500 135 125 375 0 0 0
3 4,000 10,000 137 211 6,250 3,125 211 203
4 500 230 138 115 115 0 0 0
5 1,500 100 140 17 83 0 0 0
6 300 300 143 220 50 0 30 0
7 750 3,000 145 331 2,000 0 223 446
8 5,250 101 147 5 48 48 0 0
9 100 50 150 50 0 0 0 0
10 250 250 153 235 0 0 15 0

Company E would make no payments.

Method

Take, for example, claim 3.

First fix the proportions, using the EML and retention:


4,000 / 250 = 16, hence a total of (16 – 1) lines will be required.

Proportions of (premium and) each claim will be Co : A : B = 1 : 10 : 5

The Actuarial Education Company © IFE: 2019 Examinations


Page 62 SP7-06: Reinsurance products – types

Then consider the actual claim (which at 10,000 is way over the EML):

Portions of claim give: Co 625, A 6250, B 3125.

Given the index at 137, excess points on the XL treaties are currently 200  137/130 = 211 (for C)
and 422 (for D). Hence:
C pays 422 – 211 = 211
and D pays 625 – 422 = 203.

The upper limit of the treaty with D is not breached, so Co pays 211 (the excess on the treaty with
C).

Check, the portions of the claim add to 10,000, as required.

NB for claim 7 the upper limit of the treaty with D is breached.

(vii) Alternative calculations

Part EML Claim Index Co. A B C D


(a) 4,000 10,000 137 211 4,762 4,762 211 54
(b) 250 250 163 250 0 0 0 0

6.6 Items in a catastrophe excess of loss reinsurance treaty:


 parties involved
 period of operation
 class of business
 geographical limits
 definition of a catastrophe
 hours clause
 excess point
 upper limit
 indexation arrangements
 limits on volume of business written
 underwriting restrictions
 premium rates
 exclusions
 procedures for payment of premiums and claims
 reinstatement arrangements
 claim payment arrangements above excess point

© IFE: 2019 Examinations The Actuarial Education Company


SP7-06: Reinsurance products – types Page 63

 arbitration clause
 termination clause
 interaction with other reinsurance arrangements
 procedures for verification of direct writer’s records.

6.7 Features of stop loss reinsurance:


 treaty for class or classes
 non-proportional
 reinsurer pays when total claims in the period (year) exceed an agreed level (loss ratio)
 cover may be for a proportion of claims only
 unusual unless some link between direct writer and reinsurer …
 … or where reinsured has little/no influence over underwriting and claim settlement
(eg crop insurance)
 as cover is against all adverse experience (eg poor underwriting, large claims,
catastrophes, poor experience, poor claims settlement), it could be expensive.

6.8 (i) Payments

X takes 20% of all claims. Therefore X pays 0.2  £2m = £0.4m.

The net claim after QS exceeds Y’s band of cover. Hence, Y must pay £0.75m.

The net claim after QS is £1.6m (ie 2  0.4 ) and we have established that Y pays £0.75m.

Therefore the insurer has to pay the slice below Y’s band of £0.5m.

This leaves £0.35m to be paid by Z (ie 1.6 – 0.75 – 0.5).

(ii) XOL acting first

The insurer pays 80% of £0.5m = £0.4m.

The reinsurer X pays 20% of £0.5m = £0.1m.

Y still pays £0.75m.

Z pays everything over £1.25m, ie £0.75m.

(iii) Stability clause

The calculations can be simplified because (1.08)9 almost exactly equals 2.

Therefore Y’s boundaries become £1m and £2.5m.

Hence, following the same logic in (ii), the insurer and X share the claim below Y’s level of cover in
the proportion 80:20, ie the insurer pays £0.8m and X pays £0.2m.

The Actuarial Education Company © IFE: 2019 Examinations


Page 64 SP7-06: Reinsurance products – types

The total claim is now less than Y’s upper limit, hence Z pays nothing.

The remainder of the claim is £1m which is paid by Y.

6.9 Financial quota share is a quota share arrangement under which a generous commission payment
is made from the reinsurer to the insurer at the outset.

Under normal circumstances, the commission payment should cover:


 commission payments incurred by the insurer (return commission)
 the work of attracting and administering the business (override commission).

Under financial quota share, the initial commission may significantly exceed these components.

The surplus amount can be used in order to:


 help cover new business strain
 finance a particular strategy, eg the take-over of another insurer
 improve the solvency position of the insurer.

In future years, the commission payment would be less than the sum of these components to
compensate the reinsurer for the high initial commission.

Since financial quota share is essentially a quota share arrangement, it will have the usual benefits
of a standard quota share arrangement. In particular, it:
 will spread risk
 will increase the capacity to accept risk
 may encourage reciprocal business
 will directly improve the solvency ratio without losing market share
 is administratively simpler than alternatives.

The insurer may also benefit from the expertise of the reinsurer, particularly in pricing, since the
reinsurer will have a significant interest in the premiums charged.

6.10 (i) Spread loss covers

Spread loss covers involve the insurer paying annual or single premiums to the reinsurer for
coverage of specified claims. These accumulate with interest (contractually agreed) in an
experience account; the balance of which is settled at the end of the multi-year period.

These types of contracts involve very limited underwriting risk (limited practical risk transfer), but
provide the insurer with the liquidity and security of the reinsurer.

(ii) Structured finance arrangements

Reinsurers became involved in structured finance through their finite reinsurance business and
the increasing need of financial guarantee insurers and investment banks for additional capacity.

© IFE: 2019 Examinations The Actuarial Education Company


SP7-06: Reinsurance products – types Page 65

The typical financing solution provided by the reinsurer is a credit enhancement in which the
reinsurer provides a financial guarantee or credit insurance wrap to the institution borrowing
from the capital market.

Credit enhancement or financial guarantees lower the cost of borrowing.

(iii) Industry loss warranties (ILWs)

Industry loss warranties (ILWs) are a type of reinsurance where the basis of cover is not
indemnity, ie repayment of actual losses suffered.

Here one party will purchase protection based on the total loss arising from an event to the entire
insurance industry rather than their own losses.

The original size of the industry loss is used as a trigger for a recovery.

(iv) Committed capital arrangements

Committed capital is based on a contractual commitment to provide capital to an insurer after a


specific adverse event occurs that causes financial distress.

The insurer purchases an option to issue its securities at a predetermined price in the case that
the defined situation occurs, on the understanding that the price would be much higher after
such an event.

6.11 (i) Reinstatement of cover

The reinstatement premium will usually be calculated according to the cover burnt through and
the period of cover remaining.

We have no information on the timing of claims, therefore we will ignore the issue of the period
of cover remaining – although in practice it could be a significant issue.

First claim: recover €1.5m (ie €2.5m less €1m excess) from the reinsurer. Note that €0.5m of the
reinsurance cover, ie ¼ of the layer, is yet to be used. Pay a reinstatement premium of €0.18m
(ie ¾ of 120%  200,000). In practice this is likely to be offset from the reinsurance recovery.

Second claim: recover €2.0m from the reinsurer, and pay a further reinstatement premium of
€0.285m (ie (¼ of 120% + ¾ of 150%)  200,000) to reinstate full cover. Again this is likely to be
offset from the reinsurance recovery.

Third claim: recover €1.0m (ie €2m less €1m excess) from the reinsurer, and pay the remaining
reinstatement premium of €0.075m (ie ¼ of 150%  200,000). The remaining cover is ¾ of the
layer, ie €1.5m in excess of €1m.

Fourth claim: recover €1.5m (the remaining cover). No further reinstatement premium can be
paid and no reinsurance recoveries can be made on subsequent claims during the current period
of cover.
[½ mark for each correct recovery and reinstatement premium, total 4]

The Actuarial Education Company © IFE: 2019 Examinations


Page 66 SP7-06: Reinsurance products – types

(ii) Rate on line

This is defined, for non-proportional reinsurance, as the total premium charged for the
reinsurance (excluding any reinstatement premiums) divided by the width of the layer covered. [1]

0.2
Hence the rate on line is  0.1 , ie 10%. [1]
2.0
[Total 2]

6.12 Time and distance (T&D) reinsurance

This is not really a form of reinsurance at all, since there is limited protection against insurance
risk. [½]

It is a financial instrument that should be viewed as an investment, ie consider:


 security
 marketability
 matching by nature and term of liabilities
 expected returns. [½ each]

T&D arrangements are likely to offer poor returns, marketability and security compared with
those available on other available assets. Most practical arrangements tend to be longer term
than the short-tailed property claims expected by this insurer. [1]

In practice, these arrangements may be used to improve the apparent solvency position where
explicit discounting was not permitted. [½]

However, any such loopholes may have been closed, with providers now being required to
discount the return payments making the transaction pointless from a solvency viewpoint. [½]

Hence, T&D is likely to be unsuitable for the insurer. [½]

Stop loss

Stop loss policies pay out when the overall losses on the account reach a certain proportion of
premiums. Policies usually pay out on a proportion, say 80%, of losses up to some specified upper
limit. [1]

In theory, this is a good suggestion for the insurer. By limiting overall losses, the solvency margin
could be protected from large overall losses. [½]

Unfortunately it may not be a practical suggestion. By reducing the variance in the insurer’s
profitability, reinsurers are concerned that lack of management control may lead to them
experiencing large losses. [½]

Hence, reinsurers may be reluctant to provide the cover. [½]

© IFE: 2019 Examinations The Actuarial Education Company


SP7-06: Reinsurance products – types Page 67

Where the cover is available it is likely to be expensive (reducing profitability) and on


unfavourable terms (eg high deductible). [½]

Hence stop loss protection is theoretically sound but may be impractical. [½]
[Maximum 7]

6.13 Under catastrophe bonds, if a defined index is triggered, the bondholders typically forfeit the
interest and principal on the bond to the insurer. However, unlike catastrophe bonds, the trigger
for the ILS in this case is unclear. [1]

It would be difficult to construct an objective method of establishing the required change in


reserves for this account. [½]

Demand for such a bond may be low. This may be because:


 it is an unusual type of bond, compared to a more standard catastrophe bond [½]
 capital market investors may be reluctant to buy the securities because of the moral
hazard associated with the lack of an objective method of calculating the change in
reserves [½]
 it may be difficult to persuade the capital markets to invest in risks where standard
quantification tools have not been developed. [½]

On the other hand, demand for the bond might be adequately high, since this risk may provide
diversification from other assets in investors’ portfolios. [½]

It may be an expensive way of protecting against the risk of reserve deterioration:


 capital markets may also demand a higher return because this is an unusual type of risk
with which they are unfamiliar [½]
 because this is an unusual type of bond, the expense of structuring and marketing the
security may be particularly high [½]
 since the portfolio of liability business written is fairly small, the costs of an ILS may be
prohibitive. [½]

Investors would probably want the general insurance company to maintain an interest in the
reserves, so that claims are managed properly. This would reduce the effectiveness of the ILS as a
form of risk transfer. [½]

The use of an SPV (special purpose vehicle) would mitigate counterparty risk. [½]

Although the issue of an ILS would be possible in theory, it is unlikely to be viable in practice. [½]

Possible alternatives

There may be alternatives that better meet the insurer’s need, eg internal reinsurance within the
group, or conventional external reinsurance. [½]

Another alternative could be to purchase adverse development covers, however such covers
typically only protect against losses above a specified percentile. [½]
[Maximum 5]

The Actuarial Education Company © IFE: 2019 Examinations


Page 68 SP7-06: Reinsurance products – types

6.14 (i) Working layer

A layer of excess of loss reinsurance at a level where there is likely to be a fairly regular flow of
claims. [1]

(ii) Reinstatements

The restoration of full cover following a claim. [½]

Normally, the number of reinstatements, and the terms upon which they are made, will be agreed
at the outset. Once agreed, they are automatic and obligatory on both parties. [½]

(iii) Increased limit factors

These are factors which estimate the cost for a new limit as a multiple of the basic (original) limit.
[1]

(iv) Financial risk reinsurance

This is a form of reinsurance (or insurance) involving less underwriting risk transfer and more
investment or timing risk transfer from the cedant than is customary in reinsurance. [1]
[Total 4]

If you do not know your basic definitions you are throwing away potentially easy marks.

6.15 (i) The maximum EML = 4  100,000 + 100,000 = £500,000. [1]

(ii) The EML is split in the ratio 80:220 between the insurer and reinsurer respectively. Thus
the claim is split in the same proportion, with the reinsurer paying £198,000. [1]
[Total 2]

© IFE: 2019 Examinations The Actuarial Education Company


SP7-07: General insurance markets Page 1

General insurance markets


Syllabus objectives
1.3 Describe the implications of the general business environment in terms of the:

 main features of the general insurance market

 effect of different marketing strategies

 effect of fiscal regimes

 general effect of professional guidance.

Covered in part in this chapter.

The Actuarial Education Company © IFE: 2019 Examinations


Page 2 SP7-07: General insurance markets

0 The general insurance market


So far in this course we have looked at the different types of general insurance products. In this
chapter and the following chapters we consider some important features of the general business
environment and the implications that they have for general insurance business.

This chapter begins by discussing the market for general insurance, ie who are the providers of
general insurance?

Section 1 describes different types of insurance companies, and the role of the London Market,
which includes Lloyd’s syndicates. It also covers various forms of self-insuring groups.

The general insurance market is very diverse. World-wide, general insurance is provided by
insurance companies and Lloyd’s syndicates. A corporate entity may also manage its
self-insurance through a captive insurance company.

Cover is sometimes provided also by governments as insurers of last resort – though rarely
in these situations will there be prices, reserves and capital established as in the traditional
format.

In turn, insurance providers will wish to obtain reinsurance for some of their risks. This can
be obtained:

 from the London Market

 from Lloyd’s

 from specialist reinsurance companies

 from direct insurers who also write reinsurance

 in some cases, from the capital markets.

Depending on the size of a risk, a combination of these options may be used.

Section 2, on marketing strategies, describes in more detail the methods used to obtain insurance
business. This section focuses on methods used by insurance companies and in Lloyd’s.

Sections 3 and 4 discuss the effect of regulatory and fiscal regimes, and of professional guidance
on general insurance business.

In subsequent chapters we describe further external influences on general insurance business,


such as economic, legal, political and environmental factors.

© IFE: 2019 Examinations The Actuarial Education Company


SP7-07: General insurance markets Page 3

1 The major providers

1.1 The companies

Direct insurance companies


Direct insurance companies provide insurance for individuals and companies.

In this context ‘direct’ means a company that writes insurance directly for an insured
person or company as opposed to one that writes reinsurance.

Direct market companies are very active in ‘personal lines’ insurance, which is business sold to
individuals. For example, you might be a user of private motor, household, medical insurance or
travel insurance.

In the context above ‘direct’ means ‘not reinsurance’. However, the term direct can be open to
some misinterpretation. It has historically been taken to mean that there is a direct contract
between the insurer and the policyholders, as distinct from reinsurance where the policyholder
may not even have heard of the ‘final insurer’. The term direct is also used these days to describe
insurance sold direct to the public, often by phone, without going through a broker.

The alternative meaning of a ‘direct’ insurance company – one that deals directly with its
customers rather than through an intermediary such as a broker – is not meant in this
context.

Direct insurers can be divided into the following three groups:


1. Composite insurance companies — insurance companies that write both general
insurance and life insurance.
2. (Pure) general insurance companies that write many classes of general insurance.
3. Insurance companies that specialise in writing business in a selection of classes of
general insurance (or even just a single class).

Writing both life and general insurance business


Many groups contain both life insurance companies and general insurance companies.
While these groups are conveniently referred to as ‘composite insurers’, the risk-taking
companies themselves will usually be pure life assurance companies or pure general
insurance companies.

Suppose Company X wishes to provide its customers with both life insurance and general
insurance. It may, for example, establish itself as a life insurance company and write life
insurance business, and set up an arrangement with a general insurer (or insurers) that will
‘underwrite’ any general insurance business that Company X sells.

The Actuarial Education Company © IFE: 2019 Examinations


Page 4 SP7-07: General insurance markets

Question

Discuss why Company X might want to do this, rather than write general insurance business
directly.

Solution

The company may prefer another company to underwrite its general insurance business because:
 it does not need to establish itself as a general insurer, and thus avoids the administrative
expense of going through the authorisation process
 there is less need for specific general insurance expertise, eg in underwriting, claims
control or pricing, if the general insurance risks are borne elsewhere
 management can focus solely on life insurance risks, rather than spreading its expertise
too thinly
 it may not be cost-effective to set up a general insurance operation if sales volumes are
uncertain or expected to be low
 there may be tax or regulatory capital requirement advantages in this arrangement,
eg there is less need to hold capital if risk is transferred.

Alternatively, Company X may:


 write general insurance but have its life insurance business underwritten by a life insurer
 set itself up as a composite insurer so it can write both types of business (although new
composites haven’t been allowed in the UK for many years now).

Corporate structure of general insurers


General insurance companies may be:
 proprietary (ie owned by shareholders in order to make profit)
 mutual (ie owned by policyholders).

Most insurance companies are proprietary companies limited by shares. However, some
mutual insurance companies do exist; they are more common in some markets than in
others.

Reinsurance companies
Reinsurance companies provide cover for insurance providers. Again, some reinsurance
companies will specialise in only writing some types of reinsurance, while others write
many different classes of reinsurance. Some insurance groups write both direct insurance
and reinsurance.

© IFE: 2019 Examinations The Actuarial Education Company


SP7-07: General insurance markets Page 5

1.2 The London Market


When UK general insurance is discussed you will often hear of ‘the London Market’. This does not
include all general insurance companies with London phone numbers. It does mean a specialised
group of insurers, writing business along broadly similar lines, which is based in an area within the
City of London.

You can simplistically think of it as ‘Lloyd’s plus company equivalents’. That is, Lloyd’s is an
important part of the London Market, and the rest of the market is made up of companies with
buildings near Lloyd’s offering insurance similar to Lloyd’s.

The London Market is that part of the insurance market in which insurance and reinsurance
business is carried out on a face-to-face basis in the City of London. These companies tend
to be physically located close to each other.

The London Market concentrates mainly on providing insurance and reinsurance cover to
companies. It specialises in:

 the larger direct insurance risks – both property and liability – that are beyond the
capability of other direct insurance companies (for example, energy and aerospace
risks)

 international risks

 reinsurance.

Most UK insurers who transact this type of business, or foreign insurers who transact this
business in the UK, therefore operate within the London Market. Consequently, the
participants in the London Market would include:

 Lloyd’s syndicates (see Section 1.3)

 UK subsidiaries or branches of overseas insurance or reinsurance companies

 the reinsurance departments of UK composite companies, or reinsurance


subsidiaries of such companies

 small professional reinsurance companies set up by (or acquired by) large broking
firms for the specific purpose of transacting London Market business

 captives (see Section 1.4)

 P&I Clubs (see Section 1.4)

 companies owned by a group of insurance or reinsurance companies

 pools (see Section 1.4).

Over 100 companies operate within the London Market, as well as many syndicates in Lloyd’s.

Although the London Market concentrates mainly on providing insurance and reinsurance
cover to companies, Lloyd’s does also provide private motor insurance and some other
personal lines cover.

The Actuarial Education Company © IFE: 2019 Examinations


Page 6 SP7-07: General insurance markets

1.3 Lloyd’s syndicates

What is Lloyd’s?
Lloyd’s origins lie in a coffee shop run by Edward Lloyd in the late 1680s. Lloyd’s coffee shop
became an informal meeting place for merchants, ship owners and ships’ captains. The shop soon
became known as a source of reliable shipping news, coffee and as a place for merchants to find
private individuals who would share in the risks of proposed ventures.

Edward Lloyd did not carry out any insurance himself. His role was to provide the premises (and
the coffee) for his clients. Similarly today, Lloyd’s itself does not carry out insurance business.
Instead it provides the facilities for its members to carry out insurance.

Lloyd’s of London is a unique insurance institution. It began in Edward Lloyd’s coffee shop
in the late 1680’s before being incorporated by the Lloyd’s Act of 1871. It is not an
insurance company  it is a marketplace made up of members who provide capital and
accept liability for risks that are underwritten in return for their share of any profits that are
earned on those risks.

Names
Historically, all the members of Lloyd’s were individuals, known as ‘Names’. However, in
1994, companies were allowed to become Names for the first time. This is called ‘corporate
capital’ and the companies themselves are known as ‘corporate Names’. A corporate Name
is a limited liability company whose only business is to provide capital to Lloyd’s.

‘Limited liability’ means that the corporate member cannot lose any more than the capital it has
provided. This is discussed in more detail below.

Nowadays, corporate capital accounts for more than 90% of the capital in the Lloyd’s
market.

Some insurance companies and reinsurance companies are active in Lloyd’s through
corporate Name subsidiaries.

Syndicates
Lloyd’s members conduct their insurance business in ‘syndicates’; groups of members who
collectively co-insure risks. At the start of 2017 there were 105 syndicates at Lloyd’s. The
syndicates employ underwriters to write insurance and reinsurance business on behalf of
the members. Individual syndicates often specialise in particular types of insurance. Each
member who belongs to a particular syndicate will contribute capital to that syndicate and
will accept a portion of the insurance risks written by the syndicate; the share of each
member being predetermined according to the amount of capital they have contributed.
The profit or loss made by the syndicate is then shared among its members in these
proportions. The member’s share of a syndicate is fixed during an underwriting year but
may change from year to year.

© IFE: 2019 Examinations The Actuarial Education Company


SP7-07: General insurance markets Page 7

Limited liability
Traditionally, individual Names could not limit their liability in respect of their exposures at
Lloyd’s: they were potentially liable up to the full extent of their personal wealth. However,
when corporate capital was admitted, this was on a limited-liability basis so that a corporate
member’s maximum loss was limited to the amount of capital it provided to Lloyd’s. It
subsequently became possible for individual Names also to limit their liabilities and many
have opted for this approach.

Size of syndicate
Lloyd’s members often spread their exposure by belonging to a number of different
syndicates. However, some corporate members only underwrite through a single syndicate
and some syndicates have only one Name; a corporate member that provides all the
syndicate’s capital.

Syndicates vary in size from just one Name to over a thousand. A syndicate is analogous to an
insurance company and the Names belonging to the syndicate are broadly analogous to
shareholders in the insurance company.

Question

From what we have covered on Lloyd’s so far, suggest the key difference between an individual
Name’s involvement in a syndicate and a shareholder’s involvement in an insurance company.

Solution

Some Names are liable for the whole of their personal wealth. If an insurance company has
problems, the shareholders may lose their original investment, but they will have no further
liability. However, Names would have to use their entire personal wealth to bail out their share of
the losses of a syndicate in trouble. Now that most individual Names have opted for limited
liability, there are fewer individuals in this position.

Access to global licences


One key advantage in operating a Lloyd’s syndicate is access to Lloyd’s global licences that
enable Lloyd’s syndicates to write business almost anywhere in the world.

Lloyd’s is licensed or eligible to undertake insurance business in around 80 countries. This,


together with the strong international brand that Lloyd’s has, enables syndicates to start writing
business in new territories more easily than most insurance companies.

Capital efficiency
Another key advantage arises from the capital efficiency of writing business through
Lloyd’s (capital structure, FAL, Central Fund, etc).

Funds at Lloyd’s (FAL) is the capital fund of a member, ie the amount of capital, specified by
Lloyd’s, that each member must provide. This may be lodged either through physical assets or via
a letter of credit.

The Actuarial Education Company © IFE: 2019 Examinations


Page 8 SP7-07: General insurance markets

The Central Fund is a further layer of protection for policyholders. It is built up from contributions
by members and held centrally by Lloyd’s.

1.4 Self-insuring groups

Question

Define self-insurance.

Solution

Self-insurance is the retention of risk by an individual or organisation, as distinct from obtaining


insurance cover.

Some insurers are set up with the primary purpose of providing self insurance for their
owners’ or members’ insurance risks.

Captives
A captive insurance company is an insurer that is wholly owned by an industrial or
commercial enterprise and set up with the primary purpose of insuring the parent or
associated group companies and retaining premiums and risk within the enterprise.

There is another use of the term ‘captive’ in the insurance market, but this is not what is meant
here:

The term ‘captive insurance company’ is often applied to an insurance company set up to
sell insurance to the parent’s customers. This usage is sometimes made but cannot be
considered to be technically correct.

If you come across captives in the exam, you should assume the first interpretation, ie an insurer
set up for risk retention purposes, unless an alternative meaning is clear from the question.

Question

Without reading ahead, suggest conditions under which a company may consider setting up a
(correctly defined) captive.

Solution

 The cover required is not available in the traditional insurance market.


 The insurer is large enough (by amount of premiums) to go to the effort of setting up a
captive.
 The insurer is large enough (by amount of capital) to bear some risk.
 The insurance market isn’t satisfactory, eg too expensive, too restricted or tax-inefficient.
 It allows the insurer to manage risk and buy reinsurance, as required, at a reasonable
price.

© IFE: 2019 Examinations The Actuarial Education Company


SP7-07: General insurance markets Page 9

The usual reasons for setting up captives include the following:

 To fill gaps in insurance cover that may not be available from the traditional
insurance market.

 To manage the total insurance spend of large companies or groups of companies.

 To enable the enterprise to buy cover directly from the reinsurance market rather
than direct insurers.

 To focus effort on risk management.

 To gain tax and other legislative or regulatory advantages.

To facilitate the last aim, captives are usually set up in a location where it is possible to gain
such advantages. There are a number of locations that host a significant captive-insurance
industry. However, this has now perhaps become a secondary consideration, with the
primary purposes being the other ones mentioned above.

In addition to accepting the risks of their parent companies, captives may also accept
external risks on a commercial basis. In some territories this is necessary in order for
insurance premiums paid by the parent to be tax-deductible.

An authorised captive is free to provide insurance to risks other than those of its parent, providing
this does not change its main purpose. Those who do, called ‘open market captives’, often
provide insurance to the parent company’s customers.

Reinsurance may be used by the captive to limit the extent of self-insurance. For example, the
captive may obtain reinsurance to cover very large losses whilst retaining most of the risk itself.
Alternatively, the captive may be set up in order to gain direct access to the reinsurance market
with all (or most) of the risks being passed to the reinsurer.

Growth areas, in volumes of business being put through captives, include professional liability,
extended warranty, mortgage indemnity, employers’ liability and product liability. Growth is
strongest in areas where the traditional market is:
 expensive
 volatile (the premiums are very variable)
 simply not offering the cover, as with pollution, for example.

Protection and Indemnity (P&I) Clubs


P&I Clubs are mutual associations of ship owners that were originally formed to cover
certain types of marine risks (mainly liability), that could not be covered (at an acceptable
price) under a commercial marine policy.

Today, insurance for many of these risks may be found in the commercial market. However,
owing to their mutual nature and technical expertise, P&I Clubs still currently provide a
significant proportion of the world’s coverage against marine liability claims.

One of the main points here is that if a mutual can do the job just as well as an external
proprietary insurer, the mutual would usually be preferred since, other things being equal, there’s
no ‘insurer’s profit margin’ to pay for.

The Actuarial Education Company © IFE: 2019 Examinations


Page 10 SP7-07: General insurance markets

In addition to providing insurance, P&I Clubs also provide ship owners with technical
assistance in the marine market and advice on issues relating to the shipping industry.

Some of the largest P&I Clubs themselves mutualise in respect of very large claims.

The mutual organisation being referred to is the International Group of P&I Clubs which consists
of 13 P&I Clubs, representing approximately 90% of the world’s ocean-going tonnage. The
member clubs use the International Group as a mechanism to pool their larger risks and to
arrange reinsurance to help deal with very large claims. Not all P&I Clubs belong to the
International Group.

Pools
A pool is an arrangement under which the parties agree to share premiums and losses for
specific insurance classes or types of cover in agreed proportions. To some extent, all
insurance is pooling but specific pooling arrangements are sometimes used, particularly
where the risks are very large (eg atomic energy risks) or through mutual associations that
cater for an industry.

P&I Clubs, mentioned above, are a type of pool.

The critical difference between insuring with a conventional insurer and insuring with a pool
is that the insured’s liability to an insurer is limited to the premium charged, whereas the
liability to a pool will be related to the insured’s share of the total claims and other costs
that arise.

1.5 Insurance markets


All insurance markets tend to comprise the same participants: mutual and joint-stock
(ie proprietary) insurance companies. Pools are also found in a number of markets,
although not all.

The most developed insurance markets tend to be in large developed economies, notably
the UK, US, Japan, Canada, France, Germany, Italy and Spain.

Many markets have reinsurance companies, but reinsurance is more likely to be placed
internationally than direct business; notable reinsurance markets are the UK, US,
Switzerland and Germany.

The Bermuda market


One market that should be mentioned separately is Bermuda, which has become a major
international centre for insurance and reinsurance, despite not being a large economy in its
own right.

© IFE: 2019 Examinations The Actuarial Education Company


SP7-07: General insurance markets Page 11

Question

Suggest reasons why this might be the case.

Solution

Reasons for Bermuda being a major centre for insurance and reinsurance include:
 a favourable tax environment
 regulatory advantages – the aim is to have a regulatory environment that meets
international standards (eg on solvency standards) yet is attractive for (re)insurers based
in Bermuda, and so has relatively few restrictions (eg rules regarding authorisation,
investment and disclosure are relatively relaxed)
 a government that actively encourages and promotes growth of the country’s financial
industry
 as a major tourist location with (usually) good weather, it is an attractive place to live and
work
 strong historical and cultural links with the UK (which can help attract business from
UK-based companies including Lloyd’s business)
 relatively close to the US, with which it has strong business, cultural and tourist links (and
so it can gain access to this major insurance market).

A number of large insurance companies and groups are domiciled there, and some
insurance groups have moved their principal domicile there in recent years.

Bermuda had been the world’s most important domicile for captive insurance companies for
some decades; other important captive centres include the Cayman Islands, Vermont, the
British Virgin Islands, Guernsey, Barbados, Luxembourg and Dublin.

Insurance and reinsurance groups are not limited to a single market and many operate out
of multiple markets.

1.6 Non-traditional markets


The transfer of insurance risk to the banking and capital markets is known as securitisation. Some
examples of this were covered in Chapter 6.

The capital markets are increasingly involved in taking insurance risk through Industry Loss
Warranties (ILWs), catastrophe (cat) bonds, ‘sidecars’ and even traditional reinsurance
contracts. These often need more complex legal arrangements involving transformer
vehicles and interest rate swap arrangements.

Industry Loss Warranties, which were mentioned earlier in the course, are types of reinsurance or
derivative contracts through which one party will purchase protection based on the total loss
arising from an event to the entire insurance industry, rather than their own losses.

The Actuarial Education Company © IFE: 2019 Examinations


Page 12 SP7-07: General insurance markets

Question

Explain how a catastrophe bond differs from a traditional bond, give examples of relevant trigger
events and explain how catastrophe bonds transfer risk from the insurer to the purchasers of the
bond.

Solution

A catastrophe bond is like a traditional bond issued by an insurance or reinsurance company


(usually via a separate legal entity known as a Special Purpose Vehicle), but where the repayment
of capital (and possibly of interest) is contingent on a specified event not happening.

The trigger event may be related to the insurer’s losses (eg flooding resulting in claims to the
insurer of over £100m), industry-wide losses (eg storm damage causing the insurance industry
losses in excess of $200m) or objective measures of the peril’s severity (eg an earthquake
measuring six on the Richter scale or a windstorm reaching a speed of 50 metres per second).

The company has effectively passed on the insurance risk to the purchasers of the bond because:
 if the trigger event does happen, the investor does not receive the remaining coupon
payments, and so the insurer can use the sum of money provided from the investor (in
purchasing the bond) to cover the cost of claims arising from the earthquake
 if the trigger event does not occur, the investor gets their interest and capital back in the
normal way.

A ‘sidecar’ is a financial structure that is created to allow investors to take on the risk of a group of
insurance policies. It is a means of allowing investors exposure to the reinsurance market without
having to invest in existing reinsurance companies, which may have losses from previous years. A
sidecar acts like a reinsurance company but it reinsures only one cedant and the investors need to
place sufficient funds in the entity to ensure that it can meet any claims that arise.

The above structures are commonly purchased by hedge funds and other investors.

Question

Outline possible advantages of using securitisation products, such as those mentioned in this
section, to cede insurance risk.

Solution

Possible advantages of using securitisation:


 insurance risk may be uncorrelated with other risks that capital markets are exposed to,
and so capital markets are prepared to take on this risk
 no reinsurance default risk
 may be cheaper than conventional reinsurance

© IFE: 2019 Examinations The Actuarial Education Company


SP7-07: General insurance markets Page 13

 provides cover that reinsurers may not be willing to, or have the capacity to, accept
 more effective or tailored provision of risk management
 source of capital
 tax advantages.

They tend to have a very high reliance on third party models and as a result are
predominantly (though not exclusively) involved with catastrophe risk.

For example, the trigger event for a catastrophe bond may be linked to the extent to which the
modelled loss exceeds a specific threshold once the model’s parameters have been updated to
reflect the event (eg an earthquake), rather than being based on the insurer’s actual losses.

The Actuarial Education Company © IFE: 2019 Examinations


Page 14 SP7-07: General insurance markets

2 Marketing strategies
We now consider methods of acquiring insurance business.

2.1 Non-London Market business


Insurance business is obtained through:

 intermediaries such as brokers, banks and so on

 staff directly employed by the insurance provider

 the internet, telesales, post and off-the-page advertising.

If any of these terms are unfamiliar to you then read on – they will be explained below.

Intermediaries
Brokers

Brokers act as intermediaries between the seller and buyer of a particular insurance or
reinsurance contract without being tied to either party. They are likely to be paid by
commission (brokerage) from the insurer, but when placing business (under the ‘law of
agency’) legally, they are the agent of the insured.

Brokers may also carry out some functions on behalf of insurers (for example, operating
binding authorities / line slips). When carrying out these functions they are legally agents
of the insurer.

Note the above distinction between when the broker is an agent of the policyholder and when it
is an agent of the insurance company.

Binding authorities (also called ‘binders’) are contractual agreements setting out the scope of
delegated authority, allowing cover holders to enter into contracts of insurance and to issue
insurance documents on behalf of Lloyd’s managing agents.

The contract will specify the period for which insurance can be placed, the classes of
business covered and the policy wordings that are to be used.

Many agencies are paid a percentage of premiums as commission. This causes a potential
conflict of interest for the agency because it has an incentive to increase premiums without
sufficient regard for the profitability of the business.

An agency can increase its commission income by writing a very high volume of business. This can
happen, especially for price-sensitive classes, when premium rates are too low and the business is
loss-making.

Underwriting agencies represent a very large source of London Market business. Many of
these agencies were formed by brokers. In some cases, a company may establish a
specialist agency to underwrite risks on behalf of an insurer. Some agencies have been
formed with specialist risk management functions in-house to write specialist business on
behalf of an insurer.

Outside of Lloyd’s, binding authorities can be used to allow a broker to enter into contracts of
insurance and issue insurance documents on behalf of an insurance company.

© IFE: 2019 Examinations The Actuarial Education Company


SP7-07: General insurance markets Page 15

Similarly, a line slip is a facility under which underwriters delegate authority to accept a
predetermined share of certain coinsured risks on their behalf. The authority may be exercised by
the leading underwriter on behalf of the following underwriters; or it may extend to the broker or
some other agent authorised to act for all the underwriters.

Tied agents

Organisations such as banks and building societies are sometimes tied to a particular
insurer (perhaps part of the same group) and sell that insurer’s products alongside their
own. They are usually paid by commission for this service. They may also act as brokers
or have an insurance-broking subsidiary.

These options are not exclusive – some banks own insurance companies that write some
lines (for example, property and creditor), but act as broker or tied agent for other lines (for
example, motor).

Sale of a particular line of business through a tied agent is exclusive to a particular insurer. If
Insurer A sells motor policies through a tied agent, the tied agent will not also sell motor policies
from any other insurer. The tied agent may, however, sell property policies from Insurer B.

Staff directly employed by the provider


Staff employed by the provider may be paid a fixed salary, or entirely by commission, or,
more usually, by a mixture of these two methods.

Direct marketing
Some insurers employ staff in direct sales, where potential policyholders are invited,
through advertising, to make proposals by telephone or the internet, or are attracted
through cold-call selling by e-mail, post or telephone.

Methods used
All of the above methods of acquisition are used to some extent across most lines of
business, although the main method varies by country and by type of insurance. Mass
advertising in the media tends to be used for personal lines and small commercial lines of
insurance. For larger commercial risks, personal contact through the insurer’s sales force
or specialist insurance and reinsurance brokers are the more usual methods of acquisition.

The type of intermediary will often depend on the class of business. With classes such as
domestic buildings insurance, mortgage guarantee insurance and travel insurance, insurance
brokers may not be the natural intermediaries. With buildings insurance and mortgage guarantee
insurance, much of the business is sold through the building society or bank that supplied the
mortgage for the house purchase. In the case of travel insurance, the travel agent who arranges
the travel will often sell the insurance. Some insurance companies have tried to develop this
principle for the motor market by arranging for car insurance to be provided on new car
purchases.

The Actuarial Education Company © IFE: 2019 Examinations


Page 16 SP7-07: General insurance markets

It is unlikely that the methods of sale of insurance will stay static. In the quest for improved
profitability, insurers constantly review their methods of selling business. In the UK within the
recent past there have been increased attempts to sell insurance directly to the public, avoiding
the intermediaries altogether. Direct sales to the public can be made through advertising in the
media (eg TV, newspapers), by direct mail, by phone or using the internet. Making direct sales
using the phone is often called telesales. Placing advertisements in newspapers, magazines or
phone books etc is called off-the-page. The main classes affected by the growth of direct sales in
recent years have been private motor and domestic property.

In some respects, the sale of insurance is quite different from the sale of, say, toothpaste. For
insurance companies, reviewing the efficiency of intermediaries extends beyond an analysis of the
number of sales made and the expenses incurred (ie commission). The insurer will also need to
keep a close eye on the quality of the business sold, eg what was the claim experience for
business sold through each of the sales outlets?

2.2 London Market business


The London Market insurance providers, including Lloyd’s, have traditionally acquired
business through specialist brokers and, in particular, international brokers using the slip
system (also called the subscription system).

Historically, Lloyd’s syndicates could only write risks that they received through Lloyd’s brokers.
However, for certain standard proposals in personal lines insurance a syndicate may now deal
directly with the insured or a non-Lloyd’s broker.

Under the slip system in the subscription market:


1. The insured approaches a London Market broker.
2. The broker prepares a slip that shows, in a standard format, the main features of the
risk to be insured.
3. The broker shows the slip to one or more quoting underwriters, who, on the basis of
the slip and further information as appropriate, quote a premium. The broker will
know which of the many companies and Lloyd’s syndicates are the experts in this
particular type of risk. Note the high level of trust that the underwriter places in the
broker by relying upon the details given in the slip, although they could sue if they have
been misled. The underwriters trust the brokers to have disclosed all the facts that will be
relevant in assessing the risk.
4. The cedant (with the broker’s advice) will then select a lead underwriter and a ‘firm
order’ price for the broker with which to approach the market. This firm order price
may be below any of the quoted prices.
5. The lead underwriter accepts a share of the risk by stamping and signing the slip.
On accepting a portion of the risk, the underwriters write their names (and the proportion
they are willing to accept) on the slip under the description of the risk, hence the term
underwriter.
6. The broker then approaches other underwriters (the following market) to accept the
risk on the same terms. The follow underwriters indicate the share that they are
willing to take by stamping and signing the slip under the lead underwriter line.
(Hence the etymology of the term underwriting.) All the underwriters act as
coinsurers with several liability.

© IFE: 2019 Examinations The Actuarial Education Company


SP7-07: General insurance markets Page 17

Even though the underwriters share the risk, ‘several liability’ means each underwriter is
separately liable for its obligations, and so could be sued separately for any loss that it is
due. If one underwriter went bankrupt, the other underwriters would not be liable for
that underwriter’s share of the losses.
7. The broker continues until they have finished placing the risk (that is, received offers
for 100% or more of the risk).
Note the importance of the rate quoted by the lead underwriter. As all the underwriters
follow the premium rate set by the lead underwriter (except in exceptional
circumstances), it is important that the rest of the market respects the lead underwriter.
Also, it is important that the premium rate set by the lead underwriter is not so low that
no other underwriter is prepared to follow.
8. If the written lines exceed 100% then, in agreement with the insured, they are
reduced (or ‘signed down’) so that the signed lines total 100%.
The shares of the underwriters may total something in excess of 100%. (This assists in the
continuity of placing the risk in future years.) The total is then adjusted by reducing all
the percentage shares (often proportionately, but not necessarily so). The underwriter’s
acceptance of an initial percentage of the risk means that this is the maximum they are
prepared to accept on those terms.
9. If it is not possible to find capacity to place 100% of the risk, an additional shortfall
cover may need to be placed at different terms. So the premium rate is increased or
the cover or terms renegotiated.

If the risk is over-placed, this indicates that the firm order price was probably too high.
Conversely, if the risk is not fully placed the firm order price was probably too low. Not
placing the full risk, unless intentional, is usually a bigger issue for the insured and the
broker.

In general, all (re)insurers on the slip receive the same terms. However, there are some
markets where the lead underwriter may receive a higher rate to reflect the additional work
that they carry out on behalf of the following market.

The Actuarial Education Company © IFE: 2019 Examinations


Page 18 SP7-07: General insurance markets

3 Regulatory and fiscal regimes

3.1 The need for supervision


Why should insurance business suffer more legislation than, say, umbrella manufacturers? One of
the reasons is that there is more scope for the purchaser to lose out financially. When you buy an
umbrella, you have a look at it, and if you like it, you pay the price. However, with insurance, you
pay the price at the start of the contract and you have to trust the insurer to pay valid claims as
and when they arise in the future.

The uncertainty underlying insurance business means that it is not just a question of trusting the
honesty of the insurer. The insurer may be very well meaning, but if the insurer’s business is not
soundly managed, you may find that the insurer has collapsed by the time you need to make a
claim.

In many countries, therefore, there are specific rules and regulations that apply to general
insurers. Different countries adopt different approaches to the regulation of insurers’ operations.

3.2 Effect of the regulatory regime


The following regulatory restrictions on the actions of a general insurer may be
encountered in one or more countries of the world:

 A requirement to have an external audit of the general insurer’s accounts and to


release publicly certain information pertaining to the insurer.

 Restrictions on the type of business that a general insurer can write or classes for
which the insurer is authorised. An authority could prevent an insurer from writing
volatile classes of business or classes where it had little expertise.
 Limits or controls on the premium rates that can be charged.
For example, the authorities in some US states, eg Massachusetts, set the personal motor
premium rates that must be charged. Some states require that rates are filed (sent to the
relevant state department for approval) prior to an insurer using them. An authority
could also set maximum or minimum premiums or restrict the way in which the premiums
are calculated. For example an authority could set a maximum allowance for expenses
defined as a percentage of the gross premium.
 Restrictions on the information that may be used in underwriting and premium
rating, perhaps to avoid unfair discrimination. For example, under the EU Gender
Directive, European insurers are no longer allowed to use gender as a rating factor. (This
is discussed further below.)
 A requirement to deposit assets to back claims reserves.

 A requirement that the general insurer maintains a minimum level of solvency,


measured in some prescribed manner, ie a minimum level of free assets. This might,
for example, be calculated as a proportion of premiums written.

© IFE: 2019 Examinations The Actuarial Education Company


SP7-07: General insurance markets Page 19

 Restrictions on the types of assets or the amount of a particular asset that a general
insurer can take into account for the purposes of demonstrating solvency. This
might be with the possible aim of avoiding risky investments or increasing diversification.
 A requirement to use prescribed bases for calculating premiums or for valuing the
general insurer’s assets and/or liabilities when demonstrating solvency.

 Restrictions on individuals holding key roles in companies.

 Licensing of agents to sell insurance and requirements on the methods of sale and
disclosure of commission / broking terms.

 A requirement to pay levies to consumer protection bodies.

 Legislation to protect policyholders if a general insurer fails.

Question

Suggest possible legislation that could be used to protect policyholders if a general insurer fails.

Solution

A fund could be set up to pay claims from failed general insurers. It could be funded by the
government or by charging a levy on other insurers.

Companies could be required to deposit a large initial sum to a central governing body. This could
be used to pay claims on default.

EU Gender Directive
The EU Gender Directive was passed in 2004, being aimed at ‘implementing the principle of
equal treatment between men and women in the access to and supply of goods and
services’.

In its original form, the EU Gender Directive included an opt-out in respect of financial and
insurance products provided that certain conditions were met. In March 2011, the European
Court of Justice gave its ruling on the legality of the insurance opt-out provision, concluding
that it is not valid and should therefore be removed with effect from 21 December 2012. From
that point, insurance companies have no longer been able to use gender as a rating factor.

Insurance companies are careful to avoid the use of proxy rating factors (ie highly
correlated to gender) that might be deemed to be indirect discrimination and thus also not
permitted.

Clearly, the inability to differentiate between gender when setting premium rates is having
significant implications for insurance pricing, particularly for motor insurance where there
are material observed differences in claims experience according to gender at certain ages.

Each insurer is likely to set premium rates based on the expected mix of business by gender but
there is the risk that the mix of male / female policyholders turns out not to be as expected. The
introduction of this legislation has therefore increased the uncertainty of insurers’ claims
experience and profitability.

The Actuarial Education Company © IFE: 2019 Examinations


Page 20 SP7-07: General insurance markets

It is not yet clear how premium rates or underwriting practices have changed as a result of
the ruling. However, it is likely that premiums have not simply ‘met in the middle’, but that
there have been additional contingency loadings for the risk of business mix by gender not
being as expected within the unisex pricing.

In other words, this legislation has also led to increased uncertainty in premium rates, at least in
the short term, and hence higher risk margins being charged by insurers.

Other possible regulations


Other regulations that could be imposed on general insurers include:
 requirement to provide detailed reports and accounts at prescribed intervals
 requirement to purchase reinsurance
 requirement to hold a claims equalisation reserve
 limits on contract terms
 advertising restrictions
 prescription to hold certain assets.

Regulation is covered in more detail elsewhere in the course.

3.3 Effect of the fiscal regime


In most countries the taxation of general insurers broadly follows that for other businesses
although there may be special features, such as allowing equalisation reserves to be held to
allow for the uncertain nature of general insurance business.

For example, in some countries, transfers to equalisation reserves or catastrophe reserves may be
allowable against taxable profit.

Some countries impose a tax on general insurance premiums for some or all classes of
business.

© IFE: 2019 Examinations The Actuarial Education Company


SP7-07: General insurance markets Page 21

4 Professional guidance
When carrying out work for a general insurer or reinsurer an actuary should always bear in
mind any professional guidance relating to the work being carried out and the professional
body to which he or she belongs.

Institute and Faculty of Actuaries (IFoA) members practising in the UK need to comply with the
Technical Actuarial Standards (TASs) that are issued by the Financial Reporting Council. Although
the content of the TASs is not officially part of the Core Reading, a familiarity with them may help
you pass the exam.

An actuary should also bear in mind guidance on professional standards in addition to


guidance on technical issues.

All members of the IFoA are subject to The Actuaries’ Code. This gives general guidance on
professional conduct to which all IFoA members must conform in both the spirit and the letter.

In addition to formal guidance, the professional body may issue advice from time to time on
specific issues.

The details of all the latest professional guidance issued by the IFoA can be obtained via its
website: www.actuaries.org.uk.

Professional guidance is covered in more detail in Subject SA3.

The Actuarial Education Company © IFE: 2019 Examinations


Page 22 SP7-07: General insurance markets

5 Glossary items
Having studied this chapter you should now read the following Glossary items:

 Benchmark  Members’ agent


 Binding authorities  Mutual insurer
 Capacity  Names (Lloyd’s)
 Central fund (Lloyd’s)  Open year
 Closed year  Pooling
 Committee of Lloyd’s  Premium income limit
 Composite insurer  Premiums trust fund (PTF)
 Council of Lloyd’s  Proprietary insurer
 Funds at Lloyd’s (FAL)  Protection and Indemnity (P&I) Clubs
 Going-concern basis  Self-insurance
 Lead underwriter  Slip system
 Line slip  Syndicate (Lloyd’s)
 Lloyd’s broker  UK Guarantee Fund
 Lloyd’s deposit  Underwriting agent
 Lloyd’s managing agent  Underwriting ratio
 Lloyd’s members’ agent  Underwriting year
 Lloyd’s special reserve fund  Value at Risk (VaR)
 LMX on LMX  Wind-up basis.
 LMX spiral
 London Market
 London Market excess of loss
 Managing agent

© IFE: 2019 Examinations The Actuarial Education Company


SP7-07: General insurance markets Page 23

Chapter 7 Summary
General insurance is provided by insurance companies and Lloyd’s syndicates. A corporate
entity may also manage its self-insurance through a captive insurance company.
Reinsurance can be obtained from the London Market, Lloyd’s, specialist reinsurance
companies or direct insurers who also write reinsurance.

Direct insurers (as opposed to reinsurers) write insurance business for individuals and
companies. They may be composites or they may write general insurance only (either
specialising in certain classes or writing all classes of GI business). Most insurers are
proprietary, but there are some mutual insurers.

The London Market is that part of the insurance market in which insurance and reinsurance
business is carried out on a face-to-face basis in the City of London. It includes Lloyd’s and
the specialist companies who write business in the City. Much of this insurance is for large
and/or international risks and reinsurance, although Lloyd’s does provide certain personal
lines cover.

Lloyd’s, a major component of the London Market, is a special insurance market where
wealthy individuals and corporate bodies, known as Names, group together in syndicates to
collectively coinsure risks. Lloyd’s itself does not provide the insurance.

Most members are companies (corporate names) and have limited liability. Private
members may have unlimited or limited liability. Lloyd’s syndicates can write business
almost anywhere in the world.

A number of large enterprises have set up a captive insurance company to provide for their
own insurance needs, retaining premiums and risk within the enterprise. The main reasons
for captives include:
 focusing effort on risk management
 managing the overall insurance spend
 providing direct access to the reinsurance market
 providing insurance cover not available elsewhere.

Also, there may be tax or regulatory advantages. In addition to accepting the risks of their
parent companies, captives may also accept external risks on a commercial basis.

The Actuarial Education Company © IFE: 2019 Examinations


Page 24 SP7-07: General insurance markets

Chapter 7 Summary continued


Protection and Indemnity (P&I) Clubs provide insurance to ship owners. They are a popular
way to cover marine liability claims due to their mutual nature and the provision of technical
assistance. Some of the largest P&I Clubs themselves mutualise in respect of very large
claims and the purchase of some reinsurance via the International Group of P&I Clubs.

Specific pooling arrangements, where the parties agree to share premiums and losses for
specific insurance classes or types of cover in agreed proportions, are sometimes used
particularly where the risks are very large (eg atomic energy risks) or via mutual associations
that cater for an industry.

Markets in different territories tend to differ in the concentration of insurers (by market
share), the sales methods used, the importance of mutuals and whether composites are
permitted.

Reinsurance tends to be placed internationally. Many reinsurers are based in the US,
Switzerland and Germany. Bermuda has become a major international centre for insurance
and reinsurance, with a number of captive insurers being based there.

Non-London Market business can be sold:


 through intermediaries, such as brokers or tied agents (eg building societies or travel
agents)
 by staff directly employed by the provider
 via direct marketing methods (eg internet or telesales).

The main sales method varies by class of insurance.

Traditionally, business is placed in Lloyd’s and the rest of the London Market by specially
accredited brokers, using the slip system.

A regulator could restrict the actions of a general insurer in many ways including limiting
premium rates, restricting investments and requiring a minimum level of solvency on a
prescribed basis.

When carrying out work for a general insurer or reinsurer an actuary should always bear in
mind any professional guidance or advice relative to the work being done and the
professional body to which they belong.

© IFE: 2019 Examinations The Actuarial Education Company


SP7-07: General insurance markets Page 25

Chapter 7 Practice Questions


7.1 State the types of insurer that provide insurance in the direct insurance market and the ways they
may be owned.

7.2 Describe the following aspects of a standard household insurance policy:


Exam style (i) cover and perils normally included [10]
(ii) bases of claim settlement [3]
(iii) rating and other relevant underwriting factors [5]
(iv) measurement of exposure [3]
(v) sales outlets and product features. [4]
[Total 25]

7.3 List four providers of reinsurance.

7.4 (i) Define the term captive insurer.

(ii) State the advantages and disadvantages to a large multinational company of setting up a
captive insurer.

7.5 A large industrial corporation has recently acquired a general insurance company that has
Exam style
traditionally written personal motor business. The new management wishes to expand the
business into homeowners insurance, small commercial lines insurance, larger commercial risks
insurance and personal computer extended warranty insurance. The motor portfolio has
traditionally been written through independent brokers.

(i) List the various options that the insurer may have in distributing its products. [4]

(ii) Describe how the general insurance company’s choice of distribution channels may affect
its business and the decisions it must make, under the following headings:
 volumes / market
 expenses / setup
 risk premiums. [8]
[Total 12]

The Actuarial Education Company © IFE: 2019 Examinations


Page 26 SP7-07: General insurance markets

The solutions start on the next page so that you can


separate the questions and solutions.

© IFE: 2019 Examinations The Actuarial Education Company


SP7-07: General insurance markets Page 27

Chapter 7 Solutions
7.1 The insurers within the direct market can be subdivided into:
 mutuals and proprietaries
or into
 composites, specialist insurers or multi-class insurers.

Mutuals are owned by its policyholders.

The proprietaries may be:


 publicly owned (ie a listed company with many shareholders)
 owned by other companies (eg a bank)
 owned by a mutual life office
 privately owned.

7.2 (i) Normal cover and perils

Domestic household insurance might cover contents and/or buildings. [1]

Cover:
 damage to buildings from specified perils
 damage to contents from specified perils
 public liability
 accommodation costs / loss of rent if home is uninhabitable
 possible extensions to cover noted below. [½ each]

Perils:
 windstorm
 flood
 lightning
 fire / explosion
 earthquake
 subsidence / land-heave
 impact from vehicles, animals
 theft and burglary damage
 damage from vandalism / civil commotion
 damage from other assorted perils (pipes bursting etc). [½ each]

The Actuarial Education Company © IFE: 2019 Examinations


Page 28 SP7-07: General insurance markets

Extensions / variations:
 index-linked (premiums and claims are linked to an index)
 accidental damage to possessions
 personal accident cover with defined benefits
 all risks for items outside the home (eg laptops, bicycles, credit cards, money)
 freezer contents. [½ each]
[Maximum 10]

(ii) Bases of claim settlement

Policies may be on an indemnity basis, returning the policyholder to their financial position before
the loss, or a replacement (ie new for old) basis, replacing losses with funds to buy new items. [2]

The insurer has the right to replace the item or to pay the money equivalent. However this
should be clearly set out in the policy wording. [1]
[Total 3]

(iii) Rating and other relevant underwriting factors

Assuming that the exposure measure is sum insured, the main rating factors would be:
 cover (buildings / contents / both)
 basis for cover (new for old / indemnity)
 sum insured (if not used as the exposure measure)
 location
 extensions / options selected (see part (i)). [½ each]

Other factors that may influence the premium are:


 use of property (business as well)
 no claims discount, if applicable
 excess (voluntary / compulsory)
 age of proposer
 house / flat / other
 construction material (walls and roof)
 age of property
 heating method
 ownership (freehold, leasehold, rented)
 day-time occupation
 whether there are long periods vacant

© IFE: 2019 Examinations The Actuarial Education Company


SP7-07: General insurance markets Page 29

 window locks and type of door locks


 burglar and smoke alarms. [½ each]
[Maximum 5]

(iv) Measurement of exposure

Buildings: value of property as sum insured per year. Sum insured would be based on site
clearance and rebuilding costs. Some insurers now use number of rooms, floor space, or single
unit block instead. [1]

Contents: sum insured per year. [1]

More crude measures might be ‘policy year’ (but problems with multiple policies) or ‘house
year’. [1]
[Total 3]

(v) Sales outlets and product features

Individual policies are sold through brokers, direct sales force, telephone sales, and some
mortgage providers (eg banks).
[1]

Block policies (for buildings cover) are provided by building societies, who carry out much of the
administration. [1]

Buildings and contents cover may be provided separately. If taken together a discount may be
given. [1]

Insurers may operate a limited no claims discount with, say, a 10% reduction for one claim free
year. The discount could operate for new business as well as renewals. [1]
[Total 4]

7.3 Examples of providers of reinsurance include:


 professional reinsurers
 Lloyd’s syndicates
 reinsurance pools and schemes
 captive insurers.

7.4 (i) Definition of a captive insurer

An insurer wholly owned by an industrial or commercial enterprise set up with the primary
purpose of insuring the parent.

The Actuarial Education Company © IFE: 2019 Examinations


Page 30 SP7-07: General insurance markets

(ii) Advantages and disadvantages

Advantages:
 fills gaps in cover that may not be available from the traditional insurance market
 helps manage the total insurance spend of the company and in particular to make
financial plans more predictable due to the increased stability of premiums
 allows direct access to the reinsurance market
 helps focus effort on risk management
 retains profits that would otherwise have been passed to other insurers
 may gain tax or regulatory advantages, eg if the captive is set up in a low taxation offshore
location.

Disadvantages:
 expenses of setting up a captive and hiring the insurance expertise needed
 capital is required to set up the captive
 regulatory approval may be required from all the countries that the multinational
operates in
 risk is retained within the same group
 risk of accumulations building up due to a lack of diversity in the business written
 no access to insurers’ expertise, eg in dealing with complicated risks or claims
 setting up a captive may divert management attention from the core business.

7.5 (i) Distribution channels

Possible distribution channels are:


 independent intermediaries (brokers) [½]
 … which could be specialist brokers (for large commercial risks) [½]
 intermediaries tied to a particular insurer, such as: [½]
– banks and other financial institutions [½]
– computer shops or manufacturers [½]
 direct sales through staff directly employed by the insurance provider [½]
 through direct marketing methods, such as:
– telephone sales [½]
– internet [½]
– post [½]
– off-the-page advertising [½]
 affinity groups. [½]
[Maximum 4]

© IFE: 2019 Examinations The Actuarial Education Company


SP7-07: General insurance markets Page 31

(ii) Effects on business

Volumes / market

The choice of sales channel will affect the likely volume of business sold in each class; some
channels being more popular than others. [½]

Current customers and the brokers may not be happy if the motor business is also sold through
direct marketing. [½]

Homeowners insurance may be sold using direct sales techniques or the company may also team
up with a mortgage provider. [½]

Small commercial lines may be sold using mass advertising and direct sales or using brokers. [½]

Larger commercial risks are likely to be sold in smaller volumes via specialist brokers. [½]

Extended warranty will probably be sold via a tied agent, such as the shop selling the product
covered by the insurance. [½]

Different channels may give access to different segments of the market. [½]

Each channel is likely to experience different competitive pressures and hence could lead to
different profitability. [½]

Expenses / set-up

The level of commission (eg to brokers or tied agents) will depend on the distribution channel
used. [½]

However, ongoing expenses may be reduced since the broker or agent will be able to carry out a
lot of administration (eg endorsements and dealing with claims). [½]

Set-up costs for using brokers should be relatively low, whereas the use of any direct channel will
result in significant set-up costs associated with investing in the necessary infrastructure,
training etc. [1]

Any mass advertising used to accompany direct selling methods, eg newspaper adverts, could be
expensive. [½]

Mailing existing policyholders as a way of introducing new lines of business could be relatively
cheap. [½]

The company may charge different rates for different channels to reflect the different expense
levels and types. [½]

The persistency may differ by channel, which will affect the spreading of initial expenses. [½]

Risk premiums

Different types of policyholders are likely to buy insurance using different sales channels and
hence the claims experience will differ. [1]

The Actuarial Education Company © IFE: 2019 Examinations


Page 32 SP7-07: General insurance markets

If the company uses more than one distribution channel for a class of business then it must decide
whether to charge the same premiums for each distribution channel or to allow cross-subsidies.
[1]
[Maximum 8]

© IFE: 2019 Examinations The Actuarial Education Company


SP7-08: Regulation Page 1

Regulation
Syllabus objectives
2.2 Discuss the purposes of regulating general insurance business.

2.3 Outline possible methods by which general insurers can be regulated, including
advantages and drawbacks of each.

The Actuarial Education Company © IFE: 2019 Examinations


Page 2 SP7-08: Regulation

0 Introduction
We have already introduced some of the concepts surrounding the regulation of insurance earlier
in the course. You may therefore like to revise this topic briefly before you read any further.

This chapter provides an overview of regulation and supervision of insurers, without going
into the detail of specific jurisdictions.

We have drawn on the general principles, standards and guidance issued by the
International Association of Insurance Supervisors (IAIS). The IAIS Insurance Core
Principles are intended as a benchmark for supervisors in all jurisdictions and have been
developed and agreed by insurance supervisors worldwide.

The IMF and World Bank use the IAIS Insurance Core Principles to conduct FSAPs
(Financial Sector Assessment Program) of the insurance supervisory regimes around the
world. The IMF assesses regimes in developed economies while the World Bank focuses
on less well-developed economies. The results of these assessments are made public.

Further information on the International Association of Insurance Supervisors (IAIS) can be


found at www.iaisweb.org; IAIS papers are available on the website and are copyrighted by
the IAIS.

The IAIS is formed of representatives from insurance supervisors and regulators from all over the
world. An understanding of the guidance and principles issued by IAIS is therefore key to
understanding best practice relating to insurance supervision.

In Section 1, we first discuss the broad aims of regulation, and the costs of regulation, without
focussing on the insurance industry.

This topic was covered in full detail in an earlier subject. Section 1 highlights the key
conclusions, in order to recap the framework that gives context to the discussion of the
detailed areas in the following sections.

You should not need to spend a great deal of time revising the material in Section 1, as this
material is not technical and you may be familiar with it from your previous studies.

The remainder of this chapter discusses regulation in the context of general insurance.

In Section 2 we discuss the objectives of insurance regulation.

In Section 3 we introduce the main aims and activities of the IAIS and then discuss the Insurance
Core Principles (ICPs), which are a set of guidance papers issued by IAIS for insurers and
regulators.

Section 4 gives some background on the development and scope of Solvency II.

In Section 5 we give examples of regulatory requirements and discuss some of the disadvantages
of regulation.

© IFE: 2019 Examinations The Actuarial Education Company


SP7-08: Regulation Page 3

1 The principles behind regulatory regimes

1.1 The aims of regulation


The financial markets of all developed economies are regulated to a greater or lesser extent.
The principal aims of regulation are to:

 correct market inefficiencies and promote efficient and orderly markets

 protect consumers of financial products

 maintain confidence in the financial system

 help reduce financial crime.

These aims are, of course, related.

Question

Without reading ahead, list the direct and indirect costs of regulation.

Solution

Direct costs arise in:


 administering the regulation
 compliance for the regulated firms, ie maintaining appropriate records etc.

Indirect costs arise from:


 an alteration in the behaviour of consumers, who may be given a false sense of security
and a reduced sense of responsibility for their own actions
 an undermining of the sense of professional responsibility amongst intermediaries and
advisors
 a reduction in consumer protection mechanisms developed by the market itself
 reduced product innovation
 reduced competition.

Regulation has a cost. Regulators must attempt to develop a system which can achieve the
aims specified above at minimum cost so that the benefits, which are difficult to measure,
outweigh the costs.

Direct costs arise in administering the regulation; indirect costs arise from changes in
behaviour, both of consumers and regulated firms, to react to the regulations.

The need for regulation of financial markets is seen to be greater than the need for
regulation of most other markets primarily because of the importance of confidence in the
financial system and the damage that would be done by a systemic financial collapse.

The Actuarial Education Company © IFE: 2019 Examinations


Page 4 SP7-08: Regulation

1.2 Possible regulatory regimes

Forms of regulation
Regulation can be prescriptive, with detailed rules setting out what may or may not be done.

Recall that the detailed nature of prescriptive regulation can reduce the likelihood that things go
wrong, but often with greater costs.

Alternatively, it can involve freedom of action but with rules on publicity so that third parties
are fully informed about the providers of financial services.

Finally, the regime can allow freedom of action but prescribe the outcomes that will be
tolerated.

This last form of regulation is sometimes called ‘outcome-based’ regulation.

Regulatory regimes
Unregulated markets

It has been argued that the costs of regulation in some markets, especially those where only
professionals operate, outweigh the benefits.

Voluntary codes of conduct

These operate effectively in many circumstances but are vulnerable to a lack of public
confidence or to a few ‘rogue’ operators refusing to co-operate, leading to a breakdown of
the system.

The main advantages of a voluntary code of conduct are likely to be the reduced cost of
regulation and the fact that the rules are set by those with greatest knowledge of the industry.

The main disadvantage is likely to be the greater incentive to breach the voluntary code, which
will have no legal backing and in all likelihood less severe penalties, if any, than with statutory
regulation.

Self-regulation

A self-regulatory system is organised and operated by the participants in a particular


market without government intervention. The incentive to do so is the fact that regulation is
an economic good that consumers of financial services are willing to pay for and which will
benefit all participants. An alternative incentive is the threat by government to impose
statutory regulation if a satisfactory self-regulatory system isn’t implemented.

Question

Without reading ahead, list the advantages and disadvantages of self-regulation.

© IFE: 2019 Examinations The Actuarial Education Company


SP7-08: Regulation Page 5

Solution

Advantages:
 The system is implemented by the people with the greatest knowledge of the market,
who also have the greatest incentive to achieve the optimal cost benefit ratio.
 Self-regulation should, in theory, be able to respond rapidly to changes in market needs.
 It may be easier to persuade firms and individuals to co-operate with a self-regulatory
organisation than with a government bureaucracy.

Disadvantages:
 The regulator may be too close to the industry it is regulating. There is a danger that the
regulator accepts the industry’s point of view and is less in tune with the views of third
parties – ie the consumers of financial services.
 This can lead to low public confidence in the system.
 Self-regulatory organisations may inhibit new entrants to a market, eg by imposing very
exacting capital adequacy requirements.

Statutory regulation

In statutory regulation the government sets out the rules and polices them.

This has the advantage that it should be less open to abuse than the alternatives and may
command a higher degree of public confidence.

Even here though, there may be concerns that the regulatory body takes greater heed of the
views of those it is regulating, than those of the consumer.

A statutory regulator may also be able to benefit from economies of scale.

A disadvantage of statutory regulation is that outsiders may impose rules that are
unnecessarily costly and may not achieve the desired aim. It is claimed that attempts by
government to improve market efficiency usually fail and that financial services regulation
is an economic good that is best developed by the market.

These arguments are essentially the flip side of the arguments for and against self-regulation.

Mixed Regimes

In practice many regulatory regimes are a mixture of all of the systems described above,
with codes of practice, self-regulation, and statutory regulation all operating in parallel.

It is claimed that a mixed regime has the best of both worlds, in that it has the advantages of both
a statutory regime and a self-regulated regime, while mitigating the disadvantages of both.

Even a regime that is self-regulatory in name is likely to have statutory aspects.


Regulations are often developed by market driven private institutions (such as stock
exchanges) as well as by governments.

The Actuarial Education Company © IFE: 2019 Examinations


Page 6 SP7-08: Regulation

For example, in the UK, the Financial Reporting Council (FRC) is an independent regulator
responsible for promoting high quality corporate governance and reporting to foster
investment.

The discussion above gives a broad overview of regulation. The remainder of this chapter
discusses regulation in the context of general insurance.

© IFE: 2019 Examinations The Actuarial Education Company


SP7-08: Regulation Page 7

2 Objectives of insurance regulation and supervision


Key objectives of regulation and supervision are to promote efficient, fair, safe and stable
insurance markets and to benefit and protect policyholders.

Again, this should already be familiar to you from your earlier studies.

A sound regulatory and supervisory system for insurance helps sustainable growth and
healthy competition in the insurance sector.

A well-developed insurance sector also helps to enhance overall efficiency of the financial
system by:

 reducing transaction costs,

 creating liquidity, and

 facilitating economies of scale in investment.

‘Well-developed’ here means an orderly and efficient market. In other words, one objective of
legislation is to enhance market efficiency and consequently bring about the advantages listed
above.

Question

Explain how a well-developed insurance sector helps facilitate economies of scale in investment.

Solution

An efficient, competitive insurance industry is likely to have a good understanding of the risks it
takes on, and to allocate capital to these risks appropriately. This is in contrast to individual
policyholders, who would be less likely to hold appropriate reserves (if any) for these risks, had
they chosen to retain them.

By pooling these risks, an insurance company can (a) hold a more appropriate amount of capital
than the sum total held by individuals, and (b) achieve economies of scale in the investments it
holds to back this capital.

The insurance sector must operate on a financially sound basis in order to:

 contribute to economic growth

 allocate resources efficiently

 manage risk

 mobilise long-term savings.

Sound macroeconomic policies are essential to maintain financial stability and for the
effective performance of insurance supervisory regimes.

The supervisory authority should conduct its functions in a transparent and accountable
manner.

The Actuarial Education Company © IFE: 2019 Examinations


Page 8 SP7-08: Regulation

3 IAIS

3.1 Background to IAIS


Established in 1994, the IAIS represents insurance regulators and supervisors of more than
200 jurisdictions in nearly 140 countries, containing 97% of the world’s insurance premium
income.

A significant amount of information relating to the aims and activities of the IAIS can be found on
its website, at www.iaisweb.org.

As well as its members (ie the insurance regulators and supervisors), the IAIS has developed links
with other parties involved in the insurance industry. These other parties are called ‘observers’.

The IAIS also incorporates more than 120 observers comprising industry associations,
professional associations, insurers and reinsurers, consultants and international financial
institutions.

An executive committee, whose members represent different geographical regions, heads


the IAIS. It is supported by three main committees: the technical committee, the
implementation committee and the budget committee. These committees form
subcommittees and working parties (working groups, task forces and groups) to
accomplish their objectives.

3.2 IAIS activities


The IAIS:

 issues global principles, standards and guidance papers

 provides training and support on issues related to insurance supervision, and

 organises meetings and seminars for insurance supervisors.

3.3 IAIS objectives


The objectives of IAIS are to:

 promote effective and globally consistent supervision of the insurance industry in


order to maintain efficient, fair, safe and stable insurance markets for the benefit and
protection of policyholders
This is the key aim of insurance regulation, discussed in Section 1.
 contribute to global financial stability.

The IAIS works closely with other organisations to promote financial stability. It holds an
annual conference where supervisors, industry representatives and other professionals
discuss developments in the insurance sector and topics affecting insurance regulation.

The objectives and work of the IAIS illustrate the purpose of regulating insurance in general
and the possible methods of regulation.

© IFE: 2019 Examinations The Actuarial Education Company


SP7-08: Regulation Page 9

3.4 IAIS Insurance Core Principles (ICPs)

IAIS Insurance Core Principles require a balanced perspective


The IAIS has worked with a number of supranational financial bodies to produce
recommendations for insurers and regulators alike. Thus, the global insurance community
has been presented with a set of insurance principles that hold significant implications for
company operations.

These have been set out in the Insurance Core Principles (ICPs) that the IAIS adopted in
October 2011 and last updated in 2015. The ICPs comprise an internationally developed set
of standards and guidance for insurance regulators.

The ICPs have the support of global financial organisations such as the IMF and World
Bank.

Insurers are not obliged to follow this guidance (unless the regulator has adopted the ICP
standards themselves), although it is recognised best practice for them to do so.

It is also advantageous to insurers to comply with the ICPs because it increases confidence in their
financial strength and reduces their cost of capital.

For example, consistency between local regulation and the ICPs is considered when the IMF
and World Bank organisations review loan provisions and trading status.

This encourages insurance supervisors to identify areas where the existing regulatory
frameworks do not conform to the ICPs and to move towards compliance.

However, if regulators and insurers try to adopt the ICP’s too quickly, this can lead to other
problems, such as:
 higher compliance costs
 higher premiums for policyholders
 distracting companies from their core activities
 prompting capital providers to question their investments etc.

Therefore there is clearly a trade-off to be made when deciding how quickly / rigorously a
regulator should adopt the ICPs.

The Actuarial Education Company © IFE: 2019 Examinations


Page 10 SP7-08: Regulation

Specific implications for insurers


This sub-section discusses more specific implications for insurers of being required to comply with
ICPs.

In summary, these are:


 a best estimate basis will be used when valuing assets and liabilities
 expected higher volatility of profits, due to a move to risk-based capital assessment
 disclosure of intra-group arrangements for insurance groups
 higher costs to regulators and insurers, due to the need to demonstrate effective ERM
 consistent standards may help improve efficiency and promote product development.

Current assumptions

Risks for valuation purposes would require assumptions based on a portfolio's experience
analysis and possibly move away from prudent margins.

The potential change of emphasis towards a risk-based assessment of capital could lead to
greater volatility of distributable profits and significantly alter ratios of available-to-required
capital.

Group-wide supervision

At the group level, broader reporting requirements will probably compel the disclosure of
intra-group arrangements to the supervisor.

Enterprise risk management

Revised requirements to demonstrate the ability to control, mitigate and manage risk
exposure will initiate greater demands on the resources of regulators and insurers.

Regulatory convergence

The emergence of consistent regulatory standards may suggest strategies, products and
operating methods that could provide further advantages (eg consolidating product
development and wider product introductions).

© IFE: 2019 Examinations The Actuarial Education Company


SP7-08: Regulation Page 11

4 Solvency II

4.1 Introduction to Solvency II


Solvency II is a risk-based approach to prudential requirements which brought
harmonisation at the EEA level.

Risk-based capital assessment is covered elsewhere in the course.

Question

Suggest reasons why, under a risk-based approach, one insurer may have a higher capital
requirement than another, despite having a lower written premium income.

Solution

The insurer may have written the same volume of business but at lower rates, so it has a higher
insurance risk.

It may have purchased reinsurance from reinsurers with a lower credit rating.

It may have written more risky business, eg:


 more long-tail
 more heterogeneous risks.

Solvency II governs the capital requirements for insurance companies in the European Union.

The key objectives of introducing Solvency II were to:

 increase the level of harmonisation of solvency regulation across Europe

 introduce capital requirements that are more sensitive to the levels of risk being
undertaken

 provide appropriate incentives for good risk management

 improve consumer protection.

Scope of Solvency II
Solvency II superseded the previous Insurance Directives and the Reinsurance Directive
and, by reforming the solvency requirements for life and non-life insurance undertakings,
has aimed to improve policyholder security.

Solvency II requirements became live for companies on 1 January 2016.

The Solvency II responsibilities of supervisors and EIOPA were in effect from


1 January 2014.

(EIOPA is the European Insurance and Occupational Pensions Authority, one of the EU’s
main financial supervisory bodies, previously known as CEIOPS.)

The Actuarial Education Company © IFE: 2019 Examinations


Page 12 SP7-08: Regulation

The Solvency II Directive applies to all insurance and reinsurance companies with gross
premium income exceeding €5m or gross technical provisions in excess of €25m.

4.2 Development of Solvency II

Lamfalussy process
The Solvency II framework was created in accordance with the Lamfalussy four-level
process.

The Lamfalussy framework was drawn up by the ‘Committee of Wise Men’ in 2001, chaired by
Baron Alexandre Lamfalussy.

It is basically a step-by-step guide on how to introduce new legislation. It allows legislation to be


developed and implemented gradually over a period of time.

The four stages of the Lamfalussy process are:


 Level 1  developed an EU legislative instrument that set out the key framework
principles, including implementation powers (completed in 2009, adopted in
December 2013 and finalised in March 2014).

 Level 2  developed more detailed implementing measures and technical standards


(finalised in 2015).

 Level 3  developed supervisory guidance and common standards, and conducted


peer reviews and consistency comparisons.

 Level 4  enforced across the Member States (from 1 January 2016).

Development was supported through a number of Quantitative Impact Studies (QIS) that
insurance companies were asked to complete, and through liaison with national
supervisory bodies.

EIOPA provided technical advice and support to the European Commission for the
development of the implementing measures under Level 2 and was responsible for
producing the Level 3 additional guidance.

Further information can be found on EIOPA’s website:


https://eiopa.europa.eu/regulation-supervision/insurance/solvency-ii

Solvency II is discussed in considerably more detail in Subject SA3.

© IFE: 2019 Examinations The Actuarial Education Company


SP7-08: Regulation Page 13

5 Examples of regulatory proposals


Listed below are examples of restrictions and requirements that a regulator might put in
place to achieve the objectives outlined in Section 1 above. The purposes and advantages
of these approaches are listed alongside.

5.1 Restrictions on underwriting


 Restrictions on the type / amount of business a general insurance company can
write / classes of business it is authorised to write. An authority could prevent an
insurer from writing volatile classes of business or classes where it has little expertise.
– Ensures companies have appropriate expertise / sufficient capital to write the
business classes.

 Limits on contract terms and premium rates that can be charged. For example, we saw
earlier that some US states prescribe the level of personal motor premium rates. An
authority could also set a maximum or minimum premium or restrict the way in which the
premiums are calculated. For example an authority could set a maximum allowance for
expenses defined as a percentage of the gross premium.
– Ensures premium rates are sufficient to meet future claims / ensures
policyholders not overcharged.

 Restrictions on information that may be used in underwriting and premium rating,


eg restrictions on rating factors used, or on the ability to decline cover.
– For ethical / anti-discrimination reasons.

 Requirement to file / publish premium rates before they can be used.


– Prevents anti-competitive practices and therefore protects policyholders.

 Restrictions on countries a general insurance company can write business in.


– Prevents exposure to volatile risks and unfamiliar legal systems and
regulations.

 Mandatory restrictions on cover, eg no deductible on employers’ liability.


– To protect policyholders and claimants and to ensure consistency of cover.

 Prohibiting illegal products from being sold.


– To discourage illegal practices.

 Requirements to offer cover eg even in high-risk flood areas / motor third party
liability.
– For social responsibility and helps economy as a whole.

 Statutory requirement to purchase certain cover, eg employers’ liability and motor


third party liability.
– For social responsibility and helps economy as a whole.

The Actuarial Education Company © IFE: 2019 Examinations


Page 14 SP7-08: Regulation

5.2 Capital requirements


 The requirement to deposit assets to back claims reserves.
– To ensure the company has sufficient funds to pay claims.

 Requirement to hold a claims equalisation reserve (if allowed).


 The requirement to maintain a minimum level of solvency, ie a minimum level of free
assets. This might, for example, be calculated as a proportion of premiums written.
– To ensure if claims are significantly worse than expected the company will
remain solvent.

 The use of prescribed bases to calculate premiums, asset values and liabilities to
demonstrate solvency.
– To ensure accurate / consistent estimates of liabilities and uncertainty.

 The requirement for risk-based capital calculations and capital assessment


analyses.
– To ensure accurate estimates of liabilities and uncertainty.

5.3 Investment requirements


 Restrictions on the type or amount of certain assets allowed to demonstrate
solvency.
– To prevent high-risk assets from backing liabilities, or to encourage
diversification.
 Restrictions on the currency, domicile and duration of assets allowed to
demonstrate solvency (or mismatching reserves).
– To ensure that assets match liabilities by term and currency so that
short-term changes in exchange rates will not have an impact on solvency
margins.

 Other regulations concerning investments could include:


– requirements to hold prescribed assets, eg government securities
– restrictions on holding certain assets, eg foreign securities
– restrictions on the amount of investment in any one company / group
– custodianship of assets.

5.4 Reporting requirements


 Disclosure / transparency of reporting requirements, eg a requirement to provide
detailed reports and accounts at prescribed intervals.
– To help regulators, investors, capital providers and policyholders assess the
soundness of the company.

 Requirement for a Statement of Actuarial Opinion to be produced by an approved


actuary.
– Promotes confidence in the level of reserves and helps to prevent the failure
of a general insurance company.

© IFE: 2019 Examinations The Actuarial Education Company


SP7-08: Regulation Page 15

 Restrictions on the type of reinsurance that may be used.


– To prevent exposure to risky reinsurers or reinsurance products.

 Restrictions on discounting of liabilities and discount rates that can be used.


– To ensure consistency and that reserves are sufficient.

 Requirement for general insurance companies to be audited.


– To give regulators and investors confidence in the company and to prevent
fraud.

5.5 Authorisation requirements


 Initial authorisation of new insurance companies.
– Ensures companies have appropriate expertise / sufficient capital to write the
business classes.

 Licensing agents to sell insurance and requirements on the method of sale.


– To ensure company has necessary expertise and that insured is well
informed.

 Requirements for management to be fit and proper, eg restrictions preventing specific


individuals from holding key roles in companies.
– Promotes confidence in the industry and helps prevent fraud.

5.6 Other requirements to protect policyholders


 Requirement to purchase reinsurance.
 Legislation to protect policyholders should general insurance companies fail,
eg Financial Services Compensation Scheme.
– To protect policyholders and maintain faith in insurance market.

 Requirement to pay levies to consumer protection bodies.


– To protect policyholders and maintain faith in insurance market.

 Cooling off period, eg fourteen-day cancellation rules on policies issued.


– To protect policyholders and promote confidence in the industry.

 Advertising restrictions.
 Regulations with respect to treating customers fairly.
– To protect policyholders and promote confidence in the industry.

 Restrictions with respect to anti-competitive behaviour.


– Prevents formation of cartels, concentrations of risk, and protects
policyholders.

The Actuarial Education Company © IFE: 2019 Examinations


Page 16 SP7-08: Regulation

5.7 Disadvantages of regulatory proposals


The disadvantages are less specific and surround the following general issues:

 the cost in terms of resource and finance to comply with and supervise the rules

 the loss of business opportunities that arise from any restriction on a free market

 the inability to maximise investment returns when there are controls on investment
decisions

 the amount of regulatory bureaucracy deterring new entrants

 the difficulties and hence potential inaccuracies in complying with complex


(risk-based) liability and capital calculations

 the increased premium cost to the public arising from levies and the general
increase in insurer expenses

 the inability of companies to benefit from economies of scale and cost reductions
due to anti-competitive legislation

 the failure of insurance to reach certain sectors of the population due to the
increased cost of and restrictions on methods of distribution.

© IFE: 2019 Examinations The Actuarial Education Company


SP7-08: Regulation Page 17

6 Glossary items
Having studied this chapter you should now read the following Glossary items:
 Risk-based capital (RBC)
 Value at Risk (VaR).

The Actuarial Education Company © IFE: 2019 Examinations


Page 18 SP7-08: Regulation

The chapter summary starts on the next page so that you can
keep all the chapter summaries together for revision purposes.

© IFE: 2019 Examinations The Actuarial Education Company


SP7-08: Regulation Page 19

Chapter 8 Summary
Objectives of insurance regulation
The key objectives of legislation are to promote efficient, fair, safe and stable insurance
markets for the benefit and protection of policyholders.

Further objectives of regulation are to:


 to help growth and competition in the insurance sector and contribute to economic
growth
 enhance the overall efficiency of the financial system
 reduce transaction costs
 create liquidity
 facilitate economies of scale in investment
 allocate resources efficiently
 manage risk
 mobilise long-term savings.

Direct costs
 administering the regulation
 compliance for the regulated firms

Indirect costs
 alteration in consumer behaviour
 undermining of the sense of professional responsibility amongst intermediaries and
advisors
 reduction in self-regulation by the market
 reduced product innovation
 reduced competition

The Actuarial Education Company © IFE: 2019 Examinations


Page 20 SP7-08: Regulation

Chapter 8 Summary continued


Regulatory regimes
The main types of regulatory regime are:
 unregulated markets – where no financial services specific regulations apply, market
participants are instead subject to the normal laws of the land
 voluntary codes of conduct – drawn up by the financial services industry itself
 self-regulation – organised and operated by the participants in a particular market
without government intervention
 statutory regulation – in which a government body sets out the rules and polices
them
 mixed – a combination of the above (many countries adopt such a mixture).

Each of the above regimes can adopt any of the following forms:
 prescriptive regimes – with detailed rules as to what may or may not be done
 freedom of action – with rules only on publicity of information
 outcome-based regimes – with prescribed tolerated outcomes.

The International Association of Insurance Supervisors


The IAIS represents insurance regulators and supervisors around the world.

Its key activities are to issue principles, standards and guidance papers, provide training and
support and organise seminars for insurance supervisors.

The objectives of IAIS are to:


 promote effective and globally consistent supervision of the insurance industry in
order to maintain efficient, fair, safe and stable insurance markets for the benefit
and protection of policyholders
 contribute to global financial stability.

The IAIS core principles (ICPs) are a set of guidance papers for insurers and insurance
regulators. They are supported by the IMF and World Bank.

Implications of ICPs
Potential implications to insurers of complying with ICPs include:
 a best estimate basis will be used when valuing assets and liabilities
 expected higher volatility of profits, due to a move to risk-based capital assessment
 disclosure of intra-group arrangements for insurance groups
 higher costs to insurers (and regulators) due to the need to demonstrate effective
ERM
 consistent standards may help improve efficiency and promote product
development.

© IFE: 2019 Examinations The Actuarial Education Company


SP7-08: Regulation Page 21

Chapter 8 Summary continued


Solvency II
Solvency II has brought a risk-based approach to determining capital requirements across
the European Union.

It aims to:
 increase consistency of regulation across Europe
 reward effective risk management
 improve policyholder security and consumer protection.

It came into force on 1 January 2016 and applies to all EU insurers with GPI in excess of €5m
or gross technical provisions greater than €25m.

Solvency II was developed using the Lamfalussy process, with consultation from insurers and
insurance regulators, and support from EIOPA.

Effects of regulation
Regulation may affect the following areas:
 amount / mix of business written, including location of business, mandatory cover,
restrictions on selling illegal products, etc
 authorisation of insurance companies and management
 restrictions on sales methods, eg licensing of agents and cooling off periods
 underwriting and premium restrictions, eg approval required for premium rates
 capital requirements, eg requirements to hold reserves, minimum capital
requirements, risk-based capital calculations, restrictions on discounting, etc
 investment restrictions, eg assets held, matching requirements, etc
 contributions to consumer protection bodies
 requirements to treat customers fairly
 restrictions on anti-competitive behaviour
 reporting and disclosure requirements
 audit requirements and requirements to produce a statement of actuarial opinion
 reinsurance requirements.

The Actuarial Education Company © IFE: 2019 Examinations


Page 22 SP7-08: Regulation

Chapter 8 Summary continued


The disadvantages of regulation include:
 monitoring and compliance costs
 fewer business opportunities
 lower investment returns
 barriers to entry
 higher costs passed on to policyholders
 fewer economies of scale
 less insurance provision to some sectors of the population.

© IFE: 2019 Examinations The Actuarial Education Company


SP7-08: Regulation Page 23

Chapter 8 Practice Questions


8.1 Suggest how regulation might affect the use of stress testing by insurers.

8.2 (i) State the main objectives of regulation of the insurance industry.

(ii) Outline the other objectives that regulation of the insurance industry may be expected to
achieve.

8.3 (i) State the key activities carried out by the IAIS.

(ii) Describe the implications for insurers of adopting the IAIS Insurance Core Principles.

(iii) State the key objectives of Solvency II and state the types of companies to which it
applies.

8.4 Explain the ways in which regulation might affect a general insurance company’s capital modelling
Exam style
process. [11]

The Actuarial Education Company © IFE: 2019 Examinations


Page 24 SP7-08: Regulation

The solutions start on the next page so that you can


separate the questions and solutions.

© IFE: 2019 Examinations The Actuarial Education Company


SP7-08: Regulation Page 25

Chapter 8 Solutions
8.1 An insurer might be required to:
 carry out stress tests in its overall risk management process…
 … and use these to assist in maintaining adequate capital levels.

Regulation could specify how stress tests should be carried out. For example:
 the factors to be considered in designing and carrying out stress tests …
 … including the different modelling techniques that may be used
 the other considerations, eg corporate governance requirements that impact on the
insurer’s stress testing process.

8.2 (i) Key objectives of regulation of the insurance industry

The key objectives of regulation of the insurance industry are to promote efficient, fair, safe and
stable insurance markets and to benefit and protect policyholders.

(ii) Other aims of regulating the insurance industry

 To achieve growth and competition in the insurance sector.


 To enhance the overall efficiency of the financial system by:
– reducing transaction costs
– creating liquidity
– facilitating economies of scale in investment.
 To ensure that the insurance industry is financially sound …
… which will:
– contribute to economic growth
– allocate resources efficiently
– manage risk
– mobilise long-term savings.
 To maintain financial stability, by applying sound macroeconomic principles.

8.3 (i) IAIS activities

The IAIS:
 works closely with other organisations to promote financial stability
 issues global principles, standards and guidance papers
 provides training and support on issues related to insurance supervision
 organises meetings and seminars for insurance supervisors.

The Actuarial Education Company © IFE: 2019 Examinations


Page 26 SP7-08: Regulation

(ii) Implications of complying with the IAIS Insurance Core Principles (ICPs)

Some advantages of complying with the ICPs are:


 improved confidence in the insurer’s financial strength and risk management processes ...
 ... which should reduce its cost of capital
 the IMF and World Bank will take it into account when reviewing loan provisions and
trading status
 a greater understanding of the nature of the risks taken on
 consistent standards may help improve efficiency and promote product development.

Some disadvantages of complying with the ICPs are:


 higher compliance costs ...
 ... which may be passed onto customers via higher premium rates
 being forced to use a best estimate valuation basis, which may restrict flexibility
 distracting insurers from their core activities
 potentially more volatile profits, due to using a risk-based capital approach
 more stringent disclosure requirements.

(iii) Objectives of Solvency II

The key objectives of Solvency II are:


 to increase the level of harmonisation of solvency regulation across Europe
 to introduce capital requirements that are more sensitive to the levels of risk being
undertaken
 to provide appropriate incentives for good risk management
 to improve consumer protection.

Solvency II superseded the previous Insurance Directives and the Reinsurance Directive and by
reforming the solvency requirements for life and non-life insurance undertakings, improves
policyholder security.

The Solvency II Directive applies to all insurance and reinsurance companies with:
 gross premium income exceeding €5m, or
 gross technical provisions in excess of €25m.

© IFE: 2019 Examinations The Actuarial Education Company


SP7-08: Regulation Page 27

8.4 The regulator may prescribe features of the capital model, eg: [½]
● the model that may be used … [½]
… eg whether a standard model must be used, or whether the insurer is permitted to use
an internal model … [½]
… and in the latter case, whether the internal model must satisfy particular requirements,
such as a ‘use test’ [½]
 the method used to value the liabilities … [½]
… eg whether discounting is permitted (and the extent to which it is permitted) … [½]
● the method used to value the assets … [½]
… eg market value or discounted cashflow value [½]
 the basis used … [½]
… eg a prudent basis (combined with a modest solvency margin requirement) or a best
estimate basis (combined with a substantial solvency margin requirement) [½]
 the assumptions used … [½]
… eg there may be restrictions on how to set them … [½]
… or acceptable ranges for certain assumptions [½]
 the level of detail / granularity. [½]

Alternatively, it may provide guidance and constraints, which will also affect the capital modelling
process, but probably to a lesser extent. [1]

It may also be prescriptive (or provide guidance) on the testing of the model. [½]

In particular, it might:
 require a certain degree of sensitivity testing of the main parameters [½]
 specify scenarios that must be tested … [½]
… eg assessing capital requirements assuming that the insurer closes to new business [½]
 specify extreme stresses that must be tested … [½]
… eg a 50% fall in equity prices. [½]

Regulators may also require a certain level of monitoring and reporting. [½]

This will affect the frequency with which capital requirements are assessed … [½]

… and the extent of any documentation needed. [½]

Regulation could also affect the level of cover offered, which will impact on the level of capital
required, or the way in which it is assessed. [½]
[Maximum 11]

The Actuarial Education Company © IFE: 2019 Examinations


All study material produced by ActEd is copyright and is sold
for the exclusive use of the purchaser. The copyright is
owned by Institute and Faculty Education Limited, a
subsidiary of the Institute and Faculty of Actuaries.

Unless prior authority is granted by ActEd, you may not hire


out, lend, give out, sell, store or transmit electronically or
photocopy any part of the study material.

You must take care of your study material to ensure that it


is not used or copied by anybody else.

Legal action will be taken if these terms are infringed. In


addition, we may seek to take disciplinary action through
the profession or through your employer.

These conditions remain in force after you have finished


using the course.

The Actuarial Education Company © IFE: 2019 Examinations


SP7-09: External environment Page 1

External environment
Syllabus objectives

1.3 Describe the implications of the general business environment in terms of the:

 effect of inflation and economic factors

 effect of legal, political and social factors

 effect of the climate and environmental factors

 impact of technological change.

Covered in part in this chapter.

The Actuarial Education Company © IFE: 2019 Examinations


Page 2 SP7-09: External environment

0 Introduction
In this chapter, we continue our discussion of various features of the general business
environment and their implications for general insurance business.

This chapter discusses the impact of:


 claims inflation and other economic factors, such as the underwriting cycle, investment
conditions and currency movements
 legal, political and social factors – in particular, court awards, legislative changes, and
trends in society’s behaviour and attitudes
 the climate and environmental factors, such as the weather, other catastrophic events
(both natural and human-made) and various classes of latent claims
 changes in technology.

Question

Define a latent claim.

Solution

A latent claim is a claim resulting from a cause that the insurer was unaware of when it wrote the
policy, and for which the potential for claims to be made many years later had not been
appreciated.

In common parlance, latent claims are also those that generally take many years to be reported.

© IFE: 2019 Examinations The Actuarial Education Company


SP7-09: External environment Page 3

1 Inflation and economic factors

1.1 Introduction
General economic conditions can affect insurance business in many ways. For example, in times
of low economic growth and high unemployment, the following effects can be observed:
 a greater number of claims on mortgage indemnity guarantee insurance, as more homes
become repossessed
 more claims for theft and arson, as crime rates increase
 more attempts to make fraudulent or exaggerated claims
 increased demand for pecuniary loss cover, as businesses become more concerned about
the risk of suppliers, debtors, etc becoming bankrupt.

This section discusses the following economic influences:


 different types of claims inflation and how inflation (including expense inflation) affects
insurance business
 the underwriting cycle – how it arises, influences on it and ways in which insurers deal
with it
 investment conditions – in particular, the importance of investment income to insurers
and allowance for investment income in underwriting / pricing
 currency movements – situations when this is and isn’t important and dealing with
exchange rate movements (eg by analysing business by currency and having an
appropriate investment strategy).

1.2 Claims inflation


The general levels of inflation in the economy affect the cost of providing insurance and are
therefore likely to be reflected in premiums.

Except for fixed benefit claims, inflation will affect the amount of each claim (severity). It will also
affect the level of expenses incurred by the insurer. This section focuses mainly on inflation of
claim amounts.

Different types of claims inflation


However, different types of insurance are affected by different types of inflation, and so the
overall cost may not be affected in the same way.

Question

Without looking ahead, list four different types of inflation and suggest how they might affect
insurance claims.

The Actuarial Education Company © IFE: 2019 Examinations


Page 4 SP7-09: External environment

Solution

Four different types of inflation that might affect insurance claims are:
 price inflation, which will affect the replacement cost of goods
 earnings inflation, which will affect repair costs and loss of earnings claims
 medical inflation, which will affect medical expense claims
 court inflation, which will affect claims where the claim amount is decided by a court,
eg on some liability claims.

Failure to anticipate appropriate levels of inflation can lead to insurers charging premiums
that are likely to be less adequate than they believe them to be. They can also lead to
under-reserving if reserving is done using methods that require an explicit assumption
about inflation.

The following discussion assumes that inflation is positive, ie increases claims costs, which is
usually the case. However, it should be borne in mind that there have been periods in some
territories where some types of inflation have been negative.

Household contents insurance

The most commonly quoted inflation indices are those for general consumer goods; these
are commonly known as retail price inflation or consumer price inflation. These may
reasonably be expected to affect the cost of claims of household contents policies,
although they may not be an entirely accurate predictor of claims costs since consumer
goods are likely to be the subject of claims whereas the inflation indices may include items
such as food and housing costs.

Buildings insurance

Private, commercial and industrial property claims will generally be for the repair and
rebuilding of property, particularly buildings. These costs will be linked to the cost of
building materials, but labour will be a major part of the claims costs. Labour cost is more
likely to be linked to wages than prices, and wages tend to increase more than prices so the
cost of this insurance is likely to increase more than general prices. If wages change
differently in different industries, the link to an index may be less direct.

Another aspect of inflation is ‘loss amplification’ or ‘demand surge’. This arises where
there is a temporary increase in costs of labour or raw materials due to a large number of
claims being made at the same time, for example, as a result of a single large catastrophic
event.

Medical expenses

One area, important for general insurance, in which inflation has been persistently higher
than the most quoted indices, is medical expenses. A specialist inflation index is almost
certainly necessary for projecting this type of business.

This will apply to claims on medical expense insurance (ie private medical insurance) and bodily
injury claims (eg on motor insurance).

© IFE: 2019 Examinations The Actuarial Education Company


SP7-09: External environment Page 5

In many territories medical inflation can be significantly higher than price or wage inflation
because it is a combination of several factors:
 more advanced medical treatments are used, which are more expensive
 better treatment means patients survive for longer, but still require medical care
 in some cases, doctors’ and consultants’ salaries can rise in excess of average wage
inflation.

Motor property damage

Motor property damage claims are generally for repair. This will include the supply of parts,
but motor repair is labour intensive and will be related to wage indices. Large claims will
generally be for the replacement of vehicles.

The cost of providing a replacement vehicle may vary quite differently from a general price
inflation index. It will be influenced by factors such as the competitiveness of the motor
insurance market, availability of cars from overseas and efficiencies in the manufacture of
vehicles. Specialist indices, or adjustments to a prices index, may be required. This may vary
according to the type of vehicle (eg car, van, bus).

Fixed benefits

A few types of insurance are immune from inflation because they provide fixed benefits.
These include some personal accident policies. We should expect sums insured to
increase in line with inflation, either:

 without a link to a particular index, eg because policyholders will regularly reassess their
needs, or
 in line with an index, eg because some insurers increase sums insured on some policy
types in line with an index, as a default.

However, as premiums will be proportional to sums insured, premium rating would not be
affected by inflation.

Under fixed benefit contracts, eg a personal accident policy providing a specified payout on loss of
a limb, the benefit amount is known throughout the period of cover. There is therefore no risk to
the insurer that the claim amount will be greater than expected.

Of course the number of claims might be greater than expected, but remember that this section is
just discussing inflation of claim amounts, rather than trends in claim frequency.

The sum insured may increase from one year to the next due to:
 policyholders choosing a higher level of cover, or
 the insurer increasing the benefit amounts.

In either case, the increase may be in response to the increasing cost of living, for example.
However, there could be other factors involved. These increases will also tend to be ad hoc.
Hence, the benefit level may not be increased for several years and might be expressed in round
figures.

The Actuarial Education Company © IFE: 2019 Examinations


Page 6 SP7-09: External environment

For example, the standard level of benefit for loss of limb on a personal accident policy might
progress as follows:

Year 2013 2014 2015 2016 2017 2018


Sum insured ($) 25,000 25,000 30,000 30,000 30,000 40,000

Question

Outline other possible factors that could prompt a policyholder to increase the level of cover on a
personal accident policy.

Solution

The following factors could prompt a policyholder to increase the level of cover on a personal
accident policy:
 a change in family circumstances, eg marriage or birth of a child
 increasing salary, in particular due to a change in job or promotion
 a response to marketing material from the insurer, eg promoting the benefits of, or
offering discounts on, an increased level of cover.

If the benefit amounts are increased on renewal, there will be a proportionate increase in the
premium (unless the pricing basis is also changed). There is a slight risk to the insurer that if it
increases the benefit amounts, and so demands an increased premium, this might discourage
policyholders from renewing. However, as the Core Reading points out, premium rates are often
expressed as a premium proportional to the sum insured. The premium rate would therefore be
unchanged, which may be more acceptable to policyholders.

However, if sums insured, and therefore premiums, did not increase over time then the cost
component of premiums would need to be increased from time to time.

This is referring to the allowance for expenses in the premium. As discussed below, expenses will
be subject to inflation, which is most likely to be salary-related inflation.

Liability insurance

Liability insurance presents a generally different relationship with inflation. Liability for
property damage may develop in line with the same indices as property damage claims.
However, claims for personal injury are more complex. They generally have several
components.

The components of personal injury claims are sometimes called ‘heads of damage’. The three
components that will be discussed here are:
 compensation for loss of income
 cost of medical and nursing care
 awards for pain and suffering.

© IFE: 2019 Examinations The Actuarial Education Company


SP7-09: External environment Page 7

There may be compensation for loss of income, which will be subject to the same inflation
indices as other income related costs.

Claim payments that are intended to represent the future lost earning of an individual following
an accident are likely to be based upon the present value of that future income. The courts may
from time to time change the rate of interest at which insurers are allowed to discount future
earnings.

The Ogden tables are actuarial tables that provide multipliers that are widely used by UK courts in
assessing the present value of future losses in personal injury or accidental death cases. They are
based on assumptions for life expectancies, employment risk and a set of discount rates. The
discount rate actually used is set by the Government. A low discount rate will result in a high
lump sum award.

The discount rates to be used (and indeed other assumptions in the tables) will be changed from
time to time. Some changes could have a dramatic effect upon the overall claim payments. The
unpredictability of these court decisions makes it difficult to estimate this type of inflation.

Question

Explain what is meant by the employment risk assumption within the Ogden tables.

Solution

The employment risk assumption in the Ogden tables allows for the risk that the individual may
have been unable to work (and thus earn an income) in some future periods, assuming the
accident did not happen. For example, this may be due to sickness, unemployment or retirement.

A structured settlement is also known as a ‘periodical payment’. It is where compensation is paid


in the form of an annuity rather than a lump sum. In the UK, since 2005, courts have had the
power to insist that claims for compensation of future earnings are given in this form.

Paying claims as a structured settlement is expected to increase the cost of claims to insurers,
because:
 annuities can be expensive to purchase, especially if the market for impaired life annuities
is limited
 the courts might decide to increase the regular payment amount at a later stage
 it will be more expensive to administer the regular payments compared to a lump sum.

The reluctance of insurers to have a structured settlement may encourage them to accept a
higher lump sum settlement than they would otherwise, and so increase their costs.

A second component of personal injury claims is the cost of medical and nursing care, which
is more likely to develop according to medical expenses inflation, and may also be affected
by changing approaches to discounting.

The Actuarial Education Company © IFE: 2019 Examinations


Page 8 SP7-09: External environment

Finally, awards for pain and suffering have tended to become more generous in many
countries and have grown at a faster rate than general price or wage inflation. These
awards tend to be set judgementally (subjectively) rather than according to fixed criteria.
These and other factors often called ‘social inflation’ or ‘superimposed inflation’ are further
discussed below.

These components of personal injury claims are discussed later in this chapter, in Section 2.1,
where we cover court awards.

Economic influences
The frequency of losses can also depend on economic factors.

For example, in times of recession there might be an increase in the number of claims on
creditor business, in respect of arson on property business and in theft generally.

Expense inflation
The cost loading for an insurance policy is also subject to inflation. Insurance is a relatively
labour intensive industry (although technology may improve productivity and reduce unit
costs in the future). This suggests that this component of policy costs may be likely to
increase in line with wage inflation.

1.3 The underwriting cycle


The underwriting cycle (also known as the ‘insurance cycle’) is a cycle of high then low premiums.
When insurance premiums are high and profitable, competition tends to increase as more general
insurance companies want a share of the profits. The competition then drives down premiums.
Low, unprofitable premiums lead to insurers struggling to stay afloat. Some insurers become
insolvent and leave the market. The reduced competitive pressures then lead to an increase in
premiums, and so the cycle continues.

Although it might seem difficult to believe, there has been a surprisingly clear cyclical pattern of
insurance profits in the past. Despite this clear pattern the cycle is quite difficult to predict going
forwards.

In the past it has been observed that insurance premium rates have varied in ways that do
not reflect the underlying cost of providing the insurance. This is most common in large
commercial and industrial insurance; for example that placed in the London Market, but it
affects all classes of insurance.

On many general insurance products there will be times when insurers can make a large
allowance for profit in their premiums and other times where they need to make a much smaller
allowance (if any). This is due to the effect of the underwriting cycle, which can often be
observed, not only on large commercial and industrial insurance, but also on many personal lines
products such as household and private motor insurance, particularly in competitive markets.

© IFE: 2019 Examinations The Actuarial Education Company


SP7-09: External environment Page 9

Stages of the underwriting cycle


In general, the cycle can be described in the following terms, although describing it as
starting from a position of general profitability is purely arbitrary: the sequence could be
entered at any point.
1. Insurance is generally highly profitable. This position is commonly known as a hard
market.
Insurance premiums are generally high (or ‘hard’) at this point. You can think of it as
being a time when it is hard for policyholders to buy insurance cheaply.
2. The level of profits attracts new entrants to the market and encourages existing
insurers to write more business.
These insurers are attracted by the prospect of good returns.
To fill the extra capacity, premium rates are reduced to attract business.
Premium rates start to ‘soften’ during this phase as insurers compete for market share.
3. Eventually premium rates fall to the extent that insurance is generally loss-making.
This position is commonly known as a soft market.
Insurance will often merely be less profitable rather than actually loss-making. This is the
bottom of the cycle.
4. Insurers leave the market in response to the level of losses or reduce the amount of
business they write.
5. With restricted availability of insurance, premium rates increase.
6. Eventually premium rates rise to the extent that insurance is generally highly
profitable.
We’re back at stage 1 again, and so the cycle continues…

Question

Explain why companies don’t enter at the bottom of the cycle and leave at the top.

Solution

It may be difficult for an insurer to act against the cycle because:


 It is far from obvious when the peak and trough have been reached.
 Even if it were known, such a policy is not practical for several reasons, eg:
– the insurer would need to get a large market share from a standing start, this may
involve offering very low premiums or spending lots on advertising
– the insurer would incur high expenses on closing down and starting up again
(acquiring new business incurs higher expenses than processing renewals).

Note that different classes of insurance business will tend to be at different points of the cycle at
different times. At any point in time, profits from one class of business will subsidise another, less
profitable, class.

The Actuarial Education Company © IFE: 2019 Examinations


Page 10 SP7-09: External environment

It is important that an insurer is aware of the position in the underwriting cycle of each of its
classes of business when making strategic decisions.

Influences on the underwriting cycle


It should be noted that an insurer’s ability to write insurance is limited by the amount of
capital that it holds. While the prospect of an extremely profitable market will attract new
capital that may be subscribed to existing companies and new companies, a profitable
market in itself increases insurers’ capital bases as retained profits increase capital
holdings. Since the same effects apply to reinsurers, reinsurance is also likely to become
available on easier terms, which increases insurers’ ability to write business.

So stage 2 above (an expanding market) is a natural consequence of stage 1 (a profitable market).

The actual mechanisms that reduce the size of the market when it is unprofitable will be:

 companies becoming insolvent

 companies withdrawing as a reaction to unprofitability because of unwillingness to


accept continuing loss

 reinsurance being less readily available.

In the past, soft markets have often ended when a major disaster triggered severe losses at
a time when premium levels would not support the normal level of claims. Examples of this
are Hurricane Andrew in 1992 and the terrorist attacks of September 2001 (as well as some
other substantial losses earlier in that year).

Possible reasons why the underwriting cycle exists


The reasons for the existence of the cycle are much debated.

Four factors that encourage the cycle’s progress are now discussed:
 the ease with which new entrants can join insurance markets
 the delay between writing business and knowing how profitable it is
 simplistic regulatory capital requirements that encourage insurers to write more business
when premium rates are falling and less business when they are rising
 economies of scale, which encourage marginal costing.

Low barriers to entry

Insurance is an industry in which barriers to entry are generally low. Authorisation is a


significant process in most countries, but if a new company can demonstrate capital
sufficiency and technical competence, it will usually be authorised fairly quickly. Setting up
in business does not require the establishment of specialised plant and equipment or much
development of resources. This leads to a situation in which capital providers can quickly
move into the sector.

Delay until profitability of business written is known

Another key factor contributing to the existence of the cycle is the delay between writing
business and knowing how profitable it is.

© IFE: 2019 Examinations The Actuarial Education Company


SP7-09: External environment Page 11

Simplistic capital regimes

Simplistic capital regimes may exacerbate the cycle. In many jurisdictions, at least until
recently, the capital required to write an insurance policy depended on the premium.

For example, prior to 2016 the EU regulatory minimum capital requirement (MCR) for general
insurers (known as ‘Solvency I’) was approximately 16%–18% of annual premiums.

Question

Explain the rationale behind this approach.

Solution

The rationale behind the EU minimum margin was that the amount of capital an insurer held
related to the amount of business it wrote. It was a simple measure and it was easy to calculate.
It was easy for a regulator to enforce and verify, and allowed third parties to easily make
comparisons between insurers.

However, a disadvantage of this approach is that it required less commitment of capital to


write a policy if it were under-priced than it would have done if it had been over-priced, the
exact opposite of what risk-based considerations would merit. This meant that companies
could write more business – in terms of the amount of risk taken on rather than the amount
of premium written – as premium rates fell. Conversely, as premium rates rise they had to
restrict the amount of risk taken on unless they could raise more capital.

This is because rate increases would increase an insurer’s written premium, which in turn
increased the insurer’s capital requirement.

This exacerbated the difficulty of finding cover and tended to drive premium rates even
higher.

Current EU regulation (known as ‘Solvency II’) is much more sophisticated and aims to reduce
these unwanted side-effects. (Solvency II is covered in more detail in Subject SA3.)

Economies of scale

The economics of insurance business may also help to enforce the cycle. Insurers’
overheads tend to be, if not fixed, then less variable than premium rates. There may be little
or no cost saving (apart from commission) from an insurer not writing a policy. Therefore if
business at least covers its claims cost it may be marginally profitable for an insurer to
write it, even if the business overall makes losses. However, in the depths of soft markets,
it is common for business to fail to do even this. Insurers sometimes do not want to lose
market share because of the cost of acquiring the business again in the future, loss of
reputation and other reasons.

In a competitive market, an insurer may set premiums at a level that makes an insufficient
contribution to its fixed expenses. This may be justified on the grounds that there is still some
contribution to fixed expenses, as opposed to the zero contribution which would result from
trying (and failing) to sell an uncompetitive product with a ‘correct’ contribution built in.

The Actuarial Education Company © IFE: 2019 Examinations


Page 12 SP7-09: External environment

So, it is important that an insurer is aware of the true underlying profit or loss of its business.
Otherwise it is all too easy for it to get carried away with making premium cuts in line with the
competition and end up in a position where premiums do not even cover claim costs, and this can
result in unprofitable business.

1.4 Investment conditions


Insurance companies take premiums from customers and hold them until they have to be
paid out in claims; these monies will be placed in financial investments during this period.
Most of their capital is also available for investment.

An insurance company needs to decide how to invest the money that it has at any one point in
time. It will be exposed to the investment conditions of the assets it invests in. For example
equity investments will be exposed to changes in the underlying market values (of the individual
shares or of the market overall).

The capital that the insurance company holds can be split into two broad categories:
 that required to meet the liabilities, eg the statutory reserves, plus
 the free assets, which is the excess of the company’s assets over its technical liabilities.

Generally, an insurer will try to hold assets to match its liabilities (by term, nature, certainty and
currency), but will have greater freedom over how it invests its free assets.

In this, insurers are unlike non-financial companies, whose capital is usually tied up in
capital goods or stock. This means that income from invested securities is an integral part
of insurance business.

Only a small proportion of an insurance company’s assets is likely to be tied up in fixed assets,
such as the office buildings or machinery. Therefore, where the assets are invested, and the
investment return those assets make, are more important decisions for an insurance company to
make than they are for other companies, such as manufacturing firms.

Significance of investment conditions

Question

Suggest what factors will influence the significance of investment conditions for a general
insurance company.

Solution

The significance of investment conditions is influenced by:


 the size of the company’s free assets
 the amount of business the company writes (eg annual premium income)
 the size of its in-force business (eg size of the reserves)
 the length of claim delays (both reporting and settlement delays)

© IFE: 2019 Examinations The Actuarial Education Company


SP7-09: External environment Page 13

 the term of the contracts (eg investment is less important for weekly travel insurance
contracts than for five-year extended warranty policies)
 the premium payment method and distribution channels (premiums might be more
subject to delay under some methods / channels).

The amount of investment income that is generated by insurance business depends on the
characteristics of that business: the longer-tailed the business, the greater the amount of
investment income likely to be generated.

Some personal lines business, such as home contents, is very short-tailed and premiums
may be paid monthly. Very little investment income is generated in these circumstances.
Liability insurance will not pay claims, on average, until several years after premiums are
received; in this case the investment income will be a significant proportion of the
premiums.

Question

Explain why claim delays on liability business can be for ‘several years’.

Solution

The claim may not be notified for some years after the claim event, eg the policyholder may not
be aware that they have been exposed to a harmful substance until symptoms of a disease
emerge many years later. Also, the exposure may be over a long time period.

There may be a significant delay before the claim is fully settled. It may take time to agree
whether the policyholder is liable or not, and to decide on the amount of compensation, which
may involve court proceedings and may be disputed. Some claims may be settled using partial
payments.

Allowing for investment return in pricing


Traditionally, underwriters of all lines of business aimed to achieve an underwriting profit
without taking the benefit of investment income, although in the 1990s an appreciation of
the high levels of investment income obtainable led to so-called ‘cashflow’ underwriting,
which was ultimately associated with high losses.

In simple terms, an insurer makes an underwriting profit if its premiums are sufficient to cover its
claims and expenses. In addition, the insurer will make investment returns on the assets that it
holds, which will contribute to its overall profit.

The rationale behind the ‘cashflow’ underwriting approach was that, although premiums were set
at a level below that required to cover claims and expenses, the underwriting loss would be made
up by the investment return earned on the premiums (before claims were settled). The insurer
should still make an overall profit. It was argued that, unless premiums were set at a competitive
level, no premiums would be received to make any investment return. As competitive pressures
increased, the allowance some insurers made for investment income became increasingly
significant.

The Actuarial Education Company © IFE: 2019 Examinations


Page 14 SP7-09: External environment

In some cases, the investment return actually received ended up being much less than that
anticipated, and so overall losses were made.

These approaches reflect a lack of sophistication in the pricing and underwriting process. A more
sophisticated approach allows for the expected level of investment income under current
investment conditions.

Investment income can be allowed for in the pricing calculation by discounting expected future
claims and expenses to the date at which the premium is received. Under the approach described
here, the discount rate would reflect current investment conditions on the assets backing the
policy – for example, the current gross redemption yield for fixed-interest bonds might be used.

However, a number of points should be made here:

 A high rate of interest may indicate that expected inflation rates are high. If these
interest rates are allowed for in pricing then it is important that the projected claims
reflect a consistent level of inflation.

 Insurance companies typically place funds at shorter durations than the term of their
longer-tailed liabilities. This is partly because they need to ensure liquidity and
partly because any reductions in the market value of assets, even if they do not have
to be realised, may be reflected in solvency margins. Therefore current rates, if
high, may not continue to be available as funds are rolled over.

An alternative (more common) approach is to discount at a risk-free rate of return. This rate can
be defined as the rate at which money is borrowed or lent when there is no credit risk, so that the
money is certain to be repaid. In practice, the return on bonds issued by a (secure) government is
often used as the risk-free rate.

If investment income is taken into account in pricing it would be normal to use a risk-free
rate. Insurers may invest in more risky assets than risk-free, but since the insurers
themselves assume the investment risk it is appropriate that they receive the extra income
associated with it.

Question

An insurer is earning 6% pa return on its assets, and the risk-free rate of return is 4% pa. Explain
why it should use a discount rate of 4% rather than 6% in the premium calculation.

Solution

The insurer is earning higher than the risk-free rate because it is taking on some of the risks
associated with the assets it is invested in, such as the risk that the issuer of the asset defaults. If
it discounts at a higher rate (ie 6%), it will calculate a lower premium, which may be insufficient if
the risks do materialise. Using the lower discount rate of 4% allows an adequate premium to be
charged for the risks that the insurer is exposed to.

© IFE: 2019 Examinations The Actuarial Education Company


SP7-09: External environment Page 15

Pricing should also take account of the required profit loading. If calibrated to a return on
capital, it should take into account that the invested capital will already earn the risk-free
rate. This may be done either by:

 taking as a target return on capital the target in excess of the risk-free rate

 taking account of investment income on the capital in the calculation of projected


profitability.

Also, target return on capital will generally be higher on long-tail lines of business because
they are generally more risky than short-tail lines. This is likely to reduce the effect of
longer-tailed lines’ greater investment income.

The mechanics of pricing, including allowance for investment return, is covered in more detail in
Subject SP8.

1.5 Currency movements


The impact of movements in currency exchange rates on an insurer partly depends on where in
the world its business is written and where claims are made.

Country where business is written and claims are made


Most small and many medium-sized insurers write business in only one country. All their
premiums are likely to be in that country’s currency and claims in other currencies are likely
to be few. The effect on these companies of movements in exchange rates is very limited.

As discussed below, the exposure to currency movements for these insurers will be limited to a
few overseas claims on certain policies, such as travel insurance.

The insurers that sell business in more than one territory tend to be the larger companies.

On the other hand, most large companies write business in a number of territories, as do
some smaller companies, notably London Market and reinsurance companies. In these
companies, business may be written in a number of currencies, and the currency of a policy
may not be the currency in which all that policy’s claims are incurred.

Examples of where claims may be incurred in a country that is different from that in which the
policy was written are commercial property, marine insurance and some personal lines policies
(such as private motor and travel insurance).

A single policy may cover a company’s property in several countries, for example, or a ship
may incur claims in several currencies at it sails from country to country. This is not a
problem that is limited to major industrial and commercial policies; for example, motor
policies may lead to foreign claims and travel insurance will do so in obvious ways.

The exposure to currency movements will also depend on any territorial limits applying to
policies. For example, travel insurance that is restricted to journeys within Europe (where the
Euro currency dominates) will have less exposure to currency risk than worldwide travel
insurance.

The Actuarial Education Company © IFE: 2019 Examinations


Page 16 SP7-09: External environment

Business written through Lloyd’s


Traditionally, Lloyd’s has accounted for its business in three currencies: Sterling, US
Dollars and Canadian Dollars. The Euro was added when it was launched. Most Lloyd’s
syndicates will analyse their business in these currencies, although there is no reason why
other currencies should not be treated separately if they are important to the syndicate
concerned and added to the appropriate reporting currency when drawing up official
returns.

Analysing business by currency


Analysing business by currency avoids distortions that can arise if there are changes in
exchange rates. For example, if a company writes a policy in a foreign currency that
appreciates against its home currency then claims will cost more when measured in the
home currency and a profitable policy may appear to have been sold at a loss.

For example, suppose a UK insurer writes a policy in the US for a premium of $1,000, assuming
that claims will be 80% of premiums. If claims on the policy turn out to be $800, in US dollar
terms, this would leave the insurer with $200 towards expenses, profit, etc.

Suppose the value of the dollar increased from £0.50 when the policy was written, to £0.70 when
the claim was made.

If the values were converted into the insurer’s domestic currency, then:
 the premium would be valued at 1,000  0.5  £500

 the claims would be valued at 800  0.7  £560


 and so the policy appears to have made a loss.

This policy would make a loss as shown in the insurer’s published (Sterling) accounts. However,
this loss is due to exchange rate movements, rather than poor pricing or adverse claims, and so
this would not be a good reason to increase premiums. A better approach would be to have used
some form of currency hedging (we’ll discuss this later).

Currency movements can also cause problems when reserving.

Development factors will be distorted if the underlying claims are converted to the home
currency at different rates of exchange within a single dataset, such as a triangle.

You may recall from earlier subjects how claims can be grouped according to the year of origin
(eg accident year) and year of development (eg claim payment) in a claim triangle. Statistical
methods, such as the basic chain ladder, can be applied to the triangle to project future claims.

Development factors are the ratios of claims in successive development periods from chain ladder
calculations. If different exchange rates apply at different times, this will distort the development
factors for each year of origin. This distortion can be removed if only one exchange rate is applied
to all the data.

For this reason the actuarial analysis of historical loss data is usually carried out with all
data converted at current exchange rates.

© IFE: 2019 Examinations The Actuarial Education Company


SP7-09: External environment Page 17

However, there are exceptions. In some international industries a single currency may
dominate worldwide.

For example, major claims in the oil and aerospace industries may be determined in dollars
because new equipment and expertise in control and repair is bought in that currency, even
though claims may be paid in the currency of a local company.

Judgement needs to be used in deciding how exchange rates should be applied in any
analysis.

Currency hedging
Insurers can make real profits and losses through currency movements. A basic
assumption is that an international insurer should hold assets in currencies that can match
its liabilities; in this way the value of both assets and liabilities move together and exchange
gains on one offset losses on the other, or a similar effect might be achieved through a
currency-hedging strategy.

For example, forward currency contracts could be used.

It is not possible to match currency precisely, since, as mentioned above, it is not always
possible to know in advance in which currencies claims will arise, and reserves may run-off
favourably in one currency and unfavourably in another. Insurers may also depart from a
matched position for strategic reasons.

Even a well-matched portfolio that performs as expected can give rise to profits or losses: if
the home currency strengthens, the profits and capital made and held in foreign currencies
will simply be worth less after the change in exchange rates.

The Actuarial Education Company © IFE: 2019 Examinations


Page 18 SP7-09: External environment

2 Legal, political and social factors


Insurance does not stand by itself in a country’s economy. It is affected by many factors in
the way society is ordered.

In this section we discuss:


 court awards for compensation claims in liability insurance – how the legal basis of
‘negligence’ has arisen, how the size of a compensation award is decided, recent trends in
such awards, and the impact on liability insurance
 the impact of changes in legislation on insurers – this could impact insurers directly (such
as making insurance compulsory) or indirectly, where legislation affects the cost or
frequency of claims
 trends in society’s behaviour and attitudes, such as the propensity to claim, driving whilst
drunk, crime rates, fraudulent or exaggerated claiming, and organisations (eg claim
management companies) encouraging people to claim.

2.1 Court awards


Judicial decisions, or court awards, have most impact on claims under liability insurance business.
Consider an employer who is being sued for compensation by one of its employees claiming to be
ill due to working in hazardous conditions. If the claim goes to the courts, the courts can decide:
 whether the employee is entitled to compensation, ie whether the employer is liable or
not
 the amount of (any) compensation the employee is entitled to.

Courts determine liability and award compensation for wrongs suffered by organisations or
individuals.

Hence they can directly affect both the frequency and severity of liability insurance claims.

In many territories, a decision made by a court can set a precedent for future court awards, ie in
similar cases, and for cases where the parties involved agree to settle out of court. Most liability
claims are settled out of court.

Question

Explain why the parties involved would decide to agree a compensation case out of court.

Solution

Reasons include:
 to avoid (or at least significantly reduce) legal and court fees
 to obtain an earlier settlement
 to avoid spending the time and incurring the emotional upset of a (potentially lengthy)
court case

© IFE: 2019 Examinations The Actuarial Education Company


SP7-09: External environment Page 19

 to avoid possible adverse publicity (eg a high-profile product manufacturer)


 to have more control over the final outcome, ie they can negotiate rather than leaving the
decision to the courts
 it may be less risky – if the case goes to court, one party may end up with nothing
 to avoid a structured settlement (as discussed in Section 1.2).

Claims are usually made:

 on the basis of some breach of contract between the injured party and the allegedly
responsible party, or

 (more usually) on the basis of negligence.

Negligence
A tort is a legal term to mean a civil wrong or injury, not arising out of any contract, for which
action for damages may be sought. Note that this is a civil wrong or injury, as opposed to a
criminal offence. Those that conduct such wrongs are known as tortfeasors.

The tort of negligence was developed in the 1930s in response to a case in which a
customer alleged that she had been made ill by drinking ginger beer from a bottle sold to
her that contained a decomposing snail. She could not sue the café owner under breach of
contract as she had no contract with him: the drink had been bought for her by a friend (the
drink was sold as he had received it and was in an opaque bottle, so he had no opportunity
to inspect it and the case might have failed on this also), and the case against the
manufacturer failed because she had no contract with him. The judge concerned defined
the new tort of negligence as a legal requirement not to harm a neighbour, defined as
‘persons who are so closely and directly affected by my act that I ought reasonably to have
them in contemplation as being so affected when I am directing my mind to the acts or
omissions that are called in question’.

This case is of Donoghue vs Stephenson; see:


http://www.scottishlawreports.org.uk/resources/dvs/donoghue-v-stevenson-report.html

It took place in Scotland and it took almost four years from the initial visit to the café before it
was settled, in 1932.

It is now generally accepted that those who cause harm to others or to their property
(known legally as tortfeasors) are liable to compensate the victim. Negligence may arise
even where there is a contract; for example, an actuary who is negligent could be sued for
professional negligence even if there is a contract in place between the actuary and the
client.

This has a direct effect on all types of liability insurance. Although most liability claims are
decided by negotiation between the insurer and the representatives of the victim and few
are decided in court, the decisions of courts set benchmarks for negotiators.

The Actuarial Education Company © IFE: 2019 Examinations


Page 20 SP7-09: External environment

Jurisdiction shopping
One result of different legal systems and awards is ‘jurisdiction shopping’. This is where
the claimant will try to launch proceedings in the most claimant-friendly jurisdiction in order
to maximise any potential award. It is a particular issue in aviation insurance with changes
to the Montreal Convention.

The Montreal Convention is an international treaty signed in 1999 which sets out airlines’
liabilities for passengers (eg for death or injury) and their baggage. Changes to this treaty are
thought to have raised awareness of the fact that countries that have not signed up to the
agreement may have arrangements that are more favourable to claimants, and these are the
jurisdictions that people will tend to try and claim under.

Another example can be seen with the US asbestos litigation. As the law differs between
different US states, some states are notorious for being more plaintiff friendly than other states.
Asbestos claimants will often try to have their claim filed in one of these plaintiff-friendly states.
The ability of asbestos claimants to ‘forum shop’ by selecting the most beneficial locations to have
their claims heard, has to some extent, been abated by the enactment of tort reforms in the US,
discussed below.

Compensation
We now discuss factors that are considered when determining the size of a compensation award.

Compensation is supposed to put the victim in the position he or she would have been had
there been no incident of negligence.

This is known as the principle of indemnity.

Property damage or loss

This may be straightforward in the case of the loss of some property: it is easy to replace
many things and their cost is easily found. However, even here complications arise:

 How should depreciation be calculated?

 To what extent should the inconvenience that results from the loss of the item be
compensated?

An example here would be compensation for the loss of luggage, which a passenger may claim
from an airline.

Bodily injury – ‘heads of damage’

Compensation for bodily injury is much more complicated: the tortfeasor cannot put the
victim right again, and a monetary award must suffice.

© IFE: 2019 Examinations The Actuarial Education Company


SP7-09: External environment Page 21

Compensation for major injuries is divided into several ‘heads of damages’: different
amounts are provided for different aspects of the victim’s loss. For example:

 loss of income

 medical and nursing costs and

 compensation for pain and suffering


are likely to be recognised separately in an award.

Loss of income is often split between loss of past income and loss of future income, with each
considered separately. Special heads of damage will apply in particular cases.

The courts will consider each head of damage separately and decide on the amount of
compensation for each. The total award is the sum of all the components. In some cases, new
heads of damage have emerged, such as ‘bullying’ and ‘post traumatic stress disorder’, each of
which contribute to the total award amount. This has contributed towards a general increase in
overall claim costs (see below).

Punitive or exemplary damages are intended to punish the tortfeasor over and above the
cost of compensating the victim; these are very rare in English or Scottish law, but quite
common in the US.

The increasing cost of compensation

In general it has been noted in many markets that compensation for negligence has become
more generous. There are two ways in which this is observed:
1. courts may be more willing to accept that there is liability for a victim’s suffering
(and even that suffering exists)
2. given that they decide there is liability, the courts may award larger amounts for
similar losses.

So, from a liability insurer’s point of view, this acts to increase both the frequency and the severity
of claims.

Whether or not this is a good thing is a matter of controversy that will not be discussed
here, but there seems to be general agreement that the trends are real. They may arise
through judicial decision or through legislation. In the US, tort trials are decided by juries
who are also responsible for setting awards, which is another source of this trend.

In other territories, judges may decide the amount of the award. One could argue that the people
on juries are more likely to be emotionally affected by the victim’s case and to decide in favour of
them. This is in comparison to judges, who might be more objective.

In some states of the US, there are ‘tort reform’ laws that constrain the amount of freedom that
juries can exercise. For example, the state may place a monetary limit on the amount of
compensation that may be payable (eg per head of damage or in total). However, some US tort
reforms have increased claims costs, eg in the 1980s when it became easier to claim
compensation for injury due to childhood vaccination.

An example of legislation is the Courts Act in the UK, which empowered courts to impose
structured settlements (regular payments rather than lump sums) as compensation even
when both parties preferred lump sums.

The Actuarial Education Company © IFE: 2019 Examinations


Page 22 SP7-09: External environment

The impact of structured settlements was discussed in Section 1.2.

The effects on insurance are to some extent obvious: if the cost of awards increases,
compensation will increase and liability claims will be more expensive. This will feed
through into increased premiums.

However, it is important to note that awards are made on claims that are covered by
insurance whose risk period has expired. An unexpected increase in compensation will
affect insurance for which no more premiums can be taken and so will, other things being
equal, cause loss.

While a general trend towards higher compensation can be included in premium rates, the
rate of change is hard to predict and unexpected changes may cause claims reserves to be
inadequate.

2.2 Legislation

Impact of legislative changes on claims experience


Changes in legislation may affect insurers in the same way as court awards.

In other words, the changes can directly affect the frequency or severity of claims on policies that
were written some time ago, and so there is no scope for the insurer to increase the premium of
these policies.

It is possible that legislation can be made to apply to events that occur only after the date of
the legislation, but if it concerns the way that amounts of compensation should be
determined in court this is rare.

A change in legislation that affects all claims that have not finally been settled at the date
the legislation comes into force will affect the adequacy of claims reserves in the same way
as an unexpected judicial decision, although as legislation takes some time to prepare and
enact, it may be less unexpected.

It is therefore important for insurers to keep abreast of relevant legislative proposals and
developments so that they can assess their expected impact, and allow for this in their pricing,
reserving, financial planning, etc. Insurers may also be able to influence governments in setting
legislation – for example, through industry bodies, such as the Association of British Insurers (ABI)
in the UK.

Legislation may affect the expected losses suffered by insurers.

For example, health and safety legislation might be expected to reduce the frequency and
severity of accidents, which should reduce claims costs, although if it reflects a higher
expected level of safety, courts may be more inclined to grant large awards to the victims of
tortfeasors who infringe the standards.

Insurers might expect to see fewer employers’ liability claims overall, but some third party liability
claims may be for higher amounts. In many cases the impact of new legislation is difficult to
predict.

© IFE: 2019 Examinations The Actuarial Education Company


SP7-09: External environment Page 23

A further example can be seen in the new noise regulations that were introduced in the UK in
2006, which reduced the number of decibels at which employers must provide hearing protection
for their workers. Although this measure has increased health and safety levels, it may have
actually increased the number of employers’ liability claims. This is because some employees who
are being exposed to a level of noise that is below the old threshold may now be able to claim
against their employer for negligence.

Other changes can affect the cost of insurance. For example the strictness of seat belt
rules, drink-driving rules and motor speed limits – and the strictness with which they are
enforced – should affect the number and severity of road accidents. It is usually impossible
to estimate accurately the effect of any such changes before they come into effect.

The insurer could make use of some information, such as the experience of overseas countries
that have already introduced similar legislation, to estimate the impact on its business. However,
as the Core Reading says, any estimate is likely to be fairly crude.

In particular, it will be difficult to predict:


 The degree to which legislation will be enforced, eg when checking whether motorists are
complying with seatbelt regulations, what proportion of cars will the police check, and
what penalties (if any) will they impose?
 The extent to which people will adhere to the new laws, which will depend on factors
such as social attitudes of people towards law enforcement or safety measures and the
extent to which the new laws are publicised.

Therefore, it will be important that insurers regularly monitor the impact of new legislation as
soon as it is introduced, so that they can reflect this in their business processes, eg premium
rates.

Legislation that affects insurance directly


Legislation may be aimed explicitly at affecting insurance.

One example is when insurance is made compulsory for people or organisations doing
particular things. Examples of this in most countries include:

 motor third-party liability insurance for car owners

 employers’ liability or workers’ compensation insurance.

Question

Describe possible impacts on general insurers of making certain lines of business compulsory.

The Actuarial Education Company © IFE: 2019 Examinations


Page 24 SP7-09: External environment

Solution

Possible impacts include:


 increased volume of business overall, which should lead to increased overall profits and
lower per-policy expenses
 premiums may need to drop to maintain market share, ie if more insurers join the market
 less opportunity for anti-selection, if people are forced to buy a policy
 possibly greater risk of moral hazard – eg if people are forced to buy a policy, they may be
tempted to claim fraudulently in order to get something back for their premiums.

A different example is the introduction of the Personal Injuries Assessment Board in the
Republic of Ireland. This was designed to make compensation amounts payable for
personal injury more predictable. It was intended that this would make legal action over
such injuries unlikely, thereby reducing the legal costs associated with them and ultimately
reducing the cost of insurance without reducing the compensation paid to injured people.

The impact on the Irish insurance market since the PIAB was introduced in 2005 has generally
been positive. Claims have been settled more quickly and at lower cost. Insurers have reduced
premiums as a result.

The previous chapter describes examples of the effect of the regulatory regime on the actions of a
general insurer.

Impact of legislative changes on the litigation process


Legislation also affects the process of litigation. Reforms that make it easier or cheaper to
bring or defend litigation or that affect the payment of legal costs may affect people’s
willingness to bring cases for compensation, impacting the costs of liability insurance.

No-win-no-fee arrangements

An example is when lawyers in the UK were allowed to conduct cases on the basis of
no-win-no-fee, with an enhanced fee for success. This led, as might have been expected, to
a large number of extra legal cases seeking compensation for injuries. It also led to the
emergence of a new class of insurance – after-the-event legal cover – in which the legal
costs that are foregone when a case is lost are recovered through the policy.

Under a ‘no-win-no-fee’ arrangement, the lawyer will only charge the claimant (eg an injured
employee) a fee if the claim is successful. The fee charged is more than it would be otherwise to
compensate for the fact that the lawyer does not get a fee if the case in not successful. Lawyers
can take out after-the-event insurance to cover the defendant’s legal fees where the case is
unsuccessful. (Often the courts will make the losing party pay the other party’s legal costs.)

© IFE: 2019 Examinations The Actuarial Education Company


SP7-09: External environment Page 25

The claimant is therefore not subject to any financial risk:


 if they lose the case, no fees are paid
 if they win the case, they will receive the court award – the legal fees may be deducted
from the court award or may be recoverable from the defendant.

The introduction of no-win-no-fee arrangements has therefore increased the popularity of


claiming for compensation. This was particularly marked in the UK when changes in the law made
it possible to recover legal fees from the losing party so that the claimant is entitled to the full
court award, with no deduction.

The Actuarial Education Company © IFE: 2019 Examinations


Page 26 SP7-09: External environment

2.3 Trends in behaviour and awareness


The behaviour of people and organisations directly affects the cost of insurance, and trends
in claims costs can often be traced to various trends in society. These may be in the things
people do or in the way they view their place in society, their rights and their obligations.

Increasingly litigious society


It has been observed that a large proportion of cases that might successfully be litigated are
never brought, possibly because victims:

 are unaware of their rights

 do not know how to go about the process

 do not want to dwell on past miseries, or

 are not sufficiently motivated to go through the process.

The risk of having to pay legal costs, which was the case in the UK before no-win-no-fee
arrangements were introduced, would also deter would-be claimants from attempting to obtain
compensation.

If the proportion of those eligible to seek compensation who do so is small, a change in the
proportion can have a significant effect on the amounts of claims paid. The general
tendency to seek compensation when it is likely to be available is known as ‘propensity to
claim’.

You may have heard this tendency referred to as an increasing ‘compensation culture’. It is
believed that it is becoming more acceptable for people to look for someone to blame for their
suffering and seek compensation from them. This is being seen more in some countries (eg US)
than in others.

Societal attitudes
Other societal attitudes are also important.

A societal attitude is the way in which we think of, and behave towards, others in our community.
Here we discuss changes in attitudes towards drink driving, crime and insurance.

Drink driving

For example, driving while drunk has long been illegal in most countries but in others it was
long considered to be socially acceptable; those who were prosecuted being regarded as
unlucky rather than ill-behaved. In many countries attitudes have changed and it is
generally regarded in many countries as something that responsible people do not do.
Consequently, drink driving may be less frequent now than before, with corresponding
effects on insurance claims.

There should be fewer, and less serious, accidents as a result of this change in attitude, reducing
the cost of claims on motor insurance policies.

© IFE: 2019 Examinations The Actuarial Education Company


SP7-09: External environment Page 27

This trend was reinforced in many places by legislative changes; for example

 a reduction in the level of alcohol in the blood that was permitted while driving
In some territories it is illegal to drive with any alcohol in your blood stream.
 the introduction of random breath tests

 the penalties for offending, which are generally being increased

 the resources devoted to enforcement, eg more police checks are being carried out.

Insurers themselves may have helped in this trend; for example by introducing or enforcing
strictly policy clauses that removed the insurance cover (except what was required by law)
when a driver was drunk. This would have had a direct effect on claims. It would also have
reinforced the idea that driving while drunk was socially unacceptable and would have
raised the cost of so doing, further deterring people.

Crime rates

Crime rates are another obvious area in which societal trends have a direct effect on
insurance costs. Crime often leads to insurance claims since theft, burglary and malicious
damage are often insured perils.

Crime rates tend to vary greatly from year to year, and trends in crime rates will also vary
considerably between countries. General insurance companies might actively engage in trying to
encourage policyholders to take steps to reduce crime, in the hope of reducing claim costs.

Attitude of people towards insurance

The attitude of people towards insurance will always affect the cost of insurance. Surveys
have often suggested that people view fraudulent claiming of insurance as a minor offence,
if an offence at all, especially when it takes the form of exaggerating a genuine claim rather
than presenting a claim that is wholly fraudulent. (This is particularly relevant with certain
products like single trip travel insurance.)

This gives rise to claims cost, as it is often impossible to detect these claims and it
increases the cost of claims handling. While this is a normal cost of insurance any change
in public attitudes will change the general level of claims cost.

Suppose you are using claims data for pricing (eg own insurer data) that already contains some
level of fraudulent or exaggerated claims. If there is no reason to expect this to change over time,
then this data could be used for projecting future claims. If, however, you expect a change in the
extent of fraudulent or exaggerated claims (eg due to changing attitudes), then you should make
adjustments to allow for this.

The Actuarial Education Company © IFE: 2019 Examinations


Page 28 SP7-09: External environment

Organisations encouraging the placing of claims

Some organisations encourage the placing (making) of claims, and increases or reductions
in their activity will affect the overall cost of claims.

These organisations are called claim management, or accident management, companies (or
sometimes, ‘ambulance chasers’). Their aim is to help individuals make compensation claims. For
example, they will arrange for lawyers, medical experts and expert witnesses on the claimants’
behalf. They will often actively encourage individuals to pursue compensation claims, although
the extent of this encouragement will depend on any marketing or advertising restrictions in
place.

In the US, some firms of lawyers specialise in personal injury claims, advertising widely for
people who may be able to make claims. A number specialise in particular diseases or
allegedly harmful substances, such as asbestos or silicone implants.

In other countries, similar roles may be taken by lawyers, trade unions or specialist firms
set up for the purpose, as happened in the UK after it became legal for lawyers to work on a
no-win-no-fee basis. These organisations tend to concentrate on liability for causing latent
claims (ones that manifest themselves some years after the cause), which means that the
effect on insurance is on the reserves, sometimes reserves for years of account that were
thought to be settled in full, rather than on current business.

We discuss latent claims later in this chapter. Reserving is covered in detail later in the course.

Staged accidents

There are also types of fraud that result in claims to insurance policies of innocent third
parties, for example, staged motor vehicle accidents.

An example of this would be where the car in front deliberately and suddenly brakes in order to
cause an accident. The owner of the car then pretends to have sustained serious, but difficult to
prove, injuries (such as neck whiplash or stress) and also fraudulently claims for damage to the
vehicle, which existed before the accident. The people committing staged accidents are often
part of organised crime gangs. Staged accidents are a big issue for motor insurers in many
territories, but this practice is gaining awareness among the general public.

© IFE: 2019 Examinations The Actuarial Education Company


SP7-09: External environment Page 29

3 Climate and environmental factors


The natural environment leads to many insurance claims, including some of the most
spectacular ones. Recent concerns about global warming have raised the prominence of
this issue, but normal trends in weather and the incidence of spectacular events, means
that weather related losses are inherently unpredictable from year to year. The
human-made environment can also be a cause of claims in ways that are not always
obvious in advance.

This section covers:


 the impact of the weather, including seasonal effects (which will depend on the
characteristics of the property being insured), and the possible implications of global
warming
 catastrophes, including weather-related events (particularly hurricanes), and factors
affecting the financial impact of these; earthquakes and examples of human-made
catastrophes are also discussed
 latent claims, giving lots of examples, and how insurers have tried to deal with latent
claims.

3.1 Weather

Seasonality
The most obvious way in which weather varies, in most countries, is seasonality. In
temperate climes there is the spring / summer / autumn / winter pattern; in tropical climes
there may be a dry season / wet season pattern or a monsoon season. The precise pattern
and the dangers associated with each phase will vary from country to country even within
geographic zones, with differences in weather patterns and building codes, among other
things.

Building codes are the standards to which houses, offices, bridges, etc must be constructed. For
example, in territories prone to earthquakes, such as California and Japan, there are regulations
to ensure that all new buildings are built to standards to withstand earthquakes of a specified
intensity.

In areas where the standard of building construction is high, insurers should bear lower losses.

In general, winter weather is harsher and for some classes is more likely to give rise to
claims: storm damage is more likely and driving conditions are likely to be more
treacherous, including the fact that there are fewer hours of daylight. This is rarely of
concern to insurers, since most policies are issued for a year and will be in force through all
four seasons. However, in extreme cases it may influence patterns used to earn premiums.

The unearned premium reserve (UPR) is usually calculated by taking a portion of premiums in
respect of the unexpired exposure period. Often, this is done on a straight averaging basis, eg for
an annual policy with six months to go it might be reasonable to take half of the premium.
However, if the risk is not uniformly spread over the year of cover, eg where the claim costs vary
according to the time of the year, the proportion of premium taken should reflect the expected
risk in the unexpired period.

The Actuarial Education Company © IFE: 2019 Examinations


Page 30 SP7-09: External environment

In calculating UPR, an allowance might be made for initial expenses. The calculation of the UPR is
discussed in more detail later in the course.

Subsidence and land heave


Although the weather in summer is generally more benign than in winter, the problems of
subsidence of buildings and heave are generally more likely to arise in the summer,
particularly when it’s very dry and hot. The shrinkage of land on which houses are built as
the ground becomes desiccated (dried up) leads to damage to the houses, which is
exacerbated when the drought breaks and the ground expands again. This can lead to a
large number of claims for structural damage to property, especially domestic property,
many of which can be large. Also, when the damage is not caused over a short period of
time, as it might be with a storm, catastrophe XL reinsurance may not respond to these
losses.

Heave is essentially the opposite of subsidence. Whereas subsidence involves a downward


movement of land due to a reduction in hydration, heave involves an upward movement due to
increased soil hydration. Heave could occur after trees are felled in an area because they will no
longer absorb moisture from the soil.

Question

Suggest why the catastrophe XL (excess of loss) reinsurance may not have covered many of the
subsidence claims.

Solution

Catastrophe excess of loss reinsurance covers catastrophic events. An event is defined as a


number of claims occurring within a short period of time (defined in the hours clause in the
reinsurance treaty). Many subsidence claims will have been deemed to occur outside of this
period, and so would not be counted in the total claim amount used to determine the reinsurance
recovery.

Location of property
Different areas are obviously subject to different climates, but the vulnerability of particular
properties to weather events will vary in ways that are not always obvious, and make
underwriting difficult for a mass product. Places close to each other will suffer almost the
same weather, but some locations are more sheltered than others and some will be more
prone to being flooded. Obviously, properties built on flood plains are prone to flooding, as
are those on low lying lands near the coast, but other vulnerable places may not be so
obvious: where water is channelled as it runs downhill it may make some hillside properties
vulnerable to flooding.

So two properties might be in the same small town but have a very different weather-related risk
because:
 one is in a sheltered, dry spot under a hill
 the other is in an exposed area, next to a river.

© IFE: 2019 Examinations The Actuarial Education Company


SP7-09: External environment Page 31

Some insurers are dealing with this issue by, for example:
 requesting more precise details about the location of the property and previous claim
history
 rating according to a more precise measure of location – eg using full postal code, rather
than just the first few characters.

Global warming
A number of serious weather losses have been linked to global warming.

The theory behind global warming is that increased carbon emissions is leading to increased
temperatures. This, in turn, is leading to climate change. Melting of the polar ice caps may also
lead to increased sea levels. Many experts now expect an increase in claims due to severe
weather-related events, such as floods, hurricane, storms and droughts, as a result.

Whether or not this is the true explanation is debatable, and there is some evidence that
major storms in the Caribbean and the Gulf of Mexico have fluctuated in number and
intensity as long as records exist.

However, even if you assume that it will happen, quantifying the rate of climate change is a
difficult challenge and open to debate.

The problem for insurers is two-fold:

 Firstly, it is not possible to be sure that long-term trends exist or, if they do, where
they will lead.

 Secondly, the effect on insurance is unclear.

It has been suggested that even small changes in climate can have a more than proportional
effect on insurance losses, but this is impossible to prove.

Some people consider that global warming will increase the volatility / unpredictability of
weather-related claims, which will impact insurers (even without a change in the mean).

Again, the nature of general insurance as an annual contract means that companies will be
able to adapt gradually to changes in the claims environment.

Question

Climate change is seen as a long-term effect. Suggest some short-term measures that general
insurers could take each year in response to the resulting adverse claims experience.

The Actuarial Education Company © IFE: 2019 Examinations


Page 32 SP7-09: External environment

Solution

A general insurer could respond to adverse claims experience by:


 increasing premiums, either overall or for certain risks
 changing its benefits, eg exclude specific perils
 strengthening underwriting or claims control measures
 no longer selling certain products
 restricting or attracting a different target market, eg stopping marketing activities in
certain locations
 strengthening its reinsurance programme.

However, if changes mean that some properties become substantially more vulnerable to
loss, it may affect their insurability and this can lead to political issues.

Many people feel that governments have a moral obligation to protect householders from
flooding. This is demonstrated through state-funded construction of flood defences, such as the
Thames barrier in London. In some territories, such as France and the United States, flood
protection is provided through government insurance or pooling arrangements. In others, such as
the UK, if flood defences fail or are inadequate, the householder may be left paying, and so will
need to buy private insurance.

The concern in the UK is that some people, eg those living in low lying properties or coastal areas,
will not be able to afford to insure their houses because the insurance premiums will be too
expensive.

To combat this concern, the UK Government and the insurance industry have set up a fund called
Flood Re, to provide flood insurance to high risk policyholders by taking the flood risk element of
home insurance from an insurer in return for a premium based on property’s Council tax band.
Flood Re was introduced in April 2016, and in April 2017 had 60 insurance providers signed up to
the scheme. Flood Re is financed by a levy on all insurers relative to their share of the home
insurance market.

3.2 Catastrophes
Catastrophic losses can take the form of one immense loss, such as an oil-rig explosion.
Alternatively, there may be many smaller insured losses, all stemming from a common,
identifiable event such as a hurricane.

One way to reduce the impact of catastrophic losses is to write business in a wide range of
geographical locations and across many classes. Catastrophe reinsurance will also help.

Catastrophes may be either natural or human-made in origin.

© IFE: 2019 Examinations The Actuarial Education Company


SP7-09: External environment Page 33

Examples
Natural catastrophic losses include:

Ice, snow, frost: Widespread property damage may arise from water damage caused by
burst pipes. There will also be many more claims for accidents from the
motor account.

Storms: Severe storms (eg wind, hail or rain) can cause extensive damage to
property. There may be a large number of claims from agricultural or
motor policies in a region hit by a hailstorm or household property
damage from wind storms or flooding (eg parts of the UK in the winter of
2015).

Earthquake: Potentially massive damage to property classes (eg Los Angeles


earthquake in 1994).

Human-made catastrophic losses include:

Fire: A large fire especially in hot, dry territories (eg Australian bushfires in
2017).

Air crash: Eg Shoreham Air Disaster (UK) in 2015. This could affect the aviation or
public liability classes. If the problem is a design fault, claims could fall on
the manufacturer’s product or public liability cover.

Explosion: Eg oil depot at Buncefield (UK) in 2005, the Piper Alpha oil rig in 1988.
Losses could hit property, employers’ liability, public liability and/or
consequential loss policies.

Terrorism: Eg terrorist attacks on the World Trade Centre in New York on


11 September 2001.

Hurricanes, storms
Although the US is cited for having the most expensive weather incidents, more recently,
events in New Zealand, Australia and South East Asia have reminded us that ‘weather’ is a
worldwide phenomenon.

The pre-eminence of the US in this regard is partly because of the concentration of high
insured risks and partly because of the vulnerability of the coast of the Gulf of Mexico and
the Atlantic states’ littoral area to hurricanes.

Littoral means pertaining to the coast.

The Actuarial Education Company © IFE: 2019 Examinations


Page 34 SP7-09: External environment

As mentioned before, the high total cost of insurance claims from weather-related events in the
US compared to other countries can be partly attributed to:
 the large number of properties in certain areas, particularly in cities along the eastern
coast
 the high proportion of properties that are insured
 the high average value of these properties.

In addition, the US states of Alabama, Florida, Georgia, Louisiana, Mississippi, North Carolina,
South Carolina and Texas are particularly prone to hurricanes.

It is normal seasonal behaviour for tropical storms to form over the Atlantic Ocean and
track in an easterly direction, some forming hurricanes in the Caribbean, the Gulf of Mexico
and on the US’s south-eastern coast. They tend to cause damage – sometimes very serious
damage – in Caribbean states, but the concentration of insured values there is low.
Whether or not they are serious events in global insurance terms depends partly on their
strength, but more on whether they affect the US and where in the US they land.

The most expensive weather incident ever recorded (at the time of writing) is Hurricane
Katrina, which hit Louisiana in the autumn of 2005. Although it was a very strong storm it
was not uniquely strong, and not in fact the strongest storm to hit the US in 2005 (that was
Hurricane Wilma slightly later that year). However, Hurricane Katrina passed almost
precisely over the city of New Orleans, which proved to be particularly vulnerable. This type
of effect adds to the uncertainty of underwriting property insurance in loss-prone areas.

Part of the reason for there being such extensive damage was that Hurricane Katrina weakened a
main levee (breakwater) protecting New Orleans. Six days later the levee broke, resulting in
flooding to approximately 80% of New Orleans and subsequent loss of lives, damage to property
and much looting in and around the city.

Rising cost of major losses

It has been observed that the cost of major losses has risen substantially. This is largely
due to economic development. Some of the areas in the US most vulnerable to storm
losses have been at the forefront of development. In particular, Florida is a major holiday
destination, which has led to a great deal of development near the coast. It is also a
low-lying state which makes it vulnerable to hurricanes.

Another cause of the increase in the cost of disasters is a general trend towards taking out
insurance. Insurance cover is not universal even in developed economies. In
less-developed economies it can be the exception rather than the rule. However, in almost
all countries property is more likely to be covered by insurance than it was some years ago.
This means that the proportion of economic loss covered by insurance in any catastrophe
is higher than it used to be. An obvious consequence of the difference in the proportions of
properties insured in various places is that a catastrophic event in a less-developed country
can cause only modest insurance losses whereas a similar event in a developed country
can give rise to very heavy losses.

© IFE: 2019 Examinations The Actuarial Education Company


SP7-09: External environment Page 35

Earthquakes
Earthquakes are occasional events that may lead to heavy insured losses. Geological
structures determine an area’s vulnerability to earthquakes in general, and the most
vulnerable areas are well known, although small events are not unknown elsewhere. Most
areas of greatest vulnerability are areas of low insurance intensity, but there are important
exceptions; notably Japan, including Tokyo, and the San Francisco and New Madrid areas
of the US.

On the other hand, the Asian tsunami of December 2004 – caused by an offshore
earthquake that was one of the largest ever recorded – caused about 230,000 deaths and
widespread devastation, but relatively little insured loss. In March 2011, an earthquake hit
the northeast coast of Japan, causing a 10-metre tsunami. Over 20,000 lives were deemed
dead or missing and over 100,000 buildings were totally destroyed.

Other natural perils


Other natural catastrophe perils include flood, typhoon, hail and volcanic eruption.

In some countries there are nationally-administered insurance schemes that may effectively
provide some or all of the cover for certain catastrophe perils.

Human-made catastrophes
Human-made catastrophes consist mainly of terrorist incidents, industrial accidents and
conflagrations.

A conflagration is a large destructive fire, which can be difficult to control.

Terrorism may or may not be covered by insurance, depending on local practice and law.
Terrorist incidents give rise mainly to property damage claims, but may give rise to liability
claims if security measures are found to have been inadequate.

In some territories, such as in the UK, claims arising from terrorist attacks are covered by the
government-backed arrangements. In these circumstances, insurance contracts tend to
specifically exclude claims resulting from terrorism. (In some cases, such as in the UK prior to the
2001 WTC attacks, insurers would cover claims up to a specified monetary limit, and the
government-backed arrangement would pay claims in excess of this.)

3.3 Latent claims


Latent claims derive from perils that were unforeseen when the policy concerned was
signed. Candidates should note however that the term is also applied to any insurance
claims that become known some years after the cause of loss. Most of these arise from
diseases caused by products or industrial processes, but faulty construction of buildings is
another possibility. Claims arising from the sexual molestation of people, particularly
children, is another example.

Most latent claims will therefore arise under product liability and employers’ liability insurance.
This is not necessarily the case however. Faulty building construction could probably be covered
under architect’s professional indemnity insurance or a contractor’s construction all risks (CAR)
cover. Child sex abuse claims may arise under public liability insurance.

The Actuarial Education Company © IFE: 2019 Examinations


Page 36 SP7-09: External environment

Examples of types of latent claim


The most notorious classes of claims of this nature are those arising from asbestos.

Numerous employers’ liability (and some product liability) claims are arising in respect of workers
that handled asbestos materials and products. Although some of those affected were exposed to
asbestos from as early as the 1940s, the resultant lung conditions (eg asbestosis and
mesothelioma) did not begin to materialise until about 1980.

Some other latent claim classes are listed below. These include claim causes that appear to
have run their course, others that are still in the process of manifesting themselves and
others that have caused concern, but may or may not develop into significant sources of
loss:

 Agent orange.
This was a chemical defoliant that was sprayed over Vietnam by the US army in the 1960s.
Many of the soldiers based there have sued against the manufacturers of the chemical for
consequent health problems to themselves and their families (ie for birth defects). Most
of these claims were made in the 1980s. More recently (2006), Vietnamese victims have
tried unsuccessfully to make claims.
 Radiation from mobile phones.
It has been suggested that this might be linked to an increased risk of some cancers
(particularly in children), headaches or sleeping problems. To date, no definite link has
been proven, but research continues.
 Benzene.
Exposure to benzene can cause serious health problems, including some cancers. The
chemical has been found in some carbonated soft drinks, which were, of course,
immediately withdrawn from sale.
 Diethylstilbestrol (DES).

This is a drug that was given to millions of women in the US in the middle of the 20th
century to reduce the likelihood of premature births. However, it has been linked to
genital abnormalities in daughters, and even potentially in granddaughters.
 Electromagnetic fields.
These are linked to the increased risk of leukaemia and other cancers.
 Pollution.
Exposure to pollution may last several months or years. The impact on health of being
exposed to polluted conditions may not be apparent for many years.

© IFE: 2019 Examinations The Actuarial Education Company


SP7-09: External environment Page 37

 Guns.
In the US, there have been attempted claims for compensation against gun
manufacturers:
– by victims of accidental shootings
– by city councils, for the increase in gun crime
– by gun users, where accidental injury has been caused.
To date, most of these claims have been unsuccessful.
 Noise-induced deafness.
Most commonly, this is due to working with, or being exposed to, noisy machinery
(eg with pneumatic drills or beside aircraft).
 Blood products infected with HIV or hepatitis.
During the late 1970s and early 1980s, large numbers of haemophiliacs became infected
with HIV or hepatitis after receiving tainted blood-clotting substances.
 Sick building syndrome.
The building people work in can be blamed for a range of illnesses, such as irritation to
the nose, throat and eyes, fatigue or headaches. This could be attributed to
micro-organisms within the air conditioning or the humidity of the building, but the
specific causes of these conditions are generally difficult to prove.
 Latex gloves.
Some people are severely allergic to latex rubber. Examples of compensation claims are:
– under employers’ liability coverages, from hospitals sued by workers that have
been made to wear latex gloves
– under product liability coverages, from glove manufacturers sued by customers
wearing, or patients treated by medical staff wearing, latex gloves.
 Lead paint.
Lead is added to some paint to improve its performance, eg in drying quickly. It can be
damaging to health, in particular hindering the development of young children. Now that
the dangers are known, it should only be used in certain circumstances, eg for painting
road surfaces. There are potential product liability claims against paint manufacturers,
and some public liability claims, eg against landlords.
 Bovine spongiform encephalopathy (BSE).

BSE is commonly known as ‘mad cow disease’ as it affects the brains of cattle. It was first
found in the mid-1980s, mainly in parts of Europe. However, 10 years later a brain
disease (known as vCJD) was found in humans, causing several deaths, and there is
evidence to suggest that some victims may have caught the disease by eating meat from
BSE-infected cattle.

The Actuarial Education Company © IFE: 2019 Examinations


Page 38 SP7-09: External environment

 Toxic mould.
There have been houses and other buildings in the US, particularly in Texas, where types
of mould that emit toxins have been claimed to cause health problems and damage to
property. Among the parties being litigated against are builders, architects and owners of
buildings (such as schools).
 Dalkon shield.
This is a contraceptive intrauterine device that was found to cause severe injury to a
disproportionately large number of its users.
 Repetitive strain injury (RSI).

RSI is a generic term used to describe a range of painful conditions of the muscles,
tendons and other soft tissues. It can affect the upper limbs, neck, spine, or other parts of
the musculoskeletal system. They are generally caused by performing work-related,
usually repetitive, tasks, and so they can lead to employers’ liability claims. Vibration
white finger (from using vibrating machinery, such as pneumatic drills) is a traditional
example, although conditions related to computer use (eg poor posture) are more
prevalent nowadays.
 Silica dust.
A fine silica dust can be produced when certain types of rock are cut, drilled, etc, which
can cause lung diseases if inhaled. Foundry workers and people working with the
products produced (eg potters and sandblasters) are most at risk unless proper
precautions are taken. It can take, say, 10 to 15 years following exposure before
symptoms develop.
 Tobacco.
Smokers and their families have taken tobacco companies to court for illnesses, injury or
death caused by long-term smoking. Most cases have been in the US, where some
medical insurance providers have also claimed compensation from the tobacco
companies.
 Year 2000 computer systems.
Towards the end of the 1990s, there was a huge fear that many computer systems and
products that relied on microprocessors would fail in the year 2000. This concern arose
because early computer programs often use a two digit code for the year component of
dates and the ambiguity of the date ‘00’ may lead to incorrect calculations. Products that
may have failed include computers, machinery, lifts and safety equipment. Failure on
safety equipment may also have led to employers’ liability claims. Companies and
organisations all over the world checked and upgraded their computer systems in
preparation for the ‘millennium bug’, and no significant computer failures occurred when
the time came.

© IFE: 2019 Examinations The Actuarial Education Company


SP7-09: External environment Page 39

 Nanotechnology.
Nanotechnology is the ability to work with materials on an extremely small scale, eg 100
billionths of a metre or less. This is still a developing field, but nanomaterials are already
being incorporated into many products worldwide, including cosmetics, paints, medicines
and food products. However, there is very little knowledge about how nanomaterials
may affect the long-term health of workers and consumers.
Any toxic tort litigation arising from nanomaterials could impact manufacturers,
distributors, secondary users (ie producers who incorporated nanomaterials into other
products) and retailers. Insurers writing employers’ liability, general liability and product
liability could therefore be affected.

Problems with latent claims


One problem with latent claims is that it is impossible to know where the potential claim is
lurking. Also, if the claim does materialise, the future claim cost is completely unknown.

For example:
 Will there be future employers’ liability claims for damage to people’s eyes from using
computers too much?
 If so, how much will the claim amounts be and how many people will be able to claim?

There is also the problem of identifying when exactly the claim event occurred, especially if
exposure (eg to the harmful substance or working conditions) was over many years.

Most latent claims arise in liability insurance. The normal form of these policies was the
occurrence basis in which a claim would always be paid from the year of account in which
the damage was caused. This leads to problems of definition: if a person who worked with
asbestos for a number of years, possibly with several different employers, contracts
mesothelioma some decades later, how can it be traced to a particular year of insurance?
The answer is that a legally imposed or industry agreed method of allocation must be found.

It may be difficult to identify the claim event date. However, the claim notification date should be
readily identifiable and objective.

Partly as a response to this the claims made policy was developed in the 1980s. This is now
the standard form of policy for professional indemnity insurance and some other liability
classes. It is intended to cover all claims that were first notified in the year of insurance.
However, the cover granted may be unsatisfactory from the claimant’s point of view, and
even more so from the point of view of a claimant who depends on the tortfeasor’s
insurance to obtain redress. When a claim arises the tortfeasor may no longer exist, and if
latent claims are emerging they may have trouble obtaining continued cover.

So, for example, say an employer took out liability insurance on a claims-made basis. If it is
known that the employer exposed its workers to hazardous conditions, it will be difficult for it to
get cover that is either affordable or comprehensive enough to cover future claims. This is
because insurers will fear a large number of claims being notified in the coming year.

For reasons such as this occurrence cover may be required in areas where insurance is
compulsory, such as UK employers’ liability.

The Actuarial Education Company © IFE: 2019 Examinations


Page 40 SP7-09: External environment

Liability insurance is intended to protect the insured against the cost of having to pay
compensation, rather than to protect the third party victim. However, such insurance is
compulsory to ensure that victims can be compensated. It is therefore common for victims
to be able to claim directly from insurers where the tortfeasor no longer exists, having
ceased to trade for example.

Latent claims are covered in more detail in Subject SA3.

© IFE: 2019 Examinations The Actuarial Education Company


SP7-09: External environment Page 41

4 Technological change
The business world changes at an accelerating pace. For insurers and their customers, it
has brought about increasing efficiencies but, as yet, it does not seem to have reduced the
need for human input, nor indeed hastened the arrival of the mythical ‘paperless office’.

It is now possible to do more complicated analysis than ever before, using vast quantities of data.
But however powerful the computers have become, they can’t (yet) do the thinking, and still need
to be operated by people who have an understanding of the problems to be solved.

Among the evolutions that are currently taking place, one might cite:

 increased computer power enabling much more complex models to be processed

 reduced cost of data storage, allowing more and more detailed information to be
stored and hence improving the accuracy of predictive models

 increasing availability of external data

 better strategy, planning and capital forecasting

 a growing market in modelling software solutions, including ‘machine learning’ and


the increasing sophistication of natural catastrophe models
Catastrophe models are covered in Subject SP8.
 increasing technological awareness of customers, enabling different methods of
sales, including web-based data transfer and social networking

 the growth in, and acceptance of, comparison websites


These comparison websites are commonly known as aggregators.
 the growing ability to handle claims online

 new risks emerging, such as ‘cyber risk’ from criminal activities relating to
technology

 increasingly complex rules and responsibilities around handling customer data; for
example, the EU General Data Protection Regulation (GDPR) – see Chapter 12 (Data)
for more details

 the potential in telematics, the insurer’s ‘eye’ in the insured vehicle, which can
transfer personal underwriting information on driving skill and usage
This is where an electronic device is installed in an insured vehicle in order to monitor the
location, movements and behaviour of a vehicle and its driver. The data received by the
device can be used in pricing the insurance risk more accurately.
 the growing ability to invest and switch, where not only are the products more
complex but even the decisions to buy and sell are generated electronically

 the impending arrival of driverless cars, where deciding whether the occupants or
the car’s manufacturers are liable for accidents will be among the challenges faced.

An insurer will need to embrace these new technologies and their associated implications
because their competitors will be doing the same. Any insurer that fails to do so is likely to be at a
competitive disadvantage very quickly.

The Actuarial Education Company © IFE: 2019 Examinations


Page 42 SP7-09: External environment

Technological change does not always mean increasing costs. The ‘information super
highway’ puts detail, data and comparative analysis at the fingertips of supplier and end
user, often at zero-cost, including some of the software solutions mentioned above.

In the exam, you will be expected to be able to give examples of technological advancements and
also describe how these might impact the operations and experience of an insurer.

For example, price comparison websites have increased the transparency and competitiveness of
the market and the sensitivity of consumers to changes in the rating structure.

Question

Suggest three more examples of how the technological advances listed above might affect an
insurer.

Solution

Increased computer power and more complex models could lead to:
 more accurate pricing
 higher costs, eg relating to IT costs and the hiring of expertise
 spurious accuracy and an over-reliance on the model’s output.

Better strategy, planning and capital forecasting could lead to more accurate reserves being held,
thereby increasing consumer protection and/or reducing the cost of capital.

The ability to handle claims online might tend to:


 accelerate claim settlement patterns
 reduce claim handling costs, eg stationary and postage costs
 improve customer satisfaction.

© IFE: 2019 Examinations The Actuarial Education Company


SP7-09: External environment Page 43

5 Glossary items
Having studied this chapter you should now read the following Glossary items:
 Hard premium rates
 Insurance cycle
 Losses-occurring (losses occurring during [LOD]) policy
 Ogden tables
 Risk-attaching basis
 Soft premium rates.

The Actuarial Education Company © IFE: 2019 Examinations


Page 44 SP7-09: External environment

The chapter summary starts on the next page so that you can
keep all the chapter summaries together for revision purposes.

© IFE: 2019 Examinations The Actuarial Education Company


SP7-09: External environment Page 45

Chapter 9 Summary
Among the economic factors affecting insurance business are claims inflation, the
underwriting cycle, investment conditions and currency movements.

Different insurance claims types are affected by different types of inflation, eg:
 household contents – mainly price inflation
 buildings and motor damage repair costs – mainly wage inflation
 medical expenses – medical cost inflation
 liability for personal injury – court award inflation (split into components).

Expense loadings are also subject to inflation – mainly salary-related. Failure to anticipate
appropriate levels of inflation can lead to inadequate premiums or reserves.

The underwriting cycle is a continuous cycle of hard (profitable) premiums, then increased
competition, followed by soft (less profitable) premiums, and then insurers leave the market.
Soft markets have often ended when a major disaster has triggered severe losses. Low
barriers to entry, the delay until profitability of business written is known, simplistic capital
regimes and marginal costing have facilitated the cycle.

The significance of investment conditions depends on the characteristics of the business


sold, eg greater for long-tailed business. If significant, investment return should be allowed
for in pricing, often using a risk-free rate.

The impact of currency movements depends partly on where business is sold and where
claims are made. Analysing business by currency can avoid distortions where there are
changes in exchange rates, although judgement needs to be applied in deciding how to apply
exchange rates in any analysis. Currency hedging can be used.

Among the legal, political and social factors affecting insurance business are court awards,
legislative changes and trends in behaviour and awareness.

Court awards have most impact on liability insurance claims. Claims may be on the basis of
breach of contract but are more usually on the basis of negligence. Although most cases are
settled out of court, court decisions set benchmarks for the negotiations.

Compensation awards for major bodily injuries can be complicated, and are split into several
‘heads of damage’. Punitive or exemplary damages are quite common in the US. In many
markets, compensation awards are becoming more generous, leading to increases in liability
insurance premiums. Unexpected increases in claims may lead to inadequate reserving.

The Actuarial Education Company © IFE: 2019 Examinations


Page 46 SP7-09: External environment

Chapter 9 Summary continued


Legislative changes can directly impact insurers, eg making certain types of insurance
compulsory. Changes in regulations to improve health and safety (eg at work or for
motorists) or the litigation process (eg introducing no-win-no-fee arrangements) will also
affect insurance claims.

Changes in the attitudes and behaviour of people and organisations directly affect the cost
of insurance. Examples of this are a greater propensity of people to seek compensation
(helped by the encouragement of the placing of claims for compensation with claim
management companies, for example), lower social acceptance of drink driving and a greater
tendency towards fraudulent or exaggerated claiming.

Among the climate and environmental factors affecting insurance business are the weather,
catastrophes and latent claims.

The seasonality of weather-related claims, if significant, may need to be allowed for in


earned premium payment patterns. Subsidence can be a problem in very dry, hot summers,
and can lead to large property damage claims, arising over a long period of time. A number
of serious weather losses have been linked to global warming.

An individual property’s location may need to be considered in some detail to be able to


assess its exposure to weather conditions accurately. However, if insurers fully reflect the
risk in their premiums, this could cause political difficulties, eg leading to some properties
being insurable against flood damage.

Major natural catastrophes include hurricanes and earthquakes. The largest insurance losses
tend to be in the US. This is due to the concentration of high insured risks and the
vulnerability of some areas to hurricanes. The cost of these claims is rising due to economic
development and greater use of insurance.

Human-made catastrophes consist mainly of terrorist incidents, industrial accidents and


conflagrations.

Latent claims are insurance claims that become known about some years after the cause of
loss. Most of these arise from diseases caused by products or industrial processes, and so
arise in liability insurance. The most notorious classes of claims of this nature are those
arising from exposure to asbestos, but there are many other examples.

It can be difficult to identify the occurrence date of a latent claim. Claims-made policies can
make it difficult to obtain continued cover, and so the claim-occurring basis is usually
required for compulsory types of liability business.

There are also technological factors affecting insurance, largely arising due to the increasing
capacity of computers and widespread use of the internet.

© IFE: 2019 Examinations The Actuarial Education Company


SP7-09: External environment Page 47

Chapter 9 Practice Questions


9.1 List twelve distinct examples of natural or human-made catastrophes that might lead to an
insurer experiencing large losses.

9.2 For each of the following statements, suggest two brief examples of commercial lines insurance
classes and two personal lines classes which illustrate them:

(i) ‘In a recession, amounts of written premiums often fall.’

(ii) ‘In a recession, amounts of claims incurred often rise.’

9.3 A general insurance company operating in a particular country is heavily exposed to domestic
Exam style
household insurance. One third of the company’s premium income is from household insurance.
The business is spread widely throughout the country.

On 1 December the country is to be hit by a severe storm, causing widespread damage across the
whole country: roofs blown off, walls collapsed, trees felled, river banks broken etc.

Explain which functions of the operation of the insurance company will be affected by the large
number of claims that are made as a result of the storm damage, and, by considering all aspects
of the insurer’s operations, describe the likely impact on the company.

You may assume that the company is unaware that the storm is about to take place but has
catastrophe excess of loss reinsurance in place. [12]

9.4 Describe the impact of different types of inflation on private motor insurance claims and
Exam style
expenses. [7]

The Actuarial Education Company © IFE: 2019 Examinations


Page 48 SP7-09: External environment

The solutions start on the next page so that you can


separate the questions and solutions.

© IFE: 2019 Examinations The Actuarial Education Company


SP7-09: External environment Page 49

Chapter 9 Solutions
9.1 Examples include:
 wind, storm or hurricane
 flooding
 tidal wave or tsunami
 earthquake
 major fire or conflagration
 volcano
 hail storm
 ice, snow or frost
 subsidence
 fire
 explosion
 escape of toxic chemicals
 widespread use of a damaging product
 widespread use of a damaging process
 terrorism
 crash: planes, trains, automobiles, ...

9.2 (i) ‘written premiums fall’

commercial:
 lower profits, so lower consequential loss insurance
 lower stocks, so lower contents insurance by businesses
 lower employment, so lower employers’ liability
 less construction, so less insurance for plant / machinery, indemnity of builders

personal:
 less house moving, so less mortgage guarantee
 less consumer expenditure, so less extended warranty
 possibly also some contents policies lapsing (on grounds of not being able to afford cover)
 possibly moving from motor fully comprehensive to motor third party, on cost grounds

The Actuarial Education Company © IFE: 2019 Examinations


Page 50 SP7-09: External environment

(ii) ‘claims rise’

commercial:
 crime rises, so higher claims from property insurance
 fraudulent claims under commercial fire, especially from arson
 businesses cut corners, so may be more claims from EL etc

personal:
 higher theft claims from household contents and theft from cars
 fraudulent claims under household contents may increase
 mortgage guarantee claims rise (bigger loss if house prices low)
 claims on unemployment, credit card, creditor insurance rise

9.3 Functions affected by the large number of claims

 claims department: recording, assessing, estimating and paying claims [1]


 reinsurance department: arranging recoveries from reinsurer [½]
 investment department: realising assets as required to pay claims [½]
 branch network: dealing with queries from brokers and policyholders [½]
 queries / publicity: dealing with enquiries from policyholders as to whether they are
covered [½]

Other more ‘fringe’ aspects

 computer department: ensuring the system has sufficient capacity [½]


 reinsurance department: reinstating the catastrophe reinsurance [½]
 claims reserving: at 31 December, there will be a large number of IBNR and unsettled
claims [½]
 premium rating: need to assess the likely cost of future storm damage [½]
 management: checking adequacy of free reserves, reviewing the strategy of high
exposure to household class [½]

Impact on the company

 the extent of reinsurance cover is critical [½]


 higher claim payments will affect the underwriting result [½]
 higher claims reserves (for reported claims and IBNR) will also affect the underwriting
result [½]
 investments may have to be sold at a loss [½]

© IFE: 2019 Examinations The Actuarial Education Company


SP7-09: External environment Page 51

 extra work loads may result in overtime payments for claims handling staff, and higher
expenses [½]
 reinstatement of catastrophe cover will probably require a small renewal premium
(affecting underwriting result) [½]
 future catastrophe excess of loss reinsurance premiums will be increased as the reinsurers
will aim to recover their losses [½]
 future household premiums will (probably) rise as the company itself will try to recoup
losses  this depends on the level of competition for this class [1]
 higher premiums should not result in a loss of business, on the assumption that other
companies will have experienced a similar result [½]
 the impaired underwriting result will have an adverse effect on the excess of assets over
liabilities, ie a lower solvency margin [½]
 all of this will be greater if the other classes of business written are also subject to claims
from the same storm [½]

This might have a number of knock-on effects, eg:


 lower dividends [½]
 slightly more cautious investments [½]
 more cautious reinsurance (balanced by a higher cost of reinsurance) [½]
 some restraint on growth of premium income. [½]
[Maximum 12]

9.4 In general, the importance of inflation depends on when and for how long the policies are in
force, and the type of claim that arises. [½]

Property damage claims

These claims are usually reported and settled quickly, and so inflation is not so important. [½]

The appropriate inflation measures affecting such claims would be:


 inflation of motor mechanics’ wages (ie related to standard earnings inflation), being a
major part of repair costs [½]
 inflation of car component costs (to replace damaged spare parts) [½]
 car price inflation, as an indication of the cost of total replacement (eg in a major crash
resulting in a write-off) [½]
 for cars where there is a large element of imports (either of parts or of the whole car), the
local currency strength would also be relevant. [½]

Bodily injury claims

These claims may have significant reporting and settlement delays, so inflation can have a big
impact. [½]

The Actuarial Education Company © IFE: 2019 Examinations


Page 52 SP7-09: External environment

Court award inflation is important since settlements are often guided by court awards, even if the
litigation does not go through the court process. [½]

Earnings inflation is also important, since bodily injury claims are usually related to loss of
earnings (because the injury can prevent the injured party from working). [½]

Medical expense inflation may also be relevant since the cost of rehabilitation will be accounted
for in bodily injury claims. [½]

Bodily injury claims will also be affected by judicial decisions or legislation that changes the
method used to calculate compensation amounts (eg changes to the Ogden tables in the UK). [½]

General office expenses

The main influence is salary inflation, since salaries normally form the bulk of a general insurance
company’s expenses. [½]

There will also be some effects due to price inflation, for example inflation appropriate to the cost
of office equipment (eg stationery and furniture). [½]

The inflation of expenses may be offset by improvements in efficiency over time. [½]

Claim expenses

For expenses associated with claims, again salary inflation will be the most important. [½]

Also, legal fee inflation will also have an influence, particularly on larger claims. [½]
[Maximum 7]

© IFE: 2019 Examinations The Actuarial Education Company


SP7-10: The Lloyd’s market Page 1

The Lloyd’s market


Syllabus objectives
1.4 Outline the key features of the Lloyd’s market.

The Actuarial Education Company © IFE: 2019 Examinations


Page 2 SP7-10: The Lloyd’s market

0 Introduction
A description of the Lloyd’s market was given in an earlier chapter, when we looked at the various
providers of insurance. This chapter now covers various aspects of Lloyd’s in more detail.

Question

Recall that Lloyd’s is not an insurance company. Describe briefly what Lloyd’s is.

Solution

Lloyd’s of London is a marketplace made up of members who provide capital and accept liability
for risks that are underwritten in return for their share of any profits that are earned on those
risks.

Lloyd’s is a key player in the worldwide general (non-life) insurance and reinsurance
market, with some £29.9 billion of gross written premium in 2016. It is licensed to
underwrite business in over 70 territories and can cover risks in over 200 countries,
according to local rules.

Lloyd’s accepts a wide variety of different business lines – including property, casualty
(liability), marine, energy, motor and aviation – on both a direct and reinsurance basis.

Lloyd’s is also well known for providing insurance on very large, complex or unusual risks, for
which it can be difficult to obtain cover elsewhere. Examples are cover for kidnap or ransom, fine
art, wind farms, private space shuttles and sports events.

It has a long history, during which it has faced some tough challenges and has undergone
some radical changes, but some of the principles underlying the way that risks are
underwritten at Lloyd’s have remained largely unchanged. Its unique structure means that
it has certain features that make it unlike any other insurance provider.

Section 1 gives a brief history of Lloyd’s.

Section 2 describes in detail how business is conducted at Lloyd’s.

Section 3 describes the capital and solvency requirements for individual members, and Lloyd’s as
a whole.

Section 4 pulls together the material in the previous two sections by giving a worked example of
how Lloyd’s operates.

Section 5 explains the management structure of Lloyd’s.

Section 6 describes the historical problems at Lloyd’s and how they led to the formation of
Equitas.

© IFE: 2019 Examinations The Actuarial Education Company


SP7-10: The Lloyd’s market Page 3

1 History of Lloyd’s
London was an important trading centre in the seventeenth century and this led to a
demand for ship and cargo insurance. Edward Lloyd’s Coffee House (first referenced in the
London Gazette in 1688) came to be recognised as a place for obtaining marine insurance
from rich individuals.

In 1771 the business ceased being run from a coffee house and became owned by a group
of ‘subscribers’: underwriters and brokers who had contributed to the cost of new
premises. The first committee of Lloyd’s was then established.

In 1871 Lloyd’s was incorporated in the UK by an act of Parliament. (The Lloyd’s Act of 1871.)

The 1906 San Francisco Earthquake was a major landmark in Lloyd’s history since it
established its reputation in the US (when underwriter Cuthbert Heath famously sent out the
order: ‘Pay all our policyholders in full irrespective of the terms of their policies’) and it also
led to the development of non-proportional reinsurance.

Whilst other insurers were disputing claims in the aftermath of this major disaster, Heath’s
decision to pay policies quickly and in full, irrespective of the policy wordings, helped San
Francisco to recover from the earthquake and established Lloyd’s as a major source of insurance
in the US. Cuthbert Heath developed the concept of excess of loss reinsurance in order to provide
protection against similar future disasters.

In 1925 a central guarantee fund (a shared capital resource that covered all Lloyd’s
underwriting) was established that still operates to this day.

The Central Fund is described in Section 3.2.

The 1990s saw major losses at Lloyd’s. This led to a process called ‘reconstruction and
renewal’, an element of which was the formation of a company called Equitas to manage the
related liabilities (see Section 6 for more details).

The Actuarial Education Company © IFE: 2019 Examinations


Page 4 SP7-10: The Lloyd’s market

2 How business is conducted at Lloyd’s

2.1 Lloyd’s members

Members and syndicates


Lloyd’s is not an insurer in its own right; it simply provides the necessary infrastructure
(premises, licenses and so on) to allow its underwriting members (also called Names or
members) to conduct business.

Question

Describe the two types of Name (or member) in Lloyd’s.

Solution

The two types of member in Lloyd’s are:


 individual Names, usually wealthy individuals
 corporate Names, which are limited-liability companies.

The role of the members is to subscribe to (or participate in) a syndicate, which is a
collection of one or more members who have agreed to back the underwriting activities of a
particular underwriter for one specific calendar year.

Liabilities or profits from that year’s insurance activities are allocated to the members in
proportion to their agreed participation at the start of the year, and on a ‘several’ basis,
which means that members are not liable for any other member’s liabilities.

We shall see later (in Section 3) that this means that each member has to provide sufficient
capital as security for the risks that they are exposed to.

Managing agents
A managing agent operates each syndicate. A managing agent may operate several
syndicates but each syndicate has just one managing agent. By 2017 there were 57
managing agents, managing a total of 105 active syndicates and a small number of
reinsurance to close (RITC) only and special purpose syndicates.

It is the responsibility of the managing agent to employ the underwriting staff and manage the
syndicate on the members’ behalf. The managing agent must be a company specifically
established for the purpose of managing a syndicate, and it may not carry out any other function.

Types of member and limits of liability


Nowadays the majority of the members (measured by premium volume) are companies
(corporate members) rather than private individuals. Private members may have unlimited
or limited liability, though no new unlimited liability members are admitted. Corporate
members have limited liability, and must be a separate legal entity to the managing agent,
although they are often owned by the same insurance group.

© IFE: 2019 Examinations The Actuarial Education Company


SP7-10: The Lloyd’s market Page 5

The difference between limited and unlimited relates to what mechanisms can be used to
collect liabilities: in essence, unlimited liability members can be made personally bankrupt
while limited liabilities can only be collected up to the amount of capital deposited with
Lloyd’s.

Integrated Lloyd’s Vehicles


Where a syndicate consists only of a single corporate member, which is owned by the same
company or is part of the same corporate group as the managing agent, this is known as an
Integrated Lloyd’s Vehicle (ILV).

This arrangement allows an insurance company to have control of the underwriting of the risks in
the syndicate.

With the growth of corporate membership, insurance groups have bought up all the
participation rights in a number of syndicates.

It makes sense to talk of these syndicates in the same way as if they were insurance
companies. Legally, the syndicate only exists for one year at a time, and the roles of
managing agent, member and syndicate remain separate. These syndicates are known as
‘Integrated Lloyd’s Vehicles’ (ILVs) or aligned syndicates.

There are still many syndicates with both corporate and private members who have no links
with the managing agent: these are known as ‘unaligned’ syndicates.

Members’ agents
Members’ agents advise private members on which syndicates to be subscribed to, and
carry out much of the administration of their membership. Corporate members do not have
to have a member’s agent, but private members must use one.

Question

Explain the difference between managing agents and members’ agents.

Solution

Managing agents are responsible for managing the syndicate on the members’ behalf. They act in
the interests of all members of the syndicate, which may include corporate members.

Members’ agents advise private members on which syndicates to be subscribed to, and carry out
much of the administration for them. They act in the interest of the member that appoints them.
Corporate members need not appoint members’ agents.

The Actuarial Education Company © IFE: 2019 Examinations


Page 6 SP7-10: The Lloyd’s market

2.2 Subscription business

Question

Define the London Market.

Solution

The London Market is that part of the insurance market in which insurance and reinsurance
business is carried out on a face-to-face basis in the City of London. It concentrates mainly on
providing insurance and reinsurance cover to companies.

One of the features that makes the London Market (including Lloyd’s) different from the
personal lines insurance market is that it is a subscription market. This feature is certainly
not unique to the London Market, nor is all London Market business written on a
subscription basis, but it is typical there and the way it works is bound up with how London
Market businesses operate.

Most types of Lloyd’s business must be sourced via a Lloyd’s broker. However, there are
exceptions – for example, some syndicates sell personal lines business directly to the public,
eg via direct telephone sales.

The process for writing business under the subscription (or slip) system was described in an
earlier chapter.

In essence, subscription business is just insurance, or reinsurance, written on a


coinsurance basis. The whole risk is divided up proportionally and each syndicate or
insurer (underwriter) takes a specified share of premium and claims. Failure by one
underwriter to pay claims does not affect the liabilities of any of the others.

A particular coinsurer’s percentage share of a risk is known as their ‘line’.

Coinsurance in the London Market has traditionally been arranged through the slip (or
‘subscription’) system. A broker describes proposed risks on a standard form (known as a
‘slip’). The broker circulates the slip around the market. Underwriters then sign the slip to
confirm the proportion of risk that they will accept.

There is usually (almost always) a ‘lead underwriter’ who sets the price and agrees terms for
the risk on behalf of the other underwriters on the risk. The lead will also usually decide
whether a claim can be paid or whether it should be resisted. The rest of the underwriters
are known as ‘following underwriters’. There is not a complete split, but underwriters will
quite often focus on leading or on following, as will companies or syndicates. People use
the terms ‘lead market’ or ‘following market’ to indicate that to an extent individual
underwriters and companies have these focuses.

So underwriters working for syndicates in the lead market tend to set premium rates and
underwriting terms, whereas those in the following market tend to adopt the premium rates set
by the lead underwriter.

© IFE: 2019 Examinations The Actuarial Education Company


SP7-10: The Lloyd’s market Page 7

The broker will sometimes try to over-place the risk: to receive offers for more than 100% of
the risk. If this happens, then, in agreement with the insured, the shares of the underwriters
are reduced (or ‘signed down’) so that they total 100%. If the broker is unable to find
capacity to place 100% of the risk, the terms of the insurance may need to be renegotiated.

2.3 Annual venture


The annual venture (AV) is the mechanism that allows investors to provide capital to underwriters
for one year at a time. Although many investors at Lloyd’s now see their involvement as
continuous, the AV continues to form the legal basis of activities.

Syndicates are reformed each year, and the current members have the right to re-subscribe
the same proportion of the next syndicate year. If they do not wish to do so, they may sell
this right to the highest bidder in an auction.

Three-year funding at Lloyd’s


For much of the business written at Lloyd’s, the long-tail nature of the liabilities implies that
it can take some time before their true cost can be determined. Each syndicate year of
account is therefore allowed to remain ‘open’, usually for a period of three years, before the
profit or loss is finally determined for that year. During that time, premiums received on
business written in the year are accumulated in a fund known as the Premium Trust Fund
(PTF) out of which claims and expenses are paid.

PTFs are the premiums and other monies that members receive in respect of their
underwriting at Lloyd’s, held by their managing agents in trust for them subject to the
discharge of their underwriting liabilities.

At the end of the three-year period, the managing agent would usually close the fund by
estimating the value of the outstanding liabilities and reinsuring them into the subsequent
open year of that syndicate. The reinsurance premium paid for this is known as the
reinsurance to close (RITC).

Once this transaction has occurred, the managing agent can determine the final result on
the closing year of account, and the profit or loss attributable to each member who
participated in that year. In some cases – for example, if the syndicate has ceased trading –
the RITC may be paid into another syndicate or a special Lloyd’s subsidiary. A year of
account may be held unnaturally open for longer than three years if the liabilities cannot be
reasonably quantified, due to fundamental uncertainty. In this case, the managing agent
will look to close the year of account in later years, when the liabilities are more certain.

With three-year funding, no profit is usually recognised until the end of the third year. For
example, for a syndicate writing business for the year 2019, profits would not usually be declared
until the end of 2021.

The Actuarial Education Company © IFE: 2019 Examinations


Page 8 SP7-10: The Lloyd’s market

Question

Suggest how reinsurance to close premiums might be assessed.

Solution

This is effectively the same process as establishing the reserves required at the year end. Hence,
an appropriate reserving method, such as the chain ladder method, could be used to assess the
reinsurance to close.

Under the Lloyd’s system of three-year funding, three annual funds (or ‘account years’) are
normally open for each syndicate at any one time.

For example, during the year 2019, separate funds would be open in respect of:
 business written during 2017
 business written during 2018
 business written during 2019.

The funds for business written prior to 2017 would be closed, eg the fund for business written
during 2016 would be closed at the end of 2018.

An exception to this would occur if, at the end of the third year, the syndicate could not arrange
any reinsurance to close and the fund was left open into the fourth year or beyond, ie where
there is still too much uncertainty over what an appropriate RITC would be.

Question

The following model describes a Lloyd’s syndicate that uses three-year funding:
 All premiums written in a year are paid on 1 January.
 For each £1 premium, 25p of claims and expenses are paid on 1 July in each of the three
open years, and there is a 30p reinsurance to close paid to a specialist reinsurer.
 £110m of premium is written in year X.
 Investment income is earned at 3% each 6 months.

Calculate the profit declared at the end of year X+2.

© IFE: 2019 Examinations The Actuarial Education Company


SP7-10: The Lloyd’s market Page 9

Solution

1/1/X fund = £110m premium

1/7/X fund  110  1.03  27.5  85.8

1/7/X+1 fund  85.8 1.032  27.5  63.5

1/7/X+2 fund  63.51.032  27.5  39.9

31/12/X+2 fund  39.9  1.03  33  8.1 , after payment of the RITC

The profit is £8.1m.

Note that in practice, an inwards RITC would often be paid into the fund in respect of the
outstanding liabilities on the previous syndicate year of account. This would be paid by the
previous year’s fund at the time that it conducted its own RITC exercise, ie at time 31/12/X+1.

Three-year funded accounts are covered in more detail later in the course.

Statement of Actuarial Opinion (SAO)


Each year, as at year end, the managing agent must produce a statement of actuarial
opinion (SAO) for each open syndicate year. The SAO confirms that the technical
provisions held for solvency are at least as large as the signing actuary’s best estimate.

All Lloyd’s syndicates writing general insurance business must appoint a qualified actuary (the
Syndicate Actuary) to provide the SAO. The IFoA provides guidance to actuaries on preparing
these statements.

The Actuarial Education Company © IFE: 2019 Examinations


Page 10 SP7-10: The Lloyd’s market

3 Capital and solvency


Managing agents carry out the technical insurance operations acting as agents on behalf of
members. For each year (separately) members undertake to accept risks and take the
profits or losses arising. A one-year group of members is called a syndicate.

3.1 Funds at Lloyd’s (FAL)


Since members are taking risks, they need to hold capital in case the PTFs (Premium Trust
Funds) turn out to be inadequate to pay all claims and expenses. Each member must
produce the amount of capital specified by Lloyd’s. The capital is held by Lloyd’s in trust,
and Lloyd’s has absolute authority to use it to pay claims or other liabilities arising from the
member’s activities at Lloyd’s. The capital fund of a member is called Funds at Lloyd’s
(FAL).

FAL may be lodged in two main ways:

 assets or

 via a letter of credit (a guarantee by a bank to provide funds when called upon to do
so by Lloyd’s).

3.2 Demonstrating solvency


Individual members must demonstrate solvency to the regulator, and must have sufficient
FAL if they wish to continue underwriting at Lloyd’s.

In addition, Lloyd’s in aggregate must demonstrate overall solvency (that is, based on the
aggregate of all members’ exposures or net liabilities) to the regulator (the Prudential
Regulation Authority, PRA) by holding additional assets centrally (known as the central
assets for solvency). The majority of these central assets consist of the New Central Fund
(NCF). If a member’s FAL proves to be insufficient then these central assets will be used to
meet claims.

Therefore, the total funds available to pay the claims for the syndicates are the funds held
by the syndicates, the members’ funds at Lloyd’s and the central assets.

The Central Fund is a fund of last resort. It is a safeguard for policyholders should a member fail
to meet insurance liabilities in full. Where a member has insufficient assets on an individual basis,
assets in the Central Fund can be earmarked to cover that member’s liabilities if it should prove
necessary (this is at the discretion of the Council of Lloyd’s).

All Names are required to contribute a set proportion of their authorised premium limit to the
Central Fund. The Fund can also received loans from syndicates’ premium trust funds, which are
repaid when the year of account is closed.

You may be wondering why the above Core Reading mentions a New Central Fund. The Central
Fund comprises an ‘old’ and a ‘new’ fund. The New Central Fund was established in 1996 as part
of the ‘Reconstruction and Renewal’ measures, which will be discussed in Section 6, and this
covers liabilities from 1993 onwards. The ‘old’ fund was largely used to fund Equitas.

© IFE: 2019 Examinations The Actuarial Education Company


SP7-10: The Lloyd’s market Page 11

In addition to the assets of the Central Fund, the Society of Lloyd’s has raised external funding
from capital markets by issuing subordinated debt, which has increased the central assets for
solvency purposes.

The Actuarial Education Company © IFE: 2019 Examinations


Page 12 SP7-10: The Lloyd’s market

4 An example of how Lloyd’s works


This section gives a brief description of how Lloyd’s works, from the point of view of an individual
Name.

4.1 The simplified tale of a Lloyd’s Name


Suppose that I was a wealthy individual and wanted to try making lots of money writing insurance
at Lloyd’s.

Having negotiated the hurdles and become a member with a certain writing limit, I would first
decide the maximum amount of insurance business I wished to write in the next year. Let’s
imagine it was £300,000 of premium income. The Lloyd’s authorities would require me to deposit
with them a certain minimum amount of capital and to make a defined contribution to the
Central Fund to enable me to do this.

I would appoint a member’s agent and authorise them to arrange to place me in syndicates for
the next underwriting year. Suppose that we can take this as 1 January to 31 December 2018.
Knowing that diversification reduces risk, I want my capacity spread over various syndicates,
specialising in different classes of insurance. Let’s say I ended up in 20 syndicates, with £15,000
capacity in each.

From now on let’s concentrate mainly on one of those syndicates, Syndicate X. In total, 500
different Names joined Syndicate X, with a total capacity for 2018 of £100m (called the stamp).
That means my share of all business on the syndicate is 15/100,000, ie 0.015%.

Since Lloyd’s works on the principle of coinsurance, I know that I will have a share of 0.015% of all
premiums, claims, expenses, investment income, and profits or losses for all business written by
that syndicate in 2018.

During 2018, in fact, my syndicate only writes £80m, ie only 80% of the syndicate’s capacity. This
doesn’t reduce my requirements for capital or the Central Fund levy.

The Syndicate X premiums for 2018 all go into a pot called the premium trust fund. The Syndicate
managers invest it for us, usually in short-term assets. As claims and expenses come in they are
met from the fund.

At the end of 2018 the Syndicate effectively ceases then forms itself again for 2019, but let’s
assume that I don’t wish to rejoin. The only Syndicate X pot I’m interested in is (in principle, at
least) the 2018 one.

The solvency of the premium trust fund is checked at 31 December 2018 and 2019. Let’s suppose
that it looks fine. (If it didn’t and extra money was required to support the expected losses, then I
would have to put up 0.015% of whatever extra total cost estimated was required to support the
fund.)

At 31 December 2020, exactly three years after it first opened, the fund is closed. At this point
the managers of the Syndicate decide how much money will be needed to meet the outstanding
liabilities from the cohort of business written by the syndicate in 2018.

© IFE: 2019 Examinations The Actuarial Education Company


SP7-10: The Lloyd’s market Page 13

This amount, the reinsurance to close (RITC), is then taken from the fund and paid to somebody
else who, in return, takes on the outstanding liabilities. What’s left in the fund (after payment of
the RITC) is the profit on the syndicate. Suppose it’s £20m. I receive 0.015% of that, ie £3,000,
when the sums are finalised, early in 2021.

A similar transaction takes place on all 20 of my Syndicates at that time. Some may make bigger
profits, others losses. My overall result depends on the syndicates I have been part of and how
my capital was split between them, and is unlikely to be the same as for any other Name.

Where does the RITC go?


It is normal for the RITC to be paid to the next open year on the same syndicate, in this case
Syndicate X’s 2019 trust fund. That way, since many Names stay on syndicates from one year to
the next, the transfer is between two groups of investors with many members in common.

You will realise, however, that this implies the 2018 fund would have received an RITC early in
2020, when the 2017 year was closed at 31 December 2019. Correct. That’s why the past history
of the syndicate has some relevance.

If during 2020 it became clear that the RITC received was too small to cover the claims we
eventually pay in respect of that liability accepted (after allowing for investment income), then
that’s tough. The 2017 Names have discharged their liability and we (the 2018 Names) are left
holding the proverbial baby. On the other hand, it can cut the other way, so that if the RITC
received proves too large, then we gain. Syndicate managers obviously need to be aware of the
conflicts of interest between different groups of Names.

What if the RITC can’t be agreed?


If there is just too much uncertainty at the end of the third year then it’s possible that the two
sides can’t agree an acceptable RITC. In this case the fund is not closed on schedule, but is left
open for another year. The hope is that after another year’s experience the uncertainty about the
future will have reduced enough to allow the year to be closed, at 31 December 2021 in our
example. If there is still too much uncertainty, then it remains open once more and we try again
at the end of the fifth year. Eventually we’ll be able to close it.

What if I had made big losses?


An individual (non-corporate) Name sometimes has unlimited liability (although most now have
limited liability). In that case, suppose a different syndicate, on which I had a 1% share, made
massive losses, let’s say £100m. Then I would be liable for a £1m share of those losses. This
would certainly wipe out my Lloyd’s deposits. I might have to sell everything I owned to try to
make up the difference.

If I couldn’t meet this cost then the Lloyd’s Central Fund would come into play, and make up the
shortfall so that the claims were paid in full. Notice that the Central Fund is for the benefit of
policyholders, not Names. I could still be bankrupted. (There is, though, a Names’ hardship fund
that I might just be able to tap into.)

The Actuarial Education Company © IFE: 2019 Examinations


Page 14 SP7-10: The Lloyd’s market

Now the richest person in the world might also have a share of 1% on this disastrous syndicate,
and pay up their £1m without even noticing it. The fact that they are still smiling doesn’t help me
at all. Coinsurance means that they are in no way liable to make good my losses, even though we
were Names on the same syndicate.

However, most Names now have limited liability. This mitigates the downside risk and so makes
becoming a Name a more attractive proposition.

Why does anybody do it?


The potential gains are, of course, the reason. Even through the worst years, some syndicates
and some Names still make a reasonable return.

© IFE: 2019 Examinations The Actuarial Education Company


SP7-10: The Lloyd’s market Page 15

5 Lloyd’s management structure

5.1 Corporation of Lloyd’s


The Corporation of Lloyd’s is a corporate entity, financed by subscriptions from the
members, both individual and corporate. The Corporation provides central premises,
administrative staff and services to enable members to transact business. It is led by a CEO
and several directors.

Franchise Board
The Franchise Board acts as a Board for the executive of the Corporation. It includes
executives (of Corporation), non-executives from managing agents and external non-
executives. The Franchise Board fulfils the governance role for all decisions that can be
delegated by the Council of Lloyd’s, and is Lloyd’s version of the board of a company.

5.2 Council of Lloyd’s


The Council of Lloyd’s is the governing body responsible for the overall direction of
Lloyd’s. It was established as a result of the Lloyd’s Act 1982 and consists of six working
members, six external members and six nominated members whose appointment must be
confirmed by the Governor of the Bank of England. One of the nominated members is the
Chief Executive.

The Actuarial Education Company © IFE: 2019 Examinations


Page 16 SP7-10: The Lloyd’s market

6 Reconstruction and renewal

6.1 Problems at Lloyd’s


During the 1990s, there were major upheavals and changes at Lloyd’s. These were brought
about in response to the enormous losses (totalling £7.9 billion) made at Lloyd’s between
1988 and 1992, which almost resulted in the collapse of Lloyd’s. These losses arose from
several sources including large catastrophic losses, the emergence of claims for asbestos,
pollution and so on, and poor underwriting.

The effect of these losses was accentuated by what is known as the ‘LMX spiral’.

The first step in explaining this spiral is to consider how the reinsurance market operates. An
insurer will pass the risk to a reinsurer, say for all losses above £1m. The reinsurer will accept
some of that risk, say up to £3m, and pass a higher layer to another reinsurer.

Whereas reinsurance normally spreads risk, LMX (London Market Excess of Loss
reinsurance) actually concentrated risks for a while. This concentration arose because
reinsurers ended up reinsuring themselves. One of the reasons why this happened was
that the information passed between the parties when risks were retroceded was
inadequate.

Due to this lack of data, the reinsurers did not know which risks they were accepting. Hence the
reinsurer who passed on the layer above £3m may well accept part of the same risk later on, once
it has been passed around the market.

As a result, some syndicates were to find in due course that they had unwittingly accepted
reinsurance on risks that they themselves had earlier placed in the market by way of
outward reinsurance. Hence the term spiral. It was only when claims arose that they
discovered that their reinsurance protection was much less than they had imagined.

Question

Suggest what made this problem even worse.

Hint: think of high layers.

Solution

The problem was made worse by the fact that as the layers increase, the number of reinsurers
that specialise in and are prepared to accept that risk reduces.

Because of the uncertainty associated with the LMX spiral (and problems with
asbestos-related claims), many syndicates were unable to close some years of account
because it was impossible to estimate the ultimate losses arising from them.

© IFE: 2019 Examinations The Actuarial Education Company


SP7-10: The Lloyd’s market Page 17

6.2 Reconstruction, renewal and the formation of Equitas


The losses led to a package of reconstruction and renewal that was crucial to the survival of
Lloyd’s, including:

 the admittance to Lloyd’s of limited liability corporate capital in 1994 (individual


Names too are now permitted limited liability and many have gone down that route)
Corporate capital now makes up the vast majority of the market capacity.
 the setting up in 1996, by Lloyd’s, of a reinsurance company, Equitas, that has
reinsured the outstanding liabilities from the 1992 and earlier years of account.

Equitas took on the reserves and premiums held in syndicates for those years, and received
further funding:
 from Lloyd’s own reserves, including its buildings, contributions from brokers, auditors,
underwriting agents, a charge on the ongoing membership and a loan
 through individual reinsurance contracts entered into with each Name whereby Equitas
took on the Name’s liabilities at Lloyd’s in return for a premium.

Equitas is a separate entity from Lloyd’s, and is outside Lloyd’s regulatory regime. Equitas is not
authorised to write new business.

The logic underlying the establishment of Equitas was that their concentration of resources
and exposures would enable them to deal with the outstanding liabilities more efficiently
than if all the individual syndicates (at one stage there were 430) were separately dealing
with their own liabilities.

Equitas enjoys economies of scale in investment, expenses and claims settlement. By being a
much larger entity than any one syndicate, its solvency position is enhanced. Also the results will
be more stable (you win some, you lose some).

Other aims of establishing Equitas were to:


 enable Names to retire from Lloyd’s if they wish
 restore Lloyd’s reputation in the world insurance market
 encourage new capital into the market
 allow syndicates to concentrate on the future rather than the past.

However, the ownership of Equitas has now changed.

In March 2007, a deal was approved between Equitas and Berkshire Hathaway that resulted
in subsidiary companies of the Berkshire Hathaway group of insurance companies:

 reinsuring all Equitas’ liabilities

 providing up to a further $5.7 billion of reinsurance cover to Equitas (increased to


$7 billion following the subsequent Part VII transfer)

 taking on the staff and operations of Equitas and conducting the run-off of Equitas’
liabilities.

The Actuarial Education Company © IFE: 2019 Examinations


Page 18 SP7-10: The Lloyd’s market

In June 2009, a Part VII transfer was approved that achieved finality for Names by
transferring all liabilities from them to Equitas Insurance Company. These continue to be
reinsured to Berkshire Hathaway companies.

Names here refers to those individuals that insured the 1992 and prior liabilities of Lloyd’s.

A Part VII transfer is form of risk transfer. It will be discussed in a lot more detail in Subject SA3.

The subsidiary of Berkshire Hathaway that is managing the run-off of Equitas’ liabilities has since
been renamed Resolute Management Services Limited.

© IFE: 2019 Examinations The Actuarial Education Company


SP7-10: The Lloyd’s market Page 19

7 Glossary items
Having studied this chapter you should now read the following Glossary items:
 Central fund (Lloyd’s)
 Closed year
 Committee of Lloyd’s
 Fund (or funded) accounting
 Funds at Lloyd’s
 Integrated Lloyd’s Vehicles (ILVs)
 Lloyd’s deposit
 Lloyd’s managing agent
 Lloyd’s special reserve fund
 LMX on LMX
 LMX spiral
 Managing agent
 Members’ agent
 Names (Lloyd’s)
 Open year
 Premium income limit
 Premiums trust fund (PTF)
 Realistic Disaster Scenarios (RDS)
 Reinsurance to close (RITC)
 Signing down
 Slip system
 Three-year accounting
 Underwriting agent.

The Actuarial Education Company © IFE: 2019 Examinations


Page 20 SP7-10: The Lloyd’s market

The chapter summary starts on the next page so that you can
keep all the chapter summaries together for revision purposes.

© IFE: 2019 Examinations The Actuarial Education Company


SP7-10: The Lloyd’s market Page 21

Chapter 10 Summary
Lloyd’s of London is a special insurance market, where wealthy individuals and corporate
bodies group together in syndicates to write insurance. Lloyd’s itself does not provide the
insurance.

Lloyd’s is a key player in the world-wide general insurance and reinsurance market, and
accepts a wide variety of different business lines, including some very large, complex and
unusual risks.

Members (or Names) are grouped into syndicates, which collectively coinsure the
underwriting activities of a particular underwriter for one specific calendar year. A
managing agent operates each syndicate.

Most members are companies (corporate names) and have limited liability. Private
members may have unlimited or limited liability. No new unlimited liability members may
enter.

An Integrated Lloyd’s Vehicle (ILV), also called an aligned syndicate, is where a syndicate
consists only of a single corporate member, which is owned by the same company or is part
of the same corporate group as the managing agent. Such a syndicate may be considered as
an insurance company, although legally it only exists for one year at a time and the parties
involved are separate entities.

Members’ agents advise private members on which syndicates to be subscribed to, and carry
out much of the administration for them.

Traditionally, business is placed in Lloyd’s and the rest of the London Market by specially
accredited brokers, using the subscription (slip) system. There is usually a lead underwriter
who will set the premium rates and underwriting terms for the risk, which are adopted by
the following underwriters.

Syndicates are reformed each year, and the current members can re-subscribe the same
proportion of the next syndicate year or sell this right to the highest bidder in an auction.

Three-year funding usually operates at Lloyd’s, whereby each syndicate year of account
remains open, usually for three years, during which time premiums less claims and expenses
accumulate in its Premium Trust Fund (PTF). At the end of the three years, the outstanding
liabilities are usually transferred to the next open year and a payment is made called a
reinsurance to close (RITC).

Each open syndicate year requires an annual Statement of Actuarial Opinion (SAO) to
confirm the adequacy of the technical provisions held for solvency.

The Actuarial Education Company © IFE: 2019 Examinations


Page 22 SP7-10: The Lloyd’s market

Chapter 10 Summary continued


Each member must hold sufficient capital, known as Funds at Lloyd’s (FAL) in case the PTF
turns out to be inadequate. The amount of capital is specified by Lloyd’s and it is held by
Lloyd’s in trust.

In addition, Lloyd’s in aggregate must demonstrate overall solvency to the regulator by


holding central assets for solvency, mainly from the Central Fund, which is a fund of last
resort. The Central Fund was established to meet liabilities should a member default with
insufficient assets and is primarily funded from members’ levies.

The Corporation of Lloyd’s is a corporate entity, financed by members’ subscriptions, which


provides central premises, administrative staff and services to enable members to transact
business. The Franchise Board acts as a Board for the executive of the Corporation.

The Council of Lloyd’s is the governing body responsible for the overall direction of Lloyd’s.

As a result of enormous losses in the late 80s and early 90s, Lloyd’s allowed corporate capital
from 1994 and established Equitas to run off liabilities from 1992 and prior years. Equitas is
now owned by the Berkshire Hathaway group.

© IFE: 2019 Examinations The Actuarial Education Company


SP7-10: The Lloyd’s market Page 23

Chapter 10 Practice Questions


10.1 Define a Lloyd’s syndicate, and explain the relationship between members and syndicates.

10.2 Define the following terms used in Lloyd’s:


 syndicate
 managing agent
 Integrated Lloyd’s Vehicle (ILV)
 members’ agent
 subscription (or slip) system
 lead market
 Premium Trust Fund (PTF)
 reinsurance to close (RITC) premium
 Funds at Lloyd’s (FAL)
 Central Fund
 Corporation of Lloyd’s
 Council of Lloyd’s.

The Actuarial Education Company © IFE: 2019 Examinations


Page 24 SP7-10: The Lloyd’s market

The solutions start on the next page so that you can


separate the questions and solutions.

© IFE: 2019 Examinations The Actuarial Education Company


SP7-10: The Lloyd’s market Page 25

Chapter 10 Solutions
10.1 A Lloyd’s syndicate is a group of members that collectively coinsure risks for one specific calendar
year. Each syndicate often specialises in a particular type of insurance. Each member who
belongs to a particular syndicate will contribute capital to that syndicate and will accept a portion
of the insurance risks written by the syndicate, the share of each member being predetermined
according to the amount of capital they have contributed. The profit or loss made by the
syndicate is then shared among its members in these proportions.

10.2 A syndicate is a collection of one or more members who have agreed to back the underwriting
activities of a particular underwriter for one specific calendar year.

A managing agent is a company specifically established for the purpose of operating a syndicate,
ie employing the underwriting staff and managing the syndicate on the members’ behalf.

An Integrated Lloyd’s Vehicle (ILV) is where a syndicate consists only of a single corporate
member, which is owned by the same company or is part of the same corporate group as the
managing agent.

A members’ agent advises private members on which syndicates to be subscribed to, and carries
out much of the administration for them.

The subscription (or slip) system is the face-to-face system used within the London market to
coinsure risks.

The lead market are syndicates with underwriters that tend to set the premium rates and
underwriting terms for risks under the subscription system.

A Premium Trust Fund (PTF) is the accumulated fund for a particular syndicate and year of
account, into which all premiums received are paid, and out of which expenses and claims are
paid.

The reinsurance to close (RITC) premium is the premium paid by a syndicate to reinsurer the
outstanding liabilities. It is usually paid at the end of the three-year period to another party,
usually the same syndicate, but in the next open year of account.

Funds at Lloyd’s (FAL) is the capital fund of a member, used as security to support the member’s
total Lloyd’s underwriting business. The amount of capital is specified by Lloyd’s and it is held by
Lloyd’s in trust.

The Central Fund is a fund of last resort, established to meet liabilities should a member default
with insufficient assets. It is primarily funded from levies on members.

The Corporation of Lloyd’s is a corporate entity, financed by members’ subscriptions, which


provides central premises, administrative staff and services to enable members to transact
business.

The Council of Lloyd’s is the governing body responsible for the overall direction of Lloyd’s.

The Actuarial Education Company © IFE: 2019 Examinations


Page 26 SP7-10: The Lloyd’s market

End of Part 1

What next?
1. Briefly review the key areas of Part 1 and/or re-read the summaries at the end of
Chapters 1 to 10.
2. Ensure you have attempted some of the Practice Questions at the end of each chapter in
Part 1. If you don’t have time to do them all, you could save the remainder for use as part
of your revision.
3. Attempt Assignment X1.

Time to consider …
… ‘learning and revision’ products
Marking – Recall that you can buy Series Marking or more flexible Marking Vouchers to have
your assignments marked by ActEd. Results of surveys suggest that attempting the
assignments and having them marked improves your chances of passing the exam. One
student said:

‘The marker highlighted areas of weakness and gave good, helpful


feedback on what I considered a very difficult assignment. I now know
the key areas to focus on in this part of the course.’

You can find lots more information on our website at www.ActEd.co.uk.

Buy online at www.ActEd.co.uk/estore

© IFE: 2019 Examinations The Actuarial Education Company


SP7-11: Risk and uncertainty Page 1

Risk and uncertainty


Syllabus objectives

2.1 Describe the major areas of risk and uncertainty in general insurance business with
respect to reserving and capital modelling, in particular those that might threaten
profitability or solvency.

The Actuarial Education Company © IFE: 2019 Examinations


Page 2 SP7-11: Risk and uncertainty

0 Introduction
Insurance contracts transfer elements of risk and uncertainty from customers to insurers. The
insurers accept these risks and uncertainties and therefore it is important that they are able to
control these if they are to survive and be profitable.

There are a number of risks surrounding the management of the insurance business
written, which are discussed in this chapter.

Any insurance company must be able to:

 determine the price at which it is prepared to accept risk

 assess the reserves required so that it can determine profitability and provisions
needed

 set capital to reduce the probability of becoming insolvent to within acceptable


limits.

This may require managing volumes of business and capital such that the insurance
company remains able to support the business it has written, as well as that which it is
writing – that is, stay solvent.

There is a risk that volumes of business may be lower than expected which could lead to lower
profits than expected and a failure to cover fixed expenses. There is also a risk that volumes may
be higher than expected. This would lead to strain on available resources (eg staff to handle
policy applications and claims) and higher capital requirements to back the expanding business
volumes. If capital held is insufficient, there is a risk of intervention by regulators or, in extreme
cases, insolvency.

0.1 Uncertainty
Uncertainty is the inability to predict the future with confidence. Because of the presence of
uncertainty, we need to consider the effects of possible deviations from the projected figures.

0.2 Risk
Risk is the possibility of variations in financial results, positive or negative.

It is important that a general insurer is able to identify the risks that it faces and assess the
suitability of methods available for managing those risks.

The greater the uncertainty, the greater the risk.

0.3 Uncertainty in cashflows


An insurer (or reinsurer) intends to have sufficient money, including capital, at all times to
cover all cash outgoings (including claims, expenses, reinsurance premiums, commissions
and so on), net of all inflows and assets (including reserves, premiums, reinsurance
recoveries, subrogation and third party recoveries, profit commissions, investment income
and so on), and produce an eventual surplus that meets the profit targets.

There is substantial uncertainty about the quantum and timing of the income and the outgo.

© IFE: 2019 Examinations The Actuarial Education Company


SP7-11: Risk and uncertainty Page 3

To decide on an appropriate level of capital to hold and reserves to set, insurers tend to
build models based on an analysis of past experience where possible, while adjusting the
resulting figures to reflect the requirements imposed by local and international regulators.

0.4 Allowing for risk when setting reserves


When it sets reserves or capital, the general insurer should assess the level of risk within
the current and future portfolio to decide on suitable levels to cover the risk of possible
adverse eventualities.

Therefore the insurer should assess the expected cashflows and ensure that the reserves cover
the expected net outgo. It is very unlikely that the insurer will get these estimates exactly right
and so the level of capital held should also allow for any uncertainty surrounding the estimate of
the net outgo.

We can summarise the risk in this context as the risk that the cashflows into and out of the
company do not match those expected when it determined the current reserves.

In some theoretical approaches to reserving assessment, a risk loading is added to the


‘best estimate’ figures, so that the resultant levels may be deemed prudent. Capital may be
set such that it is sufficient to meet solvency requirements with a high degree of probability.

We can express this prudence mathematically in terms of the variance or skewness, for
example. We can achieve this in other ways, such as by setting a return on capital
employed target, which will be reflected in the pricing.

There are many possible ways to subdivide the risks and uncertainties facing a company at
this point, for example:

 Random error: This covers the fundamental uncertainty (the insurable risk) that is
insured. ‘Random error’ may also be called ‘process error’.

 Parameter error: This covers the uncertainty arising from the estimation of parameters
used in a model. It often arises from inadequate / incomplete data.
 Data errors from systems and procedures: This covers the uncertainty arising from
the way past claims are reported and stored.

 Model error: This covers uncertainty in the selection of the model and how it is used.

 Adjustment factors: It is necessary to make a number of assumptions about how to


adjust observed data to reflect the future experience, each of which is uncertain.

 Market conditions: Insurance is not written in isolation. Legal, regulatory and


competitive effects can all influence the risk and uncertainty.

 Portfolio movements: Unexpected changes in the volume and mix of business can cause
an insurer uncertainty.

(The word ‘error’ is used interchangeably with ‘uncertainty’ throughout this chapter.)

The remainder of this chapter gives examples of how risks and uncertainties arise, under the
broad categorisations given above. Note however that there can be overlaps between these
categories, and some sources of risk may lead to uncertainties arising under more than one
heading.

The Actuarial Education Company © IFE: 2019 Examinations


Page 4 SP7-11: Risk and uncertainty

If you are studying Subject SP8, you may notice a significant overlap with the material in the
equivalent chapter in that subject.

Another way to subdivide the risks facing a company is to split them into different items of
experience, for example:
 claim frequencies
 claim amounts
 expenses and commission
 investment income
 new business volumes
 lapses.

Question

Suggest how these categories fit into the categories listed in the Core Reading above.

Solution

Data errors from systems and procedures – This is defined as relating to claims.

Model error – Modelling could be used for any of claims, expenses (and commission), investment
income, business volumes and lapses. (In fact modelling can be used for pretty much anything.)

Random error – This primarily relates to the number and amount of claims.

Adjustment factors – These will be needed whenever we are setting assumptions for future
business. Assumptions may be needed for claims, expenses, investment income, business
volumes and lapses.

Market conditions – Market conditions may affect:


 claim frequencies, eg from time to time, new claims will emerge, which will increase claim
frequencies
 claim amounts, eg through inflation
 expenses, eg through inflation, regulation
 commission, eg through the power of brokers
 investment income, eg during a recession
 new business volumes and lapses, eg through levels of competition.

Portfolio movements – This certainly relates to new business volumes and lapses, since these will
affect the volume and mix of business. However it may also be linked to:
 expenses, eg through per policy expense loadings
 commission, eg through broker deals.

© IFE: 2019 Examinations The Actuarial Education Company


SP7-11: Risk and uncertainty Page 5

However, there are many possible areas of overlap between different subdivisions, and no one
correct way of organising your thoughts.

The examiners would expect students to be able to describe all significant risks and uncertainties
relevant to an exam question, no matter which subdivisions or structure you choose.

The Actuarial Education Company © IFE: 2019 Examinations


Page 6 SP7-11: Risk and uncertainty

1 Process uncertainty  effect on claims


Process error refers to the inherent randomness underlying a book of business.

An insurer’s specific process uncertainties stem from:


 general claims uncertainty relating to the specific business written
 internal influences, eg changes in reserving philosophy or mix of business.

In Sections 1 and 2, we discuss a number of sources of process uncertainty. These are


complexities that arise as a result of the inherent uncertainty in statistical estimation.
Please note that this is not intended to be an exhaustive list and that other sources of
process uncertainty exist.

1.1 External sources of claims uncertainty


The amount and timing of an individual claim is always going to be uncertain. Similar
historical claims may give some idea as to how claims may develop, but there is no
guarantee that the development will be similar.

General insurance claims are extremely unpredictable. Even with a reasonable quantity of
reliable, relevant past data, it is impossible to predict claims accurately. Uncertainty exists over:
 claim frequency
 claim amounts
 claim payment patterns.

Claim frequency
Claim frequencies may be subject to random fluctuations, and may also change over time.

Changes over time may be due to a changing attitude of policyholders to claiming. In the United
States for example, policyholders have shown an increase in the propensity to submit claims.

This change in attitude could lead to a spiral effect. If an individual manages to make a successful,
but unusual, claim then due to the publicity that this is likely to attract, other policyholders in the
same position are also likely to claim.

It is often not clear if it is the attitude of policyholders that is fundamentally changing, or whether
the actions of lawyers, in encouraging people to make such claims, is the underlying cause of the
trend.

The interpretation of policy wordings can also affect claim frequencies.

© IFE: 2019 Examinations The Actuarial Education Company


SP7-11: Risk and uncertainty Page 7

Question

Suggest how policy wordings might affect claim frequencies.

Solution

Misinterpretation of policy wording leads to claims that the insurer had not intended to cover
under the policy and hence claim frequencies will be higher than expected.

If the insurer is required to meet the claim payment, it may set a precedent for many other
claims, further increasing claim frequency.

(Even if the insurer does not pay the claim, it may be involved in lengthy and costly litigation – this
will not affect claim frequencies, but will affect claim amounts and/or expenses.)

The policy document is a legal agreement between the insurer and the insured. Amongst other
things, it describes the events and circumstances under which losses are or are not covered. It is,
therefore, important to word the policy as precisely as possible, if it is to give the cover intended
and to prevent claims from being paid that were never intended to be included in the cover given.
There may nevertheless be some residual uncertainty as to whether certain types of event could
give rise to a legitimate claim.

Claim amounts
Claim amounts may also be subject to considerable variability.

For some types of business the size of possible claims covers a very wide range, and there is
consequent uncertainty as to whether the claims that have actually occurred can properly be
regarded as typical of what might be expected to occur.

Question

Give an example of how one class of business can give rise to claims of very different sizes.

Solution

In private motor business, the variability of property damage claims is relatively small, even
allowing for the possibility of a number of cars being damaged in, say, a motorway pileup.
However, the size of injury claims could range (in the UK) from a few hundred pounds to a few
million pounds.

The variance of aggregate claim amounts will also increase if there is non-independence of risks.
Therefore accumulations of risk will increase the uncertainty relating to the variability in claim
size.

The Actuarial Education Company © IFE: 2019 Examinations


Page 8 SP7-11: Risk and uncertainty

Accumulations of risk may occur due to the insurer’s business acquisition strategy (eg it might
target policyholders of a particular type) or they may arise inadvertently (eg there may be a large
concentration of policies taken out by individuals living close to the insurer’s head office).
Accumulations may also arise as a result of a catastrophe event.

Due to the variability in the size of claims, there may be uncertainty as to whether changes in
claim costs from year to year are due to changes in underlying risks or are simply the result of
random variation.

The level of random variation will be higher, the smaller the portfolio of business. This problem is
therefore greater for small companies (or small classes of business) where we would expect a
larger variation from year to year.

Claim payment patterns


There are typically delays between:
 the date of the claim event
 the date of reporting the claim
 the final settlement of the claim.

In addition, the date of the claim event itself may be uncertain. For example, for claims such as
subsidence or industrial disease, it is very hard to pinpoint the exact date of the claim occurrence.
In fact, it is likely that the claims arose over a period of time.

The delay between the claim event and the reporting of the claim will depend on the type of
claim and the speed at which policyholders report potential claims.

The delay between the claim being reported and settled will also vary by type of claim. Large
claims are likely to suffer the longest settlement delays, especially in liability classes where the
claims may need to be settled by the courts. Large claims are discussed later.

Changing development patterns


Claims development patterns may change over time. This may be due to a number of
factors.

For example, there may be political pressure on insurers to speed up the payment of claims
following a natural disaster, or payment / recording of claims may slow down due to staff
shortages.

Where development patterns change due to a known cause, such as those mentioned
above, then allowances can be made in the reserving process, although these are often
judgemental. However, there will be occasions where the development pattern changes
without explanation, and the impact of this may not have been included within the premium
or claims reserving process.

© IFE: 2019 Examinations The Actuarial Education Company


SP7-11: Risk and uncertainty Page 9

Question

Suggest two examples of factors that the insurer would be unaware of, but that could change the
development pattern.

Solution

Examples include:
 a change in the propensity of individuals to report claims quickly – this may be a gradual
change, eg due to longer working weeks (and so less free time to report claims)
 changes in processes of brokers regarding claims processing / reporting, for business sold
through brokers.

Demand surge
Following a major catastrophe, there will be increased demand for goods and services to an
unpredictable extent.

For example, the demand for builders may increase following a flood. This increase in
demand could force up the price for such goods and services to an unpredictable extent.

Higher prices could mean higher claim amounts.

Climate change
Over the last decade, global weather patterns have changed significantly from an insurance
point of view.

For example, global temperatures are slightly higher and severe weather events are
becoming more frequent and more severe in some regions, eg Asian typhoons. In other
areas, eg the Gulf of Mexico, the impact of climate change is much less clear. Various
agencies have produced climate models that predict further volatility in global weather
patterns.

Historical occurrence patterns for weather-related claims may no longer be appropriate. For
example, windstorm claims may now be more frequent and/or severe and could therefore
take a longer time to settle. Additionally, claims from areas traditionally not covered by the
major catastrophe models may also become more frequent or severe, eg Thai Floods.

Bodily injury claims


Some governments have introduced legislation concerning the payment of bodily injury
claims. The idea is that to indemnify the policyholder, the claim payment should be in the
form of income replacement, instead of a lump sum. This effectively increases
administration, resulting in higher claims servicing and reporting costs for a longer period
and hence higher claims handling reserves.

The Actuarial Education Company © IFE: 2019 Examinations


Page 10 SP7-11: Risk and uncertainty

Insurers could get around elements of this problem through the purchase of an annuity.
However, annuity prices can be volatile, and will also include a profit loading for the annuity
provider. Therefore, the insurer may prefer not to go down this route, and it does not
remove either the additional administration or reporting issues.

This type of legislation can also apply retrospectively.

This means that legislation may be applied to claims that have already been reported.

Differences in third party behaviour (eg lawyers actively seeking out


asbestos claimants)
The behaviour of third parties may also impact claim characteristics for certain classes.

For example, lawyers may actively seek out people affected by asbestos-related illnesses,
or PPI business. This would increase the claim frequency and may also have an effect on
severity.

Recall that PPI insurance is another name for creditor insurance. It has become the most
complained about financial product ever in the UK, as it was mis-sold to consumers on an
industrial scale.

Question

Suggest how claim severity might be affected by lawyers seeking out people affected by
asbestos-related illnesses.

Solution

If lawyers seek out people who have been affected by asbestos-related diseases, there may be a
general increase in awareness and/or an increase in publicity over asbestos cases. As a result,
there may be an increase in awards made by courts over asbestos-related claims.

Government legislation
Legislative actions can be divided into three main types:
 fiscal changes, such as increases in tax on insured items – many claims are settled on a
replacement basis (ie the insurer replaces the damaged item), so if the sales tax on that
item increases, the cost of replacing that item will increase and the claim cost will
increase
 changes in the law that increase the amount of cover being provided, such as removal of a
legal limit on compensation levels
 changes in the law that restrict or forbid the use of certain factors in underwriting.

© IFE: 2019 Examinations The Actuarial Education Company


SP7-11: Risk and uncertainty Page 11

In the first two cases, an insurer is unlikely to have foreseen such changes; therefore there may be
a sudden change in the reserves that need to be held. Furthermore, since premiums cannot be
changed retrospectively, the changes will adversely affect profits until some time after the
premiums or cover can be adjusted. The third type of change will be known about in advance, but
may expose the insurer to anti-selection for which the cost cannot be accurately assessed. This
may result in the need for higher reserves.

In the past, legislation has tended to increase the burden on insurers.

This will affect future claims and claim development patterns.

For example, legislation may have changed to require the insurer to pay for the cost of an
ambulance to attend a road traffic accident.

This legislation has been in force for many years in the UK, although many hospitals don’t claim
these costs from the insurer.

Such legislation can be retrospective, thereby including historical claims as well as incurred
but not reported (IBNR) claims.

The effect of economic conditions on claims


Many types of incident giving rise to claims are influenced by economic conditions, whose
changes are difficult to predict as regards both timing and extent. There is therefore a continuing
uncertainty as to the number and cost of the claims that will occur when conditions change.

Mortgage indemnity is one class that is heavily affected by economic conditions.

A number of economic variables could have a direct impact on claims. For example:
 inflation – this will directly affect claim amounts

Question

Outline three types of inflation that can affect claim amounts.

Solution

1. price inflation, which will affect the replacement costs of goods

2. earnings inflation, which will affect repair costs and loss of earnings claims

3. court inflation, which will affect claims that are settled in the courts

 unemployment – this could lead to certain sections of society being unable to afford
insurance, and so produce a different mix of business
 economic growth – this could lead to more sections of society being able to afford
insurance (and higher levels of cover in some cases), and so produce a different mix of
business

The Actuarial Education Company © IFE: 2019 Examinations


Page 12 SP7-11: Risk and uncertainty

 change in the value of the exchange rate – for business transacted in a currency other
than that of the country in which the insurer is based, there is a risk that the insurer’s
results will be adversely affected by changes in the exchange rate between the two
currencies; there will also be uncertainties stemming from currency mismatching between
assets and liabilities, and because it may be impossible to predict the currency in which a
claim will have to be settled.

Question

Suggest three examples of general insurance classes in which there is likely to be a high level of
uncertainty relating to the currency of the claim payments.

Solution

Marine, aviation and travel.

These are probably the most obvious but you could also mention product liability, commercial
motor etc.

In addition, the economic conditions can have a wider impact on the environment; for example,
crime rates may increase during recessions.

The rates of crimes such as theft and arson have shown considerable variation from year to year
and from country to country. General insurance companies might actively engage in trying to
encourage policyholders to take steps to reduce crime, in a hope of reducing claim costs.

1.2 Internal sources of process uncertainty

Planned or unplanned changes in mix


Different risks will exhibit different claims characteristics, eg claim frequency, severity, volatility,
timing of payments etc. The degree of uncertainty inherent in the business will therefore depend
on the mix of risks that have been written.

In addition, any changes in the mix of business will increase this uncertainty.

Question

(i) Suggest possible strategies that might lead to a change in business mix.

(ii) Suggest other reasons why the business mix might change.

© IFE: 2019 Examinations The Actuarial Education Company


SP7-11: Risk and uncertainty Page 13

Solution

(i) Strategies leading to a change in business mix


 strategic change in target market
 change in distribution channels used
 change in marketing method
 change in pricing structure
 introduction of new rating structures
 change in underwriting processes
 change in claims handling procedures

(ii) Other reasons why the business mix might change


 increase in anti-selection by policyholders
 change in the attitude to claiming
 change in fiscal regime, eg tax relief on certain groups of policyholders buying insurance
 change in regulatory regime, eg certain types of insurance becoming compulsory
 change in company reputation, eg a company becoming seen as a budget provider

Booked reserves different to best estimate


Although the reserving actuary will typically calculate the best estimate reserves, and
possibly reserves for financial reporting, the final decision in respect of reported reserves
(and maybe solvency reserves) will rest with the company directors. The directors may
decide to be conservative by booking a result that is more pessimistic than the best
estimate. If there is pressure on results, then the reserving actuary may be put under
pressure to produce a lower best estimate figure. If this is the case, then the actuary can
take a number of steps including discussion with other members of the profession.
Management decisions can have an unpredictable effect on published reserve levels.

Reserves booked will usually be greater than best estimate due to:

 desire for conservatism

 smoothing of results

 requirements of regulatory bodies

 influence of auditors.

New markets
Entering a new market or territory will incur expenses for the insurer, including set-up fees,
accommodation costs, fees to the regulator and legal costs. Some of these will be known in
advance with relative certainty. Others will be unpredictable.

The Actuarial Education Company © IFE: 2019 Examinations


Page 14 SP7-11: Risk and uncertainty

New distribution channels


Claims frequency, severity and development may be expected to vary by distribution
channel.

If a business acquisition or renewal strategy tends to attract policyholders with different


characteristics from those of the existing clientele, the resulting claims experience may differ from
that of the past.

For example, direct sales may be expected to develop more quickly than broker sales if
claims from broker sales are reported through the broker.

The internet is now the dominant sales channel for personal lines and smaller retail
products. The lack of a face-to-face meeting or a telephone call when buying a policy
certainly increases the possibility of fraud, which could affect frequency and claims
development patterns.

The number of distribution channels is likely to increase in the future, as insurers pursue
ever more innovative ways of attracting new business and reducing costs.

New channels may also create a knock-on effect on existing channels. For example, if the
internet channel increases, the broker channel may shrink, resulting in brokers offering
incentives to attract business. This will have an effect on premiums and claims patterns.

New claims handling procedures eg online


Some insurers offer an online claim reporting service. This may increase policyholders’
propensity to claim, thereby increasing claim frequency. It may also have other effects
such as increasing the speed of notification of losses.

It may also cause an increase in fraudulent behaviour; for example, policyholders reporting
non-existent claims, or exaggerating existing claims, which would increase severity and
possibly frequency. Both would be unpredictable, dependent on external factors such as
the economy, and would thus introduce additional uncertainty into the reserving process.

Increased use of profit share arrangements


Profit share arrangements may incentivise the broker to direct more business to the insurer.
Adjustments need to be made to reflect this, otherwise this may result in overestimated
premiums and reserves.

© IFE: 2019 Examinations The Actuarial Education Company


SP7-11: Risk and uncertainty Page 15

2 Process uncertainty  effect on other areas of the business


Process uncertainty can also affect other areas of an insurer’s business, besides claims. Some
examples are discussed below:

2.1 Broker mergers lead to more pricing power on brokers’ side


If a broker grows to a market dominant position, either through acquisition or organic
growth, it may demand increased commission payment from an insurer, in return for
placing business.

This will lead to uncertainty in future commission payments, and possibly the delegation of
underwriting authority.

This can be a particular issue for larger risks where both the insureds and the brokers are
far bigger than the insurers. It tends to be more of an issue when rating levels are falling.

2.2 Aggregators
During the last few years, a number of ‘aggregator’ companies have been set up, dealing
mainly with personal lines business. These companies find the best price for the customer
from a pre-selected panel of insurers.

Aggregator websites that operate in the UK include Compare the Market, Money Supermarket
and Go Compare.

Insurers may have to pay a fee or satisfy certain conditions to be a member of this panel,
which can have an uncertain effect on expenses.

There may also be different commission arrangements to those of a traditional broker, for
example, a per policy charge as opposed to a percentage of written premium.

2.3 Accounting changes


Accounting procedures may change from time to time. This may be the result of a decision by the
regulator, a change in market practice or simply an internal decision. This will incur additional
expenses through training staff and/or employing consultants. These costs can be unpredictable.

2.4 Off-shoring
During the last few years, there has been a trend for insurers to relocate some of their
back-office functions to different countries to access a cheaper labour market.

Back office functions include functions such as call centres and policy administration. In recent
years, India has been a popular place to off-shore functions to.

The Actuarial Education Company © IFE: 2019 Examinations


Page 16 SP7-11: Risk and uncertainty

Question

Suggest reasons why India has been a popular location for off-shoring.

Solution

India has a large pool of English-speaking people, who are technically proficient. It also meets the
criterion of ‘cheaper labour market’.

Such a move is likely to have high set-up and redundancy costs. This may or may not lead
to changes in processes, premium or claims treatment and processing etc.

Increased demand for labour in the target countries may lead to increased wage inflation in
those countries. This begins to erode any potential benefits that the insurer was hoping to
gain.

The insurer also faces the uncertainty of currency fluctuations, assuming the off-shoring
agreements are arranged in the off-shore currency.

2.5 Changes in tax rate


From time to time, governments may vary the level of tax, which will have an effect on the
insurer’s post-tax profits.

2.6 Levies
Levy payments can be used by the regulator to compensate policyholders in the event of
insurer default. If there are different levels of insurer default to those expected, there will be
an uncertain effect on the insurer’s levy contributions.

There may also be other (uncertain) charges, eg professional charges from auditors, or fines from
the regulator.

© IFE: 2019 Examinations The Actuarial Education Company


SP7-11: Risk and uncertainty Page 17

2.7 The effect of economic conditions on investments


The general economic cycle is difficult to predict and has a significant effect on investment
markets, most notably on investment returns.

Question

Describe how the government or central bank can impact investment returns.

Solution

The government or central bank may use various policies to control economic variables, such as
inflation and economic growth. These economic variables then influence investment returns.

During recessions, the value of many asset classes falls. This is due to a fall in demand for the
assets, and an increase in the uncertainty of risky assets.

In order to stimulate economic activity, the government might reduce interest rates in order to
increase spending (by reducing saving and increasing borrowing). Lower interest rates will affect
the value of many different asset classes.

Lower interest rates might increase inflation, which would then impact returns from various asset
classes.

Poor investment returns include:


 lower than expected investment income, which can lead to the reserves that are held
being inadequate to meet the liabilities
 movements in asset values, which can jeopardise an insurer’s solvency, (since solvency is
measured by comparing the value of assets and liabilities).

Movements in asset values can impair solvency if there is:


 a fall in certain sectors of the market in which investments are held, such as Japanese
equities
 a failure or adverse performance of individual assets or investments, such as Poorco PLC
 an adverse movement in currency rates, such as the Rand versus the Euro.

Investment return can sometimes be an important source of income for an insurer,


especially if it writes long-tailed classes of business.

The Actuarial Education Company © IFE: 2019 Examinations


Page 18 SP7-11: Risk and uncertainty

Question

Explain the previous sentence.

Solution

For long-tailed classes a considerable time will pass after the receipt of premiums before claim
payments are made. Therefore allowing for investment income has a significant impact.

The best way to minimise asset risks is to ensure that the assets match as closely as is practical to
the corresponding liabilities, eg by nature, term and currency. Unfortunately, with general
insurance business, this is usually impossible due to the uncertainty surrounding claim timings
and amounts.

Exchange rates
If the insurer holds investments in foreign currencies, which differ from those underlying
the liabilities, then future exchange rates will be a further source of uncertainty to the extent
that assets and liabilities are not matched.

2.8 The effect of economic conditions on expenses


General economic conditions will have an effect on expenses.

For example, an increase in inflation will have an effect on the average level of expenses.

Question

Suggest the key types of inflation that can affect expenses.

Solution

Expenses are likely to be linked primarily to wage inflation. To some extent they may also be
related to price inflation.

Economic conditions are very difficult to predict in advance and can change suddenly.

2.9 Traditional monetary policy now not as effective at controlling economy


Traditional monetary policy is becoming less effective at controlling the economy.

For example, the changing of short-term interest rates is less effective at controlling
inflation. This may be due to increased use of derivative instruments by financial
institutions, for example, using swaps or interest rate options to make their cost of
borrowing more certain.

Central banks may therefore attempt to control the economy in different ways, with
unpredictable effects.

© IFE: 2019 Examinations The Actuarial Education Company


SP7-11: Risk and uncertainty Page 19

2.10 New types of investment


With capital markets becoming increasingly complex, there is now a much wider range of
investments in which the insurer may choose to invest. Some of these investments may be
less certain than more traditional asset classes.

An example of this is securitised bonds, where the interest and capital repayments are contingent
on some event not happening, or on the business on which the bond is securitised making
adequate profits. The return on such investments is dependent on what it has been securitised
on and so could be very uncertain.

Some of these new asset classes may be restricted or prohibited by local regulation, both in
terms of eligibility and admissibility.

Admissibility refers to the rules that govern whether or not assets are allowed to be included
when determining if the insurer has sufficient capital to meet its regulatory capital requirements.

2.11 Globalisation of investment markets


With investment markets becoming more globalised, there is a wider range of assets in
which the insurer may invest, which may not be denominated in the insurer’s domestic
currency. Investment returns may be less certain.

Also, as mentioned above, if the assets and liabilities are mismatched by currency, then currency
fluctuations lead to additional uncertainty.

With large volumes of foreign investment flowing across borders, the investment markets
themselves may also be affected, and behave in unpredictable ways.

Suppose you had invested in an emerging market and were enjoying high expected returns. If
there was a sudden increase in demand for investments in this emerging market, then it could
affect asset prices and yields. It could also affect the exchange rate. Investor sentiment could
then reduce (or further increase) demand at any time, hence reversing (increasing) this impact.
Changes in investor sentiment may be hard to predict.

2.12 Influence of other investment markets


With increased globalisation comes increased interdependence within investment markets.

For example, a major stock market crash in the US will almost certainly affect European
stock markets.

Uncertainty in a foreign investment market can easily spread to the domestic investment
market.

The Actuarial Education Company © IFE: 2019 Examinations


Page 20 SP7-11: Risk and uncertainty

2.13 Competition
The level of competition can be affected by a number of factors. Two such factors are:
 globalisation of insurance markets
 changes / differences in regulation.

Any changes in the level of competition will have an uncertain effect on a general insurer’s mix of
business and claims experience.

Globalisation of insurance markets


Insurance markets are becoming far more globalised and insurers are more willing to write
business originating outside their home territory. This can lead to increased competition
between insurers, so similar business may not be as profitable as it has been in the past.

This can also have an uncertain effect on the insurance cycle.

Regulatory arbitrage
Many financial companies will base their head office in a territory where the regulation is
beneficial. This can distort competition within the insurance market (as there may be more
or fewer companies competing in that market than would be the case if the regulation was
the same everywhere), however this will not directly impact competition.

2.14 The insurance cycle


The insurance cycle is the observed tendency of insurance prices and hence profitability to
vary in a cyclical fashion over a period of several years. It can be very difficult to predict, as
the frequency and amplitude of the cycle can change every time the market turns.

How the insurer reacts to the cycle can also be an important factor.

For example, an insurer may decide not to follow the market down during a softening period
of the cycle, instead opting to lose some business and hopefully retain profitable business.

If this is the case, the insurer’s historical experience may not be a good indicator of future
outturn.

2.15 Expenses uncertainty


Specific influences on expense uncertainty stem from:
 uncertainty over commission and other sales-related expenses, eg for existing and new
distribution channels
 uncertainty arising from changes in operations, eg new markets and off-shoring.

© IFE: 2019 Examinations The Actuarial Education Company


SP7-11: Risk and uncertainty Page 21

3 Parameter uncertainty
Parameter uncertainty refers to the uncertainty arising from the estimation of parameters
used in a model. Given that any model is an artificial representation of a real-life situation
there will always be a certain degree of parameter uncertainty in the models that we use.

One of the requirements of a good model is that the parameter values used should be accurate
for the classes of business being modelled. However this is easier said than done. Indeed, there
might be several possible selections for a parameter, each appearing to be equally reasonable, yet
a judgement has to be made as to which to select.

Other requirements of a good model are discussed later in the course.

In this section, we discuss a number of sources which come under a broad heading of
parameter uncertainty. Please note that this is not intended to be an exhaustive list and that
other sources of parameter uncertainty exist.

3.1 Uncertainty arising from the data used


Data may be:
 of a poor quality
 internally inconsistent
 incomplete
 non-existent.

Poor quality data


There will be occasions where the raw data is poor; for example, claim / policy details may
be inaccurate, with claim dates recorded as being prior to policy inception.

Inconsistent data
Data may also have inconsistencies, for example, changes in claim recording procedures.

Incomplete and non-existent data


In some instances, data may be incomplete or even non-existent, for example, when a new
class of business is being written.

Insurers may also write business in new territories where they have relatively little
experience. This will lead to uncertainty in setting reserves or prices.

The Actuarial Education Company © IFE: 2019 Examinations


Page 22 SP7-11: Risk and uncertainty

Question

Suggest other reasons why data might be non-existent.

Solution

Data might also be non-existent for:


 unusual risks
 business that is not new, but has changed its terms and conditions to such an extent that
past data cannot be used.

Uncertainty at extreme values

Data may not be non-existent for a whole class of business – an insurer may have a significant
body of good quality data for the majority of risk cells, however it may lack adequate data in the
tails of any distribution.

When fitting a particular distribution to a set of data, it is usually very difficult to fit the tails
of the distribution. This may be because there is no data at these extreme values, or the
data that exists is too volatile to be usable. Assumptions will therefore have to be made
from what is available and theoretical approaches. This will give rise to uncertainty in the
model output. Care should be taken when interpreting the model’s output.

Certain events have little / no data

Certain claim events have insufficient data to build a reliable capital model, or historical
data may be deemed to be inappropriate. For example, global weather patterns may be
changing at such a pace that renders historical weather data obsolete.

On these occasions, assumptions will have to be made. These may be based on similar
classes of business, benchmark statistics or the modeller’s subjective judgement. In any
case, inadequate data will lead to uncertainty within the model, and these assumptions must
be validated based on expert judgement.

Format of data
Claims data can be stored in a number of different ways; for example, gross or net of
reinsurance, or inclusive or exclusive of claims handling costs. It is important to
understand exactly what is and what is not included in the data.

If there is any change in data storage protocols in the historical data, it should be
considered whether adjustments will have to be made, since this may affect the claims
development pattern.

Inadequate data supplied by third party claims handlers


There may be occasions when an insurer outsources its claims handling function, either to
a broker or a specialist claims handling firm. On these occasions, data recording will be out
of the hands of the insurer, and there may be some difficulty in checking data validity.

© IFE: 2019 Examinations The Actuarial Education Company


SP7-11: Risk and uncertainty Page 23

If different claims handlers are employed for different classes / sources of business, the
way in which data is recorded may be inconsistent.

There may also be delays in passing the data to the insurer and these delays may also differ
between claims handlers.

This will make it difficult to establish claims development patterns.

Question

Suggest how third party claims handlers may distort an insurer’s reserving data in terms of:
 claim frequency
 claim severity
 claim delays.

Solution

Claim frequency

Third parties may not inform the insurer of claims until some time after the claim has occurred.
Effectively, the insurer may hold a lower outstanding reported claims reserve and a higher IBNR
reserve.

Claim amount

Third parties may not spend as long validating claims as an insurer might. For example, they may
accept claim amounts submitted by the insured, without checking for reasonableness / getting
several estimates of the claim size. This could lead to higher ultimate claims.

Claim delays

There are likely to be delays in passing claims information to the insurance company. These
delays are likely to vary between different claims handlers.

3.2 Change in case estimate reserving philosophy


Reserving philosophy within a company will change from time to time. For example, if
claims handlers have under-reserved a case in the recent past, they may be inclined to
overestimate future claims to compensate.

There may also be changes in reserving philosophy following a change in senior personnel.

This could involve a change in reserving methods, or a change in the basis used for the reserve
estimates (within an acceptable range).

If changes in reserving philosophy are known, it may be possible to make adjustments.

The Actuarial Education Company © IFE: 2019 Examinations


Page 24 SP7-11: Risk and uncertainty

3.3 Large and exceptional claims

Large claims
Large claims can be expected to have different frequency and severity distributions to
attritional and catastrophe claims. They are also likely to have different development
patterns. There may also be differences in development pattern based upon the type of
large claim.

For example, a large windstorm claim may develop at a different rate to a large flood claim,
although both types of claim may be experienced in a property book.

It is normal practice to remove large claims from the development and project these
separately to attritional losses.

Uncertainty may also arise in how a large claim is defined. They could be defined as claims
over a particular threshold (possibly with a different threshold for different perils, often set
to achieve sufficient data and with an eye on the reinsurance programme), or large claims
may be a subjective management decision.

If the threshold for what constitutes a large claim is too low, then a large quantity of data will be
excluded from the attritional claims triangulation, and this will result in the triangulation data
(and the reserve estimates) being less credible.

However, if the threshold is set too high, then more large claims will be included in the attritional
triangulation data, and this will increase the volatility of the projection.

In practice, the definition of a large claim might be set at the retention limit for the
non-proportional reinsurance programme. This would make a projection of net of reinsurance
claims much easier. (Reinsurance reserving is discussed later in the course.)

If the threshold method is chosen, there is the additional uncertainty as to whether this
increases over time, and at what rate. Effectively the threshold would decrease going
backwards through cohorts due to the inflation assumption.

An insurer is likely to increase the definition of what constitutes a large claim periodically,
in order to allow for claims inflation and maintain the real value of the threshold. Hence
prior origin years might well have a lower threshold than the current origin year. The rate of
inflation to apply to the threshold limit is likely to be uncertain and will often differ from the
rate at which attritional claims inflate.

On some occasions, there may be an absence of large reported claims, and the reserving
actuary may wish to add a loading to reflect this fact. This will give rise to additional
uncertainty.

Catastrophes
Catastrophic losses can take the form of one immense loss, such as an oil-rig explosion.
Alternatively, there may be many smaller insured losses, all stemming from a common,
identifiable event such as a hurricane.

Catastrophes are typically hard to predict, so are hard to reserve for. Catastrophe modelling is
discussed further in Subject SP8.

© IFE: 2019 Examinations The Actuarial Education Company


SP7-11: Risk and uncertainty Page 25

One way to reduce the impact of catastrophic losses is to write business in a wide range of
geographical locations and across many classes. Catastrophe reinsurance will also help (more of
this later in the course).

Latent claims
Catastrophic claims can also result from sources that were unknown, or for which a legal liability
was not expected, at the time of writing the business. The cost of such claims cannot be
calculated with any accuracy for the purpose of setting reserves.

The first problem with latent claims is that it is impossible to know where the potential claim is
lurking. Secondly, if the claim does materialise, the future claim cost is completely unknown.

For example, will there be future employers’ liability claims for damage to people’s eyes from
using computers too much? If so, how much will the claim amounts be, and how many people
will be able to claim?

Judicial decisions can significantly affect the extent of an insurer’s liability for these claims. For
example:
 one court judgement can act as a precedent for other similar claims
 a single court judgement can apply retrospectively over many policies
 court award inflation is often significant.

3.4 Claims inflation not as expected


Inflation assumptions will often be required; for example, for calculating an initial loss ratio
for the Bornhuetter-Ferguson method.

The Bornhuetter-Ferguson method is discussed further later in the course, although it should be
familiar to you from your earlier studies.

Uncertainty regarding future rates of inflation, and in particular the rates of escalation of claims,
will affect the assessment of the financial outcome of the existing business and hence the
reserves required.

The actual inflationary experience will be a determinant in whether the chosen reserves are
too high or too low.

3.5 New distribution channels


Different distribution channels will have different expense profiles.

Question

Comment on whether the main expenses are fixed or variable for:


 broker sales
 internet sales.

The Actuarial Education Company © IFE: 2019 Examinations


Page 26 SP7-11: Risk and uncertainty

Solution

Broker sales may have a high variable cost and a low fixed (setup) cost.

This is because commission is incurred every time a policy is sold.

Internet sales may have a high fixed cost, ie setting up and testing the website, and very little
variable cost.

Some expenses are relatively predictable. Commissions paid to brokers and other intermediaries
are almost invariably expressed as a percentage of the premiums payable. As such, they do not
give rise to uncertainty in assessing the outcome of business already written. However, other
expenses are less certain, eg underwriting costs will depend on the level of, and time spent on,
underwriting.

Expense uncertainty also arises through a change in the relative proportions of business
coming from existing distribution channels.

If the mix of sales differs from what was expected (either between classes or between distribution
channels), so that a higher proportion of business is sold on higher commission terms, the average
commission rate will increase.

Also, if different brokers are paid different levels of commission, there might be a risk that the mix
of business by broker changes.

Additionally, where a differential rate of commission is paid on business acquisition to that paid
on renewal, there is a persistency risk in the spreading of these different commission rates across
future ‘level’ premiums. This applies equally to any business expense which is higher at the point
of policy acquisition and initial processing than it is at the renewal stage.

A further risk or uncertainty may relate to the recovery of commission on a policy proposed but
subsequently not taken up, or on early lapse where the distribution agreement specifies a return
of commissions paid.

While – on the whole – expenses for existing channels should be relatively certain, the expenses
for new distribution channels will be far less so.

It may be difficult to predict the expense profile of a new distribution channel.

Set-up costs of a new channel must also be factored in.

3.6 Planned or unplanned changes in mix


If the mix of business changes significantly, either as a result of the company pursuing a
particular strategy or through unknown causes, the development pattern is likely to change,
and in an unpredictable way.

© IFE: 2019 Examinations The Actuarial Education Company


SP7-11: Risk and uncertainty Page 27

4 Model (specification) uncertainty


In this section, we discuss a number of sources of uncertainty in the area of model
specification. Please note that this is not intended to be an exhaustive list and that other
sources of uncertainty in relation to model specification exist.

Model uncertainty arises from the choice of or specification of the model. This topic is
discussed again in Chapter 16, Stochastic reserving models.

Model error arguably gives rise to a greater degree of uncertainty than parameter error, as it
is less easy to detect.

Question

Suggest reasons why model error is less easy to detect than parameter error.

Solution

Parameter error may be identified using sensitivity tests on individual parameters. Insight can be
gained into the effect of varying different parameters, enabling the user to pinpoint the
parameters that have the biggest effect. Model error may be misinterpreted as a combination of
parameter errors, and hence is harder to detect.

4.1 Programming error


There is also the risk that errors can creep into the actual programming of the model; for
example, typing errors during model construction. This risk can be greatly reduced through
thorough checking and peer reviewing of the model.

4.2 Simulation error / too few simulations


The output of a stochastic model will be heavily influenced by the number of simulations
carried out: obviously, the greater the number of simulations, the greater the smoothness of
the output.

A ‘good’ stochastic model should use at least 50,000 simulations.

However, large and complex stochastic models can take a considerable amount of time to
run. Even with today’s computing power, the most complex models may take twelve hours
or more to run once on a sufficient number of simulations.

There is clearly a trade-off here. If the modeller has severe time constraints, there may have
to be a sacrifice in the number of simulations or in the iterations run to produce a result.

4.3 Model design

Incorrect dependencies
A number of the variables in the model will be correlated with one another; for example,
interest rates and claims inflation.

The Actuarial Education Company © IFE: 2019 Examinations


Page 28 SP7-11: Risk and uncertainty

Question

Describe the correlation between interest rates and claims inflation.

Solution

Higher interest rates tend to lead to lower levels of price inflation through:
 reduced demand-pull inflation, as borrowing becomes more expensive and saving more
attractive, which is likely to reduce demand from consumers and firms
 reduced cost-push inflation, as higher interest rates are likely to increase the value of the
domestic currency, making imports relatively cheap.

However, higher interest rates can also exert an upward pressure on price inflation. For example,
companies will tend to pass on the higher cost of borrowing to their customers, ie cost-push
inflation.

Other types of inflation, eg wage inflation and medical cost inflation, are often correlated with
price inflation as they may also be affected by these factors.

Dependencies are programmed based on expert judgement, and as such must go through
an appropriate validation process. The correlations can be regarded as additional
parameters, and it is essential that they are not overlooked.

Incorrect distributional assumptions in modelling reserve uncertainty


It is sometimes necessary to calculate a range of possible values for the reserves, in which
case distributional assumptions will be required.

Setting distributions for claim frequencies and claim amounts may be tricky (for the same reasons
as discussed above), and there is scope for both the distributions and the parameters used to be
wrong.

© IFE: 2019 Examinations The Actuarial Education Company


SP7-11: Risk and uncertainty Page 29

5 Glossary items
Having studied this chapter, you should now ensure that you are able to explain the following
Glossary items:
 Act of God
 Agents’ balances
 Bonus-malus
 Events not in data (ENIDs)
 Model uncertainty
 Parameter uncertainty
 Process uncertainty
 Protected NCD.

The Actuarial Education Company © IFE: 2019 Examinations


Page 30 SP7-11: Risk and uncertainty

The chapter summary starts on the next page so that you can
keep all the chapter summaries together for revision purposes.

© IFE: 2019 Examinations The Actuarial Education Company


SP7-11: Risk and uncertainty Page 31

Chapter 11 Summary
The reserves and capital held by a general insurance company needs to cover the risks being
accepted by the insurer.

The risks and uncertainties faced by a general insurer can be classified as:
 errors from systems and procedures
 model error
 random (process) error
 adjustment factors
 market conditions.

Process uncertainty
Claims uncertainty relating to process error stems from:
 external sources, such as:
– inherent uncertainty in individual claims (amount, frequency and timing)
– demand surge
– climate change
– legislative changes
– third party behaviour
– economic conditions
 internal sources, such as:
– changes in business mix
– booked reserves different to best estimate
– new markets
– new distribution channels
– new claims handling procedures
– use of profit share arrangements.

Process error also affects other areas of the business, and arise from:
 broker mergers
 development of aggregator sites
 accounting changes
 off-shoring
 tax changes
 levies

The Actuarial Education Company © IFE: 2019 Examinations


Page 32 SP7-11: Risk and uncertainty

Chapter 11 Summary continued


 economic factors (exchange rates, inflation, investment returns, new types of
investment etc)
 international factors (eg globalisation and other investment markets)
 competition and the insurance cycle
 uncertainty over expenses.

Parameter uncertainty
Parameter error results in reserving uncertainty, and stems from:
 the data used
– poor quality data
– inconsistent data
– incomplete and non-existent data
 particularly large / unusual risks:
– large claims
– catastrophes
– latent claims
 format of data
 inadequate data supplied by third party claims handlers
 change in case estimate reserving philosophy
 claims inflation not as expected
 new distribution channels
 planned or unplanned changes in mix.

Model risk
Uncertainty in a model’s specification arises from:
 model error
 programming error
 simulation error / too few simulations
 incorrect correlations in the model.

© IFE: 2019 Examinations The Actuarial Education Company


SP7-11: Risk and uncertainty Page 33

Chapter 11 Practice Questions


11.1 Describe how judicial decisions can exacerbate the uncertainty surrounding latent claims.

11.2 State the principal differences in the risk features of a general insurance policy compared with a
life assurance policy.

11.3 Outline the main risk and uncertainty features of property damage and liability classes of
business.

11.4 List the areas of greatest uncertainty for a general insurer when producing estimates of its claims.

11.5 A general insurance company has an established book of commercial property insurance policies.
Exam style
(i) State, giving examples, the three broad categories of claim size that the insurer might
face. [3]

(ii) Describe how each type of claim might be modelled. [5]

(iii) Suggest how the risks posed by each type of claim may be mitigated. [5]
[Total 13]

11.6 List the problems for an insurance company that may be associated with rapid growth in premium
Exam style
income. [2]

The Actuarial Education Company © IFE: 2019 Examinations


Page 34 SP7-11: Risk and uncertainty

The solutions start on the next page so that you can


separate the questions and solutions.

© IFE: 2019 Examinations The Actuarial Education Company


SP7-11: Risk and uncertainty Page 35

Chapter 11 Solutions
11.1 The development of latent claims is often uncertain: one court judgement can act retrospectively
over many policies, which can result in large losses for the insurer.

The effect of judicial decisions is very similar to that of inflation. In fact, the effect of judicial
decisions is often simply referred to as ‘court inflation’.

Court inflation results from court awards. The differences between court inflation and price
inflation are as follows:
 court inflation, historically, has been higher than price inflation
 court inflation tends to remain level for a period, then increase in sharp jumps when new
precedents are created
 court inflation is less predictable than price inflation.

From time to time, judicial decisions will set new precedents for the admission of certain claims,
and the amounts at which they will be settled.

Decisions relating to imprecise policy wordings can lead to the admission of new types of claim
that had not been allowed for in the original costings. Liability claims are particularly exposed to
this type of risk.

Courts also periodically set new levels of award or compensation for existing categories of claim.

The effect of such awards will be to increase immediately the average amount at which all future
claims of a similar nature are likely to be settled …

… including those that have already been reported.

Such awards are very hard to predict, so it is even harder to allow for this form of inflation than
normal claim inflation.

Claim payments that are intended to represent the future lost earnings of an individual, following
an accident, are likely to be based upon the present value of that future income.

The courts may from time to time change the rate of interest at which insurers are allowed to
discount future earnings.

This change could have a dramatic effect upon the overall claim payments.

Court awards can be impacted by decisions made in other countries too.

11.2 Differences between general insurance and life insurance:


 General insurance (GI) policy terms are usually one year, life assurance (LA) policies are
much longer. (This leads to greater competitive pressure to attract new business every
year and greater importance of recovering fixed expenses each year.)
 GI claims verification is more difficult than LA (ie checking whether a person is dead or
alive), hence settlement delays tend to be longer for GI.

The Actuarial Education Company © IFE: 2019 Examinations


Page 36 SP7-11: Risk and uncertainty

 Similarly can only claim once on a LA policy, multiple claims are possible and often
expected on commercial GI policies.
 Also payout for LA policy is fixed upon the event, GI claim amounts are often highly
uncertain.
 GI polices are subject to more policy endorsements (alterations) than LA.
 For most LA policies the level of risk, ie mortality, increases over time. This is not usually
true of GI policies which may have level, increasing, decreasing or seasonal risk incidence.
 Tends to be better quality data in LA than GI and it is easier to quantify the risks in LA.

11.3 Property damage

 Short tail, so shorter time period, hence less time for events to go against the insurer.
 Geographic accumulations possible. Potentially lots of claims from weather-related
incidents.
 Reasonably homogeneous risks.
 Amounts limited by sum insured or value of building.

Liability

 Claims frequency lower on average but more variable.


 Amounts much more uncertain too, possibly unlimited cover.
 Long tail so more exposure to unexpected inflation, asset inadequacies and reinsurer
failure.
 Varied heterogeneous risks.
 Possible accumulations due to concentrations of products or employees.
 Exposure to court awards and judicial inflation.
 Potential exposure to latent claims.

© IFE: 2019 Examinations The Actuarial Education Company


SP7-11: Risk and uncertainty Page 37

11.4 Areas of greatest uncertainty for a general insurer when producing estimates of its claims are:
 claims emerging from the latest period of exposure, ie IBNR claims
 the possibility of new types of latent claims emerging from liability classes
 inflation of the longest-tail liabilities
 catastrophe claims, eg the uncertainty surrounding huge floods due to climate change
 claims arising from unexpired risks.

11.5 (i) Types of claim

The general insurance company is likely to face:


 attritional claims – these are typically unremarkable claims, which make up the majority
of the claims by number (although not necessarily by cost) [½]
 large claims – these will be larger and less frequent than attritional claims [½]
 catastrophe claims – these are rare by nature, but could be very large. [½]

Examples:
 attritional claims may include minor damage to buildings (or contents), eg as a result of a
fire in part of the building, or theft [½]
 large claims may include the complete destruction of a large building (and its contents),
eg a large fire [½]
 catastrophe claims may include an earthquake or a bomb, which destroys many insured
buildings in one particular area. [½]
[Total 3]

(ii) Modelling the claims

Attritional claims

Attritional claims are likely to be modelled using the general insurance company’s own past data.
[½]

This may be sufficient in quantity, since the insurer is established in the market, and attritional
claims (by their nature) are relatively homogeneous. [½]

It may be necessary to adjust the data before modelling to remove large and catastrophe claims.
[½]

From the past data, the insurer should be able to find reasonable estimates of both the expected
claim frequency and the expected claim severity … [½]

… and also variances around these rates. [½]

The Actuarial Education Company © IFE: 2019 Examinations


Page 38 SP7-11: Risk and uncertainty

Large claims

Large claims are harder to model, as they tend to occur less frequently. Therefore there may not
be a sufficient number of them to model in the same way as attritional claims. [½]

If there is sufficient volume to model large claims based on past experience, it might still be
necessary to adjust the historical frequency to reflect anticipated experience. [½]

If not, then the insurer may have to use a more broad brush approach, eg using loss ratios. [½]

If excess of loss reinsurance has been purchased, then an alternative approach would be to cap
large claims at the excess of loss threshold and allocate the reinsurance cost back to individual
policies. [½]

In order to do this, it is necessary to make an assumption about the relative propensity for a claim
to be large between different groups of policies. [½]

Catastrophe claims

Catastrophes present similar problems to large claims. By their nature, they are rare and there is
unlikely to be adequate data volume to enable the insurer to price or reserve accurately for the
future risk. [½]

The insurer could use a proprietary catastrophe model to understand the impact of extreme
events on a portfolio. [½]

However, the actual effect of a catastrophe often differs from that predicted and uncertainties
will always remain about the frequency, timing, size and location of such events. [½]
[Maximum 5]

(iii) Mitigating the risks

Attritional claims

Pooling of a large number of similar risks can help to mitigate the risks for attritional claims. [½]

Effective underwriting can also help to ensure that the premium is sufficient to cover the risks
being written. [½]

Reinsurance, eg quota share and/or aggregate XL may also be effective. [½]

Large claims

The insurer may not have a sufficient enough volume of large risks for risk pooling to be effective.
In this case, the company is at risk of there being an unusually high number of large claims in a
single year, or the (expected) large claims could be for unusually large amounts. [1]

This risk can be mitigated by the purchase of reinsurance. [½]

Risk excess of loss reinsurance will be appropriate for this purpose. [½]

In addition, the reinsurer may be able to assist the insurer in modelling the large claims. [½]

© IFE: 2019 Examinations The Actuarial Education Company


SP7-11: Risk and uncertainty Page 39

Catastrophe claims

The insurer may be able to mitigate catastrophe risk through the purchase of appropriate
catastrophe reinsurance contracts. [½]

Alternatively / additionally, it could mitigate catastrophe risk using:


 a spread loss treaty, which may be effective at spreading the cost of a catastrophe over a
number of years [½]
 an industry loss warranty, which will be effective if the experience suffered by this insurer
is in line with market experience [½]
 a securitisation of the book of business (ie issuing catastrophe bonds), which will transfer
(some of) the catastrophe risk to the capital markets [½]
 weather derivatives, which may be effective for weather-related catastrophes. [½]
[Maximum 5]

11.6 Problems that may be associated with very rapid growth in premium income include:
 if the increase is due to low premium rates, the increase may indicate future losses [½]
 could indicate deteriorating experience because of anti-selection or reduced quality
underwriting [½]
 administration strains could cause service standards to brokers and policyholders to fall
leading to bad publicity [½]
 solvency margin could be reduced close to the statutory minimum level if the minimum is
based on premium income [½]
 internal controls may be weakened, eg risk management, expenses. [½]
[Maximum 2]

The Actuarial Education Company © IFE: 2019 Examinations


All study material produced by ActEd is copyright and is sold
for the exclusive use of the purchaser. The copyright is
owned by Institute and Faculty Education Limited, a
subsidiary of the Institute and Faculty of Actuaries.

Unless prior authority is granted by ActEd, you may not hire


out, lend, give out, sell, store or transmit electronically or
photocopy any part of the study material.

You must take care of your study material to ensure that it


is not used or copied by anybody else.

Legal action will be taken if these terms are infringed. In


addition, we may seek to take disciplinary action through
the profession or through your employer.

These conditions remain in force after you have finished


using the course.

The Actuarial Education Company © IFE: 2019 Examinations


SP7-12: Data Page 1

Data
Syllabus objectives

5.1 With regard to the use of data in reserving and capital modelling:
 describe the types of data that are used
 describe the main uses of data
 describe the requirements for a good information system
 outline the possible causes of data errors
 analyse the effects of inadequate data.

The Actuarial Education Company © IFE: 2019 Examinations


Page 2 SP7-12: Data

0 Introduction
The availability of quality data is critical to the operation of an insurance company. Many of the
functions of an insurance company have a clear and direct need for data.

Actuaries need data for two main purposes: premium rating and reserving. Reserving is covered
in this chapter. Premium rating is covered in Subject SP8.

It is important to have data of sufficient quality and detail for the purposes of reserving and
capital modelling. Quality here refers to the extent of any errors or other problems arising
from the data collection, grouping or final dataset used for the assessment of risks.

In both pricing and reserving, policy and claim details covering many years will be needed. In
trying to project future events, an actuary will need previous years’ data in order to establish
enough data for credibility and to identify trends and patterns. So the quality and quantity of
reliable data is very important for the general insurance actuary. The analysis is pointless if the
data is not reliable and relevant.

Whilst this is true for all actuarial investigations, the lack of adequate data seems to be more of a
problem in general insurance than it is in other areas. There are probably two reasons for this.
Firstly, actuaries are relative newcomers to general insurance, so there have been fewer years to
establish appropriate data collection for actuarial applications. Secondly, the range and scope of
the data needed is greater, in particular given the rapidly changing and competitive nature of
general insurance.

Data of sufficient quality is also necessary for the uses described in Section 2.

The structure of this chapter is as follows:


 Section 1 describes the main sources of data. An insurer may have its own internal data.
However it might also make use of external data, in particular data from industry-wide
data collection schemes.
 Section 2 identifies the main uses and users of data. The exact use to which the data will
be put is a key determinant of the level of data required. There may also be restrictions
on the use of data. There are a number of users of data. Our discussion focuses on the
actuary.
 Section 3 considers the data available from an existing data system. Quality and quantity
of data will be influenced by a number of factors, many of which may be outside the
control of the insurer.
 Section 4 considers how to set up a data collection system from scratch. Since the data
acquired under such a system may be used for many years to come, it is important that all
relevant data is captured (ie relating to both premiums and claims) at an appropriate level
of detail.
 Section 5 discusses errors, omissions and distortions that may arise in the claims data.
Consideration is also given to ways of preventing such errors occurring.

© IFE: 2019 Examinations The Actuarial Education Company


SP7-12: Data Page 3

 Section 6 identifies and discusses the specific data requirements for reserving. Note that
while similar data will be needed for both reserving and pricing, each has its own specific
requirements.
 Section 7 goes on to consider even more specific requirements for particular losses.
 Section 8 discusses why and how risks may be classified.
 Section 9 concludes by describing the effect of using inadequate data for the purpose of
reserving.

You should be aware that, although parts of this chapter are similar to the data chapter in
Subject SP8, there are significant differences. The main differences are from Section 6 onwards.
There is also more material on data protection in Section 2 of this chapter than in Section 2 of the
data chapter in SP8.

The Actuarial Education Company © IFE: 2019 Examinations


Page 4 SP7-12: Data

1 Sources of data
Companies may analyse either their own internal data and/or external data from industry
sources.

Internal data is primarily the insurer’s past experience of business sold.

Industry-wide (or industry-source) data is collected and compiled by member offices of


particular organisations, for example, the ABI (Association of British Insurers) and Lloyd’s of
London in the UK. The use of such data will increase the quantity, but not necessarily the
quality, of the data available for analysis. Industry-wide data is discussed in further detail
below.

Other sources of external data exist, eg reinsurer’s data or published accounts.

1.1 Internal data

Question

List briefly what a company’s internal data will include.

Solution

Internal data will include:


 policy information collected from the proposal form
 premium information
 claim information.

The majority of this chapter focuses on the quality and quantity of internal data. Industry data is
discussed below.

1.2 External data from industry sources

What are industry-wide data collection schemes?


In some countries, there are organisations that collect data from their member offices and
then make available summaries of all the data to their members.

For example, in the UK, the ABI collects and collates a wide variety of insurance data.

© IFE: 2019 Examinations The Actuarial Education Company


SP7-12: Data Page 5

Question

Explain why such schemes exist.

Solution

Managers of insurance companies use the data to confirm or refute suspicions from their own
data. Also, anybody managing any business should be aware of what is going on in the market
place.

Companies may also be required by regulators to publish data in a standard form. These
can be collected to form an industry-wide database.

Examples of industry-wide data include catastrophe model data sets, UK flood maps,
CRESTA zones, credit ratings and premium rates.

CRESTA stands for Catastrophe Risk Evaluating and Standardising Target Accumulations. The
global CRESTA zone data is used to help assess risks relating to natural hazards, particularly
earthquakes, storms and floods.

Potential benefits
An insurer participating in an industry-wide scheme can compare its own experience with
that of the industry as a whole (or that part of it represented by the participating insurers)
both at the overall level and at the level of the categories into which the data is classified.
Any significant differences should be explained. Since an insurer usually seeks to expand
by attracting business from its competitors, it may be important to understand the ways in
which the characteristics of the business it is seeking may differ from those of the business
it already has.

Industry-wide data provide a benchmark for insurers to assess their position compared to
their competitors. Industry-based development factors may be valuable for reserving,
especially for small insurers and insurers that have been established for only a short time.

Potential problems
There is potential for distortions within industry-wide data, particularly owing to
heterogeneity.

The Actuarial Education Company © IFE: 2019 Examinations


Page 6 SP7-12: Data

Question

Before reading on, list as many reasons as you can why data might differ between different
organisations.

Solution

 different geographical or socio-economic sectors


 different target markets or sales methods
 different cover, exclusions, sizes of excess, policy conditions
 different underwriting and claim settlement practices, reserving methods
 differences in the nature and quality of data stored by different companies

Possible reasons for heterogeneity


The data supplied by different companies may not be precisely comparable because:

 companies operate in different geographical or socio-economic sections of the


market

 the policies sold by different companies are not identical

 the companies will have different practices, for example, underwriting, claim
settlement and outstanding claim reserving policies

 the nature of the data stored by different companies will not always be the same

 the coding used for the risk factors may vary from company to company.

This means that you must be very careful when interpreting industry-wide data. It may well not
be relevant for your company.

Further problems
Other problems with using industry-wide data may be:

 The data will be much less detailed and less flexible than those available internally,
hence it will be more difficult to manipulate.
 External data is often much more out of date than internal data. The data takes quite
a while to collect, collate and then distribute to the insurers.
 The data quality will depend on the quality of the data systems of all its contributors.
If one company makes a mistake (eg entering a figure in millions instead of thousands)
then this invalidates the whole set of data. The more companies that contribute, the
more likely that one will make a mistake.
 Not all companies contribute.

© IFE: 2019 Examinations The Actuarial Education Company


SP7-12: Data Page 7

2 Uses and users of data

2.1 Uses of data


The main uses of policy and claims data by a general insurer are:

 administration

 accounting

 statutory returns

 investment strategy and performance analysis

 financial control and management information

 risk management

 reserving (including unexpired risk assessment)

 experience statistics

 premium rating and product costing

 marketing

 capital modelling

 catastrophe modelling.

Interactions and conflict between different uses


The interaction between these functions can be very complex and will vary from insurer to
insurer. However, the underlying data requirements will normally be similar in each case.
Ideally all these functions would be controlled through one single integrated data system.

This is not often achieved in practice, especially where insurers have developed through
mergers of companies whose systems were not compatible with each other.

The Actuarial Education Company © IFE: 2019 Examinations


Page 8 SP7-12: Data

Question

Give reasons why, ideally, all functions should be controlled by ‘one single integrated data
system’.

Solution

If data is controlled by one single, integrated system then:


 there is reduced chance of existing data being corrupted
 there is reduced chance of inconsistent treatment of information, between products or
over time
 there is likely to be a better level of control over those who may enter information or
amend information
 information will be easier to access, eg it will not involve collating information from
several systems
 time will not need to be spent reconciling data from different systems.

High level data

For some purposes, data may only be required on a ‘big picture’ basis. Here, data will be
publicly available in published accounts and statutory returns. In some instances – for
example, merger and acquisition work undertaken by a consulting firm – only public data
may be available. Such data would be used for reserving purposes.

However, for the purpose of management information, the data will be needed in more detail so
that strategic decisions can be made, such as deciding which classes and territories to expand.

Data on individual risks

Insurance organisations also need data relating to the individual risks for which they
provide cover. The quantity and quality of this data is closely related. Without sufficient
quantity, data groupings will either be non-homogeneous or lack credibility. However, even
where there is plenty of data available, poor quality data will not produce results that are
reliable.

Data groupings are discussed further in Section 8.

2.2 Restrictions on the use of data

Customer information
Sufficient information will be needed to identify the policyholder of each policy and to
provide basic details. However, customer information can be used for cross-selling,
customer relationship management and the estimation of customer lifetime value, among
other things.

© IFE: 2019 Examinations The Actuarial Education Company


SP7-12: Data Page 9

To do these things it may be necessary to cross-reference different customer databases, for


example, to determine which motor policyholders also have household policies. This may
not always be straightforward, and if details are not recorded in exactly the same way, it
may not be possible to establish these connections.

To do this, additional data may be needed that are not required for administration purposes.

Obtaining information on policyholders with different products should be made easier if all lines
of business are administered on the same integrated data system.

Data should be used carefully in these ways. Data protection laws may limit the use of data.
Contravening these laws can lead to criminal prosecution and extremely unfavourable
publicity. Permission may need to be obtained for use of personal data when a policy is
issued.

Data protection
The security of data is of paramount importance.

It may be important to both personal and commercial customers that data is protected.

It is subject to explicit law in many countries, usually to protect people’s personal


information. However, many commercial insureds provide data to insurance companies
that is commercially sensitive, so any concern that an insurer’s systems are not secure can
be highly damaging to the insurer.

Data protection laws cover what data a company may hold and for what it may be used. The
laws often allow access to personal data by the subject, who may also have a legal right to
amend incorrect data. This will normally be to the benefit of the insurer, which will not want
incorrect data on its system in any case.

Laws may also require that specified people are appointed to be responsible for certain
aspects of data gathering, processing or use, or for the correctness of data held. Third
parties, such as consultancies, should adhere to certain guidelines when using personal
data provided by an insurer. Such guidelines include providing details of how the personal
data will be used and destroying all personal data after use. One example of a data
protection law, the EU General Data Protection Regulation (GDPR), is discussed in the
following paragraphs.

Insurers should have procedures in place to ensure that data is secure and is used only for
appropriate purposes. These procedures will include limiting the use of systems to
specified people, who need to identify themselves to the computer using passwords, and
rules for the secure storage and transmission of data.

The EU General Data Protection Regulation (GDPR) replaces the Data Protection Directive
95/46/EC and was designed to harmonise data privacy laws across Europe, to protect and
empower all EU citizens’ data privacy and to reshape the way all organisations (not just
insurance companies) across the region approach data privacy.

The GDPR was approved by the EU Parliament on 14 April 2016. It will come into force on
25 May 2018 at which time those organisations in non-compliance will face heavy fines.

The Actuarial Education Company © IFE: 2019 Examinations


Page 10 SP7-12: Data

Although the key principles of data privacy still hold true to the previous directive, many
changes have been proposed to the regulatory policies and details of the key changes can
be found on https://www.eugdpr.org/. These key changes are summarised as follows:

 Increased territorial scope: extended jurisdiction as it applies to all companies


processing the personal data of data subjects residing in the Union, regardless of
the company’s location.

 Penalties: Under the GDPR, organisations in breach of GDPR can be fined up to 4%


of annual global turnover or €20m (whichever is greater). This is the maximum fine
that can be imposed for the most serious infringements but there is a tiered
approach to fines.

 Consent: The conditions for consent have been strengthened as the request for
consent must be given in an intelligible and easily accessible form, with the purpose
for data processing attached to that consent. It must be as easy to withdraw
consent as it is to give it.

 Breach notification: Under the GDPR, breach notification will become mandatory in
all member states where a data breach is likely to ‘result in a risk for the rights and
freedoms of individuals’ within 72 hours of first having become aware of the breach.

 Right to access: Data subjects can obtain from the data controller confirmation as to
whether personal data concerning them is being processed, where and for what
purpose. Further, the controller shall provide a copy of the personal data, free of
charge, in an electronic format.

 Right to be forgotten: Entitles the data subject to have the data controller erase
his / her personal data, cease further dissemination of the data, and potentially have
third parties halt processing of the data.

 Data portability: The right for a data subject to receive the personal data concerning
them, which they have previously provided in a 'commonly used and machine
readable format' and have the right to transmit that data to another controller.

 Privacy by design: Inclusion of data protection from the onset of the designing of
systems, rather than an addition, and for controllers to hold and process only the
data absolutely necessary for the completion of its duties (data minimisation), as
well as limiting the access to personal data to those needing to act out the
processing.

 Data protection officers: Rather than the current notification of data processing
activities with local agencies there will be internal record keeping requirements and
appointment of Data Protection Officers will be mandatory for certain higher risk
controllers.

The increased public and regulatory scrutiny over cyber-attacks and the insurance
coverage protecting organisations against such attacks is continuing to evolve. This makes
data protection an important topic in the context of insurance undertakings both from the
perspective of protecting their own customer information as well as considering the risk of
insurance losses as a result of potential cyber-attacks on their customers where coverage
is provided.

© IFE: 2019 Examinations The Actuarial Education Company


SP7-12: Data Page 11

2.3 Users of data

Full development team


The full development team for a computer system should include representatives from the
following departments:

 senior management

 accounting

 underwriting

 claims

 marketing

 investment

 actuarial / statistical

 computing

 reinsurance

 risk management

 catastrophe modelling.

Question

Suggest the main uses of data for each of these potential users.

Solution

Senior management: making business decisions


Accounting: collecting premiums; paying intermediaries, claimants etc,
preparing summaries
Underwriting: premium rating, identifying improvements, evidence of selection,
portfolio monitoring
Claims: processing and settling claims
Marketing: assessing marketing performance and identifying opportunities
Investment: monitoring investment performance and opportunities
Actuarial: premium rating; reserving; assessing solvency and capital
requirements and investment and reinsurance strategies;
management information
Computing: writing and implementing the IT system
Reinsurance: monitoring reinsurance performance and adequacy
Risk management: monitoring the size and nature of risks written, identifying
aggregations of risk, implementing risk controls
Catastrophe modelling: assessing and quantifying catastrophe risks

The Actuarial Education Company © IFE: 2019 Examinations


Page 12 SP7-12: Data

The different users of data within a general insurer will usually have different needs in terms
of the quantity and quality of the data.

In developing a system, it should be noted that different departments will have different
requirements.

For example, the marketing department may want data only at a very high level for their
promotional material. The actuarial department, on the other hand, will want very detailed
information when they undertake a rating review.

This may lead to conflict as some departments may be less attentive in supplying data
which is not directly for their own purposes. Such conflicts could result in a system that
does not contain data at the desired level of detail for actuarial analyses.

The role of the actuary


Actuaries are likely to be involved in the technical aspects of all the functions listed above
(in Section 2.1). Ideally, they should be involved in developing the information systems that
support these areas.

The extent of their involvement in developing such systems will depend on the extent to
which they are actively involved in each area.

For example, an actuary may be heavily involved in building and designing a database for
storing a history of policy and claims information for the assessment of risks. On the other
hand, actuaries may only be asked to provide verbal advice on developing a new
information system for marketing purposes.

In general, the actuary must work with whatever data is available. A new insurer will not
have its own historical data.

The appropriate response to data being of poor quality will depend on the nature of the task
in hand, and is discussed elsewhere in this chapter.

For example, a UK insurer should have kept at least enough data to be able to compile the
statutory returns to the Prudential Regulation Authority (PRA) and its predecessors. There
are similar requirements in some other countries.

The PRA is responsible, amongst other things, for the supervision and regulation of insurance
companies in the UK.

© IFE: 2019 Examinations The Actuarial Education Company


SP7-12: Data Page 13

3 Quality and quantity of data in existing data systems

3.1 Factors that influence the quality and quantity of data


The availability of data of good quality and quantity will vary greatly:

 between organisations, which will depend on:


– the size and age of the company
– the current data system in use, including the use of legacy systems, the integrity
of the data system(s)
– the management and staff responsible for collecting and maintaining data
– the nature of the organisation, eg direct insurer vs reinsurer
 within organisations:
– depending on the distribution method of the business.
 between the different classes of business.
These influences are discussed in more detail below.

3.2 Reasons for variation by organisation

Impact of size and age of company on data quantity


Large companies will have much more data available than smaller ones. They are likely to
make more use of their own data, rather than rely largely on industry-wide data.

A newly established company may find that it has insufficient historical data for pricing,
planning and reserving purposes. It may need to supplement its historical data with data
from industry sources.

Impact of size and age of company on data quality


The size of an organisation can also have a more direct influence on data quality, as a large
company may have better data systems in place than a small one. However, a large
well-established company may have an outdated computer system that is difficult to amend.
A new small company may have a modern system that can be readily adapted to change.

Building a new data system is a major project. It is expensive and uses up considerable
management time. The old system also has to be operated and maintained until the new
one is ready, and it may be difficult to transfer historical data from the old system to the
new.

Sections 4, 6 and 7 below describe the information that would be kept if a new system were
to be set up from scratch. In practice, the only insurers that can achieve this level of data
system are new companies and those that have been set up in the last few years. In
general, the older the insurer is, the more its data system is likely to differ from what is
ideal.

Larger companies, resulting from the merger or acquisition of other companies, can also
experience legacy system difficulties. These arise from the integration of two or more data
systems with different data items, and the data captured being structured in different ways.

The Actuarial Education Company © IFE: 2019 Examinations


Page 14 SP7-12: Data

Legacy systems
The ability to input, manage, extract and analyse data depends fundamentally on the quality
of the systems. Many data analysts will be hampered in their research by legacy systems.
These are systems which, for a variety of reasons, have not been updated or upgraded.
Legacy systems may affect the whole company or may be limited to particular product lines
(or even generations of a particular class). The problem is often at its most difficult in areas
relating to portfolio transfer or company acquisition. Quite often there is an additional
difficulty with systems incompatibility and communication.

As mentioned above, a common problem is that an insurer is formed from a merger of two
or more insurers with their own systems. It is rare in such circumstances for it to be
possible to transfer all historical data from one system onto the other. In general, we would
prefer to use only the better system after the merger, but the other system is likely to lack
some of the data items that the better system recorded. This is one of the things that is
likely to have made it better. Usually a merged insurer will move to one of its legacy
systems, but it is likely to retain the old business on the other system and may keep
renewals of the business on the other system, at least for a time.

In an insurance company an actuary is likely to be able to influence the gathering of data


and design of systems. This is especially true in a new company.

Considerations when designing systems are covered in Section 4.

However, while an actuary may be able to influence systems so they are more useful for
actuarial work, it is usually not possible to enter historical data onto a new system if they
were not previously collected and processed automatically. Therefore, it may be some time
before a new system becomes fully useful to the actuary. In the meantime, using two
systems can be problematic.

We should allow for any approximations made as a result and may need to allocate more
time to particular pieces of research involving data stored on legacy systems.

Integrity of systems
The quality of data depends on the integrity of the systems. In other words, checks are
needed to ensure that all data is entered onto the system, entered once only, and entered
correctly. Data should also be backed up regularly and securely, and have safeguards
against accidental corruption. Some of these safeguards will be built into the system itself
while others will be part of administrative procedures.

Checks that may be performed are discussed in Section 5.5.

Management and staff


Problems of data quality and quantity can result from poor management control of data
recording or its verification processes, or from poor design of the data systems.

Management should make staff aware of the importance of maintaining good quality data
records. If the administrators are not aware of this, the data input may be of a lower quality.

Management will have control over cost as well as design. Poor quality data systems may be due
to cutting corners with the budget as well as bad design. The extent to which actuaries have been
involved in the design of the system will also influence the quality of the data.

© IFE: 2019 Examinations The Actuarial Education Company


SP7-12: Data Page 15

This may not necessarily be a reflection on the current management, as good-quality data
cannot necessarily be obtained quickly. After implementing a process for maintaining
extensive records, it may take many years to collect enough data for analysis purposes.
This is especially true for long-tailed classes.

Reinsurer vs direct insurer


A reinsurer (especially one writing non-proportional reinsurance) will often receive
aggregate (or bulk) data relating to all its risks coming from a particular cedant. It is
virtually impossible to check the accuracy to any great extent, without going through the
expensive process of accessing the original records of the cedant.

For treaty reinsurance business, sometimes only grouped bordereaux data may be
available. Third parties, such as binding authorities or cedants may provide this
information.

The Glossary defines the term bordereau as:

A detailed list of premiums, claims and other important statistics provided by ceding
insurers to reinsurers, so that payments due under a reinsurance treaty can be calculated.

There are two distinct problems for the reinsurer here:


 If grouped bordereau data is all that is available then the reinsurer may not be able to
check and/or analyse its experience data to as high a level as it would like as there may be
insufficient detail provided.
 The data may be out of date by the time the reinsurer obtains it.

A further problem that reinsurers have regarding data is that the bulk data is usually
provided by the cedant long after the claim occurred. Hence the quality of data held at any
one time is poor when they try to assess future income and outgo.

In the case of excess of loss reinsurance, a cedant may fail to recognise that a claim may
exceed the retention and may therefore fail to notify the reinsurer. Thus, claims that have
been reported to the cedant may remain IBNR for some time from the point of view of the
reinsurer. In an attempt to reduce this problem, it is customary to ask cedants to notify the
reinsurer of all claims that they think may exceed, say, half of the retention.

3.3 Reasons for variation by different distribution methods


Insurers write their business:

 through brokers (intermediaries)

 through agents (eg banks or building societies that sell a certain insurer’s buildings and
contents insurance)
 directly with customers.

The data needs will vary between the various distribution channels, which will not in any
case be homogeneous.

The Actuarial Education Company © IFE: 2019 Examinations


Page 16 SP7-12: Data

Brokers and agents


Different brokers and agents will differ in:
 the role they play in the sales, administration and claims processes
 their level and form of remuneration
 the manner and speed with which they process policies / claims.

Role played in sales, administration and claims processes

Some brokers will retain funds on behalf of insurers in order to pay straightforward claims,
while others will not be involved in claims administration. The former will provide claims
details to be entered onto the insurer’s systems or onto bordereaux, while claims from the
latter will be entered by the insurer’s own staff.

Organisations that obtain large parts of their business through intermediaries, or those that
delegate underwriting and/or claims handling authority, may have similar problems to those
described in the section above on reinsurers.

Even among brokers who have authority to administer claims, the levels of authority will
vary.

Question

Give an example of this sort of arrangement (with the intermediary or agent taking responsibility
for most of the administration).

Solution

A large block of policies, such as mortgage indemnity business, sold through a building society, or
travel policies sold through a travel agent. (In each case the insurer may only receive summarised
data.)

Note that in this case the building society is only likely to do the policy administration. Since the
building society is the actual recipient of claim payments, the insurer would do the claims
administration.

Remuneration

Different brokers will have different remuneration conditions. The system needs to identify
them and calculate payments appropriately.

The remuneration conditions may also affect the willingness of the broker to provide good quality
data to the insurer in a timely fashion. This may also depend on the state of the market.

For example, it may be more difficult to obtain data from brokers when the market is soft
since brokers may be less incentivised. This is because they may receive lower
commissions in comparison to when the market is hard.

The insurer may receive bordereaux data in different formats and at varying levels of
quality. The level of detail may also differ.

© IFE: 2019 Examinations The Actuarial Education Company


SP7-12: Data Page 17

Both premiums and claims information may be bulk figures, and thus underlying policy and
claims details are hard to access.

The brokers may also have requirements (eg on the level and form of data collected) that may
impact the insurer’s own data systems and processes.

Manner / speed of processing information

Brokers and agents may provide information on paper. The insurer’s staff will need to enter
this onto the systems, or in a medium ready for computer entry, or through the internet.

It may be time consuming for staff to enter such data into a computer system and so only
major losses may be broken out from claims bordereaux, with the residual being entered as
a bulk item. It is quicker to integrate data received electronically into the system.

The system should provide appropriate management information to brokers to enable them
to compile their own management information and the information that they need to support
their own clients.

Direct sales
For business sold direct to the policyholder, the insurer needs to enter all the data. This will
be done by staff from telephone and in-branch sales, but internet sales may be processed
directly into the system without human intervention.

3.4 Reasons for variation by different classes


Data can vary between classes for a number of different reasons, principally due to the different
nature of risk, which leads to the following:
 big variations in claim frequency between classes
 the length of the tail of some classes means that it takes considerable time to collect the
necessary claims data
 for some classes the underwriting is subjective, so it is difficult to store the details of the
risk.

Claim frequencies
The more frequent the claims, the larger the available quantity of claims data.

Question

Give an example of a class of business where claim frequency tends to be very high.

Solution

Motor insurance claims tend to be fairly frequent. 25% on private fully comprehensive policies
would not be unusual, and up to 40% for motor fleet.

The Actuarial Education Company © IFE: 2019 Examinations


Page 18 SP7-12: Data

Length of tail
There is a difference between the number of years of past data required for long- and short-
tailed business. For longer-tailed classes of business, there may be significant delays until
there is sufficient data available to support an analysis of the business. This is particularly
true of classes that are subject to significant delays in claim notification or slow loss
development.

Subjectivity of underwriting
There is a difference in data quantity and quality for classes that use different forms of
underwriting. Underwriting may be based on statistics or on judgement. The range and
volume of data held will be greater for a class where underwriting has been largely based on
statistics than for a class where the underwriting has been largely based on subjective
judgement.

This does not mean that the underwriting process is necessarily haphazard. The
underwriting of individual deals may be supported by excellent information. But the fact
that the information will vary from risk to risk does not lend itself to systematic data
capture; often this is the case with London Market data.

One example is private motor business. This is generally rated by reference to a large
number of underwriting variables, which must be held in the policy database.

Another example is M&A insurance. This may be underwritten based on judgement on a


policy-by-policy basis, so that large amounts of systematic underwriting information are not
stored.

Therefore, there will be more data available on private motor insurance business than for, say,
M&A insurance, which is more subjective. Also, for some classes (eg M&A insurance), the rating
factors are qualitative, and therefore are more difficult to store on a computer. Industrial
property is another class of business for which this may be true.

Grouping of risks
Problems can also arise if an insurer reorganises its class structure, and it cannot assemble
historical data in the new classes. One consequence of a reorganisation of this type is that
claim development triangles may comprise only the most recent diagonals and the interiors
of the triangles are missing or available only in a different class structure.

© IFE: 2019 Examinations The Actuarial Education Company


SP7-12: Data Page 19

4 Establishing a good information system

4.1 Introduction
The primary objective of the wider information systems is usually that the computer systems can
be used effectively. For example:
 Data capture forms (eg proposal and claim forms) should be designed to provide precisely
the information required for the computer, in the right order and with as little ambiguity
or subjectivity as possible.
 Staff should be trained so that they know precisely how to use the computer and
understand the importance of the data they enter onto it.
 Any new computer system should be run in parallel with existing systems for a trial
period, until its reliability has been checked and optimised.
 Procedures should be set in place to monitor the performance of the systems regularly,
and to improve them where necessary.

The following stages are required in the establishment of a good information system to ensure
that good quality data is captured and stored:
 consideration of the users’ requirements
 careful design of appropriate proposal and claim forms
 ensuring that features of premiums and claims can be recorded
 consideration of policy and claim information to be collected
 adequate training of staff.

4.2 Users’ requirements


The first stage in establishing a good information system is to determine the priorities and
requirements of the system for all potential users. Many of these will overlap because many
of the items of data will be needed by various departments, but some may conflict, and the
resultant system may have to be a compromise to some extent. The important points,
however, are that the system should achieve the essential requirements, be compatible
throughout the organisation and integrate the different functions within the organisation.

4.3 Policy and claim information


Information on individual risks is obtained primarily from the proposal form and the claim form.

Proposal form
The prime information source will be the details given on the proposal form. It is therefore
important that it produces relevant and reliable information for the system. Questions
should be well designed and unambiguous, so that the proposer will give the full correct
information and the underwriting department can process the application readily, adding
any coding that is necessary.

The Actuarial Education Company © IFE: 2019 Examinations


Page 20 SP7-12: Data

For example, storing date of birth is more useful than storing age.

However, an excessive number of questions on either the proposal form or the system may
lead to poor quality data if the individuals answering the questions provide inaccurate
responses in order to complete the form quickly.

This information should be held (together with any subsequent changes) so that it can be
cross-checked against the claims information at the time of any claim, or for rating or
accounting purposes. This should enable the automatic checking of the validity of the
claim and the updating of the basic policy information (for example, the termination of cover
in the event of a total loss). The cross-checking is likely to be achieved by the storage of
claim reference numbers with the policy information.

When information is changed then a history of the previous information should be retained.

Question

Explain why a history of previous information should be retained.

Solution

It is necessary to keep a history of policy and claim records so that accurate experience analyses
can be carried out. In particular, it is necessary to ensure that claims and exposure data
correspond.

Clear links are needed between underwriting and claims databases, eg via policy reference
numbers.

In many policies, information is taken over the phone or through the internet. However, in
these cases it is normal for the insurer to set the information out in a completed proposal
form for the insured to sign as a legal contract.

The precise information will vary from class to class and from contract to contract, since
almost all policies have some optional elements. If the options are taken, details of the
cover provided will be needed, such as a list of valuable items on a household contents
policy. However, all proposals will need the name and address of the proposer. Other
items will vary, such as the sum insured, which may not be needed in some cases.

The sum insured may not be needed where the amount of cover that must be provided is dictated
by legislation.

For example, UK legislation requires unlimited cover for third party motor bodily injury
cover.

Claim form
The main information source will be the details given on the claim form. Like the proposal
form, it is important that this is designed with the aim of producing information that can be
transferred easily to the computer system. The system should refer to the corresponding
policy record and verify the existence of cover.

Again, the questions should be clear, unambiguous and objective.

© IFE: 2019 Examinations The Actuarial Education Company


SP7-12: Data Page 21

4.4 Features of premium information


Premium information is necessary for reserving, pricing and monitoring. Hence it is
important that the premiums received (and paid) are recorded accurately.

The features of premiums that should be recorded are:


 amounts
 timings
 adjustments to premiums, such as premium discounts and commission paid
 cross-selling information.

Written and signed amounts


Written premiums are the premiums an insurer expects to receive over the duration of the
policy. Written premiums may be before or after commission. The insurer should calculate
the net written premiums by deducting any reinsurance premiums from the gross
premiums.

Within the context of the London Market, signed premiums are the written premiums at the
signed share of a risk.

In practice, the terms ‘written premium’ and ‘signed premium’ are used interchangeably and in
fact the two figures are often the same. The difference comes when a risk is more than 100%
placed. For example, let’s say that a risk is 105% placed, then the written premium is the total of
all the insurers’ combined premium, before the risk is signed down. After the risk is signed down,
then an insurer’s share of the signed down amount is called the signed premium.

Payment times
Premiums may be paid annually, monthly or at another frequency. The data system should
record the date premium payments are due, the premium amount due and the date
premiums are actually received. The system should be designed so that it is easy to check
for any overdue premiums.

Premium adjustments
The data system should be flexible enough to allow for premium adjustments to be made.

For example, the system may have to record an adjustment to premiums because of
endorsements, such as changes in policy limits or exclusions.

Policy limits may need to be changed on a contents insurance policy if the policyholder buys a
new item of high value contents, eg a new television. Exclusions may need to be removed, for
example if a household insurance policy previously did not cover the contents of outbuildings but
the policyholder wanted to remove this exclusion.

Some endorsements may leave the premium unchanged. Others may result in an increase
or decrease in the premium. The data system should be able to record the premium
adjustment (and date).

The Actuarial Education Company © IFE: 2019 Examinations


Page 22 SP7-12: Data

If the policy is written under an NCD system, a discount may be applied to the renewal
premium if the policyholder made zero claims in the previous year. The system should
store both the original and discounted premiums for future reference.

The premium rating part of the system should also know (via a link) how many claims have been
made in the previous year so that it can accurately calculate the renewal premium.

Another example is that the premium could be paid on a ‘minimum and deposit’ basis. This
approach can be used in cases where the premium will depend on the level of exposure but
the final level will not be known until the end of the policy period. An example might be a
motor fleet policy where the total number of vehicles in use will not be known in advance.
In such a case, an initial premium is paid at the start of the policy period. Then, an
additional premium is paid at the end of the period depending upon the actual level of
exposure.

For treaty reinsurance business, the system should also record the dates and amounts of
any reinstatement premiums (either due to be paid or already paid) for the treaty.

The system should similarly record any facultative insurance premiums with respect to
individual policies.

Commissions
The system should record commissions paid to brokers and other intermediaries. The
commission may be a fixed percentage of premiums. In this case, the system should record
the correct percentage and update it when necessary for future reference. It should record
details of the relevant broker or intermediary.

It should record ceding commissions and profit commissions payable by reinsurers to the
ceding company.

Other deductions
It should record any other increases or reductions to premiums.

For example, insurers may award premium discounts to policyholders who pay their
premiums in a timely manner.

Cross-selling
Collecting information on cross-selling may help in seeing if a particular loss-leader
strategy is working.

A loss leader is an insurance product sold at a low price (usually below the technical price
so that it is sold at a loss) to:

 generate profitable sales for another insurance product

 help establish the insurer in a new territory or new product class.

Cross-selling is the selling of similar insurance products or the selling of products that can
be incorporated with the actual insurance cover being sold.

For example, selling home contents insurance, as a loss leader, to existing buildings
insurance customers (or to new customers applying for buildings insurance) may help
increase the number of home contents insurance policies sold to new policyholders.

© IFE: 2019 Examinations The Actuarial Education Company


SP7-12: Data Page 23

The system should record cross-selling information for the loss-leader product, such as
type of product, actual premium charged, technical price and number of policies sold, to
assess the effectiveness of the sales strategy.

4.5 Features of claim information


General insurance claims may have a number of features that must be captured by a data system
and the data collection process. These include:
 the definition of a claim
 the estimated outstanding claim amount, claims paid to date and type / cause of claim
 multiple claim payments
 reopened claims
 claims handling expenses
 reinsurance recoveries
 class-level adjustments.

Definition of a claim
Setting up a claim record

Insurers will have certain rules governing when a claim record should be set up, following
an alleged loss by a policyholder.

For example, a claim record could be set up on the system when:

 a request for a claim form is received

 the claim form is received back from a policyholder

 the claim form has been assessed by a member of the claims department staff who
decides that it appears likely to be valid

 the claims manager concludes that there is sufficient information to set a reserve.

Changes to this rule and/or office procedures could affect the:

 numbers of claims recorded

 numbers of nil claims (claims settled at no cost to the insurer)

 speed of notification.

Any such variations could affect subsequent analyses of claims experience for the classes
concerned. They will certainly affect the development of numbers of claims reported and
closed, and may affect the development of incurred claims.

For example, if the insurer changed the definition of a nil claim mid-way through the year and we
were analysing average claim amount and average frequency, the analysis would be distorted by
this change in definition (assuming nil claims are included in the number of claims for frequency
and amount calculations).

The Actuarial Education Company © IFE: 2019 Examinations


Page 24 SP7-12: Data

Closing a claim

Comparable variations can exist for the definition of the settlement of a claim. Some
insurers go through their claims files periodically (for example, at each year end) and only
then declare claims as settled. Others close the claim record as soon as it appears that the
last payment has been made. Some insurers may even close a claim file when a payment
will or may be paid at some relatively distant future date (for example, where a payment is
due on attainment of majority by the recipient or when a provisional damage award has
been made).

An attainment of majority is where a payment is made once the claimant reaches a certain age
that is pre-specified by the courts.

An insurer may also close claims following a one-off review with respect to a particular
book of business.

Estimated outstanding amounts


Insurers’ attitudes towards recording estimated amounts outstanding on each open claim
vary widely, as do the frequencies with which these are updated.

So there may be differences in:


 when the estimates are first set up
 the method used to determine the amount
 how often these amounts are revised.

We should allow for these facts in designing the information system in order to advise
users what information to input and what the information held means.

Setting the initial estimate

Outstanding amounts may be estimated in different ways:


 At one extreme, an initial estimate will be put on file when the claim is first notified.
This may be updated whenever a payment is made or periodically, whether or not an
intervening payment has been made. The revised estimate may be the earlier
estimate written down by any payments made or a completely revised amount,
based on the latest known facts of the case.

 At the other extreme, the insurer may not attach a case estimate to any one claim
individually, relying instead on a total estimated value for the risk group, based on
statistical methods. However, such insurers are very rare.

 A more common approach is to set a standard reserve for claims that are not large
until they are settled or to set a reserve based on the claim’s characteristics.

© IFE: 2019 Examinations The Actuarial Education Company


SP7-12: Data Page 25

Question

Explain why an insurer bothers with outstanding amounts, since the analysis would be more
accurate if it waits until all claims are settled.

Solution

If it waited until all claims were settled then the data would be out of date, especially for
long-tailed classes.

An insurer whose only reserves are bulk reserves for the risk group will not be able to
record individual outstanding claim estimates. In all other cases, the reserve amount
should be recorded on the system and the date when it was set. To compile
loss-development statistics, they should be retained, even when they are superseded by
revised estimates. It is possible to compile development statistics by retaining only the
latest reserve amount and adding a further diagonal to old triangles. In the past this was
sometimes done partly because of the cost of data storage. For the same reason, only an
inception-to-date-paid amount might be kept. However, this precludes new investigations
into historical data and is much less useful to the actuary. Because of low modern
data-storage costs and increased capacity, this is usually no longer an issue.

Triangulation methods are discussed later in the course.

If the reserves are set separately by type of payment (eg indemnity, compensation), the type
should be recorded. The currency should also be recorded.

Determining the amount to hold

Each company will have its own case estimate philosophy and guidelines for the claims
assessor to follow. For example:

 one company may require estimates to be as realistic as possible

 other companies may prefer to have an element of prudence in each estimate to


avoid later deterioration in the overall reserve amount.

In the London Market, policies are often coinsured by a number of different insurers. In
these cases, the lead insurer will normally be responsible for handling the claim (although
the second on the slip may also be involved) and will advise reserves to the following
insurers. It is common for following insurers to use the reserve advised by the leader,
although some insurers do alter the reserve for contentious claims where there are issues
such as policy coverage.

Large claims are often managed by loss adjusters: specialist companies that manage
claims on behalf of insurers. They will usually advise the insurer on the expected ultimate
amount of the claim and most insurers will adopt the adjuster’s advised reserve as their
reserve for the claim.

Some insurers record expected amounts to be recovered from subrogation or recovery as


negative reserves attached to individual claims.

The Actuarial Education Company © IFE: 2019 Examinations


Page 26 SP7-12: Data

Revising amounts

Reserves should be reviewed from time to time to ensure that they remain valid. A number
of claims are advised but not pursued (by the claimant), and the insurer may not hear further
from the claimant. In such instances it may be necessary to contact the claimant to assess
the current status of the claim; otherwise these reserves may remain on the books
indefinitely. However, there is often a reluctance to contact the claimant, as the act of
reminding him or her of the claim can be costly to the insurer.

Multiple claim payments


Some claims will involve two or more payments, made at different times. Further, the
insurer may be able to recover part of the gross cost of the claims through salvage,
subrogation or reinsurance. This is likely to occur some time after the original claim
payments are made and the amounts are normally recorded as negative claim payments
with a code to identify the type of receipt so that claim severities can be better assessed.

Separate details should be kept for each payment made or received under a particular claim,
including details of the dates, amounts, currency and type of payment involved.

Different types of payments will include:

 indemnity payments made to policyholders

 compensation payments made to third parties

 payments to claimants’ solicitors

 payments to loss adjusters and payments of interest.

This is not an exhaustive list. An equivalent list of amounts recovered from reinsurers will
also be needed, and it may be necessary to distinguish between different types of
reinsurance recovery.

It should be noted that payments on a single claim may be made in different currencies.

For example, a UK insurer might insure a ship for a European client, with the premium being
set in Euros. The indemnity amount might then be payable in the currency of the country
where a claim occurred and the loss-adjusting costs in Sterling if a UK loss adjuster were
engaged.

Even where all a company’s business is done in one country, claims in a foreign currency
are often possible, so it is usually desirable to be able to record currency.

This will subsequently give more scope to analyse claims experience in detail, and to
establish correct run-off patterns of the claim payments. It is important to keep the
approach consistent over time if data is to be useful for actuarial analysis.

This is an important general point. If you hold more detail, you will be able to perform many
different, detailed analyses on the data.

The problems of inadequate and incomplete data for reserving purposes are looked at in more
detail in Section 9.

© IFE: 2019 Examinations The Actuarial Education Company


SP7-12: Data Page 27

Reopened claims
Some claims that the insurer regarded as settled may have to be reopened at a later date for
various reasons:

 it may be purely due to the closure definition used by the insurer

 it may be because a further liability for payment, possibly of costs rather than of
indemnity, has come to light

 the insurer has made some recovery against a third party involved

 an error was made in closing the claim originally

 for a claim made by a minor who can reclaim upon attaining the age of 18.

It is important that the system does not regard these as new claims, as this would cause
errors in recording of claim frequency and errors in allocation of the claim by year of origin.

This is particularly a problem if there is a sudden change. If the method of recording reopened
claims has suddenly changed, the distortion will be more serious than if the allowance for
reopened claims has been the same for many years.

A note should also be kept of the original date of closure.

Question

Explain why the original date of closure should be kept.

Solution

To allow the insurer to analyse the percentage of claims that will be reopened and also to
calculate the average time from closure to reopening.

Claims handling expenses


Some systems will record claim payments and claims handling expenses separately, whereas
others will combine the two items.

Reinsurance recoveries
We should record reinsurance recoveries that have been received from reinsurers. In many
cases, reinsurance recoveries can be linked to particular claims, for example, when the
recoveries arise from proportional reinsurance or single risk XL. Recoveries from
catastrophe XL are not usually in respect of an individual claim. The data system should
still record these recoveries in some way. Inwards claims are likely to have a catastrophe
indicator and recoveries may be proportional.

The Actuarial Education Company © IFE: 2019 Examinations


Page 28 SP7-12: Data

It is normal for insurers to set up a reserve for recoverable reinsurance when a recovery is
expected against a gross claim. However, there is always a delay between the payment of
the claim and the recovery of the reinsured amount. If this can be significant, there may be
a need to distinguish between reserves for recoveries on paid claims and reserves for
recovery against gross reserves. When analysing data, it may be important to understand
how these have been treated. The difference may be important for accounting purposes
since recoveries on payments already made will be a debtor and recoveries on reserves will
be an accounting provision.

When risk XL or catastrophe XL cover has been used, it may be useful to record paid or
expected amounts of reinstatement premiums against individual claims.

The amount of the reinstatement premium would therefore be allocated to the claim(s) that had
necessitated the use of the reinstatement cover.

It is unlikely that one will allocate IBNR and paid claims to individual risks except for large
London Market contracts.

Indeed, it is very difficult to allocate IBNR to an individual risk because, by definition, we don’t yet
know about the claim and therefore can’t know which risk to allocate it to.

Class-level adjustments
It may be necessary to make adjustments at a total class level, such as adjustments for
IBNR claims. We should allow for reserves for pure IBNR claims at the total class level.
Similarly, we should adjust for profit commission at the total class level.

We should design the system so that these class-level adjustments can be entered and
updated when required.

4.6 Information to be recorded


The data requirements for each policy or claim record include:

 risk definition and details of cover


The risk definition will include the class and subclass of business, and, together
with the details of cover, will identify the cover granted. This may consist of a
number of items of information.
For example, in a household policy there is usually a range of optional covers, such
as garden equipment, bicycles, accidental damage and specified valuables. There
may be separate sums insured under some or all of these categories and they may
have different excesses. All these should be recorded.
Commercial policies may similarly have a menu of items that may or may not have
been selected.

 details of claim (if it is a claim record)


The details of a claim will normally include a claim cause code.

 status of present record


The status of a record is normally in-force / expired / cancelled for a policy and open
/ closed / reopened for a claim.

© IFE: 2019 Examinations The Actuarial Education Company


SP7-12: Data Page 29

 control dates (start and end dates of each record, dates of claims, and so on)

 relevant amounts and currencies (exposure, sums insured, premiums, claims


payments, and so on)

 administrative details.
Administrative details may be in narrative form for the use of the administrative
departments.

Ideally, the history of policy and claim records should be held indefinitely. However, you
should always be aware of local data protection laws that may place restrictions on what
customer data can be held and for how long.

This, together with the frequency of changes of information, implies holding a vast amount
of data. It may, therefore, be necessary to strike a balance between the:

 capacity of the system

 cost of data storage

 amount of data stored

 level of detail at which they are stored.

The Actuarial Education Company © IFE: 2019 Examinations


Page 30 SP7-12: Data

5 Errors, omissions or distortions in claims data

5.1 Claim estimation methods


Claims take some time to be settled from when they are incurred. Some claims may be
settled very quickly, some may take longer to settle, and others may take longer still.

The following terminology is commonly used:


 We refer to the claims incurred in the same period as a ‘cohort’. For a particular
cohort, in theory, a graph can be plotted of accumulated claims settled (or reported)
against time from the date of occurrence.

 We sometimes refer to the total accumulated claims (when all have been settled) of a
cohort as the ‘ultimate claims’.

 The rate at which accumulated claims are settled for the cohort can be referred to as
the ‘claims development’.

 The graph which illustrates claims development is called the ‘claims development
pattern’. In many of our reserving methods, we rely on the claims development
pattern to be stable for different cohorts.

The errors that we discuss in Section 5.2 can distort the claims development pattern and
therefore the results of the valuation. These points are discussed in more detail in the later
chapters on reserving.

Whatever method is used to estimate ultimate claims, a large degree of detail is required.
The results are, therefore, highly dependent on the quality of the claims data available:

 any significant errors in the data will be carried forward into the projections and may
cause serious distortions in the values calculated

 any errors that are carried into claims data are likely to distort the settlement or
reporting patterns on which reserving or pricing projections rely.

These would lead to an incorrect value being placed upon future liabilities and to incorrect
reserves being held, the consequences of which could be serious, as explained later in this
chapter.

The same is also true of wrongly grouped data or unusual data (such as a £2 million motor
liability claim), which may be correct but would still distort reserve projections, unless taken
into account.

5.2 Sources of data error


There are many potential sources of data error.

Some examples are given below:

Wrong claim number


Details of the claim could be allocated to the wrong record, and hence to the wrong claim
risk group.

© IFE: 2019 Examinations The Actuarial Education Company


SP7-12: Data Page 31

Wrong policy number


If the claim record picks up its risk details from the wrong policy record, these are likely to
be wrong. Again, the claim details may be allocated to the wrong class, year and so on.

Wrong risk details


This could happen if the current in-force details are entered, rather than those as at the date
of claim. For example, a claim might be registered on the file before changes in risk factors
that occurred before the date of claim.

This might happen if the policyholder has changed their car or their address in the meantime. As
stated above, we want to know the details when the claim happened, not now.

An error could also occur if the policy conditions have changed, but the new policy
continues to be stored in the original rating group.

Wrong claim date


This could cause the claim details to be allocated to the wrong year, distorting both the
apparent numbers of claims for those years and their development.

A wrong claim date could also mean that the claim will relate to the wrong risk details (if these
have changed).

A common cause of this is entering the date when the claim was notified, rather than the
original incident date, or vice versa.

Question

Explain why both of these dates would be required.

Solution

To allow for analysis of reporting delays.

For liability claims and some property claims (for example, for subsidence), where a precise
date of event cannot be determined, there need to be very clear rules and procedures for the
allocation of such cases to a claim year.

Wrong payment dates


Some claims are settled by several payments, made on different dates. If these are not each
identified separately, then development patterns may again be distorted.

Claims inflation
Inflation of claim payments may distort the monetary amounts being used in claims data
analysis unless the raw data is adjusted or the estimation method can make a suitable
allowance. If the unadjusted chain ladder method is used, the claims may not have to be
adjusted for inflation, since the unadjusted chain ladder method is based on the assumption
that future inflation will be similar to past inflation.

The Actuarial Education Company © IFE: 2019 Examinations


Page 32 SP7-12: Data

The different chain ladder methods should be familiar to you from your earlier studies, and will be
described in detail later in this course.

Wrong claim type


Often an analysis of the different claim types for a particular class will be required, since
they behave differently (that is, they may have different claims distributions or be settled
over longer or shorter periods). If the claim types are not identified correctly, this distorts
the separate development patterns and average values.

So in an analysis of the motor account, for example, the insurer would separate property damage
and bodily injury claims.

If the different claim types are not separated, a change in the mix of types of claim also
distorts the development patterns and average values.

These problems may extend to cause of claim (for example, fire, theft, explosion and so on),
as claims from different causes may also behave differently.

5.3 Sources of data distortion


There are other factors that, whilst not exactly errors, can still distort an analysis.

Some examples are given below.

Changes in claim handling procedures


Changing practices regarding the point at which a notified loss is formally accepted as a
claim on the company and is marked as such on the claim file, may distort the claim
characteristics for the risk groups affected.

Similarly, the failure to mark claim records as settled on a consistent basis may affect the
apparent development of claim cohorts.

Case estimates
If these are not updated correctly over time, or as payments are made, the values will be
unreliable.

Similarly, the system may fail to keep a historical record of the estimates (at each calendar
year-end or quarter-end, say). This will inhibit the use of this information for statistical
purposes.

A change to the basis for calculating case estimates – for example, a change from prudent
estimation to realistic – may distort run-off patterns (when based on reported claims).

Processing delays
If the rate at which claims are processed alters through backlogs, changes in procedures
and so on, this will distort the claim development patterns and hence the analysis of them.

This leads to particular problems when setting reserves.

© IFE: 2019 Examinations The Actuarial Education Company


SP7-12: Data Page 33

Large claims
The presence, or indeed the absence, of unusually large claims is likely to distort any
analysis unless a suitable adjustment is made. To make adjustments in the analysis, large
claims should be identified separately.

For example, an adjustment might be made which truncates claims above a certain amount and
spreads them across similar policies.

Large claims are often the subject of reinsurance recoveries. If the system does not
recognise these automatically, the insurer may fail to claim recoveries that are due from
reinsurers.

Return premiums
Return premiums can sometimes be recorded as a claim. This may depend on the
accounting principles of the insurer. However, there is a significant danger that such
practices will distort all manner of analyses.

Question

List possible sources of error in a claim analysis.

Solution

 wrong claim number


 wrong policy number
 wrong risk details
 wrong claim date
 wrong payment dates
 claims inflation
 wrong claim type
 changes in claim handling procedures
 case estimates
 processing delays
 large claims
 return premiums

5.4 The consequences of erroneous claims data


The consequences of wrong data being entered include false accounting, inappropriate
reserving, incorrect pricing, failure to make recoveries and general management mistakes.

The effect of inadequate data will be discussed further in Section 9.

The Actuarial Education Company © IFE: 2019 Examinations


Page 34 SP7-12: Data

5.5 Prevention of errors

Integrity of systems
The key here is the avoidance of errors.

Data input should be thoroughly screened and checked. Data management in this regard
will include check digits, data field integrity checks, mandatory fields and error reports. It is
important to maintain consistent practices over time and over different sectors of the
business.

Ways of minimising input errors should be considered. This may include the use of check
digits in file numbers, numeric minimum and maximum values, and so on.

Check digits

Policy numbers are often designed so that the last digit is a check digit: it is defined by a
mathematical formula based on the other digits so that the wrong entering of a policy
number is likely to result in the rejection of the transaction being processed rather than it
being processed to the wrong policy. The check digit might be alphabetic rather than
numeric to reduce to a minimum the probability of a wrong but valid number being entered.

The check digit could work by an algorithm that generates a final letter from the policy number. If
the final letter of the policy number does not tie in with the check digit then the policy number
has been input incorrectly.

Similar checks can be used for other data entered, such as agent numbers and postcodes.

A check could also be made that the postcode agrees with the address.

The policy number is often used as a link between different databases, eg claims system,
policy issuance system, reinsurance system, etc.

Minimum and maximum values

The maximum and minimum values could apply to the premium size, sum insured, policyholder’s
date of birth, street number etc.

These various checks will improve the overall data quality.

It will also be important to ensure that, although more than one department may be
responsible for use of raw data, only one department at a time is allowed access for
updating the information.

Culture and training


Senior management should emphasise to employees the importance of data systems.
Managers should ensure that employees receive adequate training before handling any data
and that further training is undertaken if any changes are made to the systems or
processes. Monitoring of staff and/or random checks may also help to ensure that high
standards are maintained.

© IFE: 2019 Examinations The Actuarial Education Company


SP7-12: Data Page 35

6 Data requirements for reserving

6.1 Statistical methods vs case estimates


When reserves for outstanding claims, IBNR and unexpired risks and claims handling
expenses are established, the data requirements depend on the approaches that are taken
to calculate those reserves.

Suppose reserves are being estimated by statistical methods and statistical checks are
being made on reserves based on case estimates. More detailed data is needed than is
required for case estimates, so that they can be split into appropriately homogeneous
groups for the projections. Many statistical methods require data that is tabulated in the
form of a run-off triangle.

The assumptions underlying the statistical methods are more likely to be borne out if the
model is applied to a homogenous group of claims. Fundamentally, what you need to
estimate outstanding claims is enough information to enable you to split the claims into
whatever sub-groups you want.

6.2 Data required


In general, the following data is required:

 dates of reporting and occurrence

 paid claims

 case estimates

 premiums.

In addition, the following data is sometimes required:

 number of claims

 other measures of exposure; eg turnover, payroll and vehicle-years

 expenses.

We could, and possibly should, take the view that the information needed to estimate claim
amounts is part of the data requirement for reserving. On that interpretation, the information
needed is just about everything: a skilled assessor is able to make use of all the available
information regarding a claim.

These are examined in more detail below.

6.3 Dates of reporting and occurrence


Depending on the reserving methodology to be used, the dates of reporting and occurrence
for each claim will be required to determine the likely extent of IBNR claims.

The Actuarial Education Company © IFE: 2019 Examinations


Page 36 SP7-12: Data

6.4 Paid claims and case estimates

Paid claims
These are actual amounts paid with respect to a particular claim. The payments may be
gross or net of recoveries, as is discussed further below.

Case estimates
These are the latest estimates of the cost of each claim of which the insurer is aware. A
history of case estimates is usually required.

Case estimates may be easy to determine for some classes of business, but complex for others:
 It may be easy to determine these case estimates if the benefit is fixed; such as the
compensation offered for the loss of a limb under a personal accident policy.

 When the estimation process is not so straightforward, a mechanical approach may


be used, or the judgement of an experienced claims handler or legal adviser.

It is important that we account in full for all the reported claims that are not regarded as
finally settled.

Considerations
Paid claims and case estimates may be required at the class level or at a more aggregated
level, depending on the reserving methodology.

For both paid claims and case estimates, the insurer may be able to obtain recoveries from
third parties with respect to some of these amounts. Such recoveries may arise from
subrogation or salvage.

The insurer should also consider likely recoveries from reinsurers.

When using the inflation adjusted chain ladder method, an inflation index is also required.
The index is applied to the past claims data to bring them into line with the latest year, and
to inflate the projected claims to the expected year of payment.

Checks
As discussed above, the insurer may be able to obtain recoveries from subrogation, salvage or
reinsurers.

We should check whether the data has already been adjusted for such recoveries and
decide whether to make such adjustments. Normally we need accurate information at gross
and net of reinsurance levels.

It should be checked that data net of any recoveries is consistent with data gross of such
recoveries.

Claims data obtained for reserving should be reconciled with the accounts to ensure that
the two sources are consistent with each other. Claims data should also be reconciled with
the general ledger.

The data should be compared to historical data to check for any unusual movements and to
understand why such movements have occurred.

© IFE: 2019 Examinations The Actuarial Education Company


SP7-12: Data Page 37

6.5 Premiums
Where premiums are used in the loss ratio calculation, the type of premium to use will
depend on the claims cohort. Earned premiums are appropriate for an accident year cohort.
Written premiums are appropriate for an underwriting year cohort. A reporting year cohort
would be very difficult to use in a loss ratio calculation, because premiums and claims
would have to be found with the corresponding exposure.

The premium that is consistent with the claims data should be used; for example, if claims
information is provided on an accident year basis, earned premiums should be provided or
written premiums should be converted to earned premiums.

It is usually more appropriate to use office premiums (that is, total premium received
including loadings for commission, expenses, profit, and so on) rather than risk premiums.

Here, the office premium means the premium that is charged to the insured. The risk premium is
the portion of the office premium required to cover only the expected claim amount.

This is because the loss ratio based on the office premium is more representative of the
actual experience since it expresses the cost of claims per actual premium charged. In
some cases, it may be more appropriate to use premiums net of commissions. These are
commonly used in the Lloyd’s market.

6.6 Number of claims


Information may be required with respect to the number of claims for some reserving
methods, for example, average cost per claim method.

The average cost per claim method will be discussed later in the course.

The information for numbers of claims may include information for numbers of reported
claims, reopened claims and closed claims (as is required in UK statutory returns).

6.7 Measures of exposure


In addition to premiums, other measures of exposure may be used, such as turnover,
payroll or vehicle-years. We typically use such exposure measures for exposure-based
statistical methods such as the Bornhuetter-Ferguson or expected claim ratio methods.
See Chapter 15 (Triangulation methods) for more details.

6.8 Expenses
The total indirect expenses (unallocated claims expenses and unallocated loss adjustment
expenses) are needed to calculate reserves for indirect expenses.

Direct claims settlement expenses (claim settlement expenses that can be assigned to
specific claims) are usually included within paid claims and case estimates, but this should
be confirmed with claims staff.

Total direct expenses (that is, claims expenses allocated to specific policy classes) are also
needed if these are not included within the case estimates and paid claim amounts.

The Actuarial Education Company © IFE: 2019 Examinations


Page 38 SP7-12: Data

Other considerations
The potential errors, omissions and distortions in the data should also be considered. This
issue was discussed in Section 5.

© IFE: 2019 Examinations The Actuarial Education Company


SP7-12: Data Page 39

7 Data requirements for reserving requiring specific consideration

7.1 Data items


For policy level (or policy group) exposure-based reserving we need to look at the following
information for each policy to establish the insurer’s potential liability given the type of
policies written:

 policy limits and excess

 policy start and end date

 insured perils

 exclusions to cover

 company share of total risk (for London Market business)

 paid claims

 reported outstanding claims

 date claim reported

 date claim occurred

 status of claim (that is, if claim is opened, closed, settled or reopened)

 name of assured / cedant

 type of loss

 event name / cause

 a reference to link the claims information to policy information.

Note: the above list is not exhaustive. Additional information may be needed, which may be
specific to the policy or policies concerned.

Certain losses may require specific considerations. This may be the case for:
 individual risks
 entire books of business
 certain perils / claim types.

We will now discuss these three cases.

The Actuarial Education Company © IFE: 2019 Examinations


Page 40 SP7-12: Data

7.2 Individual risks – large losses


Large losses could be projected using triangles or benchmarks.

If triangles are used, separate triangles may be used for large losses and non-large losses.
Large losses are usually defined as claims above a certain threshold. Large claims may
also be capped at a particular value so that they are only included up to a certain limit. Care
should be taken to ensure that the large and non-large loss triangles are not distorted, by
separating the large loss data from the total claims data.

As benchmarks for projecting large losses, loss ratios, ultimate to incurred ratios, ultimate
to paid ratios or benchmark development (or ‘link’) ratios may be used. Such benchmarks
could be based on market data or internal data from another book of business.

7.3 Entire books of business – a new book of business with little history
There will be limited historical data for a new book of business. The insurer may apply
market benchmarks or benchmarks from similar books of business to the premiums or
claims information to estimate reserves.

Examples of such benchmarks are:

 cumulative paid and incurred development patterns

 development (or ‘link’) ratios

 loss ratios.

7.4 Certain perils / claim types – asbestos


Triangulated data is not usually appropriate when reserving for asbestos claims. For example, the
claimant may be exposed to asbestos for a period of many years, so it is difficult to allocate claims
to any one origin year, and the development pattern is unlikely to bear a clear relationship to the
period since occurrence.

Therefore, exposure-based methods are usually used to estimate reserves with respect to
asbestos claims.

Latent claims will be discussed further later in the course.

© IFE: 2019 Examinations The Actuarial Education Company


SP7-12: Data Page 41

8 Data grouping and reduction of heterogeneity

8.1 What is the aim of risk classification?


The insurer will analyse the in-force data for its classes of business at regular intervals. For the
purpose of such analyses the insurer will want to ensure that there is sufficient data so that the
results are credible, but not so much data that heterogeneity is a significant issue.

We separate the insured risks into different groups suitable for our investigation, bearing in
mind that we will not distinguish between risks in the same group. The main consideration
in data grouping (ie risk classification) is to obtain homogeneous data. By reducing
heterogeneity within the data for a group of risks, we make the experience in each group
more stable and ensure that the risks within each group have similar characteristics, so that
we can use the data appropriately for projection purposes.

This is important when we monitor claims experience, review rating levels and estimate
outstanding claim values. Any heterogeneity in data groups will distort the results because
the average risk within the group may change in the future. This may cause understating or
overstating of the reserves.

For example, in motor insurance, property damage and bodily injury claims should be treated
separately for reserving purposes. For pricing purposes they should also be treated separately in
case there is a change in the mix of business, from fully comprehensive to third party cover.

8.2 You must consider the level of detail required very carefully
We should be careful when we classify risks to ensure that there is sufficient detail for all
users. Management will perform many analyses, which may require separation of
departments or intermediaries to assess performance.

In other words, some types of analysis will require data to be grouped by department or
intermediary, for example to compare the relative profitability of different distribution channels.

Personal lines classes tend to have standard policies and coverage. Hence it is usually
easier to group the claims into homogeneous groups, whilst ensuring sufficient volumes of
claims within each group.

Commercial lines business may include risks with tailor-made policies and coverage. In
comparison to personal lines business, it may be more difficult to group claims into
homogenous groups whilst ensuring sufficient claims volumes.

The Actuarial Education Company © IFE: 2019 Examinations


Page 42 SP7-12: Data

9 The effect of inadequate data on reserving


As a result of inadequate data, outstanding claim reserves and unexpired risk reserves may
be overstated or understated.

Consequences of overstatement:

 Results may appear worse than they are, leading to a loss of confidence by
shareholders, brokers, reinsurers and the stock market.
This may result in loss of new business or a reduction in the company’s share price.
 The solvency margin may seem reduced, possibly causing problems with the
regulators and rating agencies.

 Assets that could be applied more usefully to other projects by the insurer may be
tied in.
This would result in an opportunity cost to the insurer.
 Premiums based on the worsened results increase, with the consequence of
reduced market share.

 A profitable business line may be closed.

 The tax computation may reduce, at least in the short term.

Consequences of understatement:

 Overstated profits are prematurely distributed, leading to future problems meeting


liabilities.

 The insurer pays too much tax in the short term.

 Premiums will be reduced below profitable levels.

 Profit commission will be overpaid.

Incorrect reserving will affect reinsurance arrangements, with incorrect estimates of


recoveries and inappropriate premiums on renewal.

© IFE: 2019 Examinations The Actuarial Education Company


SP7-12: Data Page 43

10 Glossary items
Having studied this chapter, you should now read the following Glossary items:
 Benchmark
 Bordereau
 Claim cohort
 Credibility
 CRESTA zones
 Product costing
 Reporting year
 Risk group.

The Actuarial Education Company © IFE: 2019 Examinations


Page 44 SP7-12: Data

The chapter summary starts on the next page so that you can
keep all the chapter summaries together for revision purposes.

© IFE: 2019 Examinations The Actuarial Education Company


SP7-12: Data Page 45

Chapter 12 Summary
Sources of data
General insurance companies may make use of both internal and external data sources.

Industry-wide data collection schemes allow insurers to compare their own experience with
industry experience. Relying on industry-wide data, when available, has several problems, for
example: lack of detail and flexibility, differences in policies sold, different target markets and
sales methods.

Uses and users of data


Availability of a large quantity of good quality data is critical to a general insurance company.
It is also critical for actuaries working in general insurance. Actuaries need data mainly to
help make premium rating and reserving decisions.

Ideally all data in a general insurance company should be controlled through one single
integrated data system, although in practice this may not be possible. The actuary should be
involved in technical aspects of data collection and systems design.

Customer information will be used primarily to identify policyholders. However it may also
be used for cross-selling, relationship management etc. Data protection is of paramount
importance regarding both personal and commercial customers.

The full development team for a computer system should include senior management,
accountants, underwriters, claims managers, marketing, investment, computing staff, risk
management staff, catastrophe modellers and reinsurers, as well as actuaries.

Quality and quantity of data

The quality and quantity of data varies between different insurance organisations, and
within different organisations, between different classes of business. The key factors
affecting the quality and quantity of data are:
 size and age of company
 existence of legacy systems
 integrity of data systems
 management and staff
 nature of the organisation
 method of sale
 class of business.

The Actuarial Education Company © IFE: 2019 Examinations


Page 46 SP7-12: Data

Chapter 12 Summary continued


Establishing a good information system
The system should hold a large amount of detail from the policy and claim records.

Questions on the proposal and claim forms should be clear, unambiguous and objective. It is
necessary to capture key features relating to both:
 premiums (including written amounts, payment times, premium adjustments,
commission, other deductions and cross-selling information)
 claims (including the definition of a claim, estimated outstanding amounts, multiple
claim payments, reopened claims, claims handling expenses, reinsurance recoveries
and class-level adjustments).

The data to be recorded includes:


 risk definition and details of cover
 details of claim
 status of record
 control dates
 relevant amounts and currencies (sums insured, premiums etc)
 administrative details.

A history of records should be held (ideally) indefinitely. A balance will need to be struck
between the amount (and level of detail) of data held and the cost of the data system.

Errors, omissions and distortions in claims data

There are a large number of potential errors and distortions that must be overcome to allow
accurate analysis of the data. These include: wrong policy or claim number, wrong risk
details, wrong dates, wrong claim types, claims inflation, changes in procedures, case
estimates, delays, large claims and return premiums.

Errors can be avoided through appropriate use of check digits, data field integrity checks
(eg minimum and maximum values), mandatory fields, error reports and the training of staff.

© IFE: 2019 Examinations The Actuarial Education Company


SP7-12: Data Page 47

Chapter 12 Summary continued


Data requirements for reserving
The following data may be required for reserving:
 dates of reporting and occurrence
 paid claims (gross and net of recoveries)
 case estimates (which may or may not be straightforward to make)
 premiums
 number of claims
 other measures of exposure (eg turnover, payroll)
 expenses (both direct and indirect).

Consideration should be given to any recoveries (through subrogation, salvage or


reinsurance) that may be made, and to inflation of claims and expense amounts. Claims data
should be checked thoroughly and compared to other sources, such as the accounts and
historical data.

Losses requiring specific consideration


Special considerations and data requirements may be needed for:
 large losses – separate triangles or benchmark development factors may be used.
 certain perils / claim types, eg latent claims or large liability claims – exposure-
based methods may be used
 whole classes of business, eg a new book of business – market benchmarks or
benchmarks based on similar books of business may be used.

Risk classification
Data should be grouped into homogeneous cells, but consideration should also be given to
ensuring that there is enough data in each cell for it to be credible enough for all users.

The effect of inadequate data on reserving


If the reserves calculated from the data are incorrect, this will distort the reported results
and tax payments.

The Actuarial Education Company © IFE: 2019 Examinations


Page 48 SP7-12: Data

The practice questions start on the next page so that you can
keep the chapter summaries together for revision purposes.

© IFE: 2019 Examinations The Actuarial Education Company


SP7-12: Data Page 49

Chapter 12 Practice Questions


12.1 For a household insurance contract (covering both buildings and contents), list three pieces of
information that will be required for all contracts and six pieces of information that may only be
required for some policies.

12.2 List the scenarios in which:


(i) policy and claims data may not exist
(ii) there may be a low volume of relevant data.

12.3 State the factors limiting the usefulness of industry-wide data.

12.4 List the controls an insurance company might implement to ensure that policyholders’ personal
data is protected.

12.5 Describe the constraints that might prevent an ideal data system being maintained.

12.6 Explain why the policy and claims data for a commercial fire account may not be fully
computerised.

12.7 List the steps that could be taken to ensure that an insurance company’s policy data is accurate.

12.8 (i) Explain, in one sentence, the fundamental reason why statistical methods may not be
appropriate when estimating outstanding claim reserves.

(ii) List the possible sources of error when using statistical methods.

12.9 List the circumstances in which an insurer would put more reliance on the aggregation of case
estimates than statistical methods.

12.10 You are about to calculate the outstanding claim reserves for a block of domestic contents
policies using chain ladder techniques. List the data items and other information that you require.

12.11 Identify the effects of inadequate data on reserving and list the main consequences of such
effects.

12.12 (i) State the issues that data protection laws might cover. [5]
Exam style
(ii) Suggest the possible consequences to an insurance company of contravening such
laws. [3]
[Total 8]

12.13 You have taken over responsibility for a poorly designed and administered claims database. State
Exam style
the types of errors or distortions that you may find in that database, and suggest examples of how
they might have arisen. [16]

The Actuarial Education Company © IFE: 2019 Examinations


Page 50 SP7-12: Data

The solutions start on the next page so that you can


separate the questions and solutions.

© IFE: 2019 Examinations The Actuarial Education Company


SP7-12: Data Page 51

Chapter 12 Solutions
12.1 Information that would be required include:
 policyholder name
 policyholder address
 some indication of the type of property, eg house or flat
 some indication of the size of the house, eg estimated rebuild cost or number of
bedrooms
 the level of excess required.

Examples of information that may also be needed:


 estimated sum insured for contents
 list of valuable items to be covered
 whether the property is used for business
 whether the property is normally unoccupied during the day
 type and standard of construction
 age of the building
 types of locks / burglar alarms / smoke alarms fitted
 whether there are pets at the house
 family composition of residents
 age of residents
 whether residents are smokers / non-smokers
 type of heating
 whether claims have been made in recent years.

12.2 (i) Scenarios in which policy and claims data may not exist
Data will not exist:
 for products that are completely new to the market
 for products that are new to a particular insurance company
 for products whose terms and conditions have been altered significantly, eg different
exclusions
 where new rating factors are being used (assuming these have not been monitored
before)
 for business being written in new territories.

The Actuarial Education Company © IFE: 2019 Examinations


Page 52 SP7-12: Data

(ii) Scenarios in which there may be a low volume of relevant data


There will be a low volume of relevant data:
 for all the scenarios above, but when products are relatively new
 for unusual risks
 for large, unique risks
 for low frequency claims
 when data is incomplete, which may be due to:
– poor data collection techniques
– brokers / agents not collecting data accurately.

12.3 The main problem is the potential for distortions within the data (heterogeneity).

The data supplied by different companies may not be directly comparable because:
 it relates to different socio-economic or geographical areas of the market
 policy conditions may differ
 other practices may differ, eg underwriting, claims settlement
 the nature of the data stored may differ
 the coding used for the risk factors might vary.

Other potential problems are:


 data is usually less detailed and less flexible than that available internally to a company
 data is often more out of date than internal data
 quality may be suspect if some contributors have poor quality data systems or supply
incorrect information
 not all companies contribute.

12.4 Controls include:


 regular training of staff on:
– data protection regulation
– procedures relating to handling data
– consequences of breaching data protection rules
 employing specialists for roles relating to:
– data gathering
– data processing
– data use
 limiting the use of certain items of data to specified people through password protection
 providing guidelines to third parties (eg consultancies) that have access to the data, eg:
– on use of the data

© IFE: 2019 Examinations The Actuarial Education Company


SP7-12: Data Page 53

– requirement to destroy data after use


 employing a compliance department, that will monitor data protection legislation
 following rules on storage and transmission of data
 allowing the subject access to the data and then the right to have it corrected
 monitoring of data handling procedures.

12.5 Constraints include:


 age – over time, the system will gradually become out-of-date
 mergers and acquisitions could lead to the existence of legacy systems …
… or data being migrated on to one of the systems, leading to potentially empty data
fields
 new rating factors being added, leading to potentially empty data fields for historical data
 a reorganisation of class structures, with the insurer being unable to assemble the
historical data into the new classes
 human error, which is inevitable (eventually at least)
 conflicts between users, which could lead to a lack of data for some users …
… and too much data for others, resulting in a slower and more unwieldy system.

12.6 Commercial fire insurance data may not be fully computerised because for this class policies often
vary a great deal in terms of:
 the cover provided
 the risk factors.

Claims data may also be variable and depend on subjective reports by specialists.

It may not be practical to have a sufficiently flexible system to deal with all the variations.

Computerisation may deal with most ‘mainstream’ cases but, even then, data may not be fully
computerised. Special codes may be needed to refer to paper records for individual non-standard
cases.

12.7 Steps to try to ensure that an insurance company’s policy data is accurate (but you can never
ensure total accuracy):
 original form design: clear design that collects the correct information from policyholders
 original form design: clear and unambiguous questions
 data entry: should be typed straight from forms and input only once
 automatic validity checks on data entry
 use of unique policy numbers with check digits
 spot checks on sample data to trace any global errors

The Actuarial Education Company © IFE: 2019 Examinations


Page 54 SP7-12: Data

 spot checks that procedures are being correctly followed


 check key distributions of data for consistency
 subsequent monitoring and cross-checking
 checking totals are consistent with movements of policies
 individual checks on extreme data
 individual checks on empty fields
 ensure consistency with data from other sources, eg accounts
 give staff incentives to keep data accurate.

12.8 (i) The fundamental reason why statistical methods may not be appropriate

The underlying assumptions may be incorrect and/or there may be bias in the source
data.

(ii) Possible sources of error when using statistical methods


Possible sources of error from using statistical estimates include:
 change in the mix of business (where different mixes have different run-off speeds)
 policy conditions may change (again affecting speed of settlement or reporting)
 insufficient data generally (so random fluctuations are magnified)
 reporting delays may change, eg due to new procedures or postal delays
 settlement patterns may change, eg new procedures on partial payments
 large claim distortions, ie one-off large claims being projected
 past and future inflation assumptions are wrong
 further claims outstanding from earlier origin years
 secular or social trends not projected properly
 random fluctuations within the two sharp corners of the triangle (ie claims settled most
recently from the first and last origin years) are magnified by the methods
 change in the average cost of claim or definition of a claim will invalidate the average cost
per claim method
 if assumed run-off pattern or ultimate loss ratios are inappropriate, this will invalidate the
Bornhuetter-Ferguson method.

© IFE: 2019 Examinations The Actuarial Education Company


SP7-12: Data Page 55

12.9 New class with no past data.

Where there are a small number of claims outstanding, eg:


 small class of business
 low claim frequency
 last remaining claims from a cohort (in any class).

Classes with large variation in claim size, especially if the average claim size is large ...

… and there is a large amount of relevant data on each unsettled claim.

For very large claims (in any class – as the circumstances of these are likely to be very
unique) ...

… especially those sensitive to court decisions over liability or compensation.

Where there is no stability over time, eg in policy conditions, underlying risks or the period from
occurrence to settlement.

Where there is no suitable model, or the assumptions are unreliable.

Where the company has a strong team of experienced assessors.

12.10 Accident month / quarter / year and development triangulation data:


 paid and incurred claim amounts
 number of claims reported and settled
 recoveries and estimated future recoveries
 split by claim type, eg fire, theft, flood.

Background information on:


 changes in mix of business
 claim definition
 treatment of large / nil / multiple claims
 inflation
 changes in claim sharing agreements.

12.11 Inadequate data could lead to either over or under-reserving.

Under-reserving could lead to a shortfall of funds and an inability to meet liabilities as they
become payable.

Under-reserving will speed up the payment of dividends and tax.

Over-reserving will worsen the apparent results, possibly causing a loss of confidence in the
company if the true position is not considered.

The Actuarial Education Company © IFE: 2019 Examinations


Page 56 SP7-12: Data

Over-reserving will also reduce the apparent solvency margin, possibly causing problems with the
regulator / rating agencies.

Over-reserving may lead to more caution than would otherwise be necessary possibly reducing
overall returns to shareholders.

Over- or under-reserving may also filter through to the premium rating exercise, resulting in
incorrect rates.

An incorrect assessment of reserves may lead to inappropriate management decisions,


eg reinsurance purchasing, investment strategy etc.

12.12 (i) Data protection laws

Data protection laws may cover:


 what data a company may hold on its customers … [½]
… both personal and commercial … [½]
… and for how long [½]
 what a company may use its data for [½]
 rights of access by a customer to its own personal data [½]
 rights of a customer to have incorrect data corrected [½]
 which parties are authorised to handle data at different stages, eg:
– data gathering stage [½]
– data processing stage [½]
– subsequently [½]
 guidelines for third parties (such as consultancies) to adhere to when dealing with data [½]
 requirements to destroy personal data after use [½]
 requirements on the security of data when it is being:
– stored [½]
– transferred to other parties. [½]
[Maximum 5]

© IFE: 2019 Examinations The Actuarial Education Company


SP7-12: Data Page 57

(ii) Consequences of contravening laws

There may be the following consequences:


 criminal prosecution of the responsible parties [½]
 legal actions against the company [½]
 fines to the company [½]
 requirements to compensate the customer [½]
 restrictions on the future activities of the company [½]
 more frequent monitoring of the company [½]
 negative publicity for the company (and the individuals involved) … [½]
… which could impact business volumes. [½]
[Maximum 3]

12.13 Types of errors

Policy / claim numbers and linking


 wrong claim number – claim details allocated to wrong claim number initially [½]
 wrong claim number – claim details allocated to wrong claim number reopened [½]
 no policy number to link to [½]
 link to non-existent policy number [½]
 claim allocated to wrong policy number [½]
 reinsurance linking wrong [½]

Dates
 wrong claim date [½]
 wrong payment dates [½]
 wrong (or no) reported date [½]
 wrong processing date [½]
 reopened date not recorded [½]
 settled date not completed [½]

Amounts
 case estimates not systematically updated [½]
 case estimates not consistently updated [½]
 case estimate history not recorded – gives current position only [½]
 incorrect amounts recorded [½]
 amounts in wrong currency [½]

The Actuarial Education Company © IFE: 2019 Examinations


Page 58 SP7-12: Data

 wrong exchange rates used [½]


 inconsistent treatment of nil claims [½]
 catastrophe claims missing [½]
 recoveries information missing [½]
 type of payment not recorded [½]

Header fields
 wrong insured name [½]
 wrong policy year [½]
 wrong branch [½]

Detail fields
 wrong risk details [½]
 wrong peril code [½]
 claim description wrong / insufficient detail [½]
 poor description of large claims [½]
 poor / missing catastrophe code [½]
 no claim status flag [½]

How errors may have arisen

Processing delays
 paper records not yet input [½]
 claim settled but not input on the system [½]
 reinsurance collected but not booked [½]

Control errors
 no use of check digits [½]
 no use of validity limits [½]
 no consistency checks [½]
 no use of compulsory fields [½]
 no warning messages [½]

© IFE: 2019 Examinations The Actuarial Education Company


SP7-12: Data Page 59

Others
 poor training [½]
 high staff turnover [½]
 poor management / supervision [½]
 changes in claim handling processes [½]
 changes to basis for calculating case estimates [½]
[Maximum 16]

The Actuarial Education Company © IFE: 2019 Examinations


All study material produced by ActEd is copyright and is sold
for the exclusive use of the purchaser. The copyright is
owned by Institute and Faculty Education Limited, a
subsidiary of the Institute and Faculty of Actuaries.

Unless prior authority is granted by ActEd, you may not hire


out, lend, give out, sell, store or transmit electronically or
photocopy any part of the study material.

You must take care of your study material to ensure that it


is not used or copied by anybody else.

Legal action will be taken if these terms are infringed. In


addition, we may seek to take disciplinary action through
the profession or through your employer.

These conditions remain in force after you have finished


using the course.

The Actuarial Education Company © IFE: 2019 Examinations


SP7-13: Actuarial investigations and analyses Page 1

Actuarial investigations and


analyses
Syllabus objectives

5.2 Outline the major actuarial investigations and analyses of experience undertaken with
regard to reserving and capital modelling for general insurers.

The Actuarial Education Company © IFE: 2019 Examinations


Page 2 SP7-13: Actuarial investigations and analyses

0 Introduction
In much actuarial work, we analyse data and draw conclusions from the results. We
perform most analyses on a regular basis. Often in an investigation, we will compare the
expected forecast from the previous analysis with the actual observed experience since the
analysis was performed.

This section sets out the key investigations that are regularly performed, and others that
may be needed reasonably frequently.

You can use this chapter as an introduction to those later parts of the Course Notes. There is
plenty of overlap between the investigations outlined in this chapter and those covered in the
corresponding chapter in Subject SP8  if you are studying both subjects simultaneously, you may
want to read both chapters now.

Note: there is no limit to the investigations a general insurance actuary can be called on to
perform. It is important for the actuary to be clear on the question being asked and the data
available. The analysis that can be undertaken will depend on the available data and should
be appropriate to the question being asked.

© IFE: 2019 Examinations The Actuarial Education Company


SP7-13: Actuarial investigations and analyses Page 3

1 Reserve estimation
A general insurer needs to estimate its ultimate claims liability for a variety of purposes. All
insurers need to know the extent of their liabilities and the uncertainty around their
estimation.

The most common investigations are:

 To determine the liabilities to be shown in the insurer’s published accounts.


Published accounts are those prepared to give shareholders information. They show the
shareholders what the insurer has done with their money over the year.
We will require this at least annually, and most probably half-yearly or quarterly. It is
likely that we will present the results to senior management and the company’s
Board, and they will be audited annually. We should support the reserve estimation
exercise by documentation, include reconciliations of the base data to the
accounting records and provide an explanation of the key drivers of the results and
the main changes in assumptions and results from the previous exercise.
The analysis of figures from an insurer’s accounts is covered further elsewhere in the
course.
 To prepare separate accounts for the purpose of supervision of solvency and to
determine the liabilities to be shown in those accounts, if necessary.
The same comments apply here as to the published accounts, except that the
primary audience of these accounts will be the local regulator.
Solvency accounts are those required to demonstrate to the supervisory authorities that
the company can be expected to meet its obligations due to policyholders.
 To determine the liabilities that we show in internal management accounts of the
insurer.
This is likely to be a side product of the reporting for published accounts, although
we may perform this more regularly, for example, monthly. We may present results
at a more detailed level (broken down into smaller groups), and on a best estimate
basis, rather than with any margin for uncertainty.
Management require sufficient detail to enable them to make suitable decisions about
the direction in which the company should be taken.
Although a best estimate basis is used, the management would probably also want to see
what could go wrong, and so may also be interested in other bases or scenarios.
 To value an insurer for purchase or sale.
This is one of those investigations that we may need from time to time. As part of
the due diligence process, a purchaser will want to evaluate the liabilities of the
insurer to be acquired and make sure that there are no undiscovered liabilities. A
seller will wish to make sure that hidden margins in claims reserve estimates are not
being given away.
We can do the investigation at different stages of the transaction process, for
example, as more data is made available to the purchaser. It is important to
highlight areas of uncertainty that are relevant from the perspective of the vendor or
the purchaser, depending on which party we are advising.

The Actuarial Education Company © IFE: 2019 Examinations


Page 4 SP7-13: Actuarial investigations and analyses

 To assess the accuracy of the company’s case estimate and/or IBNR claims reserve
estimation in previous year-end exercises.
We will do this by comparing the eventual out-turn (outcome) of claims paid with the
earlier estimates (actual versus expected). We can do a similar analysis on
movements in incurred claims. The degree of detail (level of disaggregation) will
depend on the importance of the reserves to the overall insurance results and the
credibility of available data.
This might cause us to change the assumptions that we use in the future, or even
change the model or methodology.

 To provide information to management on how areas of the business are


performing, and provide an indication on the profitability of business currently being
written.
Management may become concerned about one particular area of business 
perhaps a distribution channel or a sub-sector of an account  and will ask for a
more detailed analysis of the claims experience.

Other common investigations include:

 comparison of best estimates against held reserves

 calculation of ranges of results

 transforming an underwriting year into an accounting year

 calculating movements in reserves and analysing reasons for these.

Another common investigation is to calculate reserves in order to estimate the cost of claims
incurred, as an intermediate step in the premium rating process. However, this is more relevant
to Subject SP8.

Some of these investigations are covered in more detail throughout the rest of the Course Notes.

The choice of methods and assumptions that we use depends on the purpose of the reserve
estimates and on the extent and quality of the data available.

Question

Suggest how the strength of basis will differ between the suggested main purposes for estimating
liabilities.

Solution

Published accounts: prudent side of best estimate to ensure stability and consistency from year to
year.

Supervision of solvency: the most prudent in order to protect policyholders.

Internal management accounts: best estimate to give a realistic view of the financial condition of
the company.

Purchase or sale: close to best estimate, but preferably on the optimistic side if you are selling and
on the cautious side if you are buying.

© IFE: 2019 Examinations The Actuarial Education Company


SP7-13: Actuarial investigations and analyses Page 5

Accuracy of previous reserve estimates: often no estimation is needed since we are comparing
actual paid claims with those expected by previous estimates. If we are analysing incurred claims,
then it makes sense to use the same basis as that used in the previous reserve estimation exercise
in order that a meaningful comparison can be made.

Internal profitability: best estimate in order to get a realistic view.

The Actuarial Education Company © IFE: 2019 Examinations


Page 6 SP7-13: Actuarial investigations and analyses

2 Investment and capital


The investigations relating to the capital of a general insurer that we may need include:

 Evaluation of the existing portfolio.


We will review the current portfolio to see what return has been made, how this
compares with what was expected and what the market has done.
We will need to establish the mean term of the liabilities.
This is to ensure that the assets held are appropriate, given the insurer’s liabilities.
We will calculate current market values and the total assets minus total liabilities,
compared to regulatory capital requirements and those set by the Board of the
company. We are likely to do this monthly, although in times of market turbulence
or when the capital levels look low, we may do this more frequently.
If assets are valued on a mark-to-market basis, a company that invests predominantly in
volatile assets such as equities may see its solvency fluctuate considerably, even if
liabilities remain stable. This can have adverse consequences for the financial stability
perceived by rating agencies and regulators, and so many insurers try to avoid
over-exposure to such assets.
 Analysis of cashflow and asset liability modelling to assess the capital required to
support the business, assess where the biggest risks are in the business and help to
set the investment policy.
The analysis could be deterministic or stochastic, and we should not underestimate
the complexity of either approach. More detail on the mechanics of this is covered
later in this course.

 Allocation of capital between different classes of business.


This requires us to allocate capital between different classes, and possibly to
hypothecate assets to different classes. The allocation will probably come from the
analysis of capital requirements. There are several ways of approaching this, and
none of them is perfect. The actuary should use a pragmatic and consultative
approach.

 Determination of return on capital.


Having allocated the capital to different classes of business, we can analyse the
claims experience to determine the profitability of each class  expressed as a
return on capital  once we have appropriately split investment expenses. We can
compare this to the expected return and explain the differences.

This will allow shareholders and analysts to assess management performance.

Question

Explain why cashflow is particularly important for a general insurer, compared to a life insurer or a
pension scheme.

© IFE: 2019 Examinations The Actuarial Education Company


SP7-13: Actuarial investigations and analyses Page 7

Solution

One event may cause a large number of claims. Consequently cash would be needed to meet the
claim payments. For life assurance and pension schemes the payments are more predictable and
more evenly spread over the year.

Also, the size of any individual claim is usually uncertain, whereas in life and pensions business the
claim size is often known or easy to predict.

The Actuarial Education Company © IFE: 2019 Examinations


Page 8 SP7-13: Actuarial investigations and analyses

3 Analysis of experience
We need to analyse an insurance company’s past experience for many of the functions of
the business. Management will often use the results of this analysis to assess the
performance and profitability of the business.

3.1 Areas of analysis


The analysis may cover:

 Pricing and sales of policies – premium rating, risk acceptance and profitability of
sales channels.
This is covered more fully in Subject SP8.
 Claims reserve estimation and changes to claims experience or the claims
environment.
This will be covered further in the next section.
 Exposure analysis and potential aggregations of risk within an insurer’s portfolio.
This analysis also helps with reinsurance planning.
 Policyholders’ behaviour, for example, the likelihood that they will renew the policy
or cancel midway through the policy period.
This is covered in detail in Subject SP8.
 Estimation of claim trends, in particular frequency and inflation.
This is covered in the next section.
 Other analyses dependent upon the information required by management, and the
extent to which data is available.

 Expense analysis and allocation – both direct and indirect.


You may be familiar with the analysis and allocation of expenses from your earlier studies.
It is also covered in depth in Subject SP8.

Question

Define and give examples of direct and indirect expenses for a general insurer.

Solution

Direct expenses are those identified directly as belonging to a particular class of business. On the
other hand, indirect expenses do not have a direct relationship to any one class of business.
These need to be apportioned between the appropriate classes when performing an analysis.

Direct costs, incurred directly through providing insurance cover, include underwriting costs,
commission and claims settlement expenses.

© IFE: 2019 Examinations The Actuarial Education Company


SP7-13: Actuarial investigations and analyses Page 9

Indirect costs relate to the support functions, such as:


 computing costs
 personnel department
 general management costs.

3.2 Communication of analysis


In presenting the results of such analyses, it is very important for us to communicate:

 from where we have sourced data, on what basis the data is prepared, and how it
has been verified
For example, we may need to confirm whether premium amounts provided include
or exclude commission payments to the broker.

 which results are factual, and which results have required assumptions and/or a
projection method to be derived
In the case of the latter, we should communicate the extent of uncertainty of any
projected results, perhaps by showing how the end result would look if different sets
of assumptions were used.

 how measures have been defined or the method of calculation used


Some statistics, such as cancellation rates, may be the result of a calculation, and
there may be more than one method of calculation. We should make this clear;
otherwise management may misunderstand the results and make the wrong
decisions.

 the key features of the results and their implications

 how the actual results compare against those expected

 why the features have happened.

This may require qualitative information from administration or claims areas.

The Actuarial Education Company © IFE: 2019 Examinations


Page 10 SP7-13: Actuarial investigations and analyses

4 Analysis of claims experience


We should analyse the company’s own claims experience and changes to the claims
environment in detail as part of the actuarial reserve estimation exercise. We can use a
number of methods to identify trends in the data, including the review of diagnostics. This
is covered in more detail later in this course.

Below, we outline a number of analyses. Please note that the list is not exhaustive.
Depending on the purpose of the analysis, we may do this:

 at an overall company level, or

 at a class of business level, or

 at a more granular level, for example, by risk or rating factor.

We should compare the results to market benchmarks where such data are available.

When analysing claims, we should consider the extent to which expense or policy movements
should be categorised into risk groups. For example, in private motor, should we analyse
experience split down:
(a) into fully comprehensive versus third party cover, or
(b) by all available rating factors simultaneously?

For (a), there would only be two separate categories and we should have plenty of data in each
category. However, in many cases the results of the analysis would be too crude to be of real
practical use (eg in setting premium rates for each risk group).

For (b), there could be as many as 50 million combinations of rating factors in private motor. Such
an analysis would be totally ridiculous, even for the largest motor insurers.

When deciding on the extent to which we should split the analysis by rating factor, we should
consider three key points:
(a) What is the purpose of the investigation?
(b) Do we need to split into different risk groups? For some analyses, the different rating
factors do not have much of an impact on the results.
(c) Have we enough reliable data to support a detailed analysis by risk group?

4.1 Changing frequency and severity of reported and settled claims


In analysing the cause of changes to the overall claims experience of a particular account, it
can be very useful to split the effects of:

 frequency (number of claims reported or settled per unit exposure) and

 severity (the typical or average cost of a claim reported or settled).

Different influences will impact each of these in different ways and we construct most
models so that we can implement changes and trends to these two aspects separately.

© IFE: 2019 Examinations The Actuarial Education Company


SP7-13: Actuarial investigations and analyses Page 11

Using such analysis, we may identify the reasons for better (or worse) than anticipated
performance of the account, such as a sharp increase to the average cost of settling bodily
injury claims under a third party motor liability policy, or a sudden increase in the number of
claims under a warranty policy of a particular car manufacturer.

Question

Suggest a possible reason for:


(a) A sharp increase in the average cost of settling bodily injury claims under a third party
motor liability policy.
(b) A sudden increase in the number of claims under a warranty policy of a particular car
manufacturer.

Solution

(a) This could be due to, for example, a change in legislation (or precedence) that makes
awards for consequential loss claimable on top of the indemnity amount.

(b) One reason might be the faulty design of a particular new model of car.

It may also provide much more information on the underlying experience of an account that
seems to be stable, for example, if overall claim amounts are masked by an increase in one
element and a decrease in the other.

This approach requires claim number data to be available. If this information is maintained
historically, development triangles of the numbers of claims reported and/or settled may be
available, allowing us to make projections to ultimate.

4.2 Impact and incidence of large claims


The company may wish to analyse its large-claims experience. This may be useful to
determine the drivers for such claims, and specific areas of the account that are more prone
to certain types of ‘large’ claim.

When estimating reserves, it can be useful to analyse ‘attritional’ and large claims
experience separately.

Attritional claims are what we call normal, non-large claims.

In many cases, we will use different reserve estimation methods for each of these. If left
unadjusted in the aggregate data, individual large claims would distort the experience of the
risk group.

This can cause problems when reserving as large claims often develop differently from attritional
claims, and so can distort triangulations.

We should produce a report detailing all claims that exceed a certain size so that we can
remove or truncate such claims. The threshold points for reporting and truncating will
differ according to the class involved and the purpose of the analysis.

The Actuarial Education Company © IFE: 2019 Examinations


Page 12 SP7-13: Actuarial investigations and analyses

We may also need to vary the large claim threshold over time to make allowance for the
effects of claims inflation. For example, if claims inflation has been 5% for each of the last
three years, then we would adjust a current threshold of £500,000 to £432,000 if the claim
occurred three years ago.

If we didn’t do this, we would have an inconsistent number of large claims in different years of
account. For example, if we only looked at past claims over £500,000 we wouldn’t see
particularly many from very old years (because £500,000 was more extreme then, than it is now).

The analysis of large claims will help us to decide which reinsurance coverages are
appropriate and will be a key driver of the capital which the business requires.

For example, if you are analysing claims for the purposes of planning reinsurance, don’t be
tempted to truncate (or exclude) large claims since it is these claims that you’re most interested
in.

There is further discussion of large losses in other parts of the course, particularly in
Chapter 15.

4.3 Concentrations of claims and aggregations of risk


As with individual large claims, it is important that we can identify concentrations of claims
so that we can deal with them separately from the main body of the data. In some cases, we
may be able to do this from the individual claim records by considering such factors as
class, date of claim, peril, type of claim, geographical area and so on.

For example, an aggregation of claims may arise within a domestic household book of business
due to a large number of households being insured in one geographical area.

However, this approach may not be reliable. Therefore, when it is apparent that an event of
significant proportions has occurred, it is usual for the insurer to set up a catastrophe code
for the event, which is then placed on the claim records for all claims arising from the event.

This will also assist with any reinsurance recovery. Increasingly, reinsurers require detailed
information regarding the exposure aggregations within the underlying portfolio of risks
being insured, and the resulting accumulation of claims that may arise from a single event,
such as a hurricane.

4.4 Indemnity and expense split


It is very useful for the insurer to understand how its claim costs break down between:

 indemnity payments (those that return the policyholders to the position they were in
before the loss occurred) and

 expenses (which may include costs of investigation, legal defence costs and so on).

Question

Suggest lines of business where the expenses element will be a considerable element of the total
claims cost.

© IFE: 2019 Examinations The Actuarial Education Company


SP7-13: Actuarial investigations and analyses Page 13

Solution

Liability classes or any other business where there is potential for disputes and court involvement.

In some instances, the insurer may need to review its claim settlement procedures to
reduce costs. These procedures may include how it chooses to defend claims, for example,
potential savings of settling prior to the case being referred to arbitration or a court of law.

We should not allow indirect expenses to distort the claims data as they should form part of
the administration expenses. It is possible that some of these indirect expenses have been
allocated to risk classes. If we do not separate the indirect expenses from the claim cost on
each individual record, the data could become corrupted.

Direct expenses are, however, often included as part of the claim cost on each individual
claim record. Provided that they remain consistent over time and users are aware that the
claim figures produced include such expenses, their inclusion in the claims data should
cause no problems.

However, by keeping separate records of expenses, we have the flexibility to produce


figures, inclusive or exclusive of these expenses, for example, to enable separate
allowances for inflation to be made.

Question

Suggest the main type of inflation that expenses may be exposed to.

Solution

The main type of inflation is salary, for the labour cost.

4.5 Types of claim reported


We may analyse the types of claims reported to identify:

 new types of claim emerging, or

 the effectiveness of exclusions or changes to policy excesses in removing certain


types of claim from the experience.

4.6 Recoveries made on gross claim amounts


An insurer will want to analyse any recoveries it can make on claims, to reduce its net outgo.

The three main types of recovery on an insurer’s gross claims experience are from
outwards reinsurance protections, salvage, or a third party involved in the claim incident. In
each case, we should be able to identify these separately on the claim record in an insurer’s
claims database.

The Actuarial Education Company © IFE: 2019 Examinations


Page 14 SP7-13: Actuarial investigations and analyses

Question

Define salvage.

Solution

The Glossary defines salvage as amounts recovered by insurers from the sale of insured items that
had become the property of the insurer by virtue of the settling of a claim.

In the case of reinsurance, it is usually a statutory requirement to analyse claims both gross
and net of reinsurance. This may also help to produce more reliable projections. This is
especially important where the reinsurance takes some form of XL cover and where there
are limited reinstatements.

As XL cover is non-proportional, the net and gross claims results will often be very different,
whereas for proportional reinsurance they are more similar. Reserving is that much easier for
stable data.

Reinstatements (or any other loss-sensitive feature) also act to make the net and gross results
very different.

4.7 Nil settlements


Some claims are settled at no cost  usually either because the claim amount ends up less than
the excess, or because the claim is eventually found to be invalid. Should these count as a claim
or not?

If one part of the portfolio naturally gives rise to more nil claims than another, a change of
mix of risks within the data group could also alter the number of nil settlements in the
aggregate data.

More particularly, some policyholders may report every incident that may result in a claim,
whilst others may report incidents only when it becomes apparent that a claim will be made.
If the mix of policyholders changes, this could affect the number of nil settlements in the
data for that risk group.

In each case, the effect is the same. The aggregate number of claims will change, but the
aggregate cost will remain the same. The effect of this will be to distort both the claim
frequency and average cost per claim.

It is, therefore, important to be able to separate nil claims from those settled at some cost.

4.8 Partial payments on account


For each claim, it is important to record each payment separately, with the date(s) of
payment, and the status of the claim (that is, whether settled or outstanding). Then, when
the data is aggregated, we will be able to distinguish between payments on settled claims
and those still outstanding. If not, the average cost of settled claims and the claim payment
patterns will be distorted.

© IFE: 2019 Examinations The Actuarial Education Company


SP7-13: Actuarial investigations and analyses Page 15

Payment patterns are particularly important when using triangulation methods such as
chain-ladder methods. One key assumption of the chain-ladder methods is that claim
development is stable. If this assumption is not valid, the method becomes unworkable without
adjustment.

4.9 Reopened claims


We should allocate reopened claim payments to the risk group and claim year of the original
claim. If we create a separate claim for the additional (or return) amount involved, we could
distort the apparent overall experience of the subgroups concerned, the numbers of claims
and hence the average cost per claim.

The Actuarial Education Company © IFE: 2019 Examinations


Page 16 SP7-13: Actuarial investigations and analyses

5 Reinsurance investigations
We need to analyse several aspects of the reinsurance requirements of a general insurer,
such as:

 The amount of risk that an insurer can safely retain, having regard to its solvency
position.

 The extent of the likely exposure to accumulations of risk.


Aggregate excess of loss reinsurance can be purchased to reduce the cost of aggregations
of claims resulting from an accumulation of risk. For example, an insurer might suffer an
aggregation of subsidence claims as a result of having a concentration of household
policies in a region prone to subsidence, perhaps due to the type of soil there.
 The need for catastrophe reinsurance, having regard to the insurer’s exposure to
both natural and human-made catastrophes, and the appropriate upper and lower
limits for such cover.

 The extent of the possible need for reinstatements to cover.


Following a reinsurance claim, a reinsurer will often require an additional payment (a
reinstatement premium) to be paid in order to restore full cover. There will usually be a
limit to the number of reinstatements that can be made before the reinsurance cover is
exhausted. Reinstatements were discussed in the chapters on reinsurance.
 The value for money provided by the existing reinsurance programme, and whether
it meets the objectives of the business; for example, does it sufficiently reduce the
volatility of the claims experience?

 A general assessment of the appropriateness of existing covers (proportional vs


non-proportional, for example).

 The profitability of layers.

 The effects on capital.

The above analyses are linked to the company’s capital modelling – we will run the capital
model using different reinsurance arrangements to see the effect on capital and profit
volatility under different scenarios. We will compare the results to the company’s risk
appetite to see which programme fits best. We will also consider the effect on return on
capital or profitability to see which programmes offer the best value for money.

So the insurer will test different reinsurance assumptions in the capital model to see which
arrangement gives the best trade-off between risk and return.

We note that in many territories, reinsurance will reduce the solvency capital need: so, we
should compare the analysed reinsurance cost with any potential saving in the cost of
capital.

Capital modelling is covered further in later chapters.

© IFE: 2019 Examinations The Actuarial Education Company


SP7-13: Actuarial investigations and analyses Page 17

5.1 Risk of accumulations


We can investigate the effect of accumulations of risk, particularly in the property classes
that are prone to natural catastrophe events and severe weather. We should explain any
results and the reliance on any external natural perils model and the uncertainties inherent
in the results.

The capital model should show the extreme events that may impact the insurer. This will
give an insight into the number of reinstatements needed for reinsurance (depending on the
likelihood of the events), the cost of the reinstatements and the company’s risk appetite.

Catastrophe models are covered further in Subjects SP8 and SA3.

5.2 Check on strength / solvency of reinsurer(s), and assess any bad debt
provision requirements
A direct insurer will wish to reinsure with sound reinsurers and will investigate financial
soundness before buying. An insurer will need to set up a bad debt provision for any
doubtful recoveries and assess the likelihood of future bad debts in order to assess capital
requirements. We do this by investigating external ratings of reinsurers, their financial
statements, and analyses prepared by reinsurance brokers. We should monitor aggregate
exposures with each reinsurer.

In the UK, for example, there are rules around the use of bad debt provisions, since they could be
used to reduce apparent profits (and hence defer tax payments).

Question

Describe briefly the consequences for an insurer of taking out too much or too little reinsurance.

Solution

If too little reinsurance is used, and experience is worse than assumed, there is a risk of sharply
reduced profit and lower published solvency or even insolvency for the insurer.

If too much reinsurance is used then the reinsurer will be profiting at the insurer’s expense in the
long term. Hence the insurer’s long-term profits will be lower.

The Actuarial Education Company © IFE: 2019 Examinations


Page 18 SP7-13: Actuarial investigations and analyses

6 Utilising actuarial investigations


We may use the results of the above types of investigations to:

 carry out financial projections for budgeting, strategy and solvency purposes
This would be a projection of future accounts and balance sheets, either deterministically
or stochastically. A number of key variables could be sensitivity tested, or a number of
different scenarios tested. The model could explicitly allow for new business. This is
similar to ‘model office’ projections used in life assurance.
 value a general insurer for sale, purchase or merger

 facilitate other reserve estimation and capital modelling decisions.

General insurers, like other companies, prefer to make decisions based on information and
data. Some will be regular decisions, such as which reserve estimation methodology is
best suited to particular classes of business. Other decisions will be one-offs, such as the
quantum of claims arising from a particular catastrophe or whether it might be appropriate
to outsource claims and policy management to a different country where costs are lower.

Before starting any investigation, we should be clear on the question being asked and the
limitations of the data. Management will base decisions on these analyses, and it is
important that they understand all relevant information and assumptions, particularly in
situations where the person preparing the analysis is not part of the discussion. Otherwise
results can be taken out of context and used in ways that were not envisaged when the
analysis was undertaken.

© IFE: 2019 Examinations The Actuarial Education Company


SP7-13: Actuarial investigations and analyses Page 19

7 Glossary items
Having studied this chapter you should now read the following Glossary items:
 Allocated loss adjustment expenses (ALAE)
 Claims run-off analysis
 Endorsement
 Functional costing
 Lapse
 Lapse rate
 Partial payment
 Persistency
 Unallocated loss adjustment expenses (ULAE).

The Actuarial Education Company © IFE: 2019 Examinations


Page 20 SP7-13: Actuarial investigations and analyses

The chapter summary starts on the next page so that you can
keep all the chapter summaries together for revision purposes.

© IFE: 2019 Examinations The Actuarial Education Company


SP7-13: Actuarial investigations and analyses Page 21

Chapter 13 Summary
There are a number of actuarial investigations that are necessary for a general insurer,
including:
Reserve estimation
 to determine liabilities in published, solvency and management accounts
 to value an insurer for purchase or sale
 to assess the accuracy of previous reserve estimates
 to provide management information on performance and profitability.

Investment and capital


 to evaluate the existing portfolio
 to assess capital requirements, risk, and investment policy
 to allocate capital between classes
 to determine return on capital.

Analyses of experience may cover:


 pricing and sales of policies
 claims reserves estimation and changes to claims experience or environment
 exposure and aggregation of risk
 expense analysis and allocation
 policyholder behaviour, eg renewal and cancellation rates
 estimation of claim trends
 anything else the management require.

When presenting results, we must communicate:


 the source of the data, its preparation and verification
 our reliance on assumptions and the uncertainty of the results
 methodologies and definitions used
 key features of the results, and why they have happened
 how actual compares with expected.

The Actuarial Education Company © IFE: 2019 Examinations


Page 22 SP7-13: Actuarial investigations and analyses

Chapter 13 Summary continued


The main claims analyses are:
 changing frequency and severity of claims
 impact and incidence of large claims
 assessing concentration of claims and risk
 splitting indemnity cost from expenses
 new types of claim
 recoveries on gross claims
 nil claims
 partial payments
 re-opened claims.

The main reinsurance investigations are to analyse the:


 amount of risk an insurer can safely retain
 extent of accumulations
 need for catastrophe reinsurance
 need for reinstatements
 value for money
 appropriateness of cover
 profitability of layers
 effects on capital.

These investigations can be used to:


 carry out financial projections for budgeting and solvency
 value an insurer for sale, purchase or merger
 facilitate other reserve estimation and capital modelling decisions.

Some of these investigations are considered in more detail elsewhere in the course.

© IFE: 2019 Examinations The Actuarial Education Company


SP7-13: Actuarial investigations and analyses Page 23

Chapter 13 Practice Questions


13.1 State the main purposes for which an insurer analyses claims data.

13.2 List the main areas of a general insurer’s operations in which experience analyses are used.

13.3 You are an actuarial consultant who has been asked to review the reserving calculations for a
Exam style
large household contents book of business. Suggest appropriate checks that should be
undertaken to ensure that the calculations have been carried out correctly. [8]

13.4 A large general insurer that sells a wide range of liability and property damage insurance wants to
Exam style
analyse its large and exceptional claims in order to set reserves for them.

(i) State how large and exceptional claims might be defined. [2]

(ii) Discuss the merits of each definition. [8]


[Total 10]

The Actuarial Education Company © IFE: 2019 Examinations


Page 24 SP7-13: Actuarial investigations and analyses

The solutions start on the next page so that you can


separate the questions and solutions.

© IFE: 2019 Examinations The Actuarial Education Company


SP7-13: Actuarial investigations and analyses Page 25

Chapter 13 Solutions
13.1 There are many possibilities, including:
 reviewing premium rates and risk acceptance criteria
 claims reserve estimation and analysing changes in claims experience or claims
environment, including inflation
 comparing actual claims run-off against previous estimates
 assessing the relative profitability of different blocks of business
 monitoring the adequacy and use of reinsurance
 valuing a general insurer for sale, purchase or merger
 asset evaluation for capital modelling decisions
 monitoring the insurer’s asset / liability position
 financial planning for budgeting and solvency purposes.

13.2 Areas of a general insurer’s operations include:


 rating
 reserving
 asset liability modelling and investment
 expense allocation
 risk management
 capital management
 financial planning, including profitability monitoring and solvency
 reinsurance programme performance
 marketing.

13.3 Appropriate checks that should be undertaken include:


 check for completeness of the data, eg raw claims data reconciles with the accounting
information [½]
 high level checks of frequency and average cost over periods of time [½]
 compare data with that used in a previous review [½]
 where loss ratios are used, check that claims have attached correctly to the policy data [½]
 check that data hasn’t been corrupted in the cleaning process [½]
 check that the data has fed through correctly into any reserving software packages [½]
 check estimates against those used by other departments, eg the pricing team [½]

The Actuarial Education Company © IFE: 2019 Examinations


Page 26 SP7-13: Actuarial investigations and analyses

 check that frequency and average cost models combine correctly, eg by origin year and
type of claim [½]
 check that frequency, inflation or any other trends have been correctly projected to the
expected claims payment dates [½]
 check that allowance for reinsurance recoveries is consistent with the reinsurance
programmes in place [½]
 check that appropriate allowance has been made for individual large claims and
accumulations [½]
 as you are a consulting actuary, you are likely to have information regarding competitors’
reserves for similar business and therefore can check against this [½]
 ensure that an appropriate allowance is made for claims handling expenses, or that a
separate reserve has been set up for these [½]
 check that only household contents data is included, eg where combined buildings and
contents business is sold [½]
 check that recent and appropriate data has been used [½]
 check the credibility of data in cells and homogeneity within cells [½]
 check the suitability of the model and data cells [½]
 check that any changes in the reserves look reasonable by comparing with previous
reviews. [½]
[Maximum 8]

13.4 (i) Definition of large and exceptional claims

Large claims could be defined as claims over a particular threshold limit … [½]

… possibly with different thresholds for different classes of business and/or different claim types.
[½]

These thresholds should be increased over time. [½]

Alternatively, they could be determined subjectively by management. [½]


[Total 2]

(ii) Merits of the different definitions

A single threshold for all risks

The advantages of this method are:


 it is objective [½]
 since no judgement is needed, the same claims will be classified as large or exceptional
regardless of who carries out the analysis [½]
 there will also be consistency from year to year [½]
 it is easy to apply. [½]

© IFE: 2019 Examinations The Actuarial Education Company


SP7-13: Actuarial investigations and analyses Page 27

The disadvantages are:


 it may be hard to find a threshold limit that would be appropriate to all lines of
business … [½]
… eg what is considered large will vary significantly for personal lines and commercial
lines business … [½]
… and types of claim … [½]
… eg what is considered large will vary significantly for liability and property damage
claims [½]
 the threshold will need to increase over time, and it might not be easy to determine the
rate of increase. [½]

Different thresholds for different classes of business / claim types

This would overcome some of the disadvantages above. [½]

However, it introduces the following disadvantages:


 it introduces extra complexity, so that the analysis may take longer to carry out [½]
 it requires an appropriate threshold to be determined for each type of business / type of
claim, which might not be straightforward [½]
 there may still be heterogeneity between claims within each business class / claim type,
eg claims in different geographical areas might vary markedly in size. [½]

Subjective management decision

The main advantage of this is the complete flexibility over claims classification. [½]

Experienced claims managers may be able to add valuable input into the determination of such
claims. [½]

This may be particularly important for very unusual claims. [½]

It may also be the most appropriate method for catastrophe claims and latent claims, where the
type of claim is likely to be unique. [½]

The disadvantages of this approach are that:


 it is not objective [½]
 it requires experienced assessors to determine what are large or exceptional, and any two
assessors may arrive at different conclusions [½]
 it may be hard to be consistent from year to year. [½]
[Maximum 8]

The Actuarial Education Company © IFE: 2019 Examinations


Page 28 SP7-13: Actuarial investigations and analyses

End of Part 2

What next?
1. Briefly review the key areas of Part 2 and/or re-read the summaries at the end of
Chapters 11 to 13.
2. Ensure you have attempted some of the Practice Questions at the end of each chapter in
Part 2. If you don’t have time to do them all, you could save the remainder for use as part
of your revision.
3. Attempt Assignment X2.

Time to consider …
… ‘learning and revision’ products
Face-to-face Tutorials – If you haven’t yet booked a tutorial, then maybe now is the time to
do so. Feedback on ActEd tutorials is extremely positive:

‘I find the face-to-face tutorials very worthwhile. The tutors are really
knowledgeable and the sessions are very beneficial.’

‘Forces me to study material and keep progressing. Without them there is


no/little incentive to make use of early study days.’

‘The ActEd tutorials are kept to the point and the tutors work you far
harder than you would be motivated to alone, plus they're on hand to
answer/explain any questions or difficult concepts. They are always happy
to do so in a variety of ways so I think by the end of the day most of the
class have grasped the main concepts.’

You can find lots more information in our Tuition Bulletin, which is available on our website at
www.ActEd.co.uk.

© IFE: 2019 Examinations The Actuarial Education Company


SP7-14: Reserving bases Page 1

Reserving bases
Syllabus objectives

3.2 Suggest appropriate reserving bases for general insurance business, having regard
to:
 the different reasons for calculating reserves
 the assumptions that might be appropriate in each case
 why the assumptions may differ from a rating exercise
 the allowance for future inflation
 whether or not to discount for investment income
 the approach for additional unexpired risk reserve
 communication of the reserving basis.

The Actuarial Education Company © IFE: 2019 Examinations


Page 2 SP7-14: Reserving bases

0 Introduction
To place a value on the liabilities we must use a reserving basis. There are many possible
approaches we could use in practice. Several factors help determine what an appropriate basis
may be in a particular situation, the most important of which is usually the purpose of the
investigation. This chapter is a study of these factors and how they are applied in different
practical situations.

Changing the reserving basis does not affect the true financial position of the company position
but it usually changes the disclosed result. Whilst using a different valuation basis has no direct
effect on the real situation, there may be indirect effects. For example, if an over-cautious basis
caused an insurer to conclude that its solvency was in doubt, then the insurer might choose to
change its investment policy. Thus the true situation would have been indirectly influenced by
the choice of valuation basis.

© IFE: 2019 Examinations The Actuarial Education Company


SP7-14: Reserving bases Page 3

1 Reasons for estimating reserves

1.1 What is a reserving basis?


We define a reserving basis as the methodology and assumptions chosen in a reserving
exercise.

Question

Based on your knowledge of different reserving methodologies from your earlier studies, write
down the common approaches to reserving for reported claims.

Solution

 Case estimates
 Basic and inflation-adjusted chain ladder
 Average cost per claim
 Bornhuetter-Ferguson

1.2 The need to calculate reserves


When carrying out a reserving exercise, an actuary should consider the purpose of the
exercise. A general insurer may need estimates of its liabilities for a variety of purposes,
and the methodology and assumptions may differ for each purpose.

The most common purposes, some of which have already been mentioned in Chapter 13
include:

 to determine liabilities to be shown in the insurer’s published accounts

 if separate accounts have to be prepared for the purpose of supervision of solvency,


to determine the liabilities to be shown in those accounts, eg Solvency II

 to determine the liabilities to be shown in internal management accounts, business


plans and budgets

 to provide an independent opinion on the reasonableness or adequacy of the


reserves booked by the insurer

 to provide information to management as to how areas of the business are


performing, and provide an indication as to the profitability of business currently
being written

 to estimate the claims costs incurred in recent periods as an intermediate step in the
rating process
For example, if you are setting premium rates at the end of 2018, based on the claims
experience in the years 2017 – 2018, then some of the claims will not yet have been
settled, or even reported, and so an estimate will be required of the final claim amount.

The Actuarial Education Company © IFE: 2019 Examinations


Page 4 SP7-14: Reserving bases

 to value the insurer for purchase or sale given that any estimated surplus or deficit
in the booked reserves (as compared to the best estimate of the reserves) will
directly affect the valuation of the company
A best estimate is normally defined as the actuary’s view of the mean or expected value
of the eventual outcome.
 to negotiate a commutation for the buyer or seller (including determining an RITC
premium for Lloyd’s business)

 to transfer a book of business

 to ascertain the tax liabilities of a general insurance provider

 to test the adequacy of case estimates.

The choice of methods and assumptions depends on the purpose for which the reserves
are required and the extent and quality of the data available.

Question

Suggest how the choice of assumptions to be used will differ between the various purposes for
estimating liabilities.

Solution

Published accounts: Prudent side of best estimate to ensure stability and consistency from year to
year. Alternatively, best estimate. It depends on whether the insurer is using IFRS 4, IFRS 17,
Solvency II, UK GAAP, etc.

Supervision of solvency: Best estimate under Solvency II. In some territories outside the EEA
(European Economic Area), prudent assumptions may be used in order to protect policyholders.

Internal management accounts and internal profitability: Probably a best estimate to give a
realistic view of the financial condition of the company.

Independent opinion: Likely to be a best estimate to give a realistic view, however this will
depend on the purpose of the investigation.

Rating process: Probably best estimate but may be slightly cautious if there is a lot of uncertainty,
in order to avoid premiums that are too low.

Purchase or sale, negotiating a commutation, transferring a book of business: Close to best


estimate. The seller may prefer a more optimistic estimate as a starting point for negotiations,
whereas the buyer would prefer a prudent estimate. In practice however, both parties may
recognise the need for some prudence in the estimates.

Tax liabilities: Likely to be a best estimate. Prudent reserves will result in a deferral of profit and
tax, while an optimistic reserve will accelerate tax payments. In practice, the basis will depend on
whether the company has adopted an active tax management policy.

Testing adequacy of reserves: a range of bases may be calculated, in order to examine resilience.

© IFE: 2019 Examinations The Actuarial Education Company


SP7-14: Reserving bases Page 5

2 Uses of different reserving bases


This section summarises the considerations that might apply when deriving reserves for the
different purposes identified above. These were discussed in Section 1.2 above.

2.1 Published accounts


The assumptions to be made in determining the liabilities to be shown in an insurer’s
published accounts should have regard to the legislation and accounting principles
governing the preparation of those accounts in the territory concerned. Matters to be
considered include:

 Whether the accounts are to be prepared on a going concern basis (this means that
the company will continue in operational existence for the foreseeable future).

 Whether the accounts are required to show a true and fair view. (For example, in the
UK, this term was introduced in the 1948 Companies Act, but was not defined. It
depends on the opinion of the auditor). The requirement for accounts to show a true
and fair value is also enshrined in EU law.

 Whether reserves are required to be assessed as best estimates, including some


element of margin, or on some other basis, and precisely how the terms used are to
be interpreted.

 Whether reserves are required to be discounted (required under Solvency II and


IFRS 17), and if explicit risk margins need to be held.
The risk margin or range gives an estimate of the uncertainty surrounding the best
estimate. IFRS 17 will be discussed in Chapter 26 (Accounting methods).

The accounts could also be required to be prepared on a run-off basis. This assumes that the
company will continue to service in-force business but would cease to write any new business.
This contrasts with a break-up basis that assumes the company is wound up and policies with
unexpired periods of cover are cancelled (probably with a proportion of the premium returned to
the policyholder).

2.2 Tax purposes


Reserves used for tax purposes may not be the same as those used in the published
accounts. This depends on the tax regulations in the relevant country. When deriving
reserves for tax purposes, it is therefore very important to take account of the relevant tax
regulations. Tax authorities may penalise an insurer if it is found to have over-reserved and
therefore paid less tax, or they may accept insurers booking reserves above the best
estimate.

Question

Suggest why tax authorities may penalise an insurer when it is found to have over-reserved.

The Actuarial Education Company © IFE: 2019 Examinations


Page 6 SP7-14: Reserving bases

Solution

Over-reserving delays the emergence of profit and hence payment of tax.

2.3 Solvency accounts


In many territories, insurance companies are required to submit separate accounts to the
regulator, primarily so that the authorities can satisfy themselves as to the financial strength of
the insurer.

If separate accounts are required as part of the process of supervision of solvency, the
rules governing the preparation of those separate accounts may or may not be the same as
those that apply to the published accounts.

The rules may, for example, require the accounts to be prepared on a going concern basis
or on a run-off or break-up basis. They may also require some form of risk margin.

For example, Solvency II requires a best estimate reserve which is the mean of all possible
outcomes, less a credit for discounting for expected future investment income, plus a risk
margin (which can be calculated using a capital model or the standard formula).

Under Solvency II, Technical Provisions comprise both claims and premium provisions, the
former relating to events that have already happened as at the reserving date, and the latter
relating to future events. The Technical Provisions are based on all cashflows, including
premiums and claims costs.

Solvency II replaced the former solvency regime, under which the Technical Provisions
comprised the outstanding claims provision and the Unearned Premium Reserve (UPR).

On a Solvency II basis, there is a need to hold a provision for unearned (and, in some cases,
unincepted) liabilities at best estimate. This can involve similar considerations to
recognising the future profit (or potential loss) in the UPR carried out for an AURR
assessment.

Question

We met AURR and UPR briefly in Chapter 1. Explain why an insurer would need to hold an AURR.

Solution

An insurer would not normally recognise profit until business has been earned. In other words, if
profit is expected to emerge from business that has not yet been earned, the AURR would be
zero. However, if an insurer expects to make a loss on business that has not yet been earned, a
(positive) AURR will need to be held.

AURR and UPR will be discussed in more detail later in the course.

Any reserve report produced should make reference to those rules and any guidance to
actuaries or other professionals that may have been issued as to their interpretation.

© IFE: 2019 Examinations The Actuarial Education Company


SP7-14: Reserving bases Page 7

In some contexts (eg Lloyd’s Statements of Actuarial Opinion), the test is one sided. This
can lead to pressure for margins.

A one-sided test checks only whether the reserve is sufficient. A two-sided test also considers if it
is unrealistically high.

2.4 Management accounts


We should discuss the principles to be followed in preparing internal management accounts
and business plans and agree them with the insurer’s management. The aim is likely to be
to produce expected values of the future experience, based on realistic assumptions to
ensure a realistic picture of the entity. Management may also wish to have figures
produced on alternative bases or scenarios, for example by changing key assumptions.

Question

Suggest some key sensitivity checks that management may wish to carry out.

Solution

Management will wish to test sensitivity to:


 changes in volume and mix of business
 changes in claim frequency and claim size
 changes in claims inflation
 changes in expenses and expense inflation.

The managers will want to see a realistic picture of the financial state of the company that is not
distorted by including margins in the assumptions. Internal management accounts could be
prepared for just about any purpose, for example to analyse:
 profitability
 solvency
 claims and exposure
 expenses
 reinsurance performance
 movement analyses (eg number of new policies, lapses etc)
 investment performance
 business volumes and results by source of business.

Management may elect, in many cases, to hold reserves in the published accounts which
are above the best estimate determined by the actuary.

It is important to note that the use of margins, while often helpful, can lead to smoothing of
results. Auditors, in particular, will be alert to the potential smoothing of profits through
changing reserve margins.

The Actuarial Education Company © IFE: 2019 Examinations


Page 8 SP7-14: Reserving bases

The assumptions to be made for the purpose of rating are discussed in Section 3 below.

2.5 Sale or purchase


In a valuation of an insurance company for sale or purchase, one key component is the
valuation of the liabilities on the balance sheet. The purchaser and seller will both need to
form a view on the adequacy of the booked reserves.

The purchaser will want to understand the reserving basis used by the company and in
particular estimate the level of expected surplus or deficit in the booked reserves. Given
that this may have a material bearing on the valuation of the company it is likely that the
purchaser will make a range of estimates on different assumptions, in order to understand
the impact of potentially selecting those alternative assumptions.

As a result of the commercial impact of different levels of reserves on the pricing in a


merger and acquisition (M&A) transaction, it is quite possible for the parties to use a
different basis for the reserving in an M&A context from that used in the published
accounts. Typically, the purchaser will take a more pessimistic or prudent view of the
reserves compared to that of the vendor.

Question

Explain why the purchaser will take a more pessimistic view of the reserves than the vendor.

Solution

The purchaser will be keen for a prudent basis to be used in valuing the liabilities, so that a lower
value can be paid to the seller.

2.6 Commutation
A commutation is the finalisation of an outstanding liability by payment of an agreed amount.

When negotiating a commutation we will have similar considerations to those in a sale and
purchase context. However, the analysis will normally need to be at a much more detailed
level, both in terms of the data analysed and the factors taken into consideration. Extra
considerations will be:

 impact of reinsurance recoverability

 the relative strategic / commercial importance of the commutation between the two
parties

 the actual / perceived financial strength of parties (eg where one of the parties is
insolvent).

2.7 Transfer of liabilities


When an insurer is transferring liabilities either between companies within the same
corporate group, or between itself and another insurer, we will have similar considerations
to those that apply in a sale and purchase context. In addition, we should consider any
particular regulations that apply in the country in question.

© IFE: 2019 Examinations The Actuarial Education Company


SP7-14: Reserving bases Page 9

For example, when transferring liabilities between companies within the same group, the
regulator may be concerned that such a transaction is carried out at ‘arm’s length’.

The Actuarial Education Company © IFE: 2019 Examinations


Page 10 SP7-14: Reserving bases

3 Reserving basis vs rating basis


Rating assumptions are usually realistic and so may differ from reserving assumptions depending
on the purpose of the reserving exercise.

For example, if the reserves are being set in order to demonstrate solvency then the reserving
basis is likely to be much more prudent than the rating basis.

For the purpose of rating it will usually be appropriate to begin by assessing the likely
outcome of recent blocks of business. We may make realistic assumptions without
precautionary margins and after adjusting for elements of the experience thought to be
abnormally unfavourable or abnormally favourable (for example, the impact of an unusually
large claim).

For example, a catastrophe may have occurred of a type that is only expected once every ten
years. The claims experience may be scaled down appropriately. Any adjustments made would
be subjective and so it would be preferable to use a block of business taken over a long enough
time period to avoid too many random variations.

Question

Explain what the main problem is with analysing the business written over, say, the last ten years
to try and reduce the random fluctuations.

Solution

The data may not be available and more importantly the older data may not be relevant due to
inflation, changes in cover, changes in risk etc.

We then make further assumptions to project the experience forward to the period to which
the new premium rates and the corresponding claims will apply (ie claims inflation, changes
in exposure etc). The ultimate assumptions are therefore those we consider appropriate for
rating in respect of future periods that extend beyond the periods that are relevant to
assessing the liabilities in respect of existing business.

For this reason alone, the ultimate assumptions may be different to those that might be
used for reserving purposes.

Note: the bases used in reserving or rating may differ from best estimate due to the different
purposes for which the calculations are done. For example, differing levels of prudence,
differing risk appetites and differing competitive or other comparative pressures may all
impact the bases used.

When we have calculated the rates on the basis of expected values, we will consider what
profit loading to add against the background of the uncertainties in the business and the
competitive position. The level of regulatory capital required to support the business may
also be considered.

© IFE: 2019 Examinations The Actuarial Education Company


SP7-14: Reserving bases Page 11

4 Choice of reserving method


The methodologies used for reserving are likely to vary according to:

 the class of business, in particular the length of the tail of the run-off of the liabilities
and the exposure period of the insurance / reinsurance contracts

 the types of claim that have been incurred or may be expected to occur

 the extent and quality of the available data

 the age of the business or cohort of claims, and historical claim development
information available

 key factors that determine the development of the outstanding claims and claim
payments

 historical trends and patterns

 the purpose of the reserving exercise.

The claims experience would be analysed in relatively homogeneous groups in order to make
credible estimates of outstanding claim amounts. For example, the reserves for short and
long-tail claims may be determined separately, or the reserves for a motor portfolio may be split
between property damage and bodily injury.

In this chapter, we only consider the effect of the purpose of the reserving exercise on the
reserving basis chosen, rather than the impact of the above factors on the method chosen.

Stochastic methods may be more appropriate where we are providing a range of estimates,
but they can also be used for providing best estimates. This is covered in more detail in a
later chapter.

If the purpose focuses on a particular area of business, for example the commutation of a
reinsurance treaty, we may choose a method on the basis of the data available for that
specific area.

Where items such as a payment pattern are required, or where reserves need to be
discounted, this may influence the method chosen for the best estimate.

Ideally, we should use a number of methods to derive the estimates required, and some may
produce significantly different results. The actuary should be aware of the limitations and
implicit assumptions of each method during the analysis and consider the appropriateness
of each for the purpose in question.

In addition very rough checks can be made on the results. For example, very rough checks on the
estimates for outstanding claims can be made by taking the ratio of outstanding claims to the
amounts of premiums or incurred claims, and comparing these ratios with equivalent figures from
previous years or for similar companies.

These sorts of checks are called ‘diagnostics’. We discuss these further later in the course.

In most reserving situations, it is appropriate to monitor the emerging experience against


the expected outcome. This will require us to select a reserving method where we can
derive the expected outcome (eg over the next year). In these cases, methods which
separately model the IBNER and pure IBNR components of the reserves may have
advantages over methods that model these components in the aggregate.

The Actuarial Education Company © IFE: 2019 Examinations


Page 12 SP7-14: Reserving bases

Question

Suggest what extra information can be gained from modelling IBNER and pure IBNR separately.

Solution

By modelling IBNER and IBNR separately, we can determine whether a change in our required
outstanding claims reserve is due to an increase in the number of claims reported, or due to
prudence / optimism in our original case estimate assumptions.

© IFE: 2019 Examinations The Actuarial Education Company


SP7-14: Reserving bases Page 13

5 Allowance for future inflation


In many situations, the allowance for future inflation will be implicit within the methodology,
for example where a method implicitly assumes that future inflation will be similar to the
average inflation experienced in the past.

However, we may consider the allowance for future inflation an important assumption in a
particular context, or we may need to investigate the impact of alternative future inflation
assumptions.

For example, in periods of economic instability, when future inflation may be uncertain, an
insurer may wish to investigate the likely impact of different economic scenarios on future claims
costs. This is particularly the case when an economic downturn may increase the incidence of
fraudulent claims, and therefore exacerbate the extra cost to the insurer.

This may influence the method selected because explicit allowance for inflation within the
estimates can be more easily incorporated in some methods than in others. For example,
explicit allowance for inflation can usually be more easily incorporated in average cost per
claim methods.

When making an assumption about inflation, we should carefully consider the allowance, if
any, for inflation in excess of general economic inflation, such as consumer price and wage
inflation. Such superimposed inflation could be a result of future step changes in average
claim size arising from a change in the law or be reflective of the historical level of inflation
seen in the claims.

For example, court award inflation in recent years has been well in excess of price inflation.

Relevant indices might be available from government or industry sources. Alternatively the
insurer may be able to prepare indices from its own claims experience. These may be more up to
date and relevant than the other figures, but could be more uncertain if based on scanty or poor
quality data.

The description of the reserving basis should clearly state the ‘force’ or ‘direction’ of
inflation. The method being used usually defines this. For example, the force of inflation
could be by origin year or calendar year. We should also clearly state what the inflation
represents, for example, the increase in the average size of claims or the increase in the
average burning cost per unit of exposure. In particular, the latter may be due to an
increase in either claim frequency or claim size or both.

The Actuarial Education Company © IFE: 2019 Examinations


Page 14 SP7-14: Reserving bases

6 Discounting
Considerable delays can arise between receipt of the premium by the insurer and ultimate
payment of claims. Hence there is an opportunity to earn investment income. This can be
allowed for in the calculation of reserves by discounting the expected future claim payments to
the calculation date.

The concept of discounting claims reserves is relatively new in general insurance. Traditionally,
claims reserves have been the actual nominal amounts estimated, and not the discounted value.

The practice of not discounting reserves seems bizarre to most actuarial people new to general
insurance (particularly life assurance actuaries), although there are arguments for continuing the
practice. As with all debatable issues, there are valid arguments for both views. The balance of
the argument will depend on the purpose of the reserving exercise.

The extent to which we should use discounting in respect of investment income in


estimating technical reserves for published accounts or for solvency supervision will
depend on whether the relevant legislation or the prevailing accounting rules require,
prohibit or restrict discounting for those purposes. Under Solvency II and IFRS 17, reserves
are required to be discounted.

Discounting may be more appropriate where we wish to have a realistic picture of the
economic results or financial condition of the insurer.

Discounting gives a more realistic view of the profits and makes comparison between classes
more valid. Not discounting makes long-tail classes look relatively less profitable.

Question

Explain why ‘not discounting makes long-tail classes look relatively less profitable’.

Solution

Discounting would have a larger relative impact on distant claim payments. By not discounting,
therefore, there is overstatement of the reserves required for a long-tailed class. Hence profits
are relatively lower (in the year of writing, at least), as we build up more ‘fat’ in the technical
reserves for those classes.

Not discounting has the effect of adding implicit margins to the reserves. If discounting is used
then the loss of the implicit margins may result in explicit ones being set up. These will make it
easier to assess, and adjust, the degree of prudence in the reserves.

Note that the impact of not discounting may vary from time to time, depending on the
investment outlook at the time. Similarly, the extent to which we need to allow for contingencies
may vary from time to time. However, there’s no reason why the two should be correlated.
Therefore, it would usually be much better to make explicit allowance for contingent events and
to discount claims reserves at an appropriate rate.

© IFE: 2019 Examinations The Actuarial Education Company


SP7-14: Reserving bases Page 15

If we are providing discounted estimates, it is important to consider consistency between


the economic inflation assumptions and the discount rate as well as any assumptions used
for yields and asset returns in a corresponding solvency assessment.

The difference between the two assumptions will be more important than the absolute value of
each assumption.

Accounting rules for published accounts normally prohibit implicit discounting, where the
booked reserve is lower than the undiscounted best estimate, with the justification that it
contains a margin above the discounted best estimate.

Regulation may specify the assumption for the rate of discount to be used for the purpose
in question. It could also be specified in a contract, for example the commutation clause in
a reinsurance treaty.

If the rate of discount to be used is not specified, we should determine this by reference to a
number of aspects, which could include the currency of the liabilities, the mean term of the
liabilities, the risk-free yield curve at the valuation date and the nature of the liabilities and
assets.

We should determine whether the discount rate is gross or net of tax and disclose this as
part of the reserving basis.

Sometimes these issues are put forward as disadvantages of discounting, ie it is difficult to


determine the discount rate that should be used. However, this problem should not be
insurmountable. The important thing is to realise that assets not available for investment must
be taken into account when determining the rate.

The Actuarial Education Company © IFE: 2019 Examinations


Page 16 SP7-14: Reserving bases

7 Communication of the reserving basis


We should communicate the entire reserving basis in a way that enables the audience to
form their own view on the rationale for the selected basis and the reasonableness of the
assumptions made.

The description of the basis should take into account the degree of uncertainty surrounding
the estimates and we should communicate the impact of changing key assumptions, for
example by giving sensitivities.

The objectives in communicating uncertainty should include the following:

 ensuring stakeholders understand the level of uncertainty

 being consistent with the vocabulary used by other actuaries and other
professionals and explaining any terms where there could be scope for
mis-interpretation or which may not be understood by the audience to whom the
report is directed

 focussing on the most significant issues, given the purpose of the exercise

 explaining what has been allowed for in the best estimate, and what has not

 emphasising the unusual issues

 commenting in the context of the purpose and scope of the exercise

 avoiding misunderstandings.

The candidate should realise that this list is not exhaustive.

It is usually advisable to accompany the communication of the reserving basis with a clear
explanation of which elements of the reserves are included, for example, if they include
allocated and/or unallocated loss adjustment expenses, if they are net or gross of
reinsurance and/or net or gross of salvage / subrogation and/or net or gross of reinsurance
bad debt etc.

© IFE: 2019 Examinations The Actuarial Education Company


SP7-14: Reserving bases Page 17

8 Glossary items
Having studied this chapter you should now ensure that you are able to explain the following
Glossary items:
 Break-up basis
 Going-concern basis
 Run-off basis
 Wind-up basis.

The Actuarial Education Company © IFE: 2019 Examinations


Page 18 SP7-14: Reserving bases

The chapter summary starts on the next page so that you can
keep all the chapter summaries together for revision purposes.

© IFE: 2019 Examinations The Actuarial Education Company


SP7-14: Reserving bases Page 19

Chapter 14 Summary
There are a number of reasons for assessing a general insurer’s liabilities, including:
 to estimate the insurer’s reserves for the published accounts or supervisory solvency
or internal management purposes
 to check reasonableness of reserves
 to provide information to management on business performance by area
 to estimate claims costs for premium rating
 to value an insurer for sale or purchase
 to negotiate a commutation for the buyer or seller
 to transfer a book of business
 to ascertain tax liabilities
 to test the adequacy of case estimates.

The actuary should make clear what scenarios have been included / excluded in deriving the
best estimate.

Reserving methodologies vary according to:


 class of business, in particular the timing of the run-off of the liabilities
 the types of claim that have been incurred or may be expected to occur
 the extent and quality of the available data
 the age of the business
 the key factors that determine development of claims
 historical trends and patterns
 the purpose of the reserving exercise.

Different reserving methods have different strengths and weaknesses. The key aims are to
choose appropriate methods for the situation, ideally using a number of methods and to
monitor the emerging experience against the expected outcome.

The Actuarial Education Company © IFE: 2019 Examinations


Page 20 SP7-14: Reserving bases

Chapter 14 Summary continued


Key considerations when setting a basis will depend on the purpose of the investigation.
Namely:

Purpose of exercise Considerations


Published accounts Whether a going concern basis, true and fair
view, best estimate or discounted basis is
required.
Tax calculation Tax regulation, particularly penalties for
over-reserving.
Demonstrate statutory solvency Regulation and any professional guidance to aid
actuaries and other professionals.
Management accounts Agree basis with management, often a best
estimate basis is used in conjunction with
sensitivity testing.
Sale or purchase A starting point for negotiations is the liabilities
shown on the balance sheet. A purchaser will
want a more prudent view compared to the
vendor.
Commutation Similar to sale and purchase, but also consider
any reinsurance recoveries, the importance of
the deal and the financial strength of both
parties.
Transfer of liabilities Similar to sale and purchase, but also consider
local regulation.
Rating Usually realistic, therefore often different to
reserving assumptions. Assumptions may also
relate to different periods of exposure.

Each element of the reserves should include some allowance for future escalation of costs.
The inflation assumptions will be determined allowing for recent movements in the relevant
indices and the current economic assessments on inflationary trends.

Discounting makes allowance for investment income.

When deciding whether to discount reserves, we should consider whether discounting is


allowed by regulation. If allowed, regulation may specify a discount rate.

If not specified then the discount rate needs to be determined based on:
 currency of liabilities
 nature of liabilities and assets
 risk-free yield curve at the valuation date.

© IFE: 2019 Examinations The Actuarial Education Company


SP7-14: Reserving bases Page 21

Chapter 14 Practice Questions


14.1 List the factors that will influence the choice of valuation method and assumptions when
determining the value of an insurer’s liabilities.

14.2 (i) State the most common purposes for which a general insurance company would need to
estimate its liabilities.

(ii) For each purpose in (i) suggest, with reasons, the valuation basis that would most likely be
used.

14.3 Explain what the appropriate reserving basis will be for the following two calculations:
 a calculation to provide figures for the insurer’s published accounts
 a calculation to provide figures for the insurer’s internal management accounts.

14.4 List reasons why insurers need to calculate reserves.

14.5 (i) Define the term ‘best estimate’ basis. [1]


Exam style
(ii) Explain why two actuaries may have a different view of the best estimate basis when
valuing the same liabilities. [4]
[Total 5]

14.6 You are responsible for calculating the claims reserves for a general insurance company on a ‘best
Exam style
estimate’ basis. Explain how you would choose the rate for discounting outstanding claim
payments. [3]

The Actuarial Education Company © IFE: 2019 Examinations


Page 22 SP7-14: Reserving bases

The solutions start on the next page so that you can


separate the questions and solutions.

© IFE: 2019 Examinations The Actuarial Education Company


SP7-14: Reserving bases Page 23

Chapter 14 Solutions
14.1 Factors that will influence the choice of valuation method and assumptions include:
 the purpose of the valuation, eg statutory or internal accounts
 the class of business, eg more margins for long-tail, liability business
 the size of the solvency margin, ie companies with very large margins may be relaxed
about using strong bases
 quality, amount and stability of data.

14.2 (i) Purposes for which to estimate liabilities


 to determine the liabilities for the insurer’s published accounts
 to determine the liabilities for accounts that are used for the supervision of solvency
 to determine the liabilities for internal management accounts
 to estimate the cost of claims incurred in recent periods as an intermediate step in the
rating process
 to value the insurer for purchase or sale
 to assess the accuracy of the company’s case estimate and/or IBNR claims reserve
estimation in previous year-end exercises
 to provide information to management as to how areas of the business are performing
 to compare best estimates against held reserves
 to calculate a range of results
 to transform an underwriting year into an accounting year
 to calculate movements in reserves and analyse reasons for these

(ii) Bases likely to be used, with reasons


 Published accounts – prudent basis, to ensure stability and consistency year on year.
Alternatively, best estimate. It depends on whether the insurer is using IFRS 4, IFRS 17,
Solvency II, UK GAAP, etc.
 Solvency supervision accounts – best estimate under Solvency II. In some territories
outside the EEA (European Economic Area), a prudent basis may be used, to protect
policyholders.
 Internal management accounts – best estimate basis, to give a realistic view of the
financial condition of the company.
 Rating process – best estimate basis. A prudent basis could lead to the premiums being
set too high while an optimistic basis could lead to the premiums being set too low. We
don’t want implicit margins in the premium rates.

The Actuarial Education Company © IFE: 2019 Examinations


Page 24 SP7-14: Reserving bases

 Purchase or sale – best estimate basis is likely to be the starting point for negotiations.
However, the seller might want to use an optimistic basis and the buyer might want a
prudent basis. The final basis will depend on the relative bargaining power of the two
parties.
 Accuracy of previous reserve estimates  often no estimation is needed since we are
comparing actual paid claims with those expected by previous estimates. If we are
analysing incurred claims, then it makes sense to use the same basis as that used in the
previous reserve estimation exercise in order that a meaningful comparison can be made.
 Information for management – best estimate basis, to give a realistic view. We will also
be interested in the sensitivity of the results, so we will use several different bases, for
example worst case scenarios.

14.3 The assumptions for the calculations for the published accounts will need to take account of the
legislation and accounting principles governing the preparation of those accounts in the territory
concerned.

Consideration needs to be given as to:


 whether the accounts need to be prepared on a going concern basis
 whether the accounts are required to show a true and fair view
 the level of prudence required in the estimates …
… for example, it is likely that the accounts need to be produced on a fair value approach,
in which case a best estimate basis may be required
 whether reserves are required / allowed to be discounted and if explicit risk margins need
to be held.

For the internal management accounts, discussions should be held with the management as to
the required strength of basis.

It is likely that a best estimate basis will be used, in order to give a realistic view of the business.

Management are likely to want sensitivity testing to be carried out in order to understand the
impact on the results of changes to key assumptions, eg claim frequency and claims inflation.

14.4 The most common purposes include:


 to determine the liabilities to be shown in the insurer’s published accounts
 to prepare separate accounts for the purpose of supervision of solvency and to determine
the liabilities to be shown in those accounts, if necessary
 to determine the liabilities that we show in the internal management accounts
 to value an insurer for purchase or sale
 to assess the adequacy of the company’s case estimate and/or IBNR claims reserve
estimation in previous year-end exercises
 to provide information to management on how areas of the business are performing, and
provide an indication on the profitability of business currently being written

© IFE: 2019 Examinations The Actuarial Education Company


SP7-14: Reserving bases Page 25

 to compare best estimates against held reserves


 to calculate ranges of results
 to transform an underwriting year into an accounting year
 to calculate movements in reserves and analyse reasons for these
 to calculate reserves in order to estimate the cost of claims incurred as an intermediate
step in the premium rating process.

14.5 (i) Best estimate basis

The best estimate is normally defined as the actuary’s view of the mean or expected value of the
eventual outcome. [1]

(ii) Why two actuaries may have different views

It is difficult to ascertain the mean of the outcomes with a high degree of certainty and this can
lead to differences. [1]

The best estimate calculation will be based on an actuary’s subjective views. [½]

The best estimate calculation will reflect the mean of the possible outcomes considered and one
actuary may include more possible outcomes than another. [½]

Each actuary will need to make it clear what he/she has allowed for in their derivation of best
estimate. [½]

Different views could also be the result of:


 the use of different data [½]
 the use of different models / assumptions [½]
 errors in the model / data [½]
 the calculations being carried out at different points in time. [½]
[Maximum 4]

14.6 To select the discount rate, you would:


 consider the assets backing the technical reserves for the class [½]
 estimate the expected annual rate of return from these assets … [½]
 ... over the term of the liabilities ... [½]
 ... on a basis that is consistent with the assumptions for inflation [½]
 consider allowance for taxation [½]
 remove prudence from the estimated reserve, assuming that the rest of the basis makes
suitable explicit allowance for contingencies (eg possibility of latent claims). [½]
[Total 3]

The Actuarial Education Company © IFE: 2019 Examinations


All study material produced by ActEd is copyright and is sold
for the exclusive use of the purchaser. The copyright is
owned by Institute and Faculty Education Limited, a
subsidiary of the Institute and Faculty of Actuaries.

Unless prior authority is granted by ActEd, you may not hire


out, lend, give out, sell, store or transmit electronically or
photocopy any part of the study material.

You must take care of your study material to ensure that it


is not used or copied by anybody else.

Legal action will be taken if these terms are infringed. In


addition, we may seek to take disciplinary action through
the profession or through your employer.

These conditions remain in force after you have finished


using the course.

The Actuarial Education Company © IFE: 2019 Examinations


SP7-15: Triangulation methods Page 1

Triangulation methods
Syllabus objectives
3.1 With regard to reserving work using triangulations:
 Analyse the range of general issues that can affect reserving work using
triangulations.
 Identify how to deal with these general issues in reserving work.
 Evaluate the main triangulation methods in use – namely the chain ladder
method, the Bornhuetter-Ferguson method and the average cost per claim
method.

The Actuarial Education Company © IFE: 2019 Examinations


Page 2 SP7-15: Triangulation methods

0 Introduction
This chapter covers the main triangulation methods, and the issues to consider when using
these.

Each method is based on certain assumptions about the underlying data, and there are
issues to consider when using each method. In addition, there are general issues to
consider when using any data triangulations and these are discussed first.

This is a very long chapter, and you may well wish to split your study of it into a number of
sections. However, you should be able to remember something about the run-off methods from
your study of earlier subjects.

This chapter is one in a series of chapters on the subject of reserving. This idea is developed
further in the other related chapters.

Section 1 develops some of the general ideas involved in using a run-off triangle to estimate a
reserve.

Section 2 develops the basic chain ladder method in more detail. Section 3 develops the expected
loss ratio method, Section 4 discusses the Bornhuetter-Ferguson method, and Section 5 describes
the average cost per claim method.

Section 6 considers the results of the different methods, whilst Section 7 looks at reserves for
unexpired risks. Section 8 discusses the different types of recoveries.

While you will probably be familiar with the mechanics of some of these methods from your
earlier studies, the approach here is to give more of an overview of the main issues that may arise
when these methods are used.

© IFE: 2019 Examinations The Actuarial Education Company


SP7-15: Triangulation methods Page 3

1 General issues
This section describes the main issues arising when trying to estimate an outstanding claims
reserve for a portfolio of policies.

1.1 Understanding the data


At best, triangulation methods are an imperfect representation of the overall claims
process. In order to assess which methods to use, it is important to understand what data
is available, including any limitations, and the strengths and weaknesses of the different
methods. This will assist in selecting, and may in fact determine, the appropriate grouping
of data.

1.2 Data grouping


When we want to project future claims, we need to make decisions about how best to group the
existing claims data.

The simplest claims analysis might be to compare total claims paid against exposure in a
given period (for example, calendar year), without reference to the dates that the claims
occurred or were notified / reported to the insurer. However, such an analysis includes
payments on claims that originated in earlier periods and excludes the cost of claims that
have occurred in the analysis period, but have not been settled. In such an analysis, we are
not comparing claims and exposure on a consistent basis.

We are looking here at working on a ‘total claims paid’ basis.

For example, suppose we consider the total amount of claims paid in calendar year 2018,
compared with the total number of policies written within calendar year 2018. We are not being
consistent, since a payment in 2018 may arise from a policy that was sold much earlier, for
example in 2015. Also, payments may be made in subsequent years for policies written in 2018.

If, instead, we compare reported claims against exposure in a given period, this may also be
inconsistent, because the amounts paid are likely to differ from the corresponding
estimates in the reserves at the beginning of the period. The problem is that part of the
reported claims figure is really a correction to earlier estimates rather than a claim amount
that has actually been incurred during the period in question. This component is
sometimes referred to as ‘incurred but not enough reported’ (IBNER).

Here we are working on a ‘total claims reported’ basis.

For example, suppose we consider the total amount of claims reported in calendar year 2018,
compared with the total number of policy years within calendar year 2018. In fact what we finally
pay out on these policies may be at variance with the figure for claims reported. For example, the
reported claim on a bodily injury case could be £15,000, but this may be corrected to say £20,000
if further information comes to light regarding the severity of the injury.

The Actuarial Education Company © IFE: 2019 Examinations


Page 4 SP7-15: Triangulation methods

Note that you need to be careful over the definition of ‘incurred claims’ in this context. For
example, incurred claims might mean:
 paid claims plus estimates for outstanding reported claims (eg case estimates)
 paid claims plus estimates for all outstanding claims (including an appropriate loading for
IBNR).

Both of these approaches are sometimes described as using data for ‘incurred claims’. The first
approach is more common, ie if given an ‘incurred claims’ data table without any guidance as to
the definition used, it is probably more likely to have been prepared without any IBNR allowance.
Note that if the second approach is taken the method is merely projecting the corrections to
earlier estimates, whereas the first approach largely projects the IBNR.

In an exam, you may need to define exactly what you mean by incurred claims at the start of your
answer.

Question

Explain for each of the above definitions of incurred claims why the run-off pattern usually shows
further development after the first year (ie why the amounts of incurred claims are not just the
same for each development year).

Solution

The first definition (paid to date + estimate for outstanding reported) shows further development
for two reasons:
(i) This definition makes no allowance for IBNR claims. As the triangle develops and these
become known they will lead to additional claim payments, ie we expect some positive
development (increase in the incurred claim total) by virtue of claims currently IBNR.
(ii) Our estimates of payments for outstanding reported claims will probably prove to be
wrong. If we have under-reserved then the amounts will increase as we travel along the
rows (ie positive development). The opposite tends to occur if we have over-reserved.

The second definition (paid to date + estimate for outstanding, including IBNR) shows further
development for the second reason only.

Claim cohorts
To address this problem of inconsistency, we should separate the claims data into the
different cohorts or periods of origin (for example, underwriting year). We can then analyse
the experience of each claim cohort separately, without any distortion arising from other
periods’ experience.

© IFE: 2019 Examinations The Actuarial Education Company


SP7-15: Triangulation methods Page 5

There are a number of different ways in which we can group claims into cohorts of common
origin for the purpose of following and projecting their subsequent run-off. The choice of
grouping may lead to different results from the analysis. Some of the main ways are
discussed below:

 accident year

 underwriting year

 reporting year.

Year of underwriting may also be known as policy year, or year of account.

For ease of explanation in the discussion below we use years as the cohort, since it is the
period frequently used. In practice, any period can be used (for example, a quarter or
month may be used in some classes of business, particularly for accident periods in direct
insurance classes such as motor).

Question

Explain why it will be necessary for some classes to use periods shorter than a year.

Solution

This is so that seasonal effects can be observed and dealt with appropriately.

The Actuarial Education Company © IFE: 2019 Examinations


Page 6 SP7-15: Triangulation methods

1.3 Run-off triangles


Before we consider the different ways of grouping data, let us remind ourselves about the basics
of run-off triangle data. You may remember some of this from your earlier studies.

Let’s suppose that the claim payment amounts are tabulated as in the table below:

settlement delay in years (development year)


0 1 2 3 4
2009 c09,0  c09,1  c09,2  c09,3  c09,4 
2010 c10,0  c10,1  c10,2  c10,3  c10,4 
2011 c11,0  c11,1  c11,2  c11,3  c11,4 
2012 c12,0  c12,1  c12,2  c12,3  c12,4 
year of 2013 c13,0  c13,1  c13,2  c13,3  c13,4 
accident
2014 c14,0  c14,1  c14,2  c14,3  c14,4 
(origin year)
2015 c15,0  c15,1  c15,2  c15,3  c15,4 ?

2016 c16,0  c16,1  c16,2  c16,3 ? c16,4 ?

2017 c17,0  c17,1  c17,2 ? c17,3 ? c17,4 ?

2018 c18,0  c18,1 ? c18,2 ? c18,3 ? c18,4 ?

In the particular class tabulated here, we have assumed that no claims take longer than five years
from the date of accident to settle. If any claims take longer, the table will be extended by adding
more columns as necessary.

For each cell with a , the amount paid is known. For each cell with a ?, the amount paid is
unknown. The ? cells are the amounts that will be paid in the future (assuming that the table is
constructed at 31 December 2018).

Question

Explain what each of the following cell or cells represents:


(a) c11,3
(b) c12,0 + c12,1 + c12,2 + c12,3 + c12,4
(c) c17,0 + c16,1 + c15,2 + c14,3 + c13,4
(d) c17,2
(e) the total of the ? cells.

© IFE: 2019 Examinations The Actuarial Education Company


SP7-15: Triangulation methods Page 7

Solution

(a) The claim amount paid in 2014 for accidents in 2011.


(b) The total of claims from accidents in 2012.
(c) The total of claim payments made in 2017.
(d) The claim amount that will be paid in 2019 for 2017 accidents.
(e) The total of the ? cells is the total amount that we will pay in the future for accidents that
have happened up to 31 December 2018.

Note that claim events that have occurred but not been reported by the end of 2018 (ie IBNR) will
be included within one of the ? cells in the row corresponding to the year when the accident
happened.

In the above table the c’s are incremental amounts. If the claims are tabulated as cumulative
amounts then c15,3 becomes the total claim amount paid in and before 2018 for accidents in 2015.

Using a parallelogram or a trapezium


One of the problems with a run-off triangle is that there is very little data at the tail (ie in the final
column). For example, if a run-off triangle is used in the previous example then the claims data
from origin years 2009–2013 is ignored. The single item c14,4 is used to derive estimates for c15,4,
c16,4, c17,4, c18,4. Using a run-off parallelogram including claims data from 2012 and 2013 (or even
2009 to 2011 as well) can reduce this over-reliance on a single piece of data. If only complete
origin years are used then the parallelogram becomes a trapezium.

If we had done this, the final development ratio r3:4 would have been calculated as:

C C C
r3:4  12,4 13,4 14,4
C12,3  C13,3  C14,3

Although there is a clear advantage of greater statistical reliability, this must be weighed against
the potential irrelevance of the older data.

Accident year
Using this cohort, we group claims according to the year (or other period / cohort) in which
the claim event or ‘accident’ occurred.

IBNR claims, recoveries and reopened claims will be allocated to the original accident year
cohort in which they occurred. Projection of the future development of claims (reported or
paid) in this form should therefore allow automatically for all IBNR claims, recoveries and
reopenings belonging to the cohort.

The Actuarial Education Company © IFE: 2019 Examinations


Page 8 SP7-15: Triangulation methods

The main advantage of this approach is that all claims stem from the same exposure cohort.
The claims will therefore have usually been subject to the same risk environment, although
they may have arisen from policies written under different rating and policy terms. We can
relate variations between accident cohorts to the influences operating at that time, for
example, uplift in business volume or a change in legislation.

For example, an accident occurring early in calendar year 2018 might occur on a policy written in
February 2017. An accident occurring in December 2018 might occur on a policy just written
during that month. If there has been a change over this period in the terms under which policies
are written (for example, a tightening of underwriting), then claims occurring in a particular
calendar year will come from policies with different risk environments.

One of the advantages of the accident year approach is that it can be aligned with the accounting
year.

For accounting purposes, we can compare the losses emerging with the actual charges
made to the operations of that period (that is, the accident year).

In other words, the easiest approach here is to compare the actual experience for the year with
what was booked according to the accounts. If we do this, both calculations will be on the same
basis.

The main disadvantage is that the full number or amount of claims in the cohort is not
known until the last claim is reported. Also for some classes of business (for example,
certain classes in the London Market) the date of loss is not always known and hence the
accident cohort approach may not be practical.

For example, it may not be possible to know exactly when a claim event occurred for:
 a subsidence claim on a household policy
 an asbestos-related claim on employers’ liability.

In this case rules would need to be established to treat claims consistently.

For example, asbestos claims may be allocated evenly over the period of exposure. So if an
employee was exposed to asbestos in the workplace for a period of twenty years, then one
twentieth of the claim amount may be allocated to each origin year.

The choice of allocation method will have implications for:


 how claim costs should be split between different insurers, ie where the insurer has
changed over the period of exposure
 reinsurance recoveries  indeed, an insurer will often try to select an allocation method
which maximises its reinsurance recoveries.

© IFE: 2019 Examinations The Actuarial Education Company


SP7-15: Triangulation methods Page 9

Question

You have a run-off triangle of paid claims split by accident year. Suggest possible reasons for:
(i) a row of figures that is unusually high
(ii) a column of figures that is unusually high
(iii) a diagonal of figures that is unusually low.

Solution

(i) This represents unusually heavy claims in a certain accident year. Possible reasons could
be heavy flooding or an unusually bad winter resulting in a large number of claims.
(ii) A column represents the payments made a certain number of years after the year of the
accident. A high figure may indicate that we settle many claims in the early years
(property damage claims) and then there is a fall-off in payments before liability claims
are paid.
(iii) A low diagonal of figures represents a fall-off in payments being made by the insurer in a
given calendar year. This could be due to implementation of a new system, loss of staff or
a postal strike.

The exposure measure here is often the earned premium, which aligns naturally with
accident years.

Reporting year
Using this cohort, we group claims according to the year (or other period / cohort) in which
they are reported to the insurer or reinsurer, irrespective of the original period of the claim
event or when the insurance policy incepted.

development year
0 1 2 3 4
2014     
2015     ?
year claim
2016    ? ?
reported
2017   ? ? ?
2018  ? ? ? ?

An apparent advantage of this is that, by definition, no further claims will be added to the
cohort after the end of the original reporting period. After the end of the cohort period,
there is a fixed group of claims to be tracked during the run-off. This is unlike the accident
year cohort where the development (at least in the early stages) is a combination of
movement in previously reported claims and notification of new claims. Thus, the reporting
year cohort can help us to monitor the development of notified claims to assess the delay
before reliable estimates can be observed for claims once they are notified to the insurer.

The Actuarial Education Company © IFE: 2019 Examinations


Page 10 SP7-15: Triangulation methods

The major disadvantage is that projection methods based on this cohort will not allow for
the IBNR. A separate allowance will therefore be needed for IBNR claims.

A further disadvantage is that the claims will have come from several different exposure
periods, each of which may have differed in respect of volumes of business, cover applying
and claim settlement patterns.

Another disadvantage is that these methods will thus not pick up changes in exposure or
risk profile.

For example, claims will have come from periods with different claims environments and will
therefore be subject to different weather conditions, economic conditions etc.

Now the whole square will include all claim payments in respect of claims reported by the end
of 2018. Claims reported after 2018 will never be included in this table (unless it is extended
downwards). Therefore, the ? entries represent outstanding reported claims as at
31 December 2018.

The reserve for direct claims handling expenses will be included if the data in the body of the
triangle includes direct claims handling expenses.

Likewise care must be taken with reopened claims under this approach as to which year
they are ascribed to.

If a claim is reopened due to more information becoming available, we need to consider carefully
whether the reporting date is the original date, or the date on which we receive the additional
information.

There is no natural exposure base that would correspond to the definition of risk under the
claims being developed, but average premium and current year’s premium may be proxies.

Underwriting year (also known as the ‘year of account’)


With this approach, we group claims according to the year (or other period / cohort) in
which the policy covering that claim incepted, irrespective of when the claim occurred.
Thus, using this approach, it is quite possible to group claims arising in two or more
consecutive calendar years in the same cohort.

For example, a policy written on 1 July 2018 could have a claim occurring in either calendar year
2018 or calendar year 2019.

Question

We have seen that reserves produced using a ‘reporting year’ run-off triangle will not include
IBNR reserves. Explain what reserves are produced by a run-off triangle which uses ‘year
premium written’ (or ’underwriting year’) as the claim cohort.

© IFE: 2019 Examinations The Actuarial Education Company


SP7-15: Triangulation methods Page 11

Solution

The fully developed square will include the claim payments in respect of all premiums written up
to the end of the latest year (eg up to and including 31 December 2018 in the examples above).
Therefore, the reserve for unexpired risks will also be included in the future development, as well
as the reserve for outstanding claims. The answer is:
 outstanding claims including IBNR, IBNER and reserve for reopened claims
 unexpired risk reserve
 claim payment expenses if the plotted claim data includes claim expenses.

In many reinsurance and other London Market classes (for example, at Lloyd’s), this is the
most commonly used approach.

An advantage of this approach is that we can follow the total outcome of all policies written
in each year. Similarly, we can follow claims that arise from a particular group of policies
that are subject to the same set of premium rates and use the results to test the adequacy of
the premiums.

A further advantage is that terms, rates and conditions are often more stable by
underwriting year.

The disadvantage is that it will take more than one year before all the claims under that
cohort have occurred (for example, if all policies are of a maximum period of twelve months,
it may be up to two years from the beginning of the underwriting year before all claims have
occurred). Reporting delays, including IBNER, will extend this further.

Consider annual policies written on 31 December 2018 in the 2018 underwriting year. Cover is
provided until 31 December 2019 (ie, two years after the beginning of the underwriting year). In
fact, it could be 2020 or later before all claims are reported.

Claims in a particular cohort will also have occurred over a wider risk period than a single
accident year cohort, with a consequent lengthening of the characteristic run-off pattern.
The claims will generally relate to at least two accident years. Hence, in the early stages of
development after the end of the underwriting year itself, the observed pattern of claims
emergence relates to:

 previously notified claims

 notification of previous IBNR claims, and finally

 the emergence of new claims for new periods of exposure.

Projection on an underwriting year basis will ensure that IBNR, recoveries and reopened
claims are automatically included, provided that the emergence of newly reported claims
and reopened claims are all allocated to the correct underwriting year period.

Convergence of methods
It is important to note that each of these methods will produce an estimate, and that as we
approach the extremes of the development table, the total numbers from different cohorts
would be expected to converge towards the same figures for reserves.

The Actuarial Education Company © IFE: 2019 Examinations


Page 12 SP7-15: Triangulation methods

In other words, all claims develop to their ultimate position eventually. So the total claim
numbers / amounts (summed across all origin years) will be the same in the end, no matter which
type of cohort we use.

In some instances it may be helpful to consider alternative cohort structures. For example,
where the policy durations are variable, an accident year approach may give a clearer view
of the reserves than an underwriting year approach.

Development period
This is the period or frequency at which each claim cohort is tracked over time (for example,
at annual, quarterly or monthly points in time). Annual and quarterly periods are the most
commonly used.

In our example earlier in this section we used a yearly development period across the top of the
triangle.

When there are many segmentations of the business, projections based on, for example,
quarterly development factors, can sometimes make it more difficult to identify bigger
picture trends.

It is very common for management reports to require quarterly reserve estimates. It is


worth pointing out that if one requires re-estimation of reserves for quarterly reporting
purposes, best practice would indicate that one uses quarterly development factors.

This makes sense. If we’re carrying out a reserving exercise every quarter then we may as well
use quarterly development factors.

This does not mean, however, that projections based on quarterly development factors are
necessarily the best approach, although they are very widely used. This is because in some
situations, it may be appropriate to consider whether, in addition to (say) quarterly
projections, it is also useful to review projections based on longer (for example, annual)
development periods. This is to focus on the overall claims development pattern, rather
than the sometimes more volatile quarterly claims development.

Annual development periods have an additional advantage in that figures will be more
stable.

In practice both annual and quarterly approaches are commonplace.

Note: it is not necessary to use a constant development period over time. For example, it
may be helpful to use a quarterly period for the early stages of development, followed by
annual development once the pattern becomes more stable. However in practice actuaries
using bespoke reserving systems may have problems implementing this.

It should also be noted that the selected development period does not need to be the same
length as the selected cohort period. For example, projections of underwriting year cohorts
showing quarterly development periods are common.

Prioritising work / materiality


Once the grouping of the data into cohorts has been determined, it is possible that some of
the groups will be smaller than others and/or subject to more uncertainty. As an overall
rule, we should split the data into homogeneous groups as long as each achieves critical
mass for analysis purposes.

© IFE: 2019 Examinations The Actuarial Education Company


SP7-15: Triangulation methods Page 13

In general, we should give the same attention to all groups to ensure a robust analysis.
However, the more material (both in terms of size and uncertainty) groups may require more
detailed analysis.

Depending on the context for the analysis, in some circumstances it might also be
advisable to prioritise the analysis to focus (at least initially) on the more material classes,
so that any significant issues are identified early on in the process. Materiality can also
sometimes affect the selection of methodology. Very small or unusual classes may require
a more pragmatic approach that does not require large volumes of data. However, it is
important to realise that some groups that might appear small (for example, in terms of
premium and/or past claims) may actually have large underlying exposures and hence
might need more analysis than might be suggested by merely looking at the level of
premiums and/or claims.

An example here might be catastrophe exposures, where there might be very little useful
information from past claims data.

Some classes are totally unsuited to any standard actuarial approaches. In these cases
either a frequency-severity or exposure-based development period method could be used.

Examples of these classes may be extended warranty business or mortgage indemnity guarantee
business.

Heterogeneity of data
The different users of data within a general insurer will usually have different needs in terms
of the type and quantity of the data. For some purposes, the user may only require data on
a ‘big picture’ basis. Data for such an analysis may be publicly available in published
company accounts and statutory returns.

Question

Suggest which stakeholders might need the ‘big picture’ only.

Solution

Possibly shareholders, potential shareholders, analysts and regulators.

In many other situations, much more detailed data will be necessary. In all situations, the
quality of the data used is of paramount importance.

When reserving, we need far more detailed data than is available publicly, so that we can
split the data into appropriately homogeneous groups for the projections.

For example, for motor insurance business, we would need to divide losses at least into bodily
injury and property damage claims. Depending on the volume of data, we could also split losses
into a larger number of subdivisions.

Some classes of business will have more diversity of risks within them than other classes.
Insurers will need to be aware of this diversity, both from a pricing and reserving
perspective.

The Actuarial Education Company © IFE: 2019 Examinations


Page 14 SP7-15: Triangulation methods

The risk pooling principle of insurance is statistical in nature. Within each class of
business, the insurer seeks to take on groups of risks containing a sufficient number of
similar, but independent, risks. The proportion actually becoming claims, and the amounts
payable, can then, at least in theory, be predicted within manageable margins. This is a
result of what is popularly known as ‘the law of large numbers’, but which appears in
statistical theory as the necessary relationship between the variance of a sample and its
size. This general principle applies to claims reserving as well.

When considering data groupings we make a trade-off between ensuring sufficient


homogeneity (with regard to features such as class of business and claim type) and lack of
credibility (associated with sparse data).

One can easily fall into the trap of being guided totally by the desire by company
management for results at an ever finer level of detail (product, scheme, distribution
channel and even individual broker). This can work against the desire for reasonable
homogeneity for use in (for example) reserving. It also makes the process of reserving very
time-consuming, which can lead to insufficient analysis of the data and an overly
mechanical approach. At the other extreme, aggregating claim types where there has
clearly been a change of mix of business is also problematic, and we should be careful to
allow for this.

In some situations, there is simply not enough data available to produce reasonable
development patterns to allow most reserving techniques to be applied successfully. Here,
it is likely that we will need to use alternative approaches, such as the application of
benchmark factors derived from other relevant sources (for example, industry data) and
consider these alongside risk factors such as exposure erosion.

The term ‘exposure erosion’ here is referring to the need to adjust the benchmark development
for factors such as inflation levels, sums insured, policy numbers etc.

Where data is sparse, a number of reserving techniques may be used to ascertain a likely range of
outcomes.

We can often improve claims reserving accuracy by subdividing the data into groups
exhibiting similar characteristics, such as class of business (for example, property separate
from liability), comparable claim development patterns (delay to reporting and/or delay to
settlement) or size of loss and legal environment. Other examples of groupings include
property damage / personal injury claims, personal / commercial risks and, where relevant,
between primary or low layer covers and excess or high layer covers. Within each class of
business further subdivisions may be necessary. For example, for direct private motor
business, we might carry out separate analyses for comprehensive car, non-comprehensive
car and motorcycle – or analyses for property damage and injury claims separately. In
addition, it can be beneficial to the overall reserving process to subdivide or combine the
data so as to minimise the distorting effects of operational or procedural changes.

Removal of large losses from the data to improve homogeneity of development is discussed
below in Section 1.4.

In some situations, it may not be possible to subdivide data into desired homogenous
groups, but as long as the mix of the different subdivisions within the data is reasonably
stable over time, this may not cause too much distortion in the resulting reserve estimates.
While the different subdivisions may have different development patterns, the aggregate
development pattern may remain stable, providing that the mix of the different subdivisions
remains stable.

© IFE: 2019 Examinations The Actuarial Education Company


SP7-15: Triangulation methods Page 15

1.4 Treatment of large losses and non-standard risks


For a number of classes of business there is the risk of large or unusual claims.

For some classes of business, it may be necessary to analyse certain losses separately
from the main data triangles, such as large / catastrophe losses, latent claims and other
special or non-standard risks. This is partly because of their distorting effect and partly
because triangulation methods are not always suitable for dealing with large or catastrophe
losses, and special or non-standard risks. Such losses may exhibit a claims development
profile that is different from that of the rest of the claims: the attritional claims.

Examples of such losses or risks are:

 natural catastrophe loss – such as Hurricane Katrina impacting North America in


2005

 other catastrophe loss – such as the terrorist attacks in the US on


11 September 2001 (‘World Trade Center’)

 insurance of non-standard risks, such as a big construction project or satellite


launch

 individual large losses, for example, a fire on an oil rig

 latent claims, such as claims arising from exposure to asbestos, pollution and
health hazards

 other special or non-standard risks, such as Periodical Payment Orders in respect of


private motor insurance.

Large losses may need to be analysed on an individual case by case basis. Experienced case
assessors will be needed, and the approach may be time consuming (but necessary).

Clearly, case by case estimates cannot produce estimates for claims that have not been reported.
If case by case estimates are used for the main reserving process, reserves for IBNR must be
estimated separately.

Question

Other than not allowing for IBNR, suggest other disadvantages of using case estimates.

Explain the advantages of case estimates.

Solution

Disadvantages

The estimate relies on the skill and judgement of individuals. Different individuals may produce
quite different results.

Assessors may be naturally conservative or optimistic in their assessment.

If the estimate is used for negotiation with claimants, there may be a tendency for the estimate to
be biased to the lower end.

The Actuarial Education Company © IFE: 2019 Examinations


Page 16 SP7-15: Triangulation methods

Case estimates are extremely difficult to check.

In some classes there may be thousands of outstanding claims. It will take many person-hours in
total to estimate each claim amount individually.

Assessors may not use consistent rates of inflation. Also it will be hard to produce estimates on a
range of possible bases.

In some circumstances, estimates of the outstanding claims reserves will need to be made by
outsiders who do not have access to all the data on individual claims. In these circumstances,
individual case by case estimates are not possible.

Advantages

It is the only approach that can make use of all the known data on outstanding claims.

There are many qualitative factors that will influence the amount of a claim. An experienced and
skilled assessor will be able to weigh up all these factors when estimating the amount of the
claim.

Case by case methods may still be applicable when statistical models are not reliable.

Large losses
The definition of large losses will often depend on the class of business and the purpose
and assumptions of the reserving exercise. If we separate large losses from attritional
losses, and project the latter using standard actuarial techniques, we should consider
whether to make additional allowance for the former, or simply exclude them.

Rather than excluding them, it may be necessary to deal with them separately.

Individual large losses are a very important and sensitive feature of the reserves of many
classes of business, such as catastrophe excess of loss and motor (and liability) accounts.
For example, trends in personal injury claims are usually different from those for property
damage claims.

Sometimes the largest losses take some years before they are recognised as being very
large, as information on the circumstances of the claimant and the extent of injury or
damage emerges.

There will often be a lack of available data as to how such claims will develop.

When reserving at an early stage of development, we might use a credibility-based


approach, by considering not only actual notified claims experience, but also the likely
eventual total cost of large claims above a certain size across the whole portfolio. We may
base the latter on data from more mature years, or on market data if available.

We should decide whether to cap large losses at a certain level, and to project the excess
over the cap separately, or to remove the excess altogether. In either case, it may be
appropriate to index the large loss limit (or cap) from year to year, to allow for inflation.

If large losses are covered by reinsurance, we may want to exclude their effect in the calculation
of the reserve.

© IFE: 2019 Examinations The Actuarial Education Company


SP7-15: Triangulation methods Page 17

Methods for deriving estimated ultimate values for large losses include:

 claims development methods

 exposure-based methods – this would include discussions with the claims


department to understand the terms of coverage and any related legal
developments.

These are each discussed below.

Claims development methods


Claims development methods include standard actuarial techniques such as the chain
ladder method, as discussed in Section 2.

We should project most large claims from the date of their actual loss rather than the start
of the origin period. This is to ensure that we can compare development for different large
losses on a consistent basis.

For example, let’s consider two underwriting years (origin periods) that are impacted by
very similar types of claims, with one large loss in each year. The difference between the
two is that the large loss in the first year occurred on 1 May, whereas the large loss in the
second year occurred on 1 October. Other things being equal, we would expect the
development as at the year-end (31 December) to be different between the two years, since
the large loss in the first year will have had eight months’ development, whereas the large
loss in the second year will only have had three months’ development.

Catastrophe losses
Catastrophe losses often tend to develop more quickly than attritional claims. The main
reason for this is the increased focus on these claims from claims adjusters and
policyholders due to the magnitude of the losses. It is therefore appropriate, where
possible, to review these losses separately from attritional claims.

For example, a severe flood or hurricane is likely to lead to quick reporting, and substantial
pressure for quick settlement.

Once claims experience starts to emerge, the development pattern of similar catastrophes
in the past may assist the actuary in refining initial estimates. We should not apply
mechanical curve-fitting approaches at an early stage of development without considering
the underlying exposure, since the estimates from this approach can be highly unreliable.
At later stages of development it may well be appropriate to apply claims development
methods consistent with the approach described previously for large losses.

A catastrophe is likely to distort the run-off, although it is not always obvious what effect it will
have. The effect depends upon when in the year the catastrophe happens and the run-off of the
claims from that catastrophe. For an average cost per claim method the relative size of
catastrophe and non-catastrophe claims is important too.

The Actuarial Education Company © IFE: 2019 Examinations


Page 18 SP7-15: Triangulation methods

Let’s consider a very simple example, firstly without a catastrophe. We’ll use the basic chain
ladder method approach. Consider the following incremental claims data:

development year
0 1 2 3
2015 10 6 2 1
2016 10 6 2
origin year
2017 10 6
2018 10

In this example, all the assumptions underlying the basic chain ladder method just happen to hold
perfectly, so that the outstanding claim amount would be £13m (£1m from 2021, £3m from 2020,
£9m from 2019). We shall now consider the effect of a catastrophe on this run off. We will
reapply the chain ladder to the new modified figures to see what happens.

Suppose there had been a catastrophe in 2016 which gave rise to many claims.

Version 1

The catastrophe took place in January 2016 with a total extra claim amount of £4m. All the claims
were reported and settled in 2016 and there were no outstanding claims from the catastrophe as
at 31 December 2018. The ‘correct’ outstanding claims figure is still £13m. However, if we
increase the c16,0 figure to £14m and re-work the calculations on the chain ladder, we get an
answer of £11.93m .. more than £1m below the correct figure.

Version 2

This time, the catastrophe occurred on 30 December 2016 and all the extra £4m was paid in 2017.
Again, there were no outstanding claims from the catastrophe as at 31 December 2018, so the
correct answer is still £13m. However, the run-off based on c16,1 increased to £10m gives a result
of £14.32m .. more than £1m above the correct figure.

Version 3

The catastrophe happened in the middle of 2016 and the development of the additional
catastrophe claims was identical to the development of other claims. The catastrophe gave rise
to an extra £3.8m, of which £2m was paid in 2016, £1.2m in 2017, £0.4m was paid in 2018, and
£0.2m was expected to be paid in 2019. The outstanding claims amount as at 31 December 2018
was £13.2m (ie £13m as before plus £0.2m extra from the catastrophe), and this is precisely what
the chain ladder gives if re-worked on the new figures. (Check you can agree this figure.)

If you have completely forgotten how to apply the basic chain ladder method, you may wish to
come back and revisit this example when you have read Section 2.

© IFE: 2019 Examinations The Actuarial Education Company


SP7-15: Triangulation methods Page 19

Conclusion
The only conclusion we can safely draw is that catastrophe events within the data may disturb the
run-off calculations. We cannot conclude that catastrophe claims in the data will necessarily lead
to over-estimates of outstanding claims. If the development of claims from the catastrophe is
identical to the development of the other claims, then the run-off calculations will not be
affected.

Question

At first glance the assertions above that large single claims will disturb the run-off pattern, while
for catastrophe claims the effect on the run-off pattern is uncertain, may seem contradictory.
Explain why this is not the case.

Solution

For a catastrophe event, the key thing to consider is the pattern of claim payments. If this
resembles the ‘normal’ pattern then the run-off calculations will be undisturbed.

In contrast, for a large single claim payment, the payment falls at one particular point in time and
so the pattern will always be disturbed.

Exposure-based methods
In some cases, statistical or claims development methods are not suitable. Methods that
may be used in such cases include exposure-based reserving. This may take the form of a
‘bottom-up’ or ‘top-down’ approach.

 Bottom-up: We examine on a policy by policy basis to determine the likelihood that


each policy is exposed to the loss event (by identifying factors such as the location
of the underlying assured, dates of cover, perils covered under the policy, exclusion
clauses and so on). This will usually be done by underwriting and/or claims staff. If
they decide the underlying insured is definitely or potentially exposed to the relevant
event, a claims expert needs to assess the extent of any claim on that policy (by
considering the size of the loss in relation to policy excesses and limits and so on).

 Top-down: We attribute the total market loss from an event to an individual insurer
or policy level, based on the policy terms, excesses and limits. In the case of an
insurer that has written less than 100% of a particular risk (which is a common
feature in the London Market), the reserving or claims staff will also reflect the
proportion of a particular risk that the insurer has agreed to insure. The reserves
held against the total claim will thus be the sum of the individual provisions of each
coinsurer, an amalgam of different views and risk appetites. At the very early stages
of development, when there is very little information available to derive estimates,
the insurer may estimate its losses as the product of an estimated total market loss
and its estimated market share.

Although there will be some past experience of events such as hurricanes and satellite losses,
each event will be unique and so any reserving exercise should take into account the particulars
of each individual event.

The Actuarial Education Company © IFE: 2019 Examinations


Page 20 SP7-15: Triangulation methods

For both methods, in many countries there are tools available (from brokers, consultants and
specialist companies) to help the claims expert assess the likelihood and extent of losses given a
particular event.

Catastrophe models are such a tool, which assess likely loss experience given the exposure of the
company, although these are more often used for premium rating rather than reserving. These
models are discussed further in Subject SP8.

Latent claims
These types of claim include those arising from asbestos, pollution and health hazards
(APH). For such claims, we should use other reserving methods because of the pattern of
emergence of latent claims. Many latent claims have a ‘calendar year’ development effect,
that is, a similar claims development pattern for a particular calendar year rather than a
development year.

It’s not really clear why this should be the case. It is possible that for some types of
asbestos-related claims, publicity in the media may influence the rates at which claims develop
(ie reading about other people’s claims may encourage those who have not yet claimed to do so).

Thus a group of underwriting years can show development that results from the same
underlying latent claims. Because of this, the earlier underwriting years cannot be easily
used as a guide to how the later underwriting years might develop. Therefore, chain ladder
and other triangulation methods are not usually suitable for APH or latent claims.

Instead we might use a survival ratio. A ratio often used to estimate asbestos loss reserve
adequacy is the three-year survival ratio, which is the ratio of loss reserves to the three-year paid
loss average. Assuming that average paid losses remain constant, with no additional reserving,
the survival ratio indicates how many years the existing reserves should last.

Latent claims are discussed further in Subject SA3.

1.5 Other issues to consider

Basis on which business is written


In this section, we discuss three possible ways in which losses may be attributable to
policies: losses occurring, claims made and risks attaching. There are other variations, but
these three are the most common. In general terms, for reserving purposes, it is desirable
to subdivide data between these three categories, although this may not always be practical
or necessary.

Losses occurring
Under this common basis, the policy provides cover for losses occurring in the defined
policy period, no matter when they are reported.

Many policies are written in this way, eg motor insurance, buildings and contents. This is
consistent with an accident year approach.

Such policies are thus subject to potential problems in defining the date of loss which may
be established as a result of legal action.

© IFE: 2019 Examinations The Actuarial Education Company


SP7-15: Triangulation methods Page 21

This was discussed in Section 1.3.

Claims made
Under this basis, the policy covers claims reported (or ‘made’) during a certain period rather
than claims arising out of occurrences during that period (‘losses occurring’).

Thus, a claims made policy covers losses reported during the policy year (plus, in some
cases, during an extended reporting period, particularly on renewal of coverage). It is used
to reduce the tail on liability business by removing the development caused by late
reporting. In other words the ‘pure IBNR’ is effectively removed.

Late reported claims will fall under the following years’ policies.

By the end of a policy year, all claims to the policy will be known, although some may have
subsequent changes in their value; that is, there is scope for development arising from
IBNER, but not from pure IBNR. It should be noted that there may be large numbers of
claims notifications that never give rise to payments as the insured would notify any
possible event on a precautionary basis. All other things being equal, claims made cover
will have a shorter development tail than losses occurring cover.

Question

Despite this, many practitioners say that reserving on a claims made basis is more difficult than on
a losses occurring or risks attaching basis. Suggest why this may be the case.

Solution

Because the losses can arise from policies written at very different times, or covering losses that
occurred at very different times.

The claims-made approach also overcomes the issue of establishing an occurrence date
where this may be difficult (eg a claim for negligent financial advice where advice was given
over a period of time), although there may be an exclusion of any claims that occurred
before a specific date (the retroactive date).

Risks attaching basis


This is a basis under which reinsurance is provided for claims arising from policies
incepting during the period to which the reinsurance relates. For reinsurance policies, it
ensures that they collectively match the inwards business. For example, a reinsurance
policy on a risks attaching basis running for one year from 1 January 2017 will cover claims
on reinsured policies written during 2017. Hence, if the last of those policies was a
12-month policy that started on 31 December 2017, then the reinsurance policy could, in
theory, cover a loss event from this policy that occurred on 30 December 2018.

This is consistent with an underwriting year approach.

The Actuarial Education Company © IFE: 2019 Examinations


Page 22 SP7-15: Triangulation methods

Benchmarks
When considering development period ratios as part of chain ladder techniques, we often
adjust the actual results using judgement. This can include the selection of a tail factor
(beyond the available development data), which can have a big impact on the final results.
We could base our judgement on benchmarks obtained from the development experience of
the business of other companies, from market data or from similar portfolios where more
data is available.

We can apply benchmarks in different ways, including:

 age to age development factors

 ultimate to paid or incurred factors

 ultimate loss ratios

 IBNR as a percentage of paid, outstanding or incurred

 IBNR as a percentage of premium.

An age-to-age development factor is just the development factor as calculated in the standard
basic chain ladder method approach. But here we would adjust the factors obtained from our
own data by comparing them with the corresponding benchmark data.

When applying benchmarks to derive reserve estimates, it is very important to consider


whether the source from which the benchmark is derived has characteristics that are
appropriate to the business for which the reserves are being derived. For example, the
claims-reserving philosophy for reported outstanding claims should be similar and the
underlying nature of the business should be sufficiently similar and so on. In all cases, it is
necessary for the actuary to consider carefully whether the benchmark is relevant to the
account being reserved, and whether any adjustments are necessary when applying the
benchmark.

Question

Suggest ways in which benchmarks may differ from the business being analysed.

Solution

They may refer to different:


 classes of business
 terms and conditions
 allowance for reinsurance
 time periods
 excesses
 geographical location
 mix of business
 coverage

© IFE: 2019 Examinations The Actuarial Education Company


SP7-15: Triangulation methods Page 23

 reserving basis
 inflation allowance, etc.

As well as being a practical (albeit simplistic) way of deriving reserve estimates in situations
where data are limited, we can also use these benchmarks to compare reserving strength
between insurers (that is, establish the reserve estimates using the methods described
elsewhere in this course, and then calculate the benchmark ratios and so on). We can also
use them as an alternative approach to compare against the results derived from using the
insurer’s own data.

1.6 Issues that may affect the stability of the claims development pattern

Changes in terms and conditions


The terms and conditions of insurance policies can change between one period and the
next. If the terms and conditions change substantially, this could have an impact on:

 the claims development, or on

 the level of claim amounts.

For example, if the terms include cover for risks that were not previously covered, claim
amounts may be higher. Similarly, if certain liability risks are included with property cover,
the claims development may be longer-tailed. Changes in deductibles can have a
significant impact on development patterns.

Actuaries should discuss changes in terms and conditions with claims and underwriting
staff as part of any reserving exercise.

Question

Suggest what action should be taken if additional risks are now covered under a group of policies,
due to changes in the terms and conditions.

Solution

If there is sufficiently credible data available, claims arising from these new conditions should be
stripped out and analysed separately.

Changes in the mix of business


Changes in the mix of business will increase the heterogeneity in the data. As stated in
Section 1.3, it may not be possible to analyse the data at a level where the mix of business
is relatively stable from one period to the next.

If there has been a change in the mix of business over time, it is important to estimate the
impact of this change on the claims development pattern. For example, if the proportion of
liability business increases relative to standard property cover in a portfolio insuring small
businesses, then we should consider lengthening the claims development for the more
recent cohorts.

The Actuarial Education Company © IFE: 2019 Examinations


Page 24 SP7-15: Triangulation methods

Distortions in the data


Distortions in the data due to changes in (administration) processes can affect the normal
claims development.

Such changes could involve stricter claims underwriting or a new system to speed up the claims
processing. Such changes affect the size and speed of the run-off respectively.

We should remove distortions from the data if the event is unusual. We should then
consider the distortions separately.

We discuss below some of the events that can cause distortions in the data. In any
reserving exercise, it would be appropriate for the actuary to seek to understand whether
any of these distortions exist, and if so, make appropriate allowance for them when deriving
reserve estimates that rely on the data containing the distortions.

Changes in claims handling procedures


One of the key assumptions underlying the chain ladder method is that the pattern of
historical development is a good guide to how future experience will develop. However,
changes to claims handling processes and procedures may invalidate this assumption.
Some examples of the way that this might occur are as follows:

 Changing practices regarding the point at which a notified loss is formally accepted
as a claim by the company and is marked as such on the claim file.

 A new claims process is implemented that speeds up the processing of newly


notified claims.

 The case reserving philosophy is changed. For example, from ‘best estimate’ to
‘realistic worst case’, or if the initial standard case reserve is changed for certain
types of claim.

 A change in the period before non-active claims are reviewed – either to chase for
outstanding information or to close the claim as a nil claim.

 Delays in making claims payments, whether deliberate or due to circumstances.

 The failure to mark claim records as settled on a consistent basis.

This last point might occur if the definition of what constitutes a settled claim is changed. This
could lead to inconsistencies between claims categorised as settled at different times.

Question

Without reading on, explain how you might deal with these changes.

Solution

See the Core Reading below.

The first four examples above might be expected to cause a one-off change in the way that
claims develop. This will then affect all future claims development after the change
occurred. The last two examples might cause a temporary change in the way that paid
claims develop. This would not be expected to affect the future claims development.

© IFE: 2019 Examinations The Actuarial Education Company


SP7-15: Triangulation methods Page 25

In each case it is important to recognise the impact that the change in procedures might
have on the claims development and to use judgement to make allowance in the projection
in order to remove any potential distortion.

One way to deal with the first three examples might be to base the chain ladder link ratios
only on development data from after the new procedures came into effect. Alternatively, we
may make subjective adjustments to allow for the fact that the future development is
expected to be different from that suggested by the historical data.

We may deal with the last two examples by excluding the affected link ratios. For some
changes in claims handling procedures, which only affect the development of incurred
claims, it may be appropriate to rely more heavily on paid development data for the
projection.

Market-wide initiatives
From time to time a major issue may arise that the market is keen to quantify as quickly as
possible.

For example, a market-wide initiative was put in place to speed up the reporting of energy losses
arising from damage to oil rigs, following hurricanes in the southern US and Gulf of Mexico during
2005.

If such initiatives are taken, it is important that any chain ladder model that is fitted to the
data recognises that the claims development may be different from what may be typically
expected for the type of business or loss in question. For example, it may be inappropriate
to use the benchmark from the particular development of a market-wide event to estimate
the ultimate claims from another event, without adjustment.

Claims reviews
It is common for companies to undertake periodic claims reviews to ensure that the case
reserves being held accurately reflect all currently known information relating to the claim.
If such reviews are carried out regularly then it might be reasonable to assume that the
historical development reflects the average impact that such reviews have, and that the
future development may follow a similar pattern.

However, if such reviews are infrequent, or if a large ‘one-off’ review has been carried out,
or if the company undertakes the reviews more frequently than it has in the past, it may be
necessary to adjust the development pattern derived from the data.

This is particularly the case if the company has carried out a review of ‘redundant
outstandings’, that is, where they have open claims files containing non-zero case reserves,
but where there have not been any payments or other movements for a period of time.

Such reviews are often carried out at late periods of development and can lead to
substantial reductions in reported outstanding claims if some of these are found to be
‘redundant’.

An example of a redundant claim would be where the insurer found, following long investigation,
that it was not liable to pay the claim.

If the development pattern is used to project future claims for other years, we should be
careful to ensure that the underlying assumptions are still valid.

The Actuarial Education Company © IFE: 2019 Examinations


Page 26 SP7-15: Triangulation methods

If the rate at which claims are processed alters through backlogs or ‘clear-outs’, changes in
procedures and so on, this will distort the claim development patterns, and hence the
analysis of them. Unusually low claims development factors may indicate a claims backlog.

The impact of claims clear-outs can be the opposite of claims backlogs. A concerted effort
to settle or release case reserves for reported claims which have been dormant for a long
period may result in an unusually high paid claims development factor or a decrease in
incurred claims.

In some situations (albeit rarely in most territories / companies), the claims system may fail
to keep a historical record of the estimates (at each calendar year-end or quarter-end, say).
This will inhibit the use of this information for statistical purposes.

Seasonality
Seasonality can affect the claims development pattern, both within a cohort and for one
cohort relative to another where the exposure period is not the same. Seasonality is the
tendency for certain types of policy to have more claims at certain times of the year. For
example, motor policies usually have more claims in winter when the weather conditions
are more hazardous and household insurers receive more subsidence claims in the
autumn / winter following a hot summer. Seasonality may not have a bearing on annual
development factors but may ensure that quarterly factors appear less smooth.

Seasonality can also impact the speed at which claims are processed – for example, fewer
claims may be processed during December when there are a greater number of bank
holidays.

Commencement of writing policies


If the average start date of policies changes significantly from one year to another then
there may be a change to the development pattern as a result of influences to the period of
exposure. For example, many Gulf of Mexico energy policies moved after the 2005
hurricane season so that they ‘attached’ before the next hurricane season. This would have
shortened the 2005 exposure and thus invalidated previous patterns.

The average start date might also change for example if a marketing campaign is carried out for a
short period, which would affect the average start date for policies of a particular type.

Average policy length


If the average length of policies changes from one year to the next, this can cause changes
to the underwriting year development pattern, with shorter-term policies tending to
accelerate the development, and longer-term policies tending to lengthen it.

Although most types of general insurance policy are for a fixed term of one year, there are some
types of policy that are not, for example travel insurance and extended warranty insurance.

Reserving policy
The way the case reserves are set influences the incurred claims development. For
example, it can take a number of weeks or months once a claim is reported before a
reasonable estimate of the likely cost of a bodily injury claim is known. A policy of
assigning, say, £1,000 initially to a reported claim will result in a different development of
incurred claims compared to an alternative policy of assigning, say, £10,000 initially.

© IFE: 2019 Examinations The Actuarial Education Company


SP7-15: Triangulation methods Page 27

Other issues to be considered include closing date changes, computer system changes and
noting levels.

Noting levels are also known as notification levels, ie the claims level above which claims should
be referred to a reinsurer.

A closing date is the cut-off date when the recording of claims stops for a particular period.
For example, if the recording of claims reported has a closing date of five working days
before the end of the period, then a claim reported, say, three working days before 31
December will be recorded as a claim reported in the following year. If this closing date is
unchanged from one period to the next, claims development patterns are not distorted,
other things being equal. However, if the closing date is changed to, say, three working
days before the quoted date, then the claims development pattern for the 2017 year will be
slightly slower due to the inclusion of some claims in the 2017 year that would otherwise
have been included in the 2018 year.

Developments in the business, economic and legal environment


Allowance for future inflation

Each element of the reserves should include some allowance for future escalation of costs
(implicit or otherwise).

We will be looking at the different methods later in this chapter. Some of these allow explicitly
for inflation. Others use an implicit approach.

For property claims, the main influences are likely to be price inflation and building cost
inflation. For liability claims, we should allow for a combination of earnings inflation and,
possibly, a continuing trend towards increasing generosity in court awards. We can
examine recent movements in the relevant indices with reference to current economic
assessments of inflationary trends generally, to obtain a basis for the assumptions to be
made in reserving.

The chain ladder method makes implicit allowance for future inflation, by assuming that the
inflation observed in past claims history will continue at the same rate in the future.

However, past inflation may not be appropriate for the future, for example if there was a step
change in the rate of increase of court awards affecting liability claims.

If we expect that conditions affecting the future inflation rate are likely to change from those
prevailing in the past, we should make specific allowance for this change, for example, by
adjusting the claims development factors derived from past historical development, or by
using the inflation-adjusted chain ladder method.

Recession or economic upturn

Certain claim types are more prevalent under certain economic conditions. Either a
recession or an economic upturn can impact both claim size and frequency separately and
possibly in opposite directions.

For example, the incidence of professional indemnity claims tends to be higher under
recessionary conditions. If the future economic conditions are likely to be different from
those prevailing over the observed historical period, we should allow for this.

The Actuarial Education Company © IFE: 2019 Examinations


Page 28 SP7-15: Triangulation methods

Question

Explain the likely effect of a recession on:


(i) household contents policies
(ii) mortgage indemnity guarantee policies.

Solution

(i) There is likely to be an increase in claims for theft under household contents policies.

(ii) A recession is likely to produce increased unemployment, increased difficulty in meeting


mortgage payments, more repossessions, and lower values achieved by the lender from
the sale proceeds of properties. All of these factors are likely to contribute to an increase
in claims under mortgage indemnity policies.

Legislation

New legislation, or changes in laws and regulations, can cause discontinuity in the claims
experience. An early example was seatbelt legislation that came into effect in many
countries from the 1970s to the 1990s, making it compulsory (with certain exceptions) for
drivers and passengers to wear safety belts. The effect was to reduce the severity of many
of the injuries suffered in road accidents, thus causing a change in the claims pattern in
motor business.

At the time of a legislative discontinuity, the extent of the influence on the claims pattern
may be difficult to predict. We can, however, make some reasonable allowance for this, and
then subsequently check this against the emerging experience. Also, as time passes, the
presence of the discontinuity will come to be viewed as a historical fact. Hence when the
affected experience is used in projections, we can make any necessary adjustments more
easily (for example, exclusion of individual development ratio or ratios when using the
chain ladder method).

Legislation is usually drafted to apply to future events and occurrences. Hence, new
legislation will not usually affect claims arising from prior accident periods.

However, legislation may occasionally have a retrospective effect, or may have to be put
into immediate effect in new court decisions. In such cases, some adjustment to the
reserves for prior accident periods may be necessary. One example is the change in the
Ogden discount rate in the UK which is used to calculate personal injury awards. Ogden
tables are an actuarial measure used to reflect life expectancy and designed to provide a
guide for courts in assessing damages awards in personal injury and fatal accident cases.
The discount rates are applied to the Ogden tables to assess the initial payment to a
claimant.

In early 2017 the Ogden rate was revised from 2.5% per annum to -0.75% per annum. This
caused a significant one-off increase in reserves for bodily injury claims with a future loss
of earnings and/or future care component.

© IFE: 2019 Examinations The Actuarial Education Company


SP7-15: Triangulation methods Page 29

The Ogden Tables are effectively tables of life annuities, which allow an expected present value to
be placed on (lost) future income. They allow for a certain level of impaired life mortality, and are
calculated at various interest rates. There are a number of different tables, for example loss of
income for whole future life, loss of a pension income to be taken at age 65, and so on.

Legal environment and litigation

In many countries there has recently been a move towards a more litigious society leading to
higher claims and more claims being made.

Some industry commentators have argued that the litigation culture (or ‘compensation
culture’ – a tendency for people to take legal action to gain compensation for themselves)
has tended to result in claims inflation rates significantly in excess of price or earnings
inflation for many liability and pecuniary loss claims. This not only affects the future claims
inflation rate, but may also lead to increased claim frequency and new types of claims
arising in future that may have not been inherent in the past claims data.

Changes in the legal process – such as processes introduced to speed up court cases and
judgements – may also speed up the development patterns for paid claims and so an
allowance may need to be made for this.

Legal awards and judgements could lead to a one-off step change in the claims
development patterns or change the pattern of future claims development patterns. An
example of this could be a slowing down of claims settlement while the outcome of a test
case is being awaited.

For certain classes of business, class actions (for example, from shareholders) may also
result in step changes similar to the ones mentioned above.

A class action could occur, for example against a pharmaceutical company if it transpired that a
company product had severe and unanticipated side effects, and had been widely used.

Other considerations

Accounting changes may have an impact on how the data is presented.

Medical advances could decrease claim costs; for example, allowing injured persons to
return to a normal way of life earlier than had been the case before. If medical advances
result in earlier detection of disease claims, they could accelerate claims reporting and
settlement patterns. On the other hand, they may also increase the cost of claims as
seriously injured or ill people live for longer than before.

Application of actuarial judgement


As can be seen from the above, when triangulation methods are used there are a number of
issues to consider that may invalidate the underlying assumption that the future claims
development is likely to be in line with the past claims development.

We should use actuarial judgement during the process of reserving rather than the
mechanical application of a triangulation method to the data. We should also use actuarial
judgement once we have estimated the reserves, to consider whether the results appear
reasonable. We can do this by using diagnostic tests, benchmarks and past experience,
amongst other things. This is discussed in Chapter 17.

The Actuarial Education Company © IFE: 2019 Examinations


Page 30 SP7-15: Triangulation methods

The purpose for which the reserves are estimated is also important. The same actuary
using judgement could make a different estimate of reserves for purpose A from that for
purpose B. This was discussed in Chapter 14.

For example, reserving estimates might be obtained for regulatory purposes, or for internal
management, leading to different figures.

As the process of reserving is not an exact science, two actuaries could estimate different
values for reserves even if the reserves are for the same purpose and based on the same
information.

This will reflect the role of actuarial judgement in the reserving process.

Allowance for re-underwriting


Sometimes an insurer will state that they have ‘re-underwritten’ a class of business. This
may be for:

 a generally poor performing class of business

 a class which has suffered material losses from a single event which led to claims
from a large number of policies.

In the former scenario, an insurer may identify that a particular segment of the class has
caused the poor results. In future underwriting they will either seek to avoid that business
(only writing the better performing business), or increase the premium charged.

In the latter scenario, an insurer may revisit the terms and conditions under which they are
prepared to write business following material losses from a single event.

When reserving, an actuary will need to apply judgement in assessing the impact of any
re-underwriting, and seek sufficient information to support any claimed improvement.

© IFE: 2019 Examinations The Actuarial Education Company


SP7-15: Triangulation methods Page 31

2 Chain ladder (link ratio) method

2.1 Introduction
You are likely to have met most of the terminology and methods mentioned in the rest of this
chapter in your earlier studies. It may be worthwhile revisiting your notes from these courses if
you struggle to remember any of the detail.

The ability to carry out the calculations for the various statistical methods is assumed knowledge
for Subject SP7, and you may be asked to carry out such calculations in the exam. However,
Subject SP7 also builds on your current knowledge to examine the principles underlying the
methods and the impact of distortions in the data on a given method.

With statistical methods, the aim is to estimate the ultimate claims amount using past experience.

The chain ladder method is a statistical method of estimating the ultimate value of a set of
development data, whereby an average of the past development is projected into the future.
The projection for successive periods of future development is based on the actuary’s
calculation of ratios of cumulative past development.

We tabulate the development data (based on past experience) in the form of a run-off
triangle or development table. The rows of the triangle correspond to the years (or other
period) of origin, whose definition will depend on the choice of cohort. The columns of the
triangle correspond to the claims development period. The claims development is usually
cumulative (that is, total claims for the cohort accumulated to development period).

The key assumptions underlying the chain ladder method are:

 The assumed pattern of development derived from past experience will remain
appropriate in the future. Unfortunately for some classes there may not be a
recognisable pattern.

 The first cohort is fully run-off, or that its development to an ultimate position can be
predicted with some confidence. If the latter is the case, a ‘tail factor’ will be
required.

 Where the triangle used is unadjusted for the inflation which is present in the data,
the method builds in an implicit assumption that a weighted average of past inflation
will be repeated in the future.

This is the basic chain ladder method.

Alternatively we can allow in more detail for inflation by using the inflation-adjusted chain ladder
method, which we will remind ourselves of later.

This method can be applied to premium and claim development data. A full description of
the chain ladder method is included in the Core Reading for an earlier subject.

Let’s see an example of how the chain ladder method works in practice.

The Actuarial Education Company © IFE: 2019 Examinations


Page 32 SP7-15: Triangulation methods

Question

You are given the following data in respect of claims originating from recent years, split by year of
payment (in £000s). The table below shows the total amounts paid in each development year for
each accident year (in £000s).

settlement delay in years (development year)


0 1 2 3 4
2014 125 104 65 32 9
2015 120 98 57 27
year of
accident 2016 148 124 75
(origin year)
2017 143 110
2018 138

Using the basic chain ladder method estimate the total outstanding claims, including reported
and IBNR claims, as at 31 December 2018.

Solution

Step 1 Find the cumulative payments from each origin year as at the end of each development
year:

settlement delay in years (development year)


0 1 2 3 4
2014 125 229 294 326 335
2015 120 218 275 302
year of accident
2016 148 272 347
(origin year)
2017 143 253
2018 138

Step 2 Find the development factors:

229  218  272  253


r0:1   1.8134
125  120  148  143

294  275  347


r1:2   1.2740
229  218  272

326  302
r2:3   1.1037
294  275

© IFE: 2019 Examinations The Actuarial Education Company


SP7-15: Triangulation methods Page 33

335
r3:4   1.0276
326

Step 3 Use the development factors to estimate the total claim payments outstanding from each
origin year:

2015: 302  r3:4  302  8.34

2016: 347  r2:3  r3:4  347  46.55

2017: 253  r1:2  r2:3  r3:4  253  112.57

2018: 138  r0:1  r1:2  r2:3  r3:4  138  223.59

Step 4 Add together the outstanding claim payments from each accident year for the estimate of
the total outstanding claims as at 31 December 2018 (ie 391.05).

In some circumstances, you may need to consider the amounts expected to be paid in each
separate origin / development cell (eg if you wanted to see the emerging cashflows in each year).
In this case, Step 3 above would have to be modified as follows:

Step 3 (revised) Use the development factors to find the cumulative amounts in each origin /
development year:

C15,4  302  r3:4

C16,3  347  r2:3

C18,4  C18,3  r3:4 and so on.

These cumulative amounts can then be disaccumulated to find the amount in each cell.

When to use the chain ladder method


The chain ladder method can be used to project many different categories of data, including
premiums, paid and incurred claims and numbers of claims.

Loss ratios may also be projected but best practice would normally dictate that premiums
and claims are projected separately.

The Actuarial Education Company © IFE: 2019 Examinations


Page 34 SP7-15: Triangulation methods

Question

(i) Explain why premiums should be projected.

(ii) Explain why premiums and claims should be projected separately.

Solution

(i) Because they may get paid over time, and you want to estimate the ultimate amount
paid. Projection is also useful for investment income calculations.

(ii) A loss ratio projection will give no indication of the key factors driving the results, and
whether these are premium-related factors or claim-related factors.

For example, premium development may be affected by a change in distribution


channels, whereas claims development may be affected by any number of factors, such as
weather conditions or economic conditions.

Similarly, a loss ratio projection will be unable to distinguish whether a change in


profitability is due to changes in premium rates or changes in claims experience,

To be able to estimate the ultimate value with reasonable certainty, it is necessary to have a
number of years of historical data. We need adequate data to construct a triangle with
sufficient development to give a good indication of what the ultimate value of the earliest
cohort will be. The actual amount of historical data required will vary depending on the
features of the data set being considered. We should use other actuarial methods when
insufficient data is available to apply the chain ladder method.

We will look at these later in this chapter.

We should ideally apply the method to data that is homogenous and consistent; for
example, to claims data where the individual underlying claims share similar reporting,
settlement and inflationary characteristics. Data should also be consistent in terms of
timing and content. For example, the mix of claim types should not vary across origin
periods. There should also be a sufficient volume of data to ensure credibility.

In practice it is very unusual to have a data set that is completely homogeneous. Examples
of issues that can cause heterogeneity, and ways that these issues can be addressed, are
discussed later in this section.

2.2 Issues arising


This section discusses particular problems that arise when using the basic chain ladder method.

Analysis and selection of link ratios


The link ratios for a cohort are the ratios of the accumulated claims up to a period compared
to the accumulated claims up to the previous period.

Link ratio is just another term for development factor.

© IFE: 2019 Examinations The Actuarial Education Company


SP7-15: Triangulation methods Page 35

When we estimate amounts using the chain ladder method, it is important to look for any
trends in the link ratios (such as a calendar year effect), unusual link ratios (for example,
‘high’ or ‘low’ link ratios) or cohorts which appear unusual compared to others. It can be
helpful to plot development charts to assess these.

It is essential to understand the underlying reasons for the unusual link ratios. For
example, an unusually high link ratio may be due to a late reported loss, or a distortion
affecting a single calendar year may be due to some processing errors. We must consider
whether such link ratios are appropriate to include in the projection. Particular care must
be taken in the tail, where the inclusion of an abnormally large (or small) link ratio has the
potential to have a significant impact on a number of cohorts.

Also, we may observe that a group of origin cohorts are out of line with others, and fit
separate link ratio models to them. In such cases, it is helpful to identify whether there is a
reason for the difference; this assists in deciding the appropriate approach.

Question

Give an example for each of the three types of origin cohort as to why a group of origin cohorts
might be out of line with others.

Solution

For an underwriting year cohort, it could be due to a change in terms of conditions, eg excess.

For an accident year cohort, it could be due to changes in legislation affecting all claims from a
certain date.

For a reporting year cohort, it could be due to a change in claims administration staffing.

Many other reasons are possible.

Mechanical generation of standard link ratio models for initial modelling


Claims reserving software has been developed that makes the process of chain ladder
modelling increasingly automated. There is, however, always need for judgement to
understand the factors underlying the link ratios and to ensure that data anomalies are
queried and understood.

We also need judgement to select the various options available within the mechanical
process, such as whether to use volume-weighted or simple averages within the chain
ladder process.

If we use a simple average, we are implying that each past year is equally important in
determining the link factor. However, we may feel that recent past years are more relevant than
older years, in which case we would not want to use a simple average.

In addition, we may feel that years with more volume of data are more credible, in which case
greater weightings should be placed on these years.

The Actuarial Education Company © IFE: 2019 Examinations


Page 36 SP7-15: Triangulation methods

Case reserve, paid and incurred modelling


Factors affecting claims development patterns

There are a number of external factors that can affect the observed claims development
patterns. These were discussed in Section 1.6. We should consider these when we apply
chain ladder methods to paid and incurred claims development data.

Approximations for incomplete triangles

In some instances full triangles may not be available; for example, where past calendar
years’ data is lost due to legacy systems (inherited from previous insurers) or where the
oldest cohort is not fully developed. Past data may not be relevant and may therefore be
discarded; for example, changes in claim payment and case reserving practices.

We can still apply the chain ladder method where the data set is not complete. By adjusting
the way in which link ratios are calculated, we can allow for the missing information. For
example, where we have an incurred claims triangle missing all data prior to a certain
calendar year, we can derive development factors in a similar way to a full triangle using the
column sum average.

Paid versus incurred modelling


From a theoretical standpoint, it should be possible to fit a chain ladder pattern to paid
claims development or to incurred claims development and derive a similar reserve value,
as the paid and incurred claims should converge to the same ultimate claim amount. In
practice there may be a number of reasons why this does not turn out to be the case.
Examples of such reasons include, but are not limited to:

 Payments are made after claims are incurred, so for early development periods (that
is, those that are less developed) the paid claims data can be very sparse and hence
unreliable for projection, leading to potentially different paid and incurred
projections.

 Large complex claims usually take longer to settle and there may still be significant
case reserves at later development periods.

 Disputed claims may be slow to settle and are more subject to change.

 One pattern may be more volatile than the other, which makes projections difficult.

 At later development periods there may be only a small number of open claims
remaining, with all other remaining claims being fully settled. In such cases it may
appear, by looking at the paid claims development, that there will be no future
development and hence a paid link ratio model is likely to give a different answer to
an incurred link ratio model.

 Changes to case reserving procedures over time.

For these reasons, while it is helpful to consider projections based on both incurred and
paid link ratio models, we should use judgement to determine which is more appropriate to
use. In many cases it can be useful to undertake projections on both paid and incurred
data, and to compare and understand the differences in order to gain helpful insights and
select the most appropriate approach.

© IFE: 2019 Examinations The Actuarial Education Company


SP7-15: Triangulation methods Page 37

Question

Suggest which of paid or incurred claims should be used in each of the following situations.
(i) Outstanding reported claims reserves are always too low
(ii) Property business with very high excesses.
(iii) Reserves are consistently 10% too high.
(iv) Inwards reinsurance business.

Solution

(i) Paid, because reserves are unreliable as an indication of the future.


(ii) Incurred, because paid claims will often be zero or very high, and thus too volatile to
predict.
(iii) Either paid or incurred; incurred data would give a more prudent estimates.
(iv) Paid, because incurred claims will be distorted by the cedants’ reserving bases.

Unusually heavy / light experience


An individual year might experience particularly heavy or light experience. If the year is not
very developed, we may use the Bornhuetter-Ferguson method as an alternative.

This method will be discussed in Section 4.

However, if the year is more developed we should be careful not to over or underestimate
the IBNR required when we use a chain ladder method. This could arise if the claims
experience to date is either very low or very high, which, when multiplied by the relevant
development factor to ultimate, could cause us to under or overestimate the ultimate value.
Similarly, a year with heavy experience might contain claims that have reached policy limits
and so cannot deteriorate further. It would not be appropriate to apply a development factor
to such claims, so we should consider the impact of this.

Link ratios with values less than one


Since the data triangle to which a chain ladder method is applied is cumulative, claims
development patterns often increase over time; that is, the incurred claims continue to
increase until they reach their ultimate value.

However, depending on the class of business, the case reserving philosophy and the level
of reserves established, it is often possible that the development may initially increase to a
value greater than the ultimate level and then decline over time; for example, if conservative
case reserves are set up at the outset or if case reserves are held for claims which
subsequently settle for nil.

Another issue to consider is that of subrogation and recoveries. If a run-off triangle is


constructed net of recoveries and subrogation, we may expect link ratios of less than one
for certain periods.

The Actuarial Education Company © IFE: 2019 Examinations


Page 38 SP7-15: Triangulation methods

This can also be the case for non-fault claims such as cargo insurance and other lines where
recoveries are a key feature of claims development.

The Core Reading here uses ‘non-fault’ claims to mean claims where the liability is proved to
reside with someone other than the policyholder (ie it is someone else’s fault that the claim event
occurred). In these cases, the insurer can make recoveries from the third party and so sees a
redundancy in the reserves

Cargo insurance can also experience decreasing claim development due to salvage recoveries, for
example by selling cargo that has suffered water damage.

Question

Explain what subrogation recoveries are.

Solution

If an insurer indemnifies a policyholder against a loss, it may be entitled to attempt to recover


some or all of that loss from a third party.

For example, construction company X is insured against defects in its building work by its
insurance company. If a claim for defective work is made, and is paid by the insurance company,
and it subsequently turns out that the defective work was actually carried out by a subcontractor,
and not by X, the insurer may wish to attempt to recover some or all of the cost of the claim from
the subcontractor (or the subcontractor’s insurers).

We should consider the appropriateness of including link ratios with values less than one in
business that generally has an increasing cumulative pattern, or vice versa.

Pure IBNR
If claims are grouped by a reporting year cohort, the chain ladder projection will not allow
for pure IBNR, that is, for claims that are notified after the end of the reporting year. A
projection based on reporting year cohort data will only project claims that have been
reported to the insurer. It will therefore be necessary to estimate the pure IBNR claims
using other actuarial methods, if these are to be included within the scope of the review.

One way of doing this might be to use a simple proportion of a figure from the accounts. For
example, we might use a fixed proportion of earned premiums, or of reported claims. The
proportions to be used would need to be estimated from our past knowledge of the relevant
classes of business.

© IFE: 2019 Examinations The Actuarial Education Company


SP7-15: Triangulation methods Page 39

2.3 Key strengths and weaknesses of the chain ladder method

Strengths
Some key strengths of the chain ladder method are:

 the method can be applied to a wide variety of sets of data

 provided the data can be arranged into a development triangle, the chain ladder
method can be used to project it to ultimate

 the basic method can easily be modified to allow for data distortions

 the method is conceptually straightforward, and it is easy to relate results back to


the pattern of development

 the method can be developed to serve as a starting point for a number of other
methods, for example the Bornhuetter-Ferguson method.

Weaknesses
Some key weaknesses of the chain ladder method are:

 The results can be distorted by unusual experience (for example, very good or very
bad past claims experience).
– If this is the case then the pattern of development derived from the link ratios
suggested by past experience may not be appropriate. Furthermore the
projected ultimate for an individual year which has had unusual experience
may not be reasonable if we apply the chain ladder method with no
adjustments as the unusual experience is projected just as normal
experience.

 There is limited use for recent cohorts, particularly for long-tailed classes.
– Where classes are long-tailed (for example, certain liability classes) there
may be no fully developed cohorts available. Therefore we should estimate a
tail factor and apply it to the most developed cohort. We may base this tail
factor on the result of fitting a curve to the actual claims development, other
available information (for example, benchmarks) or we may use an altogether
different method to estimate the ultimate claims.
– It may be more appropriate to use alternative approaches, such as the
average cost per claim (ACPC) method (where claim number development
may run-off quicker than claim amount development), or the Bornhuetter-
Ferguson method which incorporates an expectation of the ultimate loss.
We will look at these methods later.
– The chain ladder method, on its own, projects claims experience to date,
which may be light or heavy. At the extreme, where there are no claims or a
series of large claims, it would be inappropriate for us to project without
adjustment or considering other methods.

 Considerable care is needed in applying the method to prevent unusual features in


the data having a significant impact on the results.
– For example, where a one-off large loss distorts the development patterns or
where there is a calendar year effect due to changes in claims handling
procedures.

The Actuarial Education Company © IFE: 2019 Examinations


Page 40 SP7-15: Triangulation methods

There are many other issues that can invalidate the assumptions behind the chain ladder.
However, the method is simple to use and can provide useful results. It can provide some
very useful insights into the data and is a generally accepted standard actuarial method.
However, we should not apply the method mechanically and should use judgement to
ensure that we obtain a reasonable result.

Use of judgement is discussed later in the course.

2.4 Methods related to the chain ladder method

The inflation adjusted chain ladder method


A full description of this method is included in the Core Reading for an earlier subject.

It may be worthwhile revisiting your notes from your earlier studies if you struggle to remember
any of the detail.

As we saw above, the basic chain ladder method assumes that future claims inflation will follow
the pattern of past claims inflation. This is not always a valid assumption. The inflation adjusted
method allows the actuary to make an explicit allowance for future inflation.

The difference between this method and the basic method is that an inflation index is
applied to the past claims data to bring it into line with the latest year, and to inflate the
projected claims to the expected year of payment. This method requires an appropriate
inflation index to be available for the business being considered. Use of the commonly
published indices such as for retail prices or for average earnings may not always be
suitable for the type of business being considered; for example, if the claims are subject to
legal inflation.

The choice of index is key to the success of reserving using this method, including changes
in inflation over time and choice of suitable benchmarks where data is sparse.

Explicit assumptions are made for both past and future claims inflation.

The assumption is therefore that, for each origin year, the expected amount of claims paid, in real
terms, in each development year is a constant proportion of the total claims, in real terms, from
that origin year.

The process is similar to the basic chain ladder except that there are many more calculations:
(1) The original claims data tabulated by origin / development year needs to be converted to
constant money terms, probably to the most recent origin year. For this we need to use
our best estimates of claims inflation for each prior year.
(2) After accumulating the claim amounts for each origin year, the chain ladder technique is
used on this modified table to estimate the claim amounts to be paid in each of the
outstanding origin / development years. These amounts will be in constant money terms,
as in (1).
(3) Disaccumulate the claim amounts estimates for each year. Then, future claims inflation is
used to convert amounts in each future origin / development cell to the appropriate
monetary amount for the year of payment.

© IFE: 2019 Examinations The Actuarial Education Company


SP7-15: Triangulation methods Page 41

The main difference from the basic chain ladder is that we are working in real money terms and
not in actual monetary amounts.

Question

Use the inflation adjusted chain ladder method to estimate the total outstanding claim payments
for the data on page 32. Assume that inflation for each of the last four years has been 6% pa.
Claims inflation is expected to be 2% pa for 2018/19 and 4% pa thereafter.

Solution

You should get an answer of about £373k. Some intermediate figures are given below. This was
based on a spreadsheet calculation, carrying many decimal places. You may have a slightly
different answer if you rounded intermediate results.

Note that if you calculate an answer using the incremental paid claims data you get £369k. The
basic chain ladder method is, however, defined (by convention) to work with cumulative data.
Hence, when using the IACL, you must complete the following stages:

(i) Derive incremental data from cumulative data (if necessary).

(ii) Adjust incremental data to constant money terms (if necessary).

This gives for this question:

0 1 2 3 4

2014 157.8 123.9 73.0 33.9 9.0


2015 142.9 110.1 60.4 27.0
2016 166.3 131.4 75.0
2017 151.6 110.0
2018 138.0

(iii) Form cumulative data, ie the sum to date along the rows:

0 1 2 3 4
2014 157.8 281.7 354.7 388.6 397.6
2015 142.9 253.0 313.5 340.5
2016 166.3 297.7 372.7
2017 151.6 261.6
2018 138.0

(These figures have been produced using unrounded numbers)

The Actuarial Education Company © IFE: 2019 Examinations


Page 42 SP7-15: Triangulation methods

(iv) Calculate chain ladder ratios:

0–1 1–2 2–3 3–4


1.7685 1.2504 1.0912 1.0232

(v) Apply chain ladder ratios to project future cumulative data:

0 1 2 3 4
2014
2015 348.3
2016 406.7 416.1
2017 327.1 356.9 365.2
2018 244.1 305.2 333.0 340.7

(vi) Derive incremental data from cumulative data:

0 1 2 3 4
2014
2015 7.9
2016 34.0 9.4
2017 65.5 29.8 8.3
2018 106.1 61.1 27.8 7.7

(vii) Project to allow for future inflation:

0 1 2 3 4
2014
2015 8.0
2016 34.7 10.0
2017 66.8 31.6 9.1
2018 108.2 64.8 30.7 8.8

Inflation is 2% for the first year and 4% thereafter.

(viii) Add up the numbers to give whatever estimate you require.

This gives £373k.

© IFE: 2019 Examinations The Actuarial Education Company


SP7-15: Triangulation methods Page 43

Berquist-Sherman method
The results of the basic chain ladder method may be inaccurate if there have been changes to the
speed of claim settlement over time.

Berquist and Sherman suggest techniques for adjusting historical development for changes
to the rate of settlement (ie payment) of claims and/or changes in case reserving adequacy.
These techniques produce adjusted development patterns that are consistent with the
current reserve adequacy levels and settlement rates that are thought to apply at the time of
the latest diagonal. We derive these adjusted development patterns by restating the
historical development data to be on the current reserving or settlement basis.

For example, we restate the paid claims in the triangle by examining the historical
incremental claims settlements and restating the paid data such that they are consistent
with the current estimated settlement rate. Similarly we restate the incurred claims by
assuming that the historical level of case reserve adequacy is the same as the current level
except for claims inflation trends. We can then build up a restated incurred triangle by
multiplying the restated average outstanding claim size by the number of open claims and
adding the paid triangles.

We need these adjustments to ensure that selected development patterns are not distorted
by shifts in the data. For example, if we believe the settlement rate has significantly
increased then by projecting unadjusted paid data we would overestimate the ultimate
value. This is because the unadjusted data will include a higher proportion of settled claims
than before.

Curve fitting
If the selected development factors are to be used to project expected future development
for use in actual versus expected comparisons, it is important that the development factors
in the tail follow a smooth pattern. Otherwise, the expected development can be volatile.

Using regression techniques, we can fit curves to premium or claim development data to
assist in either projecting development factors beyond the oldest development periods,
and/or deriving smooth development factors. We can fit a number of different distributions,
for example, Weibull, inverse power and exponential. This would involve estimating a best
fit to cohort development data. We can also use curve fitting to smooth development
patterns or to select a tail factor to allow for development beyond the oldest development
period.

You may remember the procedure for fitting a straight line to a set of data from your previous
studies. The same procedure (the method of least squares) can be used to fit a curve.
Alternatively we can carry out parameter estimation using maximum likelihood estimation or the
method of moments. Computer software exists to do this very easily.

When selecting the type of curve to be fitted it is important to select a curve with the
appropriate degree of tail heaviness for the type of claim being modelled.

Trend analysis
The pattern of development may change between cohorts and development periods over
time. It is possible to use various ad hoc methods to quantify these trends to increase the
accuracy of estimates.

The Actuarial Education Company © IFE: 2019 Examinations


Page 44 SP7-15: Triangulation methods

For example, we can fit curves from one distribution to each cohort and then analyse the
change in parameters to see if there are any trends. By carrying out a numerical review of
link ratios we may observe trends, for example, faster development of claims in more recent
cohorts.

© IFE: 2019 Examinations The Actuarial Education Company


SP7-15: Triangulation methods Page 45

3 Expected loss ratio method

3.1 Introduction
A loss ratio can be defined as the cost of claims per unit of exposure. A loss ratio method
requires a measure of exposure. One common example of an exposure measure used is
premium. Other possible exposure measures might be turnover or payroll.

Investigation of the loss ratios for several different origin years might show some
consistency, provided there have been no distorting effects; for example, no significant
change to premium rates, if premiums are used as the exposure measure.

A typical example of a distorting effect is a catastrophe. It should however be quite simple to


remove the claims related to a catastrophe from the claims amount used to calculate the loss
ratio.

If premium rates increase from one year to the next then, unless there is expected to be a
corresponding increase in the size or frequency of claims, the loss ratio is likely to reduce. The
insurance cycle is another good example of a factor that can have a distorting effect on the loss
ratio. The loss ratio will vary during the cycle as premium rates rise or fall.

The estimate of the claims reserve given by the method relies on the assumption that the
loss ratio is correct. This is a simplistic approach, but it does give a useful reference point
against which to compare other methods.

3.2 When to use the method


The method is simple to apply and is especially useful when the data is sparse, unreliable or
missing altogether – for example, if a company has started to write a line of business for the
first time. It is a simple way to check other methods.

Here is a very simple example. We are given the following data on earned premiums and claims
incurred from years 2013 to 2015 which are fully run off:

Year 2013 2014 2015


Earned premiums 2,450 2,855 2,947
Claims incurred 1,837 2,142 2,210

The earned premiums for 2016, 2017 and 2018 were 3,000, 3,156 and 3,297 respectively.

Question

(i) Calculate the loss ratio for years 2013 to 2015 and decide upon a sensible ratio to apply to
years 2016 to 2018.
(ii) Use this ratio to estimate the ultimate claims from 2016, 2017 and 2018.
(iii) If the claims paid to date in respect of premiums earned in these years amounted to
4,860, determine the total reserve required.

The Actuarial Education Company © IFE: 2019 Examinations


Page 46 SP7-15: Triangulation methods

Solution

(i) The loss ratios are:

Year 2013 2014 2015


Loss ratio 74.98% 75.03% 74.99%

75% seems to be appropriate.

(ii) This implies that the ultimate claims are:

2016 3,000  0.75 = 2,250


2017 3,156  0.75 = 2,367
2018 3,297  0.75 = 2,473
Total 7,090

(iii) The total reserve required in this case would be 2,230 (7,090 – 4,860).

3.3 Issues arising


This method relies on an assumption regarding the loss ratio for an origin period without
taking into account the claims experience (if any) to date for that origin period. The
selected loss ratio can be based on items such as:

 Benchmarks derived from data from other companies or industry / market data.

 Historical loss ratios. We will usually derive these from the specific company and
class of business being analysed, possibly allowing for any trends or outliers in the
observed historical ultimate loss ratios. If available, premium rate changes and
claims inflation should be used to adjust them, as well as undue heavy or light
claims experience.

 Underwriters’ opinion – for many classes of business, the underwriter may have a
prior view of the expected ultimate loss ratio for the account concerned. Human
bias should be considered though, as there may be a tendency to underestimate the
likelihood of writing unprofitable business.

 Changes in underwriting or expected changes in the claims environment. For


example, there may be an expected increase in weather-related catastrophes.

 Break-even loss ratio assumption (that is, set loss ratio to 100%, with or without an
adjustment for expenses / commission).

 Earnings pattern of the business written. For example, if we are using earned
premiums as an exposure measure, then we should consider the claims
environment to which those policies would be exposed. This could be different from
that for the unearned premiums.

© IFE: 2019 Examinations The Actuarial Education Company


SP7-15: Triangulation methods Page 47

 Target loss ratios in the company’s business plan.

 For lines of business where pricing models are robust and reliable (eg private motor
or household), the pricing models might be applied to the actual exposure and
compared to earned premiums to obtain an estimate of a loss ratio.
For example, we might take the estimated claims per unit of exposure from the pricing
model, and multiply this by the actual exposure from the business written. We could then
divide by the earned premiums to date, to derive a loss ratio estimate.

If we use the underwriter’s opinion, we should be aware that the resulting estimated
ultimate claims will be influenced (perhaps significantly) by that opinion. We should use the
break-even assumption with caution; perhaps only if the other options are not available.

We then apply this loss ratio to an exposure measure for that origin year. This is quite often
the estimated ultimate premium, although other measures are commonly used, such as
vehicle years for motor insurance business.

3.4 Strengths and weaknesses


An advantage of this method is that it is not distorted by anomalous data. This can
particularly have an impact for longer tailed business and at early durations if claims
experience to date is particularly light or heavy.

As noted above, it is also a simple method to apply and is especially useful when data is
sparse, unreliable or missing altogether.

This is true, provided that the assumptions made about the loss ratios are correct.

The disadvantages of this method are:

 It ignores the pattern of claims development to date.

 It is difficult to adjust for large claims.

 If loss ratios are derived from past years, the method will replicate any past biases.

 The benchmarks used may not be appropriate as the business written may be
different from that to which the benchmarks relate. For example, even though the
class of business may be the same, geographical coverage or policy conditions may
differ.

 Where used, the ultimate loss ratios (ULRs) for previous years may be understated
or overstated because of fluctuations in experience. Also any changes in the
underwriting or claims environment may make them unsuitable for use, although it
may be possible to make appropriate adjustments.

 The underlying assumptions can be subjective, particularly when based on soft


information such as the underwriter’s opinion. For this reason, we must aim to
understand the business being written and form a view on the reasonableness of the
underwriter’s assumptions.

 Where premium rate changes are introduced, these are often only for renewed
business and not for new business (which may be written at lower rates).

The Actuarial Education Company © IFE: 2019 Examinations


Page 48 SP7-15: Triangulation methods

4 Bornhuetter-Ferguson method

4.1 Introduction
We can think of the Bornhuetter-Ferguson method as a credibility estimate, based on a
weighted average of an expected level of claims (estimated by the loss ratio method) and a
projection of the ultimate claims based on experience to date (estimated by the chain ladder
method). It combines the advantages of the loss ratio method (for example, stability) with
the advantages of the chain ladder method (for example, incorporating information provided
by the claims development to date). We can equally apply the BF method to paid claims or
incurred claims.

You may remember something about credibility theory and the Bornhuetter-Ferguson method
from your earlier studies. We shall demonstrate some examples of how the method is used in
practice.

Application of the Bornhuetter-Ferguson method


A fuller description of this method is given in an earlier subject, but a quick summary, as
applied to an individual origin period, is as follows:
1. Determine an initial expected ultimate claim amount for the relevant origin period
(= LR, say). The loss ratio may typically be derived from the previous year’s best
estimate results, adjusted for claims inflation, rate changes, large claims and other
factors.
This was described in Section 3.3.
2. Estimate the proportion of the claims that are currently incurred (if using an incurred
Bornhuetter-Ferguson method) or paid (if using a paid Bornhuetter-Ferguson
method) for the relevant origin period (= p, say).
3. Derive the proportion of claims yet to be incurred or yet to be paid, by subtracting
the proportions in Step 2 above from one (that is, 1 – p ).
4. Determine the value of the claims that are currently undeveloped or unpaid by
multiplying the initial expected ultimate claims by the proportion of claims currently
undeveloped or not yet paid, as derived from Step 3. This is effectively the
estimated future incurred claims (ie, IBNR) or estimated future paid = LR x (1 – p ).
5. The final expected ultimate claims for the origin period is then the current value of
the incurred or paid claims (which will be given in the data) plus the expected value
of the undeveloped or unpaid claims, taken from Step 4 above = C + LR x (1 – p ).

We can equally apply the Bornhuetter-Ferguson method to paid claims or incurred claims,
as long as the data and development patterns are available.

The above description is on the assumption that we are using the method to derive an
estimated ultimate claim amount. We can equally use the method to derive an:

 estimated ultimate loss ratio (by combining the reserves estimated with an ultimate
premium)

 estimated ultimate number of claims (by using claim numbers in the approach
described above)

 estimated ultimate premium (by using premiums in the approach described above).
The expected premium may be the underwriter’s estimate.

© IFE: 2019 Examinations The Actuarial Education Company


SP7-15: Triangulation methods Page 49

We most commonly use the Bornhuetter-Ferguson method to derive estimated ultimate


claims or estimated ultimate loss ratios. The other two uses listed above are effectively a
form of ‘credibility’ method as opposed to a ‘pure’ Bornhuetter-Ferguson method.

For example, consider the very simple run-off triangle below where all claims are fully run-off
after two years. Our initial independent estimate (based on a loss ratio method) of the ultimate
claims from 2018 accidents is 120. This is not derived from the following table – it is independent.

Development year
0 1
2017 80 20
Accident year
2018 98

These are incremental figures.

The development of claims from 2017 suggests that 80% of claims will develop in year 0 and 20%
in year 1. So out of our initial estimate for 2018 we would expect 96 (80% of 120) to develop in
year 0 and 24 to develop in year 1.

The amount expected to have developed in the past can be compared with the actual amount
developed as indicated by the run-off triangle. The difference can then be used to revise the
estimate of the ultimate claim amount.

So in our example, the actual amount of claims developed in year 0 was 98 but our estimate was
only 96. We therefore increase our estimate of ultimate claims by 2 (9896), to 122.

Alternatively, our new ultimate claims amount is found by adding the claims expected to develop
in the future (from the initial estimate this is 24), to the known amount of claims developed in the
past (98). This gives the same answer as before: 122.

Make sure you understand why the two approaches give the same answer. Note our estimate of
the claims to develop in the future (24) has not been affected by the development of claims to
date from 2018.

So far, we have made an initial estimate of claims originating from 2018 and then used the
development of claims to date to revise this estimate. If our aim is to calculate the reserve
required at 31 December 2018 then we haven’t quite finished.

To calculate the reserve we need to deduct from the revised estimate, the amount of claims paid
to date in respect of claims originating in 2018. If the claims from each origin year are tabulated
in the projection table by year of payment (ie development year equals payment year), then the
claims paid to date is 98 and the reserve is 24. (Note that this could have been obtained directly
from the initial estimate of ultimate claims and the development pattern.)

However, if incurred claims or reported claims are used to tabulate the claims from each origin
year, then the amount of claims paid will have to be obtained from another source before the
reserve can be calculated.

The Actuarial Education Company © IFE: 2019 Examinations


Page 50 SP7-15: Triangulation methods

4.2 When to use the Bornhuetter-Ferguson method


The Bornhuetter-Ferguson method is very useful where the available data for the particular
cohort is sparse. This is often the case with more recent cohorts, cohorts from
longer-tailed portfolios (for example, liability excess of loss reinsurance) or where premium
volumes are so small that claims activity is expected to be extremely volatile.

It can also be used when a blend of experience and an exposure-based estimate is deemed
appropriate.

The Bornhuetter-Ferguson method is most effective when the current data is too immature
to be developed on a projection method, but we believe the experience data to date still
gives some indication of the level of ultimate claims.

In some cases, the data being projected may develop with a negative tail. In these
situations, we may consider the assumed percentage developed unsuitable as a weight in
this credibility method. We may use different weights, or where the incurred development
displays a negative tail, we may consider it is more appropriate to apply the
Bornhuetter-Ferguson method to the paid development.

In practice, it would be unusual to use the Bornhuetter-Ferguson method after the first few
development years, once the data is reasonably well developed. A chain ladder or case
estimate approach would be preferred at longer durations.

However, the Bornhuetter-Ferguson method may still be used at longer durations if the
development is very slow.

4.3 Strengths and weaknesses


The main strength of the Bornhuetter-Ferguson method is that it can be used when the
claims data is at a very early stage of development. In these circumstances, a pure chain
ladder projection method can produce a result that is either very high (if there happens to
have been material claims paid or incurred to date) or very low (if there happens to have
been limited claims paid or incurred to date).

At the extreme, where there have been no claims incurred to date, a projection method such
as the chain ladder will simply produce a zero ultimate. In contrast, the
Bornhuetter-Ferguson method produces a result that is less dependent on the claims
experience to date, since it partly relies on some previous, or a priori, estimate of ultimate
claims.

The main weakness of the Bornhuetter-Ferguson method is that it can be difficult to gather
the information for the prior estimate for the claims. The result, particularly at the early
stages of development, can be heavily dependent on this prior estimate.

In many situations, the actuary will apply both a chain ladder projection-based method and
a Bornhuetter-Ferguson method and either select one of the results from these two
approaches, or perhaps some form of weighted average of the results. Furthermore, we
would typically use a projection-based approach (or give it more weight) for the more
developed origin periods, and a Bornhuetter-Ferguson approach (or give it more weight) for
the less developed origin periods, as indicated in Section 4.2.

© IFE: 2019 Examinations The Actuarial Education Company


SP7-15: Triangulation methods Page 51

5 Average cost per claim (ACPC) method

5.1 Introduction
In this section we provide a brief description of the average cost per claim (ACPC) method.
We discuss its two main components – the claim frequency and the claim severity – when it
is most appropriate to use the method, and finally the advantages and weaknesses of the
method. The method is described fully in an earlier subject and a detailed description is not
repeated here.

We will include an example later on in any case to remind you of this approach.

The ACPC method is really a family of methods as it is not uniquely defined. Many
variations to the method are possible. This means that when we describe the ACPC method
that has been used, it is important to give a clear definition of the frequency and average
claim size used for the method. For example, we can apply the method to incurred, reported
or settled claims or pure IBNR claims. The average claim size can be for origin years,
development years or calendar years.

If incurred claims data is used, each cell in the claim development triangle will usually consist of
claims paid to date plus estimates of outstanding payments in respect of reported claims at that
date (usually from case estimation). Each claim amount should therefore correspond to the
number of claims reported at that date.

Sometimes, ‘incurred claims’ data may include an estimate for all outstanding claims (ie including
an estimate for IBNR). In this case the corresponding number of claims should also include an
allowance for IBNR claims.

Note also that some general insurance practitioners use the terms reported claims and incurred
claims interchangeably. Make sure you state in the exam exactly what you are taking to be
included in your definition.

The situation is complicated by partial payments and claims that are settled at no cost (zero
claims). Changes in the way either of these are dealt with may disturb the average cost per claim
method.

The number of claims table could contain various information. In this course, the number of
claims settled is used most often but the number of claims handled or reported can also be used
to ensure consistency with the format of the claims amounts data.

There is no single correct definition of ‘claims handled’. Definitions that may be used are:
(1) Number of claims settled in the year plus the number outstanding at the end of year.
(2) Number of reported claims outstanding at the start of the year plus number of claims
reported in the year. (This is the same as the first definition, assuming it does not
double-count partial settlements.)
(3) Number of claims fully or partially settled in the year (including zero claims).
(4) Number of claims fully or partially settled in the year (excluding zero claims).

The Actuarial Education Company © IFE: 2019 Examinations


Page 52 SP7-15: Triangulation methods

When we use the ACPC method, we estimate two separate components for each origin year:
the claims frequency, which relates to the number of claims, and the claims severity, which
relates to the average size of those claims.

If the severity used in the method is the average ultimate claim, we can calculate the
estimated ultimate claims by multiplying together the estimated ultimate number of claims
and the estimated average claim size. In this case, we can estimate the claims reserve by
subtracting the paid claims to date from the estimated ultimate claims.

If the average claim size used in the method is the average of future payments, we can
estimate the claims reserve directly by multiplying together the estimated number of future
claims and the estimated average claim size.

Let’s remind ourselves about this with an example.

This example works with incurred claims data (ie including estimates for the outstanding
payments in respect of reported claims). For example, in Table 1 below, the 280 in the first cell
represents the estimated total ultimate claims from accidents that occurred during 2015 as at
31 December 2015 (based on claims reported up to 31 December 2015).

To ensure correspondence with this data we must form averages using numbers of claims
reported, not claims handled. We have not attempted to model inflation explicitly.

Cumulative incurred claims data, by years of accident and reporting development:

Table 1

Development year
0 1 2 3
2015 280 375 431 446
Accident
2016 255 343 398
year
2017 248 323
2018 260

Cumulative number of reported claims, by years of accident and reporting development:

Table 2

Development year
0 1 2 3
2015 56 74 87 91
Accident
2016 49 65 77
year
2017 44 61
2018 50

© IFE: 2019 Examinations The Actuarial Education Company


SP7-15: Triangulation methods Page 53

Dividing each cell in Table 1 by the corresponding cell in Table 2 gives the average incurred cost
per claim, by years of accident and reporting development:

Table 3

Development year
0 1 2 3
2015 5.000 5.068 4.954 4.901
Accident
2016 5.204 5.277 5.169
year
2017 5.636 5.295
2018 5.200

Tables 2 and 3 can be completed using the chain ladder technique and are presented as Tables 4
and 5 below:

Table 4

Development year
0 1 2 3
2015 56 74 87 91
Accident
2016 49 65 77 80.54
year
2017 44 61 71.97 75.28

2018 50 67.11 79.18 82.83

Table 5

Development year
0 1 2 3
2015 5.000 5.068 4.954 4.901
Accident
2016 5.204 5.277 5.169 5.114
year
2017 5.636 5.295 5.181 5.126

2018 5.200 5.134 5.024 4.970

So the ultimate claims amount from accident years 2015 to 2018 is:

(91 4.901) + (80.5 5.114) + (75.3 5.126) + (82.8  4.970) = 1,655

As we are working with incurred claims and not paid claims an additional piece of information,
the amount of claims paid to date, is required before we can calculate the reserves.

The Actuarial Education Company © IFE: 2019 Examinations


Page 54 SP7-15: Triangulation methods

If the claims paid to date (from accident years 2015 to 2018) amounted to 1,323, the total reserve
required would be 332.

When we estimate the average claim size, we should ensure that the aggregate claim
amounts used in the numerator, as far as possible, correspond to the claim numbers used
in the denominator. If there is no direct correspondence between the claim numbers and
claim amounts, we may need to adjust the results.

An example of when we should be careful is when a claim is settled by several partial


payments. In theory, if the claim has been fully settled so that no further payments are
expected, we should not treat the individual partial payments as individual claims payments
and should aggregate them so that we can calculate the correct ultimate average cost of the
claim. If we treat the individual partial payments as individual claims, we will understate the
estimated average cost. In practice, in these circumstances, we may well choose a different
method, especially if a significant number of claims are in a state of partial payment.

The number of claims can be based on a frequency relating to an exposure measure such
as the policy count, premiums or sum insured.

Sometimes we use the ultimate claims derived from an aggregate method (for example,
chain ladder) to estimate the average claim amount for the ACPC method by dividing the
ultimate by the estimated number of claims for the origin period. The averages for more
developed origin years are then trended and used for more recent origin periods. We
should compare the average claim size derived with the average claim sizes in the past data
to check that they are consistent.

When we use the ACPC method, we can apply other methods such as the chain ladder and
the Bornhuetter-Ferguson method as part of the analysis. For example, when modelling
claim numbers, we may use the chain ladder method and the Bornhuetter-Ferguson method
to estimate the ultimate number of claims for the most recent origin year(s). Conversely, we
may use the ACPC method as part of other methods. For example, we could use an ultimate
from the ACPC method as a prior for the Bornhuetter-Ferguson method.

When we use the chain ladder method, it is necessary to consider whether it is appropriate
to use development patterns derived from more developed origin years to project the less
developed years. Similarly when we use the ACPC method, it is necessary to consider
whether the claim frequencies and average claim sizes developed from past data are
appropriate to use for projecting ultimate claims.

5.2 When to use the ACPC method


The ACPC method is useful where there are features in the data that aggregate modelling
methods will not detect or model properly. For example, claims inflation may arise from an
escalation of settlements amounts or an increase in claims frequency; aggregate models
cannot distinguish between these two sources of inflation.

We need more data for the ACPC method than for some other methods, such as the chain
ladder method. Therefore, we can only apply the method when appropriate data is
available. In particular, we need information on claim numbers in addition to claim amounts
information.

We should not use the method where the claim count or an average claim size is not
meaningful. An example of such a situation would be in a subscription market where
insurers write different shares on each of the policies that they underwrite (unless an
adjustment can be made to allow for this).

© IFE: 2019 Examinations The Actuarial Education Company


SP7-15: Triangulation methods Page 55

5.3 Issues arising

Claim numbers
We should take into account the following considerations when we choose the appropriate
claim numbers for the ACPC method:

 Reported claims: projecting reported claim numbers can provide a short-tailed


projection of the ultimate numbers of claims, as claims are reported more quickly
than they are settled. Ideally we should use a consistent definition of a ‘reported
claim’. If claims reserving protocols have changed over time this may not be
possible.

 Nil claims: Claim numbers can either include or exclude nil claims. If they include
nil claims, it will be important to consider whether their impact varies between the
origin years in a way that might affect either the development of the claim numbers
or the overall claims frequency.
For example, the pattern of claims reported by policyholders may change because of
a change in mix of business or policy terms, or nil claims may be recorded
differently because of a change in an insurer’s claims recording approach.
Excluding nil claims from the claim numbers may help avoid the potential
distortions to claim frequencies and claim severities which may arise.

 Settled claims: If the projection of the ultimate number of claims is based on the
development of the number of settled claims it is important to consider how to
define when a claim is settled.
For example, the settlement date could be the date the file is flagged as closed on
the system, or the date of last payment. Depending upon the claims settlement
process, one of these definitions may be more appropriate to use than another.

 Reopened claims: it is important to define carefully how reopened claims are treated
in order that reopened claims cannot distort the number of settled claims and hence
the average cost per claim.

We note that the definition of claim numbers affects the average claim sizes. For example,
whether nil claims are included or excluded will affect the selected average claim sizes (all
other things being equal). It is of utmost importance to ensure that the claim frequency and
claim severity are consistent.

Claim amounts
We can apply the ACPC method to data adjusted for claims inflation.

We can adjust the incremental claim payments (for each cohort and development period)
from the historical data by an appropriate claims inflation index, either derived from the data
or from an external index. From this, we can derive an average cost per claim for each
origin year in current values. We then multiply future projected incremental average claim
amounts by an appropriate future inflation index to estimate future inflation adjusted
average claim amounts.

The inflation index could be based on general economic inflation, based on wages or prices,
and a ‘superimposed’ inflation rate which would capture any tendency for claim payments
to increase at a higher rate than general economic inflation; for example, due to legislative
changes.

The Actuarial Education Company © IFE: 2019 Examinations


Page 56 SP7-15: Triangulation methods

Whether we use data adjusted for inflation or not, when we select the average claim sizes
we should consider the allowance for inflation, in particular whether the inflation rate varies
by origin year or calendar year.

Technical considerations
There are many technical considerations to take into account when applying an ACPC
method. We may adjust either the claim frequency or severity, or both. Examples of such
considerations are:

 As with other projection techniques, we should allow for large claims that may or
may not have been notified or recognised as being large. We could adjust the data
for the large losses by either excluding them or capping the triangles with a large
loss threshold to derive an attritional claims triangle. We could then apply the ACPC
method to the attritional triangle to derive estimated claims frequencies and
severities for attritional claims, and then use a separate model for the excluded large
losses or claims in excess of the large loss threshold.

 As mentioned above, we can make an explicit assumption for past and future
inflation which can be applied to the claim amounts.

 We may need to allow for changes in the mix of business or policy terms and
conditions as these changes will distort patterns and trends observed in the data.

 As with other actuarial projection techniques, we may need to modify the ACPC
model to take into account changes in the legal, economic, social environment, or
company operation, which may have taken place or are expected to take place in the
future.

 The method can also be influenced by acceleration or slowing in claims payments or


settlements. It may be necessary to make adjustments to allow for these factors.
The considerations are similar to those described earlier in connection with the
Berquist-Sherman method.

 We can select the average claim size with reference to industry benchmarks (for
example, relevant industry studies / databases) if appropriate.

 The method can introduce cross-subsidies between origin years. For example, if a
trended average claim amount is used.

Other more general considerations were detailed in Section 1.6.

© IFE: 2019 Examinations The Actuarial Education Company


SP7-15: Triangulation methods Page 57

5.4 Strengths and weaknesses

Strengths
 Easy to understand and communicate.

 Provides information about how both claim numbers and claim amounts are
expected to develop in the future.

 For direct business in particular, the data required is generally available.

 Can be used in conjunction with other projection techniques, such as the chain
ladder and Bornhuetter-Ferguson.

 Helps to explain volatile data and results when the data contains only a small
number of claims, for example, for very small lines of business or reinsurance
losses.

 Can be applied to settled claims when claims reserving protocols have changed
over the development history, making some other methods invalid.

 If used in the correct way, it can be useful as a basis for estimating latent claims
(which are claims that are reported a significant time after occurrence) because we
can make explicit assumptions about the average claim size, the long-term effect of
inflation and the expected number of claims.

 Enables more accurate adjustments to be made (where these only affect the
frequency or the severity of claims) which would not be possible with other
(aggregate) methods.

Weaknesses
 Can be distorted by reopened claims, nil claims or partial payments.

 Assumes that the distribution of claims is the same for each origin year or
settlement year (for example, if the frequency increases it may be the case that this
is because of a new type of claim which has a different average claim size to the
past data).

 Needs more detailed information, which may not always be available.

 Small data samples may lead to volatile results; although this is in common with
other projection techniques.

The Actuarial Education Company © IFE: 2019 Examinations


Page 58 SP7-15: Triangulation methods

Question (long, but worthwhile)

Given the following data, estimate the claims outstanding as at 31 December 2018 using:
(i) the basic chain ladder method
(ii) the inflation adjusted chain ladder method
(iii) the Bornhuetter-Ferguson method.

Cumulative paid Development year


claims 1 2 3 4 5 ULT
2014 29,791 52,054 54,890 55,267 55,345 55,345
2015 28,620 31,130 48,885 57,141
Accident
2016 27,935 40,060 49,962
year
2017 37,661 47,683
2018 19,619

Inflation
2014/15 10.0%
2015/16 10.0%
2016/17 10.0%
2017/18 10.0%
Future inflation rate pa 10.0%

Benchmark year 2014 2015 2016 2017 2018


Earned premium 63,907 74,913 71,715 76,802 54,903
Ultimate loss ratio 90.0% 90.0% 90.0% 90.0% 90.0%

© IFE: 2019 Examinations The Actuarial Education Company


SP7-15: Triangulation methods Page 59

Solution

(i) Basic chain ladder

The essential figures are:

Link ratios 1.3784 1.2474 1.0832 1.0014 1


Cumulative dev factors 1.8651 1.3531 1.0847 1.0014 1

Estimated claims
Year Ultimate Outstanding
2014 55,345 0
2015 57,222 81
2016 54,195 4,233
2017 64,520 16,837
2018 36,591 16,972
Outstanding claims reserve 38,123

(ii) Inflation adjusted chain ladder

Cumulative Development year


paid claims 1 2 3 4 5 Ult
2014 29,791 52,054 54,890 55,267 55,345 55,345
2015 28,620 31,130 48,885 57,141
2016 27,935 40,060 49,962
2017 37,661 47,683
2018 19,619

Incremental Development year


paid claims: 1 2 3 4 5
2014 29,791 22,263 2,835 378 78
2015 28,620 2,510 17,755 8,256
2016 27,935 12,125 9,902
2017 37,661 10,022
2018 19,619

The Actuarial Education Company © IFE: 2019 Examinations


Page 60 SP7-15: Triangulation methods

Inflated Development year


incremental
values: 1 2 3 4 5

2014 43,617 29,632 3,430 416 78


2015 38,093 3,037 19,530 8,256
2016 33,801 13,338 9,902
2017 41,427 10,022
2018 19,619

Chain ladder applied to cumulative inflated amounts:

Development year
Projection
1 2 3 4 5
2014 43,617 73,249 76,679 77,095 77,173
2015 38,093 41,130 60,661 68,917 68,987
2016 33,801 47,139 57,041 60,642 60,704
2017 41,427 51,449 61,917 65,827 65,903
2018 19,619 26,623 32,040 34,063 34,098
Ratios 1.3570 1.203 1.0631 1.0010

Projected incremental amounts (monetary values as at 31 December 2018):

Development year
Projection
1 2 3 4 5
2015 70
2016 3,602 61
2017 10,468 3,910 67
2018 7,004 5,417 2,023 35

© IFE: 2019 Examinations The Actuarial Education Company


SP7-15: Triangulation methods Page 61

Projected incremental amounts based on future inflation assumption:

Development year
Projection
1 2 3 4 5 O/S
2015 77 77
2016 3,962 74 4,036
2017 11,515 4,730 89 16,334
2018 7,705 6,554 2,693 51 17,003
Outstanding claims reserve 37,450

(iii) Bornhuetter-Ferguson method

Year 2018 2017 2016 2015


Independent ultimate 49,413 69,122 64,544 67,422
Ppn claims paid to date 0.5362 0.7390 0.9219 0.9986
Ppn claims O/S 0.4638 0.2610 0.0781 0.0014
O/S claims 22,919 18,038 5,041 95
Outstanding claims reserves 46,093

The Actuarial Education Company © IFE: 2019 Examinations


Page 62 SP7-15: Triangulation methods

6 Comparison of results from different methods


It is generally advisable, if data permits, to use more than one method to project reserves.
This may involve a combination of paid and incurred methods, chain ladder and
Bornhuetter-Ferguson, or ACPC.

6.1 Factors to be considered if results from different methods diverge


One of the most important considerations in assessing the results from different reserving
methods is to understand what is driving the outcome from each method. For example:

 What properties of the claims experience is each method modelling?

 Are there specific data points that are driving the divergence?

 Are the trends across cohorts for each method sensible?

Some issues to look out for are where the paid projections are consistently higher than the
incurred projections, or where an incurred projection gives an ultimate that is less than the
paid to date (in the absence of recoveries).

The final result chosen may be the method that best reflects the underlying behaviour of
claims. Alternatively, some kind of weighted average may been taken, either ad hoc or
using a statistical approach, as in the Benktander method (named after its original
proponent). The Benktander method is a weighted average of the paid chain ladder and
paid Bornhuetter-Ferguson methods, with the weighting factor for the paid chain ladder
method being the proportion of ultimate claims paid. Thomas Mack (Credible Claims
Reserves: The Benktander Method, ASTIN Vol 30 No 2) demonstrated that under certain
conditions, the Benktander method gives a lower mean square error than either of the
component methods.

The Benktander Method is also called the Iterated Bornhuetter-Ferguson method, and is not very
commonly used.

© IFE: 2019 Examinations The Actuarial Education Company


SP7-15: Triangulation methods Page 63

7 Reserves for unexpired policies


In this section we look at the reserves that a company should hold at the year-end in respect of
the unexpired risk of policies that are currently in force. Given that most general insurance
policies provide cover for one year, almost any policy written during a year will have an unexpired
portion at the end of the year.

We will consider the unexpired risk in two different ways:


(a) A retrospective approach: how much of the premiums that were charged should we be
holding in respect of the unexpired risk? This will give us the unearned premium reserve
(UPR).
(b) A prospective approach: how much is needed now to cover the expected claims and
expenses from the unexpired risk? This will give us the unexpired risk reserve (URR).

7.1 The unearned premium reserve (UPR)


When accounting in the one-year accounting format, the exposure to risk under general insurance
policies usually falls into more than one accounting period. A proportion of the premiums
received in a given financial year is therefore reserved to meet the liability arising from the
exposure after the accounting year end.

Precise calculation of UPR


Let’s start by looking at a single policy. Our aim is to calculate the UPR as at 31 December 2018,
the year end. To do this we need to determine what the premium is supposed to cover, and
when precisely the premium is earned.

The premium covers:


 claim payments, which are incurred in line with the incidence of risk (which may not be
uniform over the life of the policy)
 initial expenses (ie commission, documentation, other set-up costs), which are incurred at
the start of the policy life
 other expenses, which will mainly be related to claims, and will therefore be incurred with
the incidence of risk
 profit, which arguably should also be earned in line with the incidence of the risk.

Therefore, apart from the initial expenses, which we shall call acquisition costs from here on, all
the elements of the premium are earned with the incidence of risk. We defer discussion of
acquisition costs until Section 7.6 and concentrate for now on the risk-related premium.

The Actuarial Education Company © IFE: 2019 Examinations


Page 64 SP7-15: Triangulation methods

If the policy under consideration has m% of the risk before the year end and (100-m)% of the risk
after the year end, then (100-m)% of the risk related premium is unearned at the year end.

earned unearned
m% of risk (100-m)% of risk

31/12/2018 31/12/2019

The UPR for this policy could then be calculated as:


(100 – m)%  premium

For the whole portfolio, the UPR is the sum of these amounts over all policies. However, this is
not all that practical if the pattern of risk is complex or unknown.

In practice, policies do not come with little tags specifying what proportion of the risk has elapsed
by the year-end. It is normal to base the split on elapsed time, unless this is clearly inappropriate
because the risk is not uniformly spread over the year, for example where the insurance is heavily
weather related.

7.2 Risks evenly spread


When the risk is evenly spread, proportionate methods of calculating the UPR may be used for
annual policies. These depend on the grouping of the base dates, for example daily or monthly.

365ths method
This is the most accurate method that can be used. It calculates the unearned premium by
multiplying the office premium by the ratio:
(365 – Number of days since inception) / 365

For example, a policy incepting on 6 October will have earned 87/365ths by 31 December, so the
unearned portion is 278/365ths of the office premium for the policy.

Unless the portfolio is small this method requires computerised records, because of the volume of
transactions and the fact that a separate calculation is needed for each policy.

24ths method
A similar method, with a slightly less accurate result, takes account of the month, rather than day,
of inception. It requires an assumption that every policy is incepted in the middle of the month.
Thus the unearned premium for an annual policy written in month m of the accounting period is:
((2m  1) / 24)  P
where P is the annual gross premium.

Other methods
An eighths method and a halves method also exist.

© IFE: 2019 Examinations The Actuarial Education Company


SP7-15: Triangulation methods Page 65

Question

(i) State the assumption used for (a) the eighths method and (b) the halves method.

(ii) Calculate the UPR (as a fraction of the office premium) for the 6 October policy using the
24ths method.

Solution

(i)(a) Policies incept on average at the mid-point of the quarter.

(i)(b) Policies incept on average at the mid-point of the year.

(ii) Assume the 6 October policy incepts mid-way through October and by the year-end
would be 5/24ths earned. Hence the UPR would be 19/24ths of the office premium.

7.3 Uneven risks


When calculating the UPR for policies where the risks are not borne evenly over the period of the
policy there is not a single correct formula or method and the calculation will depend on the
characteristics of the risk involved. The progression of the incidence and the size of the risk will
have to be analysed. Thereafter a suitable formula can be developed. Examples of uneven risks
are the effects of inflation and annual travel policies where the claim propensity rises in the
summer months.

Question

An annual policy starts on 1 August. The risk in the first half of the policy is three times higher
than the risk in the second half of the policy. Throughout each half year, the risk is evenly spread.
Calculate, as a percentage of the office premium, the UPR at the end of the year.

Solution

Three quarters of the premium is earned in the first six months of the policy.

At the year end the policy is 5 months old and so the earned premium is 5/6ths of three quarters:
5/8ths.

The UPR is therefore 3/8ths of the office premium.

7.4 Treatment of non-annual premiums


The above description relates to policies where the premium is paid annually at the inception or
renewal of the policy. Where premium payments are made differently or more frequently the
same principles apply. The basic methods simply require an adjustment to allow for the length of
the period to which the premium relates. For covers running for less than a year, it is only those
in force at the accounting date that give rise to a UPR.

The Actuarial Education Company © IFE: 2019 Examinations


Page 66 SP7-15: Triangulation methods

Question

(i) Suggest, and justify, how UPRs might be decided for mortgage indemnity business.

(ii) Explain why using the 24ths method may not give the best estimate of the UPR.

Solution

(i) Mortgage indemnity business

For mortgage guarantee, the single premium potentially covers a term of, say 25 years, although
the risk will probably be minimal after ten years (or less).

The insurer would need to make a decision on the incidence of risk, and spread the premium over
the policy term accordingly. Risk may plausibly rise for the first few years before it, later, tails off
to zero. The risk is closely linked to the economic cycle. The UPR should cover the balance of risk
remaining at the time.

(ii) 24ths method

 on average, policies may not be written in the middle of the month


 policies may not be for 12 months, eg two week travel insurance policies
 risk may not be spread evenly over the year due to inflationary or seasonal effects
 computerised information may permit a more accurate calculation

7.5 Choice of base dates


We have generally assumed that the premium is received the day that the insurer goes on risk. In
reality, premiums may be paid in instalments or there may be a delay of several months before
the insurer receives the premium from the broker. There may also be a delay before the insurer
is notified about the policy.

In grouping the base dates to which a formula as developed above is applied, the insurer will need
to decide which entry date is used. The possibilities are:
 the date on which the premium is paid or debited
 the date on which cover starts
 the inception (or renewal) date of the policy.

Each date could be different. For example, an annual policy may be incepted then subsequently
renewed on 1 July of each year; premiums may be paid (eg by direct debit) on the 25th of each
month; but in the first year there may be a waiting period (during which no claims can be made 
rarely used in general insurance, except possibly in personal accident or creditor insurance
policies) so cover would not start until 1 September in that year.

© IFE: 2019 Examinations The Actuarial Education Company


SP7-15: Triangulation methods Page 67

Whichever method is used there must be consistency between:


 the assumption regarding the period to which the UPR is apportioned
 the credit of the (full) premium received
 the credit or debit for outstanding or overpaid premiums respectively.

For example, consider the reserves calculated at the year end for a policy that commenced on
15 December but for which premiums were not received until 5 January. The UPR may be
calculated with regard to the policy year, ie take 23/24ths of the annual premium as the reserve.
However, if we do this, we must give credit in the accounts for the premium that we expect to
receive.

7.6 Allowance for deferred acquisition costs (DAC) in the UPR


The above description of the UPR made no allowance for the fact that the office premium
includes allowance for expenses that are unlikely to be evenly spread over the policy term. In
reality the majority of expenses are likely to be incurred at the inception of the policy (ie they are
acquisition expenses).

Two approaches exist to allow for the acquisition costs in the UPR on an ongoing basis:
 the UPR is directly reduced for acquisition expenses, ie the acquisition expenses are
deducted before the UPR calculation is applied to the office premium. This is usually
called ‘a UPR net of DAC’.
 the UPR is not directly reduced for acquisition expenses, but a Deferred Acquisition Cost
(DAC) is set up as an asset, which then reduces over the exposure period of the policy.
This type of UPR is called ‘a UPR gross of DAC’.

On a wind-up basis, the insurer’s liability would normally be the full unearned proportion of the
office premium, hence the UPR should not be reduced for acquisition expenses and a balancing
asset should not be included (unless there would be some clawback of commission).

On a wind-up an insurer will have to refund the unearned proportion of the premium to the
policyholder. The liability should therefore be the UPR gross of DAC unless the insurer is also able
to reclaim some commission from the broker.

If acquisition costs are to be taken account of, whether directly or indirectly as a deferred
acquisition cost asset, the reduced UPR needs to be calculated by reducing the office premium
proportionately for the acquisition costs.

As a simple example, if the acquisition costs amounted to 20% of the written premium, which had
been written uniformly over the year, the UPR could be calculated as:

50%  80%P  40%P

This method is commonly used as a rough approximation for UPR.

Profit margins, claim handling costs and expenses of servicing the business are implicitly assumed
to be spread in the same way as the risk.

The Actuarial Education Company © IFE: 2019 Examinations


Page 68 SP7-15: Triangulation methods

Question

Suggest a formula for the UPR of a contract, t days after the purchase of the policy (0 < t < 1825),
taken out on the purchase of an item. The contract extends a one-year warranty for another four
years.

Assuming that the risk is constant during each year, and that risk doubles on each policy
anniversary (commencing at the end of year 2), calculate the UPR for a contract with two years to
run.

Solution

This is a contract with risk zero for the first year, followed by four years on risk. Assume a single
premium, P , is paid at time t  0 . If AC is the amount of the acquisition costs and the risk is even
over the four years of cover (and it probably isn’t), the UPR would be:

1  AC   P for t  365

 5  365  t 
1  AC   P  for t > 365 (ignoring leap years)
4  365

If risk doubles each year, there are 12 out of 15 units of risk remaining with two years to run, so
UPR is:
12
1  AC   P
15

7.7 The additional reserve for unexpired risks (AURR)

What is an AURR?
We have considered the unexpired policies on a retrospective basis, ie the proportion of
premiums held as a reserve at the year end. We will now look at the unexpired policies on a
prospective basis, ie how much we think we need to cover the claims and expenses from the
unexpired policies.

This prospective amount is called the unexpired risk reserve or URR. Having assessed the URR, we
then compare it with the UPR net of deferred acquisition costs (which is the amount of money we
should put by using a retrospective approach). We might have:

(a) UPR > URR


This is what we would hope to see. It implies that premiums are greater than expected
claims and expenses, so we would expect a profit to emerge in the coming months from
the unexpired risks. We would hold a reserve equal to the UPR.

(b) UPR = URR


Most unlikely. Someone has been fiddling the calculations.

© IFE: 2019 Examinations The Actuarial Education Company


SP7-15: Triangulation methods Page 69

(c) UPR < URR

If the estimated cost of claims arising from unexpired risks is greater than the unearned
premium reserve, then an AURR is required if full provision is to be made for future
liabilities. It should be noted that the future liabilities should include unallocated loss
adjustment expenses (ULAE) and any allowed or required discounting.

There are, of course, many reasons why the original premium could be insufficient,
including higher than expected levels of claim frequency, claims inflation and expenses.

There has been some confusion of terminology in the insurance world as to whether unexpired
risk reserve means the total unexpired risk (URR here) or the additional bit in excess of the UPR
(the AURR here). Be careful when you come across the expression. These notes usually use URR
to be the total and AURR to be the additional amount.

Effect of AURR on the accounts


Where an additional reserve is held the emergence of profit between accounting periods will
be different from what it would be if no additional reserve were held. To hold an additional
reserve will defer the emergence of profit and will anticipate a loss when business has been
put on the books at unprofitable premium rates.

Question

An insurance company accounts for all its business on a quarterly basis. The amounts of premium
written for each quarter of the calendar year are:
Q1: £150m
Q2: £170m
Q3: £190m
Q4: £180m

In September, the company carried out a review of its expected claim experience for the year’s
business, and concluded that for business written to date the loss ratio for the year would be
120%. As a result of this review, the company doubled all its premium rates, the increases taking
effect on 1 October.

Calculate the UPR and estimate the AURR as at close of play at the end of the year. Acquisition
costs are 25% of premium. State all assumptions you make.

The Actuarial Education Company © IFE: 2019 Examinations


Page 70 SP7-15: Triangulation methods

Solution

We need a number of assumptions:

(a) That the assessment of AURR is carried out for the total of unexpired risks. (We could
alternatively have had a quarterly basis, which would give a bigger result since the
‘negative’ AURR for the final quarter would not be netted off against the positive AURRs
for the others.)

(b) Policies are annual and risk is even over each policy year.

(c) Policies commence mid-quarter on average.

(d) As usual the loss ratios apply to earned premium. Assume they also include allowance for
handling expenses, ie here:
claims incurred, including handling expenses
loss ratio =
premiums earned

(e) Loss ratios continue throughout the remainder of the period of cover, so are 120% for
business written in Q1, Q2, Q3 and 60% for business written in Q4.

Unearned premium Expected claims


Written UPR
(gross of DAC) from year’s
premiums (net of DAC)
at year end exposure
Q1 150 14.1 18.75 22.5
Q2 170 47.8 63.75 76.5
Q3 190 89.1 118.75 142.5
Q4 180 118.1 157.50 94.5
Total 690 269.1 358.75 336.0

AURR is 336.0 – 269.1 = 66.9

As with many general insurance questions, we could make different valid assumptions and get a
different valid answer.

There can sometimes be considerable uncertainty as to the need for, and the possible size
of, an additional reserve for unexpired risk.

For example, one class of business that may exhibit extreme variation in the scale of
reserves for unexpired risk is mortgage indemnity guarantee insurance, for which the
experience depends heavily on changes in economic conditions.

© IFE: 2019 Examinations The Actuarial Education Company


SP7-15: Triangulation methods Page 71

One question that may need to be settled is the extent to which classes of business should
be grouped together to assess whether such a reserve is required, ie anticipated future
profits from some categories of business may offset potential inadequate premiums in other
categories. This may be specified in accounting or regulatory requirements and may be
defined in terms of homogeneous groups.

The Actuarial Education Company © IFE: 2019 Examinations


Page 72 SP7-15: Triangulation methods

8 Recoveries

8.1 Types of recoveries


Most of the discussion on reserving focuses on gross reserves. However, the reserves for
solvency and published accounts are required to be presented net of recoveries.

Recoveries may generally be classified into salvage and subrogation, and reinsurance.

Salvage and subrogation recoveries arise when the insurer is able to recover some of the
cost of a claim from another party. Examples include the sale for scrap or parts of motor
vehicles written off (salvage), and recoveries from other parties to whom some liability for
the claim can be attributed (subrogation).

Trade credit insurance is an example of a class for which subrogation recoveries may be material,
eg debt obligations of the defaulting party (or underlying collateral) may be partially recoverable.

Engineering and plant machinery insurance is an example of a class where salvage recoveries are
likely to be material, eg heavily damaged material will have a scrap value and partially damaged
material may still be usable for other purposes.

Employers’ liability is an example of a class where salvage and subrogation recoveries are not
likely to be material.

8.2 Salvage and subrogation recoveries


It is important to understand whether the data provided is gross or net of salvage and
subrogation recoveries.

Wherever possible, salvage and subrogation recoveries should be projected separately


from the gross amounts, as the pattern of recoveries will not match the gross payment
patterns.

If such recoveries are not projected separately, we may end up either over-reserving or
under-reserving, depending on how the pattern for recoveries differs from the pattern for gross
amounts.

Insurers may or may not establish case estimates for salvage and subrogation recoveries.
Where case estimates are not maintained, payment-based methods will need to be used.

Methods based on reported incurred claims cannot be used because the reported incurred data
will not properly reflect the salvage and subrogation recoveries, if case estimates for these are
not included.

The considerations are similar to considerations for gross reserves. Where case estimates
are maintained, incurred analyses may be carried out in addition to paid analysis.

It is often helpful to carry out analyses on both a paid and an incurred basis. If the results are very
different, we will want to investigate why. For some classes, paid claims can be volatile so more
emphasis may be given to the incurred analysis. On the other hand, the reported incurred
amounts are affected by the prudence of reserving, so the paid analysis can provide an objective
comparator.

© IFE: 2019 Examinations The Actuarial Education Company


SP7-15: Triangulation methods Page 73

Marine Cargo is a class that commonly has salvage recoveries.

8.3 Reinsurance recoveries


Depending upon the reinsurance program(s), reinsurance recoveries can represent a
significant proportion of the gross reserves. Where the reinsurance is proportional, the
estimation of reinsurance recoveries is relatively straightforward, as the net reserves
should be proportionate to the gross reserves. However, where the reinsurance is excess
of loss, or a mixture of excess of loss and proportional, the estimation of net reserves is
more complex.

Although it is possible to set up and analyse net (of reinsurance) triangles, it is generally
recommended that ceded reserves be calculated separately and subtracted from gross
reserves to estimate net reserves. There are two main reasons for this:

 Reinsurance programs can change from year to year. While this causes similar
problems to those induced by changes in the gross claims experience or policy
terms and conditions, the impact on net data is more severe and is difficult to model.

 Where the reinsurance represents a relatively small proportion of the reserves, the
use of a net analysis may give reserves that are not consistent with the gross
reserves.

The first problem can possibly be dealt with by calculating what the net payments on each
claim would have been under each reinsurance arrangement, and reconstructing different
sets of net triangles. However, this is laborious and is only practicable if the actuary has
access to all individual claim payments and estimates.

Frequently there will be insufficient reinsurance claims data to give triangles from which
projections can be made. For example, where the reinsurance is excess of loss, these
triangles may consist mainly of zeroes.

Lots of zeroes in the triangle can clearly lead to problems when calculating development factors,
and it is unlikely that there will be a consistent pattern for all years of origin (as required for the
chain ladder method).

The main methods for dealing with this problem are:

 If there is a benchmark loss emergence pattern, and an indicative loss ratio, it may
be possible to use the Bornhuetter-Ferguson method. This method would likely
need to be used for a greater number of years than it would for gross reserves.
We tend to use the Bornhuetter-Ferguson method for less developed years.
Development patterns for reinsurance business are generally longer than those for the
corresponding direct business, so there will be a greater number of ‘less developed’ origin
years.

 Where large losses are projected separately, and particularly if the large-loss
threshold is below the reinsurance retention, the reinsurance program can be
applied specifically to each large loss, factoring in IBNER as necessary. It is then
necessary to allow for IBNR large losses. An allowance for these may be based on
the reinsurance modelling carried out for the capital modelling purposes.

The Actuarial Education Company © IFE: 2019 Examinations


Page 74 SP7-15: Triangulation methods

 A simpler approach is to use reinsurance to gross ratios. The reinsurance to gross


ratios for premiums, paid claims, incurred claims and outstanding claims are
calculated for each cohort, and a choice is made of a reinsurance to gross ratio for
the IBNR. Then once the gross IBNR has been projected the reinsurance IBNR can
be calculated.

 It is also possible to estimate the reinsurance recoveries using stochastic methods,


whereby gross claims are projected stochastically and the reinsurance program is
applied.

When applying the reinsurance program to individual gross losses, it is important to


consider whether the attachment point and/or limit are indexed. This has particular
relevance for large bodily injury claims, which may take many years to settle. Claims that
are settled later will have reinsurance recoveries eroded by the indexing of the attachment
point. This can be dealt with stochastically, but it is usually assumed that there is an
average time to settlement, and the attachment point is indexed by a corresponding
amount.

One important test of the reinsurance calculations is to compare the gross and reinsurance
ultimate loss ratios for different cohorts. While the comparison can be affected by changes
in the reinsurance program, and by the specific gross claims distribution for each cohort,
we would generally expect that for excess of loss reinsurance, the reinsurance ultimate loss
ratio will be zero for low gross ultimate loss ratios, and then increase as the gross ultimate
loss ratio increases, with a slope greater than one.

When the gross ultimate loss ratio is low, there are likely to be no (or very few) large losses, so we
would expect the loss to a Risk XL coverage to be zero or close to zero. When the gross loss ratio
is higher, there are likely to be more large losses which hit the layer, so the reinsurance ultimate
loss ratio will increase.

To see why the slope will be greater than one, consider a Risk XL treaty where the Risk XL layer
has attachment point £1m and exit point £2m. Let’s say the cedant’s gross claims total £2m with
corresponding gross premium of £4m, so that the cedant’s gross loss ratio is 50%. Suppose the
losses comprise attritional claims totalling £0.9m and a single large loss of £1.1m. If the
reinsurance premium is £200k then the reinsurance loss ratio will be 50% (because the loss to the
layer is £100k).

Now consider that an additional large loss of £2m occurs. The cedant’s loss ratio increases from
50% to 100%. However, (assuming free reinstatements) the losses to the reinsurance layer now
total £1.1m so the reinsurance loss ratio increases from 50% to 550%.

Chapter 25 (Reinsurance reserving) discusses, in more detail, methods by which an insurer can
reserve for recoveries and premiums under its reinsurance programme.

© IFE: 2019 Examinations The Actuarial Education Company


SP7-15: Triangulation methods Page 75

9 Glossary items
Having studied this chapter you should now ensure that you are able to explain the following
Glossary items:
 Accident year
 Atafs  age to age factors
 Atufs  age to ultimate factors
 Average cost per claim method (ACPC)
 Case by case estimation
 Chain ladder method
 Claim run-off analysis
 Commutation
 Commutation account
 Commutation clause
 Delay table
 Development factors
 Eighths method
 Grossing-up factor
 Incurred but not enough reported (IBNER) reserve
 Link ratios
 Loss ratio
 Office premium
 Partial payment
 Reporting year
 Run-off triangle
 365ths method
 24ths method
 Underwriting year.

The Actuarial Education Company © IFE: 2019 Examinations


Page 76 SP7-15: Triangulation methods

The chapter summary starts on the next page so that you can
keep all the chapter summaries together for revision purposes.

© IFE: 2019 Examinations The Actuarial Education Company


SP7-15: Triangulation methods Page 77

Chapter 15 Summary
For the purpose of claims analysis, the data can be grouped into cohorts of common origin:
 year of accident
 year of reporting
 year of underwriting.

The development of claims is quicker under a year of reporting approach but excludes IBNR,
which is included under the other two approaches.

The overriding principle of all claims analyses is the need to determine the basic
characteristics, values and trends of past data.

Consideration needs to be given regarding:


 materiality
 homogeneity of data
 how to deal with large / catastrophic losses
 how to deal with latent claims.

Estimates of the outstanding claims reserves can be made on a case by case basis, by using
statistical methods, or by using exposure-based reserving.

Assumptions are made about the stability of claim development, and that past patterns will
continue into the future.

Statistical methods might be applied to various features of claims that appear stable (and are
measurable), eg numbers or amounts.

However, the stability of the claims development pattern may be affected by:
 changes in terms and conditions
 distortions in the data
 changes in claims handling processes
 market wide initiatives
 claims reviews
 seasonality
 changes in commencement of writing policies
 changes in average policy length
 changes in reserving policy
 developments in the business, economic and legal environment.

The Actuarial Education Company © IFE: 2019 Examinations


Page 78 SP7-15: Triangulation methods

Chapter 15 Summary continued


The basic chain ladder method assumes that the future pattern of claims development
derived from the past experience will remain stable.

The method for carrying out a basic chain ladder calculation is:
 tabulate claims on a cumulative basis by development year / origin year
 calculate the development ratios
 apply these ratios to complete the table
 from the cumulative results, find the amounts expected to be paid in each future
development year / origin year cell.

The basic chain ladder method implicitly assumes that past inflation will continue into the
future. The inflation adjusted chain ladder can be used if explicit inflation assumptions are
required.

An expected loss ratio method can be used to estimate future claims. Historical loss ratios
based on the company or industry-wide data are applied to current premiums to obtain
future claim estimates.

The Bornhuetter-Ferguson method relies on an assumed run-off pattern and an estimate of


the ultimate claims for each cohort. The estimate is usually made using the loss ratio
method. The external estimate is apportioned between the past and future (as at the date
of the reserving exercise).

There are several statistical methods based on an assumed stability of average cost per
claim. They require development tables for both total claim amounts and claim numbers.

In practice, the statistical methods of estimating outstanding claims might be impaired by


distortions which invalidate the underlying assumptions. Problems might be caused by:
 errors in the data
 large claims
 inflation
 latent claims
 catastrophes
 changes in procedures
 changes in the mix of business
 lack of data.

These distortions do not mean that the statistical methods should not be used. In practice,
users who are aware of the problems can make compensatory adjustments to the data or
methods.

© IFE: 2019 Examinations The Actuarial Education Company


SP7-15: Triangulation methods Page 79

Chapter 15 Summary continued


Where data is insufficient or inappropriate, non-statistical methods can be used. Methods
can be exposure-based (either bottom-up or top-down) or be simple ratio methods
(eg survival ratios, or IBNR to outstanding claims ratios).

The unearned premium reserve (UPR) is the proportion of premium (net or gross of
acquisition costs) that is held back for the unexpired risk. Ideally the proportion is based on
the proportion of risk unexpired at the accounting date. In practice, a time basis is often
used on the assumption that risk is even over the policy year.

Special care is required with the calculation of UPR for non-annual policies and policies with
uneven risk.

The unexpired risk reserve (URR) is an estimate of the amount of claims and expenses that
will emerge from the unexpired policies. We would hope that it will be less than the UPR
(net of acquisition costs).

An AURR would be set up if URR is greater than UPR (net). This is effectively a reserve set up
to provide for an expected future loss on unexpired policies.

Reserves for solvency and published accounts are required to be presented net of recoveries.
Recoveries may generally be classified into salvage and subrogation, and reinsurance.

The Actuarial Education Company © IFE: 2019 Examinations


Page 80 SP7-15: Triangulation methods

The practice questions start on the next page so that you can
keep the chapter summaries together for revision purposes.

© IFE: 2019 Examinations The Actuarial Education Company


SP7-15: Triangulation methods Page 81

Chapter 15 Practice Questions


15.1 If incurred claims data already gives us the ultimate cost of claims for each accident year, explain
why we need to go any further (eg use projection techniques) to calculate the claim reserves.

15.2 State which of the following definitions of ‘claims handled in a year’ are equivalent to: ‘the
number of claims settled in the year plus the number of claims outstanding at the end of the
year’.

Definition 1: Number of claims settled in the year (including those settled at no cost to the
insurer).

Definition 2: Number of claims open at some point throughout the year.

Definition 3: Number of claims outstanding at the start of the year, plus the number reported
in the year.

A 1 and 2
B 2 and 3
C 2 only
D 3 only

15.3 The following triangle shows claims paid by accident / development year.

Development year
0 1 2 3
2015 150 80 30 5

Accident 2016 170 95 36


year 2017 160 90
2018 175

(i) Calculate the outstanding claims reserve (as at 31 December 2018) using the basic chain
ladder method.

(ii) For parts (a) to (d) below, choose your answer from:
A down by less than 10%
B up by less than 5%
C up by between 5% and 10%
D up by more than 10%

Estimate the effect on the total reserve of increasing the number in the specified cell by
20 units. (Take each change separately, ie they are not cumulative).
(a) 2016/1
(b) 2015/3
(c) 2015/0
(d) 2018/0

The Actuarial Education Company © IFE: 2019 Examinations


Page 82 SP7-15: Triangulation methods

15.4 Outline how you could assess the need for a general insurer to set up an additional reserve for
unexpired risks.

15.5 The following claims data were available as at the end of 2018 for a class of business written by a
general insurance company:

Year of Claim payments (£000s) in year of development


accident 1 2 3 4
2015 120 84 50 15
2016 124 90 57
2017 130 98
2018 140

Year of Number of claims settled in year of development Claims notified in


accident 1 2 3 4 accident year
2015 230 130 65 12 300
2016 240 134 70 320
2017 240 128 310
2018 230 300

Assuming that the 2015 accident year is fully developed, calculate the outstanding claim reserve
at 31 December 2018 using each of the following methods, stating any assumptions made:
(i) basic chain ladder
(ii) inflation adjusted chain ladder
(iii) an inflation adjusted average cost per claim method.

Where the method requires an assumption on inflation, you may assume a rate of 5% pa
throughout.

15.6 A director has commented that the use of statistical reserving methods by the company will lead
to an incorrect allowance for inflation, since past inflation will not equal future inflation, and
hence an alternative approach to reserving should be adopted.

Outline the points you would make in reply to this comment.

© IFE: 2019 Examinations The Actuarial Education Company


SP7-15: Triangulation methods Page 83

15.7 You have been given the following information regarding the private motor account of a large
Exam style
general insurance company.

Development of claims notified in each calendar year

Calendar No. of claims Cumulative Outstanding Case


year notified in year payments up Estimates as at
to 31/12/2018 31/12/2018
£000s £000s
2013 5,600 3,200 360
2014 21,700 15,965 890
2015 33,800 21,824 3,580
2016 35,200 16,385 9,790
2017 45,500 15,965 19,220
2018 38,300 6,600

All monetary amounts in the above table have been inflation adjusted to 31 December 2018.

(i) Use an average cost per claim method to estimate the outstanding claim amounts at
31 December 2018 values, for all claims notified in 2018. State the assumptions you make
and explain the reasoning behind your calculations. [10]

(ii) Describe how the required information would differ if you were to carry out a more
common form of inflation adjusted average cost per claim method. Describe the steps
you would then take to calculate the outstanding claim amount. [6]
[Total 16]

15.8 An insurer has a large portfolio of domestic contents policies, but transacts no other insurance
Exam style
business. The figures in the following table represent for each accident year the total gross
amount paid out in claims (£m) up to the end of each development quarter.

The written premium in 2018 was £350m, having been level at £300m for each of the previous
three years.

Development quarter
Accident 1 2 3 4 5 6 7 8 9 10 11 12 13 14 15 16
year
2015 19 52 80 123 155 174 191 200 208 211 215 216 217 222 223 224
2016 17 44 76 111 139 158 174 184 190 194 197 199
2017 22 55 86 134 162 186 209 219
2018 21 57 89 139

(i) Use the chain ladder method to estimate, from the data provided, the total outstanding
gross claim payments at 31 December 2018. [5]

The Actuarial Education Company © IFE: 2019 Examinations


Page 84 SP7-15: Triangulation methods

(ii) Explain which elements of the insurer’s technical reserves at 31 December 2018 would
not be covered at all by the amount calculated in (i), and which technical reserves may
only be partially covered. [5]

(iii) Estimate the total outstanding gross claim payments at 31 March 2019 for accidents
incurred prior to 31 December 2018. [3]

(iv) The underwriter initially estimated that the ultimate loss ratio for 2018 would be 75.4%.
Use this information, combined with the Bornhuetter-Ferguson method, to calculate a
revised estimate of the ultimate gross claim payments for the 2018 accident year. [3]
[Total 16]

15.9 On 1 January 2018, a general insurance company estimates its outstanding claim reserves for its
Exam style
comprehensive private motor business using the inflation adjusted chain ladder method, based
on an accident year cohort. Considering each of the following events in isolation, explain how the
results from the chain ladder may be affected, and after briefly outlining any preparatory
investigations, describe adjustments to the method or the data that would be necessary.

(i) A period of exceptionally windy weather in January of 2017.

(ii) An increase in sales tax from 17.5% to 20% early in 2016, ignoring any impact on the rate
of new car sales.

(iii) Poor claims experience throughout 2017 for all types of claim.

(iv) The introduction of a new administration process on 1 January 2018, which should speed
the processing of claims by about ten days.
[15]

15.10 A general insurance company currently estimates all its public liability claims on a case by case
Exam style
basis. Over the past few years, the total case estimates at the end of different financial years
have subsequently been found to be anything from 10% understated to 15% overstated.

(i) Suggest reasons why the claims may originally have been incorrectly estimated. [5]

(ii) The company is considering estimating claims by means of the chain ladder technique.
Outline the points you would make in a letter to the Board explaining briefly how the
chain ladder technique works, its key assumptions and whether it may give a more
reliable answer than case estimates. [12]
[Total 17]

© IFE: 2019 Examinations The Actuarial Education Company


SP7-15: Triangulation methods Page 85

15.11 An insurance company starts to write a new class of business on 1 March 2018. The policies are
Exam style
all for six months and all commence on the first day of the month in which they are written.

The table below shows the premiums written in each month in 2018, and the claim payments
arising from accidents in each month tabulated by month of development.

Claims paid by development month


from month of accident (£000)
Premium Month 0 1 2 3 4 5
£000s
50 March 3 2 1 0 0 0
70 April 7 5 3 1 0 0
60 May 12 7 4 3 0 0
90 June 12 11 6 3 2 0
85 July 23 12 6 4 0 0
70 August 27 15 8 4 0
60 September 29 16 8 5
75 October 26 15 7
80 November 31 17
80 December 28

Calculate the reserves that will be required at 31 December 2018 for this class of business.
Assume zero inflation and acquisition costs of 30% of the premium. State all the other
assumptions you make. [20]

15.12 A study of reporting delays for a class of business has shown that the proportion of claims
Exam style
reported in the nth month after the month of occurrence is given by:

2 / (n  2)2 n5
0 n5

The following table gives the number of claims reported in one year, tabulated by month of
accident:

January 456 May 395 September 383


February 430 June 389 October 342
March 427 July 380 November 301
April 400 August 397 December 256

(i) Calculate how many claims there will be in total from this year’s accidents. [6]

The Actuarial Education Company © IFE: 2019 Examinations


Page 86 SP7-15: Triangulation methods

(ii) Explain how your answer to (i) would differ if the table showed the number of claims
reported in the year, tabulated by month of claim notification. [2]

(iii) Explain why the 2 / (n  2)2 formula might not be appropriate in the future. [6]
[Total 14]

15.13 You have been given the following claims data. You have been asked to perform a reserving
Exam style
calculation using incurred claims data.

Claims paid to date:

AY/DY 0 1 2 3 (Ult)
2015 20 60 68 72
2016 28 80 90
2017 35 95
2018 40

Estimate of outstanding reported claims to date:

AY/DY 0 1 2 3 (Ult)
2015 40 30 7 0
2016 52 40 10
2017 65 55
2018 70

(i) Calculate the estimated ultimate claims for accident year 2017 using the basic chain
ladder method. [2]

(ii) You have been given an independent estimate of 105 for claims from accident year 2017.
Calculate the ultimate loss for this year using the Bornhuetter-Ferguson method. [1]

(iii) Repeat part (ii) in the case that the independent estimate for accident year 2017 was 130.
[1]

(iv) Comment on the above results, and hence on the appropriateness of using the
Bornhuetter-Ferguson method with incurred claims data in this case. [5]
[Total 9]

© IFE: 2019 Examinations The Actuarial Education Company


SP7-15: Triangulation methods Page 87

15.14 A new general insurer commences on 1 January 2018 with £8m capital. You are given the
Exam style
following information:
 Policies are annual and commence on the first day of each month. They are written
evenly over the year.
 The level of risk in December, January, February and March is 50% higher than the level of
risk in other months.
 Acquisition costs are 20% of written premium.
 Claims take exactly one month to be reported and a further two months to be settled.
 The company started writing business on 1 January 2018.
 By 31 December the company had written premiums of £30m and it had £25m in the
bank.
 The company keeps all its funds in a non-interest bearing bank account.
 There is no inflation and there have been no expenses other than those of acquiring the
business.

(i) Calculate the following components of the balance sheet at 31 December 2018:
(a) outstanding reported claims reserve
(b) IBNR reserve
(c) unearned premium reserve
(d) additional amount for unexpired risks (if any). [11]

(ii) Define free reserves, and list the methods a general insurer might use to increase its level
of free reserves. [6]
[Total 17]

15.15 An insurance company writes annual policies. The level of risk per day is zero at the start of each
Exam style
policy year and increases by a constant amount each day, so that the risk per day is greatest at
the end of the policy year.

The company writes £28m of premiums in the first quarter of 2018, and £32m in each subsequent
quarter. The company calculates UPR using the 40% method (ie it calculates UPR to be 40% of
written premiums in the previous 12 months).

Given that acquisition costs are 15% of premium, estimate the theoretical error in the company's
calculation of UPR. [5]

The Actuarial Education Company © IFE: 2019 Examinations


Page 88 SP7-15: Triangulation methods

15.16 (i) Explain the terms IBNR provision and IBNER provision. [2]
Exam style
(ii) State circumstances when the IBNR may need to be calculated as a separate amount from
the other parts of the claim reserve. [2]

(iii) Describe, with reasons, the circumstances under which statistical methods would not be
suitable for calculating outstanding claims reserves of a general insurer. [4]

(iv) Describe two methods that can be used to calculate claim reserves where statistical
methods are not suitable, giving two examples of each method. [4]
[Total 12]

15.17 You have been given the following information about a particular class of business as at
Exam style
31 December 2018.

Accident year 2015 2016 2017 2018


Earned premiums 85,450 86,290 78,110 90,370
Reported loss ratio 76.5% 70.2% 58.6% 38.2%
Expected loss ratio 76.0% 76.0% 78.0% 78.0%
Paid claims 62,660 57,960 36,120 22,730

Expected reported claims development pattern:


Development year 0 1 2 3 4
Expected proportion developed 50% 75% 89% 98% 100%

Calculate the revised ultimate loss ratio for accident years 2015 to 2018 as at 31 December 2018
and the estimated outstanding claims reserve according to the Bornhuetter-Ferguson method. [5]

15.18 You are an actuary advising a general insurance company on the reserves it should hold in relation
Exam style
to its domestic goods extended warranty business.

(i) Describe the consequences of incorrect reserving. [4]

(ii) List the types of reserves that the company may hold in relation to this business, and
indicate, with reasons, the likely relative size of each type of reserve. [13]

(iii) Outline the factors that should be taken into account when determining a case estimate
on this business. [3]
[Total 20]

© IFE: 2019 Examinations The Actuarial Education Company


SP7-15: Triangulation methods Page 89

15.19 A general insurance company writing only direct lines of business, has an accounting year-end
Exam style
date of 31 December. For the purpose of estimating the provisions for outstanding claims
required for its accounts as at that date, it has decided to take into account all claims information
recorded up to the end of the following month.

(i) List the main advantages and problems of adopting this course of action. [3]

(ii) Outline the relative suitability of case estimates and statistical methods for calculating the
provisions for reported claims in each of the following classes of business:
(a) liability
(b) property
(c) motor. [5]

(iii) Outline methods for calculating the provision for IBNR claims:
(a) in a class of business where claims are reported quickly and the required provision
is likely to be small
(b) in a class of business where the required provision is likely to be substantial. [6]
[Total 14]

The Actuarial Education Company © IFE: 2019 Examinations


Page 90 SP7-15: Triangulation methods

The solutions start on the next page so that you can


separate the questions and solutions.

© IFE: 2019 Examinations The Actuarial Education Company


SP7-15: Triangulation methods Page 91

Chapter 15 Solutions
15.1 Usually incurred claims data will be based on paid claims plus estimates for outstanding reported
claims only. In this case the claims for each accident year will develop further with time as the
incurred claims data will not include:
 IBNR claims (estimates were only based on claims known at each development stage)
 re-opened claims (further payments on claims which we thought were fully settled)
 changes to the estimate of outstanding claims (as more information becomes available
and eventually all claims are settled).

Even if the definition of incurred claims used includes an allowance for all outstanding claims
(ie including IBNR, reopened claims, etc), the figures shown will change over time due to the last
point, as the estimate of outstanding claims is updated.

Projection methods can allow for these features by referring to the historical pattern of claim
development.

15.2 B

For a claim to be open at some point throughout the year, the claim must have been outstanding
at the start of the year, or newly reported during the year.

Similarly, all claims which are open during a year must be either closed (ie settled) during the
year, or outstanding at the end of the year.

15.3 (i) Basic chain ladder

Development factors are given by:


r1:0  745 / 480 r2:1  561 / 495 r3:2  265 / 260

Correct answer: 183.3

Rounding each row to the nearest integer gives 184, which is perfectly acceptable.

If you apply the chain ladder approach directly to the incremental data provided, you get an
answer of 184.7. This is not the correct answer, though, because ‘the basic chain ladder method’
works on cumulative data, ie you have to make running totals for each row first.

(ii) Effect on reserve

(a) B

The main impact is on the reserve for the 2018 accident year. An increase of 20 increases the 0:1
factor from (480+265)/480=1.552 to (480+285)/480=1.594, ie about 3%.

There is also a minor decrease in the 1:2 factor from (320+175+66)/(320+175) =1.133 to
(320+195+66)/(320+195)=1.128 ie about ½%.

The Actuarial Education Company © IFE: 2019 Examinations


Page 92 SP7-15: Triangulation methods

Hence the overall effect is approximately a 2½% increase in the ultimate claims estimate for 2018
and a ½% decrease in the ultimate claims estimate for 2017. Together these increase the
estimate of the outstanding claims by around 3%.

(b) D

This has a significant impact on 2:3 ratio, which affects all years. We will now expect an extra 20
or a little more, for each of the other cohorts. An extra 60plus/180ish is big.

(c) A

This time the ratio 0:1 will go down. Hence the reserve for 2018 will go down slightly and the
other accident years will be unaffected. Of these choices, A must be the one (4.6% down).

(d) C

There will be no effect on any ratios, but reserves for 2018 will increase by 195/175, ie a bit over
11%. Given the significance of reserves for the latest year here (about three-quarters of the total
reserve), this will be an overall increase above 5%, but not as high as 10% overall.

15.4 We could assess the URR by an estimated claim ratio for the unexpired period applied to the gross
unearned premium (ie the UPR before deducting DAC).

The claims ratio will need to be adjusted to allow for trends, inflation, changes to risk, changing policy
conditions etc.

Also consider URR using:


expected claim frequency  expected average cost per claim  unexpired exposure

then deduct the UPR to get AURR.

© IFE: 2019 Examinations The Actuarial Education Company


SP7-15: Triangulation methods Page 93

15.5 (i) Basic chain ladder

Assume that:
 undiscounted claim reserves are required (ie express answers in terms of amounts
payable)
 a constant proportion of the total claim amount arising from each origin year is paid in
each development year
 the pattern of inflation within the existing data can be projected into the future.

Cumulative data

Year of Claim payments (£000s) to year of development


accident 1 2 3 4
2015 120 204 254 269
2016 124 214 271
2017 130 228
2018 140

Calculate ratios

r3 year 3 to 4: 269/254 = 1.0591

r2 year 2 to 4: (254+271)/(204+214)  r3 = 1.3302

r1 year 1 to 4: (204+214+228)/(120+124+130)  r2 = 2.2976

Complete the run off using the ratios

outstanding

2015 120 204 254 269 0


2016 124 214 271.......287 16
2017 130 228............……..303 75
2018 140.....................………… 322 182
273

Outstanding claim reserve on basic chain ladder = £273,000.

The Actuarial Education Company © IFE: 2019 Examinations


Page 94 SP7-15: Triangulation methods

(ii) Inflation adjusted chain ladder

Assume that:
 undiscounted claim reserves are required (ie express answers in terms of amounts
payable)
 a constant proportion of the total claim amount, in real terms, arising from each origin
year is paid in each development year
 claims inflation, both past and future, is 5% pa.

Convert claim amounts to 2018 values

Year of Claim payments in 2018 values (£000s) in year of development


accident
1 2 3 4
2015 139 93 53 15
2016 137 95 57
2017 137 98
2018 140

Cumulative data

Year of Claim payments in 2018 values (£000s) to year of development


accident
1 2 3 4
2015 139 232 285 300
2016 137 232 289
2017 137 235
2018 140

Calculate ratios

year 3 to 4: 300/285 = 1.0526


year 2 to 3: (285+289)/(232+232) = 1.2371
year 1 to 2: (232+232+235)/(139+137+137) = 1.6925

Complete the run off, using the ratios

2015 139 232 285 300


2016 137 232 289 304
2017 137 235 291 306
2018 140 237 293 308

© IFE: 2019 Examinations The Actuarial Education Company


SP7-15: Triangulation methods Page 95

Disaccumulate (so that inflation factors can be applied)

2015 - - - -
2016 - - - 15
2017 - - 56 15
2018 - 97 56 15

Increase at 5% for each year after 2015

2015 - - - -
2016 - - - 16 16
2017 - - 59 17 76
2018 - 102 62 17 181
273

Outstanding claim reserve on inflation adjusted chain ladder = £273,000.

(The fact that this answer is the same as in (a) above tells us that the basic chain ladder is using an
implied rate of inflation of 5%.)

(iii) An inflation adjusted average cost per claim

Assume that:
 undiscounted claim reserves are required (ie express answers in terms of amounts
payable)
 the average amount of claim payment (in real terms) is a constant for each development
year
 a constant proportion of the total number of claims from each origin year are settled in
each development year.

Divide the amount by the number of claims settled in each cell, giving the average amount settled
by cell (in £)

2015 604 712 808 1250


2016 570 705 814
2017 569 766
2018 609
Averages 588 728 811 1250

The Actuarial Education Company © IFE: 2019 Examinations


Page 96 SP7-15: Triangulation methods

BCL on claim number

ratios: year 3 to year 4 = 437/425 = 1.028


year 2 to year 3 = (425+444)/(360+374) = 1.184
year 1 to year 2 = (360+374+368)/(230+240+240) = 1.552

2015 230 360 425 437


2016 240 374 444 457
2017 240 368 436 448
2018 230 357 423 435

Applying projected claim numbers (incremental amounts) to average claim sizes

2015 - - - -
2016 - - - 16
2017 - - 55 15
2018 - 92 54 15

Increasing by 5% for each year after 2018

2015 - - - -
2016 - - - 17
2017 - - 58 17
2018 - 97 60 17

Total outstanding claims reserves = £266,000

15.6 Some statistical methods do make an implicit assumption that future inflation will reflect the
average of past inflation experience, eg basic chain ladder method.

However, some statistical methods can incorporate an explicit allowance for inflation,
eg inflation-adjusted chain ladder method, if inflation is expected to be different from the past.

Similarly average cost per claim methods make it easy to have an explicit inflation
adjustment.

15.7 (i) Outstanding claim amount for claims notified in 2018

Assume that:
 the average claim amount from each year of notification remains stable [1]
 the case estimates of outstanding claims for 2013 – 2017 are best estimates. [1]

© IFE: 2019 Examinations The Actuarial Education Company


SP7-15: Triangulation methods Page 97

Look at each year’s average claims (all at 31 December 2018 values)

Year notified No. of claims Total paid + case Average cost per claim
estimate (£000s) (£)
2013 5,600 3,560 636
2014 21,700 16,855 777
2015 33,800 25,404 752
2016 35,200 26,175 744
2017 45,500 35,185 773
Unweighted average 736
[1]
Points to consider include:
 2013 data looks out of line: is it because it is the most fully developed year, and that the
estimates for other years are too cautious, or is it that the data is historical and therefore
less applicable? [1]
 No clear trend emerging since 2014, so no reason to invalidate an answer based on an
average over several years. [1]
 Much more data for the most recent years, so averaging should be weighted to take
account of this. [1]

Decision:

Calculate the average over 2014 – 2017, using total claim numbers and amount. [1]

2013 should be excluded because it’s old and has an out-of-line average claim amount. [1]

Therefore, the average claim cost  1,000 


16,855  25,404  26,175  35,185 
 21,700  33,800  35,200  45,500  [1]

 £760.79

Therefore, the estimated outstanding  38,300  760.79  6,600,000


[1]
 £22,538,000

Assumptions made:
 As the average cost per claim is roughly constant over the period, the types of policy and
mix of business are also assumed to be constant. [1]
 The definition of the number of claims is constant over the period. [1]
 The inflation adjustments made are appropriate and correct. Any trends over the period
are allowed for in the inflation. No inflation beyond 2018 is required. [1]
 The estimates of outstanding claims are accurate and consistent from year to year, and
are all at 31 December 2018 values. [1]
[Maximum 10]

The Actuarial Education Company © IFE: 2019 Examinations


Page 98 SP7-15: Triangulation methods

(ii) More common form of average cost per claim

Data required would be very different.


 We need the amount of claims paid tabulated by year of origin and development year
(ie when happened and when paid). [1]
 We need the number of claims paid split in same way. Care is required over consistent
definition of a claim. [1]
 We need claim inflation over the period of investigation, including estimates of future
inflation.
[1]

Steps

1) Ensure data in cells is inflation adjusted to 2018.

2) Find the average claim amount in each cell (dividing numbers into amounts).

3) Find the average claim amount arising from each development year (by averaging the
cells).

4) Project the number of claims settled in undeveloped cells, using the basic chain ladder
method on the claim numbers.

5) Apply the average claim amount by development year to the estimated number of claims
settled in each cell. [3]
[Total 6]

15.8 (i) Chain ladder

Assume that:
 the pattern on claim development will continue into the future
 all claims have been settled at the end of the 4th year.

Construct the usual run off pattern, annually:

1 2 3 4
2015 123 200 216 224
2016 111 184 199
2017 134 219
2018 139

The three development ratios are:


1. 224 / 216
2. (216 + 199) / (200 + 184)
3. (200 + 184 + 219) / (123 + 111 + 134)

© IFE: 2019 Examinations The Actuarial Education Company


SP7-15: Triangulation methods Page 99

The total expected claims are:

ultimate – paid = outstanding


2016: 206.4 – 199 = 7.4
2017: 245.4 – 219 = 26.4
2018: 255.3 – 139 = 116.3

Estimate of total outstanding = £150.1m [5]

(ii) Reserves not covered

Not covered:
 UPR [½]
 AURR [½]
 Other reserves, eg equalisation, catastrophe. [1]

Possibly only partially covered:


 outstanding reported claims & IBNR & reopened claims (there may be more payments
after the fourth year) [1½]
 claim handling expense reserve (not covered if the claim amounts do not include suitable
allowance for handling expenses). [1]

Because the figures are all gross, we need to assess future reinsurance recoveries before setting
up actual net reserves. [½]
[Total 5]

(iii) Estimate the outstanding claims at 31 March 2019

Use the following ratio to estimate the claim payments up to 31 March 2019 for 2018 accidents:
(155 + 139 + 162) / (123 + 111 + 134) = 1.239

So the expected payments up to 31 March 2019 = 1.2391  139 = 172.2.

Estimated outstanding at 31 March 2019 from 2018 accidents = 255.3 – 172.2 = 83.1. [1]

Similarly for other accident years:


2017: 245.4  (208  190) / 384  219  18.4 [½]

2016: 206.4  (217 / 216)  199  6.5 [½]

Hence, the total is: 83.1  18.4  6.5  108.0 . [1]


[Total 3]

The Actuarial Education Company © IFE: 2019 Examinations


Page 100 SP7-15: Triangulation methods

(iv) Revised estimate of claims outstanding

Assume premiums are written uniformly over 2018 and 2017.

So 2018 earned premium = 325.

Hence the initial estimate of ultimate claims for 2018 is:


0.754  325 = 245.05 [1]

The expected amount to be paid by the end of 2018 (using chain ladder factors) is:
245.05 / 1.8365 = 133.43. [1]

The actual amount paid by the end of 2018 is 139.

So the revised estimate of ultimate gross claims is:


245.05 + (139 – 133.43) = 250.6 [1]

or:

The expected amount to be paid after the end of 2018 (using chain ladder factors) is:
(1  0.54451)  245.05  111.6 , so the estimate of ultimate gross claims is 139 + 111.6.
[Total 3]

15.9 (i) Windy weather January 2017

The unusual weather may have resulted in higher claims than usual. [½]

We should investigate the claims from this period, looking particularly at:
 whether they have been settled [½]
 the balance between damage and liability. [½]

Most of these will probably have been damage related, and as they occurred in January, they are
likely to have been settled. [½]

This increases the 2017/0 cell, which will lead to over reserving in subsequent years. [½]

The extra claims included in 2017/0 (ie over and above what would normally have been incurred
in the period) should be removed for the chain ladder exercise ... [1]

... and an additional amount for outstanding claims from the period should be added back on. [½]

There may be some reinsurance recovery if the claims above an excess limit are covered by an
excess of loss or catastrophe treaty. This needs investigation. [1]

(ii) Increase in sales tax in early 2016

From the date of the increase in the tax, some of the payments will be higher than if tax had
stayed at 17.5%. [½]

© IFE: 2019 Examinations The Actuarial Education Company


SP7-15: Triangulation methods Page 101

This would not affect all claim payments. For example, liability awards would not be affected, but
many of the claims based on repairing motor vehicles will be increased. [½]

This is effectively a one off inflationary effect. Without correction, the results would be
overstated. [1]

The problem is solved by adjusting prior years’ payments (ie before 2016) as if they had included
the tax at 20%. [1]

We will need to investigate the proportion of prior years’ claim payments that included the
tax. [½]

(iii) Poor claims experience throughout 2017

This should not cause any distortion in the effectiveness of the chain ladder. Provided the
balance between short tail and long tail claims is not disturbed, there will be no distortion. [1]

However, we might need to make some allowance for:


 the claims handling staff being slower on average, due to having more claims to settle [½]
 any distortions that might be caused if the worsening of experience was weighted to the
year end. (This would result in a bigger proportion of claim settlements falling into the
next development year.) [½]

If we use an unmodified chain ladder, both of these could result in under reserving. [½]

For the first point, we need to investigate average settlement delays. The number of reported
claims outstanding at the year end (from 2017 accidents) compared with the number of claims
settled (from 2017 accidents) might indicate an increasing average settlement delay. [1]

The chain ladder calculations could be recalculated with the 2017 diagonal figures inflated to
what they might have been had settlement patterns been as normal. [½]

For the second point, separate estimates of reported claims and IBNR would be helpful. Reported
claims could be estimated by using ‘year reported’ cohorts for the chain ladder. A delay table
based on the number of claims reported in each month would be used to estimate IBNR. [½]

(iv) New administration process on 1 January 2018

All claims tabulated in the run off triangle were reported before the new administration system
took effect. There would therefore be no inconsistency within the data. [½]

Although the outstanding claims will be settled more quickly now, there is no reason here to
doubt the amount of outstanding claim produced by the chain ladder. [1]

Claims are settled ten days quicker, so there should be ten days’ less inflation adjustment. [½]

However, this level of accuracy is probably spurious. [½]


[Maximum 15]

The Actuarial Education Company © IFE: 2019 Examinations


Page 102 SP7-15: Triangulation methods

15.10 (i) Why claims were originally incorrectly estimated

Data

 It can be difficult with liability style claims to collect sufficient data on which to base an
accurate assessment.
 Data may take a long time to materialise, for example as a condition stabilises.
 IBNR – clearly case estimates cannot be made where no claim has been reported. (It is
not clear from the wording of the question how IBNR is being treated here.)

Quality of staff

 Management guidelines given to assessors may have changed.


 Claims staff may have changed over the period.
 Claims assessors may not be competent.

Assumptions

 The basic inflation assumption may be wrong.


 The estimated time to settlement may be wrong.
 The pattern of court awards may not be correctly assessed.

Also, by chance due to random fluctuations.

[½ mark per point, total 5]

(ii) Letter to the Board

Method

On the basis of the pattern of claim payments from previous years, the chain ladder (CL) projects
the expected total payments for outstanding claims.

The method begins by splitting past claim payments into a table divided by accident year (the year
when the claim occurs) and year of payment. This will generate a triangular shaped table, as
shown below:

Number of years after accident (development year)

0 1 2 3 4 5 6
2012 x x x x x x x
2013 x x x x x x
Year of 2014 x x x x x
accident 2015 x x x x
2016 x x x
2017 x x
2018 x [1]

© IFE: 2019 Examinations The Actuarial Education Company


SP7-15: Triangulation methods Page 103

The main assumption is that the pattern of claim payments emerging from each accident year
remains constant. We will then be able to estimate the claim amount. For example, the 2013
accident year, development year 6 (2013/6), from the ratio of 2012/6 to 2012/5. Projections are
made on a similar basis for all years with outstanding claims. [1]

The projections can be extended beyond year 6 for those claims from the 2012 accident year
which haven’t yet been settled. [½]

The CL method can also be applied to other tabulations of claims data. For example, we might
tabulate claims by reporting year (rather than by accident year) in order to derive an estimate of
the outstanding reported claims. [½]

This choice will depend upon the tabulation that gives a payment pattern we believe to be stable.
[½]

Assumptions and reliability

We are relying on a regular pattern of claim payments to give us reliable results. In practice this
might be distorted by: [1]
 changes in the mix of business from year to year [½]
 changes in policy conditions [½]
 changes in claim reporting and settlement procedures [½]
 reliability or otherwise of past data. [½]

The reliability of data can be doubtful where the amount of data is small, or there are one-off
(eg catastrophic) events within past data, which should not be projected into the future. [1]

Changing inflation can have an impact on the results. However, the CL method can be adjusted to
incorporate allowance for past inflation and our expectations of future claim inflation. [1]

The CL can only provide total estimates – it cannot be used as a basis for estimating claim
amounts for individual claims. Further, it makes no allowance for known data about outstanding
claims. [1]

However, it is possible to apply the CL approach to tabulated data that includes estimates of
claims reported to date, by year of accident and year of reporting. [½]

Providing the pattern of reporting (by estimated amount) is stable, the CL approach will provide a
reliable estimate of claim amounts in relation to claims that have not yet been reported. [½]

An advantage of the CL, is that it provides an objective estimate which can be used as a check on
the total estimates from the claims assessors. It may also be appropriate to consider using the CL
for the first two or three years of claim development, when the number of outstanding claims is
greatest. [1]

Whether the method will give a more reliable answer than produced by the case estimation will
depend on how far the problems outlined in the section above are absent, and on the quality of
the case estimates. [1]

The Actuarial Education Company © IFE: 2019 Examinations


Page 104 SP7-15: Triangulation methods

Even when a mechanical use of the chain ladder method is not appropriate (ie where there have
been changes in experience), if the user is aware of the reasons why past data should not be
mechanically used as a basis for projection into the future, adjustments can be made to the standard
method. [1]

Suitability for public liability claims

The CL method is unlikely to be a suitable alternative to case estimation for public liability claims.
The reasons are: [1]
 there is unlikely to be enough data to justify total reliance on a statistical technique [½]
 there are usually a few large and perhaps unpredictable claims from public liability which
will require individual attention. [½]
[Maximum 12]

15.11 Use the basic chain ladder (as the assumption of zero inflation is given). [½]

Exclude June data, as there appears to have been some delay in settlement (note the different
run off pattern for June). [½]

Exclude earlier months too:


 they might be affected by the June blip [½]
 they are less reliable as the volumes of data were less stable prior to June, and the
amount of business on the books was still increasing. [½]

Reserve for IBNR will be included within the chain ladder calculations, since an accident year
cohort is used.
[½]

Calculations including July onwards give:


r1:0  1.5515 r2:1  1.1779 r3:2  1.0903 [1½]

Expected further claim payments from each month are:


October: 4.3 [1]
November: 13.6 [1]
December: 27.8 [1]
Total: 45.7 [1]

So the total outstanding claims reserve, including IBNR is: £45,700.

Calculations could also be carried out using different base data. For example, it would be
reasonable to argue (explicitly) that all data can be included, as the weight given to early cohorts
will be small anyway.

© IFE: 2019 Examinations The Actuarial Education Company


SP7-15: Triangulation methods Page 105

Assumptions
 There will be no further payments in respect of accidents prior to October. [½]
 The pattern of claim settlement in respect of accidents in October to December will be
consistent with the pattern of settlement for the previous three months, ie no
seasonality. [½]
 Christmas and New Year have not disturbed the pattern of claim settlement (the result
could be particularly sensitive to, say, an understatement of claims arising from December
accidents). [½]
 Claims reserves are not discounted. [½]
 Claim handling expenses are included in the data given. [½]

It could be argued that only data from September onwards is valid, since fewer policies are
exposed prior to the completion of six months from the start of writing business. This would,
however, leave too little data.

Reopened claims

Assume that no reserve is needed in respect of claims that have been reopened or may be in the
future. [½]

Unearned premium reserve

As premiums are written on the first day of each month:

Earned premium  50  70  60  90  85  65  70  46  60  63  75  26  80  16  80

 £530,833 [½]

Unearned premium  16  70  26  60  63  75  46  80  65  80

 £189,167 [1½]

Check: total premiums written = £720,000 (ie sum of earned and unearned).

Acquisition costs are 30%, so unearned premium reserve (net of DAC) is 70% of £189,167:
UPR = £132,417 [1]

Assumptions
 Premiums (net of acquisition costs) are earned evenly over the 6 month life of the policy.
That is, risk is even over the life of the policy. [½]
 There is no seasonality in claim experience. [½]

Additional reserve for unexpired risks

Total claims to date = £363,000 (by summing all the claims data given)

Outstanding claims = £45,700 (from above)

The Actuarial Education Company © IFE: 2019 Examinations


Page 106 SP7-15: Triangulation methods

Total claims incurred = £408,700 [1]

Assuming that claim handling costs are fully included within the data, the total claims cost from
£530,833 of earned premium is £408,700. [½]

Therefore the expected claims cost from unearned premium is:


408,700/530,833  189,167 = £145,644 [1]

This is greater than the UPR calculated above. We therefore need to hold an additional reserve
for unexpired risk:
145,644 – 132,417 = £13,227 [½]

Assumptions
 Claims experience for unearned premiums will be the same as for aggregate experience
for premiums earned in 2018. [½]
 The company does not hold any other contingency reserves (eg no catastrophe reserve
maintained) and there are no margins built into the other reserves. [½]

The total reserve required will be £191,344 (ie £192,000 say). [½]

This ignores any additional solvency margin required, plus the need for extra free reserves to give
greater freedom from intervention. [1]
[Maximum 20]

15.12 (i) No. of claims from the year’s accidents

n 2/(n+2)2 Cumulative
0 1/2 0.5000
1 2/9 0.7222
2 2/16 0.8472
3 2/25 0.9272
4 2/36 0.9827
5 balance 1.0000
6 0 1.0000
[2]

For n  5 , the proportion of claims must be the balancing item, so that all the items sum to 1. [1]

© IFE: 2019 Examinations The Actuarial Education Company


SP7-15: Triangulation methods Page 107

Month Total claims from month

December 256  0.5000 = 512


November 301  0.7222 = 417
October 342  0.8472 = 404
September 383  0.9272 = 413
August 397  0.9827 = 404
July – January 380 + 389 + 395 + 400 + 427 + 430 + 456 = 2,877
5,027

Therefore 5,027 claims from the year’s accidents. [3]


[Total 6]

(ii) The table shows claims reported in the year

For each month, January to May, the table would contain claims originating from the previous
year’s accidents. [½]

From the figures given, we cannot accurately separate the previous year’s data. [1]

Thus, there is insufficient data to calculate the claims from this year’s accidents. [½]
[Total 2]

(iii) Why the formula might be inappropriate in future

There may have been a changes in the reporting system used. For example, a 24-hour direct
telephone system replacing written claims via brokers. [1]

Improved internal administration systems, so that claims are logged as being ‘reported’ more
quickly than previously; or worse internal administration, due to, for example, loss of staff,
increased workload, strikes etc. [1]

Changes to policy conditions, with a change in the balance of the type of claims emerging.
Different types of claims may have a different rate of emerging and being reported. [1]

Changes in the mix of business with different types of claim coming from each type of
business. [1]

Secular changes (eg with motor, more road traffic) so that the balance of the type of claims
changes. [1]

Seasonal changes. [½]

The formula may never have been appropriate in the first place (eg poorly conducted study, just
chance result over the period of the study). [½]
[Total 6]

The Actuarial Education Company © IFE: 2019 Examinations


Page 108 SP7-15: Triangulation methods

15.13 (i) Ultimate claims for AY 2017 using BCL

Incurred claims to date:

AY/DY 0 1 2 3 (Ult)
2015 60 90 75 72
2016 80 120 100
2017 100 150
2018 110

Grossing-up factor for development year 1 is 0.8 (ie 0.833  0.96 ). [1]

Estimated ultimate claims using the basic chain ladder = 150  0.8  120. [1]
[Total 2]

(ii) Ultimate claims for AY 2017 using BF with IE=105

Future expected claims are:


1
105  (1  )  26.25
0.8

Estimated ultimate claims for 2017 are:


150 – 26.25 = 123.75 [1]

(iii) Ultimate claims for AY 2017 using BF with IE=130

Future expected claims are:


1
130  (1  )  32.5
0.8

Total ultimate claims for 2017 are:


150 – 32.5 = 117.5 [1]

(iv) Comments on results and appropriateness of method

The BF estimate of ultimate claims using an independent estimate of 105 is greater than one
would expect using basic chain ladder alone, despite the independent estimate being lower – to
make use of the independent estimate, we would expect a result that was lower than 120. [1]

The BF estimate of ultimate claims using an initial estimate of 130 is also outside the range one
would expect given the chain ladder and independent estimates. [½]

We are seeing reductions in the cumulative incurred claims for each cohort year from
development year 1. This is probably due to over-reserving. [½]

Claims could also be settled for less than expected, perhaps due to lower than expected claims
inflation, salvage and subrogation recoveries, etc. [1]

© IFE: 2019 Examinations The Actuarial Education Company


SP7-15: Triangulation methods Page 109

In this case, incurred claims to date are greater than what we would expect ultimately from this
accident year. We therefore have to apply a negative adjustment to allow for future
development. [½]

Under the BF method, this reduction is a function of the independent estimate. [½]

Since, we know (from the historical claims pattern) that we are over-reserving, it would be better
to make a reduction to the reserves based on reported claims. [1]
[Total 5]

15.14 (i) Accounts

Let 1 risk unit be the risk from £1m of annual written premium in one standard month.

Then the total exposure from £1m written premium is 8  1 + 4  1.5 = 14 risk units.

The total exposure over a policy year is, therefore, 12  14 units = 168 units. [1]

For the risks on the books on 31 December, the risk units exposed per month during the year just
ended were:
Total exposure during month

Jan 1.5 units (ie only January policies and non-standard risk)
Feb 3 units (ie 2 months’ policies with non-standard risk)
Mar 4.5 units
Apr 4 units (ie 4 months’ worth of policies with standard risk)
May 5 units
Jun 6 units
Jul 7 units
Aug 8 units
Sep 9 units
Oct 10 units
Nov 11 units
Dec 18 units. [2]

Therefore the expired risk is 87 units and the unexpired risk must be 168 – 87 = 81 units. UPR can
be calculated from a ratio of the risk exposure:

UPR (without adjustment for acquisition expenses) = 81/168  £30m = £14.46m. [1]

UPR (adjusted for acquisition expenses) = £14.46m  0.8 = £11.57m [1]

There is a three-month delay from occurrence to settlement, including a one-month delay in


reporting. Therefore, there will be claims outstanding relating to the risks for October, November
and December, with those relating to December being IBNR claims.

The Actuarial Education Company © IFE: 2019 Examinations


Page 110 SP7-15: Triangulation methods

Of the 87 expired risk units, claims will have been paid in respect of 48 units, with 21 units relating
to outstanding reported claims and 18 units to IBNR claims. [1]

The cashflows to and from the company in the year must explain the change in cash held,

ie Premiums less commission paid less claims paid = increase in cash held

ie 0.80  30 – Claims paid = 25 (year end cash) – 8 (initial cash)

Claim payments for exposure in January to September are £7m (ie 24 + 8 – 25) [1]

Therefore, the claim payment per risk unit = 7/48. Using this as a basis for estimating the
outstanding reported claims, IBNR and total URR, we get:

(a) outstanding reported claims: 21  7/48 = £3.06m [1]

(b) IBNR: 18  7/48 = £2.63m [1]

(c) UPR: (from above) = £11.57m

(d) Total URR: 81  7/48 = £11.81m [1]

Additional URR: (URR – UPR) = £0.24m [1]


[Total 11]

(ii) Free reserves

Free reserves are assets less technical reserves and current liabilities. [1]

Ways to increase the free reserves are to:


 raise capital (eg a rights issue, if this is a proprietary company)
 strengthen the valuation basis of assets
 weaken the liability valuation basis
 write more profitable business…
 …or increase premium rates, assuming volumes remain constant
 possibly write less business
 control expenses better
 improve investment returns, both income and gains
 pay out less dividends (if this is a proprietary company)
 possibly make use of equalisation reserves. [½ each]
[Total 6]

© IFE: 2019 Examinations The Actuarial Education Company


SP7-15: Triangulation methods Page 111

15.15 UPR on 40% method: 0.4  (28 + 3  32) = 49.6 [½]

Assume business is written at the mid-point of each quarter. [½]

Over the policy year, if the risk increases from 0 to say 2 units by the end of the policy year, then
the total risk over the year is 1 unit.

At the end of 2018, the business written in Q1 will be 7 8 ths of the way through the policy year.

The level of risk will then have reached 2 87 units per day. Hence, the total risk units earned up to
that point will be:

27  7  2  49
8 8 64

So the UPR from the business written in Q1 2018 is:

0.85  28  64
15  5.578 [1]

Similarly, the business written in Q2 is now 5 8 ths of the way through the policy year. The
proportion of the total risk earned to date is 25 64 and the UPR is:

0.85  32  64
39  16.575 [½]

The business written in Q3 is now 3 8 ths of the way through the policy year. The proportion of
the total risk earned to date is 9 64 and the UPR is:

0.85  32  64
55  23.375 [½]

Finally, the business written in Q4 is now 1 8 th of the way through the policy year. The
proportion of the total risk earned to date is 1 64 and the UPR is:

0.85  32  64
63  26.775 [½]

The accurate UPR is the sum of these:

5.578  16.575  23.375  26.775  72.305 [½]

Hence UPR is understated by:

72.3  49.6  22.7 (31.4% of the true UPR). [1]


[Total 5]

The Actuarial Education Company © IFE: 2019 Examinations


Page 112 SP7-15: Triangulation methods

15.16 (i) Incurred but not reported (IBNR) provision

A reserve to provide for claims in respect of events that are believed to have already happened,
but have still to be reported to the insurer. [½]

Incurred but not enough reserved/reported (IBNER) provision

A reserve reflecting changes (increases and decreases) in initial estimates of claims outstanding.
[½]

For direct writers, the reserve will usually be an implicit part of the reserve for reported
outstanding claims. However, reinsurers will often hold it explicitly to reflect the greater
uncertainty in the original estimates provided by direct writers. [1]
[Total 2]

(ii) Separate IBNR

 When outstanding claims reserves are calculated using case estimates. [½]
 When outstanding claims reserves are calculated using statistical methods with a
reporting year cohort for grouping claims. [½]
 For separate statutory returns (eg to the PRA) and tax accounts where a separate
statement of the IBNR is required. [½]
 For internal management accounts where a detailed breakdown of technical liabilities
may help management in their decision making processes. [½]
[Total 2]

(iii) When statistical methods are not suitable

Statistical methods would not be suitable when reserving for either:


 one-off large claim events, such as a major tsunami [½]
 insurance of non-standard risks, such as a satellite launch. [½]

This is because the characteristics of the claim event or risk will be so different from the past
experience that it really has to be considered on its own merits. [½]

Neither would statistical methods be suitable for types of claims where the claim development is
very unpredictable … [½]

… for example, asbestos, pollution and health hazards. [½]

The uncertainty of claim development patterns makes the assumptions underlying statistical
methods invalid. [½]

Statistical methods will not be possible if past data is insufficient. [½]

They may also be unsuitable when there have been major changes that would be difficult or
impossible to allow for, such as changes in cover, underwriting or mix of business. [1]
[Maximum 4]

© IFE: 2019 Examinations The Actuarial Education Company


SP7-15: Triangulation methods Page 113

(iv) Reserving methods

Exposure-based methods

Exposure-based methods can be used for one-off events or risks. Two approaches used are
bottom up and top down. [½]

Bottom up involves examining the conditions of each policy to determine whether the insurer is
exposed to the loss event. [½]

For any policies where the company is exposed to the risk, a claims expert will assess the extent
of any claim on that policy. [½]

The top down approach involves estimating the total market loss from an event. [½]

We then estimate how much of this is attributed to an individual insurer (or policy) by considering
the conditions (eg excesses or limits) of the policies. [½]

Ratio methods

Ratio methods can be used where there is insufficient data to use exposure-based methods. [½]

For example, we could use survival ratios so that the reserve held is such that the company is able
to sustain the current rate of claim payment or increase to reported claims for a given number of
years. [1]

Another example is IBNR to outstanding claim ratios. This involves applying a pre-determined
ratio to the outstanding reported claim reserve to determine the IBNR claim reserve. [1]
[Maximum 4]

The Actuarial Education Company © IFE: 2019 Examinations


Page 114 SP7-15: Triangulation methods

15.17 Calculations are summarised in the following table:

Accident year 2015 2016 2017 2018


Earned premiums 85,450 86,290 78,110 90,370
Reported loss ratio 76.5% 70.2% 58.6% 38.2%
Expected loss ratio 76.0% 76.0% 78.0% 78.0%
Paid claims 62,660 57,960 36,120 22,730
Remaining development 2% 11% 25% 50%
Expected ultimate 64,942 65,580 60,926 70,489
Development to date 65,369 60,576 45,772 34,521
Expected future 1,299 7,214 15,232 35,245
development
Revised ultimate 66,668 67,790 61,004 69,766
Revised loss ratio 78.0% 78.6% 78.1% 77.2%
Reserve 4,008 9,830 24,884 47,036

[1 for each year, award ½ for correct method but numerical errors]

Therefore the total reserve is 85,758. [1]

Notes:

The remaining development is derived from the development pattern given.

Expected ultimate = expected loss ratio  earned premium

Development to date = reported loss ratio  earned premium

Expected future development = remaining development  expected ultimate

Revised ultimate = development to date + expected future development

Revised loss ratio = revised ultimate / earned premiums

Reserve = revised ultimate – paid claims

An alternative (and quicker) method would be to calculate the revised loss ratio as:
reported loss ratio + remaining development  expected loss ratio.

[Total 5]

© IFE: 2019 Examinations The Actuarial Education Company


SP7-15: Triangulation methods Page 115

15.18 (i) Consequences of incorrect reserving

Reserves may be either overstated or understated. [½]

If they were overstated, this might worsen the apparent results, leading to a loss of confidence by
shareholders, brokers and the stock market. [½]

This may result in loss of new business or a reduction in the company’s share price. [½]

Or it would make the solvency position look worse than it actually is, which may cause problems
with the regulators. [½]

For example, they may require the company to produce extra reports or to take out additional
reinsurance. [½]

Overstating reserves would also tie in assets that could be applied more usefully to other projects
by the insurer. This would result in a loss of profit from the project that the assets could be more
usefully used for. [1]

If reserves were understated, there would be a danger that apparent profits could be prematurely
distributed, leading to future problems meeting liabilities. [½]

Tax payments could be larger in the short term than would otherwise be the case. [½]

Incorrect reserving can have a knock-on effect if the results are used in any pricing exercise, either
over- or understating the correct rate that should be charged for a particular risk. [½]
[Maximum 4]

(ii) Reserves and their likely size

Outstanding reported claims reserve [½]

This should be small because most claims under extended warranty are settled very quickly once
they have been reported. The delay may be, at most, a few weeks whilst some larger items are
repaired or replaced. [1]

Incurred but not reported reserve [½]

This should be very small as claims are generally reported very quickly once a fault has been
discovered. [1]

Incurred but not enough reported reserve [½]

If shown separately, this could be sizeable (compared to the outstanding reported claims reserve),
because early estimates of repair or replacement costs could be inaccurate. [1]

Claims handling expense reserve [½]

This could be large in relation to claims costs. Claims costs themselves will be relatively small, and
so expenses may have a big influence. [1]

The Actuarial Education Company © IFE: 2019 Examinations


Page 116 SP7-15: Triangulation methods

Reopened claims reserve [½]

This could exist if repairs were ineffective, but would probably be small. [½]

Catastrophe reserve [½]

This is unlikely to exist, but could (in theory) be held to cover possible faults that affect all
products  for example, all washing machines manufactured by one company. [1]

Claims equalisation reserve [½]

This will probably be zero as volatility should not be a major problem. [½]

Ensuring that sufficient free reserves are held may be an alternative to the last two reserves. [½]

Unearned premium reserve [½]

This will be the largest of the reserves. [½]

The term of the policy can be many years longer than the manufacturer’s guarantee and most risk
is towards the end of the term. In the first year the UPR may be the entire single premium in size.
[1]

Additional unexpired risk reserve [½]

This could be held if premiums are deemed insufficient to cover the unexpired risk. As the risk
towards the end of the policy can be sizeable, there may be considerable uncertainty as to the
sufficiency of the premiums, and hence the AURR could be sizeable. [1]
[Maximum 13]

(iii) Factors that should be taken into account when determining a case estimate

We might consider:
 the type of good being insured (eg washing machine)
 the make and model (or value) of good
 the duration the policy has been in force
 the age of the domestic good
 the impact of claims inflation (eg increases in repair costs or value of goods)
 who is carrying out the repair (eg an ‘approved’ service provider)
 any previous claims history on the policy
 location (possibly affects cost of repair)
 the nature of the fault (eg has an appliance stopped working completely or is it just not
functioning properly?).
[½ each point, maximum 3]

© IFE: 2019 Examinations The Actuarial Education Company


SP7-15: Triangulation methods Page 117

15.19 (i) Advantages and disadvantages

The main advantages are:


 it gives a better knowledge of outstanding claim values and IBNR, including those for
exceptional year-end weather claims [½]
 it avoids December backlogs due to holidays, postal delays etc. [½]

The main disadvantages are:


 claims belonging to the next financial year must be excluded [½]
 claims must still be accounted for according to their status at the year end [½]
 statistical development patterns may be distorted, with subsequent difficulties for
projections of IBNR, etc [½]
 preparation of statutory accounts and results may be delayed. [½]
[Total 3]

(ii) Suitability of case estimates and statistical methods

Applying to all three classes, case estimates have the following advantages:
 they give greater management control over total claims reserves [½]
 they pick up new trends more quickly than statistical methods. [½]

(a) Liability

There will normally be a relatively large number of small claims, settled quickly and a smaller
number of large claims, often subject to long delays in discovery and/or settlement. Statistical
methods may be necessary in the early development periods, when relatively little information is
available on the larger claims. [1]

At later durations, case estimates may deal best with the few cases then outstanding. [½]

(b) Property

Statistical methods will normally give stable results for personal lines with large numbers of
relatively small claims. Case estimating is relatively expensive in such cases. [1]

Larger claim size risk groups, such as commercial fire, may be better suited to case estimating. [½]

(c) Motor

At early stages of development, large numbers of property damage claims will make statistical
estimates suitable. [½]

At later stages, there will be relatively small numbers of large injury claims, for which case
estimates may become more suitable. [½]
[Total 5]

The Actuarial Education Company © IFE: 2019 Examinations


Page 118 SP7-15: Triangulation methods

(iii) Methods for IBNR

In both cases, if a statistical method has been used for reported claims, it may implicitly include
the IBNR. [½]

(a) In a class of business where claims are reported quickly and the required provision is likely
to be small simple methods will suffice. [½]

For example:
 a percentage of premiums [½]
 a percentage of claims paid [½]
 a percentage of claims reported. [½]

(b) In a class of business where the required provision is likely to be substantial more
elaborate methods should be employed. [½]

These would be based on historical development / delay patterns. [½]

For example:
 chain ladder techniques [½]
 projection of claim numbers and average amounts. [½]

Under both methods, allowance should be made for any special features. [½]

For example:
 changes in claim administration [½]
 special claims etc. [½]
[Total 6]

© IFE: 2019 Examinations The Actuarial Education Company


SP7-15: Triangulation methods Page 119

End of Part 3

What next?
1. Briefly review the key areas of Part 3 and/or re-read the summaries at the end of
Chapters 14 and 15.
2. Ensure you have attempted some of the Practice Questions at the end of each chapter in
Part 3. If you don’t have time to do them all, you could save the remainder for use as part
of your revision.
3. Attempt Assignment X3.

Time to consider …
… ‘learning and revision’ and ‘revision’ products
Flashcards – These are available in both paper and eBook format. Students have said:

‘The Flashcards are perhaps the most useful thing I have bought from ActEd.
Loving your work on this!’

‘I find Course Notes and exam questions supplemented with Flashcards the most
efficient way of learning.’

‘I think the Flashcards have been very useful. Would definitely buy the
Flashcards again for other modules.’

You can find lots more information, including samples and demos, on our website at
www.ActEd.co.uk.

Buy online at www.ActEd.co.uk/estore

The Actuarial Education Company © IFE: 2019 Examinations


All study material produced by ActEd is copyright and is sold
for the exclusive use of the purchaser. The copyright is
owned by Institute and Faculty Education Limited, a
subsidiary of the Institute and Faculty of Actuaries.

Unless prior authority is granted by ActEd, you may not hire


out, lend, give out, sell, store or transmit electronically or
photocopy any part of the study material.

You must take care of your study material to ensure that it


is not used or copied by anybody else.

Legal action will be taken if these terms are infringed. In


addition, we may seek to take disciplinary action through
the profession or through your employer.

These conditions remain in force after you have finished


using the course.

The Actuarial Education Company © IFE: 2019 Examinations


SP7-16: Stochastic reserving models Page 1

Stochastic reserving models


Syllabus objectives
3.3 Evaluate stochastic reserving processes.
3.3.1 Describe the uses of stochastic reserving methods.
3.3.2 Describe the likely sources of reserving uncertainty.
3.3.3 Describe the following types of stochastic reserving methods:
 analytic methods
 simulation-based methods.
3.3.3 Describe the Mack and bootstrapping approaches to reserving.
3.3.5 Describe the issues, advantages and disadvantages of each of the methods.
3.3.6 Describe the approach to aggregating the results of stochastic reserving
across multiple lines of business, and discuss methods of correlation.

The Actuarial Education Company © IFE: 2019 Examinations


Page 2 SP7-16: Stochastic reserving models

0 Introduction
The traditional methods of claims reserving, such as the chain ladder method that we looked at in
the previous chapter, produce a single best estimate of the claims reserve. In reality the actual
reserve required will differ from this estimate. In order to assess the likely error involved in using
the best estimate, we need to use a stochastic reserving method, which will provide us with a
confidence interval as well as a best estimate. It is these methods that we will be looking at in this
chapter.

In most cases the stochastic models produce a best estimate that is either the same as, or very
close to, the best estimate derived from the chain ladder method. They also provide us with
information about the distribution of the reserves, for example, the variance.

We can consider the run-off of claims reserves to be a random process, with many random
factors influencing the outcome. These uncertain factors include:

 the occurrence and severity of claims

 the notification delays on individual claims

 legal changes that affect the size of awards

 legal changes that affect the ‘heads of damage’ (types of loss recognised in
compensation awards for serious injuries, such as loss of income, medical and
nursing costs and so on) awarded

 changes in the litigiousness of society

 levels of claims inflation (which in turn are related to levels of price inflation and
wage inflation in the economy)

 court rulings on liability or quantum of individual claims not foreseen by claims


handlers and/or not in the historical data

 changes in the mix of claim types, either caused by an underlying change in claim
type experience or by changes in the mix of business written

 changes in claims handling, either because of policy changes or because of external


events (such as a catastrophe leading to claims handlers being over-stretched)

 the emergence of new types of claim

 changes in the way claims are settled – for example, if more claims are settled in the
form of Periodical Payment Orders (rather than as lump sums).

These factors contribute to the uncertainty underlying the process of the run-off of claims.
We will often use historical data to project the run-off of claims. The projection introduces
further uncertainties, including:

 The historical data only provides a limited sample.

 The quality of data may have varied over time.

 There are many ways of deriving an estimate of the claims reserve, and many
judgements required in each method. We know the uncertainty introduced by this
as ‘model error’ or ‘model uncertainty’.

We will discuss model error in more detail in Section 3.

© IFE: 2019 Examinations The Actuarial Education Company


SP7-16: Stochastic reserving models Page 3

When we developed reserves in the past, we were mainly interested in developing


techniques for deriving best estimates of reserves. Recently, we have become more
interested in developing methods for quantifying the levels of uncertainty around the best
estimate of the reserves.

It is often no longer considered sufficient to describe the reserves using single estimates, without
also gauging the size of the prediction errors that may be present in these estimates.

Question

Suggest a reason for this.

Solution

One reason is that the insurer needs to ensure that it has sufficient reserves to be able to pay the
claims if they exceed the levels expected. This could be for regulatory or other reasons. So it will be
interested in the probability that its reserves will be insufficient to pay the eventual cost of claims.

In this chapter we outline:

 the reasons for considering variability around estimated reserves and the uses to
which we can put such results

 the main methods for determining variability, including analytical methods and
simulation methods

 methods for allowing for reinsurance

 methods for allowing for the aggregation of results across different lines of
business

 the relative strengths and weaknesses of the main stochastic methods.

The Actuarial Education Company © IFE: 2019 Examinations


Page 4 SP7-16: Stochastic reserving models

1 Uses of stochastic reserving


In recent years, there has been growing interest from all parties in the uncertainty of
reserves. While we have generally used point estimates in the past and have been
interested in developing methods to derive these, it is becoming increasingly apparent that
such an approach is not sufficient. We are interested in the uncertainty of reserves,
including its impact on the capital backing the insurance liabilities and its sufficiency.

Question

Suggest reasons why this approach is not sufficient.

Solution

Reasons include, for example:


 the possibility of bankruptcies arising as a result of catastrophe events
 an increasing awareness of reserving issues relating to latent claims
 problems made worse by the adverse economic conditions
 changing emphasis from regulators
 new guidance from the profession.

Substantial work has gone into developing actuarial methods to quantify the uncertainty in
reserves. Many of the methods are based around existing reserving methods (such as the
chain ladder method). Other methods derive both an estimate of the mean reserves and the
variability.

We can use the results from examining claim variability to:

 Assess reserve adequacy in absolute and relative terms. Some companies may hold
precautionary margins in their reserves for one or more reasons, from regulation to
prudence. These margins may be explicit or implicit. By examining claim variability,
we can provide management with information as to the strength of the reserves.

 Compare the reasonableness of different sets of reserve estimates.

 Compare datasets at different as at dates.

 Monitor performance to see if claim movements are material.

 Allocate capital. Quantifying reserving risk is a key component of insurance


companies’ capital models.

 Inform the management / Board of the insurance company to assist with ongoing
decision making, for example in what areas to expand or contract the volume of
business being written.

 Provide information to investors. From an investor perspective, understanding the


uncertainty in claims reserves helps to compare the relative attractiveness of two
different investments. To assist investors, accounting rules tend to require more
explicit disclosure of information concerning variability.

© IFE: 2019 Examinations The Actuarial Education Company


SP7-16: Stochastic reserving models Page 5

 Inform discussions with regulators. Adverse run-off movements can impact the
solvency of a company. Regulators are interested in gaining a better understanding
of the run-off risk. To this end, regulations tend to require more explicit disclosure
of information concerning variability.

 Price insurance and reinsurance policies.

The premiums charged to policyholders need to include an appropriate loading to allow


for the possibility that claims will exceed their expected cost. Reinsurance arrangements
need to be set up that include an appropriate level of protection to cover periods in which
claims costs are higher than expected.

Question

Most of these points relate to ‘reserve risk’. Explain what this means.

Solution

This could be defined as the risk in respect of financial losses that could arise if the actual claim
payments required exceed the amounts reserved for.

Communicating the outputs of a stochastic reserving exercise is an important part of the


process, as is communicating the limitations and assumptions around the derivation of the
estimates.

The mathematical derivation of the results is complicated. But we can explain the outputs
by simple measures such as graphs of the distribution of possible outcomes and tables of
the key percentiles.

The Actuarial Education Company © IFE: 2019 Examinations


Page 6 SP7-16: Stochastic reserving models

2 Stochastic reserving methods


We base most claims reserving methods on assumptions about the underlying shape of the
claims run-off. The assumptions usually define a mathematical model of the run-off. The
difference between stochastic and non-stochastic methods is that in stochastic methods,
we model not only the underlying pattern of the claims run-off but also some of its
variations.

In other words, we will be modelling the random variation around the chosen development
pattern.

The stochastic approach offers three main benefits:

 We can estimate the reliability of the fitted model, and the likely magnitude of
random variation.

 We may apply statistical tests to the modelling process to verify any assumptions
and gain understanding of the variability of the claims process.

 We can develop models in which the influence of each data point in determining the
fitted model depends on the amount of random variation within that data point. In
other words, figures with large random components should have relatively little
influence.

Question

Suggest any drawbacks of using a stochastic approach for reserving.

Solution

There are a number of drawbacks to stochastic reserving. For example:


 it takes more time
 it requires a higher level of skill and training
 the methods are more complicated, so the risk of mistakes is greater and they are harder
to explain to a non-technical audience
 a considerable element of judgement is required in the choice of model and in selecting
the prior distribution with Bayesian methods
 using more sophisticated methods may lead to spurious accuracy and false confidence in
the results.

© IFE: 2019 Examinations The Actuarial Education Company


SP7-16: Stochastic reserving models Page 7

3 Types of error

3.1 Model error


Model error arises because actuarial models are often a simplification of a very complex
(and unknown) underlying system. By using a simplified model to project the true
underlying system, an unknown bias is introduced into the model. This results in
uncertainty in the estimates produced by the model.

In addition to model error, there are two further sources of error, parameter (or estimation)
error and process error.

Parameter error arises from the fact that the estimated parameters are random variables. So
even if we use the best possible data, the random variation inherent in the data means that we
may interpret it incorrectly and hence choose inappropriate parameter values. Another name for
parameter error is estimation error.

Process error reflects the inherent random noise in the process. In other words, even if we have
built a model which perfectly describes the real world system, the actual result will (almost
certainly) differ from our estimates due to random variation.

Once we have selected a type of model to use and a data set to apply it to, the two remaining
types of error that will be present in our final predictions of the reserves will be estimation error
and process error. These are usually quantified by considering their variances, which leads to the
following equation:
Prediction variance = Estimation variance + Process variance

In line with the usual terminology used in statistical theory, the estimated standard deviation of
the predicted value is referred to as the standard error.

The standard or prediction error of a model is the estimation error plus the process error.

3.2 Testing the model


It is important to test the appropriateness of any model that we use. We can do this in the
following ways:

 Use F tests to check the appropriateness of the number of parameters. This means
testing if we can remove one of the parameters without increasing the residual
variability significantly.
Recall that F tests are used in analysis of variance to determine whether the residual
variance can be considered to be purely random, or whether there are additional factors
that need to be explicitly incorporated in the model.
 Fit the model to old data. We ignore the most recent calendar year developments
and test whether the model fit remains reasonable.
This is a form of sensitivity testing. We carry out two projections, one including and one
excluding the most recent year’s data, and see if the results are broadly consistent.

The Actuarial Education Company © IFE: 2019 Examinations


Page 8 SP7-16: Stochastic reserving models

 Use plots or triangles of residuals. For each data point, we have an observed and a
fitted value. The difference between these is the residual error. Dividing this by the
estimated amount of variance for the data point gives a standardised residual. We
expect the mean of these standardised residuals to be nil, and the variance to be
constant. Plots of the standardised residuals against items such as origin year and
development year should show these standardised residuals to be randomly
distributed, as opposed to being clustered in places.

Question

Explain why we would expect the mean of the standardised residuals to be zero and the variance
to be constant.

Solution

If the mean is not zero, then our model is biased. We can improve it by adjusting the values of
one or more of our parameter estimates. (The equivalent in linear regression would be to change
the intercept parameter, the constant in the straight line equation, to move the line up or down
to achieve a closer fit.)

If the variance is not constant, then our model is not capturing all the sources of randomness. We
can improve it by including an extra factor or by applying a function to one of the variables. (The
equivalent in linear regression would be to change to a multivariate model or to take logs of the
original data values before fitting the model.)

In the next few sections, we describe a variety of models that we can use.

© IFE: 2019 Examinations The Actuarial Education Company


SP7-16: Stochastic reserving models Page 9

4 Examples of stochastic models


In this section we will look at some examples of stochastic reserving models.

4.1 Types of model


Stochastic reserving models can be broadly categorised as follows:
 analytical (or ‘analytic’) methods
 simulation methods
 Bayesian methods.

The word ‘analytical’ is used here in the sense that the stochastic element is incorporated directly
into the formulae or statistical distributions specifying the model. No additional statistical
calculations or assumptions about distributions are required.

Bayesian methods use a prior distribution to model the input parameters and then derive a
posterior distribution for the results. You may have met Bayesian methods in your previous
studies.

The table below summarises the main types of reserving models – deterministic and stochastic.
Other models do exist but they are not covered here.

CLAIMS RESERVING MODELS


Chain ladder
DETERMINISTIC
ANALYTICAL Bornhuetter-Ferguson (BF)*
MODELS
Average cost per claim
Mack
Over-dispersed Poisson (ODP)*
Negative binomial
ANALYTICAL Normal approximation to negative binomial
STOCHASTIC
Lognormal
MODELS
Hoerl curves
Merz-Wüthrich
SIMULATION Over-dispersed Poisson (bootstrap form)*
BAYESIAN Bornhuetter-Ferguson (Bayesian form)*

*The BF and ODP models can be used in either a deterministic or a stochastic form.

The Actuarial Education Company © IFE: 2019 Examinations


Page 10 SP7-16: Stochastic reserving models

4.2 A brief description of each model


In this section we’ve given brief descriptions of each of the models in the table to give you a ‘feel’
for the different types of models and the rationale behind them. Note that this goes beyond what
is contained in the Core Reading. Full descriptions (including the relevant equations, which you
are not expected to know for Subject SP7) can be found in the paper Stochastic Claims Reserving
in General Insurance by England & Verrall, which was published in the British Actuarial Journal
(No 37).

Mack model
The Mack model uses past claims data to derive estimates of the mean and variance of the total
ultimate claims arising from each origin period. It makes no assumption or prediction about the
precise distributions involved, and so is described as a distribution-free model.

This model is discussed further in Section 5.2 below.

Over-dispersed Poisson (ODP) model


If claims occurred completely randomly they would conform to a Poisson process, so that the
claim numbers would have a Poisson distribution. Since the variance of a Poisson distribution is
the same as the mean, this might suggest using the same deterministic estimate (= best estimate)
of the reserves to estimate the variance as well.

However, because the claim amounts are not constant, we find that the variance of the reserves
is greater than the mean. They are over-dispersed. This is especially true for the large claims in
the tail of the distribution, where the variation in size is greatest.

With the ODP model, the variance is estimated as   the deterministic estimate, where   1 is a
constant multiplier estimated from the past data.

If we incorporate the additional variance by making a specific assumption about the distribution
involved, this results in an analytical model. However, an alternative approach is to ‘bootstrap’
the past data, ie to apply a Monte Carlo method using the randomness observed in the past
claims data. This then becomes a simulation method.

This simulation approach is discussed further in Section 6 below.

Negative binomial model


This model is similar to the ODP model, except that a negative binomial distribution is used
instead. The parameters required to estimate the mean and variance of the negative binomial
distribution are estimated from the data. This model also incorporates a factor f to ensure that
it is over-dispersed.

Question

Write down what you can remember about the mean and variance of the negative binomial
distribution.

© IFE: 2019 Examinations The Actuarial Education Company


SP7-16: Stochastic reserving models Page 11

Solution

The formulae for the mean and variance of the (type 2) negative binomial distribution are given
on page 9 of the Tables:
kq kq
E(X )  and var( X ) 
p p2

Unlike with the Poisson distribution, the mean and variance are not equal. In fact, since p  1 ,
kq kq
the variance is always greater (because  ). In practice however, the estimated value of p
2 p
p
could be close to 1 (or, in some cases, greater than 1), so the over-dispersion factor  is still
included in this model to give some extra flexibility.

Normal approximation to negative binomial model


As the name suggests, this model is derived by applying a normal approximation to the negative
binomial model.

If we are using incremental claims data, ie the additional claim amounts recorded in each
individual development year, we can sometimes get negative data values. One feature of this
model is that it is not disrupted by the presence of these negative data values, since the normal
distribution, unlike the Poisson or negative binomial, can take negative values.

Question

Suggest why negative data values might arise here.

Solution

The most likely reason for negative values occurring in the incremental claims data is that the
figures are partly based on case estimates, which may have been revised downwards, so that the
total estimated payout for a particular year is now less than it was in the previous year.

Other possible reasons include: salvage, reinsurance recoveries, disputed claims and lower than
expected IBNR.

Log-normal model
The log-normal model uses estimates of the mean and variance to fit a log-normal distribution to
the reserves. Percentiles can then be deduced that allow for the possibility of a ‘long tail’
corresponding to unexpectedly high claims experience.

The Actuarial Education Company © IFE: 2019 Examinations


Page 12 SP7-16: Stochastic reserving models

Hoerl curves
The development of a cohort of general insurance claims over time usually follows a humped
pattern, with relatively few payments at first, then increasing to a maximum before starting to tail
off. A Hoerl curve is a parametric formula (similar to a gamma distribution) that can be fitted to
the graph of the development factors (plotted by development year) to estimate the proportion
of claims that are still outstanding.

Bornhuetter-Ferguson (BF) – Bayesian version


This is an extension of the BF method where, instead of assuming that the exposure measure we
are using has a precise known value, we assume that this follows a prior distribution with a
suitably chosen form.

Remember you can think of the BF method as being a weighted average of a projection based on
the past claims data (eg the basic chain ladder method) and an independent measure of the
exposure (eg the loss ratio method). In the Bayesian form of this model, the estimate based on
the exposure is assumed to be random, taking a value from a particular prior distribution that we
have assumed.

In the diagram below, the known past claims are represented by the darker-shaded upper left
triangle and the projected claims are represented by the lighter-shaded lower right triangle.

BF method – deterministic form

PAST CLAIMS DATA EXPOSURE RESERVES

£ £10m £1m

BF method – stochastic form

PAST CLAIMS DATA EXPOSURE (PRIOR) RESERVES (POSTERIOR)

£
-4 -2 2 4 -4 -2 2 4

This model is discussed further in Section 9.1 below.

© IFE: 2019 Examinations The Actuarial Education Company


SP7-16: Stochastic reserving models Page 13

5 Analytical methods

5.1 Specifying distributions


The first step in estimating the variability of reserves is to formulate an underlying
statistical model by making assumptions about the data. We can attempt this by specifying
distributions for the data or just specifying the first two moments.

Distributions which might be specified for the claims process include:

 over-dispersed Poisson (ODP)

 negative binomial

 normal approximation to negative binomial

 lognormal.

Once we have specified the distribution of either the incremental or cumulative claims, we
fit the parameters.

The normal approximation to the negative binomial has the advantage that it can handle
reductions in claims (for example, savings in incurred claims due to reductions in case
estimates or salvage and subrogation).

Question

Define ‘salvage’ and ‘subrogation’.

Solution

You may often hear the two terms discussed side by side, although they are in reality very
separate concepts.

Salvage: Amounts recovered by insurers from the sale of insured items that had become the
property of the insurer by virtue of the settling of a claim.

Subrogation: The substitution of one party for another as creditor, with a transfer of rights and
responsibilities. It applies within insurance when an insurer accepts a claim by an insured, thus
assuming the responsibility for any liabilities or recoveries relating to the claim. For example, the
insurer will be responsible for defending legal disputes and will be entitled to the proceeds from
the sale of damaged or recovered property.

The Mack model is an example of a method that specifies the first two moments. It is
effectively distribution-free as further assumptions are made with respect to the claims
process, rather than an underlying distribution. We discuss this method in more detail
below.

Many analytical methods are based around the chain ladder method, with the result that the
mean outcome matches that derived by a standard chain ladder.

The analytic methods listed above enable us to calculate the prediction error. That is, we
estimate the process and estimation error together.

The Actuarial Education Company © IFE: 2019 Examinations


Page 14 SP7-16: Stochastic reserving models

Most of the analytical methods focus on the mean and variance of the distribution of
outcomes. It is often very difficult to obtain the full distribution. We usually make
approximations for communication purposes. For example, a commonly used assumption
is that the full distribution is a log-normal distribution with a mean and variance as
calculated. It is then a simple task to calculate and present any percentile figures and
produce a graph of the full distribution.

It is important to communicate the model error risk involved with such an approach.

5.2 Mack model


The Mack model was proposed by Thomas Mack in an academic paper published in 1993.

The best-known analytical model is the Mack model. The Mack model reproduces chain
ladder estimates and makes limited assumptions about the distribution of the underlying
data, specifying the first two moments only.

The key assumptions are that:

 the run-off pattern is the same for each origin period (as for the chain ladder)

 the future development of a cohort is independent of historical factors (eg high


factors in one period do not imply high or low factors in the following period)

 the variance of the cumulative claims to development time t is proportional to the


cumulative claims amount to time t  1 .

The model produces standard errors for both individual origin periods and for all periods
combined. The formulae required for deriving the Mack standard errors are quite
straightforward to implement in a spreadsheet.

As we mentioned earlier, the Mack model uses the past claims data to derive estimates of the
mean and variance of the total ultimate claims arising from each origin period. The standard
errors are the square roots of the estimates of these quantities.

The Mack model is distribution-free, in that no distributional assumptions are made, only
assumptions about the first two moments. As with many other analytical methods,
however, a full predictive distribution is not derived, although we often approximate this by
fitting a log-normal distribution with the same mean and variance.

Question

Suggest why a log-normal distribution might be appropriate here.

Solution

The log-normal distribution is skew with an extended upper tail. For many classes of business,
this more accurately reflects the shape of the loss distribution then does a symmetrical
distribution, such as the normal distribution.

© IFE: 2019 Examinations The Actuarial Education Company


SP7-16: Stochastic reserving models Page 15

It is important to keep in mind that the non-parametric estimation of mean and variance and
often arbitrary choice of distribution makes this method particularly vulnerable to producing
inaccurate results at the extreme ends of possible outcomes.

‘Non-parametric estimation’ in this context simply means that the Mack model does not assume a
distribution about the underlying data.

The Mack model can handle negative claim increments, as are commonly found within
incurred claims data, although problems may exist when data is very sparse.

For example, the formula for the variance will break down if very large negative increments cause
the cumulative claim amount to be negative at any point.

5.3 Merz-Wüthrich model


For certain applications such as reserve risk estimation for Solvency II capital modelling, it
is necessary to obtain an estimate for reserve uncertainty over a one-year time horizon.
This risk can be measured by estimating the uncertainty surrounding the claims
development result, which is the difference between an estimate of the undiscounted
ultimate claims cost made now, and an estimate made in a year’s time, taking into account
the claims development and emergence of new information during the year.

The claims development result can therefore be thought of as the profit (or loss) in the reserves
over a one-year time horizon.

One approach for estimating the reserve uncertainty over a one-year time horizon is the
Merz-Wüthrich method, described in Section 4.6 of the book ‘Claims Reserving in General
Insurance’ by David Hindley, 2017, Cambridge University Press.

The Merz-Wüthrich method is an analytic approach and so does not rely on simulation.
Essentially, it uses the same assumptions as the Mack model, except that it considers uncertainty
over a one-year period, whereas the Mack model does so over the lifetime of the liabilities. It is
therefore effectively a one-year equivalent of the Mack model. Its close relationship to the Mack
model also means that it can be implemented in the same spreadsheet or programming
framework as the Mack model.

Without adjustment, the method does not include the functionality required to include a tail
factor and only produces an estimate of the uncertainty surrounding the claims development
result, as opposed to its full distribution.

The Actuarial Education Company © IFE: 2019 Examinations


Page 16 SP7-16: Stochastic reserving models

6 Simulation methods
Most analytic methods do not derive a full distribution of outcomes.

Usually they just give the mean and variance of the distribution.

In contrast, we can use simulation methods such as the Monte Carlo method to obtain
predictive distributions of reserves. Although we do not derive the full mathematical form
of the distribution, we obtain sufficient information (such as percentile tables and frequency
plots) to communicate results.

6.1 Introduction to bootstrapping


A simple yet very powerful simulation method is to use bootstrapping techniques.
Bootstrapping involves sampling (with replacement) multiple times from an observed data
set to create a number of pseudo data sets. We can then refit the model to each new data
set and obtain a distribution of the parameters.

Bootstrapping is a generic process that we can apply to a wide range of statistical


problems, provided the model is well-specified.

Here is an example (not based on general insurance) to illustrate the idea behind bootstrapping.
The word ‘bootstrapping’ is taken from the seemingly impossible task of trying to lift yourself up
by your own bootstraps (shoelaces). This method appears to derive additional information about
the statistical properties based only on the data itself, which might at first sight seem to be an
impossible task.

Example

You own a portfolio of UK shares and the recent turmoil on the stock markets has made you
wonder about the average return you might get over the next 10 years.

Describe how you could ‘bootstrap’ the data for the past 25 years of returns on the FTSE 100
index to estimate the distribution of the average return over the next 10 years.

Solution

You could go through the following steps:


1. Get the past returns for 25 years (eg 1993 = 20%, …, 2017 = 8%).
2. Select 10 different values at random from this set of 25 numbers (allowing repeats), to
represent a simulation of the returns for the next 10 years.
3. Calculate the average return based on this sample of 10.
So you might get:
Year 1 = 4%, Year 2 = 12%, …, Year 10 = –10% fi Average = 7%.
4. Repeat steps 2 and 3 another 999 times.
5. Draw a bar chart with the results of your 1,000 calculations from step 3.

© IFE: 2019 Examinations The Actuarial Education Company


SP7-16: Stochastic reserving models Page 17

The results might look something like this.

Average return on FTSE 100 over 10 years


(derived by bootstrapping)

10%
Probability

5%

0%
-10% -5% 0% 5% 10% 15% 20%
Average return

So we have bootstrapped the past data to find the distribution of the average return over the
next 10 years. We could use this distribution to work out confidence intervals and the ‘value at
risk’ over this period.

6.2 Bootstrapping a generalised linear model


It is important to realise that bootstrapping is not a model, it is a procedure applied to a model. It
would be equally possible to bootstrap a generalised linear model, or Mack’s model, or come to
that, many other types of model.

With regression-type problems (and GLMs), the observations are not identically distributed
because the means (and possibly variances) depend on each data point. So for these problems, it
is common to bootstrap the residuals rather than the data points themselves, because the
residuals are often assumed to be approximately independent and identically distributed.

Therefore the steps that should be followed when bootstrapping a GLM would be:
1. Define and fit a GLM, obtaining parameters and fitted values for the observed data. This
should be familiar to you from your previous studies.
2. Calculate the residuals of your fitted model.
3. Take a sample from the residuals (this is the ‘bootstrapping bit’), and invert these to obtain a
set of pseudo-data.
4. Refit the GLM using this pseudo-dataset, to obtain another set of parameters for the model,
and another forecast output.
5. Repeat steps 3 and 4 many times to derive a forecast output for each pseudo-dataset. This
gives a distribution of parameters and outputs.

The Actuarial Education Company © IFE: 2019 Examinations


Page 18 SP7-16: Stochastic reserving models

6.3 Bootstrapping the ODP model – the theory


We often use the term ‘bootstrapping’ in the context of claims reserving to refer to
bootstrapping the ODP model.

You may recall that a generalised linear model assumes that the data comes from the exponential
family of distributions. One member of the exponential family is the over-dispersed
Poisson (ODP) distribution.

The term ‘bootstrapping the ODP’ is widely used, and in essence it means that we are:
 fitting a GLM to the incremental claims data, using an ODP distribution as our underlying
assumption
 then bootstrapping the residuals using the five-stage process above.

The method is based on modelling incremental claims on the assumption that they follow
an ODP distribution (that is, that the variance is proportional to, but not necessarily the
same as, the mean).

We note that we can extend the bootstrapping process to incorporate a whole variety of
models. An example of an extension is the incorporation of the Mack method.

Assumptions
The key assumptions include:

 the run-off pattern is the same for each origin period (as for the chain ladder)

 incremental claim amounts are statistically independent

 the variance of the incremental claim amounts is proportional to the mean

 incremental claims are positive for all development periods.

The last assumption implies that the method is not always appropriate for modelling
incurred claims, where often incremental development is negative. We require adjustments
for the method to work on such data.

Question

Explain why this method would go wrong if there were negative incremental claims present.

Solution

Since the variance of the claims at each stage is assumed to be proportional to the mean (and the
proportionality constant  is positive), this can lead to negative estimates of the variance arising
in some years.

© IFE: 2019 Examinations The Actuarial Education Company


SP7-16: Stochastic reserving models Page 19

Model specification
Here we give a brief technical description of how the model is defined.

Incremental claims in origin year i and development year j are assumed to be modelled as the
random variable Cij where:

C ij ~ ODP(ij , j )

Therefore:
 expected incremental claims E[Cij ]  ij

 var[Cij ]   j ij

Note that a constant scale parameter  could be used for all development years. It is
recommended however that the scale parameter  j be defined to depend on the development
year. This makes intuitive sense, eg you might expect the variance of incremental claims in the
first development period to be very different from the variance in the tail.

As with the Poisson distribution, the log link function is used.

The linear predictor is ij  c  ai  b j . We set the link function equal to the linear predictor (as
we would do for any generalised linear model), to get:
log(ij )  c  ai  b j

This yields the same estimated future incremental claims estimate E[C ij ] as a basic chain ladder
analysis.

Pearson residuals would then be calculated as:


C ij  ij
rij 
 j ij

 
A set of pseudo data Cij* is obtained from:

C ij*  rij*  j ij  ij

where rij* is the sampled residual for origin year i and development year j.

There are plenty of bootstrapping discussions available on the IFoA website.

The Actuarial Education Company © IFE: 2019 Examinations


Page 20 SP7-16: Stochastic reserving models

6.4 Bootstrapping the ODP model – the practice


The ODP model is widely bootstrapped because it is relatively straightforward to implement
in a spreadsheet.

When using bootstrapping in claims reserving, the idea is to fit a model to the past claims data,
which we can then use to calculate expected figures for each past origin and development year.
We then calculate the residuals, the differences between the actual figures in the past data and
the expected figures that we’ve calculated. These residuals tell us about the likely size of the
random component, and it is these values that we sample from to carry out the bootstrap
process.

The process used to bootstrap reserve estimates consists of the following stages (repeated
many times):
1. Calculate the expected values and the residuals for each point in the claims triangle.
2. Re-sample (with replacement) from the residuals to obtain a new triangle.
3. Re-fit the chain ladder model to the new triangle to obtain a revised reserve
estimate.

We can look at these steps in more detail.

Step 1
In Step 1 we fit a model (eg chain ladder) and calculate what the past claim amounts ‘should’ have
been if they had conformed precisely to the model with no random errors – in other words, if
each figure in the past data had exactly followed the development factors we’ve estimated. This
process is sometimes called ‘back-fitting’.

The differences between the actual values and the fitted values then give the residuals.

PAST CLAIMS DATA FITTED VALUES RESIDUALS

£ £ £

© IFE: 2019 Examinations The Actuarial Education Company


SP7-16: Stochastic reserving models Page 21

Step 2
In Step 2 we use the residuals and the fitted values from Step 1 to calculate a large number of
possible alternative sets of past data incorporating the randomness present in the residuals.

FITTED VALUES RESIDUALS ALTERNATIVE PAST DATA SETS


...
£ £ ...
+
...

Step 3
In Step 3 we apply the same method (eg chain ladder) to carry out projections for each of the sets
of alternative past data. This gives us a distribution of the possible reserve estimates
incorporating the randomness in the residuals.

ALTERNATIVE PAST DATA PROJECTED RESERVES DISTRIBUTION


... ...
... ...
... ...
We can apply bootstrapping with varying levels of sophistication. The simplest approach is to
assume that all the residuals follow the same distribution, whereas a more advanced approach
would assume that the distribution can vary by origin year and development year, which would
involve sampling each year separately in the bootstrapping stage.

6.5 Bootstrapping the ODP  relating the theory to the practice


There is often some confusion as to why this process is called ‘bootstrapping the ODP’. After all, it
looks as though we are merely sampling the residuals from the basic chain ladder method. So
why isn’t it called ‘bootstrapping the basic chain ladder’?

Well, the ODP model is a form of GLM, and we know we can bootstrap GLMs because we
discussed this above.

Recall that in the ODP model, the incremental triangle is assumed to follow an ODP distribution
(ie, the variance of the incremental claim amounts is assumed to be proportional to the mean).

But it so happens that for a special case of the ODP model (which we have described above), the
expected values obtained happen to be exactly the same as the basic chain ladder estimates. So
for that special case (this is the clever bit) – we can perform a sleight of hand and fit the chain
ladder model at steps (1) and (4) instead of an ODP GLM.

The Actuarial Education Company © IFE: 2019 Examinations


Page 22 SP7-16: Stochastic reserving models

6.6 Estimating parameter and process uncertainty from a bootstrapped model


Bootstrapping the ODP provides parameter variance. By simulating an observed claims
pattern for each future cell from an appropriate distribution, we can estimate process
variance as well.

By considering the five-stage process above, we can see how the bootstrapping process can give
us an estimate of parameter uncertainty and process uncertainty:
 We make an assumption about process uncertainty in step 1, when we specify the
variability in the underlying model. In the ODP model, this would be determined by the
scale parameter, ie the relationship between the mean and the variance.
 However, by repeating step 4 many times, we also obtain a set of re-fitted parameters for
each dataset, ie a full distribution of parameters. So this gives us our parameter
uncertainty.

The combination of the parameter uncertainty and process uncertainty gives us the uncertainty of
the projection:
Prediction variance = Estimation variance + Process variance

We have seen this relationship already, in Section 3.

6.7 Actuary-in-the-box
Actuary-in-the-box is a method that begins with a best estimate reserve at the start of the
year and a defined algorithm that has been used to derive those reserves and can be
repeated at a future point in time.

The algorithm might be a standard chain ladder procedure or may involve other methods,
such as Bornhuetter-Ferguson.

The algorithm may include other features, such as smoothing of development factors and the
estimation of tail factors.

The requirement is for the algorithm to be repeatable without any element of subjective
input.

The next step in the process is to simulate the claims development for the following year. The
methods used here can include bootstrapping or a parametric simulation from a statistical
distribution. Allowance can also be made for process error.

The algorithm is then reapplied to the data triangle including the additional year of ‘simulated’
claims development, to produce a best estimate of the reserves at the end of the following year.

The claims development result can then be calculated by looking at the difference between the
estimates of ultimate claims at the start and the end of the year.

© IFE: 2019 Examinations The Actuarial Education Company


SP7-16: Stochastic reserving models Page 23

The process is repeated for a suitable number of simulations, whereby a different claims
development for the following year is estimated under each simulation. This produces a full
empirical distribution for the claims development result, from which any required statistics,
eg percentiles can be derived. The claims development result can be considered for individual
cohorts or across all cohorts combined.

The procedure can be enhanced further by making an explicit adjustment for inflation. For
example, if a chain ladder method has been used in the algorithm, then this could follow the
inflation adjusted approach, with future inflation being stochastic.

The approach can also be extended so that the claims development in more than one future year
is simulated. This enables the uncertainty in the claims development result for each year
between now and ultimate to be estimated.

This approach is superior to the Merz-Wüthrich method described above as it can:


 be used to derive the full distribution of the claims development result in each future year
 allow for a tail factor to be incorporated in the process.

The procedure is also known as the ‘re-reserving’ approach.

Further detail is set out in Section 4.8.2 of the book ‘Claims Reserving in General Insurance’
by David Hindley.

6.8 Recognition or emergence pattern methods


When the ultimate reserve risk has been estimated or derived from external sources the
emergence pattern method can be applied to estimate the reserve risk over a one-year
horizon. It is based on the observation that the ultimate risk must emerge over time
between now and ultimate. The method relies upon being able to estimate the proportion of
the ultimate reserve risk that emerges over the next year. Typically for short-tail classes the
ultimate risk emerges quickly, whereas for longer-tail classes the emergence is slower.

The method might be used when there is insufficient data to allow analytical (eg the
Merz-Wüthrich method) or simulation methods (eg the actuary-in-the-box method) to be used to
derive the one-year reserve risk.

The ultimate reserve risk can, for example, be expressed as the difference between the reserves
at the 99.5th percentile and the best estimate reserves. Other approaches include using the
coefficient of variation of the reserves on an ultimate time-horizon basis and finding a way to
adjust this so that it is on a one-year basis.

Options for estimating an emergence pattern for the ultimate reserve risk include:
 using an estimated claims payment pattern
 using an estimated claims payment pattern, but adding an element of stochastic variation
by assuming that the proportion paid in each year follows some chosen statistical
distribution, eg normal

The Actuarial Education Company © IFE: 2019 Examinations


Page 24 SP7-16: Stochastic reserving models

 applying one or more one-year reserve risk methods and combining them with an
estimate of the ultimate reserve risk to derive an emergence pattern, using smoothing if
required
 using suitable industry benchmarks or other data.

Further detail is set out in Section 4.8.3 of the book ‘Claims Reserving in General Insurance’
by David Hindley.

© IFE: 2019 Examinations The Actuarial Education Company


SP7-16: Stochastic reserving models Page 25

7 Aggregation across multiple lines of business and correlations


In the methods discussed so far, we consider a single line of business and derive a
distribution of possible outcomes. From the overall financial perspective of a company, we
need an aggregate distribution covering all lines of business.

We can use analytical methods to aggregate across distributions. However, this can be
difficult. Simulation methods provide a much simpler framework for aggregating across
lines of business. After simulating the run-off on a class-by-class basis, we can then sum
across lines of business by simulation. However, we need to consider dependencies
between lines of business. Otherwise, we are assuming that the run-offs between lines is
independent and (on the basis that lines of business are positively correlated) this will
underestimate the variability of the aggregate distribution.

Question

Explain why this would lead to an underestimation of the variance.

Solution

It is likely that the claims from different classes will be positively, rather than negatively,
correlated. This leads to an increase in the combined variance. For example, with two classes we
have var( X  Y )   2X   Y2  2   X  Y , and the last term will be positive if the correlation
coefficient  is positive.

Lines of business can be correlated / dependent because:

 they are impacted by similar events, eg a windstorm could impact both household
and commercial property accounts

 legal changes often affect several lines of business, eg a change to the Ogden
discount rate would affect both employers’ liability and motor classes

 the same claims team may handle claims from several lines of business and so
changes to claims handling may impact more than one line

 problems with data may affect more than one line of business.

The Ogden Tables (which we met earlier in the course) are used for determining the annuity
factors to be used in certain types of UK compensation claims.

7.1 Copulas
The dependency is usually modelled using a copula and correlation matrix.

Copulas and correlations will be introduced again later in the course, so don’t worry too much at
this stage if you’re not very familiar with the concept. In that section, we give a little more detail
about the mathematics of copulas.

A copula is a way of building a multivariate distribution such that dependencies of the


underlying variables are represented.

The Actuarial Education Company © IFE: 2019 Examinations


Page 26 SP7-16: Stochastic reserving models

Copulas are a more flexible (and complex) way of modelling multiple dependencies, rather than
using single correlations.

The user must specify:


1. Underlying loss distributions for the classes of business.
2. A two-way correlation matrix between all distributions.
3. The form of the copula.

The form of the copula describes how the copula links the underlying distributions. For example,
a Gumbel copula (described further later in the course) gives a strong correlation between the
tails and is also non-symmetric, making it suitable for many insurance applications.

The Core Reading below recaps a few forms of copulas. These are covered in more detail in
Subject CS2.

The copula approach maintains rank correlation. Hence if the user inputs a rank correlation
measure, eg Spearman’s rank correlation coefficient, this will be maintained in the
outputted result.

The simplest form of copula is the Gaussian (normal) copula. This is often criticised for not
giving enough dependency in the tail, and hence failing to model extreme events. Other
copulas such as the Gumbel copula and t-copula remedy this.

In the case where the underlying distributions and copula are normal, the copula can
maintain linear correlation. So, inputting linear correlation coefficients, ie Pearson
product-moment correlation coefficient, will result in an output preserving these
coefficients.

It can be difficult to parameterise dependencies from data; therefore judgement is


important.

© IFE: 2019 Examinations The Actuarial Education Company


SP7-16: Stochastic reserving models Page 27

8 Issues surrounding stochastic reserving


As with all methods, stochastic reserving methods are only as good as the underlying
assumptions. This section outlines some of the issues surrounding stochastic reserving.

8.1 Model forms


There can be mismatches between the type of model and the data to be used. For example,
for log-normal models we must ignore any negative increments (because we take the log of
the incremental movements). Generally, this is not a problem for paid claims triangles
(unless there are significant salvage or subrogation recoveries), but this method often does
not work well for incurred claims data, where there are likely to be more instances of
negative increments.

The ODP model is slightly more flexible because individual negative increments for any
development period are possible, provided the development factor across the development
period as a whole is greater than one.

The Mack model is very flexible in its model form because it allows negative increments and
development factors less than one across a whole development period.

In some circumstances data adjustments can be made to address these problems.

8.2 Latent claims


The stochastic methods described above tend not to be suitable for certain types of claims,
in particular latent claims, since they are only able to reflect the variability reflected in the
claims data available.

A key feature of latent claims is that, by their very nature, we don’t know how they are going to
develop in the long run.

A possible alternative is to use an exposure-based method where assumptions are made


about the volatility of the number of future claims, and the average cost of future claims.

In other words, we model the distributions of the claim numbers and the average claim amounts
separately, and then combine them to find the distribution of the total claim amount.

The issue of latent claims and the fact that they are not reflected in past data is arguably
part of a more general point about any different features not already reflected in the claims
data and that the analytical methods will not capture this kind of variability.

This is discussed further in Section 8.5 below.

Thus, it should be noted that the real variability is almost always larger than Mack or
bootstrapping would suggest.

The Mack and bootstrapping methods can only estimate variability based on the historical data
available. Since the past data will inevitably not include all possible losses, these methods (and
any other methods based on past data) will therefore underestimate variability.

The Actuarial Education Company © IFE: 2019 Examinations


Page 28 SP7-16: Stochastic reserving models

8.3 Sparse data and data peculiarities


As with best estimate reserving, sparse data sets can be problematical for stochastic
methods, as can data peculiarities, such as missing or erroneous data. In particular, small
changes in numbers can lead to significant changes in the distribution of outcomes. The
results can be quite sensitive to individual data points.

Question

Explain how we could identify the extent of this problem.

Solution

We could apply some form of stress testing. For example, we could look at the effects of
increasing or decreasing a few of the data values by 10%, or removing one or two data points.
Alternatively, we could use a Monte Carlo (bootstrap) approach to create some pseudo data sets
and then compare the answers.

Coping with individual data peculiarities is a matter of actuarial judgement. Judgement


forms an important part of stochastic reserving, as it does for best estimate reserving.

8.4 The extremes


For some purposes, we use stochastic reserving to determine the extreme tail of the
distribution of possible outcomes.

However, we parameterise the distribution based on a finite amount of historical data, which
may not be representative of the tail. In addition, in most stochastic methods we make
some simplifying assumptions. These may be reasonable for the most central distribution
of outcomes but may significantly break down at the extremes.

It follows that we should be especially careful when we estimate the tail of a claims
distribution.

For example, a normal distribution is likely to be a reasonable assumption in the centre of the
distribution, but not in the tails.

8.5 Under-estimation of variability


There is consensus that many of the methods described here tend to underestimate the true
variability of reserves. For example, the Mack method has been shown empirically to
underestimate reserve variability.

This is because its central assumption of unchanged development patterns for different
origin periods often does not hold in practice.

More generally, the historical data may not capture all sources of variability to which the
reserves may be subjected in the future (eg potential changes in the Ogden discount rate,
one-off increases in claims costs arising from court judgments, or a prolonged period of
above average inflation).

© IFE: 2019 Examinations The Actuarial Education Company


SP7-16: Stochastic reserving models Page 29

When using the methods described here it is important to use judgement and not to accept
the results of any one method without question.

8.6 Stochastic reserving in practice


As with deterministic reserving methods, an assessment of the reasonableness or validity
of the results is an essential stage of the overall process of applying stochastic reserving
methods before they are communicated to the interested parties.

The detail of this assessment will depend on the methods being used and the purpose of the
exercise and is likely to involve the application of judgement and experience. Where the results
are being used to estimate reserves at higher percentiles, eg 99.5th, then it is particularly
important to validate the reasonableness of these results as they are generally less reliable than
estimates at lower percentiles.

Most reserving software packages that include stochastic methods will include a range of
numerical and graphical analyses to assist with the validation of the results.

Possible examples of the approaches used to validate the results include:


 reconciliation of stochastic results with deterministic results
 graphical review of results
 high-level reasonableness checks of numerical diagnostics
 comparison of results against benchmarks
 back testing of results
 applying stress and scenario tests.

Further detail is set out in Section 4.10.4 of the book ‘Claims Reserving in General
Insurance’ by David Hindley.

The Actuarial Education Company © IFE: 2019 Examinations


Page 30 SP7-16: Stochastic reserving models

9 Alternative approaches
We will conclude this chapter by mentioning some alternative approaches that can be used for
stochastic claims reserving.

9.1 The Bayesian approach


The Bayesian method is another important stochastic reserving method.

Recall that in Bayesian statistics the prior distribution (which captures our beliefs based on what
we know of the exposure) is combined with the likelihood (which reflects the probabilities of the
future claims development deduced from the past claims data) to produce a posterior
distribution. The posterior distribution reflects the probabilities of the future claims development
deduced from both the past claims data and our beliefs based on what we know of the exposure.

The key result from Bayesian statistics is the relationship:


Posterior distribution  Prior distribution  Likelihood

A Bayesian approach can also be used for stochastic reserving. Under the Bayesian theory
framework, the prior distribution of the model parameters is first chosen based on judgment
or experience. Then the posterior distribution of the parameters variable is calculated using
Bayes’ Formula.

The choice of prior distribution depends on many factors including the way in which the
model is parameterised. The parameter being considered may or may not have a natural
interpretation.

Using simulation-based techniques such as the Markov Chain Monte Carlo (MCMC), a
simulated distribution of parameters can be obtained. This approach is an alternative to
bootstrapping to obtain the distribution of parameters (ie parameter uncertainty).

In other words, the two approaches will result in two different sets of model parameters. The
choice of which approach to use can require significant judgement, and is an example of
parameter uncertainty.

Note: the process variance still needs to be incorporated, which is done at the forecasting
stage by simulating from the process distribution conditional on the parameters.

Advantages
One advantage of the Bayesian method is that it provides a complete predictive distribution
of the ultimate reserve. For the other methods, even if the variance can be calculated, the
distribution is not available. Although this distribution from the Bayesian method depends
on the chosen prior distribution, it does give more information. Many statistics, such as
confidence intervals, quantiles or probabilities of extreme values, can be calculated from
the complete predictive distribution.

Recall that the Mack model, for example, is distribution-free. It only predicts the mean and
variance, not the actual shape of the distribution.

Another advantage of the Bayesian method is that it explicitly shows the impact of
judgements, which is reflected in the prior distribution. For other methods, these
judgements are usually implicitly made and it is difficult to evaluate their impact.

© IFE: 2019 Examinations The Actuarial Education Company


SP7-16: Stochastic reserving models Page 31

Similarly to the analytical method, the Bayesian method could give closed-form results
when an appropriate prior distribution is chosen.

Recall that, in Bayesian statistics, there are certain natural combinations of distributions that
arise, such as Poisson-gamma and normal-normal, where the posterior and prior distributions
come from the same family of distributions. These are called ‘conjugate’ distributions. The
posterior distribution in these cases is a ‘closed-form’ distribution, ie it can be described by a
simple formula.

Disadvantages
Despite these advantages, the use of the Bayesian method in stochastic reserving is
subject to the same criticisms as the Bayesian methods in general. In particular, the choice
of prior distribution is subjective, and the posterior distribution may be over-reliant on the
choice of prior distribution.

Other than this, the Bayesian method may not give closed-form results and numerical
integration is needed to get results. However, great progress has been made recently in
this area and the Markov Chain Monte Carlo (MCMC) method can be used to calculate the
integration.

The MCMC technique is required when there is no simple formula for the posterior distribution.
It is an iterative technique where repeated Monte Carlo sampling is used to obtaining increasingly
accurate approximations to the posterior distribution.

An example of the general procedure in which Bayesian methodology is used in a


stochastic reserving context is set out in Section 4.5 of the book titled ‘Claims Reserving in
General Insurance’ by David Hindley.

9.2 Other methods


In some circumstances, it may be necessary to estimate reserves in the absence of any past claims
data, eg when a new line of business has recently been introduced.

Other methods which can also be used to understand the uncertainty of reserves include:

 Use of cedant or market figures / reinsurers’ expertise. However, these methods can
only be used where comparable cover is already offered by other insurers, thereby
providing a source of claims data.
Market data is published by organisations such as the ABI (the Association of British
Insurers) in the UK.
 Policy limits. Where materiality is not an issue or where time pressure prevents any
significant amount of analysis being performed, policy limits provide an upper limit
for the reserve that should be held. A more reasonable approach, however, is likely
to be a proportion of the policy limits, where the proportion is determined by
benchmarking or intuitive means.
Here ‘policy limit’ refers to the maximum claim amount that could arise under the terms
of the policy.
‘Materiality’ is an accounting term that refers to whether a particular item is financially
significant (and therefore its value must be assessed accurately using an established
method) or insignificant (in which case a rough estimate is acceptable).

The Actuarial Education Company © IFE: 2019 Examinations


Page 32 SP7-16: Stochastic reserving models

 Stress and scenario tests around the most significant assumptions and key areas of
uncertainty.

 Intuition / professional judgement. Professional judgement is always a useful


cross-check for an actuary, or other person carrying out the reserving, using any
personal insight, views, beliefs and knowledge in order to produce an initial estimate
for comparison with the other methods and actual experience over time. By looking
at the differences between these estimates a much greater understanding can be
gained.

As time progresses and claims experience data can be gathered, this additional information
can be incorporated into the reserve estimation process using a Bayesian credibility
approach:
z  E  (1  z)  A

where z is the weight attached to the original premium basis estimate (E) and A is the
estimate derived from the claims data to date.

The Bornhuetter-Ferguson method is considered in Chapter 15 (Triangulation methods) and


is one example of this approach.

Questions that arise


 How do we allow for correlations? Firstly between development factors and
secondly between triangle ultimates and initial expected ultimates, particularly
where the initial expected loss ratios are trended from triangle estimates for earlier
origin years.

 If the mean values of the triangle and initial expected values are very similar, does
this give a wider range than if the chain-ladder and initial expected values are
completely different (in which case the range of possible results might be
considered to be rather narrow)?
This shouldn’t happen. If the mean values of the triangle and initial expected values are
very similar, we would expect a narrower range than if the chain-ladder and initial
expected values are completely different.
 The Bornhuetter-Ferguson method is usually considered to be a credibility approach
between two methods, the chain ladder and the naive ultimate loss ratio method,
and hence statistically superior to the two underlying methods. Mack has
suggested that the Bayesian model is a credibility approach between two methods,
namely chain ladder and Bornhuetter-Ferguson, and that using this model would
therefore give results that are statistically superior to Bornhuetter-Ferguson (but
only if the naive loss ratio is in itself a credible estimate).
In other words, the BF method is superior because it’s a credibility-weighted version of
two other methods, and the Bayesian model is more superior still, because it’s a
credibility-weighted version of the BF method and the chain ladder method. However, we
still need a suitable a priori estimate of losses.

© IFE: 2019 Examinations The Actuarial Education Company


SP7-16: Stochastic reserving models Page 33

Chapter 16 Summary
Stochastic reserving
Stochastic claims reserving can be used to:
 assess reserve adequacy
 compare different estimates and datasets
 monitor performance
 allocate capital
 provide information to investors
 facilitate discussions with regulators
 price insurance and reinsurance policies.

Model error arises because models are simplifications of reality. We also get parameter (or
estimation) error, and process error (random noise) in models.

The uncertainty present in a reserve prediction (prediction error or standard error) can be
split into two components:
Prediction variance  Process variance  Estimation variance

A model can be tested by:


 examining plots or triangles of residuals
 using F tests to establish which parameters to include
 fitting the model to past data.

The Actuarial Education Company © IFE: 2019 Examinations


Page 34 SP7-16: Stochastic reserving models

Chapter 16 Summary continued


Types of models
Stochastic claims reserving models can broadly be split into:
 analytical methods
 simulation methods
 Bayesian methods.

In analytical models the stochastic element is incorporated directly into the formulae
specifying the model.

Simulation methods often use the bootstrap method, which involves three steps:
1. Calculate the expected values for the past claims data by back-fitting, and then
calculate the residuals.
2. Take samples from the residuals to create a number of alternative past data sets.
3. Calculate projections for each alternative data set and use these to find the
distribution of the reserves.

Bayesian models use a prior distribution to model uncertainty in the model parameters.

Examples of models

The Mack model uses a similar framework to the chain ladder method. It produces an
estimate of the variance, but does not specify the precise distribution. It can be used with
either paid or incurred data. Key assumptions of the Mack model are:
 the run-off pattern is the same for each origin period
 future development of a cohort is independent of historical factors
 the variance of cumulative claims is proportional to the cumulative claim amount
from the previous period.

The Merz-Wüthrich method essentially uses the same assumptions as the Mack model,
except that it considers uncertainty over a one-year period, whereas the Mack model does
so over the lifetime of the liabilities.

The over-dispersed Poisson (ODP) model can be bootstrapped. This assumes that the
variance exceeds the best estimate by a constant factor  1 that is estimated from the
data. This model cannot handle negative values that might be present if incremental claims
data is used. Key assumptions of the ODP model are:
 the run-off pattern is the same for each origin period
 incremental claim amounts are statistically independent
 the variance of incremental claim amounts is proportional to the mean
 incremental claims are positive for all development periods.

© IFE: 2019 Examinations The Actuarial Education Company


SP7-16: Stochastic reserving models Page 35

Chapter 16 Summary continued


Actuary-in-the-box is a method that begins with a best estimate reserve at the start of the
year and a defined algorithm that has been used to derive those reserves and can be
repeated at a future point in time. This approach is superior to the Merz-Wüthrich method
as it can be used to:
 derive the full distribution of the claims development result in each future year
 allow for a tail factor.

When the ultimate reserve risk has been estimated from external sources, the emergence
pattern method can be applied to estimate the reserve risk over a one-year horizon. The
method relies upon being able to estimate the proportion of the ultimate reserve risk that
emerges over the next year. It might be used when there is insufficient data for analytical
methods (eg Merz-Wüthrich) or simulation methods (eg actuary-in-the-box).

The negative binomial model is similar to the ODP model, but is based on the negative
binomial distribution. A normal approximation can be used within this model, which
prevents problems arising from negative values.

A log-normal model can also be used to model reserves and calculate percentiles.

Hoerl curves can be used to fit a parametric formula to the claims development curve.

The Bayesian form of the Bornhuetter-Ferguson model assumes that the exposure measure
follows a particular prior distribution. If a very vague prior distribution is assumed, the
results are similar to the ODP model. Again, this model cannot handle negative values.

Certain models are limited by the type of model or data that can be fitted. A key problem is
with instances of negative increments in incurred data. Data adjustments can sometimes be
used in such cases.

Stochastic models can be unreliable when applied to latent claims. Exposure-based methods
are better here.

Models fitted using sparse data (eg claims in the extreme tail of the distribution) can be very
sensitive to small changes. Judgement is required in these cases.

Care is required in the tail of the claims distribution because data may be inadequate and
assumptions may not be valid at the extremes.

Many stochastic models can underestimate the true underlying variability, eg if this is not
reflected in the past data, or due to changes in development patterns.

The Actuarial Education Company © IFE: 2019 Examinations


Page 36 SP7-16: Stochastic reserving models

Chapter 16 Summary continued


Alternative approaches
Bayesian methods use a prior distribution for the variable in combination with the data to
produce a posterior distribution for the predicted variable.

Bayesian methods have several advantages:


 they provide a complete predictive distribution
 they explicitly state the subjective judgement used
 closed-form distributions can often be obtained.

They also have some disadvantages:


 as with other Bayesian methods, the choice of prior distribution is subjective
 numerical methods are required when there is no closed-form distribution.

In some circumstances, it may be necessary to estimate reserves in the absence of any past
claims data. Methods that can be used in these circumstances include:
 using market data or data from reinsurers
 applying a percentage to the policy limit (maximum possible claim amount)
 stress / scenario tests
 using professional judgement and experience.

The Bornhuetter-Ferguson method, which uses a credibility approach, can be used in this
context.

© IFE: 2019 Examinations The Actuarial Education Company


SP7-16: Stochastic reserving models Page 37

Chapter 16 Practice Questions


16.1 In Australia, reserves must be booked at the 75th percentile confidence level (discounted at the
risk-free interest rate). With this method, the amount of reserves held is equal to the 75th
percentile of the aggregate loss distribution (the 3rd quartile), rather than the best estimate. This
amount includes a margin to allow for reserve risk.

An Australian insurer believes that a particular claim will be settled by a single payment of
amount X in 2 years’ time. The distribution of X is such that:

 P(X  $50,000)  0.25

 P(X  $100,000)  0.5

 P(X  $250,000)  0.75

If the risk-free interest rate is i  5% , calculate the reserve that must be included in the insurer’s
accounts for this claim.

16.2 A stochastic model should allow for correlations across multiple lines of business. Give examples
of how heavy rain could simultaneously impact five different lines of business.

16.3 (i) Explain the components of the variance of stochastic reserve predictions. [3]
Exam style
(ii) Let Ci , j denote the cumulative claim amount recorded for origin year i ( i  1,2, , n ) and
development year j ( j 1,2,, n ).

The table below shows the values of Ci , j (in £ millions) for a particular general insurance
portfolio.

Ci , j j 1 j 2 j 3 j 4

i 1 60 100 120 130

i 2 50 80 95
i 3 25 45

i 4 10

According to the Mack model, the expected value and the variance of Ci , j 1 are equal to:

E C i , j 1    j C i , j and var C i , j 1    2j C i , j

Here,  j denotes the development factor for the period from year j to j 1 and  2j is a
variance parameter that is estimated as:
2
1 n j  Ci , j 1 
ˆ 2j  
n  j  1 i 1
Ci , j 
 Ci , j
 ˆj  , j  1,2,, n  2

 

The Actuarial Education Company © IFE: 2019 Examinations


Page 38 SP7-16: Stochastic reserving models

(a) Write down the formula you would use to estimate  j , and calculate the values
of the estimates ˆ1 and ˆ2 .

(b) Calculate the estimated values of E C1,2  and E C1,3  and state how these
compare with the corresponding best estimates based on the basic chain ladder
method.

(c) Calculate the value of the estimate ˆ22 .

(d) Hence calculate the estimated value of var C1,3  and explain what this quantity
represents. [11]
[Total 14]

16.4 (i) Explain why a stochastic claims reserving model might be preferred to a deterministic
Exam style
model. [6]

(ii) Describe the data required for the Mack model and the output it produces. [3]

(iii) Explain how stochastic claims reserves for several lines of business should be
amalgamated. [4]
[Total 13]

16.5 Explain the meaning of each of the following terms in the context of stochastic claims reserving:
Exam style (i) bootstrapping [6]
(ii) Bayesian methods [4]
(iii) over-dispersion. [4]
[Total 14]

© IFE: 2019 Examinations The Actuarial Education Company


SP7-16: Stochastic reserving models Page 39

Chapter 16 Solutions
16.1 From the information given, the 75th percentile of the claim amount X is $250,000.

So the reserve required equals the discounted value of this amount, which is:
250,000
 $226,757
1.052

Note that this figure could be a lot higher than the best estimate of the claim amount (which we
can’t work out based on the information given).

16.2 Heavy rain can lead to an increase in claims for:


 motor insurance (eg pile-ups on motorways)
 household claims (eg water damage caused by flooding)
 personal injury claims (eg people falling on wet surfaces)
 liability claims (eg if people defer repairs until the weather improves)
 healthcare claims (eg hospital admissions for pneumonia / hypothermia).

16.3 (i) Components of prediction variance

One of the objectives of a stochastic reserving model is to estimate the level of uncertainty
present in the best estimate of the reserve. This is quantified using the prediction variance. [1]

The prediction variance can be split into two components:


Prediction variance  Process variance  Estimation variance [1]

The estimation variance reflects the fact that our best estimate is derived from a model, which
will not be perfect because it may not be the most appropriate model, it is based on a finite
amount of data and it may depend on other assumptions as well. [½]

The process variance reflects the fact that, even if we have a perfect model, because of the
inherent variability in the claims process, the actual value of the reserves will almost certainly
differ from the best estimate we calculate. [½]
[Total 3]

The Actuarial Education Company © IFE: 2019 Examinations


Page 40 SP7-16: Stochastic reserving models

(ii)(a) Estimates of the development factors

̂1 is the development factor relating year 1 to year 2. This is estimated in the usual way, by
calculating the ratio of the cumulative figures available for development year 2 to the
corresponding figures for development year 1:
C1,2  C2,2  C3,2 100  80  45 225
ˆ1     1.6667 [1]
C1,1  C2,1  C3,1 60  50  25 135

Similarly, ̂2 is the development factor relating year 2 to year 3, which is calculated as:

C1,3  C2,3 120  95 215


ˆ2     1.1944 [1]
C1,2  C2,2 100  80 180

The general formula for these estimates is:

C1, j 1  C2, j 1    Cn j , j 1 n j n j


ˆj    Ci , j 1  Ci , j [1]
C1, j  C2, j    Cn j , j i 1 i 1

(ii)(b) Estimates of the expected values

Using the formula given, with i  j  1 , the value of E C1,2  is estimated as:

E C1,2   ˆ1C1,1  1.6667  60  100 [1]

Using the formula given, with i  1 and j  2 , the value of E C1,3  is estimated as:

E C1,3   ˆ2C1,2  1.1944  100  119.44 [1]

These are the same as the estimates based on the basic chain ladder method. [1]

(ii)(c) Estimate of the variance parameter

Using the formula given, with j  2 and n  4 , the value of ˆ22 is estimated as:
2
1 2  C i ,3 
ˆ22   C i ,2   ˆ2 
1 i 1 C 
 i ,2 
2 2
 C1,3   C2,3 
 C1,2   ˆ2   C2,2   ˆ2 
 C1,2   C2,2 
   
2 2
 120   95 
 100   1.1944   80   1.1944 
 100   80 
 0.00694 [3]

© IFE: 2019 Examinations The Actuarial Education Company


SP7-16: Stochastic reserving models Page 41

(ii)(d) Estimate of the variance

Using the formula given, with i  1 and j  2 , the value of var C1,3  is estimated as:

var C1,3    22C1,2  0.00694  100  0.694  (0.833)2 [1]

Here we are considering C1,3 , the cumulative claim amount for origin year 1 in development
year 3, to be a random variable. The actual value we observed for this variable was £120m.
According to the Mack model, this value comes from a distribution with mean £119.44m and
standard deviation
£0.833m. [1]
[Total 11]
16.4 (i) Stochastic versus deterministic reserving models

A deterministic reserving model, such as the chain ladder method, only provides a single best
estimate of the reserve required. [½]

A deterministic reserve gives no information about the uncertainty present in the reserve
estimate. [½]

In reality the actual amount ultimately required to pay the claims may differ from the best
estimate, eg because of:
 variations in the occurrence and severity of claims [½]
 notification delays [½]
 legal changes, levels of litigiousness, court rulings [½]
 inflation [½]
 changes in the mix of claims [½]
 changes in claims handling procedures. [½]

It is important to allow for the uncertainty in the estimates:


 for solvency purposes [½]
 to determine capital requirements and capital allocation [½]
 to assess reinsurance requirements [½]
 to comply with regulations [½]
 for investors and policyholders to compare different insurers [½]
 when pricing insurance and reinsurance policies [½]
 when comparing different data sets [½]
 when monitoring performance to see if claims are reasonable [½]
 when communicating results. [½]
[Maximum 6]

The Actuarial Education Company © IFE: 2019 Examinations


Page 42 SP7-16: Stochastic reserving models

(ii) Data and output for Mack model

The Mack model has similar data requirements to the basic chain ladder method, namely:
 paid or incurred claims … [½]
 split by origin year and development year … [½]
 in incremental or cumulative form … [½]
 in real amounts or money amounts. [½]

The output from the model consists of a best estimate and a standard error for the best estimate
for each origin year and for all origin years combined. [1]

The Mack model is distribution-free. It does not produce a precise distribution, although a two
parameter distribution such as normal or log-normal can easily be fitted using the mean and
variance. [1]
[Maximum 3]

(iii) Amalgamating different lines of business

The best estimates for different lines of business can simply be added together to find the overall
best estimate. [½]

However, the variances (or the distributions) cannot be simply added together because of
correlations between claims from different lines. [1]

For example, bad weather can lead to increased costs for household, motor, agricultural, personal
injury, health insurance etc. [½]

It is difficult to use analytical methods to determine an aggregate claims distribution. [½]

Simulation methods provide the best approach here. [½]

The results from individual simulations can be added up and collated to obtain the aggregate
claims distribution. [½]

However, such models must make appropriate allowance for the correlations between different
lines. [1]

These correlations are usually positive, which means that ignoring them will result in
underestimating the variability of the reserves. [½]
[Maximum 4]

© IFE: 2019 Examinations The Actuarial Education Company


SP7-16: Stochastic reserving models Page 43

16.5 (i) Bootstrapping

Bootstrapping is a technique for determining the statistical properties of a quantity by using the
randomness that is present in a sample from the underlying population, and then applying a
Monte Carlo approach. [½]

It involves sampling (with replacement) repeatedly from an observed data set in order to create a
number of pseudo-data sets that are then consistent with the original data set. [½]

Various statistics of interest can then be derived for each pseudo-data set, and the distribution of
these statistics can be analysed further. [½]

It is assumed that the sampled data are independent and identically distributed. [½]

In stochastic claims reserving we can use bootstrapping techniques to estimate the distribution of
reserve predictions. [½]

This method assumes that:


 the run-off pattern is the same for each origin period [½]
 incremental claim amounts are statistically independent [½]
 the variance of the incremental claim amounts is proportional to the mean [½]
 incremental claims are positive for all development periods. [½]

This approach involves:


 obtaining a set of past claims data, split by origin / development year [½]
 back-fitting a model to the past data to find the expected claims for each cell [½]
 calculating the residual ‘noise’ present in each cell (ie actual minus expected) [½]
 sampling from this residual distribution to produce many pseudo-data sets [½]
 calculating reserve projections based on each pseudo-data set [½]
 collating the reserve projections to determine the distribution, moments and percentiles
of the reserve distribution. [½]
[Maximum 6]

(ii) Bayesian methods

Deterministic reserving methods assume that the observed claim amounts conform to a statistical
model involving parameters that have fixed but unknown values. [½]

Bayesian methods, on the other hand, assume that the parameters in the model do not have a
fixed value, but themselves conform to a certain prior distribution. [½]

If we combine an assumed prior distribution for the parameters with a model for the
development of the claims, we can find the posterior distribution for the parameter. This
combines our initial beliefs about the parameter with the additional information provided by the
data. [½]

The Actuarial Education Company © IFE: 2019 Examinations


Page 44 SP7-16: Stochastic reserving models

The posterior distribution can then be used (analytically) to calculate moments and percentiles for
the reserves. [½]

One common Bayesian method is the Bayesian version of the Bornhuetter-Ferguson model. [½]

This approach involves:


 obtaining a set of past claims data, split by origin / development year [½]
 selecting a prior distribution to model the exposure measure (which involves selecting a
type of distribution, eg a gamma distribution, and assigning suitable values to the
parameters, eg alpha and lambda) [1]
 determining the posterior distribution for the projection of the claims reserves (which
may require a Monte Carlo approach) [1]
 using the posterior distribution to determine moments and percentiles for the claims
reserves. [½]
[Maximum 4]

(iii) Over-dispersion

‘Over-dispersion’ refers to a distribution where the variance exceeds the mean. [½]

For the Poisson distribution, the variance equals the mean. [½]

A Poisson distribution can be considered as the starting point for modelling claims since it is the
distribution underlying the Poisson process, which is the theoretical model for events that occur
completely randomly over time. [½]

In practice, claim amounts do not follow a Poisson distribution because claim amounts are not
constant and claims do not occur independently. [½]

It is usually found that claims distributions are over-dispersed. [½]

One common stochastic reserving model based on this principle is the over-dispersed Poisson
(ODP) model in which the variance is estimated as   the deterministic estimate, where   1 is
a constant multiplier estimated from the past data [1]

If we incorporate the additional variance by making a specific assumption about the distribution
involved, this results in an analytic model. [½]

An alternative approach is to bootstrap the past data. [½]


[Maximum 4]

© IFE: 2019 Examinations The Actuarial Education Company


SP7-17: Assessment of reserving results Page 1

Assessment of reserving
results
Syllabus objectives

3.4 Evaluate reserving result analyses.


3.4.1 Describe the factors an actuary should consider in assessing the
reasonableness of the results of a reserving exercise.
3.4.2 Describe typical diagnostics that are commonly used to assess the
reasonableness of the results of a reserving exercise.
3.4.3 Describe the factors an actuary should consider in assessing the
reasonableness of changes in results of a reserving exercise over time.
3.4.4 Describe how an analysis of experience might be carried out in the context
of a reserving exercise.
3.4.5 Describe how alternative results of reserving exercises can arise and
highlight some of the professional issues in resolving them.

The Actuarial Education Company © IFE: 2019 Examinations


Page 2 SP7-17: Assessment of reserving results

0 Introduction
In previous chapters we have considered the process of calculating reserves. In this chapter we
consider the reasonableness of the outcome of such reserving calculations. It will be important
that the actuary checks that the figures are justifiable and that the methodology and assumptions
adopted are appropriate. A range of diagnostic tools and analysis of emerging experience can
help with this process.

In Section 1 of this chapter we consider why it is important to analyse the results of the reserving
exercise.

Sections 2 and 3 then look at various diagnostics (measures for interpreting data or results) for
claims and the claim development pattern,

The analysis of emerging experience is covered in Section 4, considering why current experience
may be different from that expected.

Section 5 describes the underwriting cycle and explains why it should be taken into account in the
reserving exercise.

Section 6 describes the reserving cycle and the affect this can have on an insurer’s reserves.

In Section 7 we consider why different people carrying out the same reserving exercise may have
different results.

© IFE: 2019 Examinations The Actuarial Education Company


SP7-17: Assessment of reserving results Page 3

1 Assessing the results of a reserving exercise

1.1 Introduction
Reserving is a core activity for actuaries in general insurance work.

The area of assessing the results of a reserving exercise is one of the most critical stages in
the reserving process. It is here that we can be required to make the key judgements that
underpin the analysis carried out. In doing so, we often need to ask ourselves two
questions:

 Are the results selected reasonable?

 Are the new results supported by an analysis of emerging experience? In practice,


we may have begun a reserving exercise by looking at the approach and results
from the previous review.
For example, actuaries will often carry out an ‘Actual versus Expected’ exercise. This
involves comparing the actual experience in the period since the last review, to the
experience that was expected to have emerged in that time period. The selected
assumptions may then be updated to reflect the most recent trends in experience.
It is important for us to bear in mind the dangers of anchoring error whereby too
much weight is given to the previous methodology and assumptions.

Anchoring describes the common human tendency to rely too heavily, or ‘anchor’ on one trait or
piece of information, perhaps past claims inflation, when making decisions.

Usually once the anchor is set, there is a bias toward that value.

Let’s take a (non-actuarial) example: A person looking to buy a puppy may automatically search
for the same breed as their previous dog, without considering that a different breed may by
better suited to their current lifestyle.

We need to distinguish carefully between changes in methodology and changes in


assumption, that is, reasons for the changes and diagnoses of the impacts.

The approach that we follow may satisfy only one of the criteria illustrated by the following
scenarios:

 We do an analysis where we fit new models to estimate reserves. This may yield
results that appear reasonable, but the estimates may change over time (due to the
change in model), even where there has been no material change in the underlying
data and information available. While there may be circumstances where such a
change of basis is appropriate, this can make communication of results more
difficult.

 We follow an approach that considers changes in results over time in response to


data, but we do not monitor whether the results themselves still appear reasonable.
We may fail to identify situations where results gradually cease to be reasonable
over a period of time. We need to be aware of the risk of anchoring to results that
are no longer supportable.

The Actuarial Education Company © IFE: 2019 Examinations


Page 4 SP7-17: Assessment of reserving results

1.2 Approaches to analysing the results


It is important that we are careful when reviewing the results of a reserving exercise to
avoid the pitfalls that can arise from either of these scenarios mentioned above. We can
achieve this by undertaking two types of analysis:
1. applying diagnostic tests to check that results are reasonable
2. carrying out an analysis of the emerging experience.

A balance needs to be achieved between the additional understanding gained from applying
these analyses versus the additional cost and time involved. In this chapter we will look at both of
these types of analysis.

We should consider undertaking an overview of the whole exercise for reasonableness and
to ensure that there are no gaps or overlaps in any analysis performed.

Finally, we must make a judgement, taking into account the analysis and our experience,
when making a final selection of results.

Judgement will form a key part of any actuary’s assessment of results. We can think of our
experience as providing an implicit benchmark for expected results. We will have a view
based on our own experience, for example, on how frequently a particular event may occur,
or what we would expect a reasonable range for a particular diagnostic to be.

What would be reasonable will vary with the type of business written, the target market and over
time.

However, we should be careful as our experience may either be limited, or not typical.

For example, one particular actuary may have experience of personal lines business but not
commercial lines, or London Market business. Where this is the case professional conduct
standards require that help from a more experienced actuary is sought.

Peer review can help here, where another actuary can review and challenge our
methodology and assumptions.

© IFE: 2019 Examinations The Actuarial Education Company


SP7-17: Assessment of reserving results Page 5

2 Diagnostics

2.1 An introduction
A diagnostic is a measure used to assist with interpretation of data or results and to help us
test and verify the underlying methodology and assumptions. It can indicate that
experience is inconsistent with the assumptions. Some diagnostics are a test of results
(eg IBNR divided by premium). Others test the data (eg paid claims divided by incurred
claims). Others test both (eg ultimate loss ratios).

In other words, diagnostics are often used to provide a high-level ‘reasonableness’ check of
results, or to indicate which areas of the portfolio should be examined more closely.

Question

Suggest what we can learn from examining:


(i) IBNR divided by premium
(ii) ultimate loss ratios.

Solution

(i) IBNR divided by premium can highlight errors or inconsistencies in the reserving process
or a change in premium rating strength (more on this later in the chapter).

(ii) Ultimate loss ratios can indicate where there have been changes in the stringency of
claims underwriting, an improvement or worsening of claims experience or a change to
the rating basis.

2.2 Interpreting the diagnostics


We should ensure that we understand the reasons for changes in diagnostics over time,
and any unusual features highlighted by them.

Question

Suggest what we should do if the diagnostics highlight unusual features.

Solution

We should:
 investigate the reason for the unusual feature
 understand the implications for the reserving process
 take appropriate action, eg change methodology or assumptions if necessary.

The Actuarial Education Company © IFE: 2019 Examinations


Page 6 SP7-17: Assessment of reserving results

We may have to consider how these diagnostics vary over differing data groupings (that is
accident, underwriting, calendar, reporting and development period).

For example, we would expect that for any given development period, IBNR divided by premium
for employers’ liability business would be higher than for household property business.

Diagnostics relating to development periods are covered in the next section.

Changes in diagnostics over time, or unusually high or low figures, may result from
unexpected emerging experience that is considered to be a one-off.

For example, there may have been a change to terms and conditions which would preclude this
experience from happening again.

In this case, we may feel that the methodology and assumptions continue to be appropriate.
In other cases, we may identify limitations in the reserving methodology and assumptions
and decide to make changes.

It is necessary to consider materiality when addressing features revealed in diagnostics.

Where an analysis of the diagnostics does highlight unusual features in experience, an actuary
may decide not to update their methodology or assumptions if the resulting change in reserves is
immaterial relative to the size of the company’s total reserves. This decision will also depend on
the purpose of the reserving exercise.

When we interpret diagnostics, it is critical to understand the underlying reasons for the
behaviour shown. It is important to consider whether the diagnostics fall in an expected
range. If they do not, we should consider whether this indicates a concern with the results
or identifies an underlying feature of the experience that has not previously been taken into
account.

This is analogous to the feedback stage of the actuarial control cycle, ie notice any deviation,
understand the reasons for the differences and then feedback by making appropriate
adjustments.

Where possible, we should exclude special features such as large losses to avoid providing
a distorted picture, but also consider the appropriateness of any reserve for the loss.

When examining the diagnostics, exceptional items should be excluded, although of course it will
be important that the end reserve does include an allowance for such items.

It is important to consider the granularity (that is, what level to subdivide the data) at which
the diagnostics are reviewed and assumptions made. We may do this by reviewing the
results and diagnostics at a higher level.

The remainder of this section discusses how various diagnostics can be used within a
reserving analysis.

© IFE: 2019 Examinations The Actuarial Education Company


SP7-17: Assessment of reserving results Page 7

2.3 Loss ratios


We should review changes in loss ratios (some or all of paid, outstanding, IBNR, incurred
and ultimate as the situation dictates). These may highlight some of the following:

 Changes in premium rating strength (see further discussion later in this chapter on
the underwriting cycle). Adjusting underlying premiums for premium rate
movements may remove this effect (a rate index can be used for this purpose) but
beware of rating increases imposed to reflect increased risk.

 Sources of uncertainty (eg exceptionally large open claims in a particular cohort).

 Inconsistencies in the model assumptions (eg if the progression of IBNR loss ratios
is not monotonically decreasing with age of the claims cohort).
A monotonically decreasing function is a non-increasing function.
 Errors in the reserving process (eg misapplied Bornhuetter-Fergusson model).

The premiums on policies may not be known until some time after the end of the period of
exposure. For example, retrospective experience rating may be operating, or there may be an
adjustment for exposure at the end of the year (eg on employers’ liability to reflect the number of
workers covered over the year).

In the case where premium estimates are unavailable to derive ultimate loss ratios, we could
consider the ultimate claims as a percentage of another measure of exposure (for example,
policy or claim count, insured turnover and so on).

We can compare loss ratios to premium and claims indices or available benchmarks and
decide whether they look sensible in comparison. Other things being equal, an increasing
premium rate index should lead to a lower loss ratio.

By reviewing the paid and incurred loss ratios, we can see at an early stage how the
experience to date has turned out for each origin year.

For example, where loss ratios are unexpectedly high, we can then investigate further
whether this is due to a unique claim or type of claim, or is an early indicator that claims
experience is going to be materially worse than expected.

Considering IBNR as a percentage of case estimates is useful in those situations where a


complete paid or incurred claims development history is not available.

This will be useful for long-tailed classes, or immature portfolios where the earliest development
year is yet to be fully run-off.

The Actuarial Education Company © IFE: 2019 Examinations


Page 8 SP7-17: Assessment of reserving results

2.4 Paid to incurred and/or case estimates to incurred ratios


This diagnostic can indicate the strength of case estimates. An increasing ratio trend over
time, when we are reviewing a triangle of cumulative paid claims divided by cumulative
incurred claims, may have a number of possible explanations, such as:

 case estimate strength has been reduced


If the ratio of paid claims to incurred claims has increased, this may indicate that the
strength of case estimates has reduced.
However, if the ratio of case estimates to incurred claims has increased, this may indicate
that the strength of case estimates has increased.
Care needs to be taken that the definition of incurred claims (ie what has been included)
has not changed.
 an underlying change in business

 an acceleration in the claims settlement pattern

 a slow-down in the rate at which outstanding claims are established

 a distorting large loss settlement.

Question

A general insurer writes public liability insurance and reserves using the basic chain ladder
method. On examining the relationship between paid and incurred claims it is found that the
ratio is decreasing over time. Suggest what impact this will have on claims development and
hence the reserves calculated.

Solution

Actual claims development is slower than that suggested by the use of the basic chain ladder. If
no adjustment is made to the basic chain ladder method, then the reserves calculated will be an
underestimate of the amount required. If the reserves are also discounted, then the payments
will be assumed to be made earlier than will eventually be the case, and will therefore be affected
to a lesser extent by discounting. This will offset to some extent the inaccuracy in the basic chain
ladder model. A revised model should take account of both of these factors.

We should try to understand the business reasons underlying these changes and, in each
case, consider whether to make adjustments to the approach taken; for example, use a
different subdivision of the data.

2.5 Average outstanding case estimate


A review of this triangle can highlight changes in the strength of case reserves.

© IFE: 2019 Examinations The Actuarial Education Company


SP7-17: Assessment of reserving results Page 9

2.6 Ratio of IBNR to case estimates


For more mature cohorts, this diagnostic is helpful, particularly where IBNR is expected to
be mainly in respect of IBNER, rather than ‘pure’ IBNR claims.

Such a diagnostic gives a feel for the outstanding claims and the uncertainty relating to them.

2.7 Survival ratios


Survival ratios show how long a reserve or IBNR estimate will last (before all outstanding
claims are paid) if current paid or incurred claims development continues at a given rate.
We may base the ratios on an average of three to five years’ experience, when we analyse
very long-tailed classes such as those with asbestos losses. We can compare these
survival ratios with other similar portfolios or market benchmarks.

A ratio often used to estimate asbestos loss reserve adequacy is the three-year survival ratio,
which is the ratio of loss reserves to the three-year paid loss average. Assuming that average paid
losses remain constant, with no additional reserving, the survival ratio indicates how many years
the reserves should last.

2.8 Claim frequency and average cost per claim diagnostics


These diagnostics are useful where claim count information is available. Changes in
frequency and severity may highlight many of the features described above for other
diagnostics. They may also identify inflationary trends in claims costs and trends in claim
frequency per unit of exposure. Where policies have a maximum claim value, it may be
useful to compare average claim size with this value.

Question

A general insurer writes employers’ liability business. On investigation it is found that claim
frequency per unit of exposure is increasing. Suggest possible reasons for this.

Solution

Reasons include:
 change in policy terms and/or conditions
 emergence of a new type of claim
 increasingly litigious society
 weaker initial and/or claims underwriting.

A triangulation of the number of settled claims divided by the number of reported claims
can indicate the stability of the claims settlement process.

For classes of business where open nil claim counts (that is, number of claims where no
payment has yet been made) are recorded, monitoring these may provide an early warning
of an anticipated increase in claims costs or problems arising in processing claims.

The Actuarial Education Company © IFE: 2019 Examinations


Page 10 SP7-17: Assessment of reserving results

2.9 Reinsurance to gross ratios


We can apply all the above diagnostics gross and net of reinsurance. In addition, it is
valuable to consider how gross, reinsurance and net estimates interact and the ratios
between them.

If a reinsurance programme remains unchanged for various origin years, and claims
experience remains relatively constant, we would expect such ratios to remain fairly
uniform. However, changes in these ratios are common and we should understand these
when we consider the reasonableness of the reinsurance or net assumptions. While this is
often more straightforward when considering proportional reinsurance, it can be useful to
consider the ratios for non-proportional business as well; but note the impact of a fixed limit
and deductible is likely to reduce with inflation.

Question

A general insurer writes domestic household business in the US. The ratio of gross to net of
reinsurance claims has been declining over the last 10 years. Suggest the possible actions that
may need to be taken.

Solution

Lower reinsurance recoveries are being made. The insurer needs to consider whether this is
acceptable. A deliberate decision may have been made to self-insure to a greater extent, in which
case no action is required.

If this is not the case then the insurer should review the reinsurance programme, both the types
and extent of cover, in order to achieve the desired reinsurance recovery as a proportion of gross
claims.

Changes in these ratios can be a result of:

 changes in the amount of business being retained or ceded by the insurer

 changes in the mix of non-proportional and proportional reinsurance cover

 changing policy terms, such as deductibles, limits and reinstatements

 changes in the underlying gross experience

 inconsistencies in the treatment of gross and net claims estimates.

Alternatively, we can use net of reinsurance to gross of reinsurance ratios.

© IFE: 2019 Examinations The Actuarial Education Company


SP7-17: Assessment of reserving results Page 11

3 Development pattern diagnostics

3.1 Introduction
One of the most important sets of diagnostics for a reserving exercise are those comparing
assumed future development patterns with past development patterns. Clearly, this can
only be carried out where there is relevant prior history.

We can carry out such a review by considering an array showing data split by origin cohort
and development period. A common approach is to consider cumulative (paid or incurred)
claims development as a proportion of the estimated ultimate claims for a given origin
cohort. We can then compare differing origin years readily.

Example
A check for calendar-year distortions could involve examining whether there are diagonals in our
run-off triangle with a bias towards ‘high’ or ‘low’ development factors.

The median development factor is obtained (M) and then for each development period we can
determine whether each development factor is the median (M) or is high (H) or low (L) relative to
the median.

settlement delay in years (development year)


0 1 2 3 4
2014 L H H M H
2015 L M L H
2016 L H H
2017 L L
2018 H

For each diagonal of the run-off the numbers of H’s and L’s are counted. In the absence of any
calendar-year effects the numbers of H’s and L’s should be about the same.

If they are not then this may suggest some adjustments need to be made to the development
factors.

Some actuaries find this is most easily performed graphically, but we should be careful.

These graphs can mask the more detailed information shown in the triangle above, and do not
give an indication of the materiality of each origin year. However, they can give a useful
indication of whether the changes in development patterns need to be analysed more fully.

3.2 Changes in the development pattern


Such an analysis can show features of the data and the volatility of the development
pattern. It can also show differences in development of each year and evidence of changing
development patterns over time where this exists.

The Actuarial Education Company © IFE: 2019 Examinations


Page 12 SP7-17: Assessment of reserving results

Question

You have a run-off triangle of paid claims split by accident year. Suggest possible reasons for:
(i) a row of figures that is unusually high
(ii) a column of figures that is unusually high
(iii) a diagonal of figures that is unusually low.

Solution

(i) This represents unusually heavy claims in a certain accident year. Possible reasons could
be heavy flooding or an unusually bad winter resulting in a large number of claims.

(ii) A column represents the payments made a certain number of years after the year of the
accident. A high figure may imply that we settle many claims in the early years (property
damage claims) and then there is a fall off in payments before liability claims are paid.

(iii) A low diagonal of figures represents a fall-off in payments being made by the insurer in a
given calendar year. This could be due to implementation of a new system, loss of staff or
a postal strike.

We would normally expect origin years to be more developed (where development has a
positive tail) as they get older, unless there are particular reasons for this not to be the
case. Such reasons may include:

 external influences such as inflation, catastrophes or changes in the underlying


nature of the risks

 internal influences such as changes in underwriting, claim handling and settlement


and recording procedures or reinsurance arrangements

 changes in the type of business attracted within a class and types of claim emerging

 random fluctuations or large claims (or even lack of expected large claims) in a
portfolio

 if policy limits have been exhausted on some or all of the policies in a portfolio, that
would limit the scope for further deterioration of the incurred claims position.

We should seek to understand the reasons for features in the development pattern for each
individual origin year and ensure that the underlying assumptions are appropriate in light of
this.

3.3 Stability of development pattern


Where the development pattern is volatile, we should consider whether the estimates are
reliable considering this. As the scope for adverse development (that is, an increase in
aggregate claims) often exceeds that for favourable development (that is, a reduction in
aggregate claims), the distribution of final outcomes will often be positively skewed. We
should consider whether we have allowed for a sufficient tail beyond the claims
development experience to date.

© IFE: 2019 Examinations The Actuarial Education Company


SP7-17: Assessment of reserving results Page 13

Question

Give examples of factors that would lead to favourable development and those leading to adverse
development.

Solution

Favourable development could be a result of faster claims handling or a propensity towards


small / simpler claims with shorter reporting and settlement delays.

Adverse development could be a result of a change in experience leading to more complex / large
claims with longer reporting and settlement delays. An accumulation of claims due to one event
(eg a hurricane) could also cause a slowing down of settlement. If the claims team take longer to
deal with claims this will also adversely affect development.

3.4 Comparison between classes


The speed of development patterns (that is, how quickly claims develop to their ultimate
position) will vary by class.

For example, we would normally expect to see slower development patterns for:

 liability classes compared to property damage classes due to delays caused by


disease development, determination of liability through courts etc
 reinsurance classes compared to the equivalent direct classes due to the delay in
being notified of reinsurance recoveries and settlement of them
 classes with policies attaching at higher layers compared to those with policies
attaching at lower layers as it tends to take longer for larger claims to develop up to the
(higher) attachment point. Larger claims are also likely to be more complex, so it may
take longer for the extent of the claim to be fully understood
 business written on a risk attaching basis compared with a claims made basis. This
is because late reported claims will fall into a subsequent policy year. This was discussed
earlier in the course.

We should consider whether the relative speeds of the suggested development patterns for
the different classes are appropriate.

3.5 Claim development versus premium development


Where there is evidence of a changing premium development pattern, we should consider
the reasons for this, and whether this should affect the claim development pattern. In some
circumstances, it may also be useful to compare premium earning (that is, earned premium)
patterns with incurred claim development patterns.

In theory, the earned premium would be expected to develop to reflect the potential for claims
over the period of exposure. In practice however, the earned premium development pattern is
often estimated using formulaic techniques and such a comparison may be of limited use.

The Actuarial Education Company © IFE: 2019 Examinations


Page 14 SP7-17: Assessment of reserving results

3.6 Comparison to benchmarks


There are a variety of sources of development patterns that could be used as benchmarks.
These include:

 industry and market sources

 other classes of business that are closely related, eg household contents as a proxy
for household buildings
 similar portfolios within the actuary’s experience.

3.7 Residuals of fitted link ratios


The analysis of run-off patterns often places great reliance on estimating individual link
ratios from those observed in the data. Useful insights may be gained from looking at the
size and pattern of the residuals on the individually fitted link ratios.

The residuals of the link ratios could be calculated by taking the selected link ratio for a particular
origin year and development period, and subtracting the actual link ratio for that development
period.

Question

The table below shows the total amounts paid in each development year for each accident year
(in £000s).

settlement delay in years (development year)


0 1 2 3 4
2014 125 104 65 32 9
2015 120 98 57 27
year of accident
2016 148 124 75
(origin year)
2017 143 110
2018 138

Calculate the triangle of residual link ratios, assuming that the selected link ratios are those
derived from the basic chain-ladder method without any adjustment.

© IFE: 2019 Examinations The Actuarial Education Company


SP7-17: Assessment of reserving results Page 15

Solution

We calculate the triangle of development factors from development year x to development year
x 1 :

development factors
0 1 2 3
2014 1.832 1.284 1.109 1.028

year of accident 2015 1.817 1.261 1.098


(origin year) 2016 1.838 1.276
2017 1.769
BCL development factors 1.813 1.274 1.104 1.028

(We calculated these basic chain ladder development factors in an earlier chapter.)

So the residual link ratios are:

residuals
0 1 2 3
2014 -0.019 -0.010 -0.005 0.000

year of accident 2015 -0.004 0.013 0.006


(origin year) 2016 -0.025 -0.002
2017 0.044

Widely-spread or skewly-distributed residual patterns may indicate that the estimate of the
link ratio is highly uncertain or perhaps even inappropriate.

Patterns within the residuals can also help identify distortions in the underlying data. For
example, if one diagonal of development factors exhibits consistently high residuals, then this
would indicate a calendar year effect, eg an internal exercise to speed up claim settlement.

The Actuarial Education Company © IFE: 2019 Examinations


Page 16 SP7-17: Assessment of reserving results

4 Analysis of emerging experience

4.1 Why analyse emerging experience?


It is essential to monitor emerging experience to implement the actuarial control cycle
effectively. The environment in which a general insurer operates is constantly changing.
Monitoring the effect of past actions can help in revising a firm’s strategy for risk
management and in reassessing the risks that it faces.

Question

(i) Explain why an insurer might have to revise decisions after an analysis of experience.

(ii) A general insurer has analysed the experience of its commercial buildings portfolio. List
the key stakeholders to whom deviations in experience from expectations should be
communicated.

Solution

(i) Why revise decisions


The analysis may have revealed that the original decisions were not suitable because:
 they were not good enough, ie wrong
 new information has become available
 the circumstances have changed.

(ii) Key stakeholders


 senior management
 pricing team
 reserving team
 claims department
 underwriters
 reinsurers
 brokers

Where we have estimates and assumptions from the previous review, we should understand
why these may have changed at the current review. We should also communicate the key
changes and their reasons to the stakeholders, so they can understand and provide
feedback if there are additional issues that may not have been considered.

© IFE: 2019 Examinations The Actuarial Education Company


SP7-17: Assessment of reserving results Page 17

For this purpose, it is useful to break down the movement since the previous review in the
estimated ultimate (that is, the final cost when all claims in a cohort have been settled) into
its component parts, namely:

 how emerging experience compares to that expected in the previous model (this will
require the actuarial model used to have this predictive capability)

 changes in methodology

 changes in assumptions.

Such a reconciliation of the figures can aid in understanding how the changes in experience have
arisen and which factors are most significant, eg which individual assumptions have had the
greatest influence.

Different areas of the experience can be examined.

For the incurred Bornhuetter-Ferguson method, for example, these could be the:

 difference between the actual and expected movement in incurred

 change in selected development pattern

 change in selected prior loss ratio

 change in selected premium estimate

 changes in other assumptions, for example inflation, if the inflation-adjusted


Bornhuetter-Ferguson method is being used.

Where we identify a consistent deterioration or improvement in one of these diagnostics, we


should consider whether this is adequately reflected in the assumptions. This is
particularly the case where emerging experience in prior years might suggest that the prior
loss ratios should be increased.

Where we have used the incurred average cost per claim method, we should break down the
total movement into that relating to claim frequency and that relating to claim severity.

We can estimate the expected movement in incurred claims by considering the previous
development pattern and the previous estimated IBNR. For short-tail classes, we would
expect a larger movement in incurred as a proportion of the previous estimated IBNR
(sometimes known as the ‘IBNR burn rate’) than for longer-tailed classes.

It is helpful to understand the reasons for the business developing differently to what was
expected (though this is not always possible). We can then consider whether the
assumptions and methods remain appropriate considering this. For example, the selected
method may inadequately capture certain aspects of the emerging experience.

For example, if inflation is no longer stable then there may be a need to move to a method which
makes an explicit allowance for future inflation as opposed to using a method which assumes past
inflation will continue.

We should watch out for any trends emerging, and consider these in the reserving process.

The Actuarial Education Company © IFE: 2019 Examinations


Page 18 SP7-17: Assessment of reserving results

Where previous reviews have been carried out, there is a danger of anchoring. Where the
previous basis has been used as a starting point, we may not adjust the assumptions
quickly enough considering unexpected developments. However, there is also a danger
that we may overreact in these situations. We must find a balance between the two
tendencies. It is also useful to review periodically how accurate previous estimates and
methods have been. We can learn from this what methods are working well in different
situations.

© IFE: 2019 Examinations The Actuarial Education Company


SP7-17: Assessment of reserving results Page 19

5 The underwriting cycle

5.1 What is the underwriting cycle?


The underwriting cycle is the process occurring over a period of years when premium rates
for a given class of business oscillate between a high level at the top of the cycle (a ‘hard
market’) where the business written is typically profitable, and a low level at the bottom of
the cycle (a ‘soft market’) where the business written is typically unprofitable.

The process of the underwriting cycle is as follows:

In a hard market, where premiums are high, and the business written is typically profitable,
premium rates tend to fall as:

 either additional capacity enters the market in order to gain a slice of the profitable
business (thus increasing supply and therefore putting the pressure on rates to fall)
and/or

 existing insurers cut their rates in an attempt to increase their own market share of
this profitable business.

This process continues until premium rates have fallen to a level where they are no longer
sustainable, and the business being written no longer proves to be profitable: the soft
market.

At this point, rates should begin to harden as insurers either withdraw from the market (thus
reducing capacity / supply and putting the pressure on rates to rise) or those that remain
typically start to increase their rates. However, in practice, reserve releases from reserves
established during the hard market can delay the recognition of losses and the
implementation of corrective action.

Hard market
More companies enter market

Business becomes profitable Profits are squeezed

Soft market
Companies exit the market

The speed at which the bottom of the cycle is reached and rates begin to harden is often
increased by external factors such as large catastrophe claims, which can reduce
profitability and increase the pressure on rates to harden. Rates then continue to increase
(harden) until we are back in a hard market and the process starts all over again.

The Actuarial Education Company © IFE: 2019 Examinations


Page 20 SP7-17: Assessment of reserving results

The length of the underwriting cycle varies by class of business and territory. For example, in
some UK personal lines classes, it is around seven years. It will be dependent on, for example:
 macro-economic effects, eg people pay less for insurance and claim more when economic
conditions are poor
 investment conditions (if it is expected that good returns can be made on invested
premiums then the insurer may be prepared to offer softer premium rates)
 major industry losses, eg natural disasters or terrorism.

5.2 Impact of the underwriting cycle on the assessment of reserves


The underwriting cycle can have an influence on claims development. For example in a hard
market, individuals who perceive themselves as low-risk may choose to self-insure rather than
pay high premiums, resulting in anti-selection against the insurer.

When assessing the reasonableness of the results of a reserving exercise, we should


consider whether we have allowed appropriately for the underwriting cycle.

One way to allow for the underwriting cycle in reserving exercises is to use a rate index
when deriving the initial expected loss ratios for use in credibility-type methods.

We should be careful, however, when selecting appropriate rate indices because:

 Rate indices are typically only available for renewal business and therefore may not
adequately allow for any differences between new and renewed business.

 Rate indices can sometimes be constructed largely based on highly subjective


information (such as the underwriters’ views rather than hard data).

 In addition to changes in the actual premium charged, there may also be changes
over time in the terms and conditions or limits and deductibles of the cover
provided, which will also impact on the profitability of the business being written
and which are not necessarily reflected in the rate indices.
Changes in these elements of the cover tend to vary in relation to different phases of
the underwriting cycle: as the market hardens, insurers remove or reduce the more
optional – and often expensive – parts of the cover, and these then gradually come
back in again as the market softens.
In an ideal world any rate index should attempt to take account of these changes,
which are inevitably more difficult to quantify than pure changes in the premium
charged.

© IFE: 2019 Examinations The Actuarial Education Company


SP7-17: Assessment of reserving results Page 21

Question

Suggest what impact the following will have on claims development patterns:
(i) looser terms and conditions
(ii) lower deductible.

Solution

(i) Looser terms and conditions – leads to claims arising which have different reporting
and/or settlement patterns. It may also lead to more protracted claims due to litigation.

(ii) Lower deductible – leads to a shorter claim development pattern.

The Actuarial Education Company © IFE: 2019 Examinations


Page 22 SP7-17: Assessment of reserving results

6 The reserving cycle


Recent studies have also suggested the existence of a ‘reserving cycle’ which is highly
correlated with the underwriting cycle.

This appears to show that in a soft market, incurred claims development patterns are slower
to develop (or longer-tailed) than in a hard market so that an unadjusted projection can
underestimate ultimate claims in a soft market (and, equivalently, overestimate them in a
hard market, when insurers can afford it).

Potential reasons for this phenomenon include:

 the effect of weakened terms and conditions

 an increasing tendency to dispute claims

 a possibly less conservative approach to case reserving when results are worse.

The evidence of a reserving cycle is more noticeable for business which is already thought
to be longtailed.

The initial expected loss ratio can be chosen to take account of changes in the reserving cycle as
well as changes in the underwriting cycle.

This over-estimation or under-estimation of booked reserves is a decision to be made by


the Board. It should not impact the actuary’s best estimate, although he or she may wish to
indicate a range of ‘best estimates’ within which he or she believes the Board’s decision
should lie.

The general insurer is likely to wish to flatten the reserving cycle:


 so that reserves are more accurate. This reduces the likelihood of insufficient reserves
being set up in past years, which will have a detrimental impact on the ongoing business
 so that the profitability of the business can be more readily understood. Appropriate
decisions can then be made as to whether to continue, contract or expand a class.

© IFE: 2019 Examinations The Actuarial Education Company


SP7-17: Assessment of reserving results Page 23

7 Comparison to other estimates


Chapter 14 explains the various reserving bases that can lead to different results and
explains the reasons for these. Here, we consider the case where two parties independently
investigate the same data over the same period and estimate different results and different
diagnostics.

Benchmarks based on market data or peers are a useful comparison against the various
diagnostics discussed previously.

Care needs to be taken when comparing against market data since the policies offered may have
different terms and conditions affecting the expected claims.

Alternative estimates may come from another actuary or from management (including
claims and underwriting teams).

It is important to compare such results and diagnostics to identify areas of differences


which may put into question the methodology and assumptions used. We should
understand areas of difference and communicate these to the stakeholders.

Possible reasons for differences between two estimates include:

 Data used.
The two estimates may have used different data sets. One dataset may be better in
quality or have additional data or benchmark information.
An extreme example of this would be in the situation of an acquisition where the buyer of
the company will have limited access to data from which to estimate reserves.
Also, one actuary may have been provided with additional data, such as historical
large loss information. This may result in an alternative method being used in the
reserving process, which would normally lead to a different estimate. The provision
and appropriate use of additional data should normally improve the quality of the
results.
This is particularly the case when dealing with underwriting and claims staff who
have much greater familiarity with aspects of the portfolio.

 Methodology.
Very rarely will two different methods produce the same claims estimate.

 Underwriting and claims handling staff.


Additional information from interviews with these staff may influence the actuary’s
selections in the reserving process.
For example, the actuary may allow for an underwriter’s expectations of future loss
ratios. Claims handling staff may point out any speeding up or slowing down of the
claims process, which may influence selections of development patterns. They can
also explain particular claims issues. Evidence of such changes should be sought
in the data, rather than taken as a fact based on staff views.

 Genuine differences of opinion.


The two estimates may reflect the parties’ differing viewpoints on how claims
experience will turn out, based on their own experience and the information
available to them. By comparing differences in approach and information, they may
identify true differences in opinion.

The Actuarial Education Company © IFE: 2019 Examinations


Page 24 SP7-17: Assessment of reserving results

When comparing results with others, the actuary will benefit from being aware of the
following factors that could give rise to professional issues for the actuary:

 The alternative estimates may have been prepared by someone with a financial
interest in the outcome (eg underwriters’ bonuses may be directly linked to the
reserves established).

 The actuary’s estimates may only be challenged for those classes where the
actuary’s estimate is significantly greater than management’s estimate. This may
introduce a downwards bias in the actuary’s final results if only these classes are
revised.

Question

You are a reserving actuary for a general insurance company. Management have commented
that your reserving estimate for a particular class of business looks high relative to their
expectations and have asked if you could reduce the figure by 10%.

Suggest what factors you should consider in deciding whether to adjust the reserve.

Solution

Factors include:
 overall materiality, eg was it a reserving estimate of £6 million or £600 million?
 The purpose of the exercise, eg statutory reporting, or internal management
 whether you consider your current estimate to lie at the low-end or high-end of a range
of reasonable estimates
 the validity of any alternative estimates prepared for the class
 whether a second opinion should be sought, eg seek a peer review
 actions taken by competitors
 regulatory requirements and guidance from profession
 whether there are cushions, ie likely over-reserving for other lines of business
 the overall financial strength (ie level of free assets) of the insurer.

© IFE: 2019 Examinations The Actuarial Education Company


SP7-17: Assessment of reserving results Page 25

8 Glossary items
Having studied this chapter you should now ensure that you are able to explain the following
Glossary items:
 Earned premiums
 Experience rating
 Hard premium rates
 Loss ratio
 Soft premium rates.

The Actuarial Education Company © IFE: 2019 Examinations


Page 26 SP7-17: Assessment of reserving results

The chapter summary starts on the next page so that you can
keep all the chapter summaries together for revision purposes.

© IFE: 2019 Examinations The Actuarial Education Company


SP7-17: Assessment of reserving results Page 27

Chapter 17 Summary
The results of the reserving exercise need to be checked to ensure:
 they are reasonable
 they are supported by emerging experience.

This can be done by analysing:


 diagnostic tests
 analysis of emerging experience.

A diagnostic is a measure used to help interpret data or results. It can test and verify
underlying methodology and assumptions.

It is important to understand the reasons for changes in diagnostics over time.

Common diagnostics include:


 changes in loss ratios
 paid to incurred and/or case estimates to incurred ratios
 average outstanding case estimate
 ratio of IBNR to case estimates
 survival ratios (how long the reserve or IBNR estimate will last if current claim
development continues at a given rate)
 claim frequency and average cost per claim
 reinsurance to gross ratios.

Diagnostics of development patterns should also be considered. If data is split by origin and
development year then different origin years can be compared easily.

The speed of development pattern will vary by class.

The reason for a change in development pattern should be understood.

The development pattern can be checked against benchmarks such as:


 industry and market sources
 other closely related classes
 similar portfolios the actuary has encountered.

Analysis of residuals can indicate trends / distortions in the underlying data and are useful to
check whether the selected development factors are appropriate.

Analysis of emerging experience is central to the actuarial control cycle. If experience is


different from assumptions this should be communicated to key stakeholders.

The Actuarial Education Company © IFE: 2019 Examinations


Page 28 SP7-17: Assessment of reserving results

Chapter 17 Summary continued


The analysis can be broken down into differences due to:
 experience being different based on the previous model
 changes in methodology
 changes in assumptions.

Changes may be needed to assumptions or method as a result of the analysis. Care needs to
be taken to make an appropriate allowance for trends.

Premium rates and loss ratio assumptions should reflect the position in the underwriting
cycle.

Care needs to be taken to reflect the underwriting cycle in the reserving exercise, eg by use
of an appropriate rate index. Development patterns tend to be longer in a soft market, and
this needs to be reflected in the reserving exercise.

A reserving cycle also exists which may affect the published accounts.

Two parties may derive different reserve estimates, even if they use the same data over the
same time period. This may be because:
 the data used was different (eg one had additional information or better quality
information)
 the methodology used was different
 additional information was available from underwriters and claims handlers
 there may be genuine differences of opinion.

When comparing results with others, the actuary should:


 be aware that any alternative estimate may have been prepared by someone with a
financial interest
 be careful that challenges are not just to reduce figures, ie pressure from
management.

© IFE: 2019 Examinations The Actuarial Education Company


SP7-17: Assessment of reserving results Page 29

Chapter 17 Practice Questions


17.1 (i) Define the term ‘anchoring’.

(ii) Explain the danger of anchoring when reserving.

17.2 An actuary has been examining a loss ratio diagnostic based on paid claims and noted that it has
been rising over time. Suggest possible reasons for this trend.

17.3 An insurer writes public liability business. For the larger claims a case estimate approach to
reserving is adopted. The diagnostic of case estimates to incurred claims is being examined.
Explain why there may be a trend for this ratio to increase over time.

17.4 (i) Define the term ‘survival ratio’.

(ii) An insurer has paid the following asbestos losses over the previous three years: $100m,
$120m, $130m. The reserve is set at $900m. Calculate the survival ratio assuming recent
claims history is averaged over three years.

17.5 An actuary is examining a run-off table and has noted that a couple of the older origin years are
less developed than more recent origin years. Explain why this may be the case.

17.6 (i) Describe the underwriting cycle. [3]


Exam style
(ii) Explain the possible impact of the underwriting cycle on the estimation of reserves. [5]
[Total 8]

The Actuarial Education Company © IFE: 2019 Examinations


Page 30 SP7-17: Assessment of reserving results

The solutions start on the next page so that you can


separate the questions and solutions.

© IFE: 2019 Examinations The Actuarial Education Company


SP7-17: Assessment of reserving results Page 31

Chapter 17 Solutions
17.1 (i) Anchoring

Anchoring is the tendency to rely too heavily on one piece of information when making a decision
affected by a range of factors.

(ii) Anchoring and reserving

There can be a danger of anchoring on past experience when setting future claims estimates even
when new trends are beginning to emerge. By not taking enough account of these new trends,
the results gradually cease to be reasonable.

17.2 Possible reasons include:


 a fall in premium rating strength (all else being equal) – the actuary should consider past
changes to the rating structure and the impact of the underwriting cycle
 poor general claims experience, eg an increased propensity of policyholders to claim
 poor claims experience as a result of a unique / large claim or a type of claim
 less stringent claims underwriting
 less stringent policy terms and conditions, eg lower excess levels
 high claims inflation
 an increase in the speed of claim settlement (diagnostics based on claim development
patterns should also be examined)
 if the loss ratio is net, issues linked to the reinsurer, eg default.

17.3 This may be as a result of:


 case estimate strength being increased
 an underlying change in the business resulting in a deterioration of claims experience
 an underlying change in the business resulting in an deceleration in the claims settlement
pattern, eg an increase in the proportion of longer-tail claims
 slow claims settlement on the part of the claims settlement team
 an acceleration in the rate at which outstanding claims are established, ie entered onto
the data system
 a distorting large loss settlement.

17.4 (i) Survival ratio

A survival ratio shows how long a reserve or IBNR estimate will last if current paid or incurred
claims development continues at a given rate.

The Actuarial Education Company © IFE: 2019 Examinations


Page 32 SP7-17: Assessment of reserving results

(ii) Calculation of survival ratio

100  120  130


The claims averaged over the last three years are:  $116.67m
3

900
The survival ratio is:  7.7 years.
116.67

17.5 In general this is not what we would expect to see, ie we would expect older origin years to be
more developed.

However, this may not be the case due to:


 external factors such as:
– catastrophes, ie it may take longer for these claims to settle
– changes in the underlying nature of the risks relating to each origin year, eg due
to external factors
 internal influences such as changes in:
– underwriting or claims handling procedures
– recording processes
– reinsurance arrangements
 changes in the types of business attracted by the class and hence impact on claims
emerging
 random fluctuation or existence / lack of large claims
 exhaustion of policy limits restricting the potential for further development.

17.6 (i) Underwriting cycle

The underwriting cycle reflects the changes in premium rates and underwriting conditions as a
result of changing levels of competition in the market. [1]

The underwriting cycle results in market conditions fluctuating between a hard market and a soft
market:
 hard market
– limited capacity / supply means that premium rates are high, policy terms and
conditions are more stringent and business is very profitable, leading to … [½]
– … more companies entering the market, capacity increasing so that the increase in
competition leads to profit margins being squeezed until … [½]
 soft market
– … lots of capacity / supply means that rates are low, policy terms and conditions
are less stringent and business is no longer profitable, leading to ... [½]
– … some insurers leaving the market, reducing capacity and so the cycle continues.
[½]
[Total 3]

© IFE: 2019 Examinations The Actuarial Education Company


SP7-17: Assessment of reserving results Page 33

(ii) Impact of the underwriting cycle on the estimation of reserves

Appropriate allowance needs to be made for the underwriting cycle in the reserving exercise. [½]

For example, if the methodology is based on loss ratios then an allowance should be made for
changes in premium rates, eg by using a rating index. [½]

Also, if the underwriting cycle has resulted in changes to policy terms and conditions, then this
must be allowed for in any estimated loss ratio, eg when using the Bornhuetter-Ferguson method.
[½]

The development pattern is also affected by the position in the underwriting cycle. [½]

For example, in a soft market the development pattern is longer, possibly due to: [½]
 contract design features [½]
 insurers trying to reduce the rate of claim payout at a time when cashflows are tight. [½]

For business which is analysed on an accident year basis, prior year data will not fully reflect the
effect of the underwriting cycle, since claims relating to any one accident year will have been
written under different underwriting environments. Care should therefore be taken when
applying these loss ratios to future years’ business. [1]

Rate indices are typically only available for renewal business and therefore may not adequately
allow for any differences between new and renewed business. [1]

In a soft market this would lead to premiums being overstated and the prior loss ratio being
underestimated. [½]

This would lead to an underestimation of reserves. Moreover, this effect is exacerbated by the
lengthening of the tail in a soft market so the deterioration in claims experience only comes to
light in later development years. [1]
[Maximum 5]

The Actuarial Education Company © IFE: 2019 Examinations


All study material produced by ActEd is copyright and is sold
for the exclusive use of the purchaser. The copyright is
owned by Institute and Faculty Education Limited, a
subsidiary of the Institute and Faculty of Actuaries.

Unless prior authority is granted by ActEd, you may not hire


out, lend, give out, sell, store or transmit electronically or
photocopy any part of the study material.

You must take care of your study material to ensure that it


is not used or copied by anybody else.

Legal action will be taken if these terms are infringed. In


addition, we may seek to take disciplinary action through
the profession or through your employer.

These conditions remain in force after you have finished


using the course.

The Actuarial Education Company © IFE: 2019 Examinations


SP7-18: Reserving – use of ranges and best estimates Page 1

Reserving – use of ranges


and best estimates
Syllabus objectives

3.5 Assess uncertainty and its communication in reserving.


3.5.1. Discuss what is meant by a ‘best estimate’ reserve.
3.5.2. Describe the following approaches to estimating ranges of reserves:
 stochastic models
 scenario tests
 use of alternative sets of assumptions.
3.5.3. Discuss the uses, advantages and disadvantages of each of these methods.
3.5.4. Discuss the issues to be considered when communicating reserve ranges
and uncertainties.

The Actuarial Education Company © IFE: 2019 Examinations


Page 2 SP7-18: Reserving – use of ranges and best estimates

0 Introduction
Reserving is not an exact science. In various chapters, we discuss techniques that guide
us in how to set a reserve estimate. Our final recommendation will typically be based on
additional qualitative factors and on judgement.

For many types of business it is likely that our initial calculation of a reserve estimate will be
based mostly on quantitative techniques, eg the basic chain ladder method. We would then
refine our estimate using our actuarial judgement.

This judgement can be based on a qualitative assessment of the state of the underwriting cycle,
any changes in the mix of business, changes in distribution sources and changes in policy terms
and conditions. Typical examples of changes in terms and conditions that can have a significant
impact on reserve levels are the use of aggregate policy limits and the expansion of coverage
exclusions.

Inevitably, the ultimate value of the quantity is likely to differ (perhaps significantly) from
our original estimate. This presents two important issues for us:

 We must be clear what we mean by a single point estimate (or best estimate) of an
outstanding liability.
This is covered in Section 2.
 We should attempt to quantify the degree to which the eventual outcome may
diverge from the point estimate and the likelihood of such divergence (that is,
quantify the uncertainty).
This is covered in Section 3.

In this chapter, we describe what is usually meant by a ‘best estimate’ reserve and describe
techniques used to help quantify the uncertainty in eventual outcomes. In Section 4 we
discuss the communication of these topics.

As we will see later this divergence from the point estimate can be described in both quantitative
and qualitative terms.

Finally in Section 5 we discuss the regulatory and accounting issues to consider when estimating
reserves.

There is a lot of overlap between the material in this chapter and the material in Chapter 16.

© IFE: 2019 Examinations The Actuarial Education Company


SP7-18: Reserving – use of ranges and best estimates Page 3

1 Sources of uncertainty
There are a number of sources of uncertainty in actuarial estimates. The forms of
uncertainty, and how we measure them more accurately, are still a developing area of
non-life actuarial practice.

Question

From your memory of earlier in the course, write down as many factors as you can that contribute
to reserve uncertainty.

Solution

Sources of process error


 general claims uncertainty
– inherent uncertainty in individual claims (amount, frequency and timing)
– changes in mix of business
– demand surge
– normal retirement
 internal sources, such as:
– changes in business mix
– booked reserves different to best estimate
– uncertainty over commission and other sales-related expenses
– new markets
– new types of investment
 systematic sources, such as:
– the economic environment
– the insurance cycle

Sources of parameter error


 the data used
– poor quality data
– inconsistent data
– incomplete and non-existent data
 incorrect modelling assumptions, eg:
– correlations in the model
– statistical distributions
 change in case estimate reserving philosophy
 planned or unplanned changes in mix

The Actuarial Education Company © IFE: 2019 Examinations


Page 4 SP7-18: Reserving – use of ranges and best estimates

 particularly large / unusual risks:


– large claims
– catastrophes
– latent claims
 inadequate data supplied by third party claims handlers
 format of data
 claims inflation not as expected
 uncertain sales-related expenses, commission, new distribution channels etc.

Sources of model specification error


 model error
 programming error
 simulation error / too few simulations

Uncertainty in actuarial estimates can broadly result from three areas: model, parameter
and process.

Model uncertainty, parameter uncertainty and process uncertainty were introduced in earlier
chapters. We now discuss each of these areas in more detail.

1.1 Model uncertainty


Model uncertainty is the risk that an inappropriate model has been used in the estimation
process.

This arises because actuarial models are often a simplification of a very complex (and
unknown) underlying system. By using a simplified model to project the true underlying
system, we are introducing an unknown bias into the model.

This introduces uncertainty in the estimates produced by the model.

A common example of this in actuarial modelling is the use of parametric distributions for
outstanding claims reserves (like the log-normal distribution). The complexity of the claims
process and the factors influencing it make it unlikely that the real distributions match
simple statistical models.

We can reduce model uncertainty by using actuarial judgement when we select a model.
This means that we select models which best capture the key features of the process. This
is especially important when the volumes of past data are insufficient to test whether a
model is inappropriate.

For example, by splitting data into perceived homogenous groups, we can reduce model
error, but this could also affect the estimates of parameter uncertainty (including correlation
parameters) and process uncertainty.

This is because the homogeneous groups may not contain sufficient credible data, therefore
increasing uncertainty.

© IFE: 2019 Examinations The Actuarial Education Company


SP7-18: Reserving – use of ranges and best estimates Page 5

1.2 Parameter uncertainty


Parameter uncertainty refers to the uncertainty in determining the parameters for an
actuarial model. This usually results from the statistical variability present in the historical
data used to estimate the parameters. Past data will never comprise all possible outcomes.

An absence of large losses in historical data can lead to an error in the estimation of the
‘average’ claim development pattern.

For example, mortgage indemnity guarantee business can have long periods of stable (and low)
claims experience during periods when the economy is performing well. However there is always
the risk of an economic downturn and a significant increase in claims.

If we assign our parameter values by analysing past claims experience only, this will lead to
inappropriate reserve estimates.

We can sometimes reduce parameter uncertainty by using judgement when we select


parameters. Quantifying the impact of using judgement on parameter uncertainty is itself
(usually) a matter of judgement. There will always be some parameter uncertainty.

In combination, parameter and model uncertainty lead to the statistical risk that the outcome of
the exercise will not form a good reflection of the underlying claim distribution. This is a result of
insufficient / inaccurate data and an inappropriate fit of the model.

1.3 Process uncertainty


If a process is assumed to be inherently stochastic, the future outcome will be uncertain
because of the randomness of the process and the fact of course that many of these events
have yet to occur.

This uncertainty is present even if model selection is perfect and the parameters are known
with certainty.

An example of this is the uncertainty present in an unearned premium reserve for business
exposed to Gulf of Mexico hurricanes. In this case, the eventual liability could be very
different from a correct average liability if a hurricane materialises during the unexpired risk
period.

This is because there is uncertainty in the timing of the hurricane, related to seasonality, so
estimating UPR at a point in time is tricky.

These sources will contribute to the overall uncertainty of a point estimate. The most
significant source of uncertainty will depend on the situation.

For example, a key source of process uncertainty for product liability business will be the
emergence or otherwise of a new type of claim, whereas for a commercial fire portfolio it might
be the occurrence of a major catastrophe.

The process uncertainty in a large portfolio of (reasonably) independent personal motor


risks can be quite small compared to model and parameter risk. The opposite might be true
for a small book of excess liability risks.

The Actuarial Education Company © IFE: 2019 Examinations


Page 6 SP7-18: Reserving – use of ranges and best estimates

Question

Explain the last sentence of Core Reading.

Solution

For a small book of business we would expect there to be greater uncertainty surrounding the
future outcome.

Also liability business may be more likely to include large heterogeneous risks and have the
chance of latent claims emerging. Again this leads to greater uncertainty regarding the future
claims outgo, ie increased process uncertainty.

Excess business is likely to be more volatile since it captures only the upper portions of the
insured losses and of course the tail-end of any distribution is subject to a high degree of
uncertainty.

Further volatility in claim amounts is introduced if different levels of excess apply to each inwards
risk.

We sometimes refer to the combination of process and parameter uncertainty as estimation


or prediction error.

© IFE: 2019 Examinations The Actuarial Education Company


SP7-18: Reserving – use of ranges and best estimates Page 7

2 Best estimate reserves


We normally define the best estimate as the actuary’s view of the mean or expected value
(also called the unbiased probability – the weighted average) of the eventual outcome.

It is very important to recognise that in many, if not most, situations it will not be possible
for the actuary to derive the mean of the outcomes with a high degree of certainty.

Question

Explain why this is the case.

Solution

Because there is great uncertainty over the future number, size and timing of claim amounts.

Instead, the actuary will estimate a reserve value using an approach that is intended to
derive a mean or expected outcome. The actuary will not be certain that the value derived
does equate to a mean value, but will ascribe the term ‘best estimate’ to the determined
value to convey the ‘type’ of estimate that the actuary is deriving.

The term ‘best estimate’ is used in this case to distinguish it from a prudent or optimistic
estimate.

In such circumstances, we can consider the best estimate as the actuary’s subjective
derivation of the probability-weighted mean of all possible outcomes, taking into account all
available information about the business being analysed.

It should also be noted that the actuary will be calculating a sample mean as an
approximation to the population mean. This means that a best estimate should allow for
information that is available to the actuary, but may not be reflected in the underlying data
yet.

For example, an actuary may be aware that a portfolio is exposed to catastrophic events, even
though no catastrophes have occurred within the period of the past data. Therefore the actuary
will need to make an allowance for this in his/her calculation of the best estimate.

The eventual outcome of outstanding liabilities is uncertain for the reasons described
above. Therefore, we must be clear what we mean when we present point estimates of
outstanding liabilities.

A term commonly used is the actuarial ‘best estimate reserve’. This is a point estimate
reserve. This is usually taken to mean the expected value of the outstanding liabilities, after
allowing for all the areas of uncertainty outlined above.

A point estimate is a single number, rather than a range of estimates.

(You may remember from your earlier studies that a best estimate reserve is often defined as the
basis that has an equal chance of overstating or understating the cost. This is not usually the case
for general insurance however, where the expected value is more commonly used.)

The Actuarial Education Company © IFE: 2019 Examinations


Page 8 SP7-18: Reserving – use of ranges and best estimates

But in practice, when we determine the value of liabilities, we may not distinguish between
the mean, mode and median, though of course we should, as the values may be very
different.

It is good practice both for Subject SP7 and in your own work to clarify what you mean when you
use the term ‘best estimate’.

The term ‘best estimate’ reserve is also used in other areas and is not necessarily defined in
a statistical framework. For example, under the Solvency II regime, the reserving actuary is
required to identify ‘best estimate’ reserves, which are the mean of all possible outcomes,
not just those present in the data.

The key characteristics of a ‘best estimate’ in this context are that:

 It is a point estimate. The best estimate is described as a single number, not as a


range of reasonable outcomes.

 It is not inherently optimistic or pessimistic. The best estimate does not include any
deliberate bias in the setting of the underlying assumptions. It is meant to be the
actuary’s impartial view of the reserves with no margins, implicit or explicit, for
prudence or optimism.

 It is based on sound and appropriate actuarial or statistical techniques.

 It is based on current and credible information.

 The requirements say nothing about the skewness of the underlying distribution or
its inherent volatility.

Solvency II is the regulatory regime applicable to UK general insurance companies in the EU. It
requires firms to value their assets and liabilities on a market-consistent basis and more
risk-sensitive capital requirements address asset as well as liability risks. It consists of three
‘pillars’.

Pillar 1 sets out the reserving basis and the capital requirements companies are required to meet
for insurance, credit, market and operational risk. Capital requirements may be calculated using a
standard formula or, if firms have supervisory approval, they may use their own capital models.

Question

Give examples of insurance risk, credit risk, market risk and operational risk for a household
property insurer.

Solution

Insurance risk – poor weather, leading to many claims from flooding and burst pipes.

Credit risk – this is the risk of third party default, eg failure of a reinsurer / broker, failure of
assets.

Market risk – volatility of assets and liability values.

Operational risk – this is the failure of people, processes and systems, eg poor claims handling
procedures.

© IFE: 2019 Examinations The Actuarial Education Company


SP7-18: Reserving – use of ranges and best estimates Page 9

Pillar 2 consists of a supervisory review process to evaluate the adequacy of capital and the
company’s risk management systems and processes. Supervisors may decide that a company
should hold additional capital against any risks not adequately covered in Pillar 1.

The aim of Pillar 3 (disclosures) is to harness market discipline by requiring firms to publish certain
details of their risks, capital and risk management.

In previous chapters, we describe various techniques or methods that help us to estimate


the reserves for outstanding claims. In assessing the ‘best estimate’ reserve, we must
understand that these methods only give guidance and do not provide a definitive value.

For example, the impact of a current company initiative to defend vigorously every claim
will not be reflected in many of the standard models based on past data. However, using
our judgement, we may reduce outstanding claim liabilities for certain claim types, increase
reserves for claim handling expenses, and lengthen the claim development pattern.

Question

For the example above, explain why such an initiative will lead to:
(i) reduced outstanding claim liabilities for certain claim types
(ii) increased reserves for claims handling expenses
(iii) a longer claim development pattern.

Solution

(i) Reduced outstanding claim liabilities – expected due to more stringent claims
underwriting.

(ii) Increased claims handling expenses – since it will cost more to defend each potential
claim payment.

(iii) Longer claim development pattern – there will be no change to claim reporting patterns,
but the period until settlement may lengthen as each case is defended more vigorously
before a settlement is reached.

It should be noted that if certain outcomes have been excluded when making the estimate
(for example the failure of reinsurance, the emergence of extreme outcomes or latent
claims), the estimate will be different than if they had been included. The actuary should
make clear what has and what has not been included in deriving the best estimate.

For a point estimate, alternatives to a best estimate are:

 the median (the value that has a fifty-fifty chance of being adequate)

 the mode (the most probable value)

 an estimate with a particular likelihood of exceeding the outcome.

The Actuarial Education Company © IFE: 2019 Examinations


Page 10 SP7-18: Reserving – use of ranges and best estimates

2.1 Risk margin and range


Depending on the purpose, most reserving exercises involve deriving a ‘best estimate’ of
the reserves or an alternative estimate that may contain a margin.

In other words, the strength of the basis will depend on the purpose of the exercise.

We can see that the best estimate is not a single defined amount that can be derived from a
given dataset.

For example, if we gave 100 actuaries the same data and asked them to derive their best
estimate of the reserves, the results would not all be the same – in fact there would be a
‘range of best estimates’. We can define such a range as one which reflects the parameter
uncertainty and model error alone; in other words it expresses the uncertainty arising from
the selection of parameters and/or a given model, given the data available.

If we asked one of these 100 actuaries to provide an indication of the uncertainty that exists
around his or her selected best estimate (derived using a chosen model or particular
process), this would reflect some or all of the process uncertainty alone.

As well as a ‘range of best estimates’, other types of ranges that might be used by actuaries
include:

 ‘Range of possible outcomes’: this would represent the actuary’s estimate of the
complete range of outcomes for future claims. It would therefore normally be
considerably wider than the range of best estimates.
This is because it includes extreme events which might not even be considered as
plausible, for example a ‘one-in-1000-year-event’.
 ‘Range of reasonable / probable / plausible outcomes’: this would typically be wider
than a range of best estimates, but narrower than the range of possible outcomes.
It would be wider than a range of best estimates because it would allow for
outcomes that cannot be reasonably regarded as an estimate of the mean or average
outcome, but which can still be regarded as plausible outcomes.
We can think of this range as allowing for parameter / model uncertainty and some
element of process uncertainty as well. It would effectively include the outcomes
that the actuary regards as plausible or probable, but exclude those outcomes that
the actuary regards as extreme (being either very low or very high).

For a view on uncertainty wording and associated probabilities, the student might refer to
section 4.5.5 of the paper ‘Reserving and Uncertainty’ by E. R. Gibson, et al:

http://www.actuaries.org.uk/research-and-resources/documents/reserving-and-uncertainty

A range of best estimates or a range of reasonable outcomes may be useful when


management are considering what reserve estimates should be booked in their accounts. A
range of possible outcomes may be useful when considering the resilience of the company
to adverse events and/or when purchasing reinsurance.

Lloyd’s syndicates routinely carry out analyses of ‘Realistic Disaster Scenarios’ for this reason.

© IFE: 2019 Examinations The Actuarial Education Company


SP7-18: Reserving – use of ranges and best estimates Page 11

3 Estimating the range of possible outcomes


There are methods that allow us to quantify some of the uncertainty in the outstanding
reserves. However, we should use judgement when we interpret the results of such
methods, just as we do for methods which only produce point estimates.

There are no universally-agreed standards for quantifying uncertainty in the reserves. It will be
important for the actuary to select the most appropriate approach given the circumstances. This
will require the actuary to weigh up the costs versus benefits of the different approaches.

There are three commonly used approaches for quantifying uncertainty:


 stochastic models
 alternative sets of assumptions
 scenario testing.

These are discussed in detail in the rest of this section.

Exam Tip
The Core Reading for Subject SP7 draws occasionally on professional guidance, such as the
Technical Actuarial Standards. You may therefore find it useful preparation for the exam if you
familiarise yourself with the content of the professional guidance issued by the Institute and
Faculty of Actuaries on general insurance.

3.1 Stochastic models


One way to quantify the uncertainty in a reserve estimate is to construct a stochastic model
of the claim development process. One model of particular interest is the so-called over
dispersed Poisson (ODP) model.

This is a generalised linear model (GLM) applied to claims triangles where the form is
chosen so that the mean (or best estimate) reserve is equal to that resulting from a
deterministic basic chain ladder method.

We can obtain a distribution of possible outcomes from this model that reflects both
parameter risk and process risk by using a bootstrapping technique. We have discussed
this in Chapter 16 (in the section ‘Bootstrapping the ODP’).

Question

List the key assumptions underlying this method.

The Actuarial Education Company © IFE: 2019 Examinations


Page 12 SP7-18: Reserving – use of ranges and best estimates

Solution

The key assumptions used when bootstrapping the ODP are:


 the run-off pattern is the same for each origin period (as for the chain ladder)
 incremental claim amounts are stochastically independent
 the variance of the individual claim amounts is proportional to the mean
 incremental claims are positive for all development periods.

Another model is the Mack model (previously discussed in Chapter 16). This calculates the
mean and variance (including parameter uncertainty) of the distribution of possible
outcomes. We choose a parametric distribution with this mean and variance, if we require a
full distribution of possible outcomes. We typically choose a positively skew distribution
(for example, a log-normal or gamma distribution).

Using these models, we attempt to quantify process and parameter error, but we may not
obtain a reasonable answer. There are various reasons why any method might provide
unreasonable results:

 The past data upon which these methods are based might be affected by one-off
events, such as changes in claims handling procedures. As we do with a
deterministic method, we need to adjust the parameters or results to allow for these
features.

 Another problem is that, even after we have allowed appropriately for changes
within the account and claims environment, the volatility seen within the historical
claims development may not be a good indicator of the underlying process
uncertainty.
This is particularly a problem for insurers or classes of business for which there are
only a few years’ historical (and consistently derived) data. Often the assumptions
inherent in these methods may not hold true in practice.

 We have limited data, which will not reflect even a small proportion of possible
outcomes.

 The choice of model (in this case the chain ladder method) may not be a good
representation of the underlying process (eg it is not normally appropriate to use the
chain ladder method to set reserves for war insurance).

 Although it is possible to allow for the more extreme correlations that may occur in
the tail of distributions using a stochastic approach, eg by using Gumbel
correlations; these are in practice very difficult to parameterise except by the use of
judgement.
The Gumbel copula is discussed elsewhere in the course.

We should also remember that both the ODP and Mack methods are based on the chain
ladder reserving technique. The variances which are derived are related to estimates from
this method. Quite often an estimate has been derived from a different method, such as the
Bornhuetter-Ferguson method. We should consider the extent that the estimation method
may change the volatility estimate.

© IFE: 2019 Examinations The Actuarial Education Company


SP7-18: Reserving – use of ranges and best estimates Page 13

A stochastic model will not always be appropriate. An approach taking into account the actuary’s
knowledge and experience and more based on judgement can be more suitable.

3.2 Alternative sets of assumptions


Another way to quantify the uncertainty in a reserve estimate is to estimate the reserve
using parameters different to those of the best estimate. The resulting spread of reserve
estimates provides a range around the best estimate reserve. We usually determine the
sets of assumptions using judgement.

You may intuitively want to call this a scenario analysis. In this context however, a scenario
analysis refers to the method of testing extreme scenarios, rather than the more likely, less
extreme outcomes discussed here.

We note that we should view each set of assumptions as a package so that each individual
assumption may be correlated with others. In particular, inflation and discount
assumptions are typically correlated.

The main advantages of this method are:

 It is very simple to perform on deterministic or stochastic models.

 We use judgement when we select possible parameters. We can therefore allow for
atypical volatility in the historical data.
For example, if it is not expected to be repeated then it can be excluded from
consideration.
This is a possible advantage over the stochastic models described above where
parameter uncertainty is related to the volatility of the historical data.

The main disadvantages of this method are:

 We assign no explicit probability to each set of parameters. Therefore, it is not


possible to estimate the distribution of future outcomes unless we assign a
probability to each set of assumptions.

 We ignore model uncertainty using this method.

 We do not allow for process uncertainty if we use alternative sets of assumptions for
a deterministic model. However, there is no reason why we cannot use alternative
sets of assumptions for a stochastic model.

Question

Suggest what type of uncertainty we are allowing for if we use alternative sets of assumptions for
a deterministic model.

Solution

Parameter uncertainty.

The Actuarial Education Company © IFE: 2019 Examinations


Page 14 SP7-18: Reserving – use of ranges and best estimates

3.3 Scenario testing


A third way to quantify the uncertainty in a reserve estimate is scenario testing. In a
scenario analysis, we often examine the likely impact of catastrophic events on an insurer’s
outstanding liabilities. In extreme conditions, many areas of uncertainty may become more
correlated than in normal conditions. The interdependency of these uncertainties is a key
aspect of any extreme scenario.

For example, in a scenario test, we might estimate the required reserves if a latent
asbestos-type claim emerges, which affects several industries and insurance classes and
causes some of the company’s reinsurers to default.

We normally use scenario analyses to investigate the top limit of the range of possible
outcomes for the outstanding liability.

We can derive a scenario in a variety of ways:

 basing it on an historical event

 thinking up a hypothetical event using our judgement or

 from the results of a stochastic model.

Scenarios are typically based on unlikely, but not impossible, events. These events can be
financial, operational, legal or related to any other risk that might have an impact on the
insurer’s outstanding liabilities. We can develop scenarios with varying complexity.

Question

A company writes employers’ liability business. Give an example of:


(i) a financial risk
(ii) an operational risk
(iii) a legal risk
that might have an impact on the company’s outstanding liabilities.

Solution

(i) Financial risk – failure of a capital provider, failure of assets.

(ii) Operational risk – mismanagement of the portfolio, poor data recording and lax claims
handling.

(iii) Legal risk – a court precedent increases the level of claims.

The aim is to make sure all sources of risk are identified and key risks are analysed in detail and
quantified. It is useful to divide risks into appropriate categories for this purpose.

We may also wish to test the effects of plausible favourable scenarios to prepare responses
to capitalise on such events if they should occur.

© IFE: 2019 Examinations The Actuarial Education Company


SP7-18: Reserving – use of ranges and best estimates Page 15

Some typical scenarios affecting outstanding liabilities are:

 Single catastrophes.

 Major individual contracts written. We can check these in isolation, or check


combinations of them. The test should analyse all risks included within these
contracts.

 Multiple ‘large’ losses. We can consider the possibility of random events or


possible common causes (for example, economic downturn / problems with
financial institutions) causing a series of losses. These might hit a significant part
of syndicate retention (for a Lloyd’s syndicate) or potentially exhaust lower layers of
reinsurance programmes.

 Poor attritional claims experience.

 Latent claims.

 Reinsurance bad debt.

 Interest rate changes (if discounting reserves).

 Inflation levels.

 Expense levels.

 Exchange rate movements.

The main advantages of scenario testing are:

 It provides an advantage over a stochastic model by allowing a more detailed analysis of


the tail end of the reserve distribution.
A stochastic approach will produce a distribution of outcomes that will enable us to
derive the percentiles of the distribution. The tail of this distribution of outcomes
will generally occur when a number of adverse factors coincide. It is very difficult to
model such a coincidence of factors reliably using a parametric approach.
– We should treat the results from stochastic modelling with a great deal of
care when we consider the tails of the distribution.
– In performing a scenario test, we pay particular attention to the likely
coincidence of these adverse factors.

 A scenario analysis is more focused. A stochastic approach provides a full analysis


even when we do not require the full output.
Therefore it is more time consuming and expensive to run.
We might only be interested in the extreme outcomes. A stochastic analysis uses
full distributions of the constituent risk factors, even when only the extreme
outcomes will have any bearing on the result. We can aim a scenario test at the
specific question being asked.

 Because it is aimed at the specific question, we can construct a scenario test and
produce reliable results much more quickly than for a stochastic model.

The Actuarial Education Company © IFE: 2019 Examinations


Page 16 SP7-18: Reserving – use of ranges and best estimates

 It is easier to communicate the results of scenario tests than the results of


stochastic models, as they are more transparent. The management of the business
or users will typically understand the scenarios used, and can form an opinion of the
tests without requiring detailed explanation. This is not normally the case for a
stochastic model where the users need a greater understanding of the mechanics of
the model and the detailed assumptions before they can have full confidence in the
results.

 Model uncertainty is much less of a problem when we construct scenario tests


because we consider the driving factors explicitly. Stochastic models may fail to
capture some features of the real life process, especially under extreme
circumstances.

The main disadvantages are:

 There is no specific probability associated with the outcomes and so it is not


possible to construct a distribution of outcomes. But we could sometimes argue
this is more appropriate than the possibly spurious accuracy in the tail of
distributions that we calculate using a stochastic approach.

 Scenarios typically only give information on the extremes of the distribution of


eventual outcomes.
The actuary may want to disclose information on the overall distribution to stakeholders
too.
 The method is more subjective than the other two methods.
This is because the actuary has to decide which extreme scenarios are to be investigated.

Other relative merits of stochastic and deterministic approaches include the following:

 Deterministic approaches only consider a limited number of factors and one result
from each.

 Failure is often due to the interaction of many differing factors which could not be
modelled deterministically.
A stochastic model can allow for the interdependency of these key factors.
 Analysis of the impact of atypical scenarios aids understanding of variation around
expected outcomes, and assigns a distinct value to them.
In practice, we are likely to use all or a combination of the above techniques.

This will be in part dependent upon the stability of the class of business and the credibility of the
past data available.

We should select the approaches which are appropriate to the scale, complexity and
importance of the analysis, taking into account the cost / benefit balance to the user.

© IFE: 2019 Examinations The Actuarial Education Company


SP7-18: Reserving – use of ranges and best estimates Page 17

4 Communicating best estimates


The ideas discussed in this part of the Core Reading should be familiar to you from your study of
earlier parts of the course.

It is pointless to calculate reserve estimates and ranges unless we can set out and explain,
clearly and effectively, those estimates to the various interested parties, such as senior
managers, the board of directors and the regulator. We should bear in mind the use to
which the recipient of the advice is likely to put it.

The information provided should enable the intended audience to understand the nature and the
size of the uncertainty that is faced. The appropriate communication of this is as important as the
calculations themselves.

4.1 Communicating best estimates


The commonly used phrases ‘best estimate’ or ‘central estimate’ can mean different things
to different people.

Furthermore, we should not communicate the best estimate in such a way that gives the
impression that it is the only ‘right’ answer or ‘point’ estimate. Rather, when providing a
point estimate, it is important that the actuary is able to communicate effectively the
inherent uncertainty surrounding that estimate to key stakeholders.

There is a range of possible outcomes because of the uncertainties described above. Some
outcomes are more likely than others. We should make clear that the best estimate is the
mean of the distribution of the range of possible outcomes. The best estimate does not
necessarily represent the most likely outcome, especially if (as is usual) the distribution of
the range of possible outcomes is positively skewed.

A positively skewed distribution will have a tail extending out to the right (larger numbers). For
this distribution, the mean is greater than the median reflecting the fact that the mean is sensitive
to each score in the distribution and is subject to large shifts when the sample is small and
contains extreme scores.

The best estimate is just an estimate. There can be a tendency for the best estimate to be
treated as being a more reliable prediction than it is intended to be and than it really is. Any
user of the information should understand that it is just an estimate, that there are other
possible reasonable estimates and that the ultimate result is likely to be different to the
estimate.

We should also highlight the key assumptions made. There will often be a number of
critical assumptions to which the best estimate and/or reserve ranges are most sensitive.
The reliability of the result will depend on the appropriateness of these assumptions. The
user needs to be aware of what these assumptions are and the sensitivity the result has to
these.

TAS 100 states that communications shall state the material assumptions and describe their
rationale.

We should comment on the main restrictions (or shortcomings) in the analysis. These
could include incomplete data, restrictions in the scope of the work or lack of information
provided on company policies.

The Actuarial Education Company © IFE: 2019 Examinations


Page 18 SP7-18: Reserving – use of ranges and best estimates

For example TAS 100 states that communications shall:


 include explanations of any significant limitations of the models used and the implications
of those limitations
 describe any material uncertainty in the data and the approach taken to deal with that
uncertainty.

It may be that the actuary wants to present an overall measure of uncertainty for the company,
eg by combining the results from the individual classes of business. In this case some adjustment
may be made to the figures to reflect the benefits of diversification.

4.2 Communicating uncertainty


For many purposes, it will be appropriate to give an estimate of the uncertainty surrounding
the best estimate by giving a margin or range. A range of reserve estimates can help key
stakeholders understand the uncertainty inherent in the business. In this case it will also be
necessary to define the meaning of the margin or range and communicate it carefully.

For example, when selecting a reserving basis where we make key judgements that have a
material impact on the estimates, we should communicate these key judgements when
giving the estimates.

TAS 100 states that material judgements shall be communicated to users so that they are able to
make informed decisions.

We should state clearly the extent to which the margin or range is intended to reflect the
various sources of uncertainty.

Question

List the four sources of uncertainty in a reserving model.

Solution

Specification error, selection error, estimation error and process error.

The terms used to describe the sources of uncertainty can mean different things to different
people. When describing a reserving basis we should define these terms carefully and
communicate them in a way that is appropriate to the audience.

The purpose for which the reserving exercise is being carried out has a direct effect on the
importance of the uncertainty surrounding the reserving process and estimates. This in
turn may affect the reserving basis selected for the purpose in question.

The Technical Actuarial Standard 100 requires that actuarial communications indicate the
nature and extent of any material uncertainty in the actuarial information they contain.

Note: on the assumption that the phrase ‘best estimate’ means the actuary’s best view of
the mean or expected value of the eventual outcome (possibly excluding certain remote
contingencies), then we can think of a range described as ‘a range of reasonable best
estimates’ as illustrating the parameter uncertainty and model error alone.

© IFE: 2019 Examinations The Actuarial Education Company


SP7-18: Reserving – use of ranges and best estimates Page 19

Since actuarial judgement is involved to a greater or lesser extent in all of the methods of
quantifying uncertainty, different actuaries examining the same tranche of business would
produce somewhat different illustrations of uncertainty.

In recent years, actuaries have increased their focus on communicating uncertainty.


Misunderstandings can sometimes occur because our stakeholders, who may have less
technical training in the details of uncertainty, are not as familiar with some of the concepts
as actuaries. Actuaries need to be careful to communicate uncertainty in a way which is
intuitively comprehensible to non-actuaries. Stakeholders have expressed a strong
preference for being told the range of potential outcomes. This is an intuitively
straightforward concept and is directly relevant when we track the actual out-turn (outcome)
of claim costs.

Question

Without referring back to your notes, write down three types of ranges that might be used by
actuaries.

Solution

Range of best estimates, range of possible outcomes and range of reasonable / probable /
plausible outcomes.

You should ensure that you clarify which of these ranges you mean when communicating your
results.

The quantification of uncertainty requires us to communicate both size and likelihood of the
reserving requirements. The size is normally quoted explicitly, whereas the likelihood is normally
communicated in two ways.

We can communicate uncertainty in two ways:


1. using words
2. using numbers (often expressed in percentiles).

In practice, it is likely to be a combination of these two approaches.

The use of percentiles is a way of communicating uncertainty not a way of estimating the
uncertainty. For example, the actuary may have exercised judgement to examine alternative sets
of assumptions when estimating uncertainty but this can still be communicated using percentiles.

The method we choose depends on the technical knowledge of the audience, but we note
that any valuations of percentiles may imply more certainty of the distribution than is
warranted. It is worth stressing that a percentile is often a percentile within a particular
model and is not immune to residual model error (or indeed parameter error).

An example of adopting the percentile approach when communicating to the audience would be
to say:

‘… this equates to the 90th percentile, meaning that in my judgement there is a 90% chance that
the outcome will lie below this value and a 10% chance that it will lie above.’

The Actuarial Education Company © IFE: 2019 Examinations


Page 20 SP7-18: Reserving – use of ranges and best estimates

5 Summary
In summary, all actuaries must comply at all times with the relevant ethical and technical
professional guidance, as issued and revised from time to time by their profession and
regulators.

In addition, when reserving, the actuary needs to know and understand the relevant
regulations and accounting standards, particularly if the reserving work is in respect of
statutory reporting.

Reserving regulations and accounting standards vary from country to country and even
between territories within countries. Even in countries where the legal and regulatory
frameworks may be broadly similar, there can be subtle differences in the regulations and
accounting standards. Reserving actuaries must understand these and, to the extent that
they do not, they should either seek suitable advice or not act at all, in accordance with the
prevailing professional ethical standards.

This should by now be of no surprise to you and the Institute and Faculty of Actuaries is pretty
unbending in this respect (note the use of the word ‘must’ in the above paragraph). If an actuary
does not have the knowledge / expertise required (or access to others who do have it) in order to
carry out a piece of work, then they should refuse to act.

© IFE: 2019 Examinations The Actuarial Education Company


SP7-18: Reserving – use of ranges and best estimates Page 21

6 Glossary items
Having studied this chapter you should now ensure that you are able to explain the following
Glossary items:
 Generalised linear model (GLM)
 Model uncertainty
 Parameter uncertainty
 Process uncertainty.

The Actuarial Education Company © IFE: 2019 Examinations


Page 22 SP7-18: Reserving – use of ranges and best estimates

The chapter summary starts on the next page so that you can
keep all the chapter summaries together for revision purposes.

© IFE: 2019 Examinations The Actuarial Education Company


SP7-18: Reserving – use of ranges and best estimates Page 23

Chapter 18 Summary
The actual ultimate claims amount will almost certainly differ from our estimate. We should:
 clearly define what we mean by our single point (or best) estimate
 attempt to quantify the likely divergence from this estimate.

Uncertainty is a result of:


 Model uncertainty – simplifications in the model used to project an underlying
complex system introduce bias.
 Parameter uncertainty – due to difficulties in setting appropriate parameters for the
future from the past data.
 Process uncertainty – even if the model and parameters were perfectly selected this
would still exist due to the inherent randomness of the claims process.

A best estimate is normally defined as the actuary’s view of the mean or expected value of
the eventual outcome. However it will include areas of uncertainty (model, parameter,
process).

The actuary should make clear what scenarios have been included / excluded in deriving the
best estimate.

The risk margin or range gives an estimate of the uncertainty surrounding the best estimate.

Three methods may be used to estimate the range of possible outcomes:


 stochastic models
 alternative sets of assumptions
 scenario testing.

Other types of ranges that might be used by actuaries include:


 range of possible outcomes
 range of reasonable / probably / plausible outcomes.

In Solvency II the best estimate is characterised as:


 a point estimate
 not inherently optimistic or pessimistic
 based on sound and appropriate actuarial or statistical techniques
 based on current and credible information
 nothing is said in the requirements about the skewness of the underlying distribution
or its volatility.

The Actuarial Education Company © IFE: 2019 Examinations


Page 24 SP7-18: Reserving – use of ranges and best estimates

Chapter 18 Summary continued


Stochastic models
Stochastic models may involve the use of the ODP model or Mack model.

Results may not be reasonable if:


 past data is affected by one-off events
 volatility in past claims development is not a good indicator of underlying process
uncertainty.

Alternative sets of assumptions


This estimates reserves using sets of parameters different to that of the best estimate.

It should allow for correlations between assumptions.

Advantages are:
 easy to perform
 allows judgement to be exercised.

Disadvantages are:
 does not estimate the distribution of future outcomes
 ignores model uncertainty
 does not allow for process uncertainty if using a deterministic model.

Scenario testing
This examines the likely impact of catastrophic events, used to examine the top limit of the
range of possible outcomes. In extreme conditions areas of uncertainty may become more
correlated than normal.

Scenarios can be derived:


 on a historical event
 from judgement, thinking up a hypothetical event
 from the results of a stochastic model.

© IFE: 2019 Examinations The Actuarial Education Company


SP7-18: Reserving – use of ranges and best estimates Page 25

Chapter 18 Summary continued


Typical scenarios to be tested include:
 single catastrophes
 major individual contracts written
 multiple large losses
 poor attritional claims experience
 latent claims
 reinsurance bad debt
 interest rate changes (where discounting reserves)
 inflation levels
 expense levels
 exchange rate movements.

Advantages of scenario testing:


 pays particular attention to the likely coincidence of adverse factors
 it is more focused on extreme outcomes than a stochastic model
 quicker to produce results than a stochastic model
 easier to communicate results than a stochastic model
 model uncertainty is less of a problem.

Disadvantages of scenario testing:


 does not give a distribution of outcomes
 typically only gives information on the extremes of a distribution
 more subjective than the other two methods.

Communication of the best estimate


Results should be explained clearly and effectively to key stakeholders.

Highlight that best estimate is just an estimate.

Highlight key assumptions and comment on the main restrictions or shortcomings in


analysis.

The Actuarial Education Company © IFE: 2019 Examinations


Page 26 SP7-18: Reserving – use of ranges and best estimates

Chapter 18 Summary continued


Communicating uncertainty
Uncertainty can be communicated by using words or numbers.

When communicating the uncertainty of the results, an actuary should:


 ensure stakeholders understand the level of uncertainty
 take account of the audience’s knowledge
 be consistent with vocabulary used by other professionals, and explain terms
 emphasise the bigger issues
 explain what has been allowed for in the best estimate, and what has not
 emphasise the unusual issues
 comment on the uncertainty in the context of the scope and purpose
 avoid misunderstandings.

Any reserving calculations must take account of the local reserving regulations and
accounting standards.

© IFE: 2019 Examinations The Actuarial Education Company


SP7-18: Reserving – use of ranges and best estimates Page 27

Chapter 18 Practice Questions


18.1 (i) Define the term ‘best estimate’ reserve.

(ii) In respect of Solvency II, set out the key characteristics of a best estimate reserve.

18.2 (i) Outline the advantages and disadvantages of using alternative sets of assumptions to
quantify the uncertainty in a reserve estimate.

(ii) Suggest two alternative approaches used to quantify the uncertainty in reserves.

18.3 A general insurer sells private motor insurance business. Two actuaries have provided an
assessment of the reserves required: $200m and $275m respectively. Suggest possible reasons
why the two reserves differ.

18.4 Discuss the uncertainties underlying an insurer’s estimated liabilities.

18.5 Explain the issues to consider when communicating uncertainty in reserve estimates. [7]
Exam style

The Actuarial Education Company © IFE: 2019 Examinations


Page 28 SP7-18: Reserving – use of ranges and best estimates

The solutions start on the next page so that you can


separate the questions and solutions.

© IFE: 2019 Examinations The Actuarial Education Company


SP7-18: Reserving – use of ranges and best estimates Page 29

Chapter 18 Solutions
18.1 (i) Best estimate reserve

This is normally taken to be the expected value of the outstanding liabilities, after allowing for
model, parameter and process uncertainty.

(ii) Characteristics of a best estimate reserve

 It is a point estimate, in other words a single number, not a range of outcomes.


 It is unbiased, ie the underlying assumptions used do not take a deliberately prudent or
optimistic view.
 It is meant to be the actuary’s impartial view of the reserves with no margins, implicit or
explicit, for prudence or optimism.
 It is based on sound and appropriate actuarial or statistical techniques.
 It is based on current and credible information.
 The Solvency II requirements say nothing about the skewness of the underlying
distribution or its inherent volatility.

18.2 (i) Advantages and disadvantages of considering alternative sets of assumptions

Advantages
 It is very simple to carry out for both deterministic and stochastic models.
 Judgement can be exercised when considering which alternative sets of assumptions to
consider.
 We can exclude scenarios which we do not expect to be repeated. This is in contrast to a
stochastic approach where unlikely scenarios will be implicitly included in the results
(albeit in the tail of our distribution).

Disadvantages
 We cannot estimate the distribution of future outcomes unless we assign a probability to
each set of assumptions.
 We ignore model uncertainty.
 If a deterministic model is used with alternative sets of assumptions then it does not allow
for process uncertainty …
… however, there is no reason why we cannot use alternative sets of assumptions for a
stochastic model.

(ii) Two alternative approaches

 stochastic modelling
 scenario testing

The Actuarial Education Company © IFE: 2019 Examinations


Page 30 SP7-18: Reserving – use of ranges and best estimates

18.3 The reserves may differ because:


 there is a different purpose to the reserving exercise, eg one for solvency purposes and
one for internal management accounts, this will affect the basis used
 the data used was different, eg one had access to more detailed data or the data used
was as at a different date
 one actuary had access to additional data, eg on large losses
 different reserving methods were used
 one actuary had additional information from interviewing underwriting and claims
handling staff
 one used net figures and the other gross
 there was a genuine difference of opinion
 there was an error.

18.4 With a few exceptions, such as personal accident insurance, the size of payment to be made will
not be known in advance particularly if it may be subject to inflation.

When considering the patterns arising in the dates of claims payments there may be several
factors acting, for example:
 the policyholder may not even know the contingency giving rise to the claim has taken
place, for instance a slowly developing disease
 the claim may have been notified to the broker but they have not yet passed it on to you
the insurer.

Even after full details of the claim have been reported to an insurer, different members of the
claims staff could reach different but equally valid conclusions about the facts and so produce
different estimates of the liability.

There may be protracted discussions about whether or not the claim falls within the terms of the
policy.

During the period to settlement there may be long discussions with the reinsurer who may take a
different view about the claim from the insurer. This can result in delays and lower than expected
recoveries.

Even if the insurer could estimate the liabilities arising from the current year’s business, it will be
very difficult for it to estimate the liabilities arising from future years. These will be made up
from:
 claims from business which does renew with the insurer, and it will be difficult to predict
how much of the business will renew at the end of each year
 claims from completely new business, which will be even harder to estimate.

Business mix and volume may not be the same as in the past.

The chosen statistical model or its parameters may not be suitable.

© IFE: 2019 Examinations The Actuarial Education Company


SP7-18: Reserving – use of ranges and best estimates Page 31

The assumptions underlying the statistical method / model may no longer be appropriate (for
example the chain ladder assumptions).

18.5 The key points to consider when communicating the uncertainty in reserve estimates are:
 A numerical estimate of uncertainty should be included in any formal report that gives a
point estimate of reserves. [1]
 Relevant professional guidance should be adhered to … [½]
 … eg TAS 100 states that ‘communications shall indicate the nature and extent of any
material uncertainty in the actuarial information they contain’. [1]
 Consider whether it is practical to quantify the uncertainty … [½]
 … often expressed in percentiles … [½]
 … or whether it is sufficient to include a descriptive summary. [½]
 Note that a percentile approach is a percentile within a particular model and is therefore
prone to residual model error. [1]
 Consider the need to demonstrate the uncertainty in outcome rather than a range for the
best estimate. [½]
 Consider the need to communicate uncertainty in a way that the intended audience will
understand ... [½]
 … consider their level of technical knowledge. [½]
 It is likely that stakeholders will have a preference for being told the range of possible
outcomes. [1]
[Maximum 7]

The Actuarial Education Company © IFE: 2019 Examinations


All study material produced by ActEd is copyright and is sold
for the exclusive use of the purchaser. The copyright is
owned by Institute and Faculty Education Limited, a
subsidiary of the Institute and Faculty of Actuaries.

Unless prior authority is granted by ActEd, you may not hire


out, lend, give out, sell, store or transmit electronically or
photocopy any part of the study material.

You must take care of your study material to ensure that it


is not used or copied by anybody else.

Legal action will be taken if these terms are infringed. In


addition, we may seek to take disciplinary action through
the profession or through your employer.

These conditions remain in force after you have finished


using the course.

The Actuarial Education Company © IFE: 2019 Examinations


SP7-19: Investment principles and asset liability matching Page 1

Investment principles and


asset liability matching
Syllabus objectives
5.9 Describe, within the context of investment and ALM:
 the principles of investment
 the asset-liability matching requirements of a general insurer
 how projection models might be used to develop an appropriate investment
strategy.

The Actuarial Education Company © IFE: 2019 Examinations


Page 2 SP7-19: Investment principles and asset liability matching

0 Introduction
This chapter examines the important issues relating to a general insurer setting an appropriate
investment strategy.

Section 1 looks at the effect the liabilities have on the choice of assets.

Sections 2 and 3 discuss the impact of the insurer’s free assets and non-investible funds when
setting the investment strategy.

Sections 4  6 consider the importance of projecting future cashflows when determining an


appropriate investment strategy, and the need to assess the sensitivity of these projections to
changes in our model assumptions.

Sections 7  9 discuss how our cashflow projections and resulting investment strategy also need
to allow for solvency requirements, other regulatory requirements, and the insurer’s overall
company objectives.

The final sections of this chapter discuss asset liability modelling. This is a useful tool to aid
understanding and develop an investment strategy. A case study is provided at the end of the
chapter.

© IFE: 2019 Examinations The Actuarial Education Company


SP7-19: Investment principles and asset liability matching Page 3

1 The effect of the liabilities on asset selection

1.1 Features of general insurance liabilities


The main features of the liabilities of a general insurer are described earlier in the course
material, in Chapter 2 (Insurance products – background) and Chapter 3 (Insurance
products – types).

When considering which investments to hold the general insurer needs to take account of the key
investment principles applicable to all investors.

The prime objective regarding the investment of the assets supporting these liabilities is to
maximise investment return, subject to meeting all contractual obligations, whilst ensuring
the risk against not receiving the return is within the company’s tolerance.

The implication for asset choice is that the characteristics of the assets should match those
of the liabilities (claims and expenses); for example in relation to:

 term of the liabilities

 amount of the liabilities

 nature of the liabilities (ie fixed or real)

 currency of the liabilities.

In this section we refer to liabilities using the example of the main liabilities of a general
insurer: the insurer’s technical reserves.

In practice of course, an insurer will have other liabilities, eg expenses such as salary, IT costs,
rent etc. However, the investment considerations relating to these other liabilities will in general
be less significant, because they are smaller, shorter-term, and much more predictable.

1.2 Outstanding term of the investments


The interval between when an insurer receives a premium and when it pays out any
associated claim is important because part of the premium can be invested for some of that
time.

Question

Explain why in the previous sentence it says only ‘part of the premium can be invested’.

Solution

The rest of the premium is used to meet initial expenses, eg administration costs and commission.

The assets held to cover the technical reserves (the amount held to cover the future claims)
need to be selected with the aim of ensuring that the pattern of the income from the assets
corresponds reasonably closely to that of the outgo on the liabilities (the outgo being the
payment of claims).

In other words we need to match assets and liabilities closely as described earlier.

The Actuarial Education Company © IFE: 2019 Examinations


Page 4 SP7-19: Investment principles and asset liability matching

A wide range of classes of insurance give rise to a short interval between receipt of
premium and payment of most claims. These short-tail classes will make it necessary to
hold cash on deposit or very short-dated assets, such as short-dated government
securities.

For business with a long average period between receipt of premiums and payment of
claims there is the opportunity to make longer-term investments with the potential for
increasing profits. The opportunity arises because the money will remain with the insurer
for many months or years.

Long-tail liability classes would be prime examples of where this would be the case.

The advisability of investing money is twofold:

 First, matching the assets to the liabilities by term will tend to reduce the
vulnerability of the insurer to adverse effects of changes in interest rates.
This is because if the assets chosen are shorter in term there will be the need to reinvest
the assets proceeds in the future, on (currently) unknown terms.
 Second, longer-term investments are in the long run likely to provide higher yields
than short-dated assets and cash on deposit. If the insurer is to be able to charge
competitive premium rates it will need to obtain a reasonably good return, at least
comparable with those of other insurers, on the investment of its assets backing the
technical reserves.
In other words, the higher expected return on long-term assets will allow the insurer to
reduce its premium rates.

Medium-dated fixed-interest stocks, index-linked stock where available and, to a very limited
extent, equities and real estate are the likely choices of assets to support longer-tail
insurance classes.

Question

Explain why equities are likely to be used to only a very limit extent.

Solution

Equities have volatile market values  there is the risk that the insurer needs to realise the
investment sooner than expected, at a time when market values are depressed.

Equities also carry a higher security risk  in the case of company failure, equity ranks behind
bonds.

© IFE: 2019 Examinations The Actuarial Education Company


SP7-19: Investment principles and asset liability matching Page 5

1.3 The influence of inflation on investment policy


The final payment amount for the majority of claims will depend on when they are settled as
the claim amount will be subject to inflation. For example:

 price inflation will affect the costs of property repair and replacement costs for
motor and household business

 earnings inflation, medical inflation and judicial inflation will affect bodily injury
claims for liability business.

The expenses of settling claims will also be subject to inflation.

Expenses are likely to be linked to both salary and price inflation.

Claims (and expenses) are subject to inflation, therefore the insurer will need to consider
investing in assets whose income and capital values also tend to increase with inflation, in
order to achieve a matched position.

Such assets could include index-linked securities if these are available in the relevant
currency, but we should bear in mind that the inflation affecting the claims and other outgo
may not be the same as that on which the index is based.

Question

Explain what the likely relationship is between price inflation and:


 earnings inflation
 judicial inflation.

Solution

Earnings inflation will exceed price inflation in the long run. There is likely to be a fairly stable
relationship between the two types of inflation, eg a 2% gap.

Judicial inflation will also exceed price inflation, and it can be very much in excess of price
inflation, eg if a new court precedent sets a higher level of award.

Both equity shares and property are real investments and may increase at a rate in excess of price
inflation.

The income and capital values generated by investment in equity shares and property may,
in the long term, tend to keep pace with claims inflation, but may be subject to considerable
fluctuations in the short term.

However certain contract types such as personal accident and fidelity guarantee may have
fixed claims amounts. In these cases, inflation will be a minor issue and the appropriate
investment policy will lean more towards fixed-interest vehicles.

Further classes may have benefits where ‘fixed’ amounts are linked to inflation, for example,
household insurance where cover is on a new-for-old basis. This should also be
considered when setting the investment policy.

The Actuarial Education Company © IFE: 2019 Examinations


Page 6 SP7-19: Investment principles and asset liability matching

1.4 How liquid do the assets need to be?


An asset is liquid if it is very close to cash, eg money market instruments or short-term bonds.
Liquid assets will either mature in the very short term or have very stable values over time so
have a predictable sale value.

If an insurer is writing business in a class that has a history of producing widely fluctuating
levels of claims (ie there is great uncertainty regarding the number and/or size of claims), it will
be wise to maintain access to ready funds. Even with reinsurance protection, the insurer
will still be left with the obligation to pay the gross claims, perhaps far in advance of making
recoveries from reinsurers.

The short-term and highly uncertain nature of a typical general insurer’s liabilities means
that the marketability / liquidity of the assets (ie having assets which are easy to trade) is an
important factor in investment policy.

© IFE: 2019 Examinations The Actuarial Education Company


SP7-19: Investment principles and asset liability matching Page 7

2 The effect of the free assets on asset selection


The Glossary defines free reserves as:

The excess of the value of an insurer’s assets over its technical reserves and current
liabilities. Also known as the solvency margin and sometimes, in the case of a proprietary
insurer, referred to as shareholders’ funds or net asset value.

The Core Reading in this section refers to ‘free assets’ instead of ‘free reserves’. In practice the
two terms are used interchangeably.

An insurance company’s free assets will be available for investment with the aim of
maximising the long-term return for the shareholders in a proprietary insurance company or
for the policyholders in a mutual insurance company, except to the extent that free assets
may need to be drawn upon to cover any shortfall in the assets held to cover the technical
reserves.

The investment of the free assets will also reflect the insurer’s attitude to risk and its future plans,
eg to use this money to expand the business or take over another insurer.

The way in which an insurer chooses to invest the free assets will be influenced by the size
of the free assets, and by permissible holdings based on the regulatory regime to which the
insurer is subject. However, size should not be measured only in absolute terms. An
insurer also needs to compare the size of the assets available to:

 the size of the technical provisions

 liquidity from the premiums written each year

 the absolute size of the liabilities the insurer is subject to at any time (ie including
technical provisions and any other liabilities)

 any required statutory minimum solvency margin.

An insurer will want to maintain its free reserves so that it can demonstrate to policyholders,
brokers and any regulatory body that it will be able to meet future liabilities. Also the greater the
free reserves, the greater the investment freedom.

We now look at how the size of free reserves can influence investment strategy by considering a
number of examples. In each example, the insurer has a wide mix of business, with
predominantly short-term liabilities. We assume that each insurer wishes to maintain the value
of its assets to be greater than the value of its liabilities, ie hold free reserves.

The Actuarial Education Company © IFE: 2019 Examinations


Page 8 SP7-19: Investment principles and asset liability matching

Example 1: Company with low free reserves


Value of liabilities: £400m

Value of assets: £410m

The company has very little investment freedom if it wants to ensure the value of its assets
remains higher than the value of the liabilities. The company must avoid assets that have volatile
market values, so only a very small percentage (if any) should be invested in equities, long-dated
bonds, property etc. Almost all the funds should be in short-dated investments with secure
values (eg cash deposits, short bonds).

Note that in practice, companies will be required to hold a minimum solvency margin, in excess of
the value of its liabilities.

Example 2: Company with reasonable free reserves


Value of liabilities: £400m

Value of assets: £500m

This company has some investment freedom, but must still monitor the value of its assets
carefully. It might decide to invest £300m in assets that were secure (in monetary terms) and
which were broadly appropriate for the liabilities (perhaps a mixture of cash, short bonds and
five-year index-linked government securities). A more cautious approach would be to invest at
least £400m in this manner. The remainder of the assets might be invested to maximise return.
Some volatility of market value of the ‘high-risk’ assets should not cause too much concern.

Example 3: Company with very large free reserves


Value of liabilities: £400m

Value of assets: £950m

The free reserves are £550m. The company has enormous investment freedom. It could choose
to mismatch to a very large extent, without jeopardising its solvency. For example, let’s say the
fund manager thinks that that long-dated corporate fixed-interest stocks were undervalued. Even
though these assets are inappropriate for the liabilities, the company could choose to invest
towards 100% of its assets in long-dated corporate fixed-interest stocks. (We are not really
advising this. We are just illustrating the greater freedom of choice that this company has in
selecting assets.)

In summary, a company with large free reserves can pay more attention to maximising expected
return. This might involve more volatile assets, fewer ‘secure assets’, and some mismatching by
term, nature and/or currency.

© IFE: 2019 Examinations The Actuarial Education Company


SP7-19: Investment principles and asset liability matching Page 9

Question

Suppose that, for the company in the first example above, the given value of liabilities is on a very
cautious basis (ie with significant margins in the valuation of outstanding claims reserves). Explain
how this has affected the investment policy, and suggest whether the investment policy would
have been any different if the liabilities had been assessed on a ‘best estimate’ basis.

Solution

If the value of the liabilities had been, say, £300m on a best estimate basis, then we might have
proposed an investment policy of £300m in matching assets, and had £110m invested more
aggressively with the aim of maximising returns. This is very different from the conservative
investment strategy that we recommended when the liabilities were valued with significant
margins (eg with undiscounted reserves, where no allowance is made for investment income).

To maximise the return on its free assets, the insurer will normally invest long term in
equities. It might also invest directly in property if the funds available for investment are
sufficiently large for it to put together a diversified portfolio.

However it should be remembered that property suffers from poor liquidity and marketability
which would mean it is inappropriate to hold a large portion in this asset class, in case funds are
required quickly. An alternative would be to invest indirectly in property, by means of a property
fund.

The Actuarial Education Company © IFE: 2019 Examinations


Page 10 SP7-19: Investment principles and asset liability matching

3 Non-investible funds
Not all the assets of a general insurer will be available for investment; for example, monies
held by:

 Agents / brokers, eg brokers might hold premiums for two months before passing them
on.
 Policyholders, eg premium payments by instalment, or end-of-year adjustment
premiums due to exposure adjustments or experience rating. Policyholders can also be
slow to pay the renewal premium.
 Reinsurers, ie delays in making recoveries.

These funds can be a large proportion of the total assets, although it will vary between
companies depending on several factors, such as:

 the mix of business

 the arrangements with the various other parties, including reinsurers

 debt collection efficiency.

In effect, these non-investible funds are tangible, short-term assets. Therefore, the insurer
may decide that a smaller proportion of its other, investible, assets needs to be invested
short. However, these assets are not easily realisable.

In other words, they are subject to the risk of default.

The risk of non-recovery may cause the insurer to look for greater security from the
investible assets and to set aside provisions for bad debts when considering these
debtor-type assets.

© IFE: 2019 Examinations The Actuarial Education Company


SP7-19: Investment principles and asset liability matching Page 11

4 Estimating the liability outgo in future time periods


When choosing an appropriate investment strategy the general insurer will need to have a good
understanding of how liabilities will arise in the future. This can be determined by modelling
future liability outgo.

The first decision is the scope of the projections. Two fundamental choices are:
1. whether to include premium income and outgo relating to business that will be
written after the accounting date, which will depend on whether the exercise is to
assess ongoing profitability, solvency and investment policy, or to determine assets
suitable for matching the existing liabilities (ie for business already written)
2. whether to use model data, which will depend on whether the actual data is too large
or difficult to handle.
This model data will take representative policies and determine the outgo based on them,
the results can then be grossed up to reflect the business as a whole.

Whichever approach is adopted, the aim is to project, on a best estimate basis, the overall
liability outgo in each future time period.

A best estimate basis will be used as we need a realistic view of the likely outgo when determining
the investment strategy. Testing can then be carried out on different bases to check that the
strategy is robust.

The overall liability outgo can be calculated from the separate items of income and outgo
as:
total gross claim payments to be made (gross of reinsurance)
 reinsurance and other recoveries to be received
+ expenses to be paid
 outstanding premiums to be received
+ tax and dividend payments to be made.

These elements are discussed in more detail later in this section.

Each of these items can be projected in the same way as for reserving calculations.
However, it is the individual period-by-period projections that are important and
calculations should, at least initially, be done on a monthly or quarterly basis to take
account of any seasonal effects.

It is best to split the business into relatively homogeneous risk groups, for example, by
class, type of claim within class and currency.

Care needs to be taken however that the data does not start to lack credibility if the volume of
data in each risk group is lower.

The Actuarial Education Company © IFE: 2019 Examinations


Page 12 SP7-19: Investment principles and asset liability matching

4.1 Claim payments and recoveries


Claim payments are almost always the most significant item of the liability outgo. The
projection must include:

 all future payments in respect of unsettled reported claims

 IBNR and reopened claims

 claims that will emerge from unexpired risks.

Ideally, the claim payments should be treated as gross of reinsurance and other recoveries,
as the timings of these recoveries do not usually coincide with those of the claim payments.
Reinsurance and other recoveries should be included separately as an offset to the liability
outgo, allowing for when they are expected to be recovered. A chain ladder type of
projection would be possible. However, the results will probably be subject to considerable
uncertainty.

If necessary, claims could be projected net of reinsurance recoveries, but with a margin to
allow for recovery delays and defaults.

4.2 Expenses
The expenses relating to handling the claims can be allowed for either explicitly or
implicitly.

For example, an implicit allowance may just be an additional percentage loading to the claim
payment.

Other office expenses would usually need to be projected, although this would depend on
the precise purpose of the projections.

As we will see in Section 5, expenses relating to investment will normally be considered as an


offset to the projection of the asset proceeds.

4.3 Premium income


Premiums will usually be taken gross of reinsurance premiums but net of commission, and
possibly also net of internal initial administration expenses. Any premiums due to
reinsurers would then be deducted.

Where new business and renewals are not included, allowance for outstanding premiums
should be relatively easy because most of the cashflows are likely to emerge within a
relatively short time period.

This assumes that the business written is annual.

However, when including new business and renewals, allowance will need to be made for:

 expected rates of premium growth (and profitability) in the light of the company’s
business plan

 each class of business

 the competitive position

 the effect of the insurance cycle.

© IFE: 2019 Examinations The Actuarial Education Company


SP7-19: Investment principles and asset liability matching Page 13

These factors obviously also affect the projection of claims and expenses relating to the
new business and renewals.

This can be an area of significant uncertainty.

In either case, the premium income should be considered separately by source of business
to allow correctly for delays and acquisition costs.

4.4 Tax and dividend payments


Some attempt may also be made to project future tax and dividend payments. Other than in
relation to the next few months, assumptions will be needed about future investment
returns, tax rates and dividend philosophy. These components of liability outgo can only be
assessed in tandem with the projection of the asset proceeds and profitability of the
company (as dividends and tax are generally driven by profitability).

The Actuarial Education Company © IFE: 2019 Examinations


Page 14 SP7-19: Investment principles and asset liability matching

5 Estimating the investment income in future time periods


The process of estimating income from the investments is relatively straightforward
compared with the equivalent process for the liabilities.

For example, the income due from fixed-interest stock is known with certainty, notwithstanding
any default risk. However, some assumptions such as future inflation rates, dividend yields and
interest rates, will have to be made for other investment classes.

An allowance also needs to be made for any capital proceeds, eg the redemption payments on
any maturing bonds, during the period of consideration.

For all types of investment, it is important to make allowance for the expenses of
investment.

Investment expenses are usually a negative adjustment to projected asset proceeds.

The projections will need to make allowance for the future volatility of capital values and
investment income.

© IFE: 2019 Examinations The Actuarial Education Company


SP7-19: Investment principles and asset liability matching Page 15

6 Sensitivity of cashflow results to changes in assumptions


It is important to understand how the liabilities and assets react both in isolation and most
importantly relative to each other if different assumptions are used. This helps ensure that the
investment strategy is appropriate to cope with a range of possible situations.

A stochastic model can give an indication of the sensitivity of both the assets and the
liabilities to changes in the assumptions. However, in practice many companies perform a
large number of deterministic simulations at the sensitivity testing stage.

This is less time-consuming and cheaper to carry out.

Parameter values are not chosen at random, but a large number of runs are performed
where each one is based on a different combination of parameter values drawn from a large
collection of possible sets of best estimates. The range of assumptions used can reflect the
level of uncertainty of liabilities and volatility of assets.

If it is discovered that small changes to the assumptions produce dramatic alterations to the
net cashflow, the insurer will know that more substantial positive cashflow will be needed in
the overall model to protect against the adverse effect of such changes.

The Actuarial Education Company © IFE: 2019 Examinations


Page 16 SP7-19: Investment principles and asset liability matching

7 Solvency requirements
Where market values are used, the solvency position is directly influenced by the volatility
of the values of assets.

In other words, if assets are valued on a market-related approach, a sudden drop in the market
value of those assets will result in an immediate reduction in the insurer’s solvency position. The
company will therefore be concerned not to invest too heavily in assets with a volatile market
value.

This is a particular issue where the solvency test is not consistent, ie the discount rate used to
value the liabilities is not market related, and therefore does not reflect the current asset holding.

If a projection model is required to incorporate an assessment of the likely level of solvency


at each period in the future, it must be extended to allow for any statutory requirements
relating to the valuation of assets and liabilities. A comparison can then be made of the
actual level of solvency with that acceptable to the statutory authorities.

It is also likely that an insurer will place further, more stringent, class-specific solvency
requirements on itself. Such requirements are likely to be related to:

 the length of tail of the liabilities (ie technical provisions)

 the likelihood of catastrophes and accumulations

 the spread of risk groups within the portfolio, ie how well diversified the insurer is by
the classes written, and how predictable the liability outgo is.

These factors will need to be built into the model.

To consider the progress of the insurer on an ongoing basis, allowance must be made for
future new business. This is a difficult assumption to set. The purposes of such an
investigation might be to assess:

 the future solvency of the company under different volumes of new business

 the future solvency under different scenarios for asset distributions

 the likely levels of profitability.

Although a deterministic model (with variations in the assumptions) could be used, a


stochastic model would be the most effective way of incorporating allowance for the
volatility in asset values and the uncertainty in the claims experience.

The stochastic model also provides more detailed output.

If one is using a stochastic projection of the future development of the insurer, by carrying
out many simulations it should also be possible to plot a probability distribution function
for the solvency ratio of the insurer at different times in the future.

© IFE: 2019 Examinations The Actuarial Education Company


SP7-19: Investment principles and asset liability matching Page 17

8 The influence of the supervisory authorities on investment policy


The supervisory authorities may impose some restriction on the assets an insurer can hold
and the way in which they are valued. This may arise in one of the following situations:

 where the regulator deems it appropriate to impose restrictions on the market


generally, or on insurers in particular product lines or stages of development
– for example, run-off companies who will have limited liquidity, due to no
future premium, may be restricted to short-term more liquid assets

 where an insurer is in difficulty, having breached, or being in danger of breaching,


regulations.

For example, the following controls may be used depending on the level of intervention
indicated by the insurer’s breach of requirements:

 restriction on the amount of certain types of assets that can be taken into account
when assessing solvency

 custodianship of assets

 prevention from holding certain assets

 prescription to hold certain assets

 requirement to hold reserves against mismatching assets to liabilities or limitations


on ability to mismatch.

Question

In relation to the bullet points above:


(i) State the term commonly used for assets that can be taken into account for the purposes
of demonstrating statutory solvency.
(ii) Describe what is meant by a ‘custodian’ of assets.
(iii) Give an example of a type of asset that an insurer may not be allowed to hold.
(iv) Suggest what sort of assets an insurer might be required to hold.
(v) Explain why a mismatch reserve might be required.

Solution

(i) Assets which can be taken into account for the purposes of demonstrating statutory
solvency are called admissible assets.
(ii) The custodian is a party that holds the assets for the insurer and is responsible for the
safekeeping of those assets and all administrative matters relating to those assets.
(iii) Derivatives held for speculative purposes.
(iv) An insurer may be required to maintain a minimum holding in government bonds.
(v) This encourages the insurer to match closely, since the more closely it matches the lower
the mismatch reserve required.

The Actuarial Education Company © IFE: 2019 Examinations


Page 18 SP7-19: Investment principles and asset liability matching

9 Objectives and constraints


Having a successful investment policy is an integral part of the operation of a general
insurance company. The return to shareholders in a proprietary insurance company (or
policyholders in a mutual insurance company) is comprised of:

 an insurance (or underwriting) result

 an investment result (including unrealised capital gains or losses depending on the


form of local GAAP).
Recall that GAAP stands for generally accepted accounting principles. Whether or not an
insurer must report unrealised capital gains or losses in its investment result will depend
on the accounting practices in the territory concerned.

9.1 Objectives
As mentioned in Section 1.1, the primary objective with the investment of the assets that
support the liabilities of a general insurer is to maximise the return. This is subject to:

 the overriding constraint of meeting claim and expense liabilities as they fall due

 maintaining a level of solvency compliant with both the statutory regulations and the
company’s risk appetite.

An insurer would typically analyse its asset risk separately from its insurance risks and
other liabilities. Many companies have started to consider all their risks from wherever they
arise using asset liability modelling (ALM), and increasingly so under the requirements of
Solvency II. ALM is discussed later in this chapter.

9.2 Constraints
The chosen investment strategy must be appropriate for the risk appetite of the general insurer
as dictated by the insurer’s management and shareholders.

The risk appetite of a general insurer depends on a number of factors or constraints that
can be categorised under the following headings:

 liabilities

 assets

 external influences

 insurer–specific considerations.

The impact of these on investment policy is discussed below.

© IFE: 2019 Examinations The Actuarial Education Company


SP7-19: Investment principles and asset liability matching Page 19

Liabilities
Consideration needs to be given to the characteristics of the liabilities in order that appropriate
matching assets can be identified. For example, the insurer needs to consider the following
characteristics:
 The nature of the existing liabilities – are they fixed or ‘real’ in monetary terms. The
majority of general insurance liabilities will be real in nature.

 Currency of existing liabilities – many domestic, personal and commercial insurers


may have portfolios predominantly denominated in their local currency. However,
international insurers and reinsurers have portfolios that contain a range of
currencies. The principal currencies used in the London insurance market are
sterling, euro, yen and US, Canadian and Australian dollars.

 Term of existing liabilities – most general insurers’ portfolios are likely to contain a
significant proportion of short-term liabilities (1–3 years), with a smaller proportion
of medium-term (4–10 years) and long-term liabilities (10 years and above; for
example, asbestos liabilities).
This will also depend on whether the insurer has been subject to recent large losses,
and whether business volumes are growing or shrinking each year, as well as the
position in the insurance cycle and so on.

Question

(i) Without looking back at your notes, give an example of a liability that will be real and one
that will be fixed in nature.

(ii) Give an example of classes of business giving rise to short, medium and long-term
liabilities.

Solution

(i) Nature of liabilities

Property damage or bodily injury claims on public liability business are real in nature.
(There are many other similar examples.)

Claims from fixed-benefit classes (such as personal accident or medical expenses


insurance) are fixed in nature.

(ii) Term of liabilities

Short term – most property damage claims.

Medium term – bodily injury claims from liability classes such as public liability and
product liability.

Long term – bodily injury claims on employers’ liability business, eg asbestos-related


claims.

The Actuarial Education Company © IFE: 2019 Examinations


Page 20 SP7-19: Investment principles and asset liability matching

 Level of uncertainty of existing liabilities – both in amount and timing.

 Estimated future liabilities arising from the portfolio of business planned  this will
depend on the volume and classes of future business written.

 Location of liabilities  for example, trust funds for Singapore, US, and so on may
require assets to be held locally and may restrict profit payments and the assets in
which investments are held.

 Whether the liabilities are discounted.

 Whether the liabilities include expenses.

Assets
The current asset holding and potential returns on different asset classes must be understood
when making the investment decision.

Question

Explain why it is important to take account of the current asset holding when formulating an
investment strategy.

Solution

This is to ensure that sufficient diversification is achieved and the investment strategy as a whole
meets the insurer’s risk appetite.

The general insurer should consider the:


 size of assets, in relation to the current liabilities – the larger the quantity of free
assets, the more the company has freedom to invest widely

 expected long-term return from various asset classes

 expected volatility within the various asset classes

 existing asset portfolio

 non-investible funds – not all of the assets will be available for investment,
eg moneys held by brokers, policyholders or reinsurers
These funds were discussed in an earlier section.
These funds may not generate any return to the insurer and thus can distort the
overall return.
In other words, these funds form part of the asset portfolio but since they do not
generate a return they lower the average return on the portfolio.
 economic outlook
The insurer will want to assess its likely impact on security and return on investments,
eg future inflation affects the return on index-linked bonds
 risk appetite.

© IFE: 2019 Examinations The Actuarial Education Company


SP7-19: Investment principles and asset liability matching Page 21

External Influences
The impact of the external environment should be considered when determining the investment
strategy, for example:
 tax treatment of different investments and the tax position of the general insurer
For example, the insurer will consider the relative tax treatment of income vs capital gains
 statutory, legal, ethical or voluntary restrictions on how the insurer may invest

Question

List statutory restrictions that might impact a general insurer’s investment strategy.

Solution

 restrictions on the amount of certain types of assets that can be taken into account when
assessing solvency
 custodianship of assets
 prevention from holding certain assets
 a prescription to hold certain assets
 a requirement to hold mismatching reserves

 statutory valuation requirements

 solvency requirements – most territories have a step-wise process of intervention


and therefore the strategy for maintaining solvency is targeted at a much higher
level than the pure statutory minimum margin

 rating agency constraints on capital required to maintain the insurer’s desired rating
This will impact the perceived strength of the insurer, and may therefore result in more
favourable terms for raising future capital.
 competition – strategy followed by other funds
It is unwise to follow a strategy very different from other insurers selling similar business
because, if there is a failure of this strategy the insurer will be alone in suffering losses.
 regulatory constraints: for example, those imposed by Lloyd’s, the Prudential
Regulation Authority (PRA) and the Financial Conduct Authority (FCA).

Insurer-specific considerations
 Risk appetite – each general insurer will differ in the amount of risk it is willing / able
to accept, perhaps due to other considerations within the company group, wider
management objectives or board appetite.
(The risks to which a general insurer is exposed are discussed in the next section.)
 Company-specific investment objectives: eg, ethical investment.

The Actuarial Education Company © IFE: 2019 Examinations


Page 22 SP7-19: Investment principles and asset liability matching

10 Risks
The risks relating to the investment strategy of a general insurer are summarised below:

 Liquidity risk – the risk of not having sufficient cash to meet the liabilities as they fall
due.
For example, if an insurer locks into investments with a fixed term, it is exposed to
the risk that the cash will be needed to settle claims before the term expires.
A general insurer’s liabilities are very uncertain in both amount and timing and
therefore the insurer must maintain a reasonable level of liquidity in its portfolio.
Exposure to a catastrophe event creates an immediate demand for claims payment.

 Currency risk – the risk that changes in the value of the assets, or the liabilities of
the company adversely impact the available capital or investment funds. If the
assets and liabilities are not matched by currency, movements will not exactly offset
each other; that is, one could increase relative to the other.

 Market risk – the risk relating to changes in the value of the portfolio due to
movements in the market value of the assets held. General insurers typically have
relatively short-term liabilities that may be matched by cash or short-term bonds.
These asset classes are in general less volatile than long-term bonds and equities;
hence market risk is generally lower.

Question

Explain why cash and short-term bonds are generally less volatile than long-term bonds and
equities.

Solution

We can value an asset using the discounted cashflow approach. A small change in interest rates
will have less impact on the cashflows for cash and short-term bonds, since the cashflows are less
far in the future. Hence cash and short-term bonds have more stable values.

 Economic risk  the risk of investing in certain asset classes at certain


(disadvantageous) stages in the economic cycle.
 Credit risk – the risk that a counterparty to an agreement will be unable or unwilling
to fulfil its obligations. The largest credit risk for a general insurer is likely to be the
failure of its reinsurer(s). Credit risk is also correlated with insurance risk;
eg following a catastrophe loss in the market, insurers may also face increased
credit risk through default of their reinsurers.
Insurers can also be exposed to credit risk from other assets, eg the downgrading of
any bonds held would be a credit risk.
 Operational risk – includes the risk of loss due to fraud or mismanagement within
the insurer itself.

 Relative performance risk – the risk of underperforming comparable institutional


investors. For general insurers, this risk is not as significant as liquidity, market and
credit risk.

© IFE: 2019 Examinations The Actuarial Education Company


SP7-19: Investment principles and asset liability matching Page 23

 Group risk – the risk arising from belonging to a group. For example, an insurance
subsidiary might be required to change investment strategy following a change in
the parent’s requirements. Liquidity risk might crystallise if, for example, the parent
requires a dividend from the subsidiary.

 Contagion risk  that is, that the collapse of one insurer is more likely to bring down
others.

Note: pure underwriting risk is uncorrelated with market risk, making the cost of capital for
an insurer lower than, for example, banks.

The Actuarial Education Company © IFE: 2019 Examinations


Page 24 SP7-19: Investment principles and asset liability matching

11 Investment policy in practice


In this section we consider the different cashflow positions which a general insurer may be in, and
the impact on investment strategy.

Insurers are usually in one of three situations as regards cashflow:


1. insurer expects that premiums and investment income will continue to exceed claim
payments for the indefinite future
2. insurer is in run-off and expects to have to rely on the maturity and realisation of
assets
3. insurer has suffered a major insurance event and needs to obtain short-term
liquidity to settle claims.

11.1 Ongoing insurer


For an ongoing insurer, it is theoretically possible to match the expected claims and
expense outgo to the expected premium and investment income by nature (real or fixed),
term, amount and currency.

Note: ongoing premium income is an important cash inflow for a general insurer and can be
used to provide liquidity.

In theory, perfect matching can be used to immunise the portfolio from investment losses or
profits. Investments can fail for reasons that do not affect liabilities (for instance, if the
asset portfolio has a longer mean term than the liabilities, higher interest rates will result in
a loss, and vice versa). In practice, it is impossible to match the outgoings of a general
insurer perfectly given the uncertainty inherent in the timing and quantum of claims
settlement.

Even in theory, insurers would not always strive for a perfectly matched portfolio.

By taking more risk (moving to a less well matched portfolio) it may be possible to increase
returns.

If an insurer makes underwriting losses, investment profits (derived from unmatched


portfolios) are necessary to offset these losses.

It would not be uncommon for long-tail classes to make underwriting losses. However, insurers
can still be profitable overall due to good investment returns achieved over the relatively long
period between the premium being received and claims paid.

The theory of matching does, however, provide the insurer with a basis on which to assess
the risks inherent in its investment policy, and in particular, a benchmark against which to
measure its exposure to interest rate movements.

It will be important for the insurer to understand the matched position even if this is not what is
held. By doing so the insurer will understand the degree to which it has moved away from this
position and therefore the degree of mismatching risk taken.

© IFE: 2019 Examinations The Actuarial Education Company


SP7-19: Investment principles and asset liability matching Page 25

11.2 Run-off insurer


For an insurer in run-off, it is vital that the assets can be liquidated as they are needed,
because premium income is insufficient to meet future cashflow needs. If the insurer is not
matched, it needs to plan carefully to realise or reinvest assets as needed.

Question

Suggest what characteristics the insurer would like the assets held to have in this situation.

Solution

Liquid, in other words close to cash and marketable, ie easy to trade.

The assets should also have a high running yield (ie give a high percentage of their return through
income that can be used to meet claims, rather than through capital growth).

11.3 Insurer post large event


An insurer may suffer a catastrophic event and have many claims to settle at one point in time.

In this case, the need is for liquidity to settle claims and many insurers have credit lines
available from banks and other financial institutions to assist them with short-term
cashflow. Often, large settlements are done with reinsurer support so that the primary
insurer is not left having to make a claim on its reinsurer, who settles its share of the loss at
the same time as the primary insurer.

Note: an event can produce losses that lead to a rating downgrade, which could cause
severe business difficulties for the insurer due to market concerns.

11.4 New business


When determining its investment policy, an insurer should also consider income from and
liabilities associated with policies written after the opening balance sheet date. This is a
consideration of whether the company will be in a cashflow positive position given the
existence of its projected new business premiums under reasonably plausible scenarios.

The Actuarial Education Company © IFE: 2019 Examinations


Page 26 SP7-19: Investment principles and asset liability matching

12 Asset liability modelling (ALM)


The projections of assets and liabilities become most useful when the relationship between
the two is looked at.

The model used can be deterministic or stochastic.

There is no single, universally accepted method for carrying out such projections, although
the simplest approach is to use a deterministic model. The asset proceeds and liability
outgo in each future time period are then calculated using a single set of assumptions.
Actuaries often refer to such projection models as ALM.

ALM is characterised as a simultaneous projection of asset and liability cashflows over a chosen
time period. Since this is a cashflow projection, timings and amounts of both asset proceeds and
liability outgo need to be understood,

One approach is to exclude income on free reserves from the asset proceeds calculations
and treat the free reserves, and their proceeds, as a balancing item. This can be done by
using as much of these reserves as is needed to meet net liability outgo, and accumulating
the balance with interest.

When ignoring new business and renewals (provided assets are sufficient to meet
liabilities), the balance of cash should stay positive at all times. If asset proceeds net of
liability outgo are negative at any point, it implies that the assets do not match the liabilities
by term, and so could be sold to meet the required cashflows.

Inclusion of future written business, however, makes the situation much more complex. In
particular, if business levels are not declining, a mismatching of assets and liabilities may
be adopted, reducing the need for realisation of assets.

In other words, new business income can be relied upon to some extent to meet short-term
liability outgo.

An alternative to a deterministic model is a stochastic model. The latter is, arguably, the
more appropriate way of allowing for the volatility and uncertainty underlying the assets
and liabilities.

A stochastic model is the more-often employed approach to ALM when data is sufficient
and the task is large enough. This enables the insurer to scan the simulated results to
assess the probability that an unfavourable situation will occur.

The insurer could then attempt to reduce this probability to an acceptable level. However, it
is highly dependent on the chosen probability distribution functions, and other
assumptions, such as those around dependencies between risk types, products and so on.

If a stochastic approach is adopted it will be just the key assumptions, eg investment returns that
will be modelled stochastically with other assumptions modelled deterministically.

© IFE: 2019 Examinations The Actuarial Education Company


SP7-19: Investment principles and asset liability matching Page 27

13 ALM for developing an investment strategy

13.1 Introduction
Historically general insurers have modelled assets and liabilities in ‘silos’, if at all.

In other words they have considered the business class by class rather than looking across classes
and determining where there is reduced risk due to diversification or negative correlation, or
indeed increased risk due to concentration.

Typically the investment strategy of general insurers is to invest in low-risk, liquid assets
because the need to match by term and holdings of volatile assets would increase the
overall uncertainty of the business result. In these circumstances, the cost of investing in a
stochastic asset model may outweigh the benefits.

However, the Solvency II regime and the requirement for greater analysis and disclosure by
directors means that there is external pressure to better understand all of the risks faced by
the company in a holistic fashion.

This idea is closely aligned with the topic of enterprise risk management, which you may
remember from your earlier studies.

As a consequence, there has been significant movement in the industry to implement ALM
models in general insurance. (See for example IAIS guidance on internal models
(Sept 2007) – a model should cover all material risks to the company.)

13.2 What is ALM?


ALM is a term used to describe any model that covers both the asset and liabilities of an
entity within one structure. This means that an ALM is generally an extension of other
models mentioned within these notes, as the basic building blocks of an ALM will be other
stochastic or deterministic sub-models, that is, an asset model and a liability model, and the
links between the two.

The basic concept is quite simple:


 project liability outgo in each future time period
 project asset proceeds in each future period
 compare the two for each future period
 run the comparisons again using different assumptions
 decide whether the asset proceeds are appropriate for the liability outgo
 if not, investigate alternative asset distributions.

Inevitably, the reality is much more complex. For example, we need to make several important
decisions regarding the scope of the model, eg:
 Should we allow for new premium income?
 Should we project the solvency position for future time periods also?

The Actuarial Education Company © IFE: 2019 Examinations


Page 28 SP7-19: Investment principles and asset liability matching

13.3 The ALM process

The structure of a typical ALM

INPUTS CALCULATIONS OUTPUTS

Economic capital model


Asset model
 Business  Output
plans reports
Assets Asset / liability
model
 Historical
information Financial
Liability model reporting
 MI
 Input Tax (Management
Premiums Expenses
database Information)
Investment
strategy
Reinsurance Gross
recoveries claims

As described above, ALM is a catch-all term that could encompass models of very different
complexity, varying from simple deterministic scenariotesting frameworks to complex
stochastic models.

An ALM may be used to determine the optimal investment strategy in line with the firm’s
investment goals (eg maximise profitability for a chosen risk appetite).

If the purpose is to investigate the suitability of the asset distribution compared to our present
liabilities, then the projections might be made without any allowance for future new business.

However, if the purpose is to investigate the overall financial position of the office and consider
its future solvency position, then the projection will need to incorporate future business into our
projections.

The projections will be carried out on a best estimate basis considering the overall liability outgo
and asset proceeds in each future time period.

For insurers with several million policyholders it will be more efficient to carry out the projections
using model points. Each model point is representative of a set of policyholders and the modelled
experience can then be scaled up to reflect the total size of the portfolio. The advantage of this
approach is that a much smaller volume of data is required, and the model will be much more
manageable.

© IFE: 2019 Examinations The Actuarial Education Company


SP7-19: Investment principles and asset liability matching Page 29

As we saw in Section 4, the liability outgo will consist of:


total gross claim payments
– reinsurance and other recoveries
+ expenses
– outstanding premiums received
+ tax and dividend payments

The claim payments are almost always the most significant item of the liability outgo and will be
subject to the most uncertainty. Their projection must include all future payments in respect of
unsettled reported claims, IBNR and reopened claims, and claims that will emerge from unexpired
risks.

The method used for estimating asset proceeds will vary depending upon the asset category being
considered.

Let’s see some examples:


 Ignoring the possibility of sale and the risk of default, the stream of income and
redemption proceeds from fixed-interest securities is of known amount and term.
 The income stream from an equity portfolio is more difficult to estimate. It might be
based on the current dividends but assumed to increase from year to year to reflect the
expected path of overall dividend increases. This assumption must be made to be
consistent with other assumptions, eg expense and claim inflation.

An ALM may also be used to determine solvency capital or in a wider capital management
programme.

If the solvency test is based on market values then the solvency position will be directly
influenced by the volatility of the capital values of assets. The choice of asset mix will therefore
be crucial to ensuring the ongoing capital adequacy of the company.

If a projection model is required to incorporate an assessment of the likely level of solvency at


each period in the future, then it must be extended to allow for any statutory requirements
relating to the valuation of asset and liabilities. A comparison can then be made of the actual
level of solvency with that acceptable to the statutory authorities.

The strength of the basis used in the ALM will depend on the purpose of the exercise. An implicit
margin for prudence can be incorporated into the model by discounting the projected future
liability outgo. A zero rate of discount could be used, for example, to replicate a ‘strong’ valuation
basis.

A stochastic model is likely to be most appropriate since it can incorporate an allowance for the
volatility in asset values and the uncertainty in the claims experience.

The results of such a stochastic model should enable a probability distribution function to be
plotted for the solvency ratio of the insurer at different times in the future.

The Actuarial Education Company © IFE: 2019 Examinations


Page 30 SP7-19: Investment principles and asset liability matching

13.4 The typical structure of an ALM

Inputs – Assumptions Modelled Variables & Outputs


Interdependencies
To test an investment This will depend on the Output is the collection of
strategy, input assumptions scope of the model. In results for each modelled
might include: general terms: output variable.

 Target asset portfolio  Assets within asset Eg, 20,000 simulations of


(eg 20% equity / 50% portfolio (eg cash balance sheet and profit /
bonds / 30% cash) deposits, equities, loss statement
bonds) This information will be
 Investment rules /
guidelines (eg fall in  Liabilities within summarised to derive
equity market by 20 liability portfolio information of interest
implies switch to bonds; (eg classes of Eg, 1-in-200 year, 1-in-10
maximum deviation from business, split into year, 1-in-5 year adverse
target portfolio before homogeneous groups) outcome (loss)
rebalancing)
 Inflationary links Enables analysis of optimal
 A starting assumption strategies by comparing
might be the current asset  Interdependencies
various statistics of the
portfolio (eg, 22% equity / between correlated
outputs for various
43% bonds / 35% cash) variables (eg between
different input strategies
two asset types, or
Economic Scenarios between property and Projection of expected
MIG losses) outcomes for business
 From an economic
planning purposes
scenario generator (ESG)*
Liabilities

 Inputs for stochastic


modelling of GI liabilities
covered elsewhere in the
Core Reading

* An ESG will typically take the form of a specialised asset model that stochastically models
the performance and interactions of various asset classes. The output of this model will be
the performance for each economic variable (for example, inflation / asset classes) at each
future projection point, for several simulations. This table of simulation outputs will then be
used within the main ALM as if the ESG was a part of the ALM. The reason for separating
the two models is that an ESG can be a very complex model, the building, parameterisation
and running of which is often outsourced. The ESG concept is considered further in
Chapter 21.

13.5 Use of ALM in investment decisions


An investment policy considers the mix of investment types that the company holds in the
various currencies where the company has exposure. It is usual for investments to be
subdivided into cash, government stocks, index-linked securities, commercial bonds,
equities and property.

The ALM can then be used to consider different mixes of investment as part of the strategy.

An economic scenario generator (ESG) is a stochastic asset modelling tool.

© IFE: 2019 Examinations The Actuarial Education Company


SP7-19: Investment principles and asset liability matching Page 31

The ESG produces interest rates and inflation rates, and translates those into the level of
the market and asset prices for each model point. The impact on solvency of changes to
investment policy can be tested using different starting mixes of assets and different
acquisition rules for assets going forward.

Graphical output of solvency for each starting asset mix and sets of investment rules can
be compared, allowing the company to investigate the impact of different strategies on its
balance sheet and solvency level.

The insurer will select the strategy that best meets the investment objectives, that is,
maximising the return subject to the overriding constraint of meeting claim and expense
liabilities as they fall due and maintaining an appropriate level of solvency.

For example, the company could examine the impact of a strategy that invests all of its
assets in equities, generating the higher returns associated with equities but the
corresponding increased volatility. This could be compared with a low risk strategy of
retaining all assets in cash.

In theory, it would be possible to assess liquidity risk associated with a particular


investment strategy using Dynamic Financial Analysis (DFA).

DFA is a phrase given to overall actuarial modelling in financial services, part of which
might be asset-liability modelling. It is not, however, a practical approach because of the
complexity of the model required to replicate performance at the micro level.

However, it should be noted that many practitioners use the terms DFA and ALM interchangeably.

13.6 Use of stress and scenario testing


Stress and scenario testing may also be used to determine or test an investment strategy.

Stress testing involves exposing the chosen strategy to extreme events to check it is robust, ie to
check that the resulting solvency position remains within the risk guidelines of the company.
Scenario testing involves developing consistent sets of assumptions that can be used to
investigate various situations, for example downgrading of assets, a low or high inflation
environment etc.

For example, a rating downgrade would generate the need to collateralise reinsurance or
other transactions that the company may have entered into with other counterparties. This
could trigger the need to modify investment policy at the margins.

If a reinsurer’s credit rating is downgraded, cedants may require additional security to ensure that
the reinsurance recoverables will be paid. Collateralised reinsurance means that the reinsurer
sets aside capital to collateralise the risks it is underwriting. Typically, this collateral is in the form
of a letter of credit or a reinsurance trust.

Liquidity risk is usually assessed using stress and scenario testing.

For example, a scenario might be a major US hurricane coinciding with a fall in asset
values. The gross claims payments, claims handling expenses, reinstatement premiums,
reinsurance recoveries and any delays between gross claims payments and recoverables
would be considered.

The Actuarial Education Company © IFE: 2019 Examinations


Page 32 SP7-19: Investment principles and asset liability matching

Stress and scenario testing has the advantage of being easily understood and challenged
by general management as well as being used as a real-world check on assumptions
underlying a stochastic ALM.

13.7 Reverse stress and scenario testing


A complementary case of stress and scenario testing is reverse stress testing. Some
regulatory authorities require firms to carry out reverse stress testing, which is an
assessment of scenarios and circumstances that would render its business model unviable.

Reverse stress testing differs from general stress and scenario testing in that the starting
point is the assumed outcome of business failure, thus the exercise being the identification
of circumstances where this might occur, whilst the latter looks at the resulting outcomes
arising from specified changes in circumstances.

In other words, a reverse stress and scenario test identifies the circumstances / model
assumptions required for the business to fail.

This is in contrast to the more common use of a stress or scenario test which analyses the effect
on the business of a given set of circumstances / assumptions.

© IFE: 2019 Examinations The Actuarial Education Company


SP7-19: Investment principles and asset liability matching Page 33

14 Asset liability modelling case study


The purpose of this case study is to develop your understanding of how the material in this
chapter might be applied to a particular practical problem. The example below is an exam-style
question. To get maximum benefit from it you should attempt the question fully yourself before
checking your attempt against our solution.

Case study
You are the actuary to a medium-sized proprietary private motor insurer. Your insurance
company currently protects the account with an individual excess of loss reinsurance contract.
The current arrangements have been in place since 1 January 2011.

You have been asked to review the level of the lower retention limit of the treaty. To do so, you
decide to conduct some stochastic asset liability modelling to investigate the effect that different
levels of retention are likely to have upon future profitability and financial strength.

Question

(i) Describe how you would estimate the future liability outgo.

(ii) Describe how you would investigate the choice of an appropriate retention level given
that the company wants to maximise its value to shareholders subject to an acceptable
level of risk.

Solution

(i) Estimation of liability outgo

Deciding upon the retention level is a management decision. Hence, it is necessary for the
projection to be realistic. Thus, all assumptions should be made on a best estimate basis.

The main component of liability outgo will be claims, so we devote most thought to this.

We can use the historical claims data to build a model of the gross original claims expected
(numbers and amounts) in the market in the future. To do this we will need to adjust the
historical data to allow for:
 IBNR claims and development of reserves for reported outstanding claims (ie the IBNER
part)
 claims inflation
 any changes to claim definitions or cover provided
 effects of any previous reinsurance
 excesses or deductibles.

The allowance for IBNR and IBNER can be calculated using standard reserving methods such as
chain ladder techniques.

The Actuarial Education Company © IFE: 2019 Examinations


Page 34 SP7-19: Investment principles and asset liability matching

The policy limit at the date of loss should be inflated for each claim into current value terms using
an appropriate index.

Motor claims tend to be seasonal so the projection should be done on a monthly or quarterly
basis (initially at least).

Claim distributions can then be fitted to each major claim type separately to minimise problems
with changing mix of business and heterogeneity generally, eg fire, theft, accidental damage and
bodily injury.

In practice the level of detail that we can work with will be limited by the amount of data
available. We do not want to divide the data down into such small groups that it ceases to be
credible.

The adjusted data should be analysed by claim frequency and amount separately. This will enable
us to spot any trends in the data. These should be investigated and projected appropriately into
the future.

Again, in practice, trends are not easy to identify. We need to consider whether the historical
information is reliable and still relevant. This problem may not be too bad for a medium-sized
motor insurer. However, a small or newly-established insurer would clearly lack reliable historical
information.

Notice how the above comments apply to any claims investigation process whether it be for model
office projections (as here), rating or reserving.

Realistically, the company will continue to write business so we need to allow for new business.
This is a sensitive assumption, so we should explore the effect of different levels of business on
the results obtained.

Other liabilities such as expenses and commission also need to be modelled. However, these tend
to be more straightforward than the claims outgo, eg commission can usually be x% of premiums.

(ii) Investigating suitable retention level

We need to project the liabilities for, say, five years to explore an appropriate retention level.
Any longer period may well lead to spurious accuracy.

Having built our distributions for claim numbers and amounts in (i) we can carry out a series of
stochastic runs, say 10,000. Each run will generate a random number of claims, and a random
amount for each claim. In this manner we can construct our total claims experience for each time
period.

Reinsurance recoveries and premiums payable need to be established and modelled to vary with
the choice of retention level. The effects of the number of free reinstatements and reinstatement
premiums also need to be modelled.

Having obtained the gross claims experience it should then be reasonably straightforward to build
up the cashflows, including gross claim amounts out, reinsurance premiums out and reinsurance
recoveries in.

© IFE: 2019 Examinations The Actuarial Education Company


SP7-19: Investment principles and asset liability matching Page 35

Some criteria need to be established in order to decide what the optimum level of retention is.
For example, ‘to vary the retention level to find the level which gives the greatest return on
shareholders’ capital subject to the probability of ruin at each time point in the projection not
being more than 0.5%’.

By ranking the simulations we can take the fiftieth worst scenario out of 10,000 as representing a
0.5% probability.

Possible choices of criteria for the definition of ‘ruin’ include the following:
 failing to meet any solvency margin
 negative total net worth, ie assets less liabilities becoming negative
 internal management target, eg avoiding y% fall in dividend payment
 not able to meet cashflow requirements, eg due to illiquid assets.

The lower the retention level chosen, the greater the solvency protection obtained, but the lower
the average profits expected because some of the profits will be ceded to the reinsurer.
Conversely, the higher the retention level, the lower the solvency protection but the greater the
expected profits.

Investment strategy also requires special consideration. The model needs to contain interactive
links between the degree of matching, investment strategy and financial strength. This is
necessary because investment matching can have a major effect on solvency and if not careful we
may over- (or under-) state the probability of ruin.

Example

A £250,000 retention may give an expected average return on capital (ROC) of 12%, with a
probability of ruin of 0.4%. Increasing the retention limit to £500,000 may give a ROC of 14% but
a ruin probability of 0.7%. Further modelling may show that a limit of £350,000 gives a ROC of
13% and ruin probability of 0.5%. Hence, given the criteria chosen, we may recommend £350,000
as the optimum retention level.

Key uncertainties

The modelling described above relies heavily upon estimates and results should be interpreted
with care, eg:
 reinsurance may not be available at the price modelled or in the capacity required
 new business levels are uncertain
 claims experience is uncertain.

It is important to recognise the key uncertainties in the process. Otherwise, as actuaries, we may
be accused of merely operating black boxes, putting numbers in and getting numbers out with no
understanding of the bit in the middle.

The Actuarial Education Company © IFE: 2019 Examinations


Page 36 SP7-19: Investment principles and asset liability matching

15 Glossary items
Having studied this chapter you should now read the following Glossary items:
 Asset liability modelling (ALM)
 Dynamic financial analysis (DFA)
 Free reserves.

© IFE: 2019 Examinations The Actuarial Education Company


SP7-19: Investment principles and asset liability matching Page 37

Chapter 19 Summary
The investment strategy should be chosen to maximise returns subject to meeting
contractual obligations, whilst ensuring the risk against not receiving the return is within the
company’s tolerance.

Assets should match liabilities by:


 term
 amount
 nature
 currency.

If classes of business are written with widely fluctuating claims there will be a need for
liquidity.

Free reserves should be invested with the aim of maximising long-term returns, except to
the extent they are needed to cover potential shortfalls in the technical reserves. They are
usually invested in equities.

Not all money is available to be invested, examples of non-investible funds are moneys held
by:
 agents
 policyholders
 reinsurers.

Liability outgo should be assessed in order to determine investment strategy. Cashflows to


consider are:
total gross claim payments
– reinsurance and other recoveries
+ expenses
– outstanding premiums received
+ tax and dividend payments.

Investment income will also need to be assessed.

The sensitivity of asset and liability cashflows needs to be understood either through using a
stochastic model or a deterministic simulation with lots of sensitivity testing.

The solvency position is affected by the capital value of assets. The projection model may
well extend to monitoring this position over time.

The Actuarial Education Company © IFE: 2019 Examinations


Page 38 SP7-19: Investment principles and asset liability matching

Chapter 19 Summary continued


The supervisory authorities may impose controls on the insurer’s asset holdings via:
 restrictions on the amount of certain types of assets that can be used to
demonstrate statutory solvency
 custodianship of assets
 prescription to hold / prevention from holding certain assets
 a requirement to hold mismatching reserves.

Objectives and constraints


The insurer will aim to maximise return subject to:
 meeting claims and expenses as they fall due
 maintaining statutory solvency and any internal company solvency constraints.

The risk appetite of the insurer will depend upon its:


 liabilities
 assets
 external influences
 insurer-specific constraints.

External influences include:


 tax treatment
 statutory, legal, ethical or voluntary restrictions
 statutory valuation requirements
 solvency requirements
 rating agency constraints
 competition
 regulatory constraints.

© IFE: 2019 Examinations The Actuarial Education Company


SP7-19: Investment principles and asset liability matching Page 39

Chapter 19 Summary continued


In relation to its investment strategy, the general insurer is subject to the following risks:
 liquidity
 currency
 market
 credit
 operational
 relative performance
 group
 contagion.

Asset liability modelling (ALM)


The model projects asset proceeds and liability outgo simultaneously based on a
deterministic or stochastic model.

Free reserves and their proceeds are treated as a balancing item.

Different investment mixes are run through the model to determine asset mixes which give
sufficient proceeds to meet liability outgo at all times. The aim is to determine the optimal
investment strategy in line with the insurer’s investment goals.

The model may include new business, in which case it is more complex.

ALM can also be used to determine solvency capital or in a wider capital management
program.

Stress and scenario testing can also be used to set or test an investment strategy.

The Actuarial Education Company © IFE: 2019 Examinations


Page 40 SP7-19: Investment principles and asset liability matching

The practice questions start on the next page so that you can
keep the chapter summaries together for revision purposes.

© IFE: 2019 Examinations The Actuarial Education Company


SP7-19: Investment principles and asset liability matching Page 41

Chapter 19 Practice Questions


19.1 State the advantage of making some allowance for future new business in the investment strategy
of a general insurer, and explain why it is dangerous to rely heavily on a high level of new
business.

19.2 (i) State the characteristics of an insurer’s liabilities that should be taken into account so that
appropriate assets can be selected.

(ii) List reasons why in practice the assets held by a general insurer may not be a perfect
match for the liabilities.

19.3 Comment briefly on the suitability of government bonds and direct property as investments for
the following:
(i) meeting claims from household contents insurance
(ii) meeting claims from employers’ liability insurance
(iii) free reserves (ie shareholders’ funds).

19.4 (i) Outline the situations in which a supervisory authority is likely to impose restrictions on a
general insurer’s investment holdings and their valuation.

(ii) List examples of the controls the supervisory authority may impose.

19.5 A general insurer writes private motor business only. Discuss the suitability of investing the
company’s assets in government securities denominated by the insurer’s own government.

19.6 You work for a medium-sized general insurer that writes a wide range of business. A director of
the company is concerned about a report they have read that mentions non-investible funds.
Exam style

List the different types of non-investible funds that the insurer may hold and explain how they
affect the financial management of the company. [8]

19.7 (i) State the primary objective of an insurer when investing its assets. [2]
Exam style
(ii) List the external factors that may affect an insurer’s risk appetite. [3]

(iii) Describe the financial risks faced by a general insurer. [10]


[Total 15]

The Actuarial Education Company © IFE: 2019 Examinations


Page 42 SP7-19: Investment principles and asset liability matching

The solutions start on the next page so that you can


separate the questions and solutions.

© IFE: 2019 Examinations The Actuarial Education Company


SP7-19: Investment principles and asset liability matching Page 43

Chapter 19 Solutions
19.1 New premium income can be used to pay short-term liabilities, allowing the existing assets to be
invested longer with the aim of producing higher returns.

It is dangerous to rely heavily on a high level of future new business because the amount of
premium income may be less than anticipated. This could leave the company exposed to the
possibility of having to realise long-term assets to meet short-term liabilities (ie mismatching).

19.2 (i) Characteristics of liabilities

The following characteristics of the insurer’s claims and expenses should be taken into account:
 term
 amount
 nature, fixed or increasing with inflation or super-inflation
 currency.

(ii) Why not perfectly matched

 The company may choose not to match the liabilities (eg if it has plenty of free reserves).
 If the company is making underwriting losses, an unmatched position may be taken to try
to make investment profits to offset these losses.
 The company may be forced to invest in much shorter, liquid assets with a more stable
market value, in order to protect its statutory solvency position.
 The timing of the liabilities will be uncertain and therefore difficult to match.
 It may not be possible to match, eg if claims inflation is different from the inflation
protection available from real assets.
 There may be regulatory limits on the amount of assets that can be invested in certain asset
classes.
 Matching assets may not be available.

19.3 (i) Household contents

Short-term bonds are suitable for household contents.

Longer-term bonds are unsuitable as the liabilities are short tailed.

Direct property would be unsuitable for household business because of lack of marketability and
large unit size. Also investments would be too long-term.

(ii) Employers’ liability

Bonds would not really be appropriate for employers’ liability as real assets would be a better
match.

The Actuarial Education Company © IFE: 2019 Examinations


Page 44 SP7-19: Investment principles and asset liability matching

Property is also probably unsuitable. Although the nature and term may be about right (or less
wrong than other assets), the lack of marketability could be a problem.

The large unit size would also make it difficult to hold a sufficiently diversified portfolio.

(iii) Free reserves

Bonds are not normally appropriate for free reserves. We prefer assets that have a higher
expected return and offer better protection against inflation.

Direct property could be acceptable for a portion of the free reserves. The insurer’s own office
may form part of the assets.

19.4 (i) Situations where restrictions may be imposed

The regulator may deem it appropriate to impose restrictions:


 on the market in general
 on insurers selling particular product lines
 at certain stages of development
 where an insurer is in difficulty, having breached or being in danger of breaching
regulations.

(ii) Examples of controls

 restrictions on the amount of certain types of assets that can be taken into account when
assessing solvency
 custodianship of assets
 prevention from holding certain assets
 a prescription to hold certain assets
 a requirement to hold mismatching reserves
 limitations on the ability to mismatch

19.5 Positive factors:


 limited risk of default (but depends upon the government)
 highly marketable, and therefore useful if there is fluctuation in claim outgo
 short-dated securities have relatively stable market values
 right currency for matching local liabilities
 short-dated securities have the right term for matching property damage claims
 fixed-interest stocks are appropriate for property damage claims (to the extent that short-
term inflation is largely predictable)
 index-linked stocks provide inflation protection for the longer-tail, liability classes
 low dealing costs and may tax efficiency.

© IFE: 2019 Examinations The Actuarial Education Company


SP7-19: Investment principles and asset liability matching Page 45

Negative factors:
 they will not provide the best expected return
 funds backing the free reserves may be invested in more aggressive assets
 long-dated stocks can have volatile market values (not so good if statutory asset valuation
is based upon market values)
 long-dated, fixed-interest stocks provide no inflation protection and do not match
short-term liabilities
 the inflation within index-linked stocks may not provide the right type of inflation
protection.

Other:
 the extent to which government stocks are used (and the extent of attempted matching)
will depend on the level of free reserves relative to any required solvency margin.

19.6 Non-investible funds are monies held:


 by reinsurers (ie who are due to pay a reinsurance claim) …
 … this figure may be net of any bad debt provision
 by agents and brokers who owe premiums to the company
 by policyholders who are paying premiums by instalments
 by policyholders who pay end of year adjustments to premiums, because of exposure
adjustments or experience rating
 by policyholders who are slow to pay premiums. [½ each, maximum 3]

Effect on financial management

Non-investible funds will not earn any investment income. Therefore, if the insurer has a large
proportion of non-investible funds, profits will be reduced or higher premiums need to be
charged. [1]

The company may try and counter this effect by investing a higher proportion of the remaining
funds in assets with a higher expected return. [½]

As non-investible funds are short-term assets then a smaller proportion of the remaining portfolio
needs to be invested short. [1]

However, if the company is concerned about the default risk of the non-investible funds then it
may look for greater security from the investible assets. [1]

Alternatively it may set aside provisions for bad debts. [½]

There may be a regulatory limit that restricts the level of non-investible funds that can be
included when demonstrating solvency. The company should ensure that solvency is not
jeopardised. [1]
[Total 8]

The Actuarial Education Company © IFE: 2019 Examinations


Page 46 SP7-19: Investment principles and asset liability matching

19.7 (i) Primary investment objective

The primary objective regarding the investment of the assets supporting the liabilities of a general
insurer is:
 to maximise the return subject to … [1]
 … the overriding constraint of meeting all contractual obligations as they fall due, whilst
ensuring that … [½]
 the risk of not receiving the return is within the company’s tolerance. [½]
[Total 2]

(ii) External influences on risk appetite

 tax treatment of various assets and the tax position of the general insurer [½]
 statutory, legal, ethical or voluntary restrictions on how the insurer may invest [½]
 statutory valuation requirements [½]
 solvency requirements and the size of the insurer’s free reserves relative to these [½]
 capital required to maintain the insurer’s desired credit rating [½]
 competition, ie the investment strategy followed by other funds [½]
 regulatory constraints, eg imposed by Lloyd’s or the PRA [½]
[Maximum 3]

(iii) Financial risks faced by the general insurer

Liquidity risk [½]

The risk that insufficient funds can be realised in time to meet claims. [½]

Since the general insurer’s liabilities are very uncertain in both amount and timing, a reasonable
level of liquid assets will be required at all times. [½]

Currency risk [½]

The risk of adverse currency movements when the assets and liabilities are not perfectly matched
by currency. [½]

Market risk [½]

The risk of changes in the market value of assets held due to market movements. [½]

However the general insurer will mainly have short-term liabilities backed by short-term assets
(such as cash or short-term bonds) which are not particularly volatile, reducing market risk. [1]

Credit risk [½]

The risk that a third party does not fulfil their obligations. [½]

The most significant example of this would be the failure of the company’s reinsurer(s). [½]

© IFE: 2019 Examinations The Actuarial Education Company


SP7-19: Investment principles and asset liability matching Page 47

Credit risk is correlated with insurance risk, ie following a catastrophe loss in the market insurers
may also face increased credit risk through default of their reinsurers. [½]

Operational risk [½]

The risk of loss due to fraud or mismanagement within the insurer itself. [½]

This could arise in large multi-national companies where it may be more difficult to control all
areas of the business closely. [½]

Relative performance risk [½]

The risk of underperforming relative to comparable institutional investors. [½]

This is more of a risk for general insurers than for life insurers, since it is often one of the main
performance measures. [½]

Group risk [½]

The risk that an insurance subsidiary needs to change its investment strategy due to the
requirements of its parent company. [½]

Contagion risk [½]

The risk that the collapse of one insurer brings down others. [½]
[Maximum 10]

The Actuarial Education Company © IFE: 2019 Examinations


Page 48 SP7-19: Investment principles and asset liability matching

End of Part 4

What next?
1. Briefly review the key areas of Part 4 and/or re-read the summaries at the end of
Chapters 16 to 19.
2. Ensure you have attempted some of the Practice Questions at the end of each chapter in
Part 4. If you don’t have time to do them all, you could save the remainder for use as part
of your revision.
3. Attempt Assignment X4.

Time to consider …
… ‘revision and rehearsal’ products
Revision Notes – Each booklet covers one main theme of the course and includes integrated
questions testing Core Reading, relevant past exam questions and other useful revision aids.
Students have said:

‘Revision books are the most useful ActEd resource.’

‘I found the revision booklets very helpful as they grouped all past exam questions
by topic and the answers were bullet pointed into key points so it was easy to
check how many of the points you get.’

You can find lots more information, including samples, on our website at www.ActEd.co.uk.

Buy online at www.ActEd.co.uk/estore

© IFE: 2019 Examinations The Actuarial Education Company


SP7-20: Capital modelling – methodologies Page 1

Capital modelling –
methodologies
Syllabus objectives

4.1 Evaluate the key considerations in deriving and applying capital modelling
techniques.

4.2 Evaluate the following approaches to capital modelling:


 deterministic models
 stochastic models.

4.3 Discuss the following issues with regard to parameterisation of capital models:
 developing assumptions
 validation.

The Actuarial Education Company © IFE: 2019 Examinations


Page 2 SP7-20: Capital modelling – methodologies

0 Introduction
In this chapter we look at approaches that can be used in modelling the capital that is required by
a general insurance company.

The aim of a capital model is to predict the future of the general insurer under different
scenarios. This should enable a firm to better understand their risks and help them to run
their business.

Section 1 sets out important components of a capital model.

Section 2 discusses the differences between a stochastic and a deterministic model, as well as the
steps involved in running these different types of model.

Section 3 examines the characteristics of a good model.

Section 4 considers the overall modelling process, including the importance of validating results
and using stress / scenario tests.

Section 5 considers the assumptions that will be needed and the data required in order to set
them.

Many assumptions will have to be made as part of this modelling process.

In Section 6, we consider aggregation methodologies.

© IFE: 2019 Examinations The Actuarial Education Company


SP7-20: Capital modelling – methodologies Page 3

1 Components of a capital model


This section considers the key cashflows that are incorporated into a typical capital model, and
the factors to consider when projecting these cashflows.

1.1 Future written premium income


Premium income is usually projected separately for each line of business (subject to the
size of each line: grouping of lines of business may be required). The figures may also be
split by source of business.

Question

Explain why we might split premium income by source of business.

Solution

The premium income from each source of business should be considered separately in order to
allow correctly for delays and acquisition costs.

Such assumptions will usually be set following consultation with sales managers,
underwriters and senior management.

Earned premium could also be used depending on the capital model structure /
requirement.

Where new business and renewals are not included, allowance for outstanding premiums should
be relatively easy because most of the cashflows are likely to emerge within the next few months.
However, when including new business and renewals, allowance will need to be made for
expected rates of premium growth (and profitability) in the light of the company’s business plan,
the competitive position and the effect of the insurance cycle.

1.2 Future claims


For each class of business it is possible to split claims into:

 attritional / large / catastrophe claims

 the development of notified claims (IBNER)

 IBNR

 claims from the period of unexpired risk on existing business

 claims from business that is written in the future.

Outstanding claims can be estimated using projection methods (eg the chain ladder method).
These methods can also be used as the basis for estimating the claim outgo in future periods.

As an example of how the elements of insurance risk might be treated, potential claims
arising from catastrophes are usually analysed using stress tests or by using the output
from proprietary catastrophe modelling software.

The Actuarial Education Company © IFE: 2019 Examinations


Page 4 SP7-20: Capital modelling – methodologies

Stress tests are discussed in Section 4. Catastrophe modelling software is discussed in detail in
Subject SP8 and in Subject SA3.

1.3 Future expenses


Assumptions about some expenses are likely to be easy to determine as they are simply a
function of other assumptions in the business plan. In the case of commission it is
normally a percentage of the written premium.

The expenses relating to handling the claims can be allowed for either explicitly or implicitly
(ie with the corresponding claims). The analysis of expenses is covered in detail in Subject SP8. It
may also be familiar to you from earlier subjects.

Other expenses such as staff costs or rental will be more difficult to predict as they will
depend on the projected business growth and staffing plan.

If the insurer stopped writing any more business, the expenses would just be in respect of the
existing business.

Question

The allowance for the cost of running the business once an office has closed should be much
more than the normal allowance for claims handling costs. Suggest why this might be the case.
(After all, both approaches relate to the cost of settling the existing business…)

Solution

As the business runs off, fewer claims will be settled (incurring lower claims handling expenses),
so the firm’s fixed expenses will become a larger and larger proportion of overall expenses.

Usually, a firm would use new premium income to meet its fixed expenses, but in the case of a
run-off business, extra capital must be set aside to finance this.

1.4 Ceded reinsurance


Assumptions about ceded reinsurance will need to take into account:

 any existing arrangements

 any changes that could be made to those arrangements

 possible new arrangements that could be put in place, particularly where there are
forecast changes in underlying direct exposures

 any expected softening or hardening of future reinsurance costs.

Allowance should be made for recovery delays and defaults.

Alternatively, claims could be projected net of recoveries, with a margin to allow for defaults and
an adjustment to the development profiles to allow for recovery delays.

© IFE: 2019 Examinations The Actuarial Education Company


SP7-20: Capital modelling – methodologies Page 5

1.5 Investment return


Assumptions about future investment return will depend on:

 the current investment portfolio held

 investment prospects and expectations around the future economic environment

 the current and projected future investment policy

 expectations on premium and claims payment patterns, which impact the run-off of
reserves and investment assets.

For all types of investment, it is important to make allowance for:


 the expenses of investment
 the future volatility of capital values and investment income.

The method used for estimating income will vary depending upon the asset category being
considered. For example, ignoring the possibility of sale and the risk of default, the stream of
income and redemption proceeds from fixed interest securities is of known amount and term.

The income stream from an equity portfolio might be based on the current dividends but
assumed to increase from year to year to reflect the path of overall dividend increases. This
assumption must be made to be consistent with other assumptions, eg expense inflation and
claim inflation.

Question

Suggest under what circumstances it would be acceptable to treat all the equity holdings as
providing an infinite stream of dividends.

Solution

Treating equities purely as an income stream assumes that none of the equity portfolio is ever
realised. This might be the case if the value of the portfolio was less than the free reserves
(ie equities are backing the free reserves not the technical reserves), and the liability outgo would
be entirely covered either by income from assets (including equities) or the redemption proceeds
from other assets (eg index linked securities). This means that the equities would never need to
be realised to meet future liability outgo.

1.6 Allowing for the environment

The economic environment


A capital model will need to make assumptions about future inflation and future interest
rates.

These assumptions should be consistent with each other.

The Actuarial Education Company © IFE: 2019 Examinations


Page 6 SP7-20: Capital modelling – methodologies

Allowance may also be made for other features of the economy, for example, the increased
moral hazard associated with increased claim frequencies during times of recession.

The insurance cycle


The model should also take account of the insurance cycle, as should any business plan
underlying the model. In particular, it will need to allow for the fact that different classes of
business may be at different stages in the cycle.

Operating environment
The model should also take account of what is happening internally within the company and its
potential influence on future cashflows. For example the potential impact of high staff turnover
on the ability to meet regulatory deadlines or the loss of an underwriting team on the ability to
meet a business plan.

There should also be some consideration of potential changes in legislation and their impact, for
example the potential impact of a change in the Ogden discount rate on future claims payments.

1.7 Risk measure


A capital model requires a defined risk measure, on which it will be calibrated. This includes the
type and confidence, for example, a 99.5% VaR.

Question

Explain what is meant by a 99.5% VaR.

Solution

Value at Risk (VaR) generalises the likelihood of underperforming by providing a statistical


measure of downside risk. VaR assesses the potential losses on a portfolio over a given future
time period with a given confidence level.

Consider, for example, a VaR of £10m over the next year with a 99.5% confidence interval. This
means that there is only a 0.5% expected probability of the underperformance (relative to a
benchmark) being greater than £10m over the next year.

© IFE: 2019 Examinations The Actuarial Education Company


SP7-20: Capital modelling – methodologies Page 7

2 Deterministic and stochastic models


A capital model may be deterministic or stochastic.

Question

Without reading on, describe what is meant by:


 a deterministic model
 a stochastic model.

Solution

A deterministic model is one where the parameter values are fixed at the outset of running the
model, and the result of running the model is a single outcome. (Sensitivity analysis can then be
carried out to assess the potential variability of the results.)

A stochastic model estimates at least one of the parameters by assigning it a probability


distribution. The model is run many times with the values of stochastic parameters being
selected from their distributions on each run. The outcome is a range of values, which gives an
understanding of the likely distribution of outcomes.

2.1 Deterministic models


A deterministic approach is one in which we assign fixed values to the variables or
parameters (interest rate, inflation rates, claims rates and so on). Under a deterministic
approach, we produce a single run for each set of fixed values. We perform a number of
different runs, using stress and scenario tests. The standard formula under the Solvency II
regime is an example of a deterministic model.

Question

Explain what the main difference is between stress and scenario tests.

Solution

In a stress test, a single parameter is varied. Stress tests therefore analyse the impact of
individual risks in isolation. In a scenario test, a combination of parameters is varied. Scenario
tests therefore analyse the combined impact of a number of risks.

Stress and scenario tests are discussed in more detail in Section 4.

2.2 Stochastic models


A stochastic model is one in which we assume some of the variables in the business plan
have a probability distribution. This enables us to describe critical assumptions, and their
financial implications, in terms of ranges of possible outcomes, rather than in terms of fixed
values and results.

The Actuarial Education Company © IFE: 2019 Examinations


Page 8 SP7-20: Capital modelling – methodologies

A stochastic model can be very complex and its results difficult to interpret. It is worth
remembering that the output from a stochastic model is only as useful as the underlying
data input allows. As such, we should start the model process by gaining a thorough
knowledge of the underlying data. Similar data issues apply equally to deterministic
processes.

We now consider the steps involved in running a model, (both deterministic and stochastic), in
more detail. These may be familiar to you from studying earlier subjects.

2.3 Steps in running a deterministic model


A deterministic model involves the following steps:
 specify the purpose of the investigation
 set the risk measure, eg Value at Risk (VaR)
The Value at Risk (VaR) is the loss at a predefined confidence level (eg 99.5%),
specified over a particular time horizon. Consequently, if an insurer holds capital
equal to the VaR, it will remain solvent over a particular time horizon with a
probability of the confidence level (eg 99.5%) and be insolvent with probability of
one minus the confidence level (eg 0.5%).

The use of probabilities and confidence levels in the risk measure seems to imply that we
need to use a stochastic model. However, VaR can be used as a risk measure for
deterministic models too, but the decision as to what constitutes a 99.5% probability will
be very subjective.
 collect data

 group and modify data


By data, we mean information relating to the policies (ie the mix of business and claims
history) being modelled.
It may be too time consuming to run each individual policyholder through the model.
Therefore it is common to group together policyholders with similar characteristics into
‘model points’.
 choose the form of the model, identifying its parameters and variables
Parameters are any factors which would affect the decisions we make as a result of
running the model. For example, if investment returns influence the premium charged
then investment return is a parameter.
Variables are the factors we are trying to test when running the model. For example, if
we are setting the premiums for a product, then the premium is the variable.
 ascribe values to the parameters using past experience and appropriate estimation
techniques, taking into account the risk measure being used and any correlations
between parameters
The value assigned to any parameter is usually referred to as the assumption for that
parameter. The full set of assumptions is referred to as the basis of our model.
 construct a model based on the expected cashflows

© IFE: 2019 Examinations The Actuarial Education Company


SP7-20: Capital modelling – methodologies Page 9

 check that the goodness of fit is acceptable and, if not, attempt to fit a different
model
This can be done by running a past year and comparing the model with the actual results.
 run the model using the selected variables

 run the model using different parameters to check sensitivity.


The model may also be run under different scenarios, ie testing the robustness of the
results to many parameters changing at the same time, rather than changing single
parameters in isolation.

2.4 Steps in building and running a stochastic model


The process of running a stochastic model has a lot of similarities with running a deterministic
model. The stages in building and running a stochastic model will include the following:
 specify the purpose of the investigation

 select an appropriate model structure (which business areas to include)

 set the risk measure, eg VaR


 determine the types of scenarios to develop and model (for example, interest rate
environment, competitive environment)

 decide which variables (for example, claims costs, premium growth) to include, and
their interrelationships

 collect data

 group and modify data

 estimate the parameters that should be used for each variable (that is, the
mathematics that specifies the behaviour of each variable)
– choose a suitable density function for each of the variables to be modelled
stochastically
– estimate the required parameters for the chosen density function(s)
– ascribe values to the variables that are not being modelled stochastically

 specify dependencies between variables

 test and validate the reasonableness of the assumptions and their interactions (for
example, the projection of future claim payments versus historical levels of claim
payments)

 check the goodness of fit is acceptable and attempt a fit with different density
function(s) if it is not. (This can be hard to implement. In practice many different fits
are equally acceptable.)

 construct a model based on the chosen density function(s)

 run the model many times, each time using a random sample from the chosen
density function(s)

 produce a summary of results that shows the distribution of the modelled results
after many simulations have been run

The Actuarial Education Company © IFE: 2019 Examinations


Page 10 SP7-20: Capital modelling – methodologies

 run the model using different distributions / parameters to check sensitivity

 continually update any model to remain relevant in the ever-changing environment


in which insurers operate; the model must continue to reflect reality.

Choosing a suitable density function


One way to choose a suitable distribution for claim severity would be to plot each of the observed
claims in a bar chart (or equivalent), by size of claim:

number of
observations

size of claim

Now convert the left-hand scale so that the total area under the curve is 1, ie by dividing through
by the total number of observed claims.

Then select a function, y = f(x) which has a similar shape to our plotted data. This is the
probability density function (PDF).

The following loss distributions are often used:


 Frequency: Poisson, negative binomial
 Severity: log-normal, Weibull, Pareto.

We would then select a method of fitting to find parameter values for our chosen distribution.
We may use different methods (eg method of moments or method of maximum likelihood) and
then select the one that gives the best fit to our data.

The method and parameters that have been fitted would be scrutinised using a number of
statistical tests to determine how well the observed claims fit the modelled claims.

Particular attention must be paid to the ‘important’ part of the distribution. For example, some
classes of insurance have very skew claim amount distributions. Care should be taken that the
fitted distribution has a sufficiently long tail. In these cases, a distribution such as the Pareto
which has a relatively long, thick tail should be used. With excess of loss reinsurance care should
be taken to ensure there is a good fit close to the excess point.

If the goodness of fit of the model is not adequate, the distribution or the parameters should be
altered until the fit is good enough.

© IFE: 2019 Examinations The Actuarial Education Company


SP7-20: Capital modelling – methodologies Page 11

2.5 Deterministic versus stochastic – the pros and cons


A stochastic model, all other things being equal, requires more calculations than a single
deterministic model. Therefore, given that stochastic models are used, they must have
reasons for being preferred to a deterministic approach.

Advantages of stochastic models


There are some important advantages of using a stochastic approach:

 Stochastic models test a wider range of scenarios. A scenario analysis can only test a
limited set of scenarios. A stochastic model should obviate this problem since it produces
a full distribution of possible scenarios.
By exploring combinations of randomly-generated values for the key parameters, we
can explore scenarios that we might otherwise not have imagined to be important.
For example, two variables in the model may have a strong interaction that is not
obvious, and deterministic stresses might not have considered them both together.

 Similarly, a stochastic model makes it easier to explore ‘ripple effects’, that is, the
knock-on consequences of the crystallisation of a risk event.
The actuary needs to decide whether the increased amount of information a stochastic
valuation will provide justifies the significant additional computations needed. Other
important considerations are the degree of spurious accuracy that may be introduced, the
increased difficulty in interpreting and communicating the results, and the questionable
accuracy of the distribution functions that are replacing the deterministic values.
 By estimating the full distribution of the aggregate result from the model, we can
apply a number of different risk measures to the same set of final output. This may
be important in sharing results with stakeholders who have different risk appetites
or concerns.

 Even if we have calibrated individual stresses to represent a certain degree of


extremity in their individual values, it is very difficult to combine those stresses and
be sure that the final result represents a sufficiently extreme combination of
circumstances. A stochastic approach – in theory at least – explores all possible
combinations and can rank these against the chosen risk measure.

 We can derive a probability distribution from the outcomes of a stochastic model


and calculate confidence levels, if required. While the assumptions are subject to
parameter error, the model is at least explicit about the assumptions being made.
We can also test the assumptions by different techniques.

 Stochastic models are particularly important in assessing the financial impact of financial
guarantees. This is because they are good at allowing for the uncertainty involved.

The Actuarial Education Company © IFE: 2019 Examinations


Page 12 SP7-20: Capital modelling – methodologies

Question

Give an example of a financial guarantee that might arise in general insurance.

Solution

A motor policy, which promises to refund the first year’s premium if the policyholder makes no
claims in a five-year period.

These benefits do not necessarily outweigh the advantages of deterministic stress tests and
scenario tests in all circumstances.

Advantages of deterministic models


There are a number of benefits of a deterministic approach:

 The model is usually easier to design and quicker to run.


 It is important to consider potential cause and effect relationships between risks.
We may model such relationships better using deterministic relationships rather
than relying on statistical dependence structures.

 It is more straightforward and, therefore, quicker to build a deterministic model than


a stochastic one.

 By reducing the computational power necessary to generate many thousands of


simulations, we can introduce more detail in other dimensions, such as detailed
descriptions of reinsurance programmes or treatment of underlying risks. This may
aid the intelligent selection of a limited number of scenarios. It could be more
efficient than a stochastic model where we hope that the important scenarios appear
amongst a larger number of randomly generated outcomes.

 We can integrate the capital model more closely with risk management, by extending
the scenario modelling to scenario planning and ‘what-if’ analysis.

 It is clearer what economic scenarios have been tested. As discussed above, the
disadvantage of this point is that it requires thought as to the range of economic
scenarios that should be tested. Since only a limited number of economic scenarios will
be tested, there is a danger that certain scenarios, which could be particularly detrimental
to the company, are not identified.
 We commonly use stress and scenario tests for those risks that cannot easily be
modelled quantitatively and where more subjective judgment is required. This
allows us to concentrate more on the important areas of the distribution of
outcomes for the key risks when a full specification of the distributions is subject to
substantial potential error.

 By developing deterministic stresses and scenarios, we can help to link the capital
model with the risk register, helping to integrate capital and risk management (this
would also apply in a stochastic environment by considering each individual
simulation as a scenario).

 It can be easier to communicate the results of stress and scenario tests to senior
management, and to give them comfort as to the reasonableness of the overall
capital value.

© IFE: 2019 Examinations The Actuarial Education Company


SP7-20: Capital modelling – methodologies Page 13

A deterministic model is more readily explicable to a non-technical audience (eg users of


results of the model and senior management), since the concept of variables as
probability distributions is not easy to understand.
 It is important that users of the output understand the results from the model. By
showing the effect of a limited range of stresses and scenarios – some of which may
have been developed in consultation with those users – we can often make the
results more comprehensible to them.

 Deterministic models are good for checking / validating results of a stochastic


model.

 Deterministic model stress tests can be used in conjunction with the results
generated from a stochastic model. This provides additional context to the
stochastic results as well as providing either independent validation or appropriate
challenge.

2.6 Combining deterministic and stochastic approaches


In many cases a problem can be solved by a combination of stochastic and deterministic
modelling. Variables whose performance is unknown and where the risk associated with them is
high might be modelled stochastically, while other variables can sensibly be modelled
deterministically.

For these reasons, the stochastic approach is often limited to the economic assumptions, with
demographic assumptions being modelled deterministically.

It may be appropriate to use a blend of approaches:

 stochastic models for some risk categories

 stress and scenario tests for other risk categories

 ad-hoc methods for yet other categories.

In fact, there are various ways in which stochastic and deterministic approaches can be combined
in a single model. For example, when modelling claim frequency and average claim size
separately, we could:
 Determine the number of claims stochastically and associate this with a deterministic
mean claim cost. Ideally the claim numbers would be divided into various homogeneous
groups in terms of claim size.
 Determine the claim amounts stochastically for the (deterministically chosen) expected
number of claims.
 Determine both claim amounts and numbers stochastically, using a collective risk model.

The Actuarial Education Company © IFE: 2019 Examinations


Page 14 SP7-20: Capital modelling – methodologies

3 Model requirements

3.1 Features of a good model


A model should ideally meet the following requirements:

 The model being used should be valid, complete and adequately documented.
By valid we mean, for example, that we should not use a stochastic investment model
which has been developed for projecting asset proceeds over periods of 30 or more years,
if we are only interested in cashflows over the next 5 years.
A complete model is one that produces realistic (and hence useful) results under a wide
range of circumstances and conditions.
The model must always be adequately documented, so that the key assumptions and
approximations made are understood and so that it can be run by other members of staff
and improvements introduced over time.
 The model chosen should reflect adequately the risk profile of the classes of
business being modelled.

 The parameter values used should be appropriate for the classes of business, and
investments being modelled.
For example, when modelling a general insurer’s short-term claims we should not use a
discount rate based on a portfolio of equities if the assets backing these claims are
short-term deposits.
 The outputs from the model and the degree of uncertainty surrounding them should
be capable of independent verification for reasonableness and should be readily
communicable to those to whom advice will be given.

 The model should not be overly complex so that the results become difficult to
interpret and communicate or the model becomes too long or expensive to run.
Therefore we might not attempt to model every small detail of a scenario if reasonable
approximations can be used instead.
 The model should be sufficiently flexible.
The model should be capable of development and refinement – nothing complex can be
successfully designed and built in a single attempt.
In addition, a range of methods of implementation should be available to facilitate
testing, parameterisation and focus of results.
 The model should have all parameters clearly identified and justified.

 The model should be structured and documented so that it can be understood by


senior management and board members who do not have actuarial expertise.

You may well be familiar with these concepts from studying previous subjects.

3.2 Additional features of a good stochastic model


A good stochastic model should, in addition to all the ‘good model’ points described above:

© IFE: 2019 Examinations The Actuarial Education Company


SP7-20: Capital modelling – methodologies Page 15

 be rigorous (ie strictly applying to constraints or principles) and self-consistent

 be sufficiently detailed to deal adequately with the key risk areas and capture
homogeneous classes of business, but not excessively complex

 be capable of being run with changed parameters for sensitivity testing

 use sufficient simulations to reduce the simulation error that could arise from the
model  this is likely to be at least 50,000

 have a robust software platform.

The Actuarial Education Company © IFE: 2019 Examinations


Page 16 SP7-20: Capital modelling – methodologies

4 General modelling considerations

4.1 Parties involved


A model requires input from many sources, many of which are non-actuarial, eg finance,
underwriting etc. All parties involved should have an understanding of the complete
process.

4.2 Objectives
Three key objectives of any capital requirement regime are to ensure that:

 there is a link between risk and capital setting – in making an assessment of capital
adequacy, a firm should:
– identify the significant risks facing the business
– assess their impact (both prior to and post having controls in place)
– quantify how much capital is required to support the risks

 senior management focus on risk management – a risk management framework is


central to this process

 the capital model is being used within the decision-making process – this should be
demonstrated through clear documentation of all prudential risks, processes and
controls.

Overarching objectives of modelling encompass solvency and policyholder protection.

4.3 Approaches
There are two broad approaches available to firms when producing a capital model, namely:

 stress and scenario tests


Stress tests consider different factors in isolation. Scenario tests consider several factors
at a time. This approach is a deterministic approach, where the user chooses which
stresses and scenarios to test. Stress and scenario tests are discussed in more detail
below.
 economic capital models (also known as stochastic models or dynamic financial
analysis (DFA) models).
An economic capital model is a more integrated, holistic approach. It systematically
models the effects of many interrelated risk factors using simulation techniques.

Although these are significantly different in application, they are not in principle different,
as a stochastic model is based on stress and scenarios weighted by probabilities. In a DFA
model, stress tests are generated automatically and often cannot be ‘seen’. These have
been discussed in more detail in Section 2.

4.4 Uncertainty
There are many potential sources of uncertainty in the parameters used to assess the
capital required, but the most common is lack of credible relevant data on which to base the
main assumptions.

© IFE: 2019 Examinations The Actuarial Education Company


SP7-20: Capital modelling – methodologies Page 17

Question

(i) List the main assumptions that will be needed.

(ii) Suggest other possible sources of error.

Solution

(i) Assumptions

 premiums in respect of business already written, and assumed premium income for
future business
 claim frequencies
 claim amounts
 claims inflation
 claim payment profiles
 expenses (including fixed and variable expenses, and in respect of new and existing
business)
 investment returns
 lapse rates
 business volumes and mix
 reinsurance costs, recoveries and payment profiles

(ii) Sources of error

 using an inappropriate model


 assuming distributions that are not appropriate to the business being modelled
 using the wrong parameters
 not correctly allowing for interdependencies between variables
 misinterpreting the output
 not checking the output for reasonableness
 placing too much reliance on the output

The aim is for the data to represent a best estimate of current claims experience. So it is
necessary to try to:
 group the data into broadly homogeneous groups
 make sure that the estimates for outstanding claims are best estimates, rather than
deliberately cautious

The Actuarial Education Company © IFE: 2019 Examinations


Page 18 SP7-20: Capital modelling – methodologies

 consider whether the emergence of any missing IBNR claims would change the shape of
the distribution of claims
 convert into constant money terms and try to identify the sources of inflation if data is
from more than one year
 adjust for trends in claim frequency, changes in policy conditions, large claims,
catastrophes etc.

We should ensure that sufficient data has been used to assess risks. Where appropriate,
we supplement the firm’s data by using external data.

The data needed for capital modelling are discussed in Section 5.

4.5 Validation and testing

Stress testing
Stress testing quantifies the effect of varying a single parameter, and is useful for
understanding the potential impact of individual risks in isolation.

Stress tests are a necessary tool in assisting an insurer to manage its risks and maintain
adequate financial resources to deal with the risks to which it is exposed. We can use
stress tests to identify and quantify the impact of different stress scenarios on an insurer’s
expected financial position. Stress tests may be deterministic but we often develop them
with probability distributions in mind.

The stress tests should be calibrated to the appropriate risk measure (eg 99.5% VaR). We
expect that prudent, well-managed insurers would undertake stress testing as a matter of
good corporate governance, even if they also use more sophisticated models.

Question

List three reasons why stress tests may be needed when using a stochastic model.

Solution

Stress tests are needed:


 to validate the model output for reasonableness
 to help with calibrating assumptions
 to test the impact on the results of uncertain assumptions.

Scenario testing
The next step, following simple stress testing, is scenario testing. This quantifies the effect
of a change in a combination of parameters. It is useful for considering the combined effect
of a number of risks and the cumulative effect of several different mitigating actions
occurring at the same time.

© IFE: 2019 Examinations The Actuarial Education Company


SP7-20: Capital modelling – methodologies Page 19

Scenario testing should look at the impact of unlikely, but not impossible, adverse
scenarios.

Actuaries should carefully consider which risks the firm is exposed to when designing the
scenarios to be used as part of a capital assessment. They must also consider the
relationships between different types of risks. They may actually model such relationships
better using deterministic models rather than stochastic ones.

We can also use stress and scenario testing to test the output from a stochastic model.

Sensitivity testing
Sensitivity testing is the process of testing the extent to which the results of a capital model
change as a result of making a small change to an assumption in the model.

As a minimum standard and part of the validation and sign off process, all capital models
must be subject to some level of sensitivity analysis.

Purpose of sensitivity testing

The purpose of sensitivity testing is to identify the more sensitive assumptions in the
capital model; that is, which assumptions, if changed, would have the greater impact on the
results of the capital model.

We can then pay attention to the more sensitive assumptions. We can pay greater attention
to justifying and documenting the rationale for selecting values of these assumptions,
because they are more critical to the results of the capital model.

So stress testing and scenario testing are used to gain an insight into the uncertainty of our
results. Sensitivity testing is used to gain an insight into which parameters have the greatest
effect on the model result.

However, care should be taken where changes in assumptions have a smaller impact than
expected due to the design of the model. It may be inappropriate to conclude that the
assumption is less critical and may suggest that a re-evaluation of the model design is
needed.

Applying judgement

When selecting assumptions, we must often make subjective judgements. (It is rare that we
can choose assumptions on the basis of data analysis alone.) There might be several
possible selections for an assumption, each appearing to be equally reasonable, yet we
must decide which to select. In such a situation, it is informative to carry out a sensitivity
test to find out the change to the capital model result from making a different (but equally
reasonable) selection for the assumption under consideration.

Extent of sensitivity testing

Sensitivity testing should be as comprehensive as possible. However, in complex models,


particularly where there are a lot of classes of business being modelled, it may not be
practical to sensitivity test each and every assumption. For example, it may not be practical
to sensitivity test the assumed loss ratio standard deviation for each class individually. In
this case, we could test the sensitivity of the capital model results to changing the loss ratio
standard deviation of all classes by a fixed multiple.

The Actuarial Education Company © IFE: 2019 Examinations


Page 20 SP7-20: Capital modelling – methodologies

When it is not practical to sensitivity test an assumption for each class individually, we
could sensitivity test the assumption for the two or three largest classes.

Communicating the results of sensitivity analyses

The results of sensitivity analyses should be communicated to the senior management to


ensure that they understand the uncertainty associated with the setting of parameters.

4.6 Back testing


Back testing is the process of comparing actual experience with model output. It tests how
well the model predicts the range of outcomes that actually occur.

Purpose of back testing

Back testing is essential in order to ensure that the model is a sufficiently accurate
reflection of the real world.

As discussed above, one of the key requirements for a good model is that it should
adequately reflect the risk profile of the classes of business being modelled, and back
testing is a key tool in the assessment as to whether this requirement is being met.

It may indicate shortcomings in the model that are not detected by other tests.

In the development of a model, many assumptions are based on an analysis of historical


data. There is therefore a presumption that past performance is a good indicator of future
performance. Back testing can be used to assess the validity of this assumption.

Analysing the results of back testing

Any significant deviations between actual and predicted values that are identified by back
testing will need to be analysed in order to understand the reason or reasons behind them.

An assessment will need to be made as to whether the deviation is simply a consequence of


the expected random variation, a more systematic effect such as a permanent change in the
environment, or an erroneous assumption or parameter.

If the deviation is deemed to be a systematic change, then changes may be made to the model or
parameters.

4.7 Documentation (audit trail)


There should be a clear audit trail from the impact of any financial calculations to the
relevant risk capital allocation in the capital model, whatever modelling approach is
adopted.

We should justify and document the rationale for selecting assumptions, especially the
more critical assumptions. We should also document the known limitations associated with
the selected assumptions, and any testing that has been carried out to assess the
materiality of the assumptions. Documentation should also justify the methodology
selected and mention the alternatives considered and why they were rejected.

In addition, where we have considered particular risk issues, we should document the
issues considered, how we considered them and the reasons behind the conclusions and
findings.

© IFE: 2019 Examinations The Actuarial Education Company


SP7-20: Capital modelling – methodologies Page 21

5 Data and assumptions

5.1 Terminology used in this section


In this section the phrase ‘unpaid claims’ means reported outstanding claims plus IBNER claims
plus IBNR claims.

This section does not cover correlation assumptions as this is discussed in Section 6 on
aggregation methodologies.

5.2 Data

Uses of data
Data is needed in a capital model:

 to create the model of the business as at the run date of the capital model

 as inputs to selecting assumptions used to simulate the firm’s results and capital
over the period covered by the capital model (discussed in Section 5.3).

Types of data
Data will be needed in order to predict all cashflows in terms of both timing and amount. These
cashflows include:
 premiums
 claims – this data will often need to be split into unexpired risks, IBNR etc
 reinsurance premiums and recoveries
 expenses
 investment income.

Data will also be needed on risks, as these may affect the likelihood of cashflows occurring, or
lead to additional cashflows.

Information will be needed on both existing business and future business.

The Actuarial Education Company © IFE: 2019 Examinations


Page 22 SP7-20: Capital modelling – methodologies

The main items of data needed to create the model of the business are:

 Gross and net of reinsurance unexpired premiums at the date of the capital model,
by class of business.

 Gross and net of reinsurance premium planned to be written over the new business
period to be covered by the model, by class of business.
(A theoretically preferable alternative to gross written premium is planned written
exposures and gross premium per unit of exposure. However, it may not be
practical for the business to have this level of data available.)

 Gross unpaid claims at the date of the capital model, by class of business.

 Claims payment profiles (that is, sizes, frequencies and settlement patterns).

 Details of large individual claims and whether they are likely to reach limits, and the
associated limits.

 Aggregate exposure by location to assist in catastrophe modelling.

 The costs of future reinsurance.

 The reinsurance programmes to which gross unpaid claims are subject, each
reinsurer’s participation on the programme, and the extent to which claims paid
have used up coverage available on these programmes. (In practice some of this
data may not be available – see next bullet point below.)

 The total reinsurers’ share of gross unpaid claims with, to the extent that it can be
ascertained, each reinsurer’s share of the total.
(This is a theoretically ‘second best’ alternative to using the actual reinsurance
programmes to which gross unpaid claims are subject. However, there may be
situations where this is the only option, or only practical option, available. For
example, records of reinsurance programmes covering very old years may have
been lost. Also it may be that even if the actual reinsurance programmes are fully
available, using them in the model may make the model overly complex or cause it
to have overly long computer run times.)

 The reinsurance programme to which claims arising from unexpired business is


subject. If the actual programme at the time of carrying out the modelling is not
known, a planned programme is needed.

 The planned reinsurance programme to which claims relating to the new business
period are subject.

 The extent to which occurrencebased programmes may overlap with exposure


periods that would not otherwise be included in the model. It may therefore be
necessary to include allowance for such exposure to ensure the application of the
reinsurance on modelled exposures is accurate.

 The expenses of the firm.

 The value and details of assets by asset category (other than the reinsurers’ share of
technical provisions, as this is covered above).

 Credit exposures, for example, broker balances.

 Details of operational risks – often identified in a risk register.

© IFE: 2019 Examinations The Actuarial Education Company


SP7-20: Capital modelling – methodologies Page 23

Checks and documentation


It is important that the data used to build a model of the business is as close to that in the
actual business as possible. Therefore we should reconcile the above data items to the
firm’s financial statements and business plans, and document explanations of differences.

This will be possible where the model is being run at a date for which financial statements exist.
However, this will not always be the case.

We should carry out reasonableness checks on the assumptions selected for a capital
model, by checking that they produce sensible outputs. For instance, we can check that
expected gross loss ratios derived from assumptions for claim frequency and severity are
similar (or even equal) to claims ratios in the business plan.

Classification of business
The classes of business to which premiums and unpaid claims are allocated in a capital
model should be as close as possible to the classification of business used by the firm for
internal management reporting, reserving and business planning. This is not necessarily
easy to achieve, as a firm may use different classifications of business for different internal
reporting purposes.

We should aim in the capital model to use a business classification:

 with which others in the firm will be able to identify

 which is not at too high a level, such that the accuracy of the results from the capital
model would be unacceptable (that is, too few groups so that some groups are too
heterogeneous)

 which is not so detailed that the model becomes overly complex, has overly long run
times, or parameter error for many of the classes becomes unacceptably high
because there is paucity of data in many of the classes.

It is important to communicate what business classification has been used in the model and
why.

5.3 Assumptions – general


We need assumptions to simulate (stochastically or deterministically) quantities for the
various components of the capital model over the period covered by the model.

Again, it might help to consider these components as assumptions regarding the amount and
timing of future cashflows, eg premiums, claims, expenses etc.

The Actuarial Education Company © IFE: 2019 Examinations


Page 24 SP7-20: Capital modelling – methodologies

These may include:

 Gross written or earned premium (or written / earned exposures plus gross premium
per unit of exposure if this is practical).

 Ceded reinsurance premiums, including reinstatement premiums and purchases of


any reinsurance on a ‘losses occurring during’ basis needed to cover claims
occurring after the capital model date, arising from business written prior to the
capital model date.

 Ultimate gross claims (including claim management costs, but excluding natural
catastrophe claims) by class of business, split between attritional, and large claims.

 Catastrophe claims (natural and man-made).

 Claims payment profiles.

 Gross reserve movements, by class of business.

 Reinsurers’ share of gross ultimate claims.

 The proportion of reinsurers’ share of gross claims that the firm is unable to
recover, by reinsurer.

 Reinsurance exhaustion.

 Reinsurer downgrade assumptions (possible change in default risk).

 Inflation.

 Expenses, both acquisition and administrative.

 Investment returns by asset class and assumptions on economic variables (such as


exchange rates and yield curves).

 Operational losses (for example, from company mismanagement).

 Tax.

 Dividends.

5.4 Assumptions – the specifics


The remainder of this section discusses the assumptions needed to simulate quantities for
the above components of a capital model and the data needed in order to make appropriate
selections for these assumptions.

You can use this section as a reference.

© IFE: 2019 Examinations The Actuarial Education Company


SP7-20: Capital modelling – methodologies Page 25

Gross written premium

Assumptions needed Data needed


Future rate changes and planned exposures, or gross written Historical rate
premium: changes.

 If the capital model allows for more than one year of new Underwriters’
business, it may be appropriate to simulate premium rates views.
and exposures.
The firm’s business
 If we allow for just one year of new business, it may be fit plan.
for purpose to assume written premiums will be as per the
business plan (but with stress-testing to estimate the Historical retention
impact of variation from this). rates.
 If we assume changes to rates and exposures, we should Historical new
allow for the potential for these being interdependent; that business rates.
is, an increase in the former could cause a reduction in the
latter.

New business rates and business retention rates.

Ceded reinsurance premiums

Assumptions needed Data needed


Reinsurance rates in respect of contracts covering unexpired and Terms of
planned future written business or ceded premiums: reinsurance
contracts entered
 If the capital model allows for more than one year of new into to which
business, it may be appropriate to simulate reinsurance unpaid claims and
rates and apply them to simulated gross written premium claims from
or to the amount of layers of reinsurance planned to be unexpired business
purchased. are subject.
 If we allow for just one year of new business, it may be fit
Planned
for purpose to assume ceded premiums will be as per the
reinsurance
business plan (again stress-testing may be employed).
programmes to
Changes in conditions from existing to future reinsurance which claims from
arrangements. business planned
to be written will be
We should allow for the cost of reinstating reinsurance if gross subject.
claims exceed retention levels. (Reinstatement costs will be a term
in the reinsurance arrangement.) Firm’s business
plan.
The potential for market forces, say following a major loss event,
increasing the costs of reinsurance on a ‘losses occurring during’ Historical changes
basis in respect of unexpired business at the capital model date by in reinsurance
more than usually expected. rates.

The Actuarial Education Company © IFE: 2019 Examinations


Page 26 SP7-20: Capital modelling – methodologies

Gross claims (excluding catastrophes) by class of business, split by


attritional and large claims

Assumptions needed Data needed


If simulated from claims ratios (applied to Historical claims ratios, frequencies and
premium) – a distribution, mean and severities.
standard deviation of claim ratios for each
class. We should revalue these to the date of the
capital model by adjusting for inflation,
If simulated from claim frequency (applied to rate changes and changes to terms and
an exposure measure) and severity – conditions.
distribution, mean and standard deviation of
frequency and severity for each class. Historical inflation rates.

Depending on the class and available data, it Input from other areas of the business
may be appropriate to simulate large claims (mostly underwriters and claims
from frequency and severity information and handlers), particularly their views on
to simulate other (attritional) claims from expected and extreme underwriting
claims ratios. losses.

Assumptions of claims ratios and claim


severity should allow for future claims
inflation.

Catastrophe claims

Assumptions needed Data needed


Frequency and severity, or a distribution, of Frequency and severity of historical and
the firm’s exposure to each catastrophe potential catastrophes, the firm’s
event. exposure to potential catastrophes.

For natural catastrophes, the assumed Data provided by specialist catastrophe


frequency should allow for events occurring modelling firms (which may be used
in clusters due to climatic forces. because they have more expertise and
data than available to the firm), needs to
The assumed severity should allow for be applicable to the firm’s exposure to the
potential inadequate resources being catastrophes under consideration.
available to mitigate or repair damage.

© IFE: 2019 Examinations The Actuarial Education Company


SP7-20: Capital modelling – methodologies Page 27

Gross reserve movements

Assumptions needed Data needed


If simulated at aggregate class level – a  Best estimate reserve.
distribution, mean (usually best estimate
gross reserve) and standard deviation of  Carried reserve (if different).
best estimate gross reserves for each class.  Historical reserve movements.
If simulated at individual claim level – claim Input from business (mostly claims,
frequency for each past year (to be applied reserving and underwriting staff),
to exposure for that year) and claim severity particularly their views on extreme
for each past year. These assumptions reserve movements.
should be affected by:
Number of reported and estimated IBNR
 numbers of claims reported
claims.
 claims settled
Case reserves.
 case reserves plus part payments on
non-settled claims, that is, a Past settled claims (revalued to the date
Bayesian approach. of capital model).

Claims inflation on unpaid claims in respect Historical inflation rates.


of business earned to the date of the capital
model.

Reinsurers’ share of gross ultimate claims occurring post model date and
gross reserve movements, by reinsurer

Assumptions needed Data needed


If it is necessary to estimate this component, Historical ratios of net to gross claims by
(ie because it is not practical to apply accident / underwriting year for each
reinsurance programmes to simulated gross class.
claims and gross reserve movements), then
the assumption needed is ratios of net to Reinsurance programmes.
gross claims for each class.
Input from business (mostly underwriters)
on their views of ratios of net to gross
claims in respect of future earned and
written business.

Proportion of reinsurers’ share of claims not recovered

Assumptions needed Data needed


Probability of default of the reinsurer and the Historical reinsurer failures.
expected loss to the insurer if the reinsurer
does default. Credit ratings for current reinsurers.

These assumptions should take into account Views of the firm’s reinsurance
the potential of major industry wide losses department and brokers.
causing increased reinsurer defaults and
Data from rating agencies.
amount of each resultant loss.

The Actuarial Education Company © IFE: 2019 Examinations


Page 28 SP7-20: Capital modelling – methodologies

Expenses

Assumptions needed Data needed


Proportion of acquisition costs to gross The firm’s business plan.
written premium, by class of business.
Historical inflation.
Administration costs.

Inflation.

Inflation, expected returns, volatilities and dependencies between the


modelled economic series

Assumptions needed Data needed


Assumptions needed depend on the method Depends on methods used.
used to simulate these and are outside the
scope of this Core Reading. Amongst data usually needed is:
historical inflation, government bond
yields (by duration and territory),
corporate bond yields (by duration and
territory), equity returns and exchange
rates.

Economic scenario generators are discussed further in another chapter.

Operational losses

Assumptions needed Data needed


Risks that have the potential to give rise to The firm’s risk register.
operational losses.
Input from the business (mostly risk and
The likelihood of each risk materialising. operations management areas of the
business), particularly their views on the
Expected costs if the risks materialise. likelihood of operational losses
occurring and the magnitude of extreme
The variability around these costs.
operational losses.
A distribution of losses arising from each risk.

The extent to which multiple risks might


materialise during the period covered by the
capital model.

Tax

Assumptions needed Data needed


The tax rate on profit retained in a financial year. Rules from the local taxation
authority.
The tax offset from carried forward losses.

© IFE: 2019 Examinations The Actuarial Education Company


SP7-20: Capital modelling – methodologies Page 29

Dividends

Assumptions needed Data needed


Dividend payments. Board policy on dividend payments.

Aggregation inputs (eg correlation coefficients) are considered below.

The Actuarial Education Company © IFE: 2019 Examinations


Page 30 SP7-20: Capital modelling – methodologies

6 Aggregation methodologies
It is likely that we will use more than one approach, model or stress test in considering all
risks that the company faces. We should bring together the results of these different
elements and produce a single result: one that can be compared both to available capital
and other metrics.

A metric is a type of measure. For example, a company’s credit rating might be an appropriate
metric for its credit risk. The amount of capital held to protect the company from different risks is
a metric for the overall size of those risks. Another possible risk metric is earnings volatility.

In carrying out this aggregation, we should consider both the method used to combine the
results of different elements and the level of correlations that might exist between these
elements.

In a stochastic model, we typically aggregate risks by determining joint probabilities using


Monte Carlo simulation techniques. The method should capture the correlation between
variables in the more extreme stresses that are likely to be of interest to the regulator.

Correlations are by no means constant across the whole of a joint distribution. It is likely that
results are more highly correlated under extreme scenarios, ie in the tails of the distribution. The
capital model should aim to capture these effects.

We can achieve this using an appropriately heavy-tailed distribution together with a


correlation matrix, or using a copula (discussed in Chapter 22).

In some cases, modelling underlying drivers (for example, loss severity directly linked to
economic factors) may be applied to capture correlations.

In other words, if two classes of business are both separately linked to economic factors within
the capital model, then the results for these classes will automatically be correlated, without the
need to link the two classes directly. However, it will still be necessary to ensure that the
correlation in the tails of the distributions is appropriate.

If we have not fitted statistical distributions, or we cannot determine a joint distribution,


then we should use more approximate methods of combination. Across different risks, it
may be suitable to use a variance / covariance approach to aggregate stand-alone risk
measures, and apply realistic disaster scenarios that assess the reasonableness of the
calculated overall capital charge.

In either case, we should state and clearly justify the dependency assumptions between
various categories such as the following:

 underwriting classes of business (for example, between motor and household


business)

 risk types (for example, between underwriting risk and reserving risk)

 years of account

 legal entities within a group, including between entities in different territories.

Correlation assumptions will typically be subjective and based on a high level of judgement.
It is good practice, when justifying selected correlations, to consider historical events, the
range of correlations that could be deemed as equally valid, and the consequent impact on
the capital models of such correlations.

© IFE: 2019 Examinations The Actuarial Education Company


SP7-20: Capital modelling – methodologies Page 31

Chapter 20 Summary
The aim of a capital model is to predict the future of a general insurer under different
scenarios.

Modelling the cashflows


Future written premium income – Subject to adequate data, this should be projected
separately for each line of business and by source of business.

Future claims – These should be split according to the stage the claim is in (notified,
IBNR etc). Attritional, large and catastrophe claims should be modelled separately.
Catastrophes may be allowed for using stress tests or catastrophe modelling software.

Future expenses – Some expenses, eg commission will be a function of other assumptions.


Other expenses will depend on the projected business growth and staffing plan.

Future investment returns – These will depend on the insurer’s current investment portfolio,
investment prospects and the current and projected future investment policy.

Ceded reinsurance – These cashflow assumptions should take into account existing
arrangements (including future changes to these) and possible new arrangements.

Cashflows should be modelled taking into account the current environment, including:
 the investment environment, eg interest rates
 the economic environment, eg recessions and booms
 the insurance cycle.

The Actuarial Education Company © IFE: 2019 Examinations


Page 32 SP7-20: Capital modelling – methodologies

Chapter 20 Summary continued


Deterministic models
With a deterministic approach, a single run is produced for each value of parameter. Stress
tests quantify the effect of varying a single parameter. Scenario tests quantify the effect of a
change in multiple parameters.

The advantages of using a deterministic approach are:


 the model is usually easier to design and quicker to build / run, so the user can
introduce more dimensions
 it is clearer which stresses and scenarios have been tested – this may be easier for
users to understand
 stress and scenario tests provide a mechanism for entering into a dialogue with
senior management and a check on reasonableness
 stress and scenario tests can help to integrate capital and risk management
 scenario modelling can be extended to scenario planning and ‘what-if’ analysis
 stress and scenarios can be used for risks that require subjective judgement
 potential cause and effect relationships may be modelled better deterministically
 deterministic models are good for checking stochastic models.

The steps in running a deterministic model are:


 specify the purpose of the investigation
 set the risk measure
 collect, group and modify data
 choose the form of the model, identifying its parameters and variables
 ascribe values to the parameters, taking into account the risk measure being used
 construct a model based on the expected cashflows
 check that the goodness of fit is acceptable (and fit a different model if not)
 run the model using the selected variables
 run the model using different parameters to check sensitivity.

Stochastic and deterministic approaches can be combined so that some risk categories are
modelled in one way, and some in another.

© IFE: 2019 Examinations The Actuarial Education Company


SP7-20: Capital modelling – methodologies Page 33

Chapter 20 Summary continued


Stochastic models
With a stochastic approach, (at least) some of the variables are assigned probability
distributions and the output is a range of possible outcomes.

The advantages of using a stochastic approach are:


 they are computationally more intensive than a deterministic model
 they test a wider range of scenarios
 they can more easily explore ‘ripple effects’
 it is possible to apply multiple risk measures to the same set of final output
 they explore (and can rank) all possible combinations of inputs, whereas stress tests
(under a deterministic approach) are difficult to combine
 they can be used to assess the financial impact of guarantees.

The steps in running a stochastic model are:


 specify the purpose of the investigation
 select an appropriate model structure
 set the risk measure
 determine the types of scenarios to develop and model
 decide which variables to include and their interrelationships
 collect, group and modify data
 estimate the parameters that should be used for each variable:
– choose suitable density functions for the stochastic variables
– estimate the required parameters for the chosen density function(s)
– ascribe values to the variables that are not being modelled stochastically
 specify correlations between variables
 test and validate the reasonableness of the assumptions and their interactions
 check that the goodness of fit is acceptable (and fit a different model if not)
 construct a model based on the chosen density functions
 run the model many times (simulation)
 produce a summary (a distribution) of the results
 run the model using different distributions / parameters to check sensitivity.

The Actuarial Education Company © IFE: 2019 Examinations


Page 34 SP7-20: Capital modelling – methodologies

Chapter 20 Summary continued


Model requirements
A capital model requires a defined risk measure, eg a 99.5% VaR.

Models should:
 be valid, complete and adequately documented
 reflect the risk profile of the classes of business being modelled
 include appropriate parameter values for the business / investments being modelled
 produce outputs that are capable of independent verification and are readily
communicable; the degree of uncertainty surrounding the outputs should also be
verifiable
 not be overly complex
 be sufficiently flexible
 have all parameters clearly identified and justified
 be structured and documented so as to be clearly understood by senior managers
without actuarial expertise.

There are additional requirements for stochastic models.

The modelling process


In assessing the adequacy of capital, it will be necessary to:
 identify significant risks facing the business
 assess their impact (both with and without risk controls in place)
 use this to quantify how much capital is required.

The two broad approaches available to firms when producing a capital model are stress and
scenario tests and economic capital models (DFA).

The main source of risk is a lack of credible, relevant data on which to base assumptions.
External data may be used to supplement internal data.

There should be a clear audit trail of the whole process.

All parties involved should have an understanding of the process.

© IFE: 2019 Examinations The Actuarial Education Company


SP7-20: Capital modelling – methodologies Page 35

Chapter 20 Summary continued


Data and assumptions
The main items of data needed for capital modelling are:
 unexpired premiums (gross and net), split by class of business
 planned premiums (gross and net), split by class of business
 gross unpaid claims, split by class of business
 claims payment profiles
 claim limits
 aggregate exposure by location to assist in catastrophe modelling
 future reinsurance costs
 reinsurance programmes for gross unpaid claims, unexpired business and planned
reinsurance programmes
 expenses
 asset values
 details of risks, such as credit exposures and operational risks.

The assumptions that must be set include:


 gross written or earned premium
 ceded reinsurance premiums
 ultimate gross claims (including claims management costs)
 catastrophe claims
 claims payment profiles
 gross reserve movements, split by class of business
 reinsurers’ share of gross ultimate claims and the proportion of this the firm may be
unable to recover
 reinsurance exhaustion and reinsurer downgrade assumptions
 expenses
 inflation
 investment returns, split by asset class
 operational losses
 tax and dividends.

A decision will be needed as to which assets to include in the model.

The Actuarial Education Company © IFE: 2019 Examinations


Page 36 SP7-20: Capital modelling – methodologies

Chapter 20 Summary continued


The following checks should be carried out:
 a reconciliation of data used in the model to financial statements and business plans
 reasonableness checks that the assumptions produce sensible outputs.

The process should be carefully documented and communicated, including:


 any concerns over data
 the rationale for selecting assumptions and methodology
 any alternatives considered and why they were rejected
 the business classification that has been used in the model.

Aggregation methodologies
If more than one approach, model or stress test has been used in considering risks, these
need to be brought together. Risks may be aggregated by determining joint probabilities that
capture correlations between variables. If the determination of a joint distribution is not
possible, then more approximate methods of combination will need to be used.

Correlation assumptions will typically be subjective. These should be set while considering:
 historical events
 an acceptable range of considerations
 the subsequent impact on the capital models.

Validating the model


Stress testing quantifies the effect of varying a single parameter.

Scenario tests quantify the effect of changing a combination of parameters. They help the
insurer analyse severe scenarios. Allowance should be made for correlations between risks.

Sensitivity testing is the process of testing the extent to which the results of a capital model
change as a result of making a small change to an assumption in the model. The purpose of
sensitivity testing is to identify the most sensitive assumptions.

Back testing
Back testing is the process of comparing actual experience with model output. It tests how
well the model predicts the range of outcomes that actually occur.

Back testing can be used to:


 check the model reflects the real world
 check the model reflects the risk profile of the business
 indicate other shortcomings in the model
 check whether past performance is a good indicator of the future.

© IFE: 2019 Examinations The Actuarial Education Company


SP7-20: Capital modelling – methodologies Page 37

Chapter 20 Practice Questions


20.1 Describe the items of information that should be documented throughout the capital modelling
process.

20.2 Discuss the relative merits of using deterministic and stochastic models.

20.3 (i) List five key items of data that a general insurer would need in order to model gross
future claims.

(ii) State further pieces of information that would be required by the general insurer to
model net future claims.

(iii) Outline the sources of data available to the general insurer in order to set future claims
assumptions.

20.4 (i) Define the following terms:


Exam style  stress testing
 scenario testing
 sensitivity testing. [2]

(ii) Describe the uses of each test. [13]


[Total 15]

20.5 A general insurer wishes to model its private motor claims experience to help with rating,
Exam style
reserving and financial planning.

(i) List the steps involved when using a stochastic approach to model the claims experience.
[11]

(ii) Suggest some distributions that might be appropriate to use to model the claim frequency
and the claim amounts. [2]
[Total 13]

20.6 A general insurance company is setting up a new line of business.


Exam style
(i) Describe the modelling necessary to assess the capital required. [9]

(ii) List the principal experience assumptions that may be needed for the model in part (i). [8]
[Total 17]

20.7 Describe the stages in the life cycle of a capital model. [21]
Exam style

The Actuarial Education Company © IFE: 2019 Examinations


Page 38 SP7-20: Capital modelling – methodologies

The solutions start on the next page so that you can


separate the questions and solutions.

© IFE: 2019 Examinations The Actuarial Education Company


SP7-20: Capital modelling – methodologies Page 39

Chapter 20 Solutions
20.1 The model itself should be adequately documented – this is true of any model.

This is necessary so that the key assumptions and approximations made are understood and can
be communicated, so that it can be run by other members of staff and improvements introduced
over time.

This is particularly true for stochastic models which should be carefully documented so that they
can be understood by senior management and board members who do not have actuarial
expertise.

The objectives of any capital requirement regime should be clearly documented. This will help to
ensure that:
 senior management focus on risk management
 the link between risk and capital setting is clearly visible
 the capital model is being used within the decision making process.

Details of all prudential risks, processes and controls should be included.

There should be a clear audit trail from the impact of any financial calculations to the relevant risk
capital allocation in the capital model, whatever modelling approach is adopted.

Where considerations of particular risk issues have been made, it should be stated what issues
were considered, how they were considered and the reasons behind the conclusions and findings.

Any differences between data items used in the model, and the insurer’s financial statements and
business plans should be documented.

The rationale for selecting assumptions should be justified and documented, especially for the
more sensitive assumptions. Greater attention should be given to the justification and
documentation of the rationale for selecting values of these assumptions, because they are more
critical to the results of the capital model.

As well as identifying which of the parameters are most critical to the capital requirement value,
the capital model documentation should give indicative movements in the capital requirement
value for the most sensitive parameters.

Documentation should also justify the methodology selected and mention the alternatives
considered and why rejected.

The Actuarial Education Company © IFE: 2019 Examinations


Page 40 SP7-20: Capital modelling – methodologies

20.2 Advantages and disadvantages of a deterministic model

+ It is easy to explain to a non-technical audience, since it does not involve the explanation
of probability distributions, particularly because the scenarios tested may have been
developed in conjunction with this audience.
+ It is clear which economic scenarios have been examined.
+ The model is usually simpler than a stochastic model, and therefore less time consuming
to build and run …
… and therefore less costly.
+ It is easier to introduce more (subjective) detail, such as descriptions of reinsurance
programmes or the treatment of underlying risks.
+ It helps to ensure that the scenarios modelled are chosen intelligently, rather than
included somewhere amongst a myriad of other scenarios (as is the case with a stochastic
model).
+ It is easy to link the model with the risk register, ie to integrate capital and risk
management.
+ Scenarios can be used to aid scenario planning.
+ It is easier to consider interdependencies between risks without having to estimate
statistical correlations.
– It is more difficult to determine which economic scenarios to test …
… and the testing may not cover a sufficient range of scenarios.
– Some important scenarios may be missed.
– It is not a good model for valuing options and guarantees (although these are rare in
general insurance) as it is difficult to model the variability in take up rates or the
guarantee biting.

Advantages and disadvantages of a stochastic model

+ Using a stochastic model, a large number of simulations can be run to identify which
eventualities are acceptable.
+ A stochastic model may, due to its random nature, identify a potentially poor scenario
that would not have been thought of as a specific scenario to test under a deterministic
model.
+ A stochastic model takes into account the variability of the model parameters and the
covariances between them.
+ The output of a stochastic model forms a distribution of values from which statistics such
as the mean and the variance of the output and a number of different risk measures can
be calculated.
+ Confidence levels can also be calculated if required.
+ Such information is useful in understanding the risks inherent in the product design.

© IFE: 2019 Examinations The Actuarial Education Company


SP7-20: Capital modelling – methodologies Page 41

+ It is easier to assess the knock-on consequences of a particular scenario over time.


+ Simulation under a stochastic model will explore many possible combinations, and rank
them against the chosen risk measure.
+ A stochastic model is useful for modelling any options and guarantees (although these are
rare in general insurance) embedded in the contract design, since the likelihood of option
take up, or of guarantees biting, can be explicitly allowed for.
– A stochastic model can be longer and more expensive to run.
– A stochastic model is likely to be more complex to design and test, leading to potentially
increased operational risk.
– The output from a stochastic model may be difficult to interpret and to communicate to
senior management.
– The model output is only as good as the input and depends on the choice of probability
distribution and its parameters for the stochastically modelled variables.
– Whilst a stochastic model is a useful tool for making sure that all eventualities have been
tested, there is no substitute for experience.
– The best course of action is often for the actuary to consult as many people as possible
about possible eventualities and to think the unthinkable.

20.3 (i) Data needed to model gross future claims

 gross unpaid claims at the as-at date of the capital model, by class of business
 claims sizes
 claim frequencies
 claim settlement patterns
 aggregate exposure by location to assist in catastrophe modelling

(ii) Further pieces of information needed to model net future claims

 whether claims are likely to reach limits, and the associated limits
 the reinsurance programme to which claims arising from unexpired business is subject
 each reinsurer’s participation on the programme
 the extent to which claims paid have used up coverage available on these programmes
 possible new arrangements that could be put in place, particularly if there are any
forecast changes in underlying exposures
 any expected softening or hardening of future reinsurance costs
 the reinsurance programmes to which gross unpaid claims from future business will be
subject (either actual or planned)
 probability of reinsurer default (for each reinsurer)

The Actuarial Education Company © IFE: 2019 Examinations


Page 42 SP7-20: Capital modelling – methodologies

(iii) Sources of data

One of the main sources of data for an existing class of business would be historical claims data –
both for frequencies and severities.

Since these would need to be revalued to make them appropriate to the future period,
information would be needed on:
● inflation – this information may be obtained from economic forecasts …
… or as the difference between the yields on fixed interest and index-linked government
securities
 changes to the mix of policyholders – this information may be obtained from:
– an analysis of the existing mix of business, including trends
– discussions with senior management and/or the marketing department about the
target market
 changes to policy terms and conditions – this information should be available internally
within the company, eg from policy documents.

Useful information may also be gleaned from underwriters, eg their views on expected and
extreme underwriting losses.

Further data may be needed on catastrophe claims. This is likely to be obtained from catastrophe
modelling companies / software.

Information may be obtained from reinsurers as to the likely levels of reinsurance (eg limits)
available in the future.

The probability of reinsurer default may be estimated by analysing past reinsurer failures …

… and the credit ratings (and any other data from rating agencies) of the current (and planned
future) reinsurers.

There may also be internal (subjective) views on the current (and planned future) reinsurers.

20.4 (i) Stress tests, scenario tests, sensitivity tests

Stress testing quantifies the effect of varying a single parameter. It is a preliminary step in
performing a scenario test. [½]

Stress tests may be deterministic but they are often developed with probability distributions in
mind. [½]

Scenario testing quantifies the effect of a change in multiple parameters. It is a natural extension
to a stress test. [½]

Sensitivity testing is the process of testing the extent to which the results of a capital model
change as a result of making a small change to an assumption in the model. [½]
[Total 2]

© IFE: 2019 Examinations The Actuarial Education Company


SP7-20: Capital modelling – methodologies Page 43

(ii) Uses of each test

Stress tests

Stress tests are a necessary tool in assisting an insurer in managing its risks and maintaining
adequate financial resources to deal with the risks to which it is exposed. They can be used to
identify and quantify the impact of different stresses on an insurer’s expected financial position.
[1]

Stress tests are often used:


 for modelling future catastrophe claims [½]
 for analysing the impact of unlikely (but not impossible) adverse stresses of one
parameter [½]
 to perform additional tests on uncertain assumptions [½]
 as a matter of good corporate governance, even if more sophisticated models are also
used [½]
 to improve an insurer’s understanding of the potential impact of individual risks in
isolation [½]
 to identify and investigate the risks incurred by extreme movements in parameters. [½]

Scenario tests

Scenario tests are often used:


 to consider the combined effect of a number of risks and the cumulative effect of several
different mitigating actions occurring at the same time [½]
 to allow for interactions between variables that would not be considered under stress
tests [½]
 within stochastic models to explore combinations of randomly-generated values for the
key parameters by exploring scenarios that you might otherwise not have imagined to be
important [½]
 to help to understand the relationships between different types of risks [½]
 as a communication tool, to make the range of results more comprehensible [½]
 to help with scenario planning and ‘what-if’ analysis. [½]

Stress and scenario tests can also be used:


 as a broad approach when using a capital model:
– stress and scenario tests are used explicitly with deterministic models [½]
– stress and scenario tests are used implicitly with stochastic models – they are
effectively based on stresses / scenarios weighted by probabilities … [½]
… in a DFA model, stresses and scenarios are generated automatically and often
cannot be ‘seen’ [½]

The Actuarial Education Company © IFE: 2019 Examinations


Page 44 SP7-20: Capital modelling – methodologies

 to validate the output from stochastic models for reasonableness [½]


 to help calibrate assumptions in stochastic models [½]
 to provide a transparent link between the capital model and the insurer’s risk register [½]
 to provide a useful mechanism for entering into a dialogue with senior management and
give them comfort as to the reasonableness of the overall capital value [½]
 for those risks that cannot easily be modelled quantitatively and where more subjective
judgement is required … [½]
… this allows a better degree of focus on the more important areas of the distribution of
outcomes for the key risks, when a full specification of the distributions is impossible. [½]

Sensitivity tests

Sensitivity testing should be used:


 to identify the more sensitive assumptions in the capital model, ie which assumptions, if
changed, would have the greater impact on the results of the capital model … [½]
… attention can then be focused on the more sensitive assumptions, and greater
attention can be given to the justification and documentation of the rationale for
selecting values of these assumptions [½]
 as part of any stochastic modelling process (to run the model using different distributions
and parameters to check sensitivity) [½]
 to help select assumptions – where there are several possible selections for an
assumption that each appear equally reasonable, a sensitivity test may be carried out to
find out the effect of making different selections for the assumption under consideration
[½]
 as a minimum standard and part of the validation and sign off process [½]
 to help understand the variability and uncertainty in parameter values so that these can
be communicated to the board and senior management. [½]
[Maximum 13]

20.5 (i) Stochastic modelling steps

 Specify the purpose of the investigation, eg rating, reserving, or planning. [½]


 Select an appropriate model structure, ie which claims to include (bodily injury, property
damage etc). [½]
 Set the risk measure, eg VaR. [½]
 Determine the types of scenarios to develop and model, eg claims in a recession, changes
in legislation over cover limits. [½]
 Decide which variables to include, and their interrelationships. [½]
 Collect claims and exposure data going back at least five years if possible. Find the right
balance of relevance and credibility. [½]

© IFE: 2019 Examinations The Actuarial Education Company


SP7-20: Capital modelling – methodologies Page 45

 Group data, eg bodily injury and property damage. [½]


 Adjust for inflation and IBNR, which could be considerable for motor business. [½]
 Adjust for any other changes, eg trends in policyholder attitudes, terms and conditions,
levels of excess etc. [½]
 Estimate the parameters that should be used for each variable (ie the mathematics that
specifies the behaviour of each variable): [½]
– choose a suitable density function for each of the variables to be modelled
stochastically … [½]
… eg claim severity, which may depend on other variables that are often modelled
stochastically (such as inflation) … [½]
– estimate the required parameters for the chosen density function(s) [½]
– ascribe values to the variables that are not being modelled stochastically … [½]
… eg claim frequencies, which may be more predictable and therefore could be
modelled deterministically. [½]
 Specify correlations between variables. [½]
 Test and validate the reasonableness of the assumptions and their interactions. [½]
 Check the goodness of fit is/are acceptable and attempt a fit with different density
function(s) if it is not. [½]
 Construct a model based on the chosen density function(s). [½]
 Run the model many times, each time using a random sample from the chosen density
function(s). [½]
 Produce a summary of the results that shows the distribution of the modelled results
after many simulations have been run. [½]
 Run the model using different distributions / parameters to check sensitivity. [½]
 Continually update the model and its parameters to remain relevant in the ever-changing
environment in which the insurer operates, eg to reflect the prevailing economic,
legislative and fiscal conditions. [1]
[Maximum 11]

(ii) Possible distributions

Claim frequency: Poisson, negative binomial, geometric [½ mark each, maximum 1]

Claim amounts: log-normal, Pareto, Weibull, translated gamma [½ mark each, maximum 1]
[Total 2]

The Actuarial Education Company © IFE: 2019 Examinations


Page 46 SP7-20: Capital modelling – methodologies

20.6 (i) Process to assess capital required

Capital will be needed to cover:


 the product development costs, eg marketing of the new product, admin systems [½]
 the usual new business strain arising from:
– administrative expenses of selling new business [½]
– commission [½]
– the setting up of reserves, including any required solvency margin. [½]

It will first be necessary to estimate the capital needed for the product design stage, including the
costs of marketing the new product. [½]

Next, it will be necessary to create model points that will represent the mix of business that the
company expects to sell. [½]

A cashflow model will be needed that projects cashflows into the future. Cashflows will include
premiums, claims, expenses, investment return, reinsurance premiums and recoveries and tax. [1]

The model must also allow for any required solvency margins. [½]

The cashflows should be discounted back (at an appropriate risk discount rate) in order to
determine the reserves that are needed. [½]

A basis should be chosen that will be fit for the purpose:


 if the company is calculating economic capital, a realistic basis might be appropriate [½]
 if the company is calculating regulatory capital, then a prudent or cautious basis may be
more appropriate – in this case the basis may be prescribed. [½]

It will be necessary to decide:


 which variables will be modelled deterministically and which ones stochastically [½]
 the time horizon over which to model the cashflows – this should be the lifetime of the
business being modelled [½]
 the degree of accuracy required (and so the number of simulations required – a minimum
of 50,000 simulations would generally be considered appropriate). [½]

Correlations between the different variables should also be allowed for. [½]

The model should ensure consistency between the various assumptions. [½]

Allowance should also be made for any guarantees on the contract. [½]

The future tax position should also be considered. [½]

Scenario and sensitivity testing should be carried out on the different assumptions. [½]

© IFE: 2019 Examinations The Actuarial Education Company


SP7-20: Capital modelling – methodologies Page 47

The output from the model will be the amount that needs to be held as reserves, plus any
required solvency margin. This will be scaled up to reflect the volume of business that the
company expects to sell. [1]

The capital required for product development will be added in to give the total capital required to
set up the new line of business. [½]
[Maximum 9]

(ii) Principal experience assumptions

Experience assumptions may include:


 gross written or earned premium (or written / earned exposures plus gross premium per
unit of exposure)
 ceded reinsurance premiums, including:
– premium to reinstate reinsurance
– purchases of any reinsurance on a ‘losses occurring during’ basis needed to cover
claims occurring after the capital model date arising from business written prior to
the capital model date
 claim frequency
 claim severity
 claims payment profiles
 claims inflation
 ultimate gross claims (including claim management costs, but excluding natural
catastrophe claims) …
… and reinsurers’ share of gross ultimate claims …
… as well as the proportion of the reinsurers’ share of gross claims that cannot be
recovered, for example…
… reinsurance exhaustion assumptions…
… and reinsurer downgrade assumptions
 catastrophe claims (natural and man-made)
 gross reserve movements
 lapses
 expenses, including:
– acquisition expenses
– administrative expenses
– claims handling expenses
 expense inflation
 investment return
 new business volumes

The Actuarial Education Company © IFE: 2019 Examinations


Page 48 SP7-20: Capital modelling – methodologies

 new business mix


 tax
 dividends
 operational losses.
[½ each, maximum 8]

20.7 The life cycle of a capital model is no different to that of any other type of model used in
insurance. [½]

The stages include:


 designing [½]
 building [½]
 testing [½]
 parameterisation [½]
 documentation [½]
 validation [½]
 recalibration [½]
 monitoring [½]
 development [½]
 use. [½]

Designing

The model must be designed so that it:


 is appropriate to the purpose to which it will be used [½]
 will allow for any constraints, eg those that may have been imposed by the regulator. [½]

The design phase should have regard to all intended users of the model and the risk-types that
need to be modelled. [½]

Building

It may be necessary to build a new model. Alternatively, an existing model could be modified, or
a commercial model could be purchased. [1]

During the ‘building’ phase, consideration should be given to:


 the purpose of the model [½]
 the level of accuracy required [½]
 the number of times the model is to be used [½]
 the desired flexibility of the model [½]

© IFE: 2019 Examinations The Actuarial Education Company


SP7-20: Capital modelling – methodologies Page 49

 the cost of building (and subsequently running) the model [½]


 the time needed / available for the model to run [½]
 the time needed / available to review and test the model. [½]

Testing

It is important to ensure that the model used is fit for the purpose for which it is being used. This
is particularly relevant when a model is being purchased from an external provider or when an
existing model is being reused for a different purpose. [1]

Parameterisation

The parameters should be appropriate to:


 the business being modelled [½]
 the insurer’s risk appetite [½]
 the external environment [½]
 the time period to which the capital model applies. [½]

Documentation

Depending on the use of the capital modelling exercise, documentation can be a very important
part of the process. [½]

For example, if the model is used for regulatory purposes, the documentation will be used by the
actuaries working on behalf of the regulator. [½]

They will want to ensure all the key risks have been identified and modelled appropriately. [½]

Documentation will also ensure that the key assumptions and approximations made are
understood and can be communicated, so that it can be run by other members of staff, and
improvements introduced over time. [1]

Validation

To give confidence that the results are correct, it can be helpful to compare the results with an
alternative method of risk and capital assessment, such as deterministic stress and scenario tests
on the key risks. [1]

We should also build the model in such a way that it provides a breakdown of the overall capital
requirement in a sufficient level of detail to allow reasonableness checks of the results. [½]

It is also important to check that the results make sense in the real world to ensure that the
model is not giving spurious accuracy. [½]

Recalibration

If any part of the validation stage highlighted any issues, these should be addressed by altering
the parameters, the distribution functions, or possibly even the model structure. [1]

The Actuarial Education Company © IFE: 2019 Examinations


Page 50 SP7-20: Capital modelling – methodologies

Monitoring

As with all actuarial work, it is important to monitor actual experience regularly and compare it
with the assumptions used, to ensure an understanding of any material deviations, so that the
model or assumptions can be improved as required. [1]

Each time a capital modelling exercise is carried out, it is advisable to identify the reasons why the
capital requirement figure has changed from the last time, and to be able to explain – both
qualitatively and quantitatively – the effect of each key change. [1]

Development

This process should be a cycle of continuous improvement and enhancement to ensure that the
model keeps pace with:
 business needs [½]
 regulatory requirements [½]
 industry best practice. [½]

Use

The capital model may be used for a number of different purposes, including assessing solvency
requirements and allocating capital for product pricing. [1]

The life cycle of a capital model is a good example of the use of the actuarial control cycle. [½]
[Maximum 21]

© IFE: 2019 Examinations The Actuarial Education Company


SP7-21: Capital modelling – assessment of capital for various risk types Page 1

Capital modelling –
assessment of capital
for various risk types
Syllabus objectives
4.4 Describe approaches to the assessment of capital requirements for the following risk
types:
 insurance risk
 market risk
 credit risk
 operational risk
 liquidity risk
 group risk.

The Actuarial Education Company © IFE: 2019 Examinations


Page 2 SP7-21: Capital modelling – assessment of capital for various risk types

0 Introduction
To fulfil its business objectives and to meet regulatory requirements, an insurance firm (and
indeed other financial services firms such as banks) must hold financial resources. These
must allow a high degree of confidence that it can meet its obligations to policyholders, and
possibly to other stakeholders in the business.

Question

List the other stakeholders to which an insurance company might have obligations.

Solution

An insurance company may also have obligations to:


 reinsurers, eg reinsurance premiums, or repayments of financial reinsurance
arrangements
 brokers, eg commission payments
 third parties to whom the insurer outsources functions
 creditors, eg providers of goods, such as stationary
 employees, eg wages.

In this chapter, we discuss the different types of capital, the different types of risk that general
insurance companies must hold capital for and how these risks should be incorporated into a
capital model.

© IFE: 2019 Examinations The Actuarial Education Company


SP7-21: Capital modelling – assessment of capital for various risk types Page 3

1 Types of capital

1.1 Available capital


General insurance companies must hold sufficient financial resources to meet their liabilities.

Generally, this requires financial resources to be well in excess of the value of technical and
other liabilities. The excess of financial resources over the value of liabilities is colloquially
known as capital, and more specifically, available capital. ‘Own funds’ is a further term
used in Solvency II.

In banking and other circles, this is often also referred to as risk capital.

1.2 Required capital


How much capital an insurer needs – its required capital – depends on many factors, but a
typical capital modelling exercise links a measure of the required capital to a desired level
of loss absorption ability. This means that, if the insurer’s experience is worse than
planned, it can call upon the available capital to subsidise the losses up to a certain level,
without recourse to its shareholders or members.

The amount of capital that is required depends on the purpose of the modelling exercise. The
two main purposes are to calculate:
 the amount of capital an insurer is required to hold for regulatory purposes – this is
known as regulatory capital
 the amount of capital that an insurer determines is appropriate to hold given its assets, its
liabilities, and its business objectives – this is known as economic capital and will be
higher than the minimum regulatory capital.

Regulatory capital
Capital requirements are probably most commonly considered in the context of regulatory
solvency capital. Most insurance regulators require the insurer to hold sufficient solvency
capital to provide a degree of comfort that the insurer can meet its liabilities to
policyholders as and when they fall due. In other words, a fundamental purpose of
solvency capital is to protect policyholders’ interests.

This is using the terms regulatory capital and solvency capital interchangeably.

In some countries, or for some types of financial provider, the solvency capital requirement
comprises a highly prescriptive, prudent valuation basis. In other regulatory regimes the basic
provisions are established on a best estimate basis, and substantial additional capital margins
must also be held. The security given by the regulatory regime is measured by the total of the
two elements of the solvency capital required.

The Actuarial Education Company © IFE: 2019 Examinations


Page 4 SP7-21: Capital modelling – assessment of capital for various risk types

Two possible regulatory regimes which result in the same total solvency capital requirement are
illustrated below:

Free capital Free capital

Additional
Regulatory Additional capital
Solvency
value of capital requirement
capital
assets requirement
requirement

Provisions
on a prudent
Provisions basis
on a best
estimate
basis

Most insurers hold more solvency capital than the minimum specified by their regulators.
This can be for a range of reasons, but five of the most important are:
1. To reduce the risk that the available capital falls below the regulatory requirement,
which would hamper the firm’s business activities.
For example, an insurer who held only marginally more capital than the regulatory
minimum would be exposed to the risk that a fall in asset values would result in it being
declared insolvent.
2. To give a greater degree of security to policyholders than implied by the relatively
weak regulatory minimum.
3. To maintain its credit rating. The capital required to achieve a particular credit rating
will probably be higher than that to meet regulatory requirements.
4. To meet the requirements of other stakeholders such as debt providers, whose
interests may be subordinated to those of the policyholders.
In other words, the solvency capital is intended to protect the interests of policyholders.
However, the company may well be required to meet its liabilities to other parties;
holding more than the minimum amount will help ensure these liabilities can also be met.
5. To allow a buffer between the actual profitability of the business and the dividend
stream paid to shareholders, who prefer less volatile returns.

In addition to regulators, two other groups of agents for the various stakeholders – ratings
agencies and investment analysts – scrutinise insurers’ solvency capital levels.

Insurers therefore need to be prepared to discuss the output of their own capital models
with these agents, who do not all share the same views on the risk modelling framework or
the required degree of comfort.

The other agents may not believe that the statutory minimum is sufficient given the inherent risks
associated with general insurance.

© IFE: 2019 Examinations The Actuarial Education Company


SP7-21: Capital modelling – assessment of capital for various risk types Page 5

Question

(i) Suggest how the firm’s business activities might be hampered if its available capital fell
below the regulatory requirement.

(ii) Explain why it is important that ratings agencies and investment analysts believe that
insurers are holding sufficient solvency capital.

Solution

(i) Falling below the regulatory minimum

If the level of available capital falls below the regulatory requirement, then the regulator will
intervene to protect the interests of existing or prospective policyholders.

Depending on the severity of the situation, the regulator may require the insurer to establish a
recovery plan, which will be monitored closely by the regulator. Such a plan might include:
 limiting the levels of new business sold
 closing to new business
 changing the investment strategy to a more matched position or to invest in less volatile
asset classes
 appointing a custodian of its assets
 increasing the amount of reinsurance the insurer has in place.

(ii) Perception of solvency

The views of rating agencies and investment analysts will affect:


 the credit rating of the insurer
 the credit rating of the debt the insurer issues
 the attractiveness of lending to the insurer
 the attractiveness of buying shares in the insurer
 the appeal of the insurer’s products
 the insurer’s standing in the market.

As an aside, it is worth remembering that a commercial insurer typically holds additional


financial resources for reasons other than solvency. In common with most businesses,
insurers require working capital for investment in business development, amongst other
things.

Economic capital
The amount of capital that an insurer believes it should hold to fulfil its own objectives is
often called its economic capital requirement.

The Actuarial Education Company © IFE: 2019 Examinations


Page 6 SP7-21: Capital modelling – assessment of capital for various risk types

Economic capital is the amount of capital that an insurer determines is appropriate to hold given
its assets, its liabilities, and its business objectives.

This is distinguished from other measures – the regulatory requirement, in particular – in


that it matches the full range of the insurer’s objectives.

Typically it will be determined based upon:


 the risk profile of the individual assets and liabilities in its portfolio
 the correlation of the risks
 the desired level of overall credit deterioration that the insurer wishes to be able to
withstand.

Question

Explain why an insurer will not wish to hold too large an amount of capital in excess of its
economic capital requirement.

Solution

Capital has a cost, ie the providers of the capital will require a return on their capital. All else
being equal, holding a larger amount of capital means that a given level of profit is spread more
widely amongst the providers of capital.

This approach is also sometimes known as risk-based capital assessment. The amount of
economic capital required would be determined using an internal model.

The first stage in a risk-based capital assessment for an insurer is to produce an economic balance
sheet. An economic balance sheet shows:
 the market values of an insurer’s assets (MVA)
 the market values of an insurer’s liabilities (MVL)
 the insurer’s available capital, which is defined as MVA – MVL.

The available capital is then compared with the economic capital requirement to assess the
insurer’s solvency status.

© IFE: 2019 Examinations The Actuarial Education Company


SP7-21: Capital modelling – assessment of capital for various risk types Page 7

The diagram below illustrates how the economic balance sheet is structured:

Free capital
Available
Economic
economic
capital
capital
Market requirement
value of
assets

Market
value of
liabilities

There is often also a different implied degree of comfort and the requirement may well be
measured on a more realistic basis.

1.3 Modelling capital requirements


Since the required capital is intended to absorb unexpectedly high losses, a typical capital
model will estimate the potential for losses to be higher than expected. Among the many
approaches that are taken, the common thread is that they seek to approximate that
fundamental measure.

This ‘fundamental measure’ refers to the risk measure described below. An example of such a
measure is a VaR.

Differences between economic capital requirements, regulatory capital requirements and


the output from a ratings agency’s model can normally be attributed to such
approximations or differences in the nature or level of protection required.

Depending on the insurer and its regulatory regime, either economic or regulatory requirements
may drive the eventual capital held.

1.4 Core features of capital models


We can characterise most models used to derive capital requirements by three core
features: a risk profile, risk measure and risk tolerance.

Risk profile
The risk profile is defined fundamentally by:

 the risks that have been modelled (including the way that they have been modelled)

 the key outcome used to measure success or failure.

Risks modelled are typically those arising from business that has already been written and
a finite period of new business activity.

Typically, we use a financial outcome as a measure of success or failure – for example, the
profit and loss or net cashflows over a selected time horizon, or the balance sheet position
at the end of a selected time horizon.

The Actuarial Education Company © IFE: 2019 Examinations


Page 8 SP7-21: Capital modelling – assessment of capital for various risk types

Risk measure
We can define the risk measure in terms of the desired impact of the capital on the outcome
considered. For example, we might set capital to achieve a selected probability that the
balance sheet at the end of one year shows a surplus no less than zero.

The risk measure links the outcome (in this case the balance sheet deficit) to the capital required
to achieve that outcome. It will be defined in terms of a required confidence level, (ie risk
tolerance) and a time horizon. An example may be VaR.

Risk tolerance
The risk tolerance is the required confidence level stated in the risk measure.

The risk tolerance is simply a parameter (or set of parameters) that links the risk measure,
as applied to the risk profile, to a single capital amount.

For example, a risk tolerance of 0.5% would set capital such that there is one chance in 200
that the balance sheet position at the end of the year shows a deficit.

We often consider the capital impact of each of the individual risk types described in this
chapter in isolation, as part of a more holistic exercise. In such cases, we would typically
define the risk profile using the specific risk’s contribution to the overall risk profile,
otherwise using the same risk measure and tolerance.

We now consider the individual risk types that contribute to a general insurer’s overall risk profile,
and how these may be modelled for capital purposes.

© IFE: 2019 Examinations The Actuarial Education Company


SP7-21: Capital modelling – assessment of capital for various risk types Page 9

2 Insurance risk

2.1 What is insurance risk?


In this section, we consider a typical way of modelling the capital impact of insurance risk.

We define insurance risk as the risk of loss arising from the inherent uncertainties about the
occurrence, amount and timing of insurance liabilities, expenses and premiums.

In earlier subjects, insurance risk is sometimes referred to as business risk.

It is normally divided between:

 underwriting risk (also known as premium risk), relating to risks yet to be written /
earned, and

 reserving risk, relating to risks already written / earned.

However, the precise delineation may vary; in particular, starting unearned premium reserve
(UPR) may appear in either part, depending on whether the dividing line is the time of
writing or of earning. Discounting and its inherent risk will play a vital role here (see
Section 2.6).

For example, suppose a motor insurance policy is set up on 31 March 2018 and cover begins on
10 April 2018. If we are modelling the required capital as at 1 April 2018, then starting UPR will
form part of the underwriting risk. However if we are modelling the required capital as at
10 April 2018, then starting UPR will form part of the reserving risk.

Insurance risk is mitigated by ceded reinsurance, so the capital requirement depends on the
net liabilities.

Insurance risk does not cover all possible events that might threaten the gross liabilities paid. In
particular:
 Any risk to the gross liabilities through non-collection of reinsurance falls under
credit risk rather than insurance risk.
Credit risk is discussed in Section 5.
 Any risk to the payment of gross liabilities through poor investment performance
falls under market or liquidity risk rather than insurance risk.
Market and liquidity risks are discussed in Sections 3 and 4 respectively.
 Finally, any risk to the writing of premiums or the settling of claims arising from
control failure, rather than from market uncertainties, falls under operational risk
rather than insurance risk.
Operational risk is discussed in Section 6.

The Actuarial Education Company © IFE: 2019 Examinations


Page 10 SP7-21: Capital modelling – assessment of capital for various risk types

2.2 Gross underwriting risk


In this part, we consider the capital impact of gross underwriting risk.

Gross here refers to underwriting risk gross of any reinsurance programmes.

Data required to measure underwriting risk


The insurer will need to project its underwriting result over the appropriate time horizon. The risk
is that actual experience is different from that expected. Recall that underwriting risk relates to
business that is yet to be written / earned, so an insurance company will need to estimate the
volumes and mix of business that it will write (or the earnings profile of business that it has
already written). The projection should be on a best estimate basis.

As a starting point, we can consider the firm’s business plan if this is prepared on a realistic
basis. If the firm uses an aspirational business plan for motivational purposes, this should
first be adjusted to a best estimate basis.

We should be able to support the loss and expense ratios for each class by reference to
historical performance, after adjusting for changes in rates, terms and conditions.

For a new class, or one with insufficient internal experience, we should be able to support
the assumptions by reference to market experience, after adjusting for any differences.

The capital requirement for the underwriting risk is the difference between:

 the underwriting result at the firm’s chosen level of risk tolerance for the business
written / earned during the modelled period, and

 the underwriting result on the realistic basis.

One way to apply a realistic basis would be to exclude any profit expected (deducting any
such baseline profit from the capital requirement as a separate item). It would then be
based on projected eventual results, that is, with best estimates of the ultimate cost of
claims.

Question

Without reading on, suggest categories of claim sizes that should be analysed separately.

Solution

The following claim sizes should be modelled separately:


 attritional claims
 large claims
 catastrophe claims
 future latent claims.

© IFE: 2019 Examinations The Actuarial Education Company


SP7-21: Capital modelling – assessment of capital for various risk types Page 11

Measuring underwriting risk


To determine the capital requirement for the underwriting risk at the chosen level of risk
tolerance, we should divide the firm’s business into classes / currencies / territories of
sufficient granularity (that is, small enough subdivisions) that we can consider distinctive
features of the class, but not so fine that statistical methods become invalid (because of
insufficient data in the subdivisions).

We should then assess the variability of its claims and expenses, either by fitting statistical
distributions or by simpler approaches such as stress tests.

Claims should generally be split into the following classes:


 attritional claims
 large claims
 catastrophe claims
 future latent claims.

Modelling attritional claims


We generally model attritional claims in aggregate. A mildly-skewed distribution such as
the lognormal may be appropriate, although we should test this against experience.

Methods of fitting and testing distributions were discussed earlier in the course.

If the standard deviation is a sufficiently small fraction of the mean, a normal distribution
may be an adequate approximation.

For classes that are small or not subject to large claims it may be more practical to model
loss ratios rather than separately model individual large claims.

Modelling large claims


Ideally, we should model large claims separately from attritional claims so that we can
determine reinsurance recoveries directly. (The dividing line between large and attritional
claims is often the firm’s typical retention for policies in the class.)

Question

Suggest why we would model large claims separately from attritional claims.

Solution

Loss ratios will be distorted – possibly significantly – by the existence of large claims. However for
classes that are small or not subject to large claims, the effect of this may be minimal.

We generally model large claims on a frequency-severity basis.

The Poisson and negative binomial distributions are often used for frequency. The Poisson
distribution is only appropriate where the claims are independent, since if there is any
correlation between claims, this distribution will underestimate the tail risk.

The Actuarial Education Company © IFE: 2019 Examinations


Page 12 SP7-21: Capital modelling – assessment of capital for various risk types

This was discussed earlier in the course.

If claims are independent and occur completely randomly, they may conform to a Poisson
process, in which case claim numbers would have a Poisson distribution. However, if there is any
correlation between claims, then a Poisson distribution may not be appropriate, as it will
underestimate the number of claims.

For severity, sampling from revalued past claim sizes is sometimes used, but this omits the
risk of a claim greater than experienced in the past, so it is preferable to fit a distribution. A
heavily-skewed distribution such as the Pareto would normally be appropriate for severity,
and we should derive it from or test it against historical data revalued to current claims
costs.

For both attritional and large claims, it will be necessary to consider if there are any types of
claims that are not present in the historical claims data used for parameterising the
distributions. These are often referred to as Events Not In Data (ENIDs) or, less commonly,
binary events. It may be appropriate to increase the standard deviation of the distributions
beyond that derived purely from historical data to allow for ENIDs. ENIDs could also be
modelled separately – this would be a more sophisticated approach, but the materiality of
the ENIDs should be borne in mind.

If we believe that there is a risk of large losses arising that is greater than those experienced in the
past, we should make an assumption about the likely severity and frequency of these, and ensure
that our fitted frequency and severity distributions allow for these adequately.

This assumption may be subjective due to the lack of detail in the historical data. However,
information may be available from underwriters, reinsurers, brokers etc.

Similarly, if the historical data includes unusually heavy experience, then it should either be
adjusted to reflect likely future experience, or the fitted distributions should reflect this.

Question

Suggest why past claim sizes might not reflect likely future experience.

Solution

There may be changes in:


 risk due to a change in the mix of underlying risks
 the method of distribution (ie sales channel)
 cover / policy terms and conditions
 underwriting strategy, eg policy acceptance
 claims handling strategy

© IFE: 2019 Examinations The Actuarial Education Company


SP7-21: Capital modelling – assessment of capital for various risk types Page 13

 claims inflation, which may depend on inflation of:


– prices
– earnings
– medical costs
– court awards
 new procedures / types of claims
 the level of reinsurance coverage
 environment, including:
– legislation / regulation
– advances in technology
– medical advances
– processes for building / repairing property.

Modelling catastrophe claims


We should model catastrophe-type claims separately from either attritional and large claims,
especially for events that may impact more than one class.

We often cannot model catastrophe events from the firm’s experience because of their
rarity.

Due to their different nature, natural, man-made (here called human-made) and terrorist-based
catastrophes may be modelled in different ways:
 For natural catastrophes such as earthquake or windstorm, or for terrorist attack, a
proprietary model can apply a set of simulated events to the firm’s exposure to
derive a distribution of possible costs.
Catastrophe modelling is discussed in detail in Subject SA3 and in Subject SP8.
It is the firm’s responsibility to ensure that the model is suitable; for example, by
allowing for demand surge, climate cycle, and so on, and to test the results against
the known impact of recent actual catastrophes and to resolve or adjust for any
discrepancy.
Demand surges reflect the basic economic reality of reduced supply and increased
demand following a natural catastrophe. In an insurance context, they can be defined as
the temporary increase in repair / mitigation costs as a result of the natural catastrophe,
eg an increase in rebuilding costs resulting from a shortage of building materials after an
earthquake. Demand surges were introduced earlier in the course.
 For human-made catastrophes other than terrorism, the firm is likely to have to
develop a bespoke model.
For example, for the effect of a severe recession on its creditor business, we might
assess the impacts of recessions of various depths, and then model the drivers or
indicators of recession to fit a distribution to these costs.

The Actuarial Education Company © IFE: 2019 Examinations


Page 14 SP7-21: Capital modelling – assessment of capital for various risk types

Alternatively, a subjective allowance could be made, without using a sophisticated model.


For example, the insurer could estimate the likelihood of a catastrophic event of a
particular size, eg a 1 in 20 year event of a $5m loss. The insurer could then build up a
catastrophe reserve over time, ie by setting aside $250,000 each year.

Modelling future latent claims


Finally, we may need to consider future latent claims as a separate risk.

As with catastrophe claims, insurers are unlikely to be able to model future latent claims based on
past experience.

Question

Explain why insurers are unlikely to be able to model future latent claims based on past
experience.

Solution

This is due to the heterogeneity of latent claim types, eg pollution claims exhibit very different
characteristics to asbestos claims, and future latent claims will probably exhibit very different
characteristics from these.

Instead a more approximate approach such as a subjective loading is likely to be used.

2.3 Gross reserving risk


In this part, we consider the capital impact of gross reserving risk.

Data required to measure reserving risk


The insurer will need to project its reserves over the appropriate time horizon. The risk is that
actual experience is different from that expected. Recall that reserving risk relates to business
that has already been written / earned, so the insurer should have more data available than for
estimating the underwriting risk. Again, the projection should be on a best estimate basis.

As a starting point, we can consider the firm’s actual reserves if these are prepared on a
best estimate basis. If the firm includes a significant reserve margin in its published
reserves, we should first remove this.

We should be able to support any assumptions made in the reserving, such as any initial
loss ratios used for Bornhuetter-Ferguson projections, by reference to earlier years’ results
or, if necessary, benchmarks, after adjusting for changes in rates, terms and conditions.

Other adjustments will also need to be made, as we outlined above.

© IFE: 2019 Examinations The Actuarial Education Company


SP7-21: Capital modelling – assessment of capital for various risk types Page 15

The capital impact of the reserving risk is the difference between:

 the eventual cost at the firm’s chosen level of risk tolerance of settling claims for the
business written / earned before the modelled period, and

 the current reserves on a best estimate basis.

Thus, it should not include any reserve margin. (Any reserve margin, if sufficiently
evidenced, may be added to the capital resources available to meet the capital requirement.)

Measuring reserving risk


To determine the capital impact of the reserving risk at the chosen level of risk tolerance,
we should divide the firm’s business into classes of sufficient granularity, but not so fine
that statistical methods become invalid.

This is analogous to the method used to measure underwriting risk.

This may be at the granularity used for reserving, but we need more data to assess
variability than to assess a best estimate, so it may be necessary to group classes of similar
reserve variability. We should then assess the variability in the firm’s claims settling, by
statistical techniques such as bootstrapping or the Mack method, or by simpler approaches
such as stress tests.

The bootstrapping and Mack methods were discussed in an earlier chapter.

Question

Suggest what the current reserves might not allow for.

Solution

The current reserves might not allow for:


 possible future variability owing to reserve shocks
 future latent claims affecting business already written.

Additional risks
We should consider whether sufficient reserve shocks have occurred within the period of
the reserve data to indicate possible future variability (ie whether ENIDs are present).
Where this is not the case, we could either adopt a greater variability than the historical
figure, or we could model an explicit shock such as a future Ogden rate change.

It is unlikely that future latent claims will be adequately represented, since these are
generally removed from claims data and reserved separately. Therefore, we should
consider latent claims separately, perhaps by a stress test in view of the difficulty in
modelling their risk with any precision.

A firm may choose to model material claims separately, eg a large immature catastrophe
event.

The Actuarial Education Company © IFE: 2019 Examinations


Page 16 SP7-21: Capital modelling – assessment of capital for various risk types

Reasons for this may include:

 the presence of the claim distorting the variability analysis for the affected lines of
business

 the reserving team may have already carried out specific analysis over the volatility
of the loss

 extreme uncertainty over the ultimate cost

 specific reinsurance arrangements

 better management visibility of the impact of the claim.

If no latent claims have yet emerged on business already written / earned, then we would expect
the methodologies used in estimating the underwriting risk and reserve risk for latent claims to be
consistent.

2.4 Net insurance risk


Gross insurance risk is made up of gross underwriting risk and gross reserving risk.

To convert the capital impact of the gross insurance risk to that for net insurance risk within
a class, we should deduct the reinsurance that can contractually be recovered on large and
catastrophe claims (and on attritional claims in the case of proportional covers and working
layers).

We should allow for disputes or exhaustion of cover, so it is preferable to calculate


reinsurance recoveries directly, rather than assume that historical net to gross ratios will
continue in the future.

Depending on materiality, a firm may model future recoveries explicitly based on actual or
expected reinsurance covers, but use a net to gross ratio approach for prior claims. Where
a reinsurance contract covers both future and prior business, we need to allow for any
reduction in cover due to prior reinsurance recoveries.

Calculating reinsurance recoveries directly allows disputes on individual claims to be taken


account of. Assuming historical net to gross ratios will ignore such disputes on individual claims.

Similarly, historical net to gross ratios will not allow for the point at which reinsurance cover on
future claims is exhausted when no further recoveries can be made. Again, a direct allowance for
reinsurance recoveries will obviate this problem.

However, we do not need to allow here for:

 reinsurance failure (falls under credit risk), or

 lack of cover through mis-purchase (falls under operational risk).

2.5 Aggregate insurance risk


Recall that underwriting and reserving risks should be determined for each class of business
separately. This must then be aggregated into an overall estimate of insurance risk.

To assess the capital impact of aggregate insurance risk, we should aggregate the capital
impact of the individual class underwriting and reserving risks in a way that leads to an
overall estimate of insurance risk at the firm’s chosen level of risk tolerance.

© IFE: 2019 Examinations The Actuarial Education Company


SP7-21: Capital modelling – assessment of capital for various risk types Page 17

In order to do this, the insurer must allow for any correlations within the individual estimates.
Correlations may exist:
 between different classes of business
 for business written in different policy years
 between underwriting and reserving risks.

We should give credit for independence between classes, but recognise any correlations
that may exist.

In other words, when aggregating risk, we should allow for diversification benefits and
accumulation effects.

Correlations between classes of business


We should be careful because some classes may display little or no correlation under
normal circumstances but become significantly correlated under the extreme stress of the
capital requirement-level risk (‘tail correlation’).

For example, private motor insurance and fidelity guarantee insurance may show little correlation
under normal circumstances. However, if the country moved into a severe recession, private
motor insurance might experience worse claims experience (eg more accidents occurring due to
society’s increased stress levels, increased theft and increased claim severity as policyholders try
to claim for higher (possibly fraudulent) amounts) and fidelity guarantee insurance might also
experience worse claims experience (eg if employees go to more desperate measures to ensure
their financial well-being). Therefore under this particular extreme stress scenario, there may be
some correlation.

It is preferable to base correlations on real factors, where these impact more than one class.
Examples are catastrophe events (underwriting risk), reserve shocks (reserving risk), or
wage / price inflation (both underwriting and reserving risk). This helps us to develop and
understand the model.

Where this is not practical, we may have to assess correlations from the firm’s historical or
market data (remembering that any smoothing may mask the true correlation) or using
judgement.

Correlations over policy years


We should correlate underwriting risks from adjacent policy years (or earned exposure), for
example when we use a multi-year model or when there are multi-year risks.

We should correlate reserving risks from different policy years where the statistical method
applied has not already done this or where the data contain insufficient correlating events.

It is often appropriate to model correlation between years on explicit drivers, but otherwise
we can choose correlation factors using judgement.

The Actuarial Education Company © IFE: 2019 Examinations


Page 18 SP7-21: Capital modelling – assessment of capital for various risk types

Correlations between risks


Finally, we should allow for correlation between underwriting risk and reserving risk. For
long-tailed lines, there tends to be high correlation between reserving risk for the more
recent years and underwriting risk for the year ahead, since under-reserving of more recent
years may lead to under-pricing of the year ahead and vice versa.

2.6 Other considerations


Other considerations affecting the assessment of the capital impact of insurance risk are:

 Underwriting cycle: the base model should reflect any expected movement of rates,
terms and conditions, premium volumes, and claims, with rising loss ratios in a
softening market. Where the trend is uncertain, the model should reflect this
through increased variability.

Question

Suggest how such variability might be allowed for in a model.

Solution

In a deterministic model, variability is allowed for through stress and scenario testing, ie the
stresses and scenarios to which the situation is subjected should reflect the possible variability of
assumptions.

In a stochastic model, variability of stochastically modelled parameters may be allowed for


through the specification of appropriate distributions of inputs. If the relevant inputs are not
modelled stochastically, then sensitivity testing may be used to assess variability.

In the case of the underwriting cycle, the assumptions subject to increased variability are
premium rates and volumes of business, and (to a lesser extent) claims.

 Parameter error: we should include a margin in the variability assumptions to allow


for less than full credibility of historical data. This is especially the case for any new
classes, or any rapid expansion of a class that might take it beyond the firm’s niche
sources of business or areas of underwriting expertise.
A rapidly expanding class may also be subject to different (ie more attractive) terms and
conditions, and may be attracting a different mix of policyholders than previously.
Therefore a credible set of relevant data may not yet be available.
 Reinsurance of unexpired risk: if a firm has bought reinsurance on a
losses-occurring basis rather than on a risks-attaching basis, we should either
model the gross cost of risks unexpired at the end of the modelling period, or allow
for possible additional costs in renewing cover, especially in a post-loss scenario.

© IFE: 2019 Examinations The Actuarial Education Company


SP7-21: Capital modelling – assessment of capital for various risk types Page 19

In other words, if reinsurance is purchased on a losses-occurring basis, then the insurer


will not yet have purchased reinsurance to cover the (future) period that is being
modelled. Therefore the capital model should either:
– assume that this reinsurance will not be bought, or
– allow for the likely cost of purchasing this reinsurance.

Question

Explain what is meant by ‘a post-loss scenario’.

Solution

Here, a post-loss scenario refers to a scenario in which losses have been higher than expected in
the last year, so that the reinsurer has paid out more than expected. In this case, the reinsurer
might be expected to increase its reinsurance premiums in the following year in order to recover
its losses.

 Multi-year policies: if a firm writes policies with multi-year risks or guaranteed future
premium rates, or if it makes other underwriting commitments whose risk runs
beyond the end of its modelling period, such as binders, we must model these so
that the firm’s insurance risk capital will suffice for all risks taken on during the
period.
Binders (more commonly known as binding authorities) are used in the London Market.
Binding authority agreements are contracts between two parties:
– the coverholder – a firm that is authorised by a Lloyd’s managing agent to enter
into contracts of insurance and/or issue insurance documentation on the behalf
of a syndicate it manages
– the underwriting members.
The agreements set out the scope of delegated authority and thus allow coverholders to
enter into contracts of insurance and to issue insurance documents on behalf of Lloyd’s
managing agents.
In addition to the two parties to the agreement, there are other parties involved,
including reinsurers, who reinsure some of the risks taken on by the underwriters, brokers
and third parties handling claims.
 Management actions: in a multi-year model, we may assume that following
underwriting losses the firm will increase rates, providing we can demonstrate that it
has responded to past losses in this way. However, we should allow in the model
for the lags before the losses become apparent, in deciding to increase rates and in
imposing those increases. We should be wary of assuming an increase following a
loss not shared by the market, since if the competitors are not also raising rates,
competitive pressures may prevent the firm from doing so.

The Actuarial Education Company © IFE: 2019 Examinations


Page 20 SP7-21: Capital modelling – assessment of capital for various risk types

 Discounting: unless the regulator has specified otherwise, it may be acceptable to


model using either discounted or undiscounted reserves. In the latter case the profit
after removing excess capital (including reserve margins) will be greater. So,
provided profit is treated separately, there should be no impact on insurance risk. In
other words, discounting does not add to insurance risk unless the insurer
anticipates profit prematurely as a result of discounting.
Undiscounted reserves will be higher than discounted reserves, so there is already an
implicit margin for prudence in the former. The resultant insurance capital (based on
undiscounted reserves) could therefore be too high and the free capital too low. When
the excess reserves are released, the profits declared will be greater (at this point) than if
discounted reserves had been held. As discussed in Section 2.2, we should deduct any
such baseline profit from the capital requirement as a separate item.
Alternatively, we could discount reserves, which would give a lower insurance capital
requirement, without any need to treat profit separately.

© IFE: 2019 Examinations The Actuarial Education Company


SP7-21: Capital modelling – assessment of capital for various risk types Page 21

3 Market risk
In this section, we consider the capital impact of market risk.

We define market risk as the risk that, as a result of market movements, a firm may be
exposed to fluctuations in the value of its assets or the income from its assets. This risk
exists to the extent that any movement in assets is not matched by a corresponding
movement in the liabilities.

Effectively, the risk can be divided into:


 the consequences of changes in asset values (this is the most obvious implication)
 the consequent changes in the value of the liabilities if these are valued on a
mark-to-market basis
 the consequences of an insurer not matching asset and liability cashflows.

Sources of general market risk include movements in:

 interest rates

 exchange rates

 equity prices

 property prices.

It is important to note that none of these sources of risk is independent of the others. For
example, fluctuations in interest rates often have an impact upon equity and currency
values and vice versa. Giving due consideration to these correlations is an important
aspect of the prudent management of market risk.

Question

Explain how a rise in interest rates affects equity and currency values.

Solution

Equity values

A rise in interest rates is likely to lead to:


 a higher cost of existing (variable rate) debt for companies
 a higher cost of raising future debt capital for companies (so that expansions and growth
become expensive)
 a slowdown in economic growth
 potentially more certainty about future inflation, which could reduce the demand for real
assets
 an increase in the discount rate used to discount dividends when valuing equities.

All of these effects will exert a downward pressure on equity prices.

The Actuarial Education Company © IFE: 2019 Examinations


Page 22 SP7-21: Capital modelling – assessment of capital for various risk types

Currency values

A rise in interest rates will make the domestic currency attractive to overseas investors, and
hence there is likely to be an immediate increase in demand for the domestic currency, and hence
its value should appreciate.

Note that in the long run, exchange rates are determined primarily by relative rates of inflation in
different countries, so the currency may be expected to depreciate over time to reflect this (rather
than the level of interest rates).

3.1 Factors to consider when modelling market risk


A firm should ensure that it considers all sources of market risk. Factors to consider when
modelling market risk include:

 changed market values of investments

 variation in interest rates and the effect on


– the market value of investments
– the value of liabilities, if discounting is applied

 level of investment income

 counterparty / issuer defaults (this might be addressed under credit risk)

 severe economic or market downturn or upturn leading to adverse interest rate


movements and/or equity market falls

 inadequate valuation of assets (this might be addressed under operational risk)

 currency movements

 the extent of any mismatch of assets and liabilities, including reinvestment risk.

3.2 Modelling assets


We need to decide what assets will be modelled and how they will be modelled. We can
choose:

 to model each asset individually

 to group similar assets, or

 to model a notional portfolio.

Modelling each asset individually


It can be very time consuming to model each asset individually and so it may be preferable
to group similar assets for modelling purposes.

Modelling asset groups


We may choose asset groups based on the class of asset, the term of the asset and the
currency in which the assets are denominated.

© IFE: 2019 Examinations The Actuarial Education Company


SP7-21: Capital modelling – assessment of capital for various risk types Page 23

Modelling notional portfolios


Notional portfolios may be appropriate where a firm holds assets that are widely spread or
has an investment policy that relies on standard assets; for example, that follow a defined
index. However, we may need to adjust the notional portfolio for specific attributes of the
firm’s portfolio; for example, where there is a concentration in a particular class, sector or
stock.

Aside from considering the current portfolio, we should also consider any freedom
investment managers have to move away from the benchmark position for tactical trading
reasons. This may be a source of extra risk that needs to be modelled depending on the
controls in place to guard against this risk.

3.3 Modelling market risk


A market risk capital charge is the amount of capital required to cover market risk.

A market risk capital charge allows for one or more events that cause asset values to fall in
relation to liabilities at the chosen level of confidence. Asset portfolios that are better
matched to the liabilities are less risky and will give rise to lower market risk charges.

It may not always be optimal for an insurer to try to match assets and liabilities. The existence of
additional capital and future premium income gives freedom to intentionally take an unmatched
position in the hope of achieving an additional return. The capital will be used to cover the
insurer’s mismatch risk.

We can model market risk by:

 simple stress testing, or

 using an economic scenario generator (ESGs).

We will discuss each of these approaches below.

Stress testing
A firm using a simple stress test approach might undertake the following types of tests:

 a rise in interest rates of W% leading to reduced asset values and changed value of
discounted liabilities (if the model discounts the liabilities)

 an X% fall in equity prices

 currencies depreciating against sterling by Y%

 a fall in property values by Z%

 a change in the spread of corporate bonds / yields.

In each case the degree of severity of the test will reflect the chosen confidence level.

Stress tests may be appropriate for insurers with standard investment portfolios. We may
also use stress tests to provide a sense check on the output of a more complicated market
risk model.

The Actuarial Education Company © IFE: 2019 Examinations


Page 24 SP7-21: Capital modelling – assessment of capital for various risk types

When carrying out stress testing, it is important to consider the relationships between risks.
For example, an exchange rate risk for a particular firm may be negligible if its assets and
liabilities are matched by currency under normal conditions. There may, however, be a
currency mismatch under stressed circumstances; that is, when a catastrophe causes
extreme losses in foreign currencies and that currency has devalued at the time the loss
crystallises.

Economic scenario generators (ESG)


A more complex capital model to determine a market risk capital charge is an integrated
capital model using an ESG. An ESG is a model that generates values for economic
variables (such as inflation, gilt yields and equity returns). It defines the forms the variables
may take and the relationships between them.

Capital modelling usually makes use of ‘real world’ economic scenarios that are based on past
data and reflect real world volatilities.

An alternative is to use ‘market consistent’ economic scenarios, although these may be preferred
for other (more short-term) uses, such as pricing. Market consistent scenario generators aim to
replicate the market, eg by replicating market option prices, and/or providing arbitrage-free
scenarios.

The inputs to the generator include current interest rates, economic growth, inflation and yields.
Each variable is modelled using specific modelling tools, eg Brownian motion for equity prices,
and the outputs include future possible values of each variable.

ESGs can be very complex. It is therefore important that the user understands the inputs
and ensures that the ESG has been calibrated to reflect the purpose. The ESG will give a
joint probability distribution of outcomes for the economic variables (for example, equity
returns, yield curve shifts, credit spread shifts, credit defaults and so on) and a point is
chosen from the distribution that reflects the desired confidence level. The point will have
been generated by a particular scenario.

Similar to any stochastic model, any point on the output distribution will reflect a particular
scenario simulated by the model. The precise scenario is not relevant. It is the size of the impact
(ie the size of the risk) at the required probability that is important.

We should scrutinise the output from an ESG for reasonableness. We can use some of the
following techniques to help gain an understanding of the output:

 Re-express the chosen confidence level scenario in terms of stress and scenario
type benchmarks: for example, an ESG might produce an equity fall of 26%, which
can be compared against the scenario fall of X% mentioned above.

 Examine the scenarios around the chosen scenario: for example, looking at the
twenty scenarios around the chosen confidence level to gain more understanding of
the factors driving the market risk charge.

© IFE: 2019 Examinations The Actuarial Education Company


SP7-21: Capital modelling – assessment of capital for various risk types Page 25

 Examine individual variables within the overall economic scenario: for example, it
may be that a firm’s balance sheet is immune to changes in one economic variable
but highly volatile to changes in another economic variable. We can identify the
variables to which the balance sheet is most sensitive.

 Examine the implied correlations within the output: ESG models have inherent
correlations between different variables. In some ESGs these are explicitly modelled
and can therefore be readily identified, in others we can infer them from the output
scenarios. We can see from the output, for example, whether equity falls are
positively correlated with yield falls.

ESGs are often viewed as superior as they use common drivers of market risk, such as
inflation.

You may recall the Wilkie model from your previous studies. It produces similar types of output
to the ESG models discussed above.

The Actuarial Education Company © IFE: 2019 Examinations


Page 26 SP7-21: Capital modelling – assessment of capital for various risk types

4 Liquidity risk
In this section, we consider the capital impact of liquidity risk. This is often a smaller risk
for general insurance companies, where relatively liquid assets are held to meet short-tailed
liabilities.

We define liquidity risk as the risk that a firm is unable to meet its obligations as they fall
due as a consequence of having a timing mismatch or a mismatch between assets and
liabilities. This risk is associated with managing timing relationships between assets and
liabilities.

Recall that market risk deals with the risk of the value of a firm’s assets not being sufficient to
meet its liabilities. In contrast, liquidity risk deals with timings.

4.1 Factors to consider when assessing liquidity risk


To assess the capital impact of liquidity risk, we must consider the extent of mismatch
between assets and liabilities and the amount of assets held in highly liquid, marketable
forms, should unexpected cashflows lead to a liquidity crunch.

When assessing liquidity risk, we should take account of the minimum level of free funds
that are required across the underwriting cycle, taking account of the time horizon used.

The insurer will want to hold this amount of capital in liquid assets at all times. It may want to
hold liquid assets in excess of this amount, for reasons outlined below.

We should assess any capital requirement for liquidity risk in conjunction with both
insurance risk and market risk and assess the impact that various stress and scenario tests,
or stochastic simulations may have on the cash position of the firm and its ability to pay
claims.

Insurance risk includes the risk of large claims (either individually, or at an aggregate or
catastrophe level). Certain scenarios that will be tested in determining the level of insurance risk
may therefore also lead to a liquidity risk. Similarly, scenarios that are tested in assessing market
risk may lead to liquidity risk. Therefore these risks should be analysed together.

We should consider liquidity risk arising from failures to forecast cashflow requirements
accurately. ‘Process’ weaknesses may also impact on cashflow; for example, poor credit
control and management of disputes could cause liquidity strains.

As for insurance and market risk, credit risk can also lead to liquidity risk. Credit risk is discussed
in the next section.

We should also consider the firm’s ability to manage unplanned changes in funding sources
(for example, banks and reinsurers), as well as changes in market conditions that may affect
its ability to liquidate assets promptly with minimal loss.

We should consider how the impact of a loss may affect liquidity. For example, following an
extreme loss there may be delays in collecting reinsurance recoveries or increased trust
fund requirements.

© IFE: 2019 Examinations The Actuarial Education Company


SP7-21: Capital modelling – assessment of capital for various risk types Page 27

4.2 Stress tests to determine liquidity risk


Examples of stress tests to determine the capital requirement of liquidity risk are:

 an increase in attritional claims

 a delay between a large loss and receipt of reinsurance recoveries

 a catastrophe loss occurring

 a reduced level of new business and the associated impact on the insurer’s ability to
pay claims.

Question

Suggest how a reduced level of new business could impact on the ability to pay claims.

Solution

The insurer may be using new business premiums to pay current liabilities, so that it can invest its
capital in longer-term assets that:
 provide a higher expected return, but
 are less liquid.

If new business volumes are lower than expected, then premium income may be insufficient to
meet current liabilities (ie claims, salary expenses etc), leaving the insurer with a portfolio of
illiquid assets with which to meet them.

The Actuarial Education Company © IFE: 2019 Examinations


Page 28 SP7-21: Capital modelling – assessment of capital for various risk types

5 Credit risk
In this section, we consider the capital impact of credit risk.

Credit risk is the risk of loss if another party fails to meet its financial obligations, or fails to
perform them in a timely fashion. It can also include downgrade impacts. Credit risk is
typically split into the following two categories:
1. investment credit risk: for example, from holdings of non-government bonds
2. counterparty credit risk: mainly reinsurance recoverables and, where material,
premium debtors, including pipeline premiums, and other balances with
intermediaries and banks.

5.1 Investment credit risk: modelling approach


When we use a stochastic modelling approach to assess investment credit risk, we should
consider:

 the probability of default by each counterparty, and

 the degree of default, that is, the loss when the default occurs.

It is worth noting that investment credit risk is sometimes included in market risk.

A typical basis for the probability of default is the financial strength rating of the
investments held from an external rating agency together with migration matrices that set
out the transitional probability of moving from one rating grade to another over a given
year. However, these probability matrices typically vary over an economic cycle, such that
the probability of default for a given rating will increase during an economic downturn. We
should consider this when selecting the probability of default matrix.

It may be possible to incorporate this using a state-dependent model.

The degree of default, ie the size of the loss will depend on the insurer’s exposure to each
counterparty.

In a model of investment credit risk, we should also consider:

 spread risk
Spread risk refers to the risk that credit spreads may not remain constant in the future.
For example, the yields on bonds issued by a particular company might be 2% above
risk-free yields at the moment, but these could go up to 5% or 10% above risk-free yields
in the future, which would mean that the market value of any bonds they had issued
would fall significantly.
 correlations between counterparties and between asset classes

 the likelihood of systemic financial risk

 potential correlations with the financial and economic modelling factors used in the
assessment of market risk.

Recall that systemic risk is the risk that cannot be diversified away. (In the past, the term
systematic risk has also been used to refer to this type of risk.)

© IFE: 2019 Examinations The Actuarial Education Company


SP7-21: Capital modelling – assessment of capital for various risk types Page 29

5.2 Counterparty credit risk: modelling approach


The most common form of counterparty credit risk arises from reinsurance default. An
insurer might differentiate between underwriting and reserving elements of reinsurance
credit risk. Reserve credit risk (relating to risks already written) is on a known event and
expected amount, whereas underwriting credit risk (insofar as this relates to policies yet to
be written / earned) is based on unknown future events. As such there may be greater
uncertainty in estimating reinsurance credit risk for underwriting risk than for reserving
risk.

This may seem confusing at first. Consider a claim payment on an existing policy. This is a liability
to the insurer. However the reinsurance recovery in respect of that claim is an asset. Therefore
the credit risk is in respect of an asset of the insurer.

For existing policies, details of the policy (and the reinsurance arrangements) are known, apart
from reinsurance recoveries on IBNR claims. Therefore the reserve credit risk is on a known asset.
Future policies are unknown and may depend on both the losses made on existing business
(which might affect the insurer’s ability to write more business) and the reinsurance terms that
will be offered. Reinsurance recoveries (and therefore credit risk) for underwriting risk is
therefore subject to a higher degree of uncertainty. Both of these will depend on gross losses.

When we assess the capital impact of investment credit risk, we should consider both the
probability of default and the loss when default occurs.

An assessment of reinsurance default risk should consider these too.

However, when we assess reinsurance default risk, we should also consider the increased
risk of reinsurance failure in extreme loss scenarios, since published ratings may either
reflect the current economic conditions or an average default rate, rather than the likelihood
of reinsurance default at the more extreme tails. In a model, we should correlate
reinsurance failure rates with large loss or catastrophe scenarios.

Where a firm has a significant concentration with an individual reinsurer, we should


consider whether the Value-at-Risk measure adequately allows for this concentration risk.

We should explicitly consider the duration of liabilities when considering the reinsurance
credit risk as there is a higher probability of default on a more distant recovery. We can
apply a probability of default based on the expected mean term of the liabilities, or model
explicitly the rating for each reinsurer (and how this might vary) until each reinsurance
recovery has been received. This is commonly known as a rating transition model.

The Institute and Faculty of Actuaries has issued guidance on the estimation of reinsurance bad
debt, which discusses this approach.

The comments also apply to balances held by intermediaries (eg brokers). However, the
shorter period over which premiums are held mean that in practice the capital risk is often
not as significant.

The Actuarial Education Company © IFE: 2019 Examinations


Page 30 SP7-21: Capital modelling – assessment of capital for various risk types

In a model of counterparty credit risk, we should also consider:

 correlation between reinsurers / intermediaries

 the loss when default occurs – potentially linked to the current financial strength
rating

 non-recoveries due to reinsurance disputes, and the extent to which this is


considered within operational risk

 any collateral held by the insurer in relation to specific counterparties – such as


letters of credit.

5.3 Stress and scenario tests


Even when we use a stochastic model, we need stress tests to validate the model for
reasonableness and to help with calibrating the assumptions.

In designing suitable scenarios it is important to consider:

 potential ‘ripple’ effects: for example, overlaps between adverse economic or


insurance scenarios and the behaviour of counterparties

 hidden costs of adverse credit scenarios: for example, extra costs of collection, or
delays reducing the present value of recoveries.

Examples of stress tests include:

 failure of the largest reinsurer

 any existing or possible future disputes relating to reinsurance contracts on a


pessimistic basis

 failure of the largest intermediary

 one notch downgrade of all reinsurers and the impact on the stochastic model

 default of the most significant corporate investment.

© IFE: 2019 Examinations The Actuarial Education Company


SP7-21: Capital modelling – assessment of capital for various risk types Page 31

6 Operational risk
In this section, we consider the capital impact of operational risk.

We define operational risk as the risk of loss resulting from inadequate or failed internal
processes, people and systems or from external events.

Question

Give some examples of inadequate or failed internal processes, people or systems that may be a
source of operational risk to an insurance company.

Solution

Examples of inadequate or failed internal processes, people or systems:


 mismanagement, for example:
– inappropriate actions of the board of directors / staff
– failure of appropriate management systems and controls
– administrative complexity
 data errors, for example inadequate, inaccurate or incomplete data
 inadequate risk control measures
 fraud.

6.1 Overlaps with other types of risk


Since people, processes and systems are important elements of each risk category, we will
model some operational risk as a part of each risk category.

An example of this is where an insurance fraud event is left within the data. This event may
lead to an increase in the volatility assumption used for insurance risk and this risk will
therefore already be allowed for as a part of the insurance risk charge.

It is therefore possible that we double count the capital required to cover these risks.

Two difficulties with operational risk are therefore:

 to judge which risks we have already allowed for appropriately so we can avoid
double counting, and

 to identify which risks have been omitted.

The operational risk capital charge will therefore contain a capital charge in respect of the
balance of operational risks not dealt with elsewhere. Where elements of operational risk
have been captured within other risk categories, it is good practice to identify and quantify
these.

The Actuarial Education Company © IFE: 2019 Examinations


Page 32 SP7-21: Capital modelling – assessment of capital for various risk types

6.2 Examples of operational risks


The following risks are examples of operational risk that we can examine to determine a
firm’s operational risk charge:

 Administration risk: The risk of error or failure associated with the administrative
aspects of the firm’s operations. This risk will depend on the extent to which the
firm’s administration is outsourced, the extent of centralised and decentralised
functions and the level of staff expertise.

 Compliance risk: The risk of non-adherence to legislative and internal firm


requirements. We might also consider the cost of implementing future regulatory
reforms.

 Event risk: The risk associated with the potential impact of significant events on the
operations of the firm, such as a financial system crisis, a major change in the fiscal
system or a natural disaster. Event risk is not intended to cover the firm’s
underwriting losses from such events as this would normally be covered within
insurance risk.

Question

Suggest what event risk might be intended to cover following a significant external event.

Solution

The failure of plans to recover from the external event. Staff will be put under more pressure (for
example processing a large number of claims in a short period of time), which will increase stress
levels and potentially lead to errors in claims handling / processing.

 Fraud risk: The risk associated with intentional misappropriation of funds,


undertaken with the objective of personal benefit at the expense of the firm.
Considerations include: the possibility of fraudulent acts occurring within the firm,
and the extent of controls which management have established to mitigate this risk.

 Governance risk: The risk associated with the board and/or senior management of
the firm not effectively performing their respective roles.

 Strategic risk: The risk arising from the inability to implement appropriate business
plans and strategies, make decisions, allocate resources or adapt to changes in the
business environment (strategic risk is discussed further in Section 8.1).

 Technological risk: The risk of error or failure associated with the technological
aspects of its operations. This risk refers to both the hardware systems and the
software utilised to run those systems. Cyber risk (any risk of financial loss,
disruption or damage to the reputation of an organisation from some sort of failure
of its information technology systems) might be included here or separately.

© IFE: 2019 Examinations The Actuarial Education Company


SP7-21: Capital modelling – assessment of capital for various risk types Page 33

 Pension scheme risk: The risk that the firm is required to make good any shortfall in
pension scheme assets relative to its liabilities. Considerations include:
– the extent to which extreme events might impact upon the pension scheme
funding position
– the financial consequences for the firm in such cases
– any commitment to fund or eliminate any existing pension scheme deficit.
For example, we may consider holding capital to the value of five years’
worth of contributions in excess of the standard contributions required to
fund the deficit of the pension plan.

6.3 Modelling operational risk


We should identify all material operational risk scenarios specific to the firm’s business.
We may do this by compiling and maintaining a list of all the risks that may affect the
operations of the firm, referred to as a ‘risk register’. We should consider these risks when
we derive an operational risk capital charge. A brainstorming session may also be
advisable.

Identification of operational risks requires considerable input from owners, senior management
and other individuals who have a detailed working knowledge of the operations of the business.
It is important to remember that while it is possible to develop computer models to analyse and
price operational risk, such models are only as good as the parameters input.

Operational risk can be difficult to assess and requires judgement and experience.
However, broad-brush measures are not generally accepted in determining operational risk;
for example, setting an operational risk capital charge to be a defined percentage of other
risk charges. This is because this does not demonstrate a thorough assessment of
operational risk.

Methods of modelling operational risks


We may use stochastic techniques in assessing the capital impact of operational risk.
While we may have limited data to determine the parameters, we can still model it
stochastically using simulations of operational risk losses. It will usually be modelled as
frequency / severity.

Alternatively, risks may be considered separately by personnel with the skills to appraise
such risks.

In other words, a deterministic scenario-based approach is more likely.

This inevitably requires them to make judgements about the degree of loss that each risk
may give rise to, the type of event that may cause the loss, and the frequency of such a loss
occurring. In addition, we may consider each loss gross and net of any mitigating controls.

For example, a firm may have Directors’ and Officers’ insurance cover against some
sources of governance risk crystallising. A firm might also have rigorous internal
processes that reduce the likelihood of an event occurring or reduce the loss to the firm if
the event did occur.

The Actuarial Education Company © IFE: 2019 Examinations


Page 34 SP7-21: Capital modelling – assessment of capital for various risk types

7 Group risk
In this section, we assess the capital requirement of group risk.

This may be included in operational risk rather than as a standalone risk group in some models.

We define group risk as the risk a firm experiences from being part of a group as opposed
to being a standalone entity. The size of the group risk will depend on the ownership
structure of the firm and how it is funded by the parent. Capital may not be a good mitigant
of group risk.

Capital is often provided by the parent company. Group risk could lead to group-wide losses,
which could lead to the parent company being unable to assist its subsidiaries. Hence a subsidiary
may not be able to rely on capital to mitigate group risk.

Furthermore, the crystallisation of group risk is likely to lead to catastrophic losses, as in the case
of the Worldcom and Enron collapses. It is therefore not feasible to hold enough capital to cover
these risks (even if the capital requirement could be accurately assessed).

A more effective mitigant would be the introduction of effective management controls, and an
enterprise risk management (ERM) policy. ERM is discussed briefly in Section 9. However, it is a
substantial (and growing) area of development.

7.1 Types of group risk

Reputational risk
If the parent shares part of or all of the firm’s name, there may be a large degree of
association. This might then give rise to reputational risk.

Group reinsurance risk


A potentially significant area of group risk relates to formal group reinsurance
arrangements. There is a risk that the arrangements are not subject to the same contractual
terms and conditions of reinsurance agreements available in the market place.

For example, if the reinsurance arrangements are subject to special (favourable) terms and
conditions, but the reinsurer withdraws these terms as a result of an issue with the parent
company (or another part of the group), then a particular subsidiary will have to obtain
reinsurance at (less favourable) market rates.

There may also be a concentrated credit risk.

Risks in centralised functions


Where firms rely on a group company to provide centralised functions, such as marketing
or accounting and actuarial support, we should consider any risks that result from these
arrangements.

For example, accounting errors occurring centrally can be perpetuated throughout the group,
which could lead to significant (even catastrophic) losses when the error is eventually identified.

© IFE: 2019 Examinations The Actuarial Education Company


SP7-21: Capital modelling – assessment of capital for various risk types Page 35

7.2 Modelling group risk


In deriving a group risk charge, we should consider the likelihood and financial
consequences of both insolvency and credit downgrading of the firm’s parent. For
example, the parent company may withdraw support from the firm.

This is especially true if the firm has intra-group borrowing facilities or funding
arrangements in place or if the insurance subsidiary is not a part of the group’s core activity
(often true of captives).

As with operational risk, a stochastic modelling approach is unlikely to be appropriate. A


scenario-based approach may be used in practice.

The Actuarial Education Company © IFE: 2019 Examinations


Page 36 SP7-21: Capital modelling – assessment of capital for various risk types

8 Other risks
The above types of risk are not exhaustive.

Different firms will have other risks for which capital is required in mitigation. Firms should
be able to assess whether this is the case using their risk register.

These risks are included within operational risk in some models.

Other possible risks are:

 strategic risk

 political risk

 pension risk may be modelled separately from operational risk

 impact of tax on results (though not strictly a risk in its own right).

In some cases, these could form part of the enterprise risk models and in other cases we
could assess them separately, depending on their overall impact.

We discuss each of these issues further below.

8.1 Strategic risk


Strategic risk arises from the inability to implement appropriate business plans and
strategies, make decisions, allocate resources or adapt to changes in the business
environment.

The firm should assess the prudence and appropriateness of its business strategy in the
context of the competitive and economic environment.

In assessing the assumptions, forecasting and projections, we should consider the


possibility of a fundamental market change, for example:

 greater competition

 a change in distribution methods

 new products

 a legislative change

 legal precedents.

We should review whether the reinsurance program is appropriate for the firm’s business
plan.

Where appropriate, we can assess this as part of a stochastic model by using the correct
range of assumptions and correlating the correct variables, but we can also assess this by
stress testing the variables.

8.2 Political risk


We define political risk as the risk of any political change that alters the expected outcome
and value of a given economic action by changing the probability of achieving business
objectives.

© IFE: 2019 Examinations The Actuarial Education Company


SP7-21: Capital modelling – assessment of capital for various risk types Page 37

Politics influences how markets operate. Often the most unpredictable economic events
are political in origin, the result of political unwillingness or inability to maintain a
consistent and predictable economic environment.

Political risks are particularly important for firms operating in emerging markets as the
potential for rapid political changes makes estimating loss reserves a moving target.

We normally use a stress testing approach to assess political risk. We would test how the
political event would affect loss development and new business revenues. Any stress test
should be based on a known example, such as the Iraq invasion of Kuwait in 1990.

This is because the use of a known example will ensure that impact data is likely to be available.
This is particularly important since the economic consequences are so hard to forecast.

The Actuarial Education Company © IFE: 2019 Examinations


Page 38 SP7-21: Capital modelling – assessment of capital for various risk types

9 Enterprise risk management (ERM)


Enterprise risk management (ERM) refers to the methods and processes used by
organisations to manage risks (or seize opportunities) related to the achievement of their
objectives.

Hence, ERM is used to manage operational risk.

A decision must be made as to whether risk should be managed at:


 the business unit level
 the group (or enterprise) level.

Some earlier subjects call this Integrated Risk Management, but the term ERM is more commonly
used in practice.

9.1 Managing risk at the business unit level


One approach to risk management would be for the parent company to determine its overall risk
appetite and to divide this up among the business units. Just as each business unit has its own
management team to run its business, the business unit management team manages the risks of
the business within the risk appetite they have been allocated.

As risk analysis involves allocation of capital to support the risks retained by each business unit,
this approach is likely to mean that the group is not making best use of its available capital.

It is clear that this approach makes no allowance for the benefits of diversification or pooling of
risks. A crude approach to allow for diversification would be simply to allow the risk appetites
allocated to the business units to add up to perhaps 130% or 150% of the group’s overall risk
appetite.

9.2 Managing risk at the enterprise level


A preferable approach is to establish the group risk management function as a major activity at
the enterprise level. The group can then impose similar risk assessment procedures on the
various business units, which will enable the results from the various models to be combined into
a risk assessment model at the entity level.

ERM provides a framework for risk management, which might contain the following steps:
(1) identify particular events or circumstances relevant to the organisation’s objectives
(risks and opportunities)
(2) assess their likelihood and size of impact
(3) determine a response strategy
(4) monitor progress.

By identifying and acting on risks and opportunities, business enterprises protect and
create value for their stakeholders, including owners, employees, customers, regulators and
society overall. This fits into capital modelling.

© IFE: 2019 Examinations The Actuarial Education Company


SP7-21: Capital modelling – assessment of capital for various risk types Page 39

By examining risk at group level, allowance can be made for pooling of risk, diversification and
economies of scale. This should prove to be the most capital-efficient way of managing risk and
should therefore reduce the capital charge required.

It will also give the group management insight into the areas with resulting undiversified risk
exposures, where the risks need to be transferred or capital set against them. This will be an
important feed into the business planning and capital allocation cycles.

ERM is evolving to address the needs of various stakeholders who want to understand the
broad range of risks facing complex organisations to ensure they are appropriately
managed. Regulators and debt rating agencies have increased their scrutiny on the risk
management processes of companies.

The Actuarial Education Company © IFE: 2019 Examinations


Page 40 SP7-21: Capital modelling – assessment of capital for various risk types

The chapter summary starts on the next page so that you can
keep all the chapter summaries together for revision purposes.

© IFE: 2019 Examinations The Actuarial Education Company


SP7-21: Capital modelling – assessment of capital for various risk types Page 41

Chapter 21 Summary
Capital
In order to fulfil its business objectives and to meet regulatory requirements, an insurance
firm (and indeed other financial services firms such as banks) must hold financial resources
that allow a high degree of confidence that it can meet its obligations to policyholders, and
possibly to other stakeholders in the business.

Available capital is the excess of an insurer’s financial resources over the value of its
liabilities. In banking, this is referred to as risk capital.

Required capital is how much capital an insurer needs. This will depend on the purpose of
the exercise.

Regulatory capital is the amount of capital the regulator deems it appropriate to hold in
order to protect policyholders’ interests. This is most commonly thought of in the context of
regulatory solvency capital. Rating agencies and investment analysts may also make use of
an insurer’s regulatory capital.

Economic capital is the amount of capital an insurer believes it should hold to fulfil its own
objectives. Economic capital requirements may be measured on a more realistic basis than
regulatory capital.

Capital models can be characterised by three core features:


 a risk profile, which is defined by the risks being modelled and the key outcome
being used to measure success or failure
 a risk measure, which links the outcome to the capital required to achieve that
outcome; it will be defined in terms of a required confidence level and a time
horizon
 risk tolerance, which is the required level of confidence stated in the risk measure.

Insurance risk
Insurance risk is defined as the risk of loss arising from the inherent uncertainties about the
occurrence, amount and timing of insurance liabilities, expenses and premiums. It is
normally divided between:
 underwriting risk, relating to risks yet to be written / earned
 reserving risk, relating to risks already written / earned.

Both of these should be calculated on a realistic basis, based on past data (if it exists),
adjusted for any differences that will apply to the future period under consideration.

Net insurance risk can be calculated by deducting ceded reinsurance costs (and premiums)
from gross insurance risk. When calculating aggregate insurance risk (across business
classes), allowance should be made for correlations.

The Actuarial Education Company © IFE: 2019 Examinations


Page 42 SP7-21: Capital modelling – assessment of capital for various risk types

Chapter 21 Summary continued


Insurance risk may also be affected by:
 the underwriting cycle
 parameter error
 reinsurance of unexpired risk
 multi-year policies
 management actions
 discounting.

Market risk
Market risk is defined as the risk that, as a result of market movements, a firm may be
exposed to fluctuations in investment markets. Sources of market risk include movements in
interest rates, exchange rates, equity prices and property prices.

Assets may be modelled individually, in groups of similar assets or by using a notional


portfolio.

Market risk can be modelled by simple stress testing or by using an ESG.

Liquidity risk
Liquidity risk is the potential that a firm is unable to meet its obligations as they fall due as a
consequence of having a timing mismatch between assets and liabilities.

In assessing liquidity, the insurer should consider all factors that affect the quantity of liquid
assets available to meet the liabilities. These factors might need to be analysed in
conjunction with other risks (eg insurance and market risks).

Credit risk
Credit risk refers to the risk of loss if another party fails to meet its financial obligations, or
fails to meet them in a timely fashion. It can also include downgrade impacts.

It is typically split into:


 investment credit risk
 counterparty credit risk.

In each case, it will be necessary to consider the probability of default for each counterparty
and the size of the loss given that the default event occurs.

It will be necessary to consider the different levels of risk in different environments,


eg recessions.

© IFE: 2019 Examinations The Actuarial Education Company


SP7-21: Capital modelling – assessment of capital for various risk types Page 43

Chapter 21 Summary continued


Stochastic models may be used to model credit risk, however stress tests will still be needed
to check the model for reasonableness and to help calibrate the assumptions.

Investment credit risk is sometimes included in market risk.

Operational risk
Operational risk is defined as the risk of loss resulting from inadequate or failed internal
processes, people and systems or from external events. Operational risks include risks
relating to administration, compliance, impact of events, fraud, governance, strategy,
technology and pension scheme provision.

Some operational risks will overlap with other categories of risk.

Due to its uncertain and subjective nature, operational risk is difficult to model and requires
a considerable amount of judgement.

Group risk
Group risk is defined as the risk a firm experiences from being part of a group as opposed to
being a standalone entity. The size of the group risk will depend on the ownership structure
of the firm and how it is funded by the parent. Capital may not be a good mitigant for group
risk.

Group risks might include: reputational risks, concentrated credit risks and risks arising from
centralised functions.

Other risks
Other risks include:
 strategic risk – these arise from the inability to implement appropriate business
plans and strategies, make decisions, allocate resources or adapt to changes
 political risk – any political changes that affect the probability of achieving business
objectives.

ERM refers to the methods and processes used by organisations to manage risks.

The Actuarial Education Company © IFE: 2019 Examinations


Page 44 SP7-21: Capital modelling – assessment of capital for various risk types

The practice questions start on the next page so that you can
keep the chapter summaries together for revision purposes.

© IFE: 2019 Examinations The Actuarial Education Company


SP7-21: Capital modelling – assessment of capital for various risk types Page 45

Chapter 21 Practice Questions


21.1 List the parties that might be interested in a general insurer’s solvency level.

21.2 List the factors a firm should consider when modelling market risk.

21.3 Discuss the three ways in which assets can be modelled.

21.4 (i) Define liquidity risk.

(ii) List the factors to consider when assessing liquidity risk.

21.5 (i) State, giving examples, three components that a risk measure should incorporate.

(ii) Define the terms risk profile and risk tolerance.

21.6 List the factors to consider when assessing:


(i) investment credit risk
(ii) counterparty credit risk.

21.7 Describe why a general insurer needs to hold regulatory capital and why it might hold more than
Exam style
the minimum required amount. [9]

21.8 State how the following should affect the modelling of insurance risk for the purposes of assessing
Exam style
capital requirements:
(a) Uncertainty surrounding the stage in (and future trend of) the underwriting cycle.
(b) Expansion of an insurance company into new areas.
(c) Regulation requiring undiscounted reserves to be held.
(d) A rating system whereby insurers review premiums every year to reflect experience in the
previous year. [5]

21.9 Describe how the following risks can lead to liquidity risk:
Exam style  insurance risk
 market risk
 credit risk
 operational risk. [8]

21.10 Describe how to model the following claims for capital modelling purposes:
Exam style  property damage motor claims
 a liability claim against a director of a large company
 earthquake claims
 claims made in a severe recession. [10]

The Actuarial Education Company © IFE: 2019 Examinations


Page 46 SP7-21: Capital modelling – assessment of capital for various risk types

21.11 A general insurer has written personal lines buildings insurance for a number of years. It currently
Exam style
reinsures this business on a losses-occurring basis. It is considering extending its product range to
cover small commercial buildings and is intending to offer a product that has premium guarantees
from year to year.

(i) Describe how the insurer will determine the insurance risk for this class of business. Your
answer should include a discussion of the data it will use. [8]

(ii) Describe the additional factors it should consider in determining the level of insurance risk
to which it will be exposed in respect of the new product. [5]
[Total 13]

© IFE: 2019 Examinations The Actuarial Education Company


SP7-21: Capital modelling – assessment of capital for various risk types Page 47

Chapter 21 Solutions
21.1 Parties include:
 the regulator
 tax authorities
 rating agencies
 investment analysts
 policyholders
 prospective policyholders
 shareholders
 prospective shareholders
 board of directors
 employees
 reinsurers
 brokers
 creditors
 the government.

21.2 Factors to consider include:


 changes in market values of investments
 variation in interest rates and the effect on the market value of investments
 level of investment income
 counterparty / issuer defaults, unless addressed under credit risk
 severe economic or market downturn or upturn leading to adverse interest rate
movements and/or equity market falls
 inadequate valuation of assets, if not addressed under operational risk
 currency movements
 the extent of any mismatch of assets and liabilities, including reinvestment risk.

The Actuarial Education Company © IFE: 2019 Examinations


Page 48 SP7-21: Capital modelling – assessment of capital for various risk types

21.3 Three ways to model assets

1. Model each asset individually.

This can be time consuming.

2. Model asset groups.

The groups will be chosen based on the class of asset, the term of the asset and the
currency in which the assets are denominated.

3. Model a notional portfolio.

The notional portfolio may be based on the long-term investment strategy of the firm
rather than the actual asset mix currently held by the insurer.

This can involve less computation than the other methods, and ensures that investment
and capital management decisions are not influenced by any short-term deviation from
the benchmark holding.

Consideration should be given to:


– specific attributes to the firm’s portfolio, eg concentrations of asset holdings
– the freedom of the investment managers to move away from their benchmarks –
this would increase the riskiness of the investment strategy and the consequent
capital charge required.

This method may be appropriate where the actual assets are widely spread, or the
investment strategy is based on indices.

21.4 (i) Liquidity risk

Liquidity risk is defined as the risk that a firm is unable to meet its obligations as they fall due as a
consequence of having a timing mismatch or a mismatch between assets and liabilities.

This risk is associated with the processes of managing timing relationships between assets and
liabilities.

(ii) Factors to consider when assessing liquidity risk

 the extent of mismatch between the assets and liabilities


 the amount of assets held in highly liquid forms
 the uncertainty of the liability cashflows
 the variation in the level of free assets over each stage of the underwriting cycle
 correlations with both insurance and market risk
 failures to forecast cashflow requirements accurately
 process weaknesses, such as poor credit control and poor management of disputes

© IFE: 2019 Examinations The Actuarial Education Company


SP7-21: Capital modelling – assessment of capital for various risk types Page 49

 ability to manage unplanned changes in funding sources


 severity of the scenarios used in any scenario testing, and correlations between variables
in each scenario

21.5 (i) Components that should be incorporated within a risk measure

A risk measure should incorporate:


 an actual measure …
… eg Value at Risk, balance sheet surplus
 a level of confidence that this is achieved, eg 99.5%
 a time horizon over which it must be achieved.

(ii) Risk profile and risk tolerance

The risk profile is defined by:


 the risks that have been modelled (and how they have been modelled) …
… which will include risks from business already written plus a finite period of new
business activity
 the key outcome used to measure success or failure …
… eg the profit over a selected time horizon, or the balance sheet position at the end of a
selected time horizon.

The risk tolerance is a parameter (or set of parameters) that links the risk measure, as applied to
the risk profile, to a single capital amount, eg a risk tolerance of 0.5% would set capital such that
there is a 1 in 200 chance that the risk measure is not met.

21.6 (i) Investment credit risk

 the probability of default, which will be reflected in:


– the financial strength rating of investments
– state-dependent migration matrices (which give the transitional probability of
moving from one rating grade to another over a given year)
 the size of the loss given that the default occurs …
 … which will depend on the degree of exposure to each counterparty
 spread risk
 correlations between counterparties and asset classes
 likelihood of systemic financial risk
 potential correlations with the financial and economic modelling factors used in the
assessment of market risk

The Actuarial Education Company © IFE: 2019 Examinations


Page 50 SP7-21: Capital modelling – assessment of capital for various risk types

(ii) Counterparty credit risk

For each counterparty, eg each reinsurer and broker, it will be necessary to consider:
 underwriting and reserving elements separately
 the probability of default under normal conditions
 the probability of default in different extreme scenarios
 the size of the loss given that the default occurs
 non-recoveries due to reinsurance / broker disputes, and the extent to which this is
considered within operational risk
 the duration of the long-tailed liabilities (ie there is more uncertainty for long-term
liabilities)
 any collateral held by the insurer in relation to specific counterparties, eg letters of credit.

Consideration should also be given:


 if there is significant exposure to an individual reinsurer (or other counterparty)
 to correlations between reinsurers / intermediaries, particularly where counterparties are
related (eg subsidiaries of the same parent group or syndicates at Lloyd’s).

21.7 A general insurer needs to hold regulatory capital in order to provide a more or less explicit
degree of comfort that the insurer can meet its liabilities to policyholders as and when they fall
due. In other words, a fundamental purpose of solvency capital is to protect policyholders’
interests. [1]

It may hold additional capital in order to:


 increase business volumes by providing a high degree of confidence that it can meet its
obligations to policyholders – a large solvency margin will give a greater degree of security
to policyholders than holding just that implied by the regulatory minimum [½]
 to avoid the risk that the available capital falls below the regulatory requirement, eg due
to a fall in asset values … [½]
 … which would hamper the firm’s business activities, eg the regulator might require that
the insurer: [½]
– closes to new business [½]
– purchases more reinsurance [½]
– changes its investment strategy [½]
 maintain or improve its credit rating … [½]
 … which may incur other benefits, such as:
– cheaper financing terms [½]
– reduced scrutiny from the regulator [½]

© IFE: 2019 Examinations The Actuarial Education Company


SP7-21: Capital modelling – assessment of capital for various risk types Page 51

 improve its standing in the market in the eyes of investment analysts … [½]
 … which might have implications for the trading of shares in the company (and so the
share price) [½]
 reduce the risk of having to call on shareholders or members for further finance if losses
are greater than expected [½]
 smooth dividends to shareholders, who prefer less volatile returns [½]
 finance new business strain and enable it to write more business [½]
 meet the requirements of other stakeholders, eg debt providers, whose interests may be
subordinated to those of policyholders [½]
 enable it to develop the business and take advantage of opportunities, eg: [½]
– launching new products [½]
– taking over other companies [½]
 enable it to undertake a more aggressive investment (and hence pricing) strategy with a
view to increasing returns and attracting higher volumes of business [½]
 manage unexpected cashflow mismatches, eg claim payments that cannot be met by
investment and premium income. [½]
[Maximum 9]

21.8 (a) Underwriting cycle  uncertainty

This uncertainty should be reflected through increased variability of premium rates, business
volumes and (to a lesser extent) claims. [½]

(b) Expansion of an insurance company into new areas

There is an increased risk of parameter error (due to a lack of relevant data), and this should be
reflected through increased variability in the input distributions. [½]

For both parts (i) and (ii), variability may be increased:


 by modelling more severe stress and scenario tests, or [½]
 by increasing the variability parameters in the probability distribution, depending on
whether the model is deterministic or stochastic. [½]

In doing so, correlations between the following parameters should be considered: [½]
 changes in terms and conditions [½]
 business volumes [½]
 claim frequency and severity (eg policyholders may try to get the most out of their
insurance in times of high premium rates) [½]
 changes in reinsurance costs and terms [½]
 changes in the probability of reinsurer default (which might be more likely following a
period of soft premium rates). [½]

The Actuarial Education Company © IFE: 2019 Examinations


Page 52 SP7-21: Capital modelling – assessment of capital for various risk types

(c) Regulation requiring undiscounted reserves to be held

As long as profits are treated separately, ie deducted from the capital requirement as a separate
item, there should be no impact on insurance risk capital. [½]

(d) A rating system whereby insurers review premiums every year to reflect experience in
the previous year

If a multi-year model is being used, then these premium rate changes should be allowed for,
although it may be necessary to take account of lags between fully recognising the total run-off
cost (and therefore profitability) of business in each year, and also in implementing the new
premium rates. [1]

Allowance should also be made for external influences on premiums, such as competitors’ rates
and regulation. [½]
[Maximum 5]

21.9 Insurance risk

Lower than expected levels of new business, or an unexpected mix of business, eg smaller policies
than expected, will result in lower premium income than expected. [½]

Premium income is often used to pay current liabilities so that a more long-term investment
strategy can be followed. [½]

A reduction in premium income can therefore lead to a lack of liquid assets with which to meet
current liabilities. This is a type of underwriting risk. [½]

Higher than expected claims can lead to liquidity issues if there are not enough liquid assets
available to pay the claims. This is a type of reserving risk. [½]

High claims may be due to:


 a higher than expected level of attritional claims [½]
 more large claims (or larger large claims) than expected [½]
 the existence of catastrophe claims [½]
 the emergence of latent claims. [½]

Market risk

If assets and liabilities are mismatched by term, then the insurer may face liquidity issues, since it
may not be able to liquidate sufficient assets in a timely manner (if insufficiently liquid assets are
held). In this case it may need to realise assets at a time when market conditions are
unfavourable. [½]

Similarly, if asset income does match liability outgo by term, but the income on the assets is
defaulted, then this may lead to liquidity issues. [½]

© IFE: 2019 Examinations The Actuarial Education Company


SP7-21: Capital modelling – assessment of capital for various risk types Page 53

Credit risk

Counterparties who may owe the insurer money may include:


 reinsurers, owing reinsurance recoveries [½]
 brokers, owing premium balances [½]
 other debtors. [½]

Each of these counterparties might:


 delay the payment of these monies [½]
 default on the payment of these monies. [½]

Each of these events could lead to a liquidity risk if the insurer was depending on such payments
to meet its current liabilities. [½]

Operational risk

A failure to forecast cash outgo and income requirements accurately could lead to an insufficient
level of liquid assets being held. [½]

Poor credit control processes could lead to higher defaults or later payments by third parties than
expected. [½]

Poor management of disputes could lead to money not being recovered by the insurer. [½]

A failure to manage unplanned changes in market conditions could also lead to counterparties
defaulting when the insurer did not expect it. [½]
[Maximum 8]

21.10 Property damage motor claims

These claims will be primarily (if not all) classed as attritional claims. [½]

Attritional claims should be based on past experience and modelled in aggregate. [½]

Ideally, a mildly-skewed distribution would be used, eg the lognormal distribution. [½]

However, a normal distribution may be acceptable if the standard deviation is sufficiently small
compared to the mean. [½]

If the class of business is so small that a distribution cannot be determined, then loss ratios may
be used. [½]

Past data will need to be adjusted so that it is appropriate, eg by adjusting for inflation of
mechanics’ wage levels. [½]

A liability claim against a director of a large company

This could potentially be a very large claim. [½]

Large claims are usually modelled on a frequency-severity basis, based on past experience. [½]

The Actuarial Education Company © IFE: 2019 Examinations


Page 54 SP7-21: Capital modelling – assessment of capital for various risk types

The frequency might be modelled using a Poisson distribution, assuming the claim is independent
of other claims. [½]

If there is anyone else involved against whom another claim could be made, eg another director
at the same company, then a Poisson distribution will underestimate claims, so allowance should
be made for this. [½]

The severity is likely to be modelled using a heavily-skewed distribution, such as the Pareto. [½]

Past data will need to be adjusted so that it is appropriate, eg by adjusting for changes in policy
terms and conditions. [½]

Earthquake claims

The firm’s own experience will not be appropriate because earthquakes occur too rarely and so
internal data will not be valid. [½]

A proprietary model might be used to apply a set of simulated events to the insurer’s exposure.
[½]

The model should be appropriate to the insurer’s exposure, for example by geographic location
and size / type of risks it insures. [½]

Allowance should be made for the likely demand surge following a catastrophe, eg the increased
cost of materials to effect repairs (due to the sudden increase in demand) and the increased
demand for insurance due to policyholders’ increased awareness of the need for insurance
following such an event. [½]

The model results must be tested against recent earthquakes. [½]

Claims made in a severe recession

The insurer might need to develop a bespoke model to determine the effects on different classes
of business, eg: [½]
 credit insurance might experience higher claim frequency, due to the increased risk of
debtor insolvency [½]
 fidelity guarantee insurance might experience higher claim rates due to the increased
incidence of fraud amongst employees. [½]

Allowance should be made for correlations, eg: [½]


 with other types of risk, such as:
– operational risk, which may be higher due to the increased risk of operational
mismanagement if internal redundancies lead to staff shortages [½]
– market risk, which may be higher due to a fall in investment markets and
probable increased volatility of investment returns during a recession [½]
 between different underwriting years, eg if the recession straddles more than one
underwriting year [½]
 between different types of losses, eg large and attritional. [½]

© IFE: 2019 Examinations The Actuarial Education Company


SP7-21: Capital modelling – assessment of capital for various risk types Page 55

It might be possible to develop a stochastic model, if sufficient historical data is available to model
the likely increase in frequency and severity of claims. [½]

However, a scenario-based approach might be more likely, and will allow for a subjective view of
the likely scenarios that might emerge in a recession. [½]
[Maximum 10]

21.11 (i) Determining insurance risk for a new class

Insurance risk is made up of underwriting risk and reserving risk. [½]

Underwriting risk

Gross underwriting risk will be based on a realistic assessment of the insurer’s business plan. [½]

Data on personal lines buildings insurance may not be relevant, so the insurer must ensure that
its assumptions are consistent with market experience, after adjusting for any differences such as
the target market being small commercial buildings. [1]

The underwriting risk is the difference between the actual underwriting result, and that expected
on a realistic basis. [½]

One way to apply a realistic basis would be to exclude any profit expected on the business from
this calculation and deduct it from the capital requirement as a separate item. [½]

The insurer will need to consider how to divide its business for modelling purposes, eg by size of
building, level of cover and territory. [½]

This may be constrained by the lack of relevant internal data, leading to a lower level of grouping
than is ideal. [½]

Most claims will be attritional claims, although there may be some large claims, depending on the
insurer’s typical retention for this new line of business. [½]

If the insurer limits the size of the risks taken on, then it should be able to limit the number of
large claims. [½]

In this case, it may be more practical to model loss ratios rather than separately model individual
large claims, at least until relevant internal data becomes available. [½]

Attritional claims should be modelled either using:


 a lognormal distribution, if enough data is available to set appropriate parameters [½]
 a normal distribution, if a low enough level of variability is expected over claim sizes [½]
 loss ratios. [½]

The insurer will probably decide to set a maximum size for an attritional claim as its typical
reinsurance retention for the class. [½]

Any large claims should be modelled separately, and reinsurance recoveries allowed for on a
frequency-severity basis. [½]

The Actuarial Education Company © IFE: 2019 Examinations


Page 56 SP7-21: Capital modelling – assessment of capital for various risk types

Catastrophe claims are possible for this class of business, eg natural disasters and terrorist
attacks. [½]

Catastrophe claims should be modelled using simulation, based on market catastrophe models.
[½]

However, other (man-made) catastrophes will have to be modelled on a bespoke basis, since
commercial models may not be available. [½]

Future latent claims are unlikely for this class, although some broad allowance may be made in
respect of these. [½]

If the insurer is using a multi-year model, then the underwriting risks from different policy years
must be adjusted to allow for correlations. [½]

Reserving risk

Gross reserving risk is in respect of business already written / earned, so this will be zero initially,
as no business will have been sold. [½]
[Maximum 8]

(ii) Additional factors affecting insurance risk

The insurer should also consider:


 the stage in the underwriting cycle [½]
 the certainty of future trends [½]
 parameter error [½]
 reinsurance [½]
 premium rate guarantees. [½]

The stage in the underwriting cycle and certainty of future trends may be quite uncertain for a
company entering the market, although this may depend on the quality and quantity of any
available data on the market, eg reinsurers’ data. [1]

Parameter error may be relatively high, since this is a new product for this insurer so it will not
have any credible data of its own. This will increase the level of insurance uncertainty
surrounding the product. [1]

If the insurer intends to reinsure this business (and it is likely that it will, since it poses new risks to
the insurer), and it intends to continue to use a losses-occurring basis, then it should either:
 model the gross cost of risks unexpired at the end of the modelling period [½]
 make allowance for possible additional costs in renewing cover, especially in a post-loss
scenario. [½]
Since future premium rates are (to some extent) guaranteed, the insurer must model the
guarantees so that its insurance risk capital will suffice for all risks taken on during the period, as
long as the guarantees can last for. [1]
[Maximum 5]

© IFE: 2019 Examinations The Actuarial Education Company


SP7-22: Capital modelling – allowance for diversification Page 1

Capital modelling –
allowance for diversification
Syllabus objectives
4.5 Explain some of the areas to consider when approaching a capital modelling exercise.

The Actuarial Education Company © IFE: 2019 Examinations


Page 2 SP7-22: Capital modelling – allowance for diversification

0 Introduction
Diversification effects arise because the various risks from a company’s operations are not
perfectly correlated.

Since risks are not perfectly correlated, the sum of the standalone capital requirements for each
risk will usually be greater than the capital requirement for all risks combined.

In other words, an adverse event (eg a catastrophic claim event) should not affect diversified
policies simultaneously (by definition), and so some credit can be given for the level of
diversification. This will result in a lower overall capital requirement. The allowance for
diversification will often be a material aspect of a capital calculation.

Correlations may arise because specific types of risks affect different segments of the
company’s overall portfolio. Correlations may also arise between the different types of
risks.

For example, during a recession we might expect:


 theft risk to increase (general crime rates might tend to increase in recessions)
 motor insurance claim sizes to reduce as drivers are less able to afford bigger, more
expensive vehicles.

These two risks are negatively correlated, ie in a recession, one risk will get worse, and the other
will improve.

Ideally, the correlations would be negative correlations. Perfect positive correlations will give rise
to accumulations of risk, which will require a higher capital charge.

Of course, there will be many, many other factors at play for a motor insurance portfolio, but
hopefully you can see how a recession can affect different risks in different ways, and hence a
diversification risk can arise.

While the ideal is to have negatively correlated risks, uncorrelated risks can also successfully lead
to diversification benefits.

As a further example, consider two risks, which we can assume to be uncorrelated: earthquakes in
Japan and hurricanes in California.
An insurance company can write up to $2m of written premium of property damage insurance
globally. If it writes $2m of property damage business in Japan, then it will be far less diversified
than if it writes $1m of property damage business in Japan and $1m in California.

In fact, even risks that are positively correlated can benefit from some diversification effects
(provided they are not perfectly positively correlated).

In this chapter, we will discuss the importance of allowing for an insurer’s level of diversification,
and the methods for incorporating this into a capital model.

The way we model and parameterise the various risks will inevitably affect how we quantify
the diversification effects.

© IFE: 2019 Examinations The Actuarial Education Company


SP7-22: Capital modelling – allowance for diversification Page 3

1 Importance of diversification in capital modelling


The importance of diversification varies, depending on the purpose of the specific
investigation.

Capital modelling investigations include:

 assessing solvency capital requirements

 allocating the capital held between classes, products or individual policies for:
– performance measurement
– pricing
– business planning and strategy setting

 reinsurance purchasing

 asset allocation studies

 studies of enterprise level risks such as credit risk and operational risk.

Capital modelling involves combining risks to give an overall capital requirement. However, it
could involve determining capital requirements at a number of different levels. For example, we
might be interested in the capital requirements of:
 a single policy
 a particular product, eg residential buildings insurance
 a class of business, eg property damage business
 the insurer’s whole portfolio of business.

The above investigations are now discussed.

1.1 Assessing solvency capital requirements


Capital modelling involves combining risks to give an overall capital requirement.

The assessment of solvency capital requirements is a ‘ground-up’ exercise and could be at:

 product level

 class of business level, or

 whole company portfolio level.

A ‘ground-up’ or ‘bottom-up’ exercise starts with individual components, which are subsequently
aggregated to produce an overall result.

In this context, solvency capital requirements would be determined for individual policies (or
groups of policies) and then aggregated to give solvency capital requirements at a product, class
or whole company level.

The assessment would also be net of reinsurance, with reinsurance being modelled either
explicitly or implicitly.

The Actuarial Education Company © IFE: 2019 Examinations


Page 4 SP7-22: Capital modelling – allowance for diversification

Modelling reinsurance explicitly would involve modelling gross claims and reinsurance
arrangements separately (and deducting reinsurance recoveries explicitly). An implicit allowance
could involve modelling net liabilities. Methods for incorporating diversification assumptions into
a capital model are discussed in Section 3.

Typically, we would expect the sum of the capital requirements for the various risks from
each portfolio segment assessed separately (other things being equal) to be more than the
capital requirement from all risks analysed together for the whole portfolio (the sum of the
p-th percentiles is more than the p-th percentile of the sum).

This is simply due to the diversification effect discussed above.

A company may be exposed to operational risks associated with its corporate group
structures and policies. These risks, which are likely to be targeted by the group’s
enterprise risk management process, may be a source of negative diversification effects,
that is, require additional capital to be held.

Recall that managing risk at an enterprise level requires group companies to take a more holistic
view to risk management, whereby the group identifies, assesses, controls, exploits, finances and
monitors all risks from all sources to all parts of the business.

It may be the case that different business units are exposed to perfectly correlated (or perhaps
the same) risks as each other, in which case there will be fewer diversification benefits.

Furthermore, the group may be set up so that risks are actually amplified as a result of being a
group (rather than many individual companies).

By way of example, consider a number of small companies. They are each at risk of fraud by
employees, but it is unlikely that every company will experience fraud at the same time.

Conversely, consider one big group company. This is exposed to fraud risk, and arguably more so
than the sum of the individual companies’ fraud risks, because one employee may have access to
the resources of the whole group.

In quantifying the diversification adjustment, we will need to make assumptions about the
extent of applicable correlations. To do this, we need to understand the various
assumptions underlying the modelling of the risks in each portfolio and how they could
interact when combined. This is necessary before we can assess the capital requirements
of the overall portfolio.

Correlation can occur between:

 risks in the same class – this is often called risk clash

 risks in different classes – this may be called class correlation.

The allowance for correlation within the stochastic model can either be implicit within the
modelling process or explicit through the use of correlation matrices.

An implicit allowance might involve reducing the variability assumption when fitting a
distribution. These are discussed further in Section 3.

© IFE: 2019 Examinations The Actuarial Education Company


SP7-22: Capital modelling – allowance for diversification Page 5

1.2 Capital allocation for performance measurement


Capital allocation for performance measurement is a ‘top-down’ exercise.

Question

Explain what is meant by a ‘top-down’ exercise and why it would be appropriate here.

Solution

A ‘top-down’ exercise is the opposite of a ground-up exercise. It involves starting with an overall
position and subsequently breaking it down into individual components.

In this context, the overall capital required would be modelled and the results then broken down
(ie allocated between) different classes of business or products.

If a ground-up approach was used, then capital would be calculated for each policy (or group of
policies), but since there would be no allowance for diversification benefits, capital would be
overstated for each policy (or group of policies) and so profits understated.

We will typically use the results from the capital assessment exercise to allocate the capital
held by the company down to individual classes of business and products for performance
measurement purposes.

So, we will start with an overall capital requirement (which was determined as part of a capital
assessment exercise and which allows for diversification effects) and we will allocate that capital
between different parts of the business (classes of business or products).

As the capital allocated includes a diversification credit, the diversification assumption for
each class of business in this case would be implicit. This depends on how much
diversification benefit is allocated to each class.

When we allocate the capital, we may use a different risk measure from that used in
assessing the capital requirement.

Recall that a risk measure is defined in terms of the desired impact of the capital on the outcome
considered, eg a balance sheet deficit might be the risk measure and capital may be set to achieve
a 99.5% probability that the balance sheet in one year shows a surplus of no less than zero.

For example, a group’s solvency capital requirement may be based on a target percentile in
the tail of the underlying aggregate loss distribution. But we may allocate the diversified
capital down to individual classes of business or products for a company in the group with
reference to a lower percentile, or with reference to various percentile-defined layers to
prevent over-allocation to catastrophe-type business.

The Actuarial Education Company © IFE: 2019 Examinations


Page 6 SP7-22: Capital modelling – allowance for diversification

Other methodologies used for allocating the capital held between classes of business or
products include:

 Marginal capital method (a ‘last in’ method): We allocate the capital with reference to
the marginal capital requirements of each segment.

 The Shapley method: This is an extension to the marginal capital method based on
game theory. We allocate the capital with reference to an average of the marginal
capital requirements, assuming that the class under consideration is added to the
overall portfolio first, second, third and so on.
In other words, for any one class of business, we calculate each capital amount that would
be required if we add the class to the overall portfolio first, second, third etc. We then
take the actual capital requirement as that average of these amounts.

Question

An insurer writes three different classes of business (A, B and C). It has used the marginal capital
method to allocate capital between the three classes and has performed the analysis in all
possible permutations. The results are as follows:

Allocated first Allocated second Allocated third Total


A – $7m B – $1m C – $2m $10m
A – $7m C – $1.5m B – $1.5m $10m
B – $5m A – $3m C – $2m $10m
B – $5m C – $1.5m A – $3.5m $10m
C – $4m A – $4.5m B – $1.5m $10m
C – $4m B – $2.5m A – $3.5m $10m
Calculate the capital that would be allocated to each class using the Shapley method.

Solution

The average of the marginal capital requirements is:


7  7  3  3.5  4.5  3.5
Class A:  4.75
6

1  1.5  5  5  1.5  2.5


Class B:  2.75
6

2  1.5  2  1.5  4  4
Class C:  2.5
6

Note: the Shapley method can be unworkable in practice. The number of scenarios that
needs to be run is the factorial of the number of classes. However, it can be used for a
small number of classes.

© IFE: 2019 Examinations The Actuarial Education Company


SP7-22: Capital modelling – allowance for diversification Page 7

In the question above, note that there were three classes of business, so that the marginal capital
method had to be applied six (ie three factorial) times.

In general, when we select the method for the allocation of capital across lines of business,
we should have regard to the use to which the results will be put and consider desirable
properties of the results, such as stability over time.

Other desirable properties include:


 representative of the risk
 not dependent on the order in which capital is allocated
 objective
 easy to understand, interpret and explain
 capable of independent verification.

There is not necessarily one method that is best suited in all cases. Typically, we would
compare the results from several methods of allocation. We would use our judgement when
recommending or setting the final allocation.

Note: a company may hold more capital than its capital model may suggest is needed.
Amongst other reasons, the company may require the excess capital to support its credit
rating or choose to hold surplus capital so that it can take advantage of business
opportunities in the future. With the publication of SCR ratios, there is now also market
pressure to be seen to have ratios within a certain range.

Question

State some other possible reasons why a company may hold more capital than its capital model
may suggest is needed.

Solution

 to increase business volumes by providing a high degree of confidence that it can meet its
obligations to policyholders
 to benefit from cheaper financing terms
 to improve its standing in the market in the eyes of investment analysts
 to reduce the risk of having to call on shareholders or members for further finance if
losses are greater than expected
 to smooth dividends to shareholders, who prefer less volatile returns
 to meet the requirements of other stakeholders, eg debt providers, whose interests may
be subordinated to those of policyholders
 to enable it to develop the business, eg launching new products or for financing the
takeover of a rival insurer
 to enable it to undertake a more aggressive investment (and hence pricing) strategy

The Actuarial Education Company © IFE: 2019 Examinations


Page 8 SP7-22: Capital modelling – allowance for diversification

 to manage unexpected cashflow mismatches, eg claim payments that cannot be met by


investment and premium income
 to comply with any regulatory requirements

Types of capital to allocate


In a capital modelling exercise, it is important to distinguish between the following:

 total capital

 economic capital

 excess capital.

The total capital available to a company is generally fixed at a single point in time. The
stochastic model will ascertain the amount of economic capital. The excess capital is the
balancing item.

So: economic capital + excess capital = total capital

Recall that economic capital is the amount of capital that a provider determines is appropriate to
hold given its assets, its liabilities, and its business objectives.

We will normally allocate economic capital to each class of business in proportion to its
contribution to the risk metric on a standalone basis.

In other words, we first calculate the required capital separately for each line of business, and
work out what proportion each of these is of the total. (Note that this does not yet allow for any
diversification benefits in the aggregate amount, it merely gives us the proportions we need to
allocate the capital between classes.)

We then calculate the aggregate capital requirement, allowing for any diversification benefits,
and allocate this to each class, according to the proportions calculated above.

We may allocate the excess capital between classes of business pro rata to its risk-based
capital or certain components of it, depending upon the purpose of the exercise.

1.3 Allowing for the cost of allocated capital for pricing


In determining a ‘technical price’ benchmark, we will consider how to allocate the capital to
a particular class of business or product down to the individual policies in a way that
reflects their relative risk profiles.

In other words, the total capital needs to be allocated across each individual policy in order to
determine the theoretical (or ‘technical’) price that should be charged for each policy.

This is non-trivial in the real world. Many issues come into play simultaneously, including:

 risk management, eg concentration limits

 large differences based on allocation methods

 longer-term strategic growth strategies etc.

© IFE: 2019 Examinations The Actuarial Education Company


SP7-22: Capital modelling – allowance for diversification Page 9

We should consider how the allocated capital compares to that needed on an underwriting
year basis, which may be subject to different diversification effects.

An underwriting year grouping of claims will combine all the claims relating to loss events that can
be attributed to all policies that commenced cover within a given calendar year, irrespective of
when they are actually reported or paid and irrespective of the year in which the incident actually
arose.

Question

Explain why capital modelled on an underwriting year basis could be subject to different
diversification effects than say capital modelled on an accident year basis.

Solution

Capital modelled on an underwriting year basis may be subject to different diversification effects
because claims in any one underwriting year cohort can arise from incidents that occur in
different accident years. They can therefore be subject to different (diversifying) claims
environments.

For expanding or contracting portfolios, the capital needed to support the reserves that
would be held until all claims from the specific underwriting year are fully paid may differ
from the reserve risk component of the allocated capital. This would be based on the size
of the total reserves brought forward.

So, the capital needed to support claims from a specific underwriting year will form just a part of
the total reserves. The total reserves will be the sum of the reserves for each underwriting year.
For a stable portfolio, each underwriting year reserve would form a similar portion of the total
reserve. However, for an expanding (contracting) portfolio, the most recent underwriting year
reserves will form a relatively large (small) portion.

Similar differences may arise with the underwriting risk component. Where material
differences arise, we should discuss with the underwriter and the management of the
company the implications of setting pricing loads with reference to the allocated capital.

Recall that the two types of insurance risk are the gross reserving risk and the gross underwriting
risk.

The effect from diversification in pricing would be partly offset by the cost of any excess
capital.

Starting from a well-diversified position, in order to diversify further, we may need to sell more
business. This will have offsetting effects on the capital requirements:
 higher business volumes will increase the amount of capital required, eg to meet new
business strain; typically, capital gets more and more expensive to raise, so the cost of
capital may increase
 higher business volumes should improve diversification, which will reduce the capital
required.

The Actuarial Education Company © IFE: 2019 Examinations


Page 10 SP7-22: Capital modelling – allowance for diversification

In determining a ‘technical price’ benchmark, we should consider if the relevant load for the
cost of capital should be based on the diversified or non-diversified capital requirement.

Allowing for diversification will reduce the capital required and so should reduce the premiums,
which will improve competitiveness. However, we cannot necessarily guarantee that the business
sold will be diverse, so it may not be prudent to allow fully (or at all) for this.

1.4 Capital modelling investigations for business planning / strategy setting


As a company will be looking to optimise the use of its capital, in any modelling
investigations, we should allow for:

 expected changes in the mix of business by class

 the marginal effects of adding new classes or products, and

 the effectiveness of the company’s reinsurance protection programme in reducing


the volatility of the retained risks.

In cases where new classes of business or products are being considered, we will
investigate the relevant additional solvency capital requirement and the corresponding
diversification effects on the overall company’s portfolio. We should consider carefully the
assumptions made for the new class. For a new class, we will need external data or prior
experience to parameterise the modelling assumptions.

For all these investigations, it will be important that we quantify the diversification effects as
accurately as possible and allow for interactions between classes and products as well as
economic and territorial correlations.

Question

Explain why accuracy is particularly important here.

Solution

In order to plan business strategies, we would want to have as realistic a picture as possible. If
diversification effects are not quantified accurately, then poor decisions may be made, which
could adversely affect the future direction of the company.

1.5 Capital modelling investigations for reinsurance purchasing


We often investigate the capital requirement of a particular class of business or product
alongside the relevant reinsurance protection requirement.

Holding capital and purchasing reinsurance are, to some extent, alternatives. An insurer with high
levels of free capital will be able to cope better with volatile claims experience, single large claims
or accumulations / catastrophes, and so will have less need for reinsurance.

When we advise a company on optimising its reinsurance protection programme, we must


consider the effects of different reinsurance strategies on the company’s overall solvency
and economic capital requirements.

© IFE: 2019 Examinations The Actuarial Education Company


SP7-22: Capital modelling – allowance for diversification Page 11

The modelling required will depend on the reinsurance that we are considering:

 If the reinsurance covers just single classes, we will need to choose the
assumptions within the modelling of the gross losses for that class so that we can
evaluate the reinsurance recoveries.

 If the reinsurance covers multiple classes, we will also need to consider how to
allocate the recoveries back to the individual contributing classes.

 For whole account reinsurance protections, it is again important to allocate the


recovery back to individual classes.

We will need to allocate reinsurance recoveries to each class in order to calculate the net
insurance capital for each class. However, for reinsurance that covers more than one class of
business, this can be a difficult exercise. For example, a whole account stop loss programme will
provide recoveries across every class of business, but the split of recoveries across each class will
depend on the gross losses of each class only after the retention limit has been breached.

A stochastic model will probably be of use here since it can generate individual claims (and
timings of claims) for each class of business, and the precise reinsurance programme can be
applied to each individual claim.

In addition to allocating the recoveries from these policies, we can also use the stochastic
model as a basis for allocating the reinsurance premium at a class level.

Depending on the type and amount of reinsurance purchased, this will change the
diversification of the retained (net) business. Drivers of such change will include:

 tail diversification (insurance risk)

 mix of risk by type (for example, less insurance risk but more credit and liquidity
risks).

By purchasing reinsurance, insurance risk (of the retained business) should be reduced, in
particular in the tails, ie reinsurance is likely to cover extreme risks. However, with reinsurance,
the insurer will be exposed to reinsurer default, hence credit risk may increase.

Question

Explain how reinsurance might increase liquidity risk.

Solution

Liquidity risk may arise as a result of having to pay reinsurance premiums.

It may also arise if reinsurance recoveries are not made immediately, so that the insurer has to
pay the full claim amount(s) some time before receiving recoveries from the reinsurer, despite the
fact that ultimately it will only be paying a part of them.

The Actuarial Education Company © IFE: 2019 Examinations


Page 12 SP7-22: Capital modelling – allowance for diversification

1.6 Asset allocation studies


The ESG used within the stochastic model will enable us to analyse the impact of different
asset allocations. The ESG will ensure the consistency of assumptions throughout the
modelling process.

Using the stochastic model, we can assess the impact of the company’s asset portfolio on
the overall capital requirement. The model will help us to consider questions such as:

 Do we have enough liquidity in our portfolio?

 Should we invest in bonds for longer-tail liabilities?

 What should the security profile of bonds be, given the liability profile and the
current investment environment? For example, how much government-backed, how
much can be AAA, AA and so on?

 Should we consider equities / property for the longest liabilities or the excess
capital?

ESG models can incorporate correlations between different variable, and can therefore allow for
any diversification benefits of the asset allocation.

1.7 Enterprise level risks


In addition to insurance risk, the company faces a range of other risks that we need to
consider within the stochastic model. We should consider risks such as liquidity, market,
credit and operational risks.

We should model each of these, either implicitly or, more likely, explicitly. We can then
assess the impact of each risk on the overall capital requirement. We then need to consider
how to allocate any change in the economic capital assessment as a result.

Correlation assumptions in a stochastic model should allow for the diversification effects of these
risks, for example, by an adjustment to the variability assumptions or by an explicit assumption to
the correlation matrix.

Alternatively, a deterministic capital model should investigate a range of scenarios in order to


assess the impact on capital of diversification between these risks.

© IFE: 2019 Examinations The Actuarial Education Company


SP7-22: Capital modelling – allowance for diversification Page 13

2 Setting diversification assumptions

2.1 Data required


Ideally when we estimate the effect of diversification, we would begin by gathering
appropriate historical data and applying statistical tests to measure correlation. However,
for many risks there may be insufficient historical data for analysis. Or where such data
exists, it may only be sufficient for us to assess a correlation around the means of the
distributions, but not sufficient for us to assess correlation around the tails of the
distributions.

When capital modelling, it’s normally the extreme events (ie the tails of the distribution) that are
of most interest, and yet, it’s those tails that we have most uncertainty about and least historical
data on.

2.2 Assessing correlations


Two alternative approaches are:
1. for us to consider causes and effects to assess the key drivers of correlation
2. to consider the likely scenarios in which the particular risks will occur at the same
time.

A cause and effect reasoning considers the underlying logical links between different events, using
for example, scientific theory. For example, the risk of a tidal wave of a certain magnitude can be
assessed using flood models, and then the effects of that tidal wave on the land can be assessed
using mapping techniques.

Much of the cause and effect reasoning could be derived from the use of catastrophe models.

We should consider correlations:

 within risk types (for example, considering the diversification between asset classes
when assessing market risk)

 between risk types (for example, applying an appropriate correlation between market
risk and insurance risk).

Correlation between two factors can vary from a full positive correlation (where high values
of one variable are always paired with high values of the other variable) to full negative
correlation (where high values of one variable are always paired with low values of the other
variable). There can also be different correlations in the tail of the distribution. This occurs
when the correlation in the lower part of the distribution is positive, but in the tail, it is
strongly positive.

Tail correlations (or tail dependencies) are discussed in the next section.

The Actuarial Education Company © IFE: 2019 Examinations


Page 14 SP7-22: Capital modelling – allowance for diversification

3 Implementing diversifications in a capital model


There are a number of ways in which we can allow for diversification effects in capital
modelling work.

These are as follows.

3.1 Linking assumptions


If two assumptions within a capital model are explicitly linked through formulae, this will
introduce an implicit correlation between them. An example would be to express a claims
inflation assumption as a margin above price inflation. This is often referred to as a ‘driver’
correlation.

Another example, within motor insurance, would be to link the inflation assumption for bodily
injury claims to an index of average earnings.

3.2 Implicit correlations


It is important to consider whether there are implicit correlations within a capital model and
assess the appropriateness of them.

For example, if the model includes a natural catastrophe event, this can give rise to
significant tail dependency between insurance results for different property classes of
business. It can also give rise to significant non-property losses; for example, workers’
compensation claims and life claims in the event of an earthquake.

By measuring the correlation of the results, we can validate whether this is consistent with
the explicit correlation assumptions we have made. This will enable us to assess the extent
of any such implicit dependencies.

Therefore we can test for the existence of implicit dependencies (that we may not have been
aware of) using the results of the model.

3.3 Explicit correlation between distributions


For some assumptions, it may be appropriate to apply an explicit correlation factor (or a
correlation matrix for multiple parameters) within a capital model.

Question

Give examples of explicit correlations that could be built into a capital model for a company that
sells only domestic household business.

Solution

Possible links might be:


 rebuilding cost inflation and sums insured for buildings insurance
 unemployment rates and theft claim rates for contents insurance
 retail price inflation and claim severities on contents insurance.

© IFE: 2019 Examinations The Actuarial Education Company


SP7-22: Capital modelling – allowance for diversification Page 15

3.4 Copulas
Simple correlation factors give rise to symmetric dependency structures. For some risks,
the correlation between them may vary in a more complex way (for example, having a
greater level of dependency in the tail of the distributions than around the mean).

In other words, the use of correlation factors may be too simplistic, as many risks are correlated in
quite a complex way. This is particularly true in the tails of distributions.

Suppose that we have two variables X and Y. Each of these variables will have a marginal
distribution, which can be described by the two (cumulative) distribution functions
FX (x)  P( X  x) and FY (y)  P(Y  y) . These describe the probability distributions of each
individual variable when we disregard information about the other variable.

However, if there is a statistical link between the two variables, we will only get the full picture for
how the variables behave if we consider the joint distribution function FXY (x , y)  P( X  x ,Y  y) .

A copula function (C) is a function that allows us to calculate the joint distribution function from
the values of the marginal distributions so that:

FXY (x , y)  C [FX (x), FY (y)]

A copula is a mathematical relationship between the individual distributions of random


variables and the joint distribution of those variables. Copulas allow us greater flexibility
when we model multiple dependencies than single correlation factors allow us.

In other words, copulas are statistical tools used to formulate a multivariate distribution in such a
way that various general types of dependence can be represented. Copulas can allow for more
general types of dependencies than a theoretical approach would.

In general terms, suppose we have two random variables with distribution functions FX ( x )
and FY (y ) . We can express the joint distribution function as a function of the individual
distributions using a third function C:

F ( x , y )  C(FX ( x ), FY (y )) .

If we define:

u  FX (x) and v  FY (y ) ,

we can define C as:

C(u,v )  F (F 1(u ), F 1(v ))

which gives

C(FX ( x ), FY (y ))  F (F 1(FX ( x )), F 1(FY (y )))  F ( x , y )

The function C(u,v ) is known as a copula.

The Actuarial Education Company © IFE: 2019 Examinations


Page 16 SP7-22: Capital modelling – allowance for diversification

Question

Derive the copula function, C(u,v) when the variables X and Y are statistically independent.

Solution

If X and Y are statistically independent, we have:

FXY (x , y)  P( X  x ,Y  y)
 P( X  x)P(Y  y)
 FX (x)FY (y)

Therefore:

FXY (x , y)  C [FX (x ), FY (y)]  FX (x)FY (y)

ie:

C (u, v)  uv

(This is known as the ‘product copula’.)

There are many different copula structures based around various probability distributions.
They differ primarily in the part of the distribution in which they apply the strongest
relationship between the two variables. One which is commonly used in insurance
applications is the Gumbel copula:

C(u,v )  exp[ (( lnu)a  ( lnv )a )1 a ]; a  1

The Gumbel copula gives a strong tail correlation, and is also non-symmetric, giving more
weight to the right of the distributions.

This is useful in a general insurance application since, for example, large losses within a class and
across different classes of business are often strongly correlated.

If you want to learn more about the statistical theory of copulas, there is no shortage of literature.
Copulas are discussed in more detail in Subject CS2.

3.5 Deterministic allowance for diversification


The above methods work well in a stochastic capital model. However, where we have
assessed risks using a deterministic or semi-stochastic method, we will still have to allow
for diversification. In this case, a common method is to apply the standard methodology for
summing variances of distributions:

Var  X  Y   Var  X   Var Y   2Cov  X ,Y 

We can use a correlation matrix to extend this method to more than two risks.

© IFE: 2019 Examinations The Actuarial Education Company


SP7-22: Capital modelling – allowance for diversification Page 17

Note: on this method, we assume that we can combine capital amounts in the same way as
standard deviations of distributions. However, where the capital amounts are extreme
percentiles in the tail of skewed probability distributions, this may not be mathematically
correct.

This goes back to the point made earlier about tail dependencies. The application of
mathematical approximations and theories to capital considerations is not an exact science.

The Actuarial Education Company © IFE: 2019 Examinations


Page 18 SP7-22: Capital modelling – allowance for diversification

4 Sensitivity and actuarial judgement

4.1 Sensitivity
In many cases, particularly when we allow for diversification between major risk groups,
diversification may give a significant reduction in the overall economic capital requirement.
Actual capital does not change as a result of the modelling. It is the allocation of capital
between economic capital and excess capital that changes.

In other words, it is the capital requirement that changes, not the total actual amount of capital
that an insurer has available.

It is therefore important to be aware of the sensitivity of the overall capital requirement to


the key correlation assumptions. By stress testing the correlation assumptions, we can try
to identify which diversification assumptions are the most important.

4.2 Actuarial judgement


Given the lack of historical data on correlations between risks under stressed situations, we
will have to apply our judgement when we set correlation assumptions.

We should have regard to:

 the purpose of the capital modelling work

 the available data (if any), and

 the sensitivity of the results to the level of diversification chosen.

In cases where the advice given may vary significantly depending upon the correlation
assumptions underlying the diversification adjustment, it is important that we communicate
the relevant uncertainty. It may be helpful that we provide results on alternative
assumptions, to assist the users of the capital modelling results to understand the financial
implications of deviations from the assumptions made.

© IFE: 2019 Examinations The Actuarial Education Company


SP7-22: Capital modelling – allowance for diversification Page 19

Chapter 22 Summary
Allowance for diversification in capital modelling depends on the purpose of the
investigation. These include:
 assessing solvency capital requirements – this is a ground-up exercise; we can
allocate capital using:
 a percentile method
 a marginal capital method
 the Shapley method
 a proportion method
 allocating the capital held between classes, products or individual policies for:
 performance measurement – this is a top-down exercise
 pricing – any pricing exercise should allow for the diversification benefits,
including diversification by underwriting year, but should allow for the
offsetting cost of excess capital, particularly for a growing business
 business planning and strategy setting – this should allow for diversification
effects of changes in the mix of business, and territorial correlations
 reinsurance purchasing – we should assess the impact of reinsurance on tail
diversification and the mix of risk by type
 asset allocation studies – an ESG model should allow for any diversification benefits
of the asset allocation strategy
 studies of enterprise level risks.

Types of correlation include risk clash and class correlation.

It is important to give credit for diversification of risks within a capital model.

In order to set diversification assumptions, we will need to apply statistical tests to historical
data to measure correlations. There may be a lack of historical data – particularly in the tails
of the distributions.

The Actuarial Education Company © IFE: 2019 Examinations


Page 20 SP7-22: Capital modelling – allowance for diversification

Chapter 22 Summary continued


When setting the diversification assumptions, we can use past data if available, or cause and
effect reasoning. To implement the assumption into the capital model, we can:
 link assumptions
 incorporate implicit correlations (allowing for any implicit assumptions already
incorporated in the model)
 incorporate explicit correlations between distributions (eg by using correlation
matrices)
 use more advanced modelling techniques such as copulas for stochastic models or by
summing variances for deterministic models.

We will need to exercise judgement and be aware of (and communicate) the sensitivity of our
results to our assumptions.

© IFE: 2019 Examinations The Actuarial Education Company


SP7-22: Capital modelling – allowance for diversification Page 21

Chapter 22 Practice Questions


22.1 Explain why a general insurer would want to hypothecate (ie allocate) capital between different
classes of business.

22.2 Define the following:


(i) risk clash
(ii) class correlation.

22.3 List ten possible capital modelling investigations. [5]


Exam style

The Actuarial Education Company © IFE: 2019 Examinations


Page 22 SP7-22: Capital modelling – allowance for diversification

The solutions start on the next page so that you can


separate the questions and solutions.

© IFE: 2019 Examinations The Actuarial Education Company


SP7-22: Capital modelling – allowance for diversification Page 23

Chapter 22 Solutions
22.1 This is needed to give realistic comparisons of the profitability of different classes.

If capital is not allocated accurately and as a result class X appears more profitable than class Y
then the company might, incorrectly, decide to expand the less profitable class (ie class X).

It also helps with pricing, enabling the actuary to make the correct allowance for investment
income within each class of business.

In general we would expect a higher return on capital for a long-tailed class than for a short-tailed
class.

22.2 (i) Risk clash is correlation that occurs between risks in the same class.

(ii) Class correlation refers to the correlation that occurs between risks in different classes.

22.3 Capital modelling investigations include:


 assessing capital requirements for:
– existing business (ie setting reserves) [½]
– a new product (ie ensuring that sufficient capital is available for product
development and covering new business strain) [½]
 assessing solvency capital requirements at a product level, class of business level or whole
account level [½]
 allocating the capital held between classes, products or individual policies for:
– performance measurement [½]
– pricing [½]
– business planning and strategy setting [½]
 reinsurance purchasing [½]
 the purchase of any other risk management instruments, eg insurance-linked securities[½]
 asset allocation studies [½]
 assessment of capital needed to cover different risks, eg insurance risk and market risk [½]
 assessment of minimum liquid capital requirement over all stages of the
insurance / economic cycles [½]
 studies of enterprise level risks such as credit risk and operational risk. [½]
[Maximum 5]

The Actuarial Education Company © IFE: 2019 Examinations


All study material produced by ActEd is copyright and is sold
for the exclusive use of the purchaser. The copyright is
owned by Institute and Faculty Education Limited, a
subsidiary of the Institute and Faculty of Actuaries.

Unless prior authority is granted by ActEd, you may not hire


out, lend, give out, sell, store or transmit electronically or
photocopy any part of the study material.

You must take care of your study material to ensure that it


is not used or copied by anybody else.

Legal action will be taken if these terms are infringed. In


addition, we may seek to take disciplinary action through
the profession or through your employer.

These conditions remain in force after you have finished


using the course.

The Actuarial Education Company © IFE: 2019 Examinations


SP7-23: Capital modelling – practical considerations Page 1

Capital modelling – practical


considerations
Syllabus objectives
4.6 Describe the practical considerations which should be borne in mind when
undertaking capital modelling.

The Actuarial Education Company © IFE: 2019 Examinations


Page 2 SP7-23: Capital modelling – practical considerations

0 Introduction
This short chapter builds slightly further on the material you will have learned so far in
Subject SP7 on capital modelling. A lot of this material should now be familiar to you from your
study of the previous three chapters, so you should be able to work through this chapter very
quickly.

This chapter discusses:


 the principles of proportionality and practicability, as they apply to capital modelling
 issues that may arise when conducting a capital modelling exercise, and how these can be
allowed for
 further considerations when designing a capital model.

© IFE: 2019 Examinations The Actuarial Education Company


SP7-23: Capital modelling – practical considerations Page 3

1 Proportionality and practicability


Before we commence a capital modelling project, we should consider the principles of
proportionality and practicability outlined in the relevant actuarial professional guidance.

Question

Define the principles of proportionality and practicability.

Solution

The principle of proportionality says that an action should be proportionate to the problem or
task in hand and need not go beyond what is necessary to achieve its objective.

The principle of practicability says that an action can only be required if practicable.

For many reasons, including limited resources, time constraints or materiality, a complex
stochastic model may not be the most appropriate solution.

Perhaps the most significant driver of the level of complexity appropriate to a model, will be the
model’s purpose.

Question

State, with reasons, how much complexity would be required in a capital model in the following
circumstances:
 a small company pricing direct motor business
 a large reinsurer reserving for its catastrophe excess of loss business.

Solution

A small company pricing direct motor business will not need much complexity. This class of
business is reasonably homogenous and predictable. A small company will also have resource
constraints. On the other hand, there are many rating factors in motor insurance, and a GLM
would be useful.

A large reinsurer reserving for its catastrophe excess of loss business will need reasonable levels
of complexity. The business is highly capital intensive due to its volatility. A large reinsurer
should have no problems resourcing the build of the model.

In addition, in any capital modelling work, some risks will be less material than others, and
lack of data may limit the level of detail or accuracy with which we may model other risks.
In such circumstances, we should use our professional judgement to decide how best to
approach the modelling exercise.

Some of these data issues are discussed in Section 2.4.

The Actuarial Education Company © IFE: 2019 Examinations


Page 4 SP7-23: Capital modelling – practical considerations

The size of reserves for a particular portfolio would usually be a good starting point for
deciding upon the level of detail for modelling the relevant capital requirements.

Other factors to consider, if applicable:

 large volumes of catastrophe business

 riskier investment portfolios

 complexity of reinsurance covers

 classes of business with losses linked to economic cycles

 credit or surety business etc.

In general, the bigger the reserves, the more detailed the model is likely to be. This complies with
the principle of proportionality.

© IFE: 2019 Examinations The Actuarial Education Company


SP7-23: Capital modelling – practical considerations Page 5

2 Practical issues in capital modelling


This section discusses a few of the practical issues involved in capital modelling. There is some
overlap with the material you will have studied in earlier chapters, so there is little added
explanation. You should be able to work through this section fairly quickly.

There is a wide range of issues that may arise when conducting a capital modelling
exercise.

2.1 Issues from product knowledge and underwriting practices


If we have a poor understanding of the product being modelled and the relevant
underwriting practices, this adversely affects our ability to model the risks in a portfolio and
consequently the reliability of our advice on the basis of such modelling.

Understanding the cover provided


We should work with underwriters, claims managers and marketing executives to
understand the cover provided by the policies in the portfolio being modelled.

We should also consider information regarding current and future claims and hence reserve
uncertainty; that is, we should consult the claims department regarding likely future court
precedents, possible legislative changes (such as changing Ogden rates) and economic
and social factors that may affect attitudes to claiming and the size of claims.

Understanding the level of risk


It may also help to understand the motivation of the potential buyers concerning the risks
they may be looking to transfer to the portfolio. This will help us decide upon the number of
parameters needed for modelling the risk of the portfolio.

Other information that may help us formulate a view of the risk profile of the portfolio being
modelled would be:

 the range of limits and sub-limits on offer, eg the maximum pay-outs under each policy,
or on any one item covered
 the limit and attachment profile of the portfolio
This is referring to business where the cover provided is a layer of cover between a lower
limit, also known as an attachment and an upper limit, also known as the limit.
 how underwriters select risks and decide upon the terms to offer in each case, as
well as underwriting guidelines

 the line size profile of the portfolio and how this is used to manage the overall risk in
the portfolio
Recall that the line size refers to the proportion of the risk taken by a particular insurer,
eg under a slip system, an insurer might sign up for a 50% line.
 risk appetite limits set by management

 the consistency and efficiency of the operating and underwriting controls.

The Actuarial Education Company © IFE: 2019 Examinations


Page 6 SP7-23: Capital modelling – practical considerations

By understanding the portfolio’s risk profile, this can help us to determine the materiality of
individual risks and correlations between these risks.

The pricing actuary supporting the relevant product could provide additional information on
the risks covered, how they may have changed over time and how they have been allowed
for in the pricing of the contract.

Because capital would typically support premiums and reserves held to ensure adequacy to
a certain target percentile within the distribution of potential outcomes, we should consider:

 the extent to which prices achieved or planned compare to the target technical price

 the granularity (how far we subdivide the business) and strength of reserves held.

By understanding how the technical price is calculated and the relevant assumptions, this
helps us to decide on the level of detail at which the underlying risks should be modelled.

The level of detail at which the underlying risks should be modelled is discussed further in
Section 2.5.

2.2 Issues associated with the business operating environment


Changes in the legal, regulatory and business operating environments can have a
significant impact on the capital requirements of a particular portfolio as they affect the
reliability of the past as a basis for estimating the future.

Regulatory bodies may extend the cover provided under a policy beyond that intended and
potentially beyond that stated in the contract wording.

For example, from time to time the regulators in the US state of Florida consider legislation
that would impact the terms and conditions which can be attached to an insurance contract.

In other words, the Florida insurance regulator may deem that policy terms, conditions and
exclusions on some types of reinsurance business will be invalid if not approved by the regulator.
The frequency and severity of claims on future business (and potential on business already
written) are likely to increase as a result.

In assessing the capital requirements of a portfolio of risks domiciled in Florida, we would


need to consider the potential effect from such regulation on both premium and reserving
risks.

Another example is where a state scheme is set up with an aggregate limit beyond which
claims are passed on to all authorised insurers / reinsurers operating in the relevant state or
market.

The stage in the insurance cycle may also have an impact on the capital requirements of an
insurer, or the method of determining the capital requirements. For example, at the bottom of
the insurance cycle, where premiums are low, it is more likely that the URR will be higher than the
UPR and so extra reserves (and hence capital) would be required.

In some lines of business, there may be significant additional risk associated with
inefficiencies of the relevant policy wordings.

An example here would be standard forms used by admitted insurers in the US versus
manuscript wordings often used in the E&S and London markets.

© IFE: 2019 Examinations The Actuarial Education Company


SP7-23: Capital modelling – practical considerations Page 7

An admitted insurer is one that is licensed to conduct business in a certain US state. Manuscript
wordings are bespoke, customer-specific contract wordings. E&S is an abbreviation for excess
and surplus lines insurance.

Excess and surplus lines insurance is a segment of the insurance market that allows
consumers to buy property and casualty insurance through the state-regulated insurance
market, where policyholders, agents, brokers and insurance companies all have the ability
to design specific insurance coverages and negotiate pricing based on the risks to be
secured.

A further issue, for larger companies, is the group / entity structure and how it should be
modelled. This will vary depending on regulatory reporting needs and fungibility of capital.

Fungible capital is capital that can be transferred between different legal entities within the
group.

2.3 Issues associated with the reinsurance programme


We could model the capital requirement of a particular portfolio on a gross basis with
explicit allowance for reinsurance, or on a net basis where we consider the liabilities net of
reinsurance; that is, we make an implicit allowance.

There are issues associated with modelling the reinsurance recoveries:

 The reinsurance treaty protecting a particular portfolio will typically be limited to a


certain number of reinstatements, some of which may be ‘free’ (that is, prepaid) or
paid. When we consider the risks in the tail of the relevant distribution, we must
consider the possibility of exhausting available reinsurance protection and the
contingent cost of any reinstatement premiums. This is much more difficult when
we model reinsurance implicitly. Aggregate reinsurance covers and parametric
triggers are now also becoming more common in some sectors, adding to the
complexity of modelling the risks in the tail of the relevant distribution.
A reinsurer will have incorporated the expected cost of ‘free’ reinstatements into the
original premium for the cover. In this sense, the reinstatements have been paid for in
advance.
 If the reinsurance treaty protecting a particular portfolio is not on a risks-attaching
basis, there will be an additional pricing risk associated with the cost of renewing
the treaty. We should consider the potential inability to renew a treaty, and the
credit risk associated with potential reinsurance recoveries.
We should consider in particular the situation following a catastrophe because,
following a large catastrophe, reinsurers are likely to have significant claims on their
resources and so are much more likely to be in a distressed situation. Large
catastrophes are also more likely to coincide with falling equity markets and so we
should allow for the correlation of these two effects (that is, tail dependency).
For example, following the terrorist attacks of 11 September 2001 on the Twin Towers in
New York, there was a significant decline in the US equity markets.

In general, it is better to model gross claims and reinsurance recoveries separately so that
we can accurately model reinsurance limits and so on, and the risk of reinsurer default. If
we only model net liabilities, it is very difficult to allow accurately for counterparty credit
risk and the net impact of varying claim experience.

The Actuarial Education Company © IFE: 2019 Examinations


Page 8 SP7-23: Capital modelling – practical considerations

2.4 Lack of data

Issues with internal data

Question

Without reading on, suggest circumstances that are likely to lead to a lack of internal data.

Solution

There is likely to be a lack of data when we are trying to model claims:


 in the tail of the distribution
 for complex products, where there may be no claims experience for particular risks
 for new products.

Typically, there will be insufficient internal data available to model adequately the various
types of risks and calibrate the relevant model parameters, including any interactions and
correlations. This applies particularly in the tail of the distribution.

We may need to make assumptions about the frequency and severity of claims beyond the
experience data available. For complex products, there may be no claims from particular
types of cover in the experience. For new products, there may be no internal data available
for modelling the relevant risks.

The experience of a particular portfolio or class of business may be dominated by a small


number of large contracts, which may warrant special treatment.

A multi-line insurer or reinsurer may experience clash risks from exposure to potential
losses from different parts of its portfolio that it may not be able to identify from the data
available. But it is more likely to benefit from diversification.

Question

Define clash risks.

Solution

Clash risks are risks in the same class of business.

Issues with other sources of data


Market exposure curves and increased limit factors that may be used to allocate losses
between layers and project loss experience to higher layers may not be appropriate for the
portfolio being modelled. The market exposure curves and increased limit factors would
typically reflect a market view of the underlying claim severity.

Note that market exposure curves are also known as original loss curves. These are discussed in
detail in Subject SP8.

© IFE: 2019 Examinations The Actuarial Education Company


SP7-23: Capital modelling – practical considerations Page 9

The level of detail in the exposure data of treaty reinsurers may not be sufficient to enable
us to model adequately the underlying risks. A similar situation could arise with lineslips
and other binding authorities.

Recall that proportional reinsurance is written by treaty and that the insurer and reinsurer share
the risks in pre-specified proportions. The reinsurer is unlikely to have access to data to the same
level of detail as the insurer, which will make it harder for it to assess the risks.

A lineslip is a facility under which underwriters delegate authority to accept a


predetermined share of certain coinsured risks on their companies’ behalf. The authority
may be exercised by the leading underwriter on behalf of the following underwriters; or it
may extend to the broker or some other agent authorised to act for all the underwriters.

So, similar to treaty reinsurers, the insurers who are not ‘lead underwriters’ may have more
limited access to data, since they will not have to conduct such a detailed rating exercise.

Approaches to dealing with a lack of data


Possible approaches to dealing with a lack of data include:

 The use of benchmark data, such as industry statistics can be useful to fill a gap in
the firm’s own internal data. However, we should note that industry data may not
match the firm’s own risk profile. For example, when we are seeking to model the
early claim experience of a start-up motor insurance portfolio, we may have paid
property damage information, but we might use industry-based claims run-off
patterns in order to incorporate bodily injury losses.
Another example is when using benchmark loss ratios from industry data. These may
need adjustment before use, as they may have been based on, for example:
– a different time period
– different classes of business or different levels / mix of cover
– different allowances for reinsurance or expenses.
 The data issues an actuary faces in a capital modelling exercise are likely to concern
the actuary working on the pricing and the reserving of the relevant portfolios. It
would therefore be useful that the capital modelling actuary discusses these issues,
and how they have been addressed, with the other actuaries in the company.

 By discussing the risk with department managers and non-actuarial technical


specialists from around the company, we will get a qualitative understanding of the
nature and extent of the risk faced. This understanding can help us to make an
informed initial parameterisation. An example of this would be scenarios such as
realistic disaster scenarios populated by underwriters for regulatory submissions or
internal risk management of exposures.

2.5 Level of detail in a capital model


A key consideration in setting the level of granularity of risk modelling (in addition to the
materiality of the risk and the availability of data) is the level of detail required by the
business. For example, if the firm would like to use the capital model to investigate the
change in risk and capital requirements caused by an investment in, say, a particular equity
sector, then it would be inadequate to model equity investments as a single asset category.

The Actuarial Education Company © IFE: 2019 Examinations


Page 10 SP7-23: Capital modelling – practical considerations

Another consideration in setting the level of granularity is the size of the models being built
and the time needed for them to run. We also need to allow time to review and test the
models.

This is consistent with the principle of practicability.

There is often a compromise between the granularity levels required in theory with those
used in practice. It may be appropriate for some approximations to take place, particularly
for some of the less material risks.

2.6 Sensitivity testing and communicating uncertainty


Given the large number of assumptions in a typical capital model, and the uncertainties
inherent in them, it is good practice to perform sensitivity tests to judge which parameters
are most critical. This can help to prioritise resources on investigating and improving the
model and assumptions.

Good practice may be achieved by following guidance over testing and communicating the
sensitivity of key assumptions and the uncertainty of the results.

TAS 100 discusses the importance of communicating uncertainty.

To ensure that the end users of the capital model are aware of its limitations, we should
discuss the key uncertainties in the main risks and the results of stress tests with senior
managers or the board. They can then take ultimate responsibility for the final choice of
these assumptions.

It is also important to understand who makes the decisions with regard to capital allocation;
that is, understand who our client is. This will assist us when we communicate the results
and give advice. The recipient could be an individual such as the chief actuary, chief risk
officer or chief financial officer. Alternatively, the recipient could be a committee made of a
group of people, each with different perspectives.

2.7 Checking the results


To give confidence that the results are correct, it can be helpful to compare the results with
an alternative method of risk and capital assessment, such as deterministic stress and
scenario tests on the key risks.

We should also build the model in such a way that it provides a breakdown of the overall
capital requirement in a sufficient level of detail to allow reasonableness checks of the
results.

With a stochastic model, there are two ways of providing such a risk breakdown:

 Firstly the model can report on each individual risk and subsets of risks to the same
level of confidence. This gives us information on the stresses applied and the
explicit or implicit correlations assumed.
Correlations were discussed in Chapter 22.

© IFE: 2019 Examinations The Actuarial Education Company


SP7-23: Capital modelling – practical considerations Page 11

 Secondly the model can provide a breakdown of the individual risk values in the
critical scenarios. This gives us an understanding of typical scenarios in which a
loss of the magnitude of the capital requirement occurs.
The overall required capital is a point chosen from the output distribution that reflects the
desired confidence level. This point will have been generated by a particular (random)
scenario.
We can look at the individual risk values generated in this scenario to give an indication of
the capital to be allocated to each risk type. As we discussed in Chapter 21, we will also
look at the scenarios generated around this chosen (critical) scenario, to gain a greater
understanding of the factors driving the overall capital requirement.

It is important to check that the results make sense in the real world to ensure that the
model is not giving spurious results.

It is usually more straightforward to check this by considering a detailed results breakdown,


rather than assessing the reasonableness of the overall capital requirement figure directly.

Where we have carried out a capital modelling exercise previously, it is good practice to
identify the reasons why the capital figure has changed from the last exercise, and explain
qualitatively and quantitatively the effect of each key change.

Producing a detailed analysis of change, or ‘walkthrough’, quantifying the impact of each


smaller change, using a series of model runs to build up to the new result could be one way
of achieving this. This can also be used to quantify the impact of changes in exposure data
versus parameter changes versus model changes.

More detail is provided in Chapter 20 on the validation of capital models.

The Actuarial Education Company © IFE: 2019 Examinations


Page 12 SP7-23: Capital modelling – practical considerations

3 Further considerations

3.1 Capital modelling for regulatory or internal use


We may use models for both:

 regulatory capital requirement calculations

 internal capital assessment.

Regulatory requirements may not be the same as those required by the business for
internal capital assessment. For example, the regulator may require more prudent
assumptions than best estimate (particularly where data is limited), or may want the
company to assess capital assuming that it closes to new business.

Where we use a model for regulatory purposes, there may be guidelines (such as those
issued by the EIOPA in Europe, the PRA and Lloyd’s in the UK) that may give constraints on
the assumptions used and level of detail in the model. In addition, there may also be
guidance issued by organisations overseeing the actuarial profession, which should also be
borne in mind.

Note that in the UK, the regulators only give guidelines and constraints, rather than completely
prescribed methods. EIOPA is the European Insurance and Occupational Pensions Authority, and
is an advisory body formed to support the stability of the European financial system, improve the
transparency of financial markets, and deal with policyholder protection.

Under Solvency II, the regulatory regime which governs capital requirements for insurance
companies in the European Union, capital requirements may be determined using a ‘standard
formula’, that allows for underwriting, market, credit default and operational risks. The
alternative is for an insurer to use an internal model of its risks. Internal models aim to create a
stochastic model that reflects the company’s own business and risk structure.

However, there are regulatory regimes (such as Solvency II) where the model has to satisfy
a ‘use test’; that is, the company must use the model to help manage the business, not
simply to produce numbers for regulatory purposes.

It follows that a company may need more than one model (or set of assumptions or both) for
a capital modelling exercise. However, it can be a useful check on the results to be able to
explain the difference between capital assessments on regulatory and internal bases.

© IFE: 2019 Examinations The Actuarial Education Company


SP7-23: Capital modelling – practical considerations Page 13

3.2 Using the model in the management of the business


We can use capital models throughout the business to inform management decision
making. Some key areas of the business in which we can use capital models are:

 reinsurance: optimising the purchase of reinsurance


this may involve deciding on the retention level that optimises the savings in reinsurance
premiums and the capital required
 investment: assessing the impact of a change in the investment mix

 pricing: assessing return on capital for pricing and performance measurement

 reserving: quantifying the uncertainty in claims reserves


the capital model may be used to give a range of outcomes around a deterministic best
estimate
 planning: comparing different plans in terms of their risks, not just expected profits

 strategy: assessing the risks and diversification benefit of new strategies


this could include assessing the capital implications of a proposed acquisition to
determine whether the capital diversification benefits from some risks will outweigh the
impact of increased aggregations in other risk areas
 risk management: identifying key risks and assessing the impact of mitigation.

3.3 Consistency of assumptions


It is important to ensure consistency (where appropriate) between assumptions used for
capital modelling and other areas of the business, such as pricing, reserving and planning.
Where these assumptions are material, it may be helpful to apply stress test scenarios to
understand the impact if the assumptions used in these other areas are not correct.

As with all actuarial work, it is important to monitor actual experience regularly and
compare it with the assumptions used, to ensure that we can understand any material
deviations, and improve the model or assumptions as required.

Again, none of this should be new to you, as it has been discussed in other subjects and in earlier
chapters of this course.

3.4 Documentation
Depending on the use of the capital modelling exercise, documentation can be a very
important part of the process. For example, if the model is used for regulatory purposes,
the documentation will need to be comprehensive, and will be used by the actuaries
working on behalf of the regulator. They will want to ensure all the key risks have been
identified and modelled and independently validated appropriately.

The Actuarial Education Company © IFE: 2019 Examinations


Page 14 SP7-23: Capital modelling – practical considerations

The chapter summary starts on the next page so that you can
keep all the chapter summaries together for revision purposes.

© IFE: 2019 Examinations The Actuarial Education Company


SP7-23: Capital modelling – practical considerations Page 15

Chapter 23 Summary
When developing a capital model, the actuary should consider:
 the principles of proportionality and practicability
 issues:
 from product knowledge and underwriting practices
 associated with the business operating environment
 associated with the reinsurance programme
 lack of data
 the level of detail required
 sensitivity testing and the communication of uncertainty
 how results will be checked.

Capital models may be used for regulatory or internal use. Consequently, an insurer is likely
to need more than one model (or set of assumptions) for modelling its capital requirements.

Capital models can be used when investigating:


 reinsurance
 investment
 pricing
 reserving
 planning
 strategy
 risk management.

It is important to ensure consistency between assumptions used for capital modelling and
other areas of the business.

Documentation is a very important part of the modelling process.

The Actuarial Education Company © IFE: 2019 Examinations


Page 16 SP7-23: Capital modelling – practical considerations

The practice questions start on the next page so that you can
keep the chapter summaries together for revision purposes.

© IFE: 2019 Examinations The Actuarial Education Company


SP7-23: Capital modelling – practical considerations Page 17

Chapter 23 Practice Questions


23.1 List the factors that will affect the level of detail or accuracy with which risks may be modelled.

23.2 Describe the factors that should be considered in setting the level of granularity when modelling
Exam style
capital requirements. [8]

The Actuarial Education Company © IFE: 2019 Examinations


Page 18 SP7-23: Capital modelling – practical considerations

The solutions start on the next page so that you can


separate the questions and solutions.

© IFE: 2019 Examinations The Actuarial Education Company


SP7-23: Capital modelling – practical considerations Page 19

Chapter 23 Solutions
23.1 Factors include:
 size of the reserves
 materiality of the risk, ie size of the risk
 extremity of the risk, ie frequency with which risk is expected to occur
 relevance of available data
 credibility of available data
 sensitivity of any proposed parameters
 risk management measures in place to deal with the risk
 regulatory constraints
 practical constraints
 level of detail required by the business, ie the purpose of the investigation
 acceptability of approximations for less material risks, ie the principle of proportionality
applies
 acceptable size of the model and the time needed for it to run, ie the principle of
practicability applies
 time needed to review and test the model.

23.2 Practical factors

The data must be divided so that the subdivisions are small enough to define distinctive features
of the subdivision, ie to avoid heterogeneity in the data cell … [½]

… but not so small that statistical methods become invalid (because of insufficient data in the
subdivisions). [½]

It is therefore necessary to consider how sufficient the available data is in terms of: [½]
 relevance [½]
 credibility [½]
 accuracy. [½]

While internal data is likely to be the most relevant, it might lack credibility, particularly in the
tails of distributions. [½]

Industry-wide data is likely to be far more credible, but will probably lack relevance. [½]

It may also contain errors, since the quality of industry-wide data depends heavily on the
contributors of the data. [½]

The complexity of the models available may also be a factor, with more sophisticated models
being better able to cope with finer subdivisions of data. [½]

The Actuarial Education Company © IFE: 2019 Examinations


Page 20 SP7-23: Capital modelling – practical considerations

Time may also be a constraint, in particular:


 the run time [½]
 the time needed to develop, review and test the models. [½]

Materiality of the risk

The materiality of the risk will also be a key determinant of the level of granularity. The more
material the risk, the more important it will be to subdivide the data as far as the available data
will allow. [1]

The materiality of the risk may depend on:


 the purpose of the capital modelling exercise [½]
 the level of detail required by the business. [½]

The level of detail may also be affected by any existing (or expected future) regulation. [½]

The level of granularity used in capital modelling is likely to be linked to the level of granularity
used in pricing, which in turn will have been affected by factors such as data and materiality. [½]

There is often a compromise between the granularity levels required in theory with that used in
practice. It may be appropriate for some approximations to take place, particularly for some of
the less material risks (ie the principle of proportionality applies). [1]
[Maximum 8]

© IFE: 2019 Examinations The Actuarial Education Company


SP7-23: Capital modelling – practical considerations Page 21

End of Part 5

What next?
1. Briefly review the key areas of Part 5 and/or re-read the summaries at the end of
Chapters 20 to 23.
2. Ensure you have attempted some of the Practice Questions at the end of each chapter in
Part 5. If you don’t have time to do them all, you could save the remainder for use as part
of your revision.
3. Attempt Assignment X5.

Time to consider …
… ‘revision and rehearsal’ products
ASET – This contains past exam papers with detailed solutions and explanations, plus lots of
comments about exam technique. Students have said:

‘The ASET packs were great and I would highly recommend them for everyone.’

‘The ASET Pack is great because it gives tips on how to start a question.’

You can find lots more information, including samples, on our website at www.ActEd.co.uk.

Buy online at www.ActEd.co.uk/estore

The Actuarial Education Company © IFE: 2019 Examinations


All study material produced by ActEd is copyright and is sold
for the exclusive use of the purchaser. The copyright is
owned by Institute and Faculty Education Limited, a
subsidiary of the Institute and Faculty of Actuaries.

Unless prior authority is granted by ActEd, you may not hire


out, lend, give out, sell, store or transmit electronically or
photocopy any part of the study material.

You must take care of your study material to ensure that it


is not used or copied by anybody else.

Legal action will be taken if these terms are infringed. In


addition, we may seek to take disciplinary action through
the profession or through your employer.

These conditions remain in force after you have finished


using the course.

The Actuarial Education Company © IFE: 2019 Examinations


SP7-24: Determining appropriate reinsurance Page 1

Determining appropriate
reinsurance
Syllabus objectives
5.3 Describe the factors influencing the choice of an appropriate reinsurance programme
for a general insurer.

5.4 Describe how to test the appropriateness of alternative reinsurance structures for a
general insurer.

5.5 Describe how reinsurance purchasing decisions might be impacted by capital


management considerations.

The Actuarial Education Company © IFE: 2019 Examinations


Page 2 SP7-24: Determining appropriate reinsurance

0 Introduction
This chapter builds on knowledge from Chapters 5 and 6.

Section 1 of this chapter explains the factors that will influence the amount and type of
reinsurance we should purchase for an insurance company. The factors influencing the
reinsurance programme depend on specific features of the insurance company and other
issues. Where those factors apply to the insurance company, this section suggests which
reinsurance classes may be appropriate.

Section 2 explains how reinsurance purchasing decisions might be impacted by capital


management considerations for an insurance company.

Section 3 explains how to determine the type and extent of reinsurance that should be purchased
by an insurer. This is illustrated by means of a case study in Section 4.

Section 5 explains how to test the appropriateness of alternative reinsurance structures for
an insurance company.

© IFE: 2019 Examinations The Actuarial Education Company


SP7-24: Determining appropriate reinsurance Page 3

1 Factors influencing the amount and type of reinsurance


The purpose of the insurance company is to accept risks in return for a premium. However, the
company needs to manage the portfolio of risks very carefully in order to achieve the right
balance of risk and return. In particular, the insurance company will try to minimise
concentrations of risk.

The insurance company will use reinsurance to help achieve diversification of risks and, by ceding
unwanted risks, achieve the correct balance of risk and return.

Question

Explain why insurers just don’t reinsure risks that they think will be unprofitable.

Solution

A poor risk, even if ceded, is still a poor risk. If the insurer ceded only these, the reinsurance
would either be unavailable or very expensive, or it may cause strained relationships between
cedant and reinsurer.

For many classes of business, underwriting and reinsurance are interwoven. For example,
if the insurer has plenty of reinsurance it can usually write larger risks. If there is no
reinsurance, the insurer needs to be much more careful and its freedom to write business is
restricted. How a general insurer actually determines appropriate reinsurance
arrangements is influenced by a number of factors.

We now discuss those factors.

1.1 Class of business


Along with the size of the free reserves (discussed in Section 1.2), this is the biggest factor
determining the type and amount of reinsurance required. The class directly indicates the
likely size, range and volatility of the risks. In particular, the claims characteristics of each
class of business are important in determining the most appropriate form and structure of a
reinsurance programme.

We discussed the claims characteristics of common classes of business in an earlier chapter. If


you can’t remember them we recommend you revise them now.

Size and range of risks


Where an insured risk is large but finite, the reinsurance cover needs to go only as far as
the sum insured (or fixed benefit) less the retained risk. Types of cover for which this is
usually the case include property damage, fixed benefit and financial loss business.

Surplus reinsurance is likely to be appropriate for cover for damage to large properties,
such as commercial property. For risks that can give rise to individual claims that are large
in comparison to the size of the insurer, several layers of risk XL reinsurance may be
required.

The Actuarial Education Company © IFE: 2019 Examinations


Page 4 SP7-24: Determining appropriate reinsurance

Surplus reinsurance is used for classes with large heterogeneous risks. It may be combined with
excess of loss cover to protect against very large claims.

For those liability types of cover where there is no clear limit on the size of claims, the top
layer of reinsurance cover should, ideally, be unlimited; but in practice, unless there are
legislative reasons, there are always upper limits.

Some types of liability business have unlimited cover, and in some countries it is possible to
choose maximum limits of indemnity.

Volatility of experience
For some classes, the overall claims experience may vary considerably from year to year.
Everything else being equal, an insurer would choose to use more reinsurance where there
is greater volatility.

If the volatility is largely in the size of individual claims then individual risk XL will be more
appropriate. Arguably then, if the volatility is more in the number of claims than their
individual size then a stop loss treaty or an aggregate XL, based on cause, would be
appropriate.

1.2 Size of free reserves


Free reserves are available to cover, amongst other things, the cost of excessive claims:
The larger the free reserves, the less the need for reinsurance. They are not there to
eliminate the need for reinsurance but will enable the insurer to have a higher retention
before reinsurance is needed. However, the purchase of reinsurance is likely to be more
cost effective than holding an excessively high level of free reserves.

This is because shareholders will require a return on their capital, which will have to be generated
largely from profit loadings within the premium rates. If you hold an excessively high level of free
reserves, this return will be much more difficult to achieve. Reinsurance may give you a better
deal.

With the arrival of Solvency II, the requirement to change claims provisions into technical
reserves and risk margins has changed the amount and calculation of free reserves.

For example, Solvency II may change the split of total reserves between technical provisions and
free reserves or the split between the statutory minimum and anything held over and above that.

1.3 Total premiums written


This is closely related to the size of the free reserves, as an insurer will usually require large
free reserves to support large volumes of business.

However, where the premiums written are large in comparison to the free reserves, the
insurer may choose to set up some quota share treaties in order to reduce its net written
premium relative to its free reserves.

Note also that a company writing a large amount of business is likely to have a bigger spread of
risks than a small company. This would reduce the relative variability of the experience, and
imply that less reinsurance (of all types) is required by large companies.

© IFE: 2019 Examinations The Actuarial Education Company


SP7-24: Determining appropriate reinsurance Page 5

1.4 Geographical regions in which the risks are located


Many perils relate only to particular geographic regions. For example, in buildings
insurance, subsidence and flood occur only in certain areas. Where such perils can lead to
large claims, the need for risk and aggregate XL reinsurance is greater for an insurer
exposed to those regions than for one that is not.

Similarly, there is a much greater need for catastrophe XL reinsurance for insurers who
write risks in the specific regions that are commonly exposed to hurricanes, typhoons or
earthquakes, eg European windstorms.

1.5 Accumulations of risk


The continued success of a general insurer depends on the operation of the principle of
spreading risks. The insurer should cover a balanced portfolio of risks. Accumulations of
risk occur where there is an unbalanced portfolio. There are several ways in which this may
happen, some of which are controllable.

In many respects accumulations pose more danger for an insurer than a single large risk, because
they are much harder to identify and therefore harder to reinsure. The total amounts involved
can also be very large.

For example, the risk posed by giving fire insurance to a single large building is very clear.
(Unwanted risk can be passed on through a surplus reinsurance treaty.) But how should an
insurance company cope with, for example, the risk that there will be many thousands of
employers’ liability claims emerging from all employees whose sight is impaired by spending too
long staring at computer screens?

Too much risk underwritten in one particular area


This is an extension of the points made in relation to the relevance of geographical region
to the need for reinsurance (see Section 1.4).

Where regional perils occur from any one event, any accumulations of risk increase the
maximum loss that may occur and hence the need for aggregate and/or catastrophe XL
cover. Quota share cover may also be used to spread the risk, possibly with a reciprocal
agreement relating to business in another independent region.

To determine the amount of reinsurance required, we should estimate the total probable
maximum loss under each type of event. We can then consider this alongside the other
matters that influence the need for reinsurance. We should also consider the exposure at
company or group level to avoid accumulations across several classes of business.

Even where there is no obvious specific regional peril, a regional accumulation of risk could
still lead to an accumulation of losses due to perils such as a local economic downturn,
aircraft crashes, explosions or fires.

The application of risk identification, risk quantification and risk management will be crucial
to actuarial best practice here.

The Actuarial Education Company © IFE: 2019 Examinations


Page 6 SP7-24: Determining appropriate reinsurance

Too much risk of one particular type or class has been underwritten
This danger is subtler in its occurrence as few causes would be expected to lead to
excessive claims throughout a whole class. However, such an accumulation can certainly
be a danger where there is an economic link to the claims; for example, in a class such as
mortgage indemnity guarantee cover.

The risk here is that house prices tumble and unemployment rises. The latter triggers defaults on
mortgage repayments and thus increases the number of repossessions, whilst lower house prices
increase the likelihood of the lender not being able to recover the outstanding debt.

Quota share and stop loss reinsurance, if available, may be appropriate ways of reducing
such risks.

Risks where claims may arise under different classes of business


Here the same risk generates different claims in different classes. For example, for marine
hull and liability classes of business, the full insurance of a tanker may give rise to several
types of claims if it goes aground or is in a collision. The hull will need repair, oil cargo will
be lost, crew members may be injured or killed, or oil spillage may pollute a coastline.

In such situations, clash cover or facultative excess of loss might be used to reduce the
risk. Clash cover is XL cover for liability business, obtained by insurers to limit their
exposure to the risk that one event gives rise to claims on more than one policy, where
otherwise the insurer might be liable for claims up to any retention limit for each individual
policy. It should be noted that some versions of clash cover will not be effective in the
above situation, where the cover applies to different policies on a liability account.

It will therefore be important to check the wording of any clash cover policy to ensure that the
cover meets the need of the insured.

Because clash cover is for liability business, the hull repair mentioned in the above example is for
repair to a third party’s hull. Any damage to the insured’s own hull would not normally be
covered by clash cover (but will undoubtedly be covered by other insurance).

In practice, the clash cover may be subsumed within reinsurance covering the whole book
of business.

Inward reinsurance
The same considerations apply to inward reinsurance, which must be considered together
with the direct business to determine the true accumulation.

A further problem exists, however, where the insurer has accepted retrocessions for which
it is not possible to determine the detailed background of the risks. Here the insurer may
not be aware of an accumulation of risk by region, or indeed may have accepted an
individual risk to which it is already exposed.

Retrocession is reinsurance of another company’s inward reinsurance liabilities. It may not be


possible for the insurer accepting the retrocession to find out the details of the original risk. It is
even possible that the insurer is already exposed to the original risk.

© IFE: 2019 Examinations The Actuarial Education Company


SP7-24: Determining appropriate reinsurance Page 7

1.6 Current market conditions


Irrespective of the profile of the insurance portfolio, we will still consider the prevailing
market conditions when purchasing reinsurance.

Availability of reinsurance
An insurer’s ability to obtain reinsurance of the desired type and the desired amount at an
acceptable price will depend on the willingness and capacity of the reinsurance market at
any time. Often insurers may find that they are only able to place a proportion of a
reinsurance layer. A related point is the price at which reinsurance can be placed.

Insurers have encountered availability and pricing issues in the traditional reinsurance
market following major catastrophic events, such as 9/11 or Hurricane Katrina.

Opportunities available to find coinsurers


Coinsurance is a method of sharing a risk among a number of direct insurers, each of which has a
separate direct contractual relationship with the insured and is, therefore, liable only for its own
contractual share of the total risk. The insured needs to make a separate claim to each coinsurer
in respect of its stated proportion of the risk. If any coinsurer defaults, the insured would not
receive a recovery for that part of the claim.

This contrasts with reinsurance where only the direct writer has a contractual relationship with
the insured. The insured is unlikely to even be aware of any reinsurance arrangements the direct
writer may have. If the reinsurer defaults, the insured would not be affected and would receive
the full claim amount, providing this does not lead to insolvency for the direct writer.

The term coinsurance is also used in certain excess of loss contracts to refer to the proportion of
claims retained by the cedant.

In theory, if very large risks are written, the insurer merely needs additional layers of
reinsurance. If the risk becomes extremely large, the proportion retained by the insurer can
become very small. In this situation the risk of default by a reinsurer may become too large
to bear. In practice, therefore, there comes a limit beyond which a large risk has to be
coinsured, provided suitable coinsurers can be found, rather than using proportional
reinsurance. Each coinsurer will then reinsure its share of the risk as appropriate. This is
the basis under which most of the London Market operates.

Question

An policyholder insures a large building they own with Company A, who then pass on 40% of the
risk to Company B.

(a) If the policyholder makes a claim for £15 million, describe the cashflows that would take
place.

(b) Explain how your answer would differ if the risk was coinsured with Company A (accepting
60%) and Company B (accepting 40%).

The Actuarial Education Company © IFE: 2019 Examinations


Page 8 SP7-24: Determining appropriate reinsurance

Solution

(a) Company A would pay the policyholder £15 million and Company B would pay Company A
£6 million.

(b) Company A would pay the policyholder £9 million and Company B would pay the
policyholder £6 million.

Perceived value for money of additional reinsurance


Provided that the reinsurance is adequately priced, the purchase of it is likely to mean the
passing on of potential profits. However, this cost has to be weighed against the value of
the increased stability and security that the reinsurance may provide.

Note that exactly the same factor is considered when a policyholder is considering the purchase
of insurance. That is, a risk-averse policyholder is willing to contribute to an insurer’s profit for
the sake of the financial security and stability that the policy gives.

Security status of the available reinsurers


Reinsurance is a relatively high-risk business and so the possibility that a reinsurer
becomes insolvent should not be overlooked by a ceding insurer. The cedant is interested
in the long-term survival of its reinsurers, because of the long period that may elapse
between the occurrence of an insured event and the eventual recovery from reinsurers.
Thus, insurers will be influenced by the rating agencies’ views on the security status of
reinsurers.

However, reinsurers with a higher default risk may be cheaper than those with a better
credit rating, which needs to be considered when deciding which quotes to accept.

Diversification is also important here. As far as is practical, direct writers should use many
different reinsurers, eg for the different layers of surplus or excess of loss reinsurance. In any
case, the direct writer should, as much as possible, manage the risks they write prudently and try
not to be over-reliant on reinsurance.

Regulatory environment
There are various ways that the regulators may influence the use of reinsurance.

Regulatory authorities may reduce any required statutory solvency levels to allow for the
beneficial impact of the reinsurance asset purchased. We can use quota share reinsurance
to reduce the net written premium income of general insurers that have capacity restrictions
(for example, Lloyd’s syndicates). The extent of any reduction to statutory capital
requirements is likely to be limited in proportionate and/or absolute terms.

© IFE: 2019 Examinations The Actuarial Education Company


SP7-24: Determining appropriate reinsurance Page 9

Question

Suggest why there is often a limit to the reduction for the use of the reinsurance.

Solution

To allow for the possibility of reinsurer default, in which case the responsibility for meeting the
gross claim payments would revert to the insurer.

Regulators might also specify a maximum level of credit that can be taken for exposure to
any one reinsurer, perhaps based on the reinsurer's financial strength.

Alternatively, the insurer may be obliged to demonstrate to regulatory authorities that it has
adequate reinsurance. Or the regulator may even prescribe the use (or type) of reinsurance that
must be used.

In some regimes there are tax advantages in purchasing reinsurance.

1.7 Insurers’ preferences


Given two insurers with identical portfolios, there will still be differences in their
reinsurance requirements.

Risk appetite and tolerance


As for individuals, insurance companies will have differing opinions on the amount of risk
that they are willing to take. A more cautious insurer will be likely to cede more risk away to
reinsurers than a company adopting a less risk-averse strategy.

An insurer that has more financial backing, whether it is through a parent company or easy
access to additional funds, will have a higher level of risk tolerance.

Question

Suggest which you think would need more reinsurance, a proprietary insurer (ie owned by
shareholders) or a mutual.

Solution

It’s not that simple.

Whilst a proprietary company has easier access to further capital following poor claims
experience (via a rights issue) than a mutual, shareholders prefer smooth dividends, which rely on
smooth profits, which are more likely to be achieved with appropriate reinsurance in place.

The Actuarial Education Company © IFE: 2019 Examinations


Page 10 SP7-24: Determining appropriate reinsurance

Underwriter influences
Company underwriters may influence how much is spent on reinsurance. For example, an
underwriter given the aim of growing his or her company’s commercial property portfolio by
10% per annum, may decide to take on a particularly large risk and thus influence the
reinsurance purchases by the need to arrange corresponding facultative cover.

An underwriter with influence over the reinsurance purchase may be more or less risk
averse than would normally be appropriate. There may also be occasions when the
underwriter is influenced by more personal motives, such as protecting his or her bonus or
job.

A general insurance company should ensure that appropriate underwriting guidelines are in place
to control the risk profile of each underwriter’s portfolio. The reinsurance strategy should then
be written in accordance with this target risk profile.

The need for technical assistance


Reinsurers can provide assistance for reserving, pricing and establishment, if a company is
considering a new product launch or the development of business for the first time in a
different territory. However, in the UK at least, much of this assistance is provided by
specialist reinsurance brokers, and not by the reinsurers themselves.

The reinsurer or broker could also, for example, provide advice on product design, underwriting
and claims control.

Financial objectives
Financial objectives can be a strong motivator in reinsurance purchases. By purchasing an
efficient reinsurance programme, we can use reinsurance as a means of achieving certain
financial objectives. For example, an insurance company may have an overall business
objective of reducing its probability of ruin in the next 10 years to below 0.5%. By carefully
modelling the insurer’s business, we should be able to structure its reinsurance so as to
minimise its exposure to claims aggregations or catastrophic losses that could ruin it.

When discussed in mathematical terms, this sort of objective is called ‘Ruin Theory’. You may be
familiar with this from your previous studies.

Alternatively, suppose a company has the objective of providing its shareholders with a
smooth return over a five-year period. Then we could use non-proportional reinsurance of
the more volatile lines such as liability to help it to achieve this.

1.8 Expectations of the reinsurer


Often reinsurers will require the insurer to have a minimum level of retention of a risk or
collection of risks. They may achieve this by providing surplus reinsurance or by requiring
the insurer to retain a layer, or proportion of a layer, of an excess of loss programme. This
will help to keep the interests of the cedant and reinsurer aligned.

By requiring the cedant to retain a share of each risk, the direct writer will have an incentive to
maintain tighter underwriting standards and claim handling procedures, thus improving the
claims experience of the reinsurer.

© IFE: 2019 Examinations The Actuarial Education Company


SP7-24: Determining appropriate reinsurance Page 11

In practice, the insurer will always be expected to operate to protect the position of the
reinsurer.

So, for example, an insurer would be expected to continue to handle claims in a rigorous manner,
even if it is expecting to receive full recoveries for those claims from its reinsurers. In other
words, the principle of uberrima fides works both ways.

1.9 Relationships
A general insurer may also consider its existing relationships with reinsurers and reinsurance
brokers. For example, an existing reinsurer may offer a good price in order to maintain a good
relationship with its clients.

The Actuarial Education Company © IFE: 2019 Examinations


Page 12 SP7-24: Determining appropriate reinsurance

2 Impact of capital management on reinsurance purchasing


This section builds on the comments in Section 1.2, which discussed the use of capital (or free
reserves) as an alternative to reinsurance.

An insurance company’s reinsurance strategy will largely be dictated by its capital


management policy. For example, an insurer with access to cheap capital in its group is
less likely to take out reinsurance compared to a small company with no parental backing.

Insurers can change the amount of capital that regulators require them to put aside, by
purchasing reinsurance. More so now with the arrival of Solvency II in Europe, the interplay
of capital and reinsurance has become an important consideration in reinsurance
structuring and buying decisions.

2.1 Using reinsurance to reduce capital requirements


An insurer may wish to purchase (additional) reinsurance to reduce the amount of capital it
needs to set aside to meet regulatory requirements.

Regulators will require insurers, on writing a book of business, to allocate capital against
this business to cover future losses.

Through an effective reinsurance strategy, we reduce the amount of capital required to


support this book because the insurer is transferring exposure to the risk of losses to the
reinsurer.

2.2 Using reinsurance as a form of capital


Following the previous point that reinsurance can be used to reduce the amount of capital
required, we can consider it to be a form of capital. By purchasing an effective reinsurance
programme, an insurer can free up capital that is otherwise allocated to support the
premium income against future uncertain losses. Therefore, reinsurance can be an
effective means of reducing the capital required and may be cheaper than raising extra
capital by other means. The insurer could use the extra capital to generate increased
revenue by writing additional business.

Note, however, that reinsurance and capital are not completely compatible. There are some
things that reinsurance can do that capital cannot, and vice versa. For example, reinsurance also
enables an insurer to receive technical assistance.

2.3 How does the cost of reinsurance capital compare with the cost of an
insurer’s capital or alternative forms of capital?
The cost of reinsurance capital will differ from the cost of alternative sources of capital,
such as that obtained through the more traditional capital markets. Reinsurance may be a
cheaper way for an insurer to reduce the capital it needs, enabling it to increase its overall
return on equity and increase its value to shareholders.

© IFE: 2019 Examinations The Actuarial Education Company


SP7-24: Determining appropriate reinsurance Page 13

Question

Explain how reinsurance can increase value to shareholders.

Solution

Reinsurance reduces the need for capital, so that there is more efficiency in the use of existing
capital and therefore better returns to shareholders.

2.4 Reinsurance for new companies


New companies use reinsurance to reduce the capital strain of financing new business.
They usually do this by ceding large proportions of business to reinsurers via quota share
reinsurance. This is an addition to many of the points made earlier in this chapter which
covers issues that also apply to new companies.

Question

Explain why else a new company would use reinsurance.

Solution

In addition to relieving new business strain, a new company has limited experience, so may
appreciate technical assistance, eg guidance on pricing, policy design, claims management,
underwriting management and access to data.

There is considerable uncertainty for a new company in the form of parameter risk (it may get its
pricing wrong), in which case reinsurance will relieve this uncertainty, through sharing
arrangements such as proportional reinsurance, and through capping arrangements such as
non-proportional reinsurance.

The Actuarial Education Company © IFE: 2019 Examinations


Page 14 SP7-24: Determining appropriate reinsurance

3 Determination of appropriate reinsurance


If you are asked to discuss an insurer’s reinsurance strategy in the exam, you should work through
the following four key processes:
1. Analyse the class(es) of insurance. Consider particular features that justify calling on the
reinsurers, for example:
– What different types of claim exist (ie property, liability, etc)?
– How big is each risk?
– Could there be any very large claims, and if so how likely are they (consider the
tail of the distribution)?
– Is there a possibility of accumulations of risk?
– What is the claim frequency and the distribution of claim amounts?
2. Analyse the needs of the direct writer. Consider the key points from Section 1 concerning
the company:
– How big are the free reserves?
– How well do the new risks complement the risks already covered?
– What do we want to achieve by using reinsurance?
– Do we have experience of this class?
3. Consider all the different reasons for using reinsurance (summarised in Section 3.1
below). Do any of these reasons apply here?
4. Finally (and only as a final check) consider the different types of reinsurance and quickly
justify using or not using each type.

If you work through these four ideas intelligently, you should be able to give a clear and complete
summary of what reinsurance (if any) is needed.

© IFE: 2019 Examinations The Actuarial Education Company


SP7-24: Determining appropriate reinsurance Page 15

3.1 Summary of reasons for using reinsurance


We have already looked at the uses of reinsurance but a final recap might be useful. These
reasons are summarised in the table below.

Reason Comment
Diversification Rather than writing a few large risks, the direct writer can write lots of
smaller risks. This is achieved by surplus and excess of loss. Reciprocity
(through quota share) is also very helpful in diversifying the portfolio.
Larger risks The office can increase the scope of business it writes, for example
through surplus and also perhaps excess of loss.
More business By reducing the variation in claim payments, less risk capital is needed.
Conversely, more gross business can be written on the same amount of
capital.
Protection The solvency margin (excess of assets over liabilities) is protected from
large claims (through excess of loss).
Smooth profits Net results are more stable over time (especially with excess of loss).
Catastrophes Helps prevent risk of insolvency from catastrophes or other
accumulations. Use catastrophe excess of loss or other aggregate excess
of loss as appropriate for the class.
Technical Reinsurers and brokers may provide useful technical services and advice
assistance to insurers that are not experienced in a particular class.
New classes Insurers can get experience in a new class without being exposed to too
much risk (although they are unlikely to make much profit).
Regulation The authorities may reduce the required level of statutory solvency to
allow for the extent to which reinsurance has reduced the company’s level
of risk.
Financial Via favourable commission agreements (proportional) or through the
assistance direct writer hanging on to the gross premium before passing anything on
to the reinsurer. Also, using reinsurance as a favourable alternative to
capital.

Assorted other reasons are sometimes also put forward; for example, reinsurance might be cheap
if there is excess supply in the market (non-proportional).

As reinsurers need to make a profit, this reason should be viewed as a short term opportunity,
rather than genuine justification for reinsurance. Remember that reinsurers do (at least in the
long term) make profits. In the main, these profits are at the expense of the direct writer, so the
direct writer will not want to take out more reinsurance than is really needed.

The Actuarial Education Company © IFE: 2019 Examinations


Page 16 SP7-24: Determining appropriate reinsurance

Question

List the factors that influence the decision as to what and how much reinsurance to obtain, and
convince yourself that you can explain them (eg list the headings and subheadings from Section 1
and then check you can develop the ideas from those headings).

Solution

 classes written: size and range of risks; claims characteristics; volatility


 size of free reserves
 total written premium
 geographical regions of risks
 accumulations: geographical; by type or class; exposure across classes; inward
reinsurance
 current market conditions: availability of reinsurance
 value for money
 reinsurer security
 regulation
 possible tax advantages
 risk appetite
 underwriter influences
 the need for technical assistance
 financial objectives
 reinsurers’ requirement for a minimum retention
 the impact of capital management
 relationships
 alternatives, eg coinsurance, equalisation reserves, etc.

© IFE: 2019 Examinations The Actuarial Education Company


SP7-24: Determining appropriate reinsurance Page 17

4 Case study
Describe, giving reasons, the main features of a reinsurance package that you think would be
most appropriate for a small rapidly-expanding company writing only domestic household
insurance.

Before reading on have a think about how you would approach this question and what your
answer might be.

Our sample solution to this question is shown below. Additional comments are shown in italics.

Company requirements
The small rapidly expanding part of the scenario leads us to make the following comments. If you
missed any of these comments ask yourself why, and think about how you can make sure that you
think (note that we say think and not remember) of them next time you see a similar question.

Quota share could be arranged to reduce new business strain. If the reinsurer pays override
commission to the insurer, this would help meet the insurer’s acquisition expenses. Quota share
could also lead to the insurer receiving profit commission.

Quota share reciprocal arrangements would also help the insurer to get a more diverse portfolio
and help prevent accumulations of risk building up.

If the company is fairly new (possibly implied by the small, rapidly expanding comment), it may
not have enough of its own data and experience. Technical assistance from reinsurers and
brokers would be available in return for business placed with them.

Domestic property requirements


Having focused on the particular needs of this company, we now turn our attention to the claim
characteristics of the business written, ie domestic property.

Property portfolios are often exposed to the possibility of many claims from one event, eg floods,
storms. Catastrophe XL will give the insurer protection against many claims arising from the same
storm.

The retention level chosen should reflect the total expected maximum loss from a major storm
and the insurer’s solvency position.

An hours clause (probably 72 hours) would be necessary to define the period over which claim
events could be aggregated.

Several storms may hit this area in any particular year. Thus, we need to consider numbers of
reinstatements available and size of any reinstatement premium.

Many subsidence claims may occur within a short period of time given a long dry summer.
Insurer should use aggregate XL reinsurance to protect against such aggregations.

The Actuarial Education Company © IFE: 2019 Examinations


Page 18 SP7-24: Determining appropriate reinsurance

The insurer will also need to use individual XL reinsurance to protect the solvency position against
the occasional large public liability claim. Given the small size of the company and possible capital
restrictions, retention limits should start quite low.

Cover should be arranged in several layers with different reinsurers to reduce the credit risk of
reinsurer failure.

The insurer is unlikely to be concerned about individual property claims, except possibly the very
largest domestic properties. The volume of policies and reasonable homogeneity means that the
sophistication of varying the percentage of each risk retained is unwarranted. Hence surplus is
inappropriate here. (It is used primarily for larger commercial properties where volumes of
business are lower and the risks are more heterogeneous.)

Stop loss cover may well be appropriate to protect the whole account, but is unlikely to be
available at a reasonable price for this class of business.

Financial reinsurance is unlikely to be a main feature of the required reinsurance package in this
scenario, as a small company may not have expertise in this area. Arguably, however, financial
quota share could be used to help alleviate new business strain.

© IFE: 2019 Examinations The Actuarial Education Company


SP7-24: Determining appropriate reinsurance Page 19

5 Testing the appropriateness of alternative reinsurance structures


Reinsurance is typically one of the most significant areas of decision making for a general
insurer, involving large upfront costs and having a material impact on a business’s risk
profile, capital requirements and ultimate profitability.

The impact is most pronounced for companies that are, for example:
 small
 inexperienced, eg lacking data and expertise
 writing volatile lines of business, eg aviation or excess of loss reinsurance.

Most purchases will be scrutinised in a variety of ways to ensure that they are appropriate.
In the following sections we outline some possible ways to assess reinsurance structures,
but this is by no means exhaustive and other tests may be appropriate, particularly if there
are unusual features of the programmes being considered or the underlying business being
protected.

We have grouped the tests into two broad categories, although there will be overlap
between these. In the first category we outline quantitative tests to assess the efficiency
and performance of a programme. In the second we consider more qualitative tests to
consider the alignment of the programme to the exposures covered and the risk objectives
of the overall business.

Regardless of the methods used, any testing should be considered by an appropriate group
of stakeholders to ensure that decisions are being made well. This should ideally include
representation from actuarial, claims, underwriting, risk, exposure management and, for
material purchases, senior management. We may also want to seek the views of reinsurance
brokers or external actuarial consultancies.

5.1 Quantitative testing


In this section, we suggest some possible quantitative tests to carry out to assess the
performance of a reinsurance programme. When considering the outcome of these tests, it
is important to ensure that decision makers understand the credibility of the parameters or
the prior data used for testing and any limitations that might place on the credibility of the
test outcomes. Otherwise there is a significant risk that tests may appear robust because
they are presented in technical and quantitative terms, while in fact being essentially
qualitative as they are based on highly subjective parameters or extremely sparse data.

It is also important to take a long-term view when assessing the suitability of a reinsurance
programme. In some years, there may be a net loss from the programme and in others a net gain.
Reinsurers will set their rates so that, in the long run, they make a profit.

The Actuarial Education Company © IFE: 2019 Examinations


Page 20 SP7-24: Determining appropriate reinsurance

Modelled loss ratios


The mean loss ratio for the reinsurer is based on underlying modelling assumptions
(whether pricing assumptions or capital model parameters). When calculating this,
consideration should be given to the treatment of reinstatements. These can be added to
the reinsurance premium used as the denominator, which is aligned to the accounting
treatment if a loss did occur. Alternatively, they can be allowed for as a deduction to the
likely level of recoveries to be received, which may be of more value if assessing the level of
profit to be ceded on an initial upfront reinsurance spend.

Modelled impacts on net or final net loss ratios can also be considered.

This could be done by, for example, comparing the expected:


 net and gross loss ratios of the insurer
 loss ratios of the insurer with those of the reinsurer.

Capital impacts
The amount of capital saved as a result of the reinsurance programme being purchased is a
useful metric to consider, as this is both a practical indication of the amount of capital
saved and an indicator of the extent of mitigation provided to the overall risk profile.

Also note that, under Solvency II, insurers using an internal model to demonstrate solvency must
satisfy the ‘use test’. This requires that the internal (capital) model should play a significant role
in all decision-making, including reinsurance purchase decisions.

As with any capital allocation exercise, there are a variety of potential methods, each of
which have various advantages and disadvantages as capital modelling involves the
interaction between a number of different variables. As each reinsurance programme
mitigates the risk for a line of business, the relative significance of other risk drivers will
increase. The modelled impact for any individual programme can therefore change as other
programmes are amended.

One approach is to consider the marginal capital impact of adding or removing a particular
programme. For this approach, one would carry out two capital model runs, one with and
one without the programme being considered, and compare the two capital outputs. This is
one of the simpler potential approaches, but may risk overstating the capital significance of
an individual programme if it tests against a portfolio that is generally well balanced other
than the line of business being tested. This can also place high demands on the capital
modelling team as it requires a series of separate runs for each programme being tested.

Another approach is to consider the contribution of recoveries from different programmes


to simulations around the target return period (eg 1 in 200), and to use this as a way of
allocating the overall capital benefit of reinsurance (which can be calculated by running the
capital model with all reinsurance cover removed). This recognises some of the interaction
between reinsurance programmes and avoids the need for multiple separate capital model
runs.

© IFE: 2019 Examinations The Actuarial Education Company


SP7-24: Determining appropriate reinsurance Page 21

There may also be value in considering a lifecycle capital view. The capital benefit of a
catastrophe programme for example, might be significant to current capital requirements
but the continuing benefit of that same programme to future years’ capital requirements
may be limited once the business has earned. Conversely, capital benefits from a quota
share of a long-tailed portfolio may be low in any individual year but might continue to
provide benefits over multiple years as the underlying exposures remain volatile for a
number of years.

Ceded return on equity


Expected profit ceded divided by modelled capital saved (whichever method is used)
provides a useful single metric for the effectiveness of a reinsurance programme.

This produces a return on capital metric that can then be compared against other return on
capital measures used for other decision making; although it should be noted a lower value
is preferable, ie better reinsurance programmes do not need to cede away as much profit to
achieve the same reduction in capital.

For example, if a reinsurance programme is expected to cede away $100,000 of profit for a
$1m reduction in capital then it would have a 10% ceded return on equity. If there are
alternative opportunities available at 8% return on capital, then the $1m of capital saved in
theory can only be deployed to generate $80,000 of profit so the programme would reduce
the overall profit achievable. It may however, still be appropriate to place if it achieves risk
mitigation or other objectives.

Other objectives might include:


 maintaining a target credit rating standard
 smoothing year-on-year results
 improving the solvency margin.

Return periods
It can be valuable to assess the likelihood of certain outcomes with different reinsurance
programmes, for example the return periods at which:

 the reinsurance programme begins to be utilised

 individual losses go above the upper limit of the programme

 the reinsurance programme and associated reinstatements are exhausted leaving no


cover remaining

 any parametric triggers (eg for an industry loss warranty) are reached.

It may also be useful to assess the modelled outcomes of the overall loss distribution or
components of a loss distribution at various return periods, either to align to regulatory
reporting requirements or to internal risk appetites. An example might be managing the
1 in 30 modelled aggregated catastrophe losses for Lloyd’s reporting.

Lloyd’s requests reporting of syndicates’ aggregate exceedance probability (AEP) losses at a


30-year return period for various perils. This is the value that annual aggregate losses would be
expected to exceed only once in every 30 years. Together with Realistic Disaster Scenario
analysis, this is useful in monitoring syndicates’ exposure to catastrophe risks.

The Actuarial Education Company © IFE: 2019 Examinations


Page 22 SP7-24: Determining appropriate reinsurance

Historical ‘as-if’ performance


Programmes can also be tested against internal historical experience, by assessing the
recoveries and reinstatements that would have been incurred if the programme had been in
place historically. This provides an alternative to a modelled loss ratio view, although
credibility is dependent on data volumes which may be limited for many organisations,
particularly for higher layers.

Consideration should be given to the need for any adjustments to historical losses, for
example indexation for claims inflation or adjustments for exposure levels.

We need to allow for any differences between the past exposure periods and the future period to
which the new reinsurance programme will apply. For example, if exposure was lower in the past,
it will need to be adjusted up when trying to determine the value of losses we should expect in
the future period of reinsurance. Similarly, historical claims need to be inflated to put them all in
the monetary terms of this future period. We call this ‘on-levelling’ because all the losses are
brought up to the same level (eg a 2019 level, if the new reinsurance programme is to apply in
2019).

5.2 Qualitative testing


So far, we have considered numerical tests to assess the efficiency and performance of a
reinsurance programme. Now we move on to more qualitative assessments of the suitability of
reinsurance arrangements.

In this section, we suggest some examples of more qualitative testing to ensure that the
programme is appropriate for the underlying portfolio, the organisation and any internal risk
appetite or other requirements.

Risk appetites
Organisations will have internal risk appetites in place (some of which are put in place by
the regulators). Reinsurance is an effective way of managing these risk appetites, which
may often be explicitly expressed on a net or final net basis.

The ‘net loss’ usually refers to the loss net of reinsurance. The ‘final net loss’ usually refers to the
loss net of reinsurance and reinstatements.

Examples of potential risk appetites might be:

 Maximum outputs from catastrophe models, either for aggregate (AEP) or event
(OEP) losses. These might be in aggregate or for specific regions or perils, eg North
Atlantic Windstorm.
There are two bases for catastrophe model output files:
– OEPs – an occurrence exceedance probability file, which considers the probability
that the largest individual event loss in a year exceeds a particular threshold.
– AEPs – an aggregate exceedance probability file, which considers the probability
that the aggregate losses from all loss events in a year exceeds a particular
threshold.
 Maximums for defined risk scenarios, such as the Lloyd’s Realistic Disaster
Scenarios or internally defined extreme events or accumulation scenarios.

© IFE: 2019 Examinations The Actuarial Education Company


SP7-24: Determining appropriate reinsurance Page 23

 Maximum allowable line sizes for a particular class of business, either in the
absolute or relative to premium expectations.

 Portfolio balance metrics requiring minimum or maximum size thresholds for net
premiums in any line of business, ensuring that no class is too small to justify
internal management time or too large to unbalance the overall business.

Each organisation will have its own risk appetites, and these should be considered in the
design of any reinsurance assessment framework to ensure that they are not inadvertently
breached.

Alignment to underlying exposures


Reinsurance programmes should, as far as possible, provide appropriate back to back
protection for the underlying inwards exposures being mitigated.
The ‘underlying inwards exposures’ are simply the risks insured by the (ceding) insurer. Recall
that inwards business is the insurance business that the company has written, as opposed to
outwards business, which refers to the company’s reinsurance programme.

Some possible factors to consider are:

 Term of the contract. If protecting multi-year inwards business, reinsurance should


ideally be written on a ‘risks-attaching’ rather than ‘losses-occurring’ basis or
should provide for a similar term of cover. If annual losses-occurring cover is
purchased, this can become extremely expensive in a run-off situation.
This is because, as the business runs off, there will be fewer and fewer policies still on-risk
in the underlying portfolio. In this situation, the expenses of providing and administering
reinsurance (and hence the price charged) become disproportionately high compared
with the volume of underlying business still on-risk.

 Exclusions or any sub-limits should be consistent with the terms used for inwards
business, to avoid the risk of any uncovered exposures. If this is not possible, these
uncovered exposures should be explicitly monitored and managed to different risk
appetites.
For example, it would not be sensible for the reinsurance programme to exclude flood risk
if the underlying policies cover this risk, as this would create a ‘hole’ in the cover for the
ceding insurer.

 Territorial or coverage scope should also align to the inwards business, with any
minor exposures out of scope monitored separately.
We should try to ensure that none of the territories in which the underlying business is
written are excluded from cover under the reinsurance contract. Similarly, we should try
to ensure that the scope of coverage under the reinsurance programme is as wide as that
provided by the underlying policies, eg if business interruption cover is included in the
(ceding) insurer’s commercial fire policies, then it should also be covered by the
reinsurance programme.

 Currency of the contract should be aligned to the likely currency of inwards losses
and limits, potentially with multiple currency limits or specified exchange rates to
avoid disputes over timing or currency of payments.

The Actuarial Education Company © IFE: 2019 Examinations


Page 24 SP7-24: Determining appropriate reinsurance

For example, without specified exchange rates, a US reinsurer could time recovery
payments to a UK-based cedant to take advantage of movements in the exchange rate for
converting US dollars to pounds sterling. This could leave the cedant feeling aggrieved or
lead to a dispute.

Reinsurance contract
A programme might potentially appear efficient on quantitative measures, but then provide
unacceptable gaps in cover. Purchasing decisions should be supported by an expert
review of the particular wording of the contract to ensure that its intended operation would
be upheld in any legal dispute.

The majority of contracts will use well-established standard market wordings which should
have a limited risk of dispute, with bespoke terms likely to be the highest risk. Examples of
potential areas that benefit from additional scrutiny are:

 exact definitions of qualifying events and loss triggers

 specification of hours clauses and the basis for selecting an appropriate time period

 any named event clauses, for example, allowing insurers to accumulate hurricane
losses from a single named storm impacting the Caribbean and the mainland US,
even if falling outside of the hours clauses

 rules governing the assessment of loss outcomes, for example, for stop loss
business or for profit commission calculations on quota share business

 provisions around the timing, nature and currency of payment

 indexation of loss or layer clauses

 collateralisation rules, including release of ring-fenced funds


In collateralized reinsurance, the reinsurer lodges collateral in an account at the start of
the contract, to provide security against its obligations to the cedant. The source of the
collateral could be, for example, investors purchasing insurance-linked securities.

 treatment of ex-gratia payments. There may be situations in which an insurer makes


a payment outside of their absolute contractual entitlement; for example, to support
a good commercial relationship with a broker or to avoid a protracted loss dispute.
Reinsurers may or may not choose to cover such payments.

Reinsurer security
Even a well-designed reinsurance contract is only effective if the reinsurer remains
adequately solvent to be able to pay claims when they arise. The credit rating of a reinsurer
will be one of the key indicators, or the amount of funds ring-fenced for collateralised
contracts.

It is likely that there will be risk appetites relating to concentrations with any particular
reinsurers, so the extent of other existing arrangements may affect the choice of reinsurer
for a new contract.

For example, an insurer may already have several key reinsurance contracts placed with a given
reinsurer, meaning that any additional reinsurance contracts need to be placed with a different
reinsurer.

© IFE: 2019 Examinations The Actuarial Education Company


SP7-24: Determining appropriate reinsurance Page 25

Potential reinsurers may also have a history of disputing claims, which should be
considered before including them on the reinsurer panel.

In most instances these factors may not affect the selected programme structure provided
sufficient alternative reinsurers are available to place the required cover, but if the only
reinsurer offering a particular contract poses a potential security risk then this may affect
the decision-making process.

Other factors
There may be other reasons driving individual purchasing decisions, with some examples
being:

 Strategic partnerships or strong commercial relationships with a reinsurer,


potentially involving some element of reciprocity or strong support for unusual
reinsurance requirements.

 Acting as a fronting agent for a particular portfolio.

 Technical assistance, analysis, advice or other support from the reinsurers.

 Protecting new portfolios with significant parameter uncertainty. A programme


might, for example, provide poor value based on the assumed profile of the risk
being protected, but would also mitigate the risk of materially misunderstanding the
underlying exposures.

 Desire to maintaining credit ratings, eg with this in mind, an insurer may feel that it is
safest to keep the reinsurance programme the same.
 Maintaining relationships with reinsurance brokers.
 Improving the solvency position.
 Market conditions, eg insurers may be more inclined to purchase reinsurance when it
appears to be cheap.

The Actuarial Education Company © IFE: 2019 Examinations


Page 26 SP7-24: Determining appropriate reinsurance

6 Glossary items
Having studied this chapter you should now read the following Glossary items:
 Clash cover
 Coinsurance
 From the ground up.

© IFE: 2019 Examinations The Actuarial Education Company


SP7-24: Determining appropriate reinsurance Page 27

Chapter 24 Summary
The main factors determining an insurer’s use of reinsurance are:
 the classes of business written, the size and range of risks and their volatility
 the size of the insurer’s free reserves
 the extent to which accumulations of risk are possible
 the size (total premiums) and diversification of the insurer’s portfolio
 the general cost (value for money) and availability of reinsurance
 alternatives including coinsurance and protection afforded by parent company
 the financial strength of reinsurers
 regulatory requirements and fiscal system
 the impact of capital management
 the need for technical assistance
 risk appetite and tolerance
 underwriters’ influences
 financial objectives
 the reinsurers’ requirement for a minimum retention
 relationships with reinsurers and brokers.

Both quantitative and qualitative approaches can be used to test the appropriateness of
different reinsurance structures.

Quantitative tests include:


 modelled loss ratios
 capital impacts
 ceded return on equity
 return periods
 historical ‘as-if’ performance.

Qualitative tests include:


 risk appetite
 alignment to underlying exposures
 reinsurance contract
 reinsurer security
 other factors, eg strategic partnerships, fronting, technical assistance.

The Actuarial Education Company © IFE: 2019 Examinations


Page 28 SP7-24: Determining appropriate reinsurance

The practice questions start on the next page so that you can
keep the chapter summaries together for revision purposes.

© IFE: 2019 Examinations The Actuarial Education Company


SP7-24: Determining appropriate reinsurance Page 29

Chapter 24 Practice Questions


24.1 For each of the scenarios below, suggest the single most likely type of reinsurance and briefly
explain the logic for your choice:
(a) A medium-sized insurer writing commercial property risks.
(b) A small, regional insurer writing household insurance.
(c) A company writing a new class of insurance.
(d) A large company writing private motor insurance.
(e) A small company writing private motor insurance.
(f) A company writing employers’ liability insurance.
(g) A small company expanding very rapidly.

24.2 State the conditions under which stop loss reinsurance is most likely to be made available to a
direct writer.

24.3 State the reasons why a general insurance company might wish to buy reinsurance.

24.4 (i) List three situations where stop loss cover is available and used in practice.

(ii) Explain why reinsurers are frequently unwilling to offer stop loss covers.

(iii) Describe the conditions reinsurers may impose before providing stop loss cover.

24.5 (i) Describe the facultative and treaty methods of reinsurance, listing the advantages and
Exam style
disadvantages of the two methods. [9]

(ii) Describe the surplus, excess of loss and stop loss forms of reinsurance. [7]

(iii) Suggest, with reasons, the type and extent of treaty reinsurance that would be most
suitable for:
(a) a medium-sized company writing motor insurance
(b) a large insurance company writing industrial fire insurance. [4]
[Total 20]

24.6 List the factors that should be considered by an insurer when considering the type and amount of
Exam style
reinsurance for a particular class of business. [4]

The Actuarial Education Company © IFE: 2019 Examinations


Page 30 SP7-24: Determining appropriate reinsurance

The solutions start on the next page so that you can


separate the questions and solutions.

© IFE: 2019 Examinations The Actuarial Education Company


SP7-24: Determining appropriate reinsurance Page 31

Chapter 24 Solutions
24.1 (a) A medium-sized insurer writing commercial property risks: surplus reinsurance, to allow
the company to quote for properties that otherwise would be too big.

(b) A small, regional insurer writing household insurance: Cat XL, to guard against storm,
flood, or burst pipe damage. (Quota share with reciprocity would almost have been
acceptable as a means of diversifying, but Cat XL is probably more important.)

(c) A company writing a new class of insurance: quota share, as a means of limiting initial
exposure and gaining technical help from the reinsurer.

(d) A large company writing private motor insurance: XL covering any one risk (eg large single
claim) or any one event (eg a motorway pile-up).

(e) A small company writing private motor insurance: same as (d), but with a much lower
excess limit and potentially QS to gain diversification.

(f) A company writing employers’ liability insurance: XL covering any one risk (eg a single
large claim) or any one event (eg many claims from a factory explosion).

(g) A small company expanding very rapidly: quota share, as a means of spreading risks and
maybe financial reinsurance to improve capital position and increase capacity to write
new business.

24.2 Stop loss most likely to be made available to a direct writer:


 where there is a close relationship between the insurer and reinsurer
 where the underwriting behaviour has less impact on the outcome of the insurance,
eg where claim events are entirely random (bad weather)
 where quality of claims handling or underwriting cannot prejudice the loss ratio.

24.3 Reasons for buying reinsurance include:


 To limit the effect of large individual or aggregated losses that may threaten solvency.
 To protect the insurer’s solvency margin more generally.
 To smooth the underwriting profits, perhaps to aid a smooth dividend policy, by
smoothing claim costs.
 In some countries a certain amount of reinsurance may be compulsory.
 To gain a better spread of risks:
– keeping a smaller part of more risks gives better diversification
– similarly, to benefit from reciprocal arrangements.

The Actuarial Education Company © IFE: 2019 Examinations


Page 32 SP7-24: Determining appropriate reinsurance

 Some companies may suffer from a lack of their own capital or experience. Thus the
insurer may use reinsurance to:
– enable the insurer to accept risks that would otherwise have been too large to
accept
– obtain technical assistance in return for placing business with the reinsurer
– get experience of a new class of insurance without being exposed to too much risk
– get return commission from the reinsurer to help relieve the insurer’s new business
strain.
 The insurer may believe that reinsurance rates are currently good value.

24.4 (i) Available in practice

Crop insurance against hail (danger of selection is reduced because the events causing the
damage are natural events, eg storm, frost).

Lloyd’s personal names on their whole portfolio of risk (to prevent individual names becoming
bankrupt). Stop loss cover may also be available at syndicate level to prevent calls on the Lloyd’s
Chain of Security, eg the Central assets.

Extended warranty insurance (danger of selection is reduced because the insurance is sold by the
retailer and not the insurer).

Mortgage indemnity guarantee insurance (danger of selection is reduced because the events
giving rise to the claims are external to the insurer and the insured, eg economic downturn).

Insurance subsidiaries from their insurance parent (possibility of selection irrelevant because
companies are part of the same group).

(ii) Reasons why not commonly available

Reinsurer is exposed to poor underwriting and claims control of the insurer; there is a possibility
of moral hazard.

Business was historically unprofitable.

(iii) Conditions reinsurer may impose

Controls on underwriting and claims standards.

Insurer must retain a part of the risk.

24.5 (i) Facultative and treaty

Facultative

An individual arrangement between a direct writer and reinsurer where the direct writer
approaches the facultative reinsurer to reinsure part of a particular risk. No obligation on the
direct writer to offer business and no obligation on the reinsurer to agree terms. [1]

© IFE: 2019 Examinations The Actuarial Education Company


SP7-24: Determining appropriate reinsurance Page 33

Possible on proportional or non-proportional bases. Might be used, for example, on a large


individual property risk which was too large to be covered entirely by an existing treaty. Also used
for unusual risks not covered by the treaty. [1]

+ flexibility
+ used to cope with risks falling outside the terms of treaty
– needs to be arranged for each risk …
– … therefore can involve extra work in setting up the reinsurance
– direct writer may not be able to find suitable reinsurance
– delay for direct writer in finding cover before accepting risk [½ mark each]

Treaty

An arrangement between a direct writer and a reinsurer. The treaty will specify class(es) and
territories and time period of cover. [1]

Reinsurer required to accept risks offered that fall within the treaty conditions. Direct writer may
or may not be obliged to use reinsurance, depending on type of treaty. [1]

The treaty could be proportional (quota share or surplus) or non-proportional (excess of loss). A
treaty is useful for proportional cover on small risks, catastrophe cover and stop loss cover. [1]

+ gives direct writer certainty of being able to have reinsurance cover


+ can be simple to operate (quota share)
– takes time / expense to set up in first place
– treaty terms may restrict direct writer’s freedom [½ mark each]
[Maximum 9]

(ii) Descriptions

Surplus

 Proportional. [½]
 Direct writer retains a proportion that may vary from risk to risk. Reinsurer’s proportion
also therefore varies from risk to risk. [½]
 Premiums and claims for each risk are shared in the proportion set for that risk. [½]
 Premium paid to reinsurer will be reduced for commission (return and overriding). [½]
 Treaty would specify maximum proportion to be reinsured (via the number of lines) and
maximum retention. [½]

The Actuarial Education Company © IFE: 2019 Examinations


Page 34 SP7-24: Determining appropriate reinsurance

Excess of loss

 Non-proportional. [½]
 Excess point is set (the retention), above which reinsurer pays the claims up to an upper
limit. [½]
 Limits might be linked to inflation (stability clause). [½]
 Might be on one of several bases. [½]
 Individual: whereby cover applies to claim amount from an individual risk or policy
(eg one large building being destroyed). [1]
 Aggregate: all claims from one occurrence or event are aggregated, eg all claims from a
single drug (product liability) or all claims from one storm (buildings cover). [1]

Stop loss

 Non-proportional. Treaty applying to a class or classes. [½]


 Reinsurer pays the total claims (or a proportion of them) from the account in excess of an
agreed level (amount or proportion of premiums). [½]
 There will usually be an upper limit to reinsurer’s payments. [½]
 Difficult to obtain for many classes. [½]
[Maximum 7]

(iii) Treaty reinsurance

Note question says type (ie singular) so we have given only the most important type in each case.

(a) Medium-sized motor insurer

A motor account can give rise to the occasional enormous liability claim (eg one driver proved to
cause a multiple pile-up). So the insurer needs individual (and/or aggregate) excess of loss for an
occasional large liability claim: excess point, say, £100,000 because medium-sized. [1]

Sufficient layers required to provide cover for any size claim (ie unlimited). [1]

(b) Large insurance company writing industrial fire

Industrial fire risks can be very large, so even a large insurer is not likely to want to retain all the
risk for each policy. [½]

So the insurer needs proportional insurance (which might vary for different types of building). [½]

So a surplus treaty is required, probably with quite a high maximum retention. [½]

The number of lines offered depends on the reinsurer’s position, but a second surplus treaty is
possible to cope with very large risks. [½]
[Total 4]

© IFE: 2019 Examinations The Actuarial Education Company


SP7-24: Determining appropriate reinsurance Page 35

24.6 Factors to consider when determining the amount and type of reinsurance

 Class of business under consideration:


– size and range of risks
– volatility of experience and hence stability of profits
– the need for technical expertise.
 Size of free reserves.
 Total written premium.
 Ratio of risks in this particular class to insurer’s total business.
 Geographical regions in which risks are situated.
 Accumulations of risk:
– too much risk in one geographical region
– too many similar types of risk
– risks where claims may arise under a different class of business.
 Opportunities to find coinsurers.
 Perceived value for money.
 Availability of suitable reinsurance.
 Reinsurer’s requirement for a minimum retention.
 Financial strength of reinsurers in the market.
 Alternatives to reinsurance.
 Amount used by competitors. [½ each, maximum 4]

The Actuarial Education Company © IFE: 2019 Examinations


All study material produced by ActEd is copyright and is sold
for the exclusive use of the purchaser. The copyright is
owned by Institute and Faculty Education Limited, a
subsidiary of the Institute and Faculty of Actuaries.

Unless prior authority is granted by ActEd, you may not hire


out, lend, give out, sell, store or transmit electronically or
photocopy any part of the study material.

You must take care of your study material to ensure that it


is not used or copied by anybody else.

Legal action will be taken if these terms are infringed. In


addition, we may seek to take disciplinary action through
the profession or through your employer.

These conditions remain in force after you have finished


using the course.

The Actuarial Education Company © IFE: 2019 Examinations


SP7-25: Reinsurance reserving Page 1

Reinsurance reserving
Syllabus objectives

5.6 Describe the following approaches to reserving for outwards reinsurance:


 gross less net
 application of standard techniques to reinsurance data
 use of appropriate factors
 application of detailed contract terms.

5.7 Compare the advantages and disadvantages of each of the above methods and the
appropriate circumstances in which to use them.

5.8 Discuss suitable approaches to reserving for inwards reinsurance.

The Actuarial Education Company © IFE: 2019 Examinations


Page 2 SP7-25: Reinsurance reserving

0 Introduction
Reinsurance can have quite different characteristics from primary insurance in terms of, for
example, data availability and claims payment patterns. These differing characteristics mean that
alternative reserving approaches are often more suitable than those appropriate for primary
insurance.

By way of definition, ‘inwards reinsurance’ will be the term used to describe the acceptance
and treatment of direct insurance business in the hands of the reinsurer. Thus ‘outward
reinsurance’ will be the same business when viewed from the perspective of the party
(insurer) having originally passed on that risk.

In other words, inwards reinsurance is reinsurance business sold by the reinsurer. Outwards
reinsurance is reinsurance bought by the cedant (so it’s the same business, just from the cedant’s
viewpoint). Reinsurance bought by reinsurers is called retrocession.

The basic methodologies that we use to reserve for inwards reinsurance are no different to
those for direct insurance. The main difference is in how easy it is to use these methods in
practice.

The inwards reinsurer encounters more data problems when reserving for reinsurance and
particularly for non-proportional reinsurances of long-tailed business. This makes it more
difficult to apply many of the methods. The inwards reinsurer will also need to exercise
more judgement.

Question

Suggest why non-proportional reinsurance of long-tailed business is particularly problematic.

Solution

Non-proportional reinsurance claims for long-tailed business are very volatile.

It can take a long time for a loss to be recognised as large enough to need to be reported to
reinsurers, particularly if the retention on the excess layer is high. This is because long-tailed
claims tend to be more complex, and it takes a while for loss adjusters to assess the full extent of
the claim.

In some years, there will be no reinsurance recoveries at all (particularly early on), and in other
years there could be a large number of claims. The eventual outcome of long-tailed primary
business is uncertain enough itself, and so its reinsurance recoveries are even more uncertain.

In Section 1 of this chapter, we describe the additional data issues associated with
reserving for inwards reinsurance.

In Section 2, we discuss the reserving process for a reinsurer. From the perspective of the
reinsurer, we call this inwards reinsurance.

In Section 3, we discuss how an insurer reserves for recoveries and premiums under its
reinsurance programme(s). From the perspective of the insurer, we call this outwards
reinsurance.

© IFE: 2019 Examinations The Actuarial Education Company


SP7-25: Reinsurance reserving Page 3

1 Data issues associated with inwards reinsurance


Data can be a problem particularly for reinsurance because:
 claim reporting delays are longer
 there is a greater tendency for claims to develop upwards
 exposure can be very heterogeneous
 data can be sparse
 benchmarks are often less relevant
 there can be IT constraints
 there is more opportunity to group data differently.

These are now covered in more detail.

1.1 Claim reporting delays are longer for the reinsurer


Claim reporting delays for reinsurers are longer even for proportional reinsurances,
compared to the delays experienced by insurers. Reinsurance contracts typically require
the insurer to report premium and losses to reinsurers on a quarterly basis. As a result, the
reinsurer may be notified of its claims under proportional covers up to a quarter of a year
later than the insurer is notified.

In some cases, the cedant may be required to provide details only of individual claims
above a certain amount. So for excess of loss covers, the claim reporting delays are longer
still, as it can take time for the insurer to recognise that a loss is large enough to be
reported to the reinsurer, particularly if the retention on the excess layer is high. As a
result, casualty (ie liability) reinsurers typically require the insurer to report claims when the
incurred claim size exceeds half the excess retention.

In most countries, and with most lines of business, half the retention is reasonable as a ‘census
point’ given that inflation may act whilst the claim develops. For longer-tailed classes, or in
countries where severe inflation is common-place, it is not unusual to have far lower reporting
points.

Lower reporting points are useful even if some smaller claims will never breach the retention,
because more data is then available for claims analysis purposes.

Question

An insurer writes liability business that has a 10-year tail. Inflation in the past has averaged 10%
per annum. Estimate a reasonable census point for claims in order to price an excess of loss
reinsurance treaty that attaches at £1m.

The Actuarial Education Company © IFE: 2019 Examinations


Page 4 SP7-25: Reinsurance reserving

Solution

Ten years of inflation at 10% per annum means that the census point would ideally be no greater
than 1,000,000v10  £385,543 . However, for prudence, perhaps a census point of £350,000
would be more suitable. More complex approaches are possible, for example by allowing for the
fact that not all claims would take ten years to settle.

For both proportional and non-proportional reinsurance, the reporting process can increase
the delays. The reporting process can involve the outwards reinsurance team at the
insurer, the reinsurance broker, and the claims team at the reinsurer. At each stage of this
process, additional delays (eg adjusting and agreeing the claim) are likely to occur before
the information finally reaches the reinsurer’s claims systems.

1.2 Greater tendency for claims to increase


With claims of such size that they hit the reinsurance cover, it is likely that the claim amount will
increase significantly over time, before final payment.

There is a greater tendency for claims to develop upwards for non-proportional reinsurance,
particularly on liability classes where liability and amount can often be in dispute for a
number of years. This is largely because large claims have longer delays to settlement so
that there is more time for social and economic inflation to affect the final settlement
amount. There can also be a tendency to underestimate the legal costs and other direct
claims handling costs associated with these lengthy claims. For some property claims,
however, the reverse can be true, as initial estimates tend to be overcautious.

Underestimation of legal costs has been particularly a problem in the UK over the last couple of
decades, as claims become larger and litigation becomes more drawn out and complex. Over the
years, efforts have been made to reduce the size and uncertainty associated with legal costs but
to date they have had little success.

1.3 Greater heterogeneity of exposure


Reinsurers may write a wide range of lines of business and a wide range of contract types
with very different terms and conditions.

Even within any one class of business, the reinsurer will write one treaty to reinsure one insurer’s
portfolio, and another treaty to reinsure another insurer’s portfolio, yet each cedant may have
very different underwriting standards, size of risks, target market and terms and conditions.

This increases the heterogeneity of the exposure for reinsurers (and hence the volatility of
the aggregate results). But most reserving methods are based on the assumption that risks
are homogeneous and there are large data volumes, so that aggregate results are more
stable.

This means that data grouping will be very important. This is discussed further in Section 1.7.

© IFE: 2019 Examinations The Actuarial Education Company


SP7-25: Reinsurance reserving Page 5

1.4 Sparse data


Particularly for high excess non-proportional business, there may be very few actual claims
so that the usual reserving data triangles may be very sparsely populated. This makes it
difficult to determine reliable development patterns resulting in more volatile projections.

One way around this is to use benchmark data to supplement our own experience. However, as
we will see in the next section, even this may not work very well, and the results will be
particularly sensitive to any assumed development factors.

1.5 Reduced applicability of industry benchmarks


Because of the heterogeneity of their exposures, different reinsurers can experience very
different claims development behaviour. This makes industry-wide benchmarks potentially
less appropriate, particularly for smaller reinsurers writing business in very specific areas.

In any event, there are relatively few industry benchmarks for reinsurance claims
development, particularly for non-proportional reinsurance.

In the UK, some actuarial consultancies have pooled data from a number of reinsurers and
cedants in order to develop benchmarks.

In the US, there are perhaps more benchmarks available. However, applying one country’s
benchmarks to another’s reserving methods is not ideal. In particular, the countries may differ
significantly in terms of, for example:
 terms and conditions, including deductibles and limits on underlying business
 social and legal trends, including attitudes to claiming
 inflation.

1.6 Data and systems constraints


The information that a reinsurer receives about losses can have less detail than the
information that an insurer receives.

The most obvious example of this is that the insurer will often only notify the reinsurer of claims
which exceed (say) half the retention, as discussed above. Therefore the data received will be a
truncated claim distribution. The reinsurer may also not receive any information on the
development pattern of claims before they reach that census point.

It is also likely that the reinsurer will receive only quantitative data, and not the additional
qualitative loss adjuster’s information.

Beyond these problems, a major issue can be that the reinsurer will often receive only aggregate
claims information. This can lead to problems surrounding the use of different claims cohorts
between insurer and reinsurer.

The Actuarial Education Company © IFE: 2019 Examinations


Page 6 SP7-25: Reinsurance reserving

This is a bigger problem for proportional covers, where the insurer may report losses on an
aggregate basis (or provide information only on claims in excess of an agreed amount to
the cedant), on the same basis as the (reinsurance) contract. For example, for a quota share
written on a ‘risks attaching’ basis, the insurer may report aggregate paid and incurred
losses to date, associated with risks written in the reinsurance policy period. If the
reinsurer uses an accident year reporting basis for its accounts and regulatory returns, it
must somehow split the risks attaching data supplied between accident years.

Usually the insurer provides more information about losses to a non-proportional contract.
But even then, a reinsurer may need to spend a material amount of time and resource in
requesting additional information which it feels that it needs to allocate and treat the losses
appropriately.

Where a reinsurance contract covers several lines of business, the loss data may show
which losses are associated with which lines. But often the premium is allocated to the
lines according to a pre-agreed percentage split, perhaps based on the expected split of
business when the contract was underwritten. To the extent that this split differs from the
actual mix of business or does not appropriately allow for different levels of risk between
lines of business, the premium split may be inaccurate. As premium (adjusted for rate
changes) may be the only measure of exposure a reinsurer has, this can lead to a mismatch
between exposure and losses in the reserving process.

Because of the complexity and individuality of reinsurance risks, it is difficult for a reinsurer
to have IT systems that capture perfectly all the contracts written and their key features. It
is more difficult than for an insurer, whose contracts tend to be more homogenous. This
makes storing and accessing accurate information harder for reinsurers. The actuary
working for a reinsurer should be aware of the potential shortcomings of the data that he or
she is using.

1.7 Data grouping for reserving


For a reserving exercise, we ideally subdivide loss and premium data into groupings that
are as homogenous as possible, and still have enough data in any one group to be able to
identify development patterns and so on.

However, as we mentioned in Section 1.3, a reinsurer’s business is normally subject to greater


heterogeneity than an insurer.

When we decide how to group the data for reinsurance reserving, we might want to
consider the following factors:

 type of contract (that is, facultative, treaty, finite/non-traditional)

 type of cover (that is, quota share, surplus, per risk excess, per event excess,
aggregate excess, stop loss, and so on)

 basis of cover (that is, losses occurring, risks attaching, claims made)

 line of business (casualty – probably sub-divided into underlying classes and


perhaps also excess vs primary underlying business due to differing reporting
delays: property, marine, aviation, construction, APH)

 attachment point (for excess covers)

 territory – US, Western Europe, Asia-Pacific, South/Central America, Africa, MENA


(Middle East and North Africa) and so on

 type of cedant (small, large, pools and/or associations, regional, multinational).

© IFE: 2019 Examinations The Actuarial Education Company


SP7-25: Reinsurance reserving Page 7

There are a large number of potential groupings; more than for a direct insurer and far more
than the volume of data is likely to allow. We need to make a compromise between
homogeneity and data volumes. We may do this by discussing it with underwriters and
claims handlers and examining development patterns to find a grouping of categories that
are expected to behave in a similar fashion.

The Actuarial Education Company © IFE: 2019 Examinations


Page 8 SP7-25: Reinsurance reserving

2 Reserving for inwards reinsurance


This section discusses the difficulties associated with using different methodologies when
reserving for reinsurance business written, rather than direct business. The reasons for using a
different approach than for direct business largely stem from the longer length of tail of the
reinsurance business.

What data should we use?


One decision we first need to make is whether to use paid or incurred data.

For a reinsurer, the incurred losses are the result of an aggregation of loss data from a
number of different insurers. All these insurers will have slightly different reserving
practices, and these may have changed in the past. In addition, a reinsurer’s pool of cedant
insurers may change from year to year.

The incurred data for a reinsurer will be much less consistent than for a direct insurer. The
reinsurer’s claims team would have to review all the losses submitted and adjust the data
submitted to make the year-by-year and cedant-by-cedant data more consistent.

As a result, it is appropriate for a reinsurer to rely more on paid development based


reserves than incurred, because the payments depend less on individual insurers and more
on legal process. The actuary may still need to estimate a tail based on analysis of case
estimates and projected number of future claims or possibly a suitable benchmark if
available. For a direct insurer the reverse might be more common.

If we are using reserving methods such as the Bornhuetter-Ferguson method, we will also need an
independent assumption of the loss ratio.

For methods such as the Bornhuetter-Ferguson, where an initial assumption for the
expected loss ratio is required, we may be able to use the target loss ratios from the pricing
of the contracts. If we do not use these directly, we may combine the pricing loss ratios
(perhaps using a credibility weighting) with inflated and rate-adjusted historical loss ratios,
say, as an input into the process.

2.1 Reserving for shorter-tailed business


Shorter-tailed business would typically include all types of property business (facultative or
excess, catastrophe or per risk) other than high excess property treaty and might include
marine or aviation, if the insured business is hull rather than products or liability.

Here, ‘products’ refers to the product liability element of marine or aviation insurance, covering,
for example, faulty manufacture of aircraft parts.

© IFE: 2019 Examinations The Actuarial Education Company


SP7-25: Reinsurance reserving Page 9

Question

Suggest why high-excess property treaty business might not be counted as short tailed.

Solution

The fact that the business is written via a treaty means that there will be delay as potentially large
losses may not be reported to the reinsurer immediately. Once a claim does hit the layer, there
will be some doubt as to its ultimate value, meaning that settlement could take longer.

For example, consequential losses can be complex and time-consuming to quantify. In addition,
they may be payable for a long period of time, for example until the property is rebuilt or until the
policy limit is reached.

The resulting reinsurance losses are therefore unlikely to be short tailed.

We are likely to treat construction risks as medium-tail because their long periods of
coverage (typically several years) increase the reporting delays.

These risks will therefore be discussed in Section 2.2.

Short-tailed business has relatively little IBNR or IBNER, and so it does not matter too much
(subject to the purpose of the reserving exercise) if the resultant reserves calculated are not as
accurate as perhaps they could be.

For shorter-tailed business, we will use relatively straightforward reserving approaches,


such as chain ladder or perhaps even an expected loss ratio, using rate and inflation-
adjusted historical loss ratios as a guide. The effort required for more complicated methods
is unlikely to be rewarded with a materially different or more accurate result. We may spend
time and resources more productively in improving the analysis for other parts of the
reinsurer’s inwards business.

Note: the chain ladder approach may have the added benefit of being easier to extend to a
stochastic approach, where local regulation (whether for reserving or capital) requires the
ability to produce a distribution for the ultimate losses.

Question

List two stochastic reserving methods that can be applied as an extension of a chain ladder
approach.

Solution

Mack’s method and Bootstrapping are probably the best known examples. These were discussed
elsewhere in the course.

We should be careful with major catastrophe losses, particularly recent catastrophes.

The Actuarial Education Company © IFE: 2019 Examinations


Page 10 SP7-25: Reinsurance reserving

Major catastrophes have very different claims characteristics (in particular, payment patterns) to
attritional claims. Recent catastrophes are a particular problem because there can be many
unreported and/or unsettled claims, and therefore reserves are more uncertain and sizeable.

It may be best to exclude all major catastrophes from the main methodology, reserve for
them separately and then add them back at the end of the process. We could use historical
experience from other similar types of catastrophe (for example, US windstorms, US
earthquakes, European windstorms, European floods and so on) as a guide to the likely
future development of more recent events.

Note: (particularly for catastrophes) we may review the development in finer detail, for
example, looking at monthly or weekly development, rather than annual, to allow better for
the time of year that the event occurred.

2.2 Reserving for medium and long-tailed business


Medium-tailed reinsurance business might include construction, high excess property
treaty, marine and non-casualty aggregate excess covers.

Long-tailed reinsurance business would typically include proportional casualty treaty,


casualty excess treaty, casualty aggregate excess, casualty facultative and APH.

As we start to get more significant delays in reporting and/or settlement of claims, it becomes
more important to rely slightly less on past experience, as it may no longer be such a good guide
to the future. It therefore makes sense to start using hybrid methods that place growing
emphasis on alternative data particularly for the most recent, undeveloped years.

We commonly use chain ladder-based methods, usually in conjunction with the


Bornhuetter-Ferguson, or Cape Cod approaches, with separate treatment of major
catastrophes.

The Glossary defines the Cape Cod method to be similar to the BF method where, instead of an a
priori loss ratio, it uses weights proportional to a measure of exposure and inversely proportional
to claims development.

There could be reserving problems for new or recently established reinsurers where the
oldest development year is not yet mature.

If triangulation methods are used, a tail factor will have to be applied.

In many cases, the incurred (notified) claims development pattern is more stable than the paid
claims development pattern. This is because notified claims amounts contain more information
(ie the estimates of outstanding) than paid claims alone. However, this is not always the case.

In some instances, the paid development patterns can be more stable than the incurred. It
may be worth bearing this in mind, although the most recent year will be so undeveloped
that the result of a basic chain ladder method will still be very volatile.

© IFE: 2019 Examinations The Actuarial Education Company


SP7-25: Reinsurance reserving Page 11

Question

Suggest in what circumstances the paid development pattern might be more stable than the
incurred development pattern.

Solution

This would occur when case estimates are wildly volatile or prone to error. This could be, for
example, due to inexperienced loss adjustors or high uncertainty in ultimate costs due to volatile
inflation, changes in the reserving basis over time or other factors affecting ultimate settled claim
values.

2.3 Inwards reinstatement premiums


Inwards reinstatement premiums are the premiums for the restoration of the insurance limit
of an inwards insurance contract to its full amount after a loss occurrence by the insured.
These are premiums payable to the reinsurer so are an asset rather than a liability to the
reinsurer and should be recorded and tracked separately.

From an estimation perspective, they can be calculated explicitly at a policy level in relation
to specific claims which trigger reinstatement premiums or at an aggregate level using
chain ladder-based methods. The circumstances under which reinstatement premiums are
payable are specific to individual contracts and vary with contract terms, making aggregate
methods more difficult to apply and may make actuarial projections unreliable if
reinstatement terms have changed over time (eg an increase in free reinstatements in a soft
market), or comparisons to historical years cannot be made (eg catastrophe years
compared to non-catastrophe years).

As with all premium estimation in early development, involvement of underwriters will be


important to assess the income for individual portfolios. In particular, where reinstatement
of coverage is in relation to catastrophe coverage post-event, underwriters will be involved
in providing estimates of both the claims cost payable as well as any additional
reinstatement premium receivable.

In practice, payment of claims to the insured may be net of reinstatement premiums payable
by the insured. Care should be taken that claims development triangles are not distorted by
reinstatement premium deductions. Ensuring consistency with the claims liabilities is also
important to avoid over (or under) projecting profits.

The Actuarial Education Company © IFE: 2019 Examinations


Page 12 SP7-25: Reinsurance reserving

3 Reserving for outwards reinsurance


In this section, we will consider the problem of reserving for reinsurance from the
perspective of a direct insurer wanting to reserve for its expected reinsurance recoveries
and premiums (to value the reinsurance asset on the balance sheet).

In this instance, the volume of reinsurance data could be very small or the actual materiality
of reinsurance recoveries could be low (or high), so that the range of practical approaches
can vary widely. The type of reinsurance cover in place also affects the methods used to
project ultimate losses.

The aim is to quantify the required reinsurance adjustments, ie to find the difference between the
net and gross reserves, and the net and gross premiums.

Fundamentally, the six main methods described are:


 use data gross and net of reinsurance, then find the difference
 perform standard triangulation techniques directly on reinsurance data alone
 adjust gross data using a broad brush approach
 use a case-by-case approach on only the largest losses
 develop all individual losses then apply the reinsurance to each one
 derive a reserve distribution net of reinsurance.

These are now discussed in more detail. Note the slight inconsistency with the syllabus objective
at the start of this chapter, which only mentions four methods.

3.1 Reserving using data gross and net of reinsurance

Description of approach
This is one of the more common, and simple, approaches used to allow for reinsurance
recoveries expected on a (re)insurer’s outwards programme.

We reserve for gross losses as normal, then apply the same methodology (typically chain
ladder and Bornhuetter-Ferguson, perhaps also Cape Cod) to the corresponding data
triangles net of reinsurance (both recoveries and premiums). For the Bornhuetter-Ferguson
approach, we could base the a priori assumption on recoveries as a percentage of gross
losses, if this appears more predictable than the reinsurance loss ratio.

In other words, we estimate the gross ultimate claims using the BF method. We then also
estimate the net ultimate claims using the BF method, but our a priori assumption is based on the
gross claims, which are netted down using an assumed ‘net-to-gross’ ratio. (The alternative
approach would be to use a subjective loss ratio assumption.)

We may need to consider particular large events separately, as for gross reserving.

Natural catastrophe losses, particularly very large insured events, can have a very different
development profile to other types of losses.

© IFE: 2019 Examinations The Actuarial Education Company


SP7-25: Reinsurance reserving Page 13

We can compare the resulting gross and net projected ultimate claims and premiums. The
difference represents the reserves for reinsurance.

Example
Let’s compare an attritional domestic buildings claim event and a catastrophic event, such as the
UK floods that occurred in the winter of 2015.

The development of a claim falls largely into three stages of delay:


 the delay to when the claim event is noticed
 the delay it takes to report the claim
 the delay it takes to settle the claim.

For a normal individual domestic buildings loss, let’s say a fence blowing down as a result of a
localised storm, we might expect a few days delay before the homeowner discovers that their
fence has blown down. They then wait a few days before they claim (it’s only a small claim, after
all, so not really urgent). The insurer checks the validity of the claim, and a few weeks later at
most, full and final payment is made. Reinsurance was not involved (let’s assume the insurer was
large and so had no proportional reinsurance).

For the catastrophic flood, however, the event is noticed immediately. The first response is for
the policyholder to arrange emergency items  sandbags, alternative accommodation and so on.
Claiming is not on their mind at this stage, although the insurer will undoubtedly be setting up a
large IBNR reserve in anticipation. Eventually, once the initial urgency has died down, the
policyholder will notify the insurer of the claim, although the amount is uncertain at this stage as
they do not know the repair costs. The insurer might at this stage start making partial payments
so that the policyholder can fund their ongoing repairs. Eventually, perhaps after a year or so,
once the household is dried and repaired, the final payments can be made from insurers. The
insurer will have notified the (cat XL) reinsurer of impending claims early on, although again,
recoveries will only be made once there is some certainty of ultimate claims costs.

Advantages and disadvantages


Advantages include:

 simple to apply and understand

 simple to add to a semi-automated reserving process


Such a process might involve, for example, an automatic feed of the net and the gross
claims data into commercially-available reserving software.
 we can use it to assess the volatility of net outcomes (but not of reinsurance
recoveries per se)
Any volatility of reinsurance recoveries, for example between different cohorts or rating
groups, will be hidden within the overall net result.

The Actuarial Education Company © IFE: 2019 Examinations


Page 14 SP7-25: Reinsurance reserving

 appropriate for proportional reinsurance or very high excess reinsurance where


there are relatively few reinsurance recoveries made
This is because any projection of net claims amounts will require relatively stable net
data.
 appropriate where the reinsurance programme has been relatively stable over a
number of years
Any changes to the reinsurance programme are likely to affect the development pattern
of net claims, making triangulation methods inappropriate.
 simple to adjust the method to allow for major catastrophes.

The disadvantages of this method stem from the fact that the method works on aggregated
triangles without looking at the effects of each reinsurance treaty separately.

Disadvantages include:

 possibility of implied negative reinsurance recoveries (net reserves higher than


gross)

Question

Explain why this method might imply negative reinsurance recoveries.

Solution

Each reserving exercise (ie net and gross) will make choices about development patterns
(amongst other things, inflation from year to year). For example, the insurer may routinely
include some prudence in its gross case estimates with the result that a downward trend emerges
towards the end of the development period. Meanwhile, the corresponding net data may not
reveal a downward trend (for example, because the reinsurance programme may already have
capped claims at a lower amount).

Consequently, while the overall figures across all years of account may be a reasonable estimate
of the ultimate position, for any one cohort it could mean that the net factor-to-ultimate is
considerably higher than the gross factor-to-ultimate, resulting in what appears to be negative
recoveries.

 where reinsurance protections have changed (for example, change of type of


reinsurance purchased or material change in retention levels for excess of loss
programmes), it may not be appropriate to apply the net of reinsurance development
patterns prior to the change after the change
This could be overcome by making manual adjustments to the net and gross figures
before the triangulations are performed. For example, the net data could be adjusted to
act as if the current reinsurance programme had applied all along.

© IFE: 2019 Examinations The Actuarial Education Company


SP7-25: Reinsurance reserving Page 15

 may be less appropriate for non-proportional covers


For example, if the excess points have not been inflated at the same rate as the gross
claims, then the net claims severity and development patterns will not have remained
stable.
 cannot accurately allow for some features of individual reinsurance contracts such
as aggregate limits, aggregate retentions and profit commissions
If an individual reinsurance contract has such an aggregate feature, for example a profit
commission, then this method would not automatically be able to take this into account
since it works only on aggregate triangles.
 cannot accurately allow for claims that breach the vertical cover available unless
these are adjusted for separately (eg major property catastrophes)
These could be extracted from the triangles and dealt with separately so that the
development pattern is not distorted.
 will not allow accurately for interaction between whole account and line-of-business-
specific covers or stop-loss covers (some, one or all of the line-specific covers may
inure to the benefit of whole accounts, ie the reinsurance covers may produce
payments under certain lines)  such cases need to be analysed separately
A cover is said to inure to the benefit of a second contract when it acts to reduce claims
before the terms of the second contract are applied. For example, if an excess of loss
treaty inures to the benefit of a quota share treaty, then under the quota share, it will be
claims net of the excess of loss treaty that will be split between insurer and reinsurer.
 the lack of direct link between the gross and net experience could lead to
inconsistent results for capital / enterprise risk management
In other words, because the effects of each reinsurance arrangement are not looked at
separately, incorrect decisions on capital (or risk) management might be made.
 does not permit accurate assessment of credit risk
This is because the volatility of claims made against an individual reinsurer cannot be
assessed.
 may not adequately reflect changes in the gross book of business.

Note: some of the advantages and disadvantages under this section may apply under
Section 3.2 also, eg the fifth and sixth advantages and the fourth and sixth disadvantages.

3.2 Applying standard reserving techniques to reinsurance premium and claims


data triangles

Description of approach
Another relatively common approach is to triangulate reinsurance premium and recovery
data, usually on a consistent basis with the gross data. We then apply standard reserving
techniques (for example, chain ladder and Bornhuetter-Ferguson) to the reinsurance data to
estimate ultimate reinsurance premiums and recoveries.

The Actuarial Education Company © IFE: 2019 Examinations


Page 16 SP7-25: Reinsurance reserving

Where reinsurances are purchased on a whole account basis, rather than line-specific, it
may be more appropriate to reserve for reinsurance on a whole account, rather than
line-of-business, basis. This may not be consistent with the grouping of data for the gross
reserving. We may need to allocate the resulting projected ultimate recoveries and
premiums back to line of business for accounting and regulatory reporting purposes. As a
starting point, we could do this in proportion to the reported incurred reinsurance
recoveries, and adjust the result if necessary (for example, if the resulting allocation
appears inequitable for a particular line of business and/or year). In practice, even when
reinsurances are purchased on a whole-account basis, future changes in reserves will
probably be considered by class.

Advantages and disadvantages


Advantages include:

 relatively simple to understand

 simple to add to a semi-automated reserving process

 can be used to assess volatility of reinsurance recoveries (for example, using


bootstrapping, Mack, stochastic chain ladder and so on) and so also assess credit
risk

 simple to adjust the method to allow for major catastrophes.

Most of the disadvantages of this method are the ‘reinsurance equivalents’ of those applying
when reserving for primary business, except that the data limitations discussed in Section 1 also
have an effect.

Disadvantages include:

 it may be hard to assess development patterns as data can be sparse (particularly


for higher retention non-proportional covers)

 where reinsurance protections have changed (for example, change of type of


reinsurance purchased or material change in retention levels for excess of loss
programmes, excess points in particular) it may not be appropriate to apply the
reinsurance recovery development patterns prior to the change after the change
This is because the net claims development pattern is an implicit mix of the gross and the
reinsurance development patterns. The effect on the net claims data of any change in the
reinsurance program will therefore be masked by the (probably weightier) gross claims
data. In contrast, by considering the reinsurance development pattern in isolation we
avoid this diluting effect.
The change in development patterns here may be more marked than for Section 3.1,
but because of the sparseness of the data, this can still be hard to detect or assess.

 changes in reinsurer panel can change payment development patterns and this
method will not capture this accurately
In other words, different reinsurers will pay reinsurance recoveries at different speeds.

© IFE: 2019 Examinations The Actuarial Education Company


SP7-25: Reinsurance reserving Page 17

 this approach does not allow accurately for individual contract aggregate features

 this approach does not allow accurately for individual claims breaching limits in
vertical cover unless we adjust for the claims separately

 we cannot be sure that the gross and net positions are consistent. If we extend the
approach to look at volatility, we are unable to match a particular gross scenario to
the corresponding reinsurance reserve outcome and so have a consistent net result.
This problem arises because you are reserving on an aggregate basis. Aggregate triangles
may look predictable, but this may hide some underlying volatility. Volatility is a key
driver of capital requirements.

Note: some of the advantages and disadvantages under this section may apply under
Section 3.1 also, eg the fourth advantage and the first and fourth disadvantage.

3.3 Applying broad brush factors to projected gross data

Description of approach
This approach can help us to deal with more complex reinsurance programmes with a
mixture of proportional and non-proportional covers.

However, we will see that whilst the method is useful for proportional covers, if we are
considering a non-proportional reinsurance programme, it doesn’t offer any advantages over the
methods described in Section 2 (reserving for inwards reinsurance).

The method may also be suitable when only an approximate reserve calculation is required,
perhaps for a small or short-tailed line of business.

For proportional business in most instances, we should know the cession percentage. The
ultimate recoveries should be that percentage of the projected ultimate gross losses.

For example, if we have a 20% quota share arrangement, then the reinsurance recoveries will
simply be 20% of the gross losses.

Question

Explain why a similar argument would not hold for surplus treaty reinsurance.

Solution

This is slightly more complicated, as the proportion ceded for each risk will vary, depending on the
number of lines used. However, it should be possible to calculate an overall percentage split fairly
easily, either exactly from policy records, or approximately.

We can calculate ultimate reinsurance premiums in a similar fashion, but we should use the
cession rate net of ceding commission (if the insurer accounts for ceding commission as a
premium item). We may be able to allow for loss ratio aggregate cover limits, and sliding-
scale ceding commissions, as both ultimate recoveries and corresponding premiums
should be available.

The Actuarial Education Company © IFE: 2019 Examinations


Page 18 SP7-25: Reinsurance reserving

In some cases, there could be more than one proportional contract for the line of business
(or for the data grouping being used for the reserving analysis). That may make the
underlying calculations (of cession rates) less straightforward, but it should be possible to
allow for them. Cession rates could be an input to the reserving process. We should only
need to set up the values (or formulae for the value) by line and year once at the start of the
year, and adjust these if new cover is purchased at a later date.

Cession rates is another name for ‘proportion ceded’.

For non-proportional business, we can apply the approach described in Section 2 for
inwards reinsurance.

Advantages and disadvantages


Advantages include:

 more accurate treatment of proportional business

 still reasonably simple.

Disadvantages include:

 all the issues with Section 2 (reserving for inwards reinsurance) still apply for
non-proportional business (complexity, eg order of operation of contracts, types of
cover)

 requires the ability to identify reinsurance premiums and recoveries data (paid and
case reserves) by cover type and line of business

 requires more information, such as detailed knowledge of current and historical


proportional covers (a good idea but adds to the data and resource requirements).

3.4 Case-by-case reserving

Description of approach
This approach is arguably the simplest of all. We only reserve for reinsurance on the
largest, most material losses.

Reinsurance recoveries on smaller losses are ignored completely.

We set the reinsurance reserves on a case-by-case basis, after discussing these with claims
handlers and underwriters, and perhaps with reference to any previous experience for events of
that type. Typically we might reserve in this way for losses from the major property catastrophe
(storm and quake) events, plus perhaps unusually large single risk events (explosions, fires and so
on). This may also be a suitable approach for reserving for facultative reinsurance business.

© IFE: 2019 Examinations The Actuarial Education Company


SP7-25: Reinsurance reserving Page 19

Advantages and disadvantages


Advantages include:

 simple

 consistent with corresponding gross losses (for those losses for which adjustments
are made)

 appropriate for catastrophe covers and high excess reinsurance.

Disadvantages include:

 requires detailed knowledge of current and historical reinsurance covers

 need to estimate losses that have not yet occurred (ie separate allowance needs to be
made for recoveries on large IBNR claims)
 not practical for stop loss, proportional reinsurance or working layer
non-proportional reinsurance.

3.5 Developing individual losses and applying the reinsurance programmes to


them

Description of approach
This approach is the most complex and time-consuming.

We develop each individual loss to ultimate settled value. We run each loss through all the
applicable reinsurance contracts to calculate the ultimate recoveries. We track the
recoveries for each contract and aggregate them so that we can apply aggregate limits and
retentions. We can also calculate reinstatement and additional premiums.

We aggregate the recoveries and premiums by line and year and compare them to paid and
incurred recoveries and premiums to calculate the reinsurance reserves.

One of the better ways to utilise this approach would be to use stochastic reserving at
individual claim level, to develop the gross losses. We can then calculate a distribution of
reinsurance recoveries as well as an expected value. This is essential where regulation
requires identification of particular percentile outcomes.

Advantages and disadvantages


Advantages include:

 this method should treat the features of each contract accurately

 consistent with corresponding gross losses

 allows the assessment of the distribution of reinsurance recoveries

 can allow for complex programmes, such as for stop losses and aggregate covers
as part of the process.

Disadvantages include:

 complex and time consuming to set up and so not suitable for simple programmes

 development of individual losses is a non-trivial and not uncontroversial process

The Actuarial Education Company © IFE: 2019 Examinations


Page 20 SP7-25: Reinsurance reserving

There can be a significant element of subjectivity when developing individual losses


(particularly large ones).
 how to allow for reinsurance recoveries for unreported claims

 requires detailed knowledge of all current and historical covers

 order of operation of contracts

 types of cover.

3.6 Net distributions


In this section ‘gross’ and ‘net’ refer to claim amounts before and after allowing for reinsurance
recoveries.

We can use the following simple approaches to derive a reserve distribution net of
reinsurance under non-proportional covers:

 Derive a gross distribution, and scale down the distribution such that the mean
equals the net best estimate. Under this method, we will usually overestimate the
uncertainty surrounding net reserves because reinsurance protection will dampen
down the volatility associated with individual large claims.

 Estimate a distribution of reinsurance to gross reserve ratios and apply this to the
gross reserve distribution.

 Use a net triangle rather than a gross triangle to derive the predicted distribution.
Under this method, we will underestimate volatility if (for example) reinsurance
retentions are increasing for the more recent origin periods and overestimate
volatility if, for example, reinsurance retentions are decreasing for the more recent
origin periods.

 Simulate individual claims and net down explicitly before aggregation.

Question

(i) Give an example to explain how reinsurance would have the effect described in the first
bullet point.

(ii) Demonstrate how the method in the second bullet point would work.

Solution

(i) Scaling down the reserve distribution

Suppose that we have excess-of-loss reinsurance in place with no upper limit. We may find that
the overall effect of the reinsurance is to halve the average claims cost. So we can reliably
estimate the mean of the claims distribution by halving the gross mean claims estimate.

However, because the large claims are capped by the reinsurance, the variance of the net claim
amounts may be much smaller in percentage terms than the variance of the gross claim amounts.
So, if we simply halved the standard deviation of the gross claims to estimate the standard
deviation of the net claims, this would give an overestimate.

© IFE: 2019 Examinations The Actuarial Education Company


SP7-25: Reinsurance reserving Page 21

Note that if the reinsurance consisted of a simple quota share agreement, this method would
work nicely, since the distribution and moments of the gross and net claims distributions would all
be related proportionately.

(ii) Reinsurance to gross ratios

Here we would subdivide the claims distribution by size. For example, we might find that, with
the reinsurance arrangements we have in place, we typically experience the following ratios:

Gross claim amount Ratio (net claims / gross claims)


< £50k 90%
£50k – £1m 75%
£1m – £5m 50%
£5m – £50m 10%
> £50m 0%

We can then estimate a distribution for the gross claim amounts and apply the factors in the table
to the different ranges to obtain a distribution for the net claims.

3.7 Combinations of methods


In practice it is likely that a variety of methods will be used to allow for reinsurance
recoveries, depending on the specific factors that apply for different lines of business and
their related reinsurance programmes.

Combinations of methods are also likely to be used depending on the component of


reserves under consideration and the methods used.

For example, recoveries on unearned or unwritten exposures can be allowed for using
reinsurance factors from business plans if available, although any seasonality issues
should be considered as appropriate.

This approach is likely to be used in conjunction with methods that focus on individual
claims, whether at current or projected values, as these methods inherently only focus on
earned exposures. It may also be useful in conjunction with, or as a check on, statistical
methods, particularly where accident period data is used.

Case-by-case approaches also only allow for recoveries on known claims, and not losses
on pure IBNR or on any IBNER that may be appropriate for the known claims. Again, a
factor-based approach to allow for any additional recoveries on IBNR or IBNER could
complement these methods.

This may be easier if reserving is carried out at a greater level of granularity, for example,
with attritional, large and catastrophe losses reserved separately. Business plan recovery
expectations (relative to business plan large loss and catastrophe expectations) may
provide appropriate factors to apply to any large loss or catastrophe IBNR, although it may
be appropriate to make some allowances for the likelihood of genuinely large events (and
any catastrophe events) to be known of at an earlier stage.

The Actuarial Education Company © IFE: 2019 Examinations


Page 22 SP7-25: Reinsurance reserving

3.8 Other considerations

Outwards reinstatement premiums


Depending on the method or methods used, there are various ways of allowing for outwards
reinstatement premiums.

If using statistical methods, reinstatement premiums may also be projected as part of


projections of reinsurance or net premium triangles, although as with the recoveries
themselves there are many factors that could limit the accuracy of this approach.

Where the analysis is carried out at the granularity of individual large losses (whether at
current claims view or on a projected basis), the reinstatements relevant to a loss can also
be calculated with reference to the applicable reinsurance programme.

If using a factor-based approach, a similar approach can be adopted for reinstatement


premiums. A reinstatement factor can be applied to the assumed level of recovery – this
should be a blended factor if the assumed recoveries are intended to be a probability
weighted distribution of potential losses, rather than applying the reinsurance programme
to the modelled recoveries as if they were the best estimate (which would likely overstate
the reinstatements as these are likely to be higher on lower layers of reinsurance).

If there are a number of known losses already, this should be considered in any allowance
for reinstatements as the final recoverable loss would not be eligible to be reinstated.

The timing of reinstatement premiums should be linked to the losses that are assumed to
generate the reinstatements.

Losses occurring during vs risks attaching during


The basis of reinsurance cover will have implications for the development of reinsurance
recoveries when considered at the level of individual reinsurance programmes.

Losses occurring during (LOD) contracts will be faster earning and will be fully earned as of
the end of the period of cover. Risks attaching during (RAD) contracts will continue to earn
until the expiry of the last policy that incepted during the period of cover, operating more
like a binding authority earning pattern.

While this is highly relevant when considering the individual contracts themselves, it may
not be as significant when allowing for outwards reinsurance recoveries in a typical
reserving exercise.

In most instances there will be an allowance for future management actions that continue to
purchase comparable reinsurance cover to protect the run off of risks in scope after the
current in-force reinsurance has expired. There will also be a recognition that previous
programmes will continue to provide cover on RAD programmes.

If the reinsurance programme is assumed to be relatively consistent across years and there
is no need to determine recoveries at the granularity of individual years’ programmes (eg if
reserving is just carried out at a class and accident / underwriting year level) then there may
be no need to make allowances for individual contract terms.

© IFE: 2019 Examinations The Actuarial Education Company


SP7-25: Reinsurance reserving Page 23

Where there is a material change in the reinsurance, some allowances will need to be made.
As noted above, material changes to reinsurance are likely to invalidate any statistical
methods used to estimate recoveries, and any case-by-case approach can already consider
the programmes in scope at the time, so factor-based approaches are most likely to be
impacted.

If an estimate can be made of the proportion of the gross losses that fall into different
reinsurance structures (eg a simple linear allowance recognising inception and expiry
dates), different factors can then be used for different components.

Indexation clauses
The intention of an indexation clause is to offset likely claims inflation so that the return
period of any reinsurance attachment remains similar over time.

If this is adopted as a reasonable working assumption, there may not be a need to make an
explicit allowance for indexation clauses with some methods. Statistical methods would
inherently capture the interaction between inflation and indexation. Factor-based methods
should remain robust as the indexation will keep the appropriate factors stable over time.

Case-by-case methods (including projections on individual known claims) may benefit from
some allowances for indexation if there is a particular view of the time at which a claim may
settle, although this should be consistent with the approach used to set the estimated claim
value. Discussions with the claims teams and underwriters may help to clarify likely
timescales.

Reinsurance exhaustion
Many reinsurance programmes will have a limited number of reinstatements after which
cover is exhausted. This should always be considered in any method applied.

For statistical methods, if the history shows a trend for further recoveries and
reinstatements after a point in time, this should not be applied without modification to years
where reinsurance is exhausted or has a high risk of becoming exhausted.

Factor-based methods should allow for the possibility of exhaustion, and if experience to
date increases the likelihood of exhaustion then consideration should be given to amending
the factors used.

Case-by-case methods should accurately capture any exhaustion in the detailed calculation
of recoveries.

As noted above, exhaustion also affects potential reinstatements which will stop becoming
payable on the last tranche of available recovery before final exhaustion.

For example, if an excess of loss layer has two reinstatements, then once the layer has been fully
reinstated for the second time, no further reinstatement premiums will be payable even though a
further full loss can still be recovered from the reinsurer.

The Actuarial Education Company © IFE: 2019 Examinations


Page 24 SP7-25: Reinsurance reserving

4 Glossary items
Having studied this chapter you should now read the following Glossary item:
 Cape Cod method.

© IFE: 2019 Examinations The Actuarial Education Company


SP7-25: Reinsurance reserving Page 25

Chapter 25 Summary
Additional data issues for inwards reinsurance reserving
Data can be a problem particularly for reinsurers because:
 claim reporting delays are longer
 there is a greater tendency for claims to develop upwards
 exposure can be very heterogeneous
 data can be sparse
 benchmarks are often less relevant
 there can be IT constraints
 there is more opportunity to group data differently.

Reserving for inwards reinsurance


For most reinsurance reserving exercises, paid claims development patterns are more
reliable than incurred.

For short-tailed business, normal triangulation (or even simpler) methods may well be used,
although catastrophes should be treated separately.

For medium and long-tailed business, we can use the same methods, but taking extra care
where development years have not matured, for example by using hybrid methods. We
should consider both paid and incurred development patterns.

Reserving for outwards reinsurance


The six main methods are:
 use data gross and net of reinsurance, then find the difference
 perform standard triangulation techniques directly on reinsurance data alone
 adjust gross data using a broad brush approach
 case-by-case approach on only the largest losses
 develop all individual losses then apply the reinsurance to each one
 derive a reserve distribution net of reinsurance.

The Actuarial Education Company © IFE: 2019 Examinations


Page 26 SP7-25: Reinsurance reserving

Chapter 25 Summary continued


The suitability of any method can be assessed by considering:
 its simplicity (eg how easily it can be added to an automatic reserving process) versus
its accuracy
 the consistency of gross and net estimates
 whether it can be used to assess volatility of net outcomes or reinsurance recoveries,
eg by using stochastic techniques such as bootstrapping
 how the method copes with:
– different types of reinsurance, eg proportional, non-proportional, working
layer, high layer
– sparse data
– changes in reinsurance programme or panel over time
– reinsurance recoveries on unreported claims
– catastrophes and large claims, eg whether it allows for claims that breach the
vertical cover
– aggregate features such as profit commissions, and loss-sensitive contracts
such as stop loss cover
– interactions between covers, eg whole account covers
 whether the method can be used to investigate:
 capital requirements and enterprise risk management
 credit risk.

© IFE: 2019 Examinations The Actuarial Education Company


SP7-25: Reinsurance reserving Page 27

Chapter 25 Practice Questions


25.1 Determine an appropriate method (based on the five broad methods listed below) for the
following reinsurance reserving scenarios:
(a) household contents insurance covered by only a quota share treaty
(b) high risk commercial liability business covered by a range of reinsurance treaties,
including XL reinsurance that includes a stability clause.

Methods:
1. use data gross and net of reinsurance, then find the difference
2. perform standard triangulation techniques directly on the reinsurance data alone
3. adjust gross data using a broad brush approach
4. case-by-case reserving on only the largest losses
5. develop all individual losses then apply the reinsurance programme to each one.

The Actuarial Education Company © IFE: 2019 Examinations


Page 28 SP7-25: Reinsurance reserving

The solutions start on the next page so that you can


separate the questions and solutions.

© IFE: 2019 Examinations The Actuarial Education Company


SP7-25: Reinsurance reserving Page 29

Chapter 25 Solutions
25.1 (a) Household contents insurance covered by only a quota share treaty

The business is short tailed. Original claims will be numerous but recoveries will be minimal apart
from catastrophes. We can use a simple method such as Method 1, 2 or 3 but treat catastrophes
separately. Other methods are too complex.

(b) High risk commercial liability business covered by a range of reinsurance treaties, including
XL reinsurance that includes a stability clause.

This is more complex. The programme is likely to have changed over time. Claims will be volatile
and there will be accumulations and many recoveries. Methods 3 and 4 are possible but given the
complexity, Method 5 is most appropriate.

The Actuarial Education Company © IFE: 2019 Examinations


All study material produced by ActEd is copyright and is sold
for the exclusive use of the purchaser. The copyright is
owned by Institute and Faculty Education Limited, a
subsidiary of the Institute and Faculty of Actuaries.

Unless prior authority is granted by ActEd, you may not hire


out, lend, give out, sell, store or transmit electronically or
photocopy any part of the study material.

You must take care of your study material to ensure that it


is not used or copied by anybody else.

Legal action will be taken if these terms are infringed. In


addition, we may seek to take disciplinary action through
the profession or through your employer.

These conditions remain in force after you have finished


using the course.

The Actuarial Education Company © IFE: 2019 Examinations


SP7-26: Accounting methods Page 1

Accounting methods
Syllabus objectives
5.10 Describe the methods and principles of accounting for general insurance business
and interpret the accounts of a general insurer.

5.11 Describe the changes to accounting methods expected under IFRS.

Covered in part in this chapter.

The Actuarial Education Company © IFE: 2019 Examinations


Page 2 SP7-26: Accounting methods

0 Introduction
There are two distinct methods used by general insurers to present their accounts:
 annual (or accident year) accounts, which consider all income earned and outgo incurred
in a year and permit the release of profits at the end of that year
 funded (or underwriting year) accounts, which consider the business written in each year
and do not permit the release of profits until the end of a subsequent year (usually the
third year).

Section 1 gives a broad overview of the two methods, which are then discussed in detail in
Sections 2 and 3. In Section 4 we discuss whether the accounts of a general insurance company
reflect the true underlying profitability of the company. Section 5 goes on to explain how to
construct simple accounts.

This chapter contains a lot of additional explanation and a number of questions, to help illustrate
the Core Reading and explain how these principles work in practice.

© IFE: 2019 Examinations The Actuarial Education Company


SP7-26: Accounting methods Page 3

1 Methods of accounting

1.1 Annual accounting


Annual accounting is the basis of accounting for general insurance business in the UK.

Annual accounts are usually used for most forms of direct insurance (eg motor, household,
employers’ liability, commercial property).

Under annual accounting, we consider all income earned and outgo incurred in a year and
can release profits at the end of the year. That is, income is recognised as earned - and the
consequence is that profit or loss might arise from the business earned on that year.

We carry forward unearned income into the next accounting period. The premium element
of this is called unearned premium and unearned premium reserves and unexpired risk
reserves are created.

For example, if an annual policy is written on 1 September 2018, then for the 2018 calendar year,
the earned premium will be one-third of the premium (assuming uniform risk over the policy
year) and the other two-thirds of the premium will be carried forward into the 2019 calendar
year.

We also refer to this approach as ‘deferral and matching’ because we defer profit release
until the accounting period when the contracts are exposed to the risk of insured events.

Under the accrual principle (as opposed to a cash basis), profit is crystallised in the year the
premium is earned. If you have a two-year policy where the risk is spread evenly, you earn
half the premium in the first year and half in the next, so that the income recognition is
matched to the risk taken in each period. Out of this flows the profit or loss made on the
business.

Annual accounts have traditionally been referred to as one-year accounts. You may also see them
referred to as accident year accounts.

The term accident year refers to a grouping of claims according to the year in which the loss event
actually occurred, irrespective of when they are reported or paid, and the year in which cover
commenced.

1.2 Funded (three-year) accounting


An alternative accounting approach, known as ‘funded accounting’ or ‘three-year
accounting’, was previously used by Lloyd’s of London and London Market companies
operating in the Lloyd’s market.

Funded (or three-year) accounts were introduced earlier in the course. They are typically used for
Lloyd’s, reinsurance and marine and aviation business.

Under funded accounting, we considered the business written in each year and could not
release profits until the end of three years.

Lloyd’s of London now also uses an annual accounting basis, but the legacy of funded
accounting remains in internal financial management as the underwriting year accounting
approach is consistent with the way Lloyd’s syndicates manage capital.

The Actuarial Education Company © IFE: 2019 Examinations


Page 4 SP7-26: Accounting methods

Funded accounting is used in Lloyd’s when we calculate the reinsurance to close (RITC)
and release profits to the Lloyd’s Names.

It is also used by companies when we calculate the emerging profit and when we calculate
underwriting and reserve risk, which may be reduced to reflect investment income that will
be earned on assets held against reserves and on premiums received in relation to the
current and prior underwriting years.

Funded accounts are also referred to as underwriting year accounts.

The term underwriting year refers to a grouping of claims according to the year in which cover
commenced, irrespective of when the loss event occurred, and when the claims are reported or
paid.

1.3 Reserving models


We typically associate accident year reserving models (based on accident year cohorts)
with annual accounting, and we associate underwriting year reserving models (based on
underwriting year cohorts) with funded accounting. However, many Lloyd’s syndicates and
London Market companies continue to use underwriting year approaches, but then apply an
adjustment to determine reserves on an annual accounting basis.

The process of converting underwriting year results to accident year results is discussed further in
Section 3.3.

© IFE: 2019 Examinations The Actuarial Education Company


SP7-26: Accounting methods Page 5

2 Annual accounting

2.1 Introduction
In the annual accounting model for insurance contracts, we need to consider the following:

 Income – earned premiums, reinsurance recoveries received or accrued when the


relevant claim has been recognised / paid, investment income, and changes in
premium reserves / risk reserves / reinsurance reserves.

 Outgo – claims, claims handling expenses and other expenses paid and changes in
claims outstanding (including IBNR) in the accounting period, reinsurance
premiums, commissions, profit commissions, underwriting charges / taxes.

 Assets – deferred acquisition costs, reinsurers’ share of unearned premium


reserves, reinsurers’ share of claims outstanding.
When policies are written the insurer pays commission and other initial expenses. At the
accounting date those acquisition costs have been paid out but not wholly incurred for
any policy that is unexpired at the time, just as part of the premium received has not yet
been earned. (You could think of them as ‘unincurred expenses’, but the name ‘DAC’ is
universally used.) DAC is usually a significant asset in the insurer’s accounts. It is like a
negative reserve. Decreasing DAC reduces the insurer’s profit, just as increasing reserves
does.
 Liabilities – unearned premium reserve, additional unexpired risk reserve, claims
outstanding.

 Debtor and creditor balances reflecting outstanding payments due to or from


policyholders, brokers and reinsurers.

Alternatively one can consider the building blocks as the premium (involves income) and
the risk assumed (reflected in a liability)  the model is based on the insurer being able to
price the risk sufficiently well so that after paying the claims / paying the admin and getting
the return on the investment of the money paid in premium, they will be able to make some
profit.

The precise layout of the published accounts will vary depending on the accounting rules in place.

2.2 The technical account


The technical account (or revenue account) shows the basic trading profit from writing insurance
business for a given period. Note that strictly speaking, this terminology is only needed for
Subject SA3 (it has an asterisk in the Glossary).

To develop a clear understanding of the intricacies of a general insurer’s accounts, it is useful to


start with the most basic model of how profits are made up.

In its most basic form, profit for a given year can be expressed as:
profit = money in – money out – increase in reserves

For example, if a company has total receipts of £10m and outgo of £7m, but had to set aside a
further £2m in reserves (eg for outstanding claims) then profit would be £1m.

The Actuarial Education Company © IFE: 2019 Examinations


Page 6 SP7-26: Accounting methods

If we now identify all the main components of these three headings then we have all the
ingredients of the revenue account.

Gross premiums written


Money in A Reinsurance and other recoveries
Investment income on insurance funds
Reinsurance commission received
Gross claims paid
Money out B Reinsurance premiums paid
Expenses paid
Commission paid
Increase in outstanding claims reserves
Increase in reserves C Increase in unearned premiums
Decrease in deferred acquisition costs

The conventional format


We could calculate the basic trading result for the year from writing insurance business as all the
items in A less all the items in B and C. Although the answer would be correct, it is not, as it
stands, in the conventional layout.

The conventional format for preparing insurance company revenue accounts is as follows:

Premiums
– Claims
– Expenses
+ Increase in DAC
= Underwriting result
+ Investment income
= Insurance result

© IFE: 2019 Examinations The Actuarial Education Company


SP7-26: Accounting methods Page 7

We construct the revenue account as in the left-hand column below. The right hand column is
included to show the detail of all the components in each item of the account.

Premiums earned gross premiums written


– reinsurance premiums paid
= net premiums written
+ unearned premiums brought forward (gross of DAC,
net of reinsurance)
– unearned premiums carried forward (gross of DAC,
net of reinsurance)
= net premiums earned
– Claims incurred gross claims paid
– reinsurance and other recoveries
+ outstanding claim reserve carried forward (net of
reinsurance)
– outstanding claim reserve brought forward (net of
reinsurance)
= net claims incurred
– Expenses (paid) commission paid
+ expenses paid
– reinsurance commission received
= net expenses paid
+ Increase in DAC deferred acquisition costs carried forward (net of
reinsurance)
– deferred acquisition costs brought forward (net of
reinsurance)
= Underwriting profit
+ Investment income ie investment income on insurance funds
= Insurance profit

You should now check back to satisfy yourself that all the items in our initial ‘money in/out’ model
have been incorporated within this revenue account and that the correct signs have been used
throughout. A typical mistake is to put the wrong sign on the reserve items. You should always
be able to check quickly from first principles which way round the signs go, eg any increase in
reserves will decrease profits. Another approach that you may find useful is to consider first how
special cases must work, eg a new company in its first year, with no reserves brought forward.

We will now look at these items in more detail.

The Actuarial Education Company © IFE: 2019 Examinations


Page 8 SP7-26: Accounting methods

Earned premiums
The earned premiums are the total premiums attributable to the exposure to risk in an
accounting period. They are different from written premiums, some of which may provide
cover outside the accounting period.

For annual policies, the premiums earned in the latest accounting year, and hence allocated
to that year, will include part of the premiums written in that accounting year and part of the
premiums written in the previous accounting year. Policies that provide cover for periods
longer than one year may contribute to the earned premiums in several years of account.

Thus the earned premiums for the accounting year will be:

unearned premiums brought forward from earlier years

+ premiums written during the year

 unearned premiums carried forward to later years

One-year accounts consider all income earned and expenses incurred in a year. The exposed to
risk period is completed at the end of the year, since any subsequent events will be included in
the exposure for some later year.

Claims outstanding
At the start of this section, we saw how the profit in a given year is reduced by transfers to
reserves. So the bigger the increase in outstanding claims reserves, the smaller the profit.

This is clear if you rewrite the claims incurred in the equivalent form:
claims incurred = claims paid + increase in claim reserves

The figure for ‘outstanding claims brought forward’ is the outstanding claims reserve as in the
balance sheet at the start of the year. The ‘outstanding claims reserves carried forward’ is the
equivalent figure from the balance sheet at the end of the year.

The claims outgo are the claims incurred during the accounting period (that is, attributable
to the exposure to risk, consistent with the earned premiums).

At the end of each accounting year, it is necessary to establish reserves in respect of all
claims that have been incurred in that year or earlier years (whether they have been
reported or not) and that are still outstanding.

The reserve for outstanding claims should include allowances for:

 outstanding claims (claims that have been reported but not yet paid)

 IBNR (claims that have been ‘incurred but not reported’)

 IBNER claims (claims that have been ‘incurred but not enough reported’, eg the
reported claim amount is incorrect, or settled claims that may reopen)
ie it is the reserve needed to cover expected increases (or decreases) in estimates for
reported claims

© IFE: 2019 Examinations The Actuarial Education Company


SP7-26: Accounting methods Page 9

 claims handling expenses (see below)

 reinsurance offset / recoveries.

The components of the outstanding claims reserve were introduced in Chapter 1. They give a
snapshot of the insurer’s liabilities on the accounting date, so they will be itemised in the balance
sheet.

The technical account summarises how these outstanding claims reserves have changed during
the year leading up to the accounting date.

The change in the claims reserves together with claim payments during the accounting year
make up the claims outgo.

Thus the claims outgo for the year of account will be:

claim payments during the year

+ reserve for claims outstanding at the end of the year

 reserve for claims outstanding at the end of the previous year

Thus the claims outgo is the movement in the estimated cost of claims, which is the sum of
(new) claims incurred in the latest accounting year and movements to the estimated cost of
claims incurred in earlier accounting years.

Question

Given the following figures, calculate the claims outgo for the year of account using each of the
methods stated above:
 claims paid in the year were £1.2m, £0.8m of which were in respect of claims incurred in
the most recent year
 the outstanding claims reserve at the end of the year in respect of claims incurred in the
most recent year is £1m
 the outstanding claims reserve at the end of the year in respect of claims incurred in
earlier years is £0.3m
 the outstanding claims reserve at the end of the previous year was £0.9m.

Solution

Method 1

Claim payments during the year were £1.2m.

The reserve for claims outstanding at the end of the year is:
£1m  £0.3m  £1.3m

The reserve for claims outstanding at the end of the previous year was £0.9m.

The Actuarial Education Company © IFE: 2019 Examinations


Page 10 SP7-26: Accounting methods

So the claims outgo for the year of account is:


£1.2m  £1.3m  £0.9m  £1.6m

Method 2

Payments on claims incurred during the year were £0.8m.

The reserve for claims incurred during the year and still outstanding is £1m.

The payments on claims incurred in earlier years is:


£1.2m  £0.8m  £0.4m

The reserve for claims incurred in earlier years and still outstanding at the end of the year is
£0.3m.

The reserve for claims outstanding at the end of the previous year was £0.9m.

So the claims outgo for the year of account is:


£0.8m  £1m  £0.4m  £0.3m  £0.9m  £1.6m

Claims handling expenses


These are claims handling expenses incurred during the accounting period.

We should establish a reserve for the future expenses of handling outstanding claims. The
expense outgo in the accounting year will in general consist of the expenses paid in that
year together with the excess of the expense reserve at the end of the year over the expense
reserve at the end of the previous year. There may, however, be an adjustment in respect of
deferred acquisition expenses.

In practice however, this adjustment is made to the earned premiums, rather than to the claims
handling expenses.

Additional unexpired risk reserves


We may deem the unearned premiums less deferred acquisition costs carried forward to be
insufficient to cover the cost of the claims and expenses that will be incurred in the period
of unexpired risk. That is, if we wrote a potentially loss-making contract (onerous contract)
in the previous period, then not only will we need to recognise any premium to be earned in
future years, but we will also recognise the reserve that we need to put aside to cover the
risk that we undertook: We should therefore establish an additional reserve for unexpired
risk.

The excess of that reserve over any corresponding reserve at the end of the previous year
will form part of the outgo in the latest accounting year.

If the company suddenly decided to weaken its basis for reserving for outstanding claims (eg by
assuming a much lower value for future inflation), then there would be a sudden boost in profits
in the year of change.

© IFE: 2019 Examinations The Actuarial Education Company


SP7-26: Accounting methods Page 11

Question

An accountant’s first assessment of an insurance company’s underwriting profits for 2018 was
£243m. After further discussion with the management of the company, the following changes
were made to the calculations:
(a) IBNR reserve increased by £3.5m.
(b) Reserve for future claims handling expenses increased by £0.3m.
(c) DAC at the year-end increased by £1.2m.
(d) An additional unexpired risk reserve for £2.6m to be set up.
(e) A transfer of £3.6m from the claims equalisation reserve to the revenue account.

Calculate the revised underwriting result.

Solution

243 – 3.5 – 0.3 + 1.2 – 2.6 + 3.6 = £241.4m

Remember that increases to reserves defer the emergence of profit, whereas DAC is a notional
asset.

2.3 The profit and loss account


The structure for a profit and loss account is:

Insurance profit

+ Other investment income Note 1

+ Profits from other activities Note 2

– Interest on loans Note 3

= Profit before taxation

– Taxation Note 4
= Profit attributable to shareholders
– Dividends Note 5
= Retained profits Note 6

The Actuarial Education Company © IFE: 2019 Examinations


Page 12 SP7-26: Accounting methods

Note 1: The treatment of investment income can give rise to considerable inconsistencies
between different companies. We will take this as investment income on the
insurer’s free reserves, ie on the assets held in excess of the insurance funds. The
investment income on the technical reserves has already been included within the
insurance profit.

Note 2: Many insurance companies have associated businesses (eg life assurance,
banking, estate agency). If so, then the consolidated profit & loss accounts would
include the contributions from these businesses. This item does not usually
feature in the exam as it has little direct relevance to general insurance.

Note 3: Because of the nature of general insurance business, shareholders’ profits are
volatile anyway. Borrowing would further increase this volatility and so insurance
companies do not generally have much long term borrowing. Shareholders would
probably find this unacceptable. The interest charge is unlikely to be large.

Note 4: The taxation amount will depend upon the basis for taxation of general insurance
companies in the country of operation.

Note 5: For a proprietary insurance company.

Note 6: You should be able to reconcile the year’s retained profit with the change in
‘shareholders’ retained profit’ between the start and end of year balance sheets.
This link between the profit and loss account and successive balance sheets has
often been used by examiners in accounts questions.

Question

State whether each of the following statements is true or false:


(a) DAC is largely made up of commission which has been paid in respect of premiums that
will be earned after the accounting date.
(b) DAC is, effectively, an asset of the insurance company.
(c) Reducing DAC as a proportion of premium income defers profit emergence.
(d) It is imprudent to make the DAC very large compared with premiums written.

Solution

All these statements are true.

© IFE: 2019 Examinations The Actuarial Education Company


SP7-26: Accounting methods Page 13

2.4 The balance sheet


The balance sheet shows the breakdown of the assets and liabilities of the company at a given
point in time (ie typically at the end of a financial year).

The Core Reading does not give a set layout for a general insurer’s balance sheet. We present one
possible format below but many are used in practice. For the purposes of the exam it is
important to know the main items that appear in the balance sheet. Any sensible layout will be
acceptable.

Fixed assets Note 1


+ Investments Note 2
+ Other current assets Note 3
= Total assets
– Current liabilities Note 4
– Deferred taxation Note 5
– Unearned premium reserve Note 6, 7
+ Deferred acquisition costs
– Outstanding claims reserve Note 8
= Shareholders’ net assets Equals shareholders’ funds below

Share capital Note 9


+ Share premium account Note 10
+ Profit and loss account Note 11
+ Revaluation reserve Note 12
= Shareholders’ funds Equals shareholders’ net assets above

Note 1: Fixed assets include the basic items the company needs to operate, such as
offices, vehicles and computer equipment.

Note 2: This includes fixed interest stocks, equities, investment properties, etc.

Note 3: Current assets will include funds held by agents and other debtors, cash and
short-term deposits.

Note 4: Current liabilities are amounts owing, for example to trade creditors, bank
overdraft and outstanding dividends.

Note 5: It is prudent to hold explicit reserves for accrued tax liabilities that will crystallise
in the foreseeable future.

The Actuarial Education Company © IFE: 2019 Examinations


Page 14 SP7-26: Accounting methods

Note 6: The unearned premiums in this account are gross of DAC. The deferred
acquisition costs have been shown separately below, rather than used to reduce
this liability item.

Note 7: If the company holds other reserves (eg additional reserve for unexpired risk),
these should be shown explicitly.

Note 8: The outstanding claims reserve includes the reserves for unsettled reported
claims, for IBNR, for claims handling expenses and for re-opened claims. The
balance sheet is very unlikely to show the separate components, although a
limited break down might be included in accompanying notes.

Note 9: The share capital is the number of shares multiplied by the par value of each
share.

Note 10: The share premium account is the aggregate of the excess paid for the shares
(when issued) above the share’s par value.

Note 11: This item is increased each year by the retained profit from the profit and loss
account. This explains why this item is called ‘retained profits’ in some insurers’
balance sheets.

Note 12: Revaluation reserve (or possibly called investment reserve) is increased each year
by the net write up for unrealised gains. If asset values fall sharply, the company
might reduce the revaluation reserve.

In many exam questions, the balance sheet is likely to be given in a very simplified form, for
example:

total assets
– UPR net of DAC
– outstanding claims reserves

 Shareholders’ funds

This simplified balance sheet implicitly assumes that the total assets are net of current liabilities.

Question

Explain what problems might arise if assets are valued at full market value in the balance sheet.

Solution

 The apparent financial strength would be very volatile (ie reflecting the volatility of
market values).
 The financial strength would be overstated if the market values were on a ‘high’ (ie this
would not be prudent).

© IFE: 2019 Examinations The Actuarial Education Company


SP7-26: Accounting methods Page 15

2.5 The current accounting regime


In the UK there are two accounting regimes that may apply:
1. All companies, including insurance companies that have any debt or equity listed on
a European stock exchange, must produce financial statements in accordance with
International Financial Reporting Standards (IFRS).
2. All unlisted companies, which include insurance companies that are unlisted
subsidiaries, may report either on an IFRS basis or in accordance with UK Financial
Reporting Standards and related guidance often referred to as UK GAAP.
Recall that GAAP stands for generally accepted accounting principles.

IFRS accounting standards are for insurers only partially developed. Under Phase I, IFRS 4:
Insurance Contracts was introduced. IFRS 4 sets out a definition for insurance contracts
and disclosure requirements but continues to permit insurers to apply their previous
measurement basis subject to a liability adequacy test and certain restrictions and rules on
changes to accounting policies.

The liability adequacy test determines whether the company’s recognised insurance
liabilities are adequate by requiring them to use current estimates of future cash flows
under its insurance contracts.

The minimum requirements under IFRS of the liability adequacy test are as follows:

 in the test, we consider current estimates of all contractual cash flows, and of
related cash flows such as claims handling costs, as well as cash flows resulting
from embedded options and guarantees

 if the test shows that the liability is inadequate, we recognise the entire deficiency as
a loss on the income statement.

UK GAAP for general insurers meets the minimum IFRS 4 requirements for a liability
adequacy test, both for outstanding claims and, through the unexpired risks reserve, for
unearned premium. UK insurers reporting under IFRS use the annual accounting model to
report their general insurance operations.

There are a few other differences between UK GAAP and IFRS  mainly because, for
instance, IFRS 4 permits a range of accounting policies from previous regimes. Hence there
is currently some divergence across the board from IFRS reporters.

For instance:

 Under UK GAAP, discounting is only permitted under very limited circumstances for
general insurers (run-off of more than four years on average for the classes to be
discounted).

 Under IFRS you could have a different definition if the previous reporting regime
was different.

 Under UK GAAP the definition of insurance contracts is the same as under IFRS, but
the disclosure requirements (information in addition to the amounts in the financial
statements) are not as onerous for general insurers.

Updated International Financial Reporting Standards (IFRS) standards for insurance were
issued in 2017, for implementation by 2021. The 2020 core reading for Subject SP7 will
include additional content on these new standards.

The Actuarial Education Company © IFE: 2019 Examinations


Page 16 SP7-26: Accounting methods

The IFRS 4 Phase II standard will be known as IFRS 17 when it comes into force, which is currently
anticipated to be on 1 January 2021. The insurance contract standard was published by the
International Accounting Standards Board on 18 May 2017.

The IFRS 17 standard establishes principles for the recognition, measurement, presentation and
disclosure of insurance contracts within the scope of the Standard. Its objective is to ensure that
an entity provides relevant information that faithfully represents those contracts. This
information gives a basis for users of financial statements to assess the effect that insurance
contracts have on the entity’s financial position, financial performance and cashflows.

The IFRS 17 standard introduces new ways of measuring insurance contracts. The default model,
also referred to as the Building Block Approach, measures an insurance contract by breaking it down
into individual blocks which cover:
 the future cashflows that arise from the provision of the insurance contract
 a discount rate to reflect the time-value of money
 a risk adjustment element for non-financial risks
 a ‘Contractual Service Margin’ that represents unearned profit.

The IFRS 17 standard is broadly in line with the requirements for measuring insurance liabilities
under Solvency II, although at a detailed level there are significant differences.

The standard is covered in more detail in Subject SA3.

If you are interested in the details of IFRS 17, you can find more information on the International
Accounting Standards Board website, www.iasb.org.uk.

© IFE: 2019 Examinations The Actuarial Education Company


SP7-26: Accounting methods Page 17

3 Funded (three-year) accounting


In Section 2 we saw how one-year accounts are based on the premiums earned within a year. The
structure of funded accounts is quite different. Funded accounts are based on premiums written
in a given year. The premiums for policies written in a particular year are kept in a special fund
for a period that is specified at outset. The most common period is three years, hence the
alternative name of three-year accounts. At the end of the third year the profits for that
particular block of business are assessed.

This chapter and the Core Reading use the terms funded accounting and three-year accounting
interchangeably. However, it is important to remember that other accounting periods (eg two or
four years) are also used on a funded basis.

3.1 Example of three-year accounts


Suppose an insurer commences business on 1 January 2016, accounting for profits on a
three-year basis.

All the premiums for business written in 2016 are notionally kept in a separate fund and all claims
and expenses relating to this business are paid from this fund. We shall refer to the amount in
this fund at time X as Fund2016,X

At the end of the first year of trading (ie at the end of 2016), there is one fund of amount
Fund2016:31/12/2016

During 2017, a new fund is started which will hold all premiums and cover all claims and expenses
in respect of business written in 2017. Note that the 2016 fund will still be maintained to account
for premiums, claims and expenses relating to business written in 2016. At the end of 2017 there
will be two funds: Fund2016:31/12/2017 and Fund2017:31/12/2017.

A third fund is opened on 1 January 2018 for business written in 2018. All the premiums, claims
and expenses for these policies are accounted for within the 2018 fund.

The convention with funded accounts is to ‘close’ a fund at the end of its third year. (Hence,
‘three-year accounts’.) So at 31 December 2018, the 2016 account will be closed.

Closing an account year


When the account is closed, an assessment is made of the profit arising from that tranche of
business.

Consider the 2016 fund in the above example as at 31 December 2018. Suppose that the fund
stands at £12 million. There will almost certainly be some residual liabilities at this date for the
business written in 2016 (eg IBNR and other outstanding claims) so we cannot take the entire
£12 million as profit.

The Actuarial Education Company © IFE: 2019 Examinations


Page 18 SP7-26: Accounting methods

The convention at Lloyd’s is to pay a premium to another account to pass on (or reinsure) these
liabilities. The expression for this type of transaction is ‘reinsurance to close’, (or RITC). Although
there are specialist reinsurers who accept this type of business, it is quite common to ‘reinsure’
the liabilities from a fund being closed within the next ‘open’ fund. So, in our example, there
could be a payment from the 2016 account to the 2017 account so that all outstanding claims are
handled by the 2017 account. If the reinsurance to close premium is £9 million, then a profit of
£3 million will be declared as at the end of 2018 in respect of 2016 business. This profit is then
carried through the profit and loss account in the usual way.

With three-year accounting, no profit is usually recognised until the end of the third year.

The main element of reserving under three-year accounts is usually the assessment of
outstanding claims (including IBNR) at the end of the third year.

Assuming that all the policies are for terms of one year or less, then the reserves at the end of the
first year will cover unexpired risks and outstanding claims. At the end of the second year the
reserve should be for outstanding claims only.

Question

Explain why there might still be some unexpired risks at the end of the third year of an account.

Solution

This is possible if the policies provide cover for a period longer than two years. Suppose a single
policy provides cover for three years. Such a policy written on 1 July 2016 would have 6 months’
unexpired exposure at 31 December 2018 when the 2016 accounting year is closed.

3.2 Classes dealt with under three-year accounts


Most classes of business which we deal with in this course are usually handled under a one-year
accounting basis (eg private motor and household). However, classes such as marine and aviation
are traditionally accounted for using the three-year basis. This is partly historical and based on
convention.

Turning to logic (rather than convention), there are two key reasons why three-year accounting
might be preferred to one-year accounting:
 fundamentally important underwriting years
 delays in premium and claim settlement.

Fundamentally important underwriting years


The underwriting year may be fundamentally important in the Lloyd’s market and for reinsurance
contracts that operate on a policies-incepting basis.

© IFE: 2019 Examinations The Actuarial Education Company


SP7-26: Accounting methods Page 19

Within the Lloyd’s market, the underwriting year is very important because syndicate
membership is determined by underwriting year. A Lloyd’s Name may take part in a particular
syndicate for a particular year’s underwriting. So, to achieve an equitable assessment of each
Name’s profit or loss, we need to prepare accounts according to the profit or loss from each
year’s underwriting separately. Three-year accounting is therefore still used by Lloyd’s syndicates
for internal reporting purposes.

Another example of a fundamentally important underwriting year is proportional reinsurance.


Here all risks incepting in a given year are shared with the reinsurer.

Delays in premium and claim settlement


In some classes, it takes several years to get a clear picture of the amount of premium written
(eg inwards reinsurance) or the amount of claims that will ultimately become payable (eg long tail
classes). Because profit is not assessed until the end of the third year, there is less uncertainty
with three-year accounts than with one-year accounts.

We mentioned above that three-year accounting is often used for proportional reinsurance
because the underwriting year is fundamentally important. It is also often used for
non-proportional reinsurance, mainly for historical reasons, but also because there are often
delays in premium and claim settlement.

3.3 Underwriting year accounting models


We can use both underwriting year and accident year recording and reserving systems to
determine annual accounting results.

Accident year recording is compatible with an annual accounting basis. Underwriting year
recording is compatible with funded accounting and so in order to produce annual accounts, we
will need to make certain adjustments.

Claims outstanding calculated by underwriting year include both amounts from incidents
already reported and amounts in respect of claims on events that will occur in future
accounting periods.

If we wish to convert from underwriting year to accident year, we should reduce the
estimated claims outstanding so that they only cover events occurring up until the end of
the accounting period.

We will also need to set up an Unearned Premium Reserve (UPR), in respect of claims that are yet
to occur on policies that have been written, but not yet fully earned.

Usually we do this in proportion to the earned premiums. However, we may need to adjust
for abnormal loss experience, such as large events, that are not expected to recur in future
periods.

We need also to add in claims arising from incidents occurring in the year in question but in
respect of policies written in earlier years.

The Actuarial Education Company © IFE: 2019 Examinations


Page 20 SP7-26: Accounting methods

4 True underlying profitability and apparent profit


Do the accounts of a general insurance company give a true assessment of the profitability of the
company? A variety of answers are possible to this question. ‘Yes’, ‘sometimes’, ‘perhaps’ and
‘no’ might all be reasonable responses. There are two principal reasons for this.
 Many of the factors determining the true profitability are unknown when the accounts
are put together and for several years thereafter. The apparent profit disclosed is highly
dependent on estimates of future claim payments, expenses etc.
It would therefore only happen by accident (and some very good reserving estimates) that
the true profitability would be given by the accounts.
 The profits declared are also a function of the particular accounting basis used. If the
basis is changed, then the amount of profits declared will change. The true underlying
profitability of the business is not directly affected by the accounting basis. However if, as
a result of the apparent profit disclosed, an insurer changed its behaviour then there
might be some indirect effect on profitability.

We will examine the concept of true underlying profitability through a few very simple examples.
These examples are for one-year accounts, but similar considerations apply to funded accounts.

Example 1: very short-tail business with no uncertainty

Consider a special and very unusual class of business where all policies begin on 1 January, expire
on or before 31 December and all claims from the year are settled by the year end. Suppose
premiums for 2017 are £100m, expenses are £30m, claims are £68m and investment income
generated by the premium income is £3.5m. Then at the end of 2017, we could say that the
profits generated by this class of business in 2017 are £5.5m (100  30  68  3.5) . This is the true
profitability, since there is no outstanding risk at the year end, and we have no doubt in declaring
£5.5m as the profit.

Example 2: short-tail business with no uncertainty

Suppose now that we have a similar position to Example 1, except that all claims are settled for a
total of £72m exactly on 20 December 2018. Suppose also that the company can buy, on
31 December 2017, an investment for £68m which will definitely give £72m on
20 December 2018. In this case we can declare profits of £5.5m at 31 December 2017, with
certainty.

Example 3: short-tail business with some uncertainty

As Example 2, except that we cannot be certain how much we will have to pay in claims on
20 December 2018. We think it will be about £72m, but we don’t know what inflation will be and
we don’t know whether particular court cases will be decided for or against us.

Because we are not sure about the size of the claims, we invest £70m (rather than £68m) on
31 December 2017 and declare profits of £3.5m.

© IFE: 2019 Examinations The Actuarial Education Company


SP7-26: Accounting methods Page 21

It turns out that the total claim payments on 20 December 2018 are £71m, and that with
investment income the company has £74m giving further profit of £3m. Clearly, the profits
declared on 31 December 2017 of £3.5m were not the true profits. Note also that the profits
declared in 2018 will include £3m arising from 2017 business.

Example 4: long-tail business with lots of uncertainty

Similar set up as before. Premiums of £100m written on 1 January 2017, expenses of £30m and
investment income in 2017 of £3.5m. We estimate that in 2022 we will settle all the claims for
£89m and that a reserve of £68m should be adequate. This time we have considerable
uncertainty about the liabilities:
 What will claims inflation be throughout the period?
 Will court judgements generally be favourable/unfavourable?
 Will there be further latent claims that we have not allowed for?
 Are we certain that our reinsurers will all be able to honour valid claims?
 When exactly will all the claims be settled?
 How much investment return will be earned?

So how certain are we that the profit we are thinking of declaring (£5.5m) is the true profitability?
Not very. What’s more, whatever level of profit we declare, it’s very unlikely to be the true profit.

Conclusion
At the end of an accounting year, insurance companies have unsettled claims with varying levels
of uncertainty as to how much the ultimate liability will be. Insurers will not know the true
profitability of each year’s business until all the claims from that period are fully settled. This may
take many years. Therefore, the profits declared in any one year are highly dependent on the
estimates made for outstanding claims.

There is much more uncertainty for long-tail classes and for classes which can have extensive
reporting delays. We give an example below in which shows the effect of the reserving basis on
the emergence of profit.

Impact of the accounting basis on true underlying profitability


The profits declared in any given year are a function of a particular reserving basis. If the basis is
changed, then the amount of profits declared will change. However, the true underlying
profitability of the business is not directly affected by the accounting basis used.

Consider two companies with identical business:


 premiums of £32m on 1 January 2018
 expenses of £6m on 1 January 2018
 a single claim payment of £28m to be made on 31 December 2021.

The Actuarial Education Company © IFE: 2019 Examinations


Page 22 SP7-26: Accounting methods

Company A has a very cautious approach. At the end of 2018, it decides to use a very strong
reserving basis, allowing for very high inflation, high latent claims and zero investment return. It
decides to hold a reserve of £38m. Declared profit for the year is –£12m (ie a loss).

Company B has a much less cautious approach. At the end of 2018, it sets up a claim reserve of
£25m on the grounds that inflation will be low, and allowing for future investment income at a
modest level. Company B declares a profit of £1m.

The apparent profits for the two companies are very different, yet their underlying profitability is
in fact identical.

Indirect impact on true underlying profitability


It is conceivable that the accounting basis will ultimately have some impact on the underlying
profitability. However, it is important to stress that these will be due to knock on (or second
order, indirect) effects.

If, in the above example, Company A changed its behaviour compared to Company B, because of
the lower declared profit and the stronger accounting basis, then there might be some effect on
profitability. For example, Company A might decide to adopt a lower risk investment strategy
than it might otherwise have taken. This should not cloud the main point that the accounting
basis affects the pace of the emergence of profits rather than the true underlying profitability.

Question

Explain briefly why the apparent profitability might not be a true reflection of the true profitability
of an insurer.

Solution

There are estimates for ‘unknowns’ which make up the stated profit. The estimates will almost
certainly turn out to be incorrect. In particular, the estimates of reserves needed for outstanding
claims are subject to uncertainty and to differences in the strength of the reserving basis used.

Also, the basis for the valuation of assets will affect the level of apparent profitability.

© IFE: 2019 Examinations The Actuarial Education Company


SP7-26: Accounting methods Page 23

5 Constructing accounts
The examiners will expect candidates to be able to construct simple accounts by applying
the principles described in this chapter and in earlier subjects.

In this section we work through a fairly simple question. The best way to learn how to construct
insurance company accounts is to practise as many examples as possible.

With all accounts questions it is essential to approach the question logically with a step by step
approach and to set out your working clearly. You can rely on the fact that you have enough
information to answer the question. All you have to do is fit the pieces together.

Example question
An insurance company has tangible assets of £520m on 1 January 2018. This is invested entirely
in interest bearing bank accounts. The following financial information on the company is
available:
(a) Written premium in 2017 is £300m and is expected to grow to £330m in 2018. It is
assumed that, in each calendar year, the premium is received uniformly and that
acquisition expenses and commission are 25% of written premium.
(b) Total commission and expenses are 30% of written premium.
(c) Total claims incurred are expected to be 65% of earned premium in 2018.
(d) Outstanding claims reserves are 100% of premium written in the previous 12 months.
(e) The company pays tax at 25% on its total profit in 2018.
(f) Dividends will be paid by the company on each 31 December so as to leave retained
profits in the company which will maintain the shareholders’ funds (as a percentage of
written premium) at its current relative level.
(g) The investment income earned each year is 12% on the average of the assets held at the
beginning and the end of the year.

Construct the balance sheet of the company at 1 January 2018, and the expected balance sheet at
1 January 2019. Also, construct the revenue and profit & loss account for 2018.

Spend some time making a proper attempt at this question, before reading ahead.

The Actuarial Education Company © IFE: 2019 Examinations


Page 24 SP7-26: Accounting methods

Worked example solution


One of the themes of accounts questions is that they are sometimes a bit like doing puzzles. You
know that you have enough information, you must just figure out how to piece it all together.

This question is more transparent than many as it is relatively easy to see how and where to start,
because the main components of the first year’s balance sheet are clearly available.
 Tangible assets and outstanding claims are £520m and 100% of £300m respectively.
 Unearned premiums are straightforward if we make sensible assumptions. Assume that
risk is even over the year so that the unearned premium at the year end is 50% of the
2017 premiums written, ie £150m. DAC is then 25% of this amount, ie £37.5m. (A UPR
net of DAC would therefore be 75% of 50% of £300m.)
 If we assume that there are no other liabilities or assets (we would not be expected to
make figures up for other items), the balancing item is the value of the shareholders’
funds.

The balance sheet for 1 January 2019 seems more complex initially, but is very easy after some
preliminary reasoning.

 With the same assumptions as at 1 January 2018, the unearned premium and DAC must
be 10% bigger at 1 January 2019 because the written premiums have increased by 10%.
 Similarly, the outstanding claims will be 10% bigger (ie 100% of £330m).
 We are told that the shareholders’ funds remain at the same percentage of written
premiums. Therefore, the shareholders’ funds increase by 10% too.
 The tangible assets must also increase at 10% (because all the other components have).

Therefore the balance sheets are (in £m):

1 Jan 2018 1 Jan 2019


tangible assets 520.00 572.00
+ DAC 37.50 41.25
– unearned premiums 150.00 165.00
– o/s claims 300.00 330.00
= shareholders’ funds 107.50 118.25

Turning to the revenue account, the premium earned for 2018 is the average of the premiums
written in 2017 and 2018. This is because premiums are written evenly over each year and we
have assumed that risk on each policy is even over the year.

The claims incurred figure is 65% of the premiums earned (given).

Expenses are 30% of premiums written (given).

The increase in DAC is given directly by the start and end year balance sheets.

© IFE: 2019 Examinations The Actuarial Education Company


SP7-26: Accounting methods Page 25

We could have assessed the investment income in total by taking 12% of the average of £520m
and £572m. However, it is not difficult showing the investment on the insurance funds and other
funds separately, so we have, by splitting the total income in proportion to the average funds.

Tax is 25% of the total profit (given).

Finally, we know that the retained profit must be the difference between shareholders’ funds at
the start and end of the year. This leaves the dividend as the balancing item.

The revenue and profit and loss account for 2018 (in £m) is therefore:

Premiums earned 315.00


– Claims incurred 204.75
– Expenses 99.00
+ Increase in DAC 3.75
= Underwriting result 15.00
+ Investment income on insurance funds 51.98
= Insurance result 66.98
+ Balance of investment income 13.55
= Total profit 80.52
– Tax 20.13
– Dividends 49.64
= Retained profit 10.75

Final comment
The presentation of this worked example has slightly more explanation than you need to give in a
solution to an exam question. However, you should always:
 show your calculations so that a marker can follow what you are doing
 explain all assumptions you make (eg about incidence of risk when estimating UPR).

Many students will lose marks by not doing this. It is very easy to gloss over the assumptions.

The Actuarial Education Company © IFE: 2019 Examinations


Page 26 SP7-26: Accounting methods

6 Glossary items
Having studied this chapter you should now read the following Glossary items:
 Accident year
 Annual basis of accounting
 Claims handling expenses (CHE)
 Non-technical account
 One-year (accident year) accounting
 Underwriting year.

© IFE: 2019 Examinations The Actuarial Education Company


SP7-26: Accounting methods Page 27

Chapter 26 Summary
Annual accounts consider the business earned in each year. Funded accounts consider the
business written in each year.

The reserves required for one-year accounts are:


 outstanding reported claims
 IBNR
 reopened claims
 expenses of handling these claims
 UPR (and possibly AURR).

Funded accounting is used when the underwriting year is fundamentally important or when
there are significant delays in premium and claim settlement.

Most personal lines business and shorter-tail commercial business is reported using one-year
accounts.

Lloyd’s syndicates, marine and aviation business and non-proportional reinsurance still use
funded accounts for internal purposes, and apply an adjustment to determine reserves on a
one-year accounting basis.

In the UK, all companies that are listed on a European stock exchange must produce their
financial statements under IFRS. All unlisted companies (and unlisted subsidiaries) can
report on an IFRS or on a UK GAAP basis.

The apparent profit disclosed in a set of accounts is dependent on estimates of the future
and is a function of the accounting bases used.

The accounting basis will ultimately have some impact on the underlying profitability but
these will only be indirect effects.

You will be expected to be able to construct simple accounts.

The Actuarial Education Company © IFE: 2019 Examinations


Page 28 SP7-26: Accounting methods

The practice questions start on the next page so that you can
keep the chapter summaries together for revision purposes.

© IFE: 2019 Examinations The Actuarial Education Company


SP7-26: Accounting methods Page 29

Chapter 26 Practice Questions


26.1 The following data is available for an insurance company for a given year:
1. Written premiums of £121m.
2. Expenses of £39m.
3. DAC increases by £3m.
4. Investment income at 10% of the average of start of and end of year assets.
5. Tax at 50% of total profit.
6. No dividends.
7. Balance sheets at start and end of year:
Assets £154m £186m
UPR (net of DAC) £40m £47m
o/s claims £82m £103m
Shareholders’ funds £32m £36m

Calculate the claims paid during the year.

26.2 You are estimating the reserves for a class of business accounted for on a three-year basis. Your
goal is to produce a single estimate as at the end of 2018 for the three funds (one of which is
about to be closed). Set out a single run-off triangle that will give you the answers you need.

26.3 By referring to results declared at the start of 2018, compare accident-year and three-year
accounts in terms of the periods exposed to risk. (You may assume that all accounting periods
run from 1 January to 31 December.)

26.4 You have been asked to investigate the profitability of the insurer over its first year of trading.
You are given the following data:
 The insurer sold 100 policies on the first day of each month, each for a premium of $500.
 The insurer incurs initial expenses of 15% of the premium size.
 Claims take ½ month to be reported to the insurer and a further 3 months to be paid.
 The claim frequency is double in January, February, November and December than it is in
the other months of the year.
 Claims all have a value of $5,000.
 The insurer initially had capital of $1m and now (ie at the end of the year) has assets of
$1.5m.
 Interest is earned on the assets at 1% per month and reserves are discounted also using
1% per month discount rate.
 The insurer pays no tax or dividends.

The Actuarial Education Company © IFE: 2019 Examinations


Page 30 SP7-26: Accounting methods

Prepare a revenue and profit and loss account for the year stating clearly all assumptions that you
make.

26.5 An insurance company that writes only motor insurance has a financial year running from
Exam style
1 January to 31 December.

On 1 May, a broker sold an annual policy with a premium of £450. The company received the
premium, net of 14% commission, on 15 June. No claims were made under the policy. In addition
to the commission, the marginal costs incurred from this policy totalled £10 on acquisition and £1
at the start of each month thereafter, finishing on 1 May one year later.

The company earned 1% per month on its assets in the first calendar year (ie from 1 May to 31
December), and 0.8% per month in the second year (ie 1 January to 30 April).

Assume that the company prepares its accounts on the basis of acquisition costs being 20% of
written premium, and that IBNR is calculated on a very crude basis, namely as 5% of premiums
earned in the year.

(i) Stating all further assumptions and ignoring the impact of taxation, show how this
individual policy will affect the company’s accounts for each of the two calendar years in
question. [15]

(ii) Explain, in detail, how your answer would differ if the company decided at the end of the
first calendar year that for policies written during that year, the premiums would be
inadequate to cover the claims that would emerge from the policies in future. [4]
[Total 19]

26.6 An abridged version of the accounts of a general insurer, which does not at present discount
Exam style
outstanding claims, is as follows:

£m
Earned premium 107.1
Incurred claims 88.2
Commission and expenses 32.7
Underwriting profit (13.8)
Investment income (allocated to technical reserves) 12.7
Trading profit (1.1)
Outstanding claims brought forward 142.6
Outstanding claims carried forward 181.6

© IFE: 2019 Examinations The Actuarial Education Company


SP7-26: Accounting methods Page 31

The chief executive of the company has suggested that these outstanding claims should be
discounted.

(i) Assuming an interest rate for discounting of 10% per annum and a mean term for
outstanding claims of two years, calculate the underwriting profit on a discounted basis
and explain why the underwriting profit has changed. [5]

(ii) Comment on whether the investment income allocated to the revenue account in future
should change on the new basis. [2]

(iii) The chief executive has also asked whether the underlying profitability of the company
will change. Outline the main points of your reply and illustrate your answer with an
example. [7]
[Total 14]

The Actuarial Education Company © IFE: 2019 Examinations


Page 32 SP7-26: Accounting methods

The solutions start on the next page so that you can


separate the questions and solutions.

© IFE: 2019 Examinations The Actuarial Education Company


SP7-26: Accounting methods Page 33

Chapter 26 Solutions
26.1 Start with the retained profit and work up the account to find that paid claims = £63m:

Written premiums 121 given


– Increase in UPR (net of DAC) 7 47 – 40
– Claims paid x the answer. Find x = 63
– Increase in o/s claims 21 103 – 82
– Expenses paid 39 given
+ Investment income 17 0.1  (154 + 186) / 2
= Profit before taxation 8 because tax is at 50%
– Taxation 4
= Profit attributable to shareholders 4
– Dividends 0 given as zero
= Retained profits 4 Increase in shareholders’ funds:
36 – 32

Note that, as the UPR given on the balance sheets is already net of DAC, you do not need to use
the information that you are given in the question relating to the increase in DAC.

26.2 The key to this question is to define the cohorts correctly. In particular, a run-off triangle with
‘year written’ as the origin year will give the answer required.

For example, if the claims were fully run off by the end of development year 4, a chain ladder of
the following form would do the trick (provided the necessary assumptions for the chain ladder
held).

The liabilities for the 2016 account year (about to be closed) are C16,3 + C16,4 + C15,4

(Assume that C15,4 was transferred to the 2016 account using a RITC premium.)

The liabilities for the 2017 account year are C17,2 + C17,3 + C17,4

The liabilities for the 2018 account year are C18,1 + C18,2 + C18,3 + C18,4

Settlement delay in years (‘development year’)


0 1 2 3 4
2014
Year 2015 C15,4
Written 2016 C16,3 C16,4
2017 C17,2 C17,3 C17,4
2018 C18 ,1 C18,2 C18,3 C18,4

The Actuarial Education Company © IFE: 2019 Examinations


Page 34 SP7-26: Accounting methods

26.3 For accident-year accounts, the period of exposure will primarily be for events occurring in 2017.

This may be distorted by adjustments in 2017 relating to prior years’ exposure, eg there will be a
deficit from prior years if the claims reserves brought forward at the start of 2017 were not
sufficient.

There may also be a distortion if an additional unexpired risk reserve is set up in relation to
exposure after 2017.

For three-year accounts, the exposure will relate to policies written in 2015. For annual policies,
this means exposure to claim events relating to these policies, which may have occurred in 2015
or 2016.

There may also be some distortions, eg:


 losses recognised early in respect of 2016 and 2017 underwriting years
 any under or over provision in respect of prior years’ reserves that were carried into the
2015 account.

26.4 Define the exposure unit of a month as the number of policies exposed in that month multiplied
by the likelihood of a claim. Therefore the ‘exposure units’ for each month are:

Year 1

Jan Feb Mar Apr May Jun Jul Aug Sep Oct Nov Dec Tot
2 4 3 4 5 6 7 8 9 10 22 24 104

Year 2

Jan Feb Mar Apr May Jun Jul Aug Sep Oct Nov Dec Tot
22 20 9 8 7 6 5 4 3 2 2 0 88

Premiums written  500  100  12  600,000

Expenses paid  600,000  0.15  90,000

UPR  88 (104  88)  600,000  275,000

DAC  88 (104  88)  90,000  41,250

Let the claim amount per unit of exposure equal C.

The accumulated capital of $1m, plus the accumulated premiums (net of expenses) less the
accumulated claims paid must equal the assets at the end of the year.

© IFE: 2019 Examinations The Actuarial Education Company


SP7-26: Accounting methods Page 35

Therefore, accumulated claims paid must equal:

1,000,000  (1.01)12  1,500,000  100  500  0.85  


S12| at 1% per month

= $171,221.47

Now assume that claims occur midway through the month and we are told that they are paid 3½
months after they occur.

So, January’s claims are paid on 1 May. Therefore the accumulated exposure units that have been
paid up to, 31 December are:
2(1.01)8 + 4(1.01)7 + 3(1.01)6 + 4(1.01)5 + 5(1.01)4 + 6(1.01)3+ 7(1.01)2 + 8(1.01)
= 40.44838

Therefore:
40.44838  C  171,221.47  C  4,233.086

and claims paid = 39 (the number of exposure units during Jan-Aug) C = 165,090

The present value of outstanding reported claims (claims from Sep–Nov) is:

C  (9  10 / 1.01  22 / 1.012 )  171,302

The present value of IBNR claims (claims from Dec) is:

C  24 / 1.013  98,606

URR (for claims occurring after the year-end) is

C  (22 / 1.014  20 / 1.015    2 / 1.0114 )  348,522

AURR  URR  UPR  DAC  114,772

The total investment income is:


end of year assets less cashflows and initial capital

 1,500,000  600,000  90,000  165,090  1,000,000  155,090

The Actuarial Education Company © IFE: 2019 Examinations


Page 36 SP7-26: Accounting methods

Revenue and profit and loss account

Written premium 600,000


UPR (275,000)
Claims paid (165,090)
O/s reported claims (171,302)
IBNR (98,606)
AURR (114,772)
Expenses paid (90,000)
DAC 41,250
Underwriting profit (273,520)
Investment income 155,090
Profit (118,430)

Investment income is made up of $126,825 on initial capital and $28,265 on premiums less claims
and expenses.

26.5 The impact on the revenue and P&L accounts of the policy described will be to increase the
entries in calendar years 1 and 2.

Year 1 Year 2
Written premium 450.0 0.0
+ UP b/f 0.0 150.0
– UP c/f 450  4/12 150.0 0.0

Earned premium WP + UP b/f – UP c/f 300.0 150.0


– Claims paid 0.0 0.0
– o/s claims c/f 0.05  300 15.0 0.05  150 7.5
+ o/s claims b/f 0.0 15.0
– Expenses 0.14  450 + 10 + 7 80.0 5 months of £1 5.0
+ Incr. in DAC 0.2  450  4/12 30.0 –30.0
U/W profit 235.0 122.5
+ Investments see Note 1 24.8 see Note 3 39.2
Total profit 259.8 161.7

[9 marks for correct entries]

© IFE: 2019 Examinations The Actuarial Education Company


SP7-26: Accounting methods Page 37

Note 1: Based on 1% per month, and assuming that this rate operates immediately the
company receives or pays out money, the investment income can be calculated by
considering each of the cashflows:

Money in on 15 June (450  0.86)  (1.016.5 – 1) = 25.9

Expenses out 1 May – 10  (1.018 – 1) = (0.8)

Monthly expenses – (1.017 – 1) / (.01/1.01) – 7 = (0.3)


______
Total = 24.8

Note 2: Assume that dividends are unaffected by this single policy. [1]

Note 3: Assume that the retained profit is held by the company throughout the following
year. Therefore the money earning investment income at 0.8% per month at the
start of year 2 is the retained profit plus the technical reserves:

Total funds on 1 Jan (150 – 30 + 15 + 259.8)  (1.00812 – 1) = 39.6

Monthly expenses – (1.0085–1)/(0.008/1.008)  1.0087 – 5 = (0.4)

Total = 39.2

Items in the balance sheet at 31/12/Y1 would be increased as follows:

Shareholders’ Reserves 259.8 Assets 394.8


(450 – 80 + 24.8)
UPR 120.0
o/s claims 15.0

[2]

Items in the balance sheet at 31/12/Y2 would be increased as follows:

Shareholders’ Reserves 421.5 Assets 429.0


(394.8 – 5 + 39.2)
UPR 0.0
o/s claims 7.5

[2]

In both cases it is assumed that tax is paid before the year end. If not, assets would be increased
by the amount of tax payable, and there would be an increase in current liabilities. [1]
[Total 15]

The Actuarial Education Company © IFE: 2019 Examinations


Page 38 SP7-26: Accounting methods

(ii) Risk of premiums being inadequate

This statement implies that the company decides that it is necessary to hold an additional reserve
at 31/12/Y1 for unexpired risk (ie additional to the UPR of 120 in this case). [1]

If an AURR was held at 31/12/Y1, it would decrease the profit from Y1 and increase the profit in
Y2, ie it would act to defer the emergence of profit from the contract in the same way as IBNR has
above. [1½]

There should be no change in the amount of investment income. The actual amounts of money
held in total should be unchanged. The reduction in free reserves at 31/12/Y1 would be offset by
the increase in AURR at 31/12/Y1. [1½]
[Total 4]

26.6 (i) Calculating on a discounted basis

Note: We have assumed that the question requires us to discount both the b/f and c/f figures,
ie what would the accounts have looked like if discounting had always been used. An alternative
approach would be to discount the c/f figure only. This is consistent with introducing discounting;
the b/f would remain as the actual claims reserve c/f at the end of the previous year.

Claims incurred = claims paid + o/s claims c/f – o/s claims b/f

Thus, claims paid = 88.2 – 181.6 + 142.6 = 49.2

As the claims are expected to be 2 years away, to be discounted at 10% pa:


discounted claims b/f = 142.6  1.21 = 117.9
discounted claims c/f = 181.6  1.21 = 150.1 [1]

So claims incurred (using discounting) = 49.2 + 150.1 – 117.9 = 81.4 [1]

The result is given by:

£m
Earned premium 107.1
Incurred claims 81.4
Commission & expenses 32.7
Underwriting profit –7.0 [1]

The reason that the loss falls, is that the increase in the outstanding claims reserve charged to the
year is reduced by discounting. [1]

Profit has been brought forward by anticipating future investment income. [½]

With the non-discounted reserves, the investment income will emerge to increase profits (or
reduce losses) in future years. [½]
[Total 5]

© IFE: 2019 Examinations The Actuarial Education Company


SP7-26: Accounting methods Page 39

(ii) Change in allocation of investment income

How the investment income allocated to the revenue account may change in future depends
partly on how the £12.7m arises in the example. [½]

The investment income allocated to the revenue account relates to the reserves backing the
business. As we now discount these reserves, the amounts and therefore the investment income
produced will be reduced. (Other things being equal, the total investment return is the same.) [1]

It may also be considered appropriate to allocate income to the revenue account at the rate used
to discount the reserves, and credit any difference to the profit and loss account. [½]
[Total 2]

(iii) Whether discounting alters the underlying profitability

Introducing discounting does not affect the actual amounts of claims paid (and hence the
underlying level of profitability of the office is not changed). [½]

It merely alters the pace at which profits emerge. [½]

If discounting is introduced to a tranche of business, profits will be higher for the first year, and
thereafter they will be slightly lower for each subsequent year. [1]

If the discount rate is the same as the investment returns earned, all the profits would be
recognised in the year the business is earned (subject to any errors in reserving). [1]

An example would help illustrate the mechanics of discounting:

Consider a new tranche of business where (rather artificially) all annual premiums (£14m) are paid
on 1 January and expenses (£2m) are paid at the same time. Claims (£11m) are paid on the
31 December of the following year, and assume that 10% is earned on investments. Discount at
the rate that you earn on investments (ie 10%).

If discounted reserves are used

£12m held after expenses and invested for a year at 10% (giving £1.2m investment income).

Incurred claims in year 1 is claims paid plus increase in o/s reserve. Nil paid. Reserve is
11/1.1 = £10m. Thus incurred is £10m.

At end of year, profit (£3.2m) removed for tax and shareholders. The balance (£10m) is invested
to pay claims at end of year 2.

The Actuarial Education Company © IFE: 2019 Examinations


Page 40 SP7-26: Accounting methods

Year 1 Year 2
Earned premium 14.0 0.0
– Claims paid 0.0 11.0
– o/s c/f 10.0 0.0
+ o/s b/f 0.0 10.0
– Expenses 2.0 0.0
Underwriting result 2.0 (1.0)
+ Investment 1.2 1.0
Trading result 3.2 0.0

If undiscounted reserves are used

Similar process, but incurred claims will be £11m in year 1 and £0m in year 2.

At end of year 1, profit (£2.2m) removed for shareholders. The balance (£11m) is invested to pay
claims at end of year 2. This time there is profit in year 2 as well.

Year 1 Year 2
Earned premium 14.0 0.0
– Claims paid 0.0 11.0
– o/s c/f 11.0 0.0
+ o/s b/f 0.0 11.0
– Expenses 2.0 0.0
Underwriting result 1.0 0.0
+ Investment 1.2 1.1
Trading result 2.2 1.1

Note the different profile of profits between the two. However, the present value of each is
identical at 10% interest.
[4 marks for clear example]
[Total 7]

© IFE: 2019 Examinations The Actuarial Education Company


SP7-27: Interpreting accounts Page 1

Interpreting accounts
Syllabus objectives
5.10 Describe the methods and principles of accounting for general insurance business
and interpret the accounts of a general insurer.

5.11 Describe the changes to accounting methods expected under IFRS.

Covered in part in this chapter.

The Actuarial Education Company © IFE: 2019 Examinations


Page 2 SP7-27: Interpreting accounts

0 Introduction
In the previous chapter we looked at the principles for the construction of accounts for a general
insurance company. In this chapter, we look at what we can learn about an insurer from the
accounts.

© IFE: 2019 Examinations The Actuarial Education Company


SP7-27: Interpreting accounts Page 3

1 Accounting principles
For the purposes of compiling and interpreting the accounts of insurers it is important to be
familiar with the relevant accounting concepts and principles. These may vary from country
to country, although efforts are being made to achieve greater harmonisation of
international accounting practice. The principles used may also depend on the purposes
for which the accounts are designed.

You may be familiar with some of the basic accounting concepts from your previous studies.

We can illustrate the meaning of the expression ‘accounting concepts’ by reference to


concepts that have been widely used in relation to the periodic financial accounts of all
businesses, including insurers. These are:

Concept Explanation

Going concern The enterprise will continue in operational existence for the
foreseeable future.
Accruals basis Revenue and costs are recognised as they are earned or
incurred, not as money is received or paid.
Consistency There is consistency of accounting treatment of like items
within each accounting period and from one period to the
next.
Since actuaries are crucial in the reserving process it is
important to understand that variations in the level of
prudence applied year on year lead to releases of profit
(potential for profit manipulation) which is against the
accounting principles. Therefore one of the most important
things in the reserving cycle is consistency.
Prudence and Revenue and profits are not anticipated (that is, must be
realisation realised), and provision is made for all known liabilities,
whether the amount of these is known with certainty or is a
best estimate in the light of the information available.
Separate valuation of When determining the aggregate amount of any item the
assets and liabilities enterprise must determine separately the amount of each
individual asset or liability that makes up that item.

We note that the concept of ‘prudence’ includes a reference to a ‘best estimate’ and does
not refer to including a margin. Indeed, an accountant would maintain that estimates
should be free from deliberate or systematic bias.

Prudence has been described as the inclusion of a degree of caution in the exercise of the
judgements needed in making the estimates required under conditions of uncertainty, such
that gains or assets are not overstated and losses or liabilities are not understated. We may
interpret this as meaning that the greater the uncertainty, the greater should be the
tendency to aim at reserves exceeding the expected value of the liabilities, as a natural
consequence of seeking to avoid understating the liabilities.

The Actuarial Education Company © IFE: 2019 Examinations


Page 4 SP7-27: Interpreting accounts

Question

General insurers have historically not discounted their reserves. Discuss whether this practice has
been consistent with the interpretation of the prudence concept described above.

Solution

Probably. Not discounting reserves will result in including an implicit margin on top of the best
estimate reserve. This means that the reserves are less likely to be understated.

The further the payments are in the future, the bigger the margin will be. Longer-tail classes tend
to be associated with more uncertainty and hence bigger margins will be held for more uncertain
liabilities.

The reason for constructing the accounts will influence the size of margins that will be included in
the accounts. Accounts that are produced to demonstrate solvency are likely to be prepared on a
more cautious basis than those prepared for management, which are produced to give a realistic
picture of the profitability and financial strength of the company.

Other than as described in this chapter, candidates for Subject SP7 examinations will not be
expected to be familiar with the accounting concepts and principles that apply in any
particular country. They may, however, be expected to discuss the problems that arise in
defining such concepts and principles and putting them into practice, and the implications
for the interpretation of insurers’ accounts.

© IFE: 2019 Examinations The Actuarial Education Company


SP7-27: Interpreting accounts Page 5

2 Interpreting accounts
Before attempting to interpret the accounts of an insurer, we should become familiar with
the statutory rules governing the preparation of the accounts and the accounting rules and
conventions that apply in the country concerned.

This paragraph can be related back to the actuarial control cycle. Before we can monitor the
experience of an insurer, we should be familiar with the economic and commercial environment
in which the insurer operates.

Though subject to some variation from country to country, the published financial
statements of insurers are usually prepared on a going concern basis and are intended to
give a true and fair view of the insurer’s performance and financial position. We should
examine them to see whether they have been affected by any changes in accounting
practice and, if so, to find what the effects of such changes may have been.

We should consider the basis used for the valuation of the assets and the treatment in the
accounts of realised and unrealised capital gains and losses. If assets are shown at market
value, we should consider the vulnerability of the asset values to changes in market
conditions.

Let’s consider Company A and Company B and for each company calculate the solvency ratio, a
useful measure of financial strength. The solvency ratio is defined as free reserves divided by net
written premiums.

Company A Company B
Total assets £740m £740m
Total liabilities £650m £615m
Net written premiums £500m £500m
Solvency ratio 18% 25%

On examination of the accounts, we see that Company A has used a conservative basis to value
assets, whereas Company B has used full market value. If both companies have assets valued on
market value the figures become:

Company A Company B
Total assets £850m £740m
Total liabilities £650m £615m
Net written premiums £500m £500m
Solvency ratio 40% 25%

This illustrates how the basis for valuation of assets can have a major influence on apparent
financial strength.

The asset valuation basis and the method used for recognising capital gains can have a
corresponding impact on the stated profits.

The Actuarial Education Company © IFE: 2019 Examinations


Page 6 SP7-27: Interpreting accounts

The reports accompanying the accounts may reveal the extent to which the results for the
latest period have been affected by the occurrence or absence of exceptional events.

For example:
 catastrophe type claims
 a year with unusually low claims experience
 high expenses from internal reorganisations (eg redundancies).

Question

Suggest why two consecutive years of unusually low claims experience might potentially be a
‘mixed blessing’.

Solution

The good news will be the high profits achieved in the two good years. The potential downsides
are:
(a) If the company used the low claims experience as a basis for future premiums, the
premium rates may be set at too low a level (note that the question says unusually).
(b) Other companies may be attracted into the insurance market, thereby triggering the
‘softening of premium rates’ stage of the insurance cycle.

Insurance business is subject to cyclical effects that may affect many insurers at more or
less the same time. We should compare the profitability of an insurer’s business (as
distinct from the profit disclosed by the latest set of accounts) with the results disclosed by
the accounts of other insurers, especially those transacting similar types of business.

Although an insurer may have made a genuine attempt to prepare the accounts in
accordance with the consistency concept, this is often difficult to achieve in practice
because of the uncertainty, and consequent margins of error, in determining the various
items in the accounts, in particular the reserves.

An insurer aims to establish reserves within a reasonable range. However, insurance


liabilities are inherently uncertain and we normally have to use judgement in setting
reserves and determining where in an acceptable range the chosen reserves are.

If the reserves established at the end of the year are on a weaker basis, in relation to current
conditions, than those established at the end of the previous year, then the profit for the
year will, in this respect, be greater, and vice versa. The same applies, of course, to the
accounts of any other insurers that may be used as a basis for comparison.

Question

List questions you might ask about the reserving basis of a general insurer.

(We thought of eight. Ideally all your questions will be significantly different from each other.)

© IFE: 2019 Examinations The Actuarial Education Company


SP7-27: Interpreting accounts Page 7

Solution

 What allowance was made for latent claims and late reported claims?
 What rate of inflation was used?
 Was discounting used, and if so, at what rate?
 What allowance was made for future expenses?
 Were reserves for re-opened claims set up?
 Was an additional unexpired risk reserve required?
 What margins were included in the estimates?
 How did the margins differ between different classes of business?

Financial strength
Let us now compare Company X with Company Y.

Company X Company Y
Total assets £260m £260m
Total liabilities £180m £150m
Net written premiums £300m £300m
Solvency ratio 27% 37%

Which company is stronger?

You should have no hesitation in saying that, from the available data, Company Y looks stronger.

But if we discovered that:


 Company Y writes more volatile business
 Company Y discounts claim reserves and has few margins
 Company X assesses reserves with very large contingency margins
… then we might establish that Company X was in fact much stronger.

For example, if the liabilities of both companies were to be assessed on a consistent and
moderately conservative undiscounted basis, the results might be:

Company X Company Y
Total assets £260m £260m
Total liabilities £160m £200m
Net written premiums £300m £300m
Solvency ratio 33% 20%

The Actuarial Education Company © IFE: 2019 Examinations


Page 8 SP7-27: Interpreting accounts

Profitability
Different reserving bases can also distort the picture for profitability. However, it is much less
clear whether profit will be understated or overstated.

Question

Justify whether a company with a strong reserving basis will always show smaller profits than an
equivalent company with a weaker reserving basis.

Solution

Not true. A critical factor is the rate of growth. A company with a stronger reserving basis will
usually show smaller profits if the volume of net written premiums is expanding because the
reserves will be increasing. If it is contracting then the reserves being released will be greater
than the new margins in reserves.

Further changes in the reserving basis also have a big impact, eg if the basis is getting even
stronger than a year ago then some of the additional (probable) over-reserving would be a hit
against this year’s profit figure.

Conclusion
The strength of the reserving basis can have a distorting influence on the apparent financial
strength and apparent profitability.

We may be able to get an indication of the strength of the reserves by examining individual
accounting items (both gross and net of reinsurance if separate figures are available) and
various ratios of these accounting items, and comparing them with their counterparts in the
accounts of earlier years.

The extent to which this is feasible will depend on the amount of detail given in the financial
statements.

There are some useful disclosures that are normally present in any set of accounts, such
as:

 triangles – in the case of IFRS

 fair value hierarchy of investments – to check asset valuations and their potential
volatility

 disclosures around reserves – to work out reserve releases / deterioration etc.

© IFE: 2019 Examinations The Actuarial Education Company


SP7-27: Interpreting accounts Page 9

Among the ratios that we may consider are:

 incurred claims to premium income

 incurred expenses to premium income

 commission to premium income

 operating ratio / combined ratio

 outstanding claims reserve to claims paid

 outstanding claims reserve to premium income

 outward reinsurance premium income to gross premium income

 reinsurers’ share of reserves to gross reserves.

A sharp rise in premium income may be a sign of competitively low, and perhaps
unprofitable, premium rates. It may also affect the progression of other items such as the
ratio of the reserve for outstanding claims to the earned premiums, the new business strain
and the statutory minimum solvency margin.

In the next section we will look at the key values and ratios that might be considered when
analysing a general insurer’s accounts including the ratios mentioned above.

The Actuarial Education Company © IFE: 2019 Examinations


Page 10 SP7-27: Interpreting accounts

3 Key values and ratios

3.1 Basic ‘key values’


For analysis purposes, each figure in the accounts is a ‘key value’ in one way or another. In
particular, the values of the following items, and how they change from year to year, are
fundamental:
 premiums (written and earned)
 claims incurred
 expenses
 investment income
 total assets
 total liabilities.

3.2 Key values from the accounts


Given that our primary aim is to assess the profitability and financial strength of an insurance
company from its accounts, the key values to consider are as follows:
 Underwriting profit (from the revenue account). This is the excess of earned premiums
over incurred claims and expenses. It is a crude measure of trading profit.
 Insurance profit (from the profit and loss or revenue account). This is the underwriting
profit plus the investment income earned on the technical reserves. The insurance profit
represents the profit achieved through writing insurance business.
 Profit before tax (from the profit and loss account). This is the insurance profit plus
investment income from other assets (ie the free reserves). This is the total profit earned
by the shareholders’ funds.
 Retained profit (from the profit and loss account). This is the profit remaining after
payment of tax and dividends.
 Total shareholders’ funds (from the balance sheet). This is the excess of assets over
liabilities. It is a measure of the financial strength of the company.

Analysing monetary amounts over time could lead to the wrong conclusions if no account is made
for the effects of price inflation. Also, comparing monetary amounts between companies makes
no allowance for the different sizes of companies. Therefore ratios should also be considered.

Ratios are useful for identifying trends, because the effects of inflation and company size are
largely removed. The purpose of ratios is to give a quick indication of the position of an insurer’s
business.

© IFE: 2019 Examinations The Actuarial Education Company


SP7-27: Interpreting accounts Page 11

Question

Suggest ratios (ie give the numerator and denominator) which might be derived from the
accounts of a general insurer for assessing:
 claims experience
 total expense levels
 commission rates
 investment return
 overall profitability of writing insurance
 financial strength
 rate of return on shareholders’ capital.

Solution

 claims experience: claims incurred / earned premiums


 total expense levels: expenses and commission / written premiums
 commission rates: commission / written premiums
 investment return: investment return / average assets
 insurance profitability: insurance profit / earned premiums
 financial strength: free reserves / written premiums
 return on capital: total profit / capital employed

3.3 Ratios
The ratios covered in this section are:
 claim ratio
 expense ratio
 commission rate
 combined ratio
 proportion reinsured
 investment performance

 profit margin
 return on capital

 solvency margin
 assets to liabilities.

The Actuarial Education Company © IFE: 2019 Examinations


Page 12 SP7-27: Interpreting accounts

We have deliberately split the ratios above into three groups. The first group can be thought of as
relating to the operational performance of the company. The second group relates to the
insurer’s profit. The third group help illustrate the insurer’s financial strength.

We will also consider the claim settlement pattern.

Claim ratio (or loss ratio)


This is usually calculated as:

incurred claims
claim ratio 
earned premiums

However it may also be calculated using any combination of claims and premiums as long as the
numerator and denominator are consistent with each other. For example if written premiums is
used as the denominator then the numerator should be all claim payments from the premiums
written.

It is usually calculated net of reinsurance, but may be applied gross to assess the overall
performance of the company’s underwriters.

If net of reinsurance, the premiums will be net of outwards reinsurance premiums and the claims
will be net of reinsurance recoveries.

It gives a basic measure of the level of claims. A high level might reflect inadequate premiums,
poor underwriting standards, poor claims control or a strong reserving basis. It can, however, be
quite volatile from year to year, so one year’s bad results does not necessarily imply bad
underwriting.

Question

A company writes premiums at the beginning of each quarter according to the following
sequence: £40m on 1/1/2016, £60m on 1/4/2016, …, £200m on 1/1/2018. All policies are annual
and have risk uniform over the year.

Given that acquisition costs are a constant 20% of premium, calculate the correct earned
premium figure for 2017 for the purpose of the loss ratio.

Solution

Acquisition costs should be ignored, giving something equivalent to the following working:
1 1 3 3 1 1
60   80   100   120  140   160   180   £480m
4 2 4 4 2 4

© IFE: 2019 Examinations The Actuarial Education Company


SP7-27: Interpreting accounts Page 13

Expense ratio
This is the proportion of premiums taken up in expenses (including commission). Because the
expenses are largely incurred on writing business, correspondence is maintained by relating the
expenses to premiums written (rather than premiums earned).

expenses paid
expense ratio 
written premiums

The expenses would be net of reinsurance commission received and the premiums would be net
of outwards reinsurance premiums. Exceptional one off expenses, eg those incurred in the
purchase of another company, might be excluded from the ratio.

Commission rate
This would be calculated as commission paid divided by written premiums.

Changes in the commission rates from time to time may indicate changes in the sources of
business, changes in the mix of business or aggressive marketing strategy by the company.

Combined ratio
This is also known as the operating ratio or the underwriting ratio.

Although not strictly a ratio in itself (unless the two denominators are the same), the sum of the
claims ratio and the expense ratio is sometimes used to analyse the performance of an insurer.

Although the denominators for these two ratios may be different, they correlate with the
individual numerators. The fact that the rates are derived from slightly different time periods is
an anomaly, but nothing more. Any error is probably minor compared with the distortions of the
claims ratio caused by the reserving basis.

Proportion reinsured
The extent to which a company relies upon reinsurers is usually determined by calculation of the
ratio of net to gross written premiums. Alternatively, we could look at the complement of these
ratios to show the amount of business ceded. Changes in this ratio may indicate a change in the
use of reinsurance.

The ratio of net to gross claims incurred may also be calculated. This may indicate the value for
money of the reinsurance, but one-off events or a year of poor claims experience may distort it.

Investment performance
The ratio of investment return divided by the average asset value during the year may give an
indication of the investment performance achieved by the insurer.

The result will be dependent on how the company recognises realised and unrealised capital gains
and if unrealised gains are not included, the balance of the investment policy between seeking
income and capital gains.

The Actuarial Education Company © IFE: 2019 Examinations


Page 14 SP7-27: Interpreting accounts

The result may also be distorted if the insurer holds a large amount of nil bearing assets, such as
broker balances.

Return on capital
One of the most fundamental profitability measures is the profit generated by the capital backing
the business. This can be calculated as total profit after tax divided by capital employed, ie:

post-tax profit
return on capital =
free reserves

Strictly, the return on capital should be based on profits earned in a given year divided by capital
at the start of that year. However, sometimes calculations are based on capital at the end of the
year or on the average of capital at the start and at the end of the year.

Solvency ratio
This is a basic measure of financial strength. It is the ratio of free reserves to net written
premiums. It is very important in many countries where the statutory minimum solvency
requirement is calculated as a percentage of net written premiums.

Assets to liabilities
The ratio of assets to liabilities is another useful ratio that you will have met in an earlier subject,
for assessing the financial strength of a company.

Profit margin
This is usually calculated as insurance profit divided by net earned premium. This ratio is sensible
because the numerator does correspond with the denominator. It effectively tells us how much
profit the company made in the year for each £1 of premiums earned by the company.

The profit margin calculation could be based on total profits including the investment income on
free reserves. Arguably, this ratio is less useful for comparison purposes because the investment
income on free reserves is not generated by writing insurance business and obviously depends on
the size of the free reserves.

Claim settlement pattern


This can be investigated by calculating the ratio of the total outstanding claims reserve to claims
paid. The higher the ratio, the longer the average tail of the business written.

Similarly, reporting delays can be demonstrated by looking at the ratio of notified and incurred
claims or IBNR to the total claims reserve. Processing delays can be shown by claims paid divided
by the reserve for reported outstanding claims.

© IFE: 2019 Examinations The Actuarial Education Company


SP7-27: Interpreting accounts Page 15

4 Interpreting the features of a set of accounts


The examiners will expect candidates to be able to comment sensibly on the conclusions (if
any) that can be drawn from the figures derived in analysing a simple set of accounts,
making reference where necessary to the effect of the strength of bases, business growth
and so on.

In the previous sections, we listed the key values and ratios that may be used when assessing the
results of a general insurance company. In this section, we now use these values and ratios to
compare the results of two example companies over a four-year period.

Question

Using the data on the next page, calculate the key values and ratios for both Company A and
Company B for each of the four years. (You do not have the data to calculate every ratio for every
year.)

Comment briefly on the results of your analysis.

Solution

The answers follow on in the notes.

The Actuarial Education Company © IFE: 2019 Examinations


Page 16 SP7-27: Interpreting accounts

Data for Company A and Company B, years 1 to 4


The data in this table is in £m.

Revenue and P&L data Company A Company B

Year 1 2 3 4 1 2 3 4

Gross premiums written 781 795 784 825 147 175 203 245

Net premiums written 602 617 622 675 129 150 177 211

Gross premiums earned 760 782 792 806 135 160 182 223

Net premiums earned 589 605 622 655 118 140 164 196

Net claims incurred 409 417 407 452 86 106 126 145

Increase in DAC 3 3 0 9 3 4 4 4

Expenses and commission 160 163 166 193 44 54 60 66

Investment income on insurance 85 81 90 104 10 10 12 15


funds

Other investment income 30 28 32 37 10 11 11 11

End of year data Company A Company B

Year 1 2 3 4 1 2 3 4

Unearned premium reserve:


 gross 325 334 327 334 62 73 87 106

 reinsurance ceded 80 77 70 57 8 9 10 14

Outstanding claims:
 gross 800 816 802 891 51 60 66 74

 reinsurance recoveries 178 196 170 164 7 8 7 10

Other liabilities 75 68 89 92 4 6 3 7

Shareholders’ funds 273 318 383 432 111 109 107 112

Total assets 1215 1263 1361 1528 213 231 246 275

You should now proceed with your own analysis of and comments on the key values and ratios for
each company. Do not look at the solution until you have written down your own answers.

© IFE: 2019 Examinations The Actuarial Education Company


SP7-27: Interpreting accounts Page 17

Analysis of company A and company B, years 1 to 4

Key values Company A Company B

Year 1 2 3 4 1 2 3 4

Growth in net premiums n/a 2% 0% 9% n/a 16% 18% 19%

Underwriting profit 23 28 49 19 –9 –16 –18 –11

Insurance profit 108 109 139 123 1 –6 –6 4

Total profit 138 137 171 160 11 5 5 15

Retained profit n/a 45 65 49 n/a –2 –2 5

Ratios Company A Company B

Year 1 2 3 4 1 2 3 4

Claim ratio 69% 69% 65% 69% 73% 76% 77% 74%

Expense ratio 27% 26% 27% 29% 34% 36% 34% 31%

Combined ratio 96% 95% 92% 98% 107% 112% 111% 105%

Percentage reinsured 23% 22% 21% 18% 12% 14% 13% 14%

Investment return n/a 8.8% 9.3% 9.8% n/a 9.5% 9.6% 9.9%

Profit margin 18% 18% 22% 19% 1% –4% –4% 2%

Return on capital n/a 50% 54% 42% n/a 5% 5% 14%

Solvency ratio 45% 52% 62% 64% 86% 73% 60% 53%

Assets to liabilities 129% 134% 139% 139% 209% 189% 177% 169%

Comments
Growth

Company A’s volume of business has been very stable from years 1 to 3, with moderate growth in
year 4. Company B has maintained a consistently high level of premium growth.

Profitability

Company A has been consistently very profitable, with a large underwriting profit in all years.
Underwriting profit slipped in year 4, quite possibly linked to the growth in premium income at
the time (eg possibly more competitive premium rates). Insurance profit has also been very good.
Insurance profit expressed as a percentage of premiums earned (to give the profit margin) shows
a very high and stable level of profitability.

In each year, Company A achieved a very high return on capital, probably higher than is
sustainable in the long term. In economics terminology, these profits would be described as
super-normal profits.

The Actuarial Education Company © IFE: 2019 Examinations


Page 18 SP7-27: Interpreting accounts

Company B, in contrast, has had very low and slightly less stable profitability. Underwriting losses
were made in all years. Even at the insurance result level, losses outweighed profits. Overall
therefore, Company B lost money writing insurance over the four years. It is possible that this
was part of a strategy by Company B to gain market share (note how net premiums written
increased by 64% in three years). Despite the insurance losses, the company managed to make
an overall profit in each year.

Consistent with the low profitability, Company B’s return on capital was poor, except for year 4
when it was moderate.

Financial strength

Both companies appear to be financially sound.

Company A started with an acceptable level of solvency, which improved over the next two years
because of the large retained profits. Solvency stabilised in year 4, when the impact of higher
premium income was neutralised by the retained profits.

Company B started the period with very large free reserves (and, hence, a high solvency ratio).
This allowed the company to rapidly increase the business volumes, without endangering
solvency. However, the high growth and low profitability has taken its toll. The solvency ratio has
fallen from 86% to 53%. Although the current level is comfortable, Company B may find it very
difficult to support continued rapid growth in volumes.

Claim ratios

Company A has maintained a low claim ratio. Other than the dip in year 3, there is no clear
pattern or trend.

Company B’s claim ratio is consistently higher and a little more erratic. Again, no clear trend.

Comparison of the claim ratios of the two companies is distorted by a number of key factors.
Firstly, if Company B had adopted a deliberate strategy of gaining market share through lower
premiums, we would expect to see higher claim ratios for Company B. Secondly, the two
companies might operate in different parts of the insurance market with different competitive
pressures.

There is some evidence from the figures that the companies have different mixes of business.
The reserves for outstanding claims are much smaller for Company B than for Company A (about
35% of premiums earned for B compared with over 100% for A) suggesting that Company B writes
short-tail business. Perhaps this market is more competitive.

Expense ratios

Company A has lower expense ratios than Company B. Although this might reflect greater
efficiency and possibly economies of scale, it might also reflect different commission
arrangements in their respective markets. Possibly Company B has been paying high commission
or high advertising costs to encourage growth in premium income.

In neither case is there enough data to be conclusive, but since year 2, Company A’s expense ratio
has been on a deteriorating trend and Company B’s expense ratio on an improving trend.

© IFE: 2019 Examinations The Actuarial Education Company


SP7-27: Interpreting accounts Page 19

Reinsurance

Company A uses more reinsurance than Company B, perhaps this is consistent with the different
classes of business.

Company A’s use of reinsurance has fallen slightly (especially in year 4), whereas Company B’s use
of reinsurance has been more stable.

Investment

Both companies have made slight improvements in their investment returns over the period with
Company B achieving marginally better returns in each year.

We need to establish what happens to capital gains and whether the investment policies differ
between the two companies.

Other observations

Given that insurance is a volatile business, the results for both companies are remarkably stable.

The Actuarial Education Company © IFE: 2019 Examinations


Page 20 SP7-27: Interpreting accounts

5 Glossary items
Having studied this chapter you should now read the following Glossary items:
 Claim ratio
 Combined ratio
 Expense ratio
 Operating ratio
 Underwriting ratio.

© IFE: 2019 Examinations The Actuarial Education Company


SP7-27: Interpreting accounts Page 21

Chapter 27 Summary
The main accounting concepts are:
 accruals
 going concern
 consistency
 prudence
 separate valuation of assets and liabilities.

The strength of the reserving basis is subject to many factors which potentially require a
subjective view. Differences in reserving bases between different companies can distort the
comparison of profitability and strength between the companies.

Strong reserving bases increase the size of reserves and defer the emergence of profit.

Differences in the basis for valuing assets also make it difficult to compare the values and
ratios for different companies.

Changes in the mix of business from one year to the next or unusual events may have a
distorting impact on the picture from the accounts.

Accounting ratios help with comparisons over time and also between companies. The ratios
you should calculate from the accounts are:
 claim ratio (loss ratio)
 expense ratio
 commission rate
 combined ratio
 proportion reinsured
 investment return
 profit margin
 return on capital
 solvency ratio
 assets to liabilities
 claims settlement pattern.

The Actuarial Education Company © IFE: 2019 Examinations


Page 22 SP7-27: Interpreting accounts

The practice questions start on the next page so that you can
keep the chapter summaries together for revision purposes.

© IFE: 2019 Examinations The Actuarial Education Company


SP7-27: Interpreting accounts Page 23

Chapter 27 Practice Questions


27.1 Suggest under what circumstances a company might choose to use a less conservative basis for
estimating its claims reserves.

27.2 List the parties that may be interested in the future financial strength and profitability of a general
insurance company.

27.3 (i) A general insurance company has a solvency ratio of 50% of gross written
premiums. Calculate this ratio at the end of the financial year assuming that:
 the company pays no tax or dividends
 investment income on free reserves is earned at 5% per annum
 the reserves are exactly correct to meet claim payments
 business is sold with an insurance profit of 3% of earned premiums
 new business has increased by 15% over the previous year.

(ii) Assume that this rate of expansion at this level of profitability continues indefinitely.
Explain briefly the impact on the company’s solvency ratio in the long term.

27.4 State the information you would need to calculate the return on capital employed for an
insurance company.

27.5 (i) Explain why it is common to make use of ratios when comparing the accounts of a
number of different companies over a number of different years.

(ii) The following information is drawn from the accounts of a general insurance company:
Gross written premiums: 79.2
Reinsurance premiums: 1.9
Increase in unearned premiums: 11.9
Investment income: 6.8
Gross claims paid: 38.4
Reinsurance recoveries: 0.6
Increase in reserve for outstanding claims: 6.0
Expenses: 13.8
Net commissions: 9.1
Increase in deferred acquisition expenses: 1.8

Calculate the operating ratio for the company in this year.

The Actuarial Education Company © IFE: 2019 Examinations


Page 24 SP7-27: Interpreting accounts

27.6 State the formula normally used to calculate each of the following items and list any other names
for the item that are in reasonably common use:
(i) loss ratio
(ii) expense ratio
(iii) commission rate
(iv) operating ratio
(v) proportion reinsured
(vi) investment return
(vii) profit margin
(viii) return on capital
(ix) solvency ratio
(x) asset to liability ratio.

27.7 An insurer writes annual policies uniformly over the year. The risk is uniform over the year. At
the start of the year the company has a solvency ratio of 50%. It expects to write 18% more
business in the next year than it did in the last one.

Calculate the profit margin it must secure to maintain a 50% ratio at the end of the year, if it
expects to earn 10% on its initial free reserves and pay dividends of 60% of its post-tax profit.

Assume that tax is paid on profits at 30% per annum.

27.8 A general insurance company has been using undiscounted technical reserves for many years. On
Exam style
this basis, the key accounting data for one particular year is as follows:
 written premium was £1,800 million
 loss ratio has been stable at 70% for several years
 the underwriting result for the year was a loss of £2 million
 acquisition costs (ie commission) were 25%
 investment return was 8% on funds invested
 brokers hold premiums for 3 months before passing them to the insurer
 solvency ratio at the end of the year was 40%
 tax at 30% of gross profit is paid on the last day of the year
 there have been no dividends for many years
 premium rates are increased by 5% each 1 January
 business volumes have been unchanged in the last five years
 claim inflation has been steady at 5% per year.

© IFE: 2019 Examinations The Actuarial Education Company


SP7-27: Interpreting accounts Page 25

The pattern of emerging claims costs (in nominal terms) has been stable for many years:

1st year 40%


2nd year 30%
3rd year 20%
4th year 10%

Prepare the accounts of the company, using a 5% discount rate for outstanding claims. [18]

27.9 You are given the following information about an insurance company that starts trading at 1/1/X,
Exam style
with free reserves of $10m:
 written premium in calendar year X is $45m, and $60m in X+1
 premiums (and risk) are written evenly over the year
 all assets are invested in government stocks
 investment income in X is $3.3m. There were no realised capital gains or losses, but the
company has unrealised losses of $1.5m
 investment income in X+1 is $4.2m. There were no realised capital gains or losses, but the
company has further unrealised losses of $1.5m
 claims paid in X were $12m, and reserves for outstanding claims (including IBNR) at
31/12/X were $6m
 claims paid in X+1 were $30m, and reserves for outstanding claims (including IBNR) at
31/12/X+1 were $14m
 expenses for X and X+1 were $5m and $6m respectively
 commission payments for years X and X+1 were $7m and $9m respectively
 only commission is used in calculating deferred acquisition costs
 no tax or dividends are paid.

Prepare the revenue accounts for the company for X and X+1, and calculate:
(a) the claim and expense ratios for X and X+1
(b) the free reserves as at 31/12/X and 31/12/X+1. [8]

The Actuarial Education Company © IFE: 2019 Examinations


Page 26 SP7-27: Interpreting accounts

27.10 A general insurance company, which started writing business on 1st January year X, has made the
Exam style
following projections (€000):

Year X Year X+1 Year X+2


Written premiums 5,018 5,800 6,500
UPR c/f 2,010 2,450 2,750

Expense ratio 33% 30% 28%


Claim ratio 75% 75% 72%

The company:
 has not discounted claim reserves in calculating the claim ratio
 expects to earn 8% pa on its assets as a risk-free rate of return
 will pay no dividends and no tax
 has funds of €4m at the start of year X.

The claim settlement pattern is expected to be:


 50% when the claim event occurs
 25% after 1 year
 25% after 2 years.

Prepare a projected revenue account and profit and loss account for management using
discounted claims reserves for the year ending X+2. You may assume that the experience over
the period will be as projected. State any other assumptions you make. [14]

© IFE: 2019 Examinations The Actuarial Education Company


SP7-27: Interpreting accounts Page 27

27.11 A general insurance company that writes only one-year travel insurance policies and extended
Exam style
warranty insurance started trading on 1 January 2017, with $2m capital, and ceased trading on
31 December 2017.

The company wrote $10m of written premium of which 60% was travel insurance. The extended
warranty business was written evenly throughout the year but the average start date of the travel
business was 1 April.

The loss ratios from the business written in the year were 55% for the travel business and 75% for
the extended warranty business. The company paid 20% initial commission on all business and
other expenses amounted to $0.5m in 2017 and $0.3m in 2018.

The company received investment income of $0.4m in 2017 and $0.45m in 2018 and paid tax at
rates of 5% of written premiums and 8% of profits, net of the tax on premiums.

The company calculates reserves without discounting cashflows but on average claims are
reported one month after they occur and are settled a further one month later. The two classes
are aggregated together in order to calculate any additional reserve for unexpired risk.

(i) Prepare the balance sheet as at 31 December 2017 and calculate the profit declared at
the end of 2017 and 2018 if the company accounts using a one-year accident year basis.
[10]

(ii) Calculate the revised profit declared had the company discounted future claim payments
at 10% pa when calculating its outstanding claims reserves and list the advantages and
disadvantages of such actions. [9]

(iii) Calculate the profit that would be declared at the end of 2017 and 2018 if the company
uses a two-year underwriting year basis and comment briefly on your results. [3]
[Total 22]

The Actuarial Education Company © IFE: 2019 Examinations


Page 28 SP7-27: Interpreting accounts

27.12 You have been given the following information, extracted from the accounts of a general
Exam style
insurance company. All figures are in $(millions).

Revenue account
2018 2017
Earned premiums, net of reinsurance
Gross premiums written 561.6 620.9
Outwards reinsurance premiums (83.4) (221.7)
478.2 399.2
Change in the provision for unearned premiums – gross 18.2 37.2
– reinsurers’ share (66.5) (20.7)
429.9 415.7

Allocated investment return 92.3 (4.5)


522.2 411.2

Claims incurred, net of reinsurance


Claims paid – gross 320.7 433.1
– reinsurers’ share (56.4) (105.2)
264.3 327.9
Change in the provision for claims – gross (19.8) (116.3)
– reinsurers’ share 25.7 22.1
270.2 233.7

Change in other technical provisions, net of reinsurance (5) 2.4


Net operating expenses 185.4 162.7
450.6 398.8
Balance on technical account 71.6 12.4

Non-technical account
2018 2017
Balance on technical account 71.6 12.4
Non insurance result
Investment income and realised gains 130.9 108.3
Unrealised gains 42.2 –
Investment expenses and realised losses (55.0) (73.1)
Unrealised losses – (49.0)
118.1 (13.8)
Allocated investment return transferred to the technical (92.3) 4.5
account
25.8 (9.3)
Other income 5.4 2.6
31.2 (6.7)
PROFIT BEFORE TAX 102.8 5.7
Tax on profit 26.4 2.1
PROFIT FOR THE FINANCIAL YEAR 76.4 3.6

© IFE: 2019 Examinations The Actuarial Education Company


SP7-27: Interpreting accounts Page 29

Balance sheet
2018 2017
ASSETS
Investments
Land and buildings 3 4
Shares and variable yield securities 158 146
Fixed interest securities 970 877
1,131 1,027

Reinsurers’ share of technical provisions


Unearned premiums 41 108
Outstanding claims 111 137
Unexpired risks 27 3
179 248

Deferred acquisition costs 92 77


Miscellaneous 96 83

TOTAL ASSETS 1,498 1,435

LIABILITIES
Capital and reserves
Share capital 128 128
Share premium account 41 38
Profit and loss account 176 122
345 288
Technical provisions
Unearned premiums 368 386
Outstanding claims 626 646
Unexpired risks 119 100
1,113 1,132

Miscellaneous 40 15

TOTAL LIABILITIES 1,498 1,435

(i) Calculate the underwriting profit for the accounting year 2018. [1]

(ii) Discuss the features of this company that can be determined from these accounts under
the following headings:
 nature of the business
 profitability
 financial strength. [17]
[Total 18]

The Actuarial Education Company © IFE: 2019 Examinations


Page 30 SP7-27: Interpreting accounts

27.13 You are a consulting actuary who is conducting an analysis of the general insurance market in a
Exam style
developing country.

Explain why and how you might analyse accounting ratios of general insurers within the country,
including any problems you might encounter. You need not discuss any particular ratios. [10]

27.14 A proprietary general insurance company has recorded an operating ratio in excess of 100%.
Exam style
Discuss the suggestion that the company is failing to achieve a satisfactory level of profits and
should be closed down immediately. [10]

© IFE: 2019 Examinations The Actuarial Education Company


SP7-27: Interpreting accounts Page 31

Chapter 27 Solutions
27.1 A company might choose to use a less conservative basis for estimating claims reserves if it wants
or needs to boost its apparent profitability or financial strength.

Reasons for this might be:


 the solvency position would look weak if the reserving basis were conservative
 to enhance the value of the company in the eyes of a potential buyer.

27.2 Parties that might be interested include:


 regulators
 management and staff
 shareholders
 policyholders
 brokers
 rating agencies
 competitors.

27.3 (i) Solvency ratio

Let premiums written in the previous year be WP0.

So, free reserves at the start of the year will be 0.5 WP0.

Assume that premiums are written evenly over the year.

Therefore earned premium will be 1.075 WP0.

Free reserves will increase due to profit added and interest on the free reserves.

Therefore, free reserves at the end of the year are:


1.05  0.5 WP0 + 0.03  1.075  WP0 = 0.55725 WP0.

Therefore, the solvency margin is (0.55725 WP0 / 1.15 WP0) = 48.5%.

(ii) Long-term impact

The increase in new business will cause a reduction in the company’s solvency ratio, since the rate
of growth in premium income is greater than the growth in the level of free reserves.

However, the business that has been written is profitable, so the free reserves are growing, in
absolute terms. The solvency margin will, therefore continue to grow.

Eventually the solvency margin will stabilise at a new level, which is lower than the current
solvency ratio.

The Actuarial Education Company © IFE: 2019 Examinations


Page 32 SP7-27: Interpreting accounts

27.4 Assume that we want to calculate the return after tax; so we need:
 free reserves at start of year
 total pre-tax profits, ie insurance profit + investment income on free reserves
 tax rate.

27.5 (i) Use of ratios

Ratios are used to compare accounts of different companies over many different years because
they:
 are quick to produce and compare
 can indicate trends clearly, by eliminating the effects of inflation
 are flexible: different ratios can shed light on different aspects
 are robust: then can deal with different types of situation, eg:
– big company vs small company
– companies writing different types of business
– companies with different accounting layouts.

(ii) Operating ratio

Net written premium = 79.2 – 1.9 = 77.3


Net expenses & commission paid = 13.8 + 9.1 = 22.9
Hence expense ratio = 22.9 / 77.3 = 29.6%

Net claims incurred = 38.4 – 0.6 + 6.0 = 43.8


Net earned premium = 77.3 – 11.9 = 65.4
Hence claim ratio = 43.8 / 65.4 = 67.0%

So combined ratio = 29.6 + 67.0 = 96.6%

27.6 (i) loss ratio: net claims incurred / net earned premiums
Also known as: claim(s) ratio.
(ii) expense ratio: net expenses and commission paid / net written premiums
(iii) commission rate: net commission paid / net written premiums
Also known as: commission ratio and could be gross.
(iv) operating ratio: loss ratio + expense ratio
Also known as: combined ratio, underwriting ratio.

© IFE: 2019 Examinations The Actuarial Education Company


SP7-27: Interpreting accounts Page 33

(v) proportion reinsured: reinsurance premiums paid / gross written premiums


(vi) investment return: total investment income / average total assets
(vii) profit margin: insurance profit / net earned premiums
(viii) return on capital: post-tax profit / initial free reserves
(ix) solvency ratio: free reserves / net written premiums
Also known as: solvency margin, free reserve ratio, solvency margin ratio.
(x) asset to liability ratio: total assets / total liabilities

27.7 Let written premium during the year be 100, so free reserves at the end of the year are 50% of
100, ie 50.

Expect next year’s written premium to be 118, so want free reserves at the end of the next year of
50% of 118, ie 59.

Hence need retained profit for the year of 59 – 50 = 9.0

With dividends at 60% of post-tax profit, need post tax profit of 9.0 / 0.4 = 22.5

With corporation tax at 30%, need pre-tax profit of 22.5 / 0.7 = 32.14

ie insurance profit + investment income on free reserves = 32.14

Insurance profit = 32.14 – 0.10 x 50 = 27.14

Required profit margin is 27.14 / 109 = 24.9%.

27.8 Basic elements of the revenue account

The constant business volumes and constant 5% increase in premiums means than written
premium has increased by 5% each year.

Earned premium = 1,800  2 + 1,800  1.05  2 = 1,757.14 [½]

This assumes that risk and policies written are uniform over each year.

Claims incurred (undiscounted) = 0.7  1,757.14 = 1,230.00 [½]

Pattern of claim payments are given in nominal terms. We can use the patterns of claim
development to see the amounts of claim payment at each point. (We have completed the table
only for those numbers used.)

Oldest – – – 106.25
Previous – – 223.13 111.56
Last – 351.43 234.29 117.14
Current 492.00 369.00 246.00 123.00
[½]

The Actuarial Education Company © IFE: 2019 Examinations


Page 34 SP7-27: Interpreting accounts

From the table, the following amounts (undiscounted) can be calculated:

claims paid in the year = 492.00 + 351.43 + 223.13 + 106.25 = 1,172.81 [½]

o/s claims b/f = 106.25 + 223.13 + 111.56 + 351.43 + 234.29 + 117.14 = 1,143.80 [½]

o/s claims c/f = 111.56 + 234.29 + 117.14 + 369.00 + 246.00 + 123.00 = 1,200.99 [½]

[check, increase in o/s = Paid + incurred ie 1,172.81  1,143.8 1.05  1,143.80  1,230.00 ]

Increase in DAC = 0.25  (UPR c/f – UPR b/f)

= 0.25  (1,800  2 – 1,800  1.05  2) = 10.71 [½]

Expenses & Commission = X, such that EP – IC – X + Inc. DAC = U/W

ie 1,757.14 – 1,230.00 – X + 10.71 = – 2.00

so, X = 539.85 [½]

of which 450 is commission and 89.85 of other expenses.

Investment income

This is the tricky bit. The easiest approach is (probably):


 Start with assets at the start of the year and add 8% investment.
 Subtract the investment income not earned (ie 8% of the money held by broker).
 Add the cashflow (P – C – E) with 4% interest (ie for half a year).
 Subtract the tax paid at year end.
 Equals the assets at year end.

Assets at start of year

Free reserves unknown, FR

UPR net of DAC at end of previous year = 0.5  1,800  0.75  1.05 = 642.86 [½]

o/s claims b/f = 1,143.80

Mid year cashflow

Cashflows occur mid year on average, but the broker holds premiums for 3 months. So:
0.75  1,800 net of commission premiums in, 1,172.81 claims out, 89.85 other expenses out

Total is 87.34 in. [1]

The figure must be adjusted to allow for the delay in premiums held by the broker.

© IFE: 2019 Examinations The Actuarial Education Company


SP7-27: Interpreting accounts Page 35

Non-invested funds

The broker holds a quarter of the year’s premium (net of commission) at any time. For the first
three months, this is based on the premiums paid in the previous year. For the remainder of the
year, it is based on the 1,800 written premiums in this year. Therefore, the broker holds the
following amount through the year on average:

0.25  ( 0.25  (0.75  1,800  1.05) + 0.75  (0.75  1800) ) = 333.48 [1]

Assets at year end

Solvency margin at end of year = 40%, so free reserves = 720.00 [½]

UPR at end of year = 0.75  1,800  0.5 = 675.00 [½]

o/s claims c/f at end of year = 1,200.99

Total assets at end of year = 720.00 + 675.00 + 1,200.99 = 2,595.99 [½]

Tax

We know that the retained profit (after tax) is the change in the free reserves. The tax at 30% can
be deduced from this as:
3/7  (720 – FR) [1]

Calculation of FR

(FR + 642.86 + 1,143.80)  1.08 – (333.480.08) + 87.34  1.04 – 3/7  (720 – FR) = 2,595.99 [1]

Solution to this is FR = 603.76. [1]

Investment income

This can be calculated in two ways:


 By working back from the equation: u/w result + I = pre-tax profit, so:
I = (720 – 603.76)/0.7 + 2 = 168.06.
 By extracting the investment income bits from the equation above:
I = (FR + 642.86 + 1,143.80)0.08 – (333.480.08) + 87.340.04 = 168.05. [1]

Discounted accounts

Most items in the accounts will be unchanged, but we need to revise the amounts for outstanding
claims and for incurred claims.

o/s claims b/f = (106.25 + 223.13 + 351.43)/1.05½ + (111.56 + 234.29)/1.051½ + 117.14/1.052½

= 1,089.54 [1]

o/s claims c/f = o/s b/f 1.05  1,144.01 [½]

claims incurred = 1,172.81 + 1,144.01 – 1,089.54 = 1,227.29 [½]

The Actuarial Education Company © IFE: 2019 Examinations


Page 36 SP7-27: Interpreting accounts

Discounted revenue account (assumes b/f and c/f discounted)

Earned premium 1,757.14


Claims incurred 1,227.29
Expenses 539.85
Increase in DAC 10.71
Underwriting result 0.71
Investment return 168.05
Gross profit 168.76
Tax 50.63 assuming tax applies to discounted result
Retained profit 118.13
[2]

Balance sheet

Assets Funds with broker 337.50 ie 0.251,8000.75


Investments 2,258.49 ie 2,595.99 – 337.50
Total 2,595.99

Liabilities O/s claims 1,144.01


UPR 675.00
Free reserves 776.98 balancing item
Total 2,595.99
[2]
[Total 18]

© IFE: 2019 Examinations The Actuarial Education Company


SP7-27: Interpreting accounts Page 37

27.9 Revenue account

Most of the items in the account below can be written straight into the account from the data
given.

Year X X+1
Written premium 45.0 60.0
+ unearned premium b/f – 22.5
– unearned premium c/f 22.5 30.0
= Earned premium 22.5 52.5 [1]

Claims paid 12.0 30.0


– outstanding claims b/f – 6.0
+ outstanding claims c/f 6.0 14.0
Claims incurred 18.0 38.0
Expenses and commission 12.0 15.0
Increase in DAC 3.5 1.0
_____ ____
Underwriting result (4.0) 0.5 [1]

Increase in DAC is shown separately from earned premium so that claims ratio is not distorted.

Underwriting result (4.0) 0.5


Investment income 3.3 4.2
____ ___
Insurance profit/(loss) (0.7) 4.7 [2]

Note: this account includes investment income, but makes no provision for capital gains (but only
unrealised losses in this case anyway).

(a) Ratios

Claims Ratio = claims incurred/earned premium

Claim ratio, year X = 18.0/22.5 = 80.0% [½]

Claim ratio, year X+1 = 38.0/52.5 = 72.3% [½]

Expense Ratio = expenses & commission/written premiums

Expense ratio, year X = 12.0/45.0 = 26.7% [½]

Expense ratio, year X+1 = 15.0/60.0 = 25.0% [½]

The Actuarial Education Company © IFE: 2019 Examinations


Page 38 SP7-27: Interpreting accounts

(b) Free reserves

How should we treat unrealised losses? Assume that they are recognised.

Year X X +1

Free reserves at 1/1 10.0 7.8

+ profit (0.7) 4.7

+ unrealised gains (1.5) (1.5)

Free reserves at 31/12 7.8 11.0 [2]


[Total 8]

27.10 Earned premium = written premium + UPR b/f – UPR c/f

(ignoring any distortion caused by deferred acquisition costs; not ideal, but there’s no data to
derive the earned premium more accurately)

Incurred claims = earned premium  claim ratio

X X +1 X +2
Earned premium 3,008 5,360 6,200
Incurred claims (undisc.) 2,256 4,020 4,464 [2]

Claim payment run off (based on 50%/25%/25%) can be deduced because the incurred claims
represents the actual (undiscounted) amounts to be paid. This assumes the pattern allows fully
for inflation (ie is not 50%/25%/25% in constant money terms). [½]

We assume that claims occur on average in the middle of the calendar year: [½]
Payment year
X X+1 X+2 X+3 X+4
Origin year
X 1,128 564 564
X+1 2,010 1,005 1,005
X+2 2,232 1,116 1,116
Claims paid 1,128 2,574 3,801 ? ? [2]

Cashflows each year: (premiums – claims – expenses = cashflow)

© IFE: 2019 Examinations The Actuarial Education Company


SP7-27: Interpreting accounts Page 39

Expenses are simply the expense ratio multiplied by written premium.

X: 5,018 – 1,128 – 1,656 = 2,234


X+1: 5,800 – 2,574 – 1,740 = 1,486
X+2: 6,500 – 3,801 – 1,820 = 879 [2]
Assuming premiums and expenses are spread evenly over each year, we can calculate assets at
1/1/X+2, and hence X+2 investment income, from the cashflows above: [½]
Fund at 31/12/X = 4,000  1.08 + 2,234  1.080.5 = 6,642 [½]
Fund at 31/12/X+1 = 6,642  1.08 + 1,486  1.080.5 = 8,718 [½]
Investment income in X+2 = 8,718  0.08 + 879  (1.080.5 – 1) = 732 [1]
We assume that claims reserves are discounted at the rate of 8%. [½]

Discounted claims reserves at 1/1/X+2: (564+1,005)/1.080.5 + 1,005/1.081.5 = 2,405 [½]

Discounted reserves at 31/12/X+2: (1,005+1,116)/1.080.5 + 1,116/1.081.5 = 3,035 [½]

Hence:
Technical reserves at 1/1/X+2 = 2,405 + 2,450 = 4,855 [½]
Technical reserves at 31/12/X+2 = 3,035 + 2,750 = 5,785 [½]
Investment income on technical reserves = (4,855+5,785)/2  0.08 = 426 [½]
Investment income on free reserves = 732 – 426 = 306 [½]

We assume no additional unexpired risk reserve is required (which seems reasonable given the
claim ratio). [½]

X+2 Revenue and P&L a/c €000


Written premium 6,500
+ UPR b/f (net of DAC) 2,450
– UPR c/f (net of DAC) (2,750)
Earned premium 6,200
Claims paid 3,801
– o/s claims b/f (2,405)
+ o/s claims c/f 3,035
less Claims incurred 4,431
less Expenses & commission 1,820
Underwriting result (51)
plus Investment income on tech reserves 426
Insurance result 375
plus Investment income on free reserves 306
Total profit 681 [2]
[Maximum 14]

The Actuarial Education Company © IFE: 2019 Examinations


Page 40 SP7-27: Interpreting accounts

27.11 (i) Balance sheet and declared profit

The following table summarises the key values required:

Travel Extended
warranty
Written premiums (WP) 6 4
2017 earned premiums (EP17) (using average start date) 4.5 2
2017 UPR = 2018 earned premiums 1.5 2
Loss ratio (LR) (given) 55% 75%
Total claims incurred (LR WP) 3.3 3

2017 claims incurred (LR  EP17) 2.475 1.5

2017 claims paid 1.925 1


Outstanding reported claims 0.275 0.25
IBNR 0.275 0.25
2018 claims paid 1.375 2
2018 claims incurred 0.825 1.5
DAC (20% of UPR) 0.3 0.4
[4]

Notes:

The 2017 claims paid for the travel business represents 7 months of claims because of the 2
month payment delay and the average start date being 1 April, ie:
7
 3.3  1.925
12

Similarly, the 2017 claims paid for extended warranty is in respect of 4 months of claims.

Outstanding reported claims are in respect of one month’s claims, eg for extended warranty:
1
 3  0.25
12

IBNR claims are also equal to one month’s claims due to the one month reporting delay.

Clearly, 2017 claims paid + outstanding reported + IBNR = 2017 claims incurred.

2018 claims paid = total claims less 2017 claims paid.

2018 claims incurred = total claims less 2017 claims incurred or 2018 claims paid less 2017
outstanding reported claims and IBNR.

© IFE: 2019 Examinations The Actuarial Education Company


SP7-27: Interpreting accounts Page 41

Profit and loss accounts

2017 2018
Premiums earned 6.5 3.5
Claims incurred (3.975) (2.325)
Expenses paid (2.5) (0.3)
Increase in DAC 0.7 (0.7)
Investment income 0.4 0.45
Tax on premium 0.5 0
Profit 0.625 0.625
Tax on profits 0.05 0.05
Net profit 0.575 0.575
[3]
Note that the 2017 expenses paid = 20% WP + 0.5

Therefore the profit declared at the end of 2017 and 2018 is $0.575m.

Balance sheet

The total assets at the end of 2017 are calculated as:

assets at start of year 2


+ premiums 10
– claims paid (2.925)
– expenses paid (2.5)
+ investment income 0.4
– tax paid (0.55)
Assets at end of year 6.425
[1½]

The Actuarial Education Company © IFE: 2019 Examinations


Page 42 SP7-27: Interpreting accounts

Therefore the balance sheet at 31 December 2017 (in $m) is:

Assets 6.425
Liabilities
UPR 3.5
DAC (0.7)
o/s reported 0.525
IBNR 0.525
Total 3.85
Shareholders’ funds 2.575
[1½]
The increase in shareholders’ funds equals the 2017 profit as required. [Total 10]

(ii) Discounting claims

The outstanding reported claim payments will on average be in half a month’s time and the IBNR
payments in 1½ months’ time. Therefore the revised reserves are:

o/s reported 0.5251.11/24  $0.523m [1]

IBNR 0.5251.13/24  $0.519m [1]

The total reserve has therefore decreased by $0.008m. This will impact on the claims incurred
and hence profit, tax on profit and net profit. Assuming that the investment income is
unchanged, the revised figures are shown in the following table:

2017 2018
Claims incurred (3.967) (2.333)
Profit 0.633 0.617
Tax on profits 0.051 0.049
Net profit 0.583 0.567
[2]
Advantages

 Profit recognition is accelerated.


 Apparent solvency margin is improved.
 A more realistic view of the financial position of the company.
 Makes it easier to compare the performance of different classes of business. Although for
this company the claim characteristics of the two classes are very similar.
 Tax authorities may require discounting.
 Better to hold explicit reserves rather than rely on implicit prudence from not discounting.

© IFE: 2019 Examinations The Actuarial Education Company


SP7-27: Interpreting accounts Page 43

Disadvantages

 It may be a sign of weakness if the market practice is not to discount.


 Discounting would accelerate tax payments.
 Far less prudent (unless explicit margins are held, but these may not be tax deductible).
 The effort involved may not be worth the potential benefit for short-tail business, as
demonstrated by above calculations.
 There are difficulties in determining a suitable discount rate.
[½ mark per point, maximum 5]
[Total 9]

(iii) Funded accounts

If a two-year basis is being used then no profit will be declared at the end of 2017. [1]

All the profit will be declared at the end of 2018, ie $1.15m. [1]

This could be worked out by summing all the cashflows from the business, which in this case
equals the sum of the profits declared on an accident year basis. This is because the investment
income is assumed to be unchanged. You would normally expect the profit on the underwriting
year basis to be greater than the sum of the profits on the accident year basis as income could be
earned on the profits not declared at the end of the first year.

Therefore using a two-year underwriting year basis defers the recognition of profit until the end
of the two-year period. [1]
[Total 3]

27.12 (i) Underwriting profit

The underwriting profit can be found by deducting the allocated investment return from the
balance on the technical account, ie:
71.6  92.3  $20.7m [1]

(ii) Company features

Nature of the business

The investment mix is split roughly: 14% real assets, 86% fixed interest. This is fairly typical of a
general insurer’s asset mix reflecting the short-term nature of the company’s business and the
desire to reduce the volatility of results. [1]

The ratio of gross claims paid to claims outstanding is 51% for 2018 and 67% for 2017. This
implies that the average length of the tail of the business written is 1.5 to 2 years, ie relatively
short term. [1]

The change in this ratio might indicate that the business is becoming more long-tailed, perhaps
due to a change in the mix of business but we do not have enough data to draw any firm
conclusions. [1]

The Actuarial Education Company © IFE: 2019 Examinations


Page 44 SP7-27: Interpreting accounts

We are given the whole company’s accounts and so the company may write a variety of general
insurance business, the largest proportion being short-tail. [½]

The gross written premiums have fallen by 10% from 2017 to 2018 and from the change in the
unearned premiums figure we can deduce that business volumes also fell from 2016 to 2017. [1]

Net written premiums have increased by 20% caused by the ratio of net to gross written
premiums increasing from 64% to 85%. [1]

This indicates that there has been a significant reduction in the amount of reinsurance being used,
perhaps due to the amount retained under a quota share treaty being increased. [1]

This is supported by the ratio of net claims incurred to gross claims incurred increasing from 74%
to 90%. [½]

The ratio of unearned premiums at the end of the year to written premiums during the year is
about 65%. This might indicate that the majority of the business is written in the second half of
the year. Alternatively, it might due to an uneven incidence of risk or because some of the
policies written are for a term of more than one year. [1½]

Profitability

First let’s look at a number of performance and profit measures.

Loss ratio: defined as claims incurred divided by earned premiums, calculated on a gross or net of
reinsurance basis.

316.8 300.9
The gross loss ratio for 2017 is:  48% and for 2018 is:  52% [½]
658.1 579.8

233.7 270.2
The net loss ratio for 2017 is:  56% and for 2018 is:  63% [½]
415.7 429.9

Note that the earned premiums should be calculated using the UPR gross of DAC. The accounts do
not state explicitly whether the unearned premiums are net or gross of DAC, but by looking at the
change in gross unearned premiums given in the balance sheets, it looks like the gross figures are
given in the profit and loss account. The change in DAC must be included in the line for net
operating expenses.

The loss ratios look reasonable and are consistent with each other. The higher net ratios reflect
the cost of reinsurance. [½]

Expense ratio: use the definition of expenses divided by net earned premiums since the expense
item includes the change in DAC.

162.7 185.4
2017:  39% 2018:  43% [½]
415.7 429.9

This ratio looks quite high but may be due to the level of commission that must be paid to obtain
the business. [½]

Combined ratio: calculated here as the net loss ratio plus the expense ratio.

© IFE: 2019 Examinations The Actuarial Education Company


SP7-27: Interpreting accounts Page 45

This increases from 95% to 106%. This is consistent with the underwriting profit reducing from
$16.9m to –$20.7m. [½]

The balance on the technical account, sometimes called the insurance profit, is $12.4m in 2017
and $71.6m in 2018.

The profit margin, defined as the insurance profit divided by net earned premiums, increases from
3.0% in 2017 to 16.7% in 2018. [1]

The return on capital, defined as total profit divided by shareholders capital at the start of the
year is 26.5%. We are not given the capital at the start of 2017 but the return will be very low
since the profit is only $3.6m. [1]

The profits increase dramatically from 2017 to 2018. This is not due to volatile claims experience
which is similar in 2018 to 2017, as shown by the loss ratios. [½]

It is instead due to the volatile investment returns allocated to the account. This is because
unrealised gains are included in the profit and loss account and have varied considerably between
the two years. [1]

The investment return for 2018 calculated using the total income and gains divided by the
average investments is 10.9% but for 2017 is negative. [½]

The presence of a reserve for unexpired risks implies that some of the business written may be on
unprofitable rates. However, the valuation basis should be considered before a conclusion is
reached. [1]

Financial strength

The ratio of assets to liabilities (where liabilities do not include capital and reserves) increases
from 125% to 130%. [½]

The solvency ratio, defined as shareholders’ capital divided by net written premiums stays roughly
constant at 72%. [1]

This is usually considered healthy but not excessive. This is consistent with the company writing
mainly short-tail and probably relative low-risk business. [1]

However, the ratios considered will depend on the strength of the valuation basis and the values
should be compared with any regulatory requirements and those of any competitors. [1]
[plus up to two bonus marks for relevant points, maximum 17]

The Actuarial Education Company © IFE: 2019 Examinations


Page 46 SP7-27: Interpreting accounts

27.13 Reasons for analysing ratios

These include:
 to assess trends in the general insurance market, eg insurance cycle [½]
 to assess the market potential in terms of, for example: [½]
– size [½]
– competitive behaviour [½]
– profitability [½]
 for comparison with competitors, to assess the relative: [½]
– financial strength [½]
– competitiveness [½]
– profitability [½]
 ratios are more useful then absolute figures as they can allow for inflation [½]
 they allow automatically for certain differences or changes, eg in size of company [½]
 they are quick to analyse, easy to calculate and flexible to use. [1]

How they can be used

The ratios will be obtained from published accounts and other publicly-available sources. [½]

The amount of information may be severely limited in a developing country. [½]

We will need to allow for differences between companies and over time, for example in:
 reserving bases
 asset valuation bases
 products sold, terms and conditions
 underwriting and claims control
 definitions of figures used
 mix of business
 methods of sale
 changes in accounting practice over time
 reinsurance usage. [½ for each relevant example, maximum 2]

We should read the accompanying reports to see if there were any (or lack of) exceptional events
and their effects on the figures. [½]

We can look at changes over time to assess the strength of reserves – by comparing previously
reported figures with how they developed in reality. [1]

Looking at the differences over time for each company will allow us to assess trends. [½]

Looking at the differences between companies will allow us to see the relativities above. [½]

© IFE: 2019 Examinations The Actuarial Education Company


SP7-27: Interpreting accounts Page 47

We should take into account any legislation, guidance or accounting conventions on the
preparation of accounts. [1]

We will need to bear in mind the purpose for which the accounts were prepared. [½]
[Maximum 10]

27.14 Profits are made up of the operating ratio plus any investment income on technical and free
reserves. [½]

The investment income could be significant, … [½]

… especially for long-tail classes. [½]

We do not know how expenses and depreciation have been dealt with in the accounts. [½]

If it is a new company with high expenses, the operating ratio may be expected to be above 100%.
[½]

Any decision regarding closure to new business should be based upon the expected profitability of
future business, not the size of past losses. [1]

For example, the operating ratio can still be above 100% without it relating to the current
business being written, since the loss ratio reflects changes to past reserve estimates. [½]

We should consider the strength of the reserving basis, the operating ratio may be much lower if
the reserves are calculated on a realistic basis. [½]

Similarly, if the allowance for DAC, or other parts of the accounting basis are unrealistic. [½]

The difference will be high if the business is expanding and reserving is cautious. [1]

This would also be true if past reserving had been less cautious than at present. [½]

Also, the operating ratio is made up of expenses / written premiums. Strictly speaking, it should
be incurred expenses and earned premiums. [½]

This can give distorted results if the business is rapidly expanding. [½]

We should look at the figures over many years, not just one year in isolation. [½]

One year’s results may be distorted for a number of reasons including:


 catastrophes [½]
 a court award leading to a new latent claim [½]
 a jump in inflation [½]
 failure of a reinsurer. [½]

Even if one year’s business is unprofitable, there are probably more appropriate courses of action
than closure, for example increasing the premium rates. [1]
[Maximum 10]

The Actuarial Education Company © IFE: 2019 Examinations


Page 48 SP7-27: Interpreting accounts

End of Part 6

What next?
1. Briefly review the key areas of Part 6 and/or re-read the summaries at the end of
Chapters 24 to 27.
2. Ensure you have attempted some of the Practice Questions at the end of each chapter in
Part 6. If you don’t have time to do them all, you could save the remainder for use as part
of your revision.
3. Attempt Assignment X6.

Time to consider …
… ‘rehearsal’ products
Mock Exam and Marking – You can attempt the Mock Exam and get it marked. Results of
surveys have found that students who do a mock exam of some form have significantly higher
pass rates. Students have said:

‘I find the mock a useful tool in completing my pre-exam study. It helps


me realise the areas I am weaker in and where I need to focus my study.’

‘Brilliant feedback! SO thorough and encouraging. I really appreciate it. It’s


helped me know where I’ve gone wrong, how to improve and has given me so much
encouragement to know I can and will do better.’

You can find lots more information on our website at www.ActEd.co.uk.

Buy online at www.ActEd.co.uk/estore

And finally ...


Good luck!

© IFE: 2019 Examinations The Actuarial Education Company


SP7: Glossary Page 1

Glossary
Syllabus objectives
0.1 Define the principal terms in use in general insurance.

The Actuarial Education Company © IFE: 2019 Examinations


Page 2 SP7: Glossary

0 Introduction
In general insurance many terms vary by company, class of business, market or country.
An important part of any actuarial investigation is to verify the exact meaning of any
important terms used. This glossary gives the definitions mainly used in practice and an
appendix at the end provides a list of abbreviations used.

The glossary includes terms used in Subjects SP7, SP8 and SA3. An asterisk denotes
terms applicable to Subject SA3 only. Some terms are defined that are not used elsewhere
in the Core Reading – these are included because the candidate may come across them in
background reading.

A potential source of confusion is the term used to denote the value assigned to the
liabilities. It has been the practice of accountants to use the word ‘provision’ to denote the
value of a liability that is known or assumed to exist at the accounting date, and to confine
the term ‘reserve’ to an estimate of additional liabilities, over and above the provisions,
either in respect of future events or in respect of past events for which the provisions may
prove to be inadequate. However, among insurers, and also among actuaries, there has
been a long-established practice of applying the term ‘reserve’ to both categories.

In the European Union, following the adoption of the Insurance Accounts Directive and its
enactment in the legislation of each of the Member States, it has become the practice to
distinguish between provisions and reserves in insurance companies’ shareholders’
accounts and also in the accounts that form part of the statutory returns to the insurance
supervisory authorities. However, in North America and to some extent in the Lloyd's
market, the practice of applying the term ‘reserve’ to both categories continues. It seems
likely that among actuaries and others the habit of using the term ‘reserves’ for what are
often called provisions will persist for some time even in the UK, notwithstanding the
legislative changes.

This glossary generally uses ‘reserve’ in text, and in headings uses the word ‘reserve’
followed by ‘provision’ in brackets; for example:

Additional reserve (provision) for unexpired risk

It should be noted that the precise form of words may vary. Candidates may, for example,
find references to an additional unexpired risk reserve (or simply an unexpired risk reserve).

© IFE: 2019 Examinations The Actuarial Education Company


SP7: Glossary Page 3

24ths method
A method of estimating the unearned premium reserve, based on the assumption that
annual policies are written evenly over each month and risk is spread evenly over the year.
For example, policies written in the first month of the year are assumed to contribute 1/24th
of the month’s written premium to the unearned premium reserve at the end of the year.

365ths method
A method of estimating the unearned premium reserve, based on the assumption that
annual policies are written evenly over a year and that the risk is spread evenly over the 365
days of a year of cover. For example, where a policy was written 100 days ago, 265/365ths
of the premium is taken as being unearned.

Accident year
An accident year grouping of claims means that all the claims relating to loss events that
occurred in a 12-month period (usually a calendar year) are grouped together, irrespective
of when they are actually reported or paid and irrespective of the year in which the period of
cover commenced. Alternative methods of grouping are underwriting year or reporting
year.

Accumulation of risk
An accumulation of risk occurs when a single event can give rise to claims under several
different policies. Such an accumulation might occur by location (property insurance) or
occupation (employers’ liability insurance), for example.

Acquisition costs
Costs arising from the writing of insurance contracts, such as commission.

Act of God
An event, such as a storm or flood, that is unexpected and outside human control. From
the perspective of insurers, it is a cause of insurance losses.

Actual total loss*


A form of total loss, defined by the Marine Insurance Act 1906.

Actual total loss is deemed to occur in one of three ways:

 The insured item is totally destroyed.

 The item is so damaged that it can no longer be classed as the type of object
originally insured.

 The insured is irretrievably deprived of the insured item.

See also constructive total loss.

The Actuarial Education Company © IFE: 2019 Examinations


Page 4 SP7: Glossary

Additional reserve (provision) for unexpired risk (AURR)


The reserve held in excess of the unearned premium reserve, which allows for any
expectation that the unearned premium reserve will be insufficient to cover the cost of
claims and expenses incurred during the period of unexpired risk.

Adjustment premium
The adjustment premium is an additional premium payable at the end of a period of cover.
This may result from the use of retrospective experience rating or from a situation where
the exposure cannot be adequately determined at the start of the period of cover.

Adverse development cover


A reinsurance arrangement whereby a reinsurer agrees, in return for a premium, to cover
the ultimate settled amount of a specified block of business above a certain pre-agreed
amount.

Agents’ balances
Moneys (typically premiums) that belong to an insurer but are held by an agent.

Aggregate excess of loss reinsurance


A form of excess of loss reinsurance that covers the aggregate of losses, above an excess
point and subject to an upper limit, sustained from a single event or from a defined peril (or
perils) over a defined period, usually one year.

All risks
Cover that is not restricted to specific perils such as fire, storm, flood, and so on. The cover
is for loss, destruction or damage by any peril not specifically excluded. The exclusions
will often be inevitabilities like wear and tear. The term is sometimes loosely used to
describe a policy that covers a number of specified risks, though not all.

Allocated loss adjustment expenses (ALAE)


See claims handling expenses.

Annual basis of accounting


Annual accounting is one of two commonly used statutory bases of accounting, the other
being fund accounting. Annual accounting is based on the cover provided during the
accounting period, regardless of when the contracts of insurance start and end.

This is also known as one-year (accident-year) accounting.

Anti-selection
The preference of some insurance applicants for policies whose underwriting requirements
are less stringent than others. Thus anti-selection refers to an asymmetry of information
between policyholder and insurer where the former has more knowledge of the negative
aspects of the risks presented than the latter.

© IFE: 2019 Examinations The Actuarial Education Company


SP7: Glossary Page 5

Asset liability modelling (ALM)


A form of actuarial projection which analyses future flows of asset income against liability
outgo.

Atafs  Age to age factors


Used by the CAS to refer to link ratios or development factors.

Atufs  Age to ultimate factors


Used by the CAS in triangulation reserving methods to refer to the grossing-up factor to get
from an intermediate period of development to ultimate.

Average
In non-marine insurance, the term relates to the practice of reducing the amount of a claim
in proportion to the extent of underinsurance.

In marine insurance, the term is generally used to describe damage or loss.

Average cost per claim method (ACPC)


A method of reserving which relies on the average cost of claims paid or incurred.

Balance of a reinsurance treaty


The ratio of the total premiums receivable by a reinsurer under a surplus treaty to the
reinsurer’s maximum liability for any one claim, based on estimated (or expected) maximum
loss (EML).

Bancassurance
An arrangement in which a bank and an insurance company form a partnership so that the
insurance company can sell its products to the bank's client base. This partnership
arrangement can be profitable for both companies.

Benchmark
A benchmark is any statistic derived from external sources; for example, loss ratio,
expense-related measure, claim reporting or claim payment development pattern.

Binding authorities
Contractual agreements setting out the scope of delegated authority, allowing cover holders
to enter into contracts of insurance and to issue insurance documents on behalf of Lloyd’s
managing agents.

Bonus hunger
The reluctance of policyholders under a no-claim discount (NCD) or bonus-malus system to
notify claims or claim amounts when faced with a potential increase in premiums. Also
known as hunger for bonus.

The Actuarial Education Company © IFE: 2019 Examinations


Page 6 SP7: Glossary

Bonus-malus
A rating system in which the base premium can be discounted or loaded in response to the
policyholder’s claims experience.

Bordereau
A detailed list of premiums, claims and other important statistics provided by ceding
insurers to reinsurers (or by coverholders to insurers in direct insurance), so that payments
due under a reinsurance treaty (or delegated authority schemes in direct insurance) can be
calculated.

Bornhuetter-Ferguson method (BF or BHF)


A reserving method which uses weights based on an a priori loss ratio and claim
development.

Break-up basis
A valuation basis that assumes that the writing of new business ceases and cover on
current policies is terminated. Current policyholders would normally be entitled to a
proportionate return of the original gross premium and deferred acquisition costs would
probably have to be written off. Also known as a wind-up basis. An alternative to this is
using a going-concern basis.

Broker
An intermediary between the seller and buyer of a particular insurance contract who is not
tied to either party. A reinsurance broker is similarly defined where reinsurance contracts
are bought and sold. See also Lloyd’s broker.

Burning cost
The actual cost of claims paid or incurred during a past period of years expressed as an
annual rate per unit of exposure. This is sometimes used (after adjustment for inflation,
incurred but not reported claims (IBNR), and so on) as a method of calculating premiums for
certain types of risks or monitoring experience, for example, motor fleets and
non-proportional reinsurance.

Business interruption insurance


Insurance cover for financial losses arising following damage (for example, by a fire) to
business premises. Also called loss of profits or consequential loss insurance. This is
sometimes referred to as BI insurance.

Cancellation
A mid-term cessation of a policy, which may involve a partial return of premium.

Capacity
The amount of premium income that an insurer is permitted to write or the maximum
exposure it is permitted to accept. It could refer to an insurance company, a Lloyd’s Name,
a Lloyd’s syndicate or a whole market.

© IFE: 2019 Examinations The Actuarial Education Company


SP7: Glossary Page 7

Cape Cod method


A reserving method, similar to the Bornhuetter-Ferguson method which, instead of an a
priori loss ratio, uses weights proportional to a measure of exposure and inversely
proportional to claims development.

Captive
An insurer wholly owned by an industrial or commercial enterprise and set up with the
primary purpose of insuring the parent or associated group companies, and retaining
premiums and risk within the enterprise. Some insurers are set up with the primary
purpose of selling insurance to the customers of the parent. These are often known as
captives but, as they write third-party business, this is not strictly correct. If the word
‘captive’ is used without qualification it should be assumed that only the parent or
associated group companies are being insured. Lighter regulatory capital requirements for
captive reinsurers only apply if the purpose of the captive is to provide cover exclusively for
the risks of the undertaking or group to which it belongs and so does not provide cover for
third parties.

Casualty Actuarial Society (CAS)


The Casualty Actuarial Society is a professional society of actuaries based in the US. Its
members are mainly involved in the property and casualty areas of the actuarial profession.

Case by case estimation


A method of determining the reserves for outstanding reported claims. Each outstanding
claim is individually assessed to arrive at an estimate of the total payments to be made. The
shorter term ‘case estimation’ is often used and the estimates are referred to as case
estimates.

Casualty insurance
Specifically, the term is used in the US, and to a lesser extent in the UK, as an alternative to
liability insurance. In a wider context ‘casualty insurance’ may be used as a phrase to cover
all non-life insurance.

Catastrophe
In the context of general insurance, a catastrophe is a single event that gives rise to an
exceptionally large aggregation of losses.

Catastrophe reinsurance
This is a form of aggregate excess of loss reinsurance providing coverage for very high
aggregate losses arising from a single event. It is common that these will contain a clause
to limit the claims that can be made on the policy to be spread over a set number of hours;
often 24 or 72 hour periods are used. See related hours clause.

Catastrophe reserve
A reserve built up over periods between catastrophes to smooth the reported results over a
number of years. The purpose of a catastrophe reserve is smoothing profit not solvency.

The Actuarial Education Company © IFE: 2019 Examinations


Page 8 SP7: Glossary

Ceding company (cedant)


An insurer or reinsurer that passes (or cedes) a risk to a reinsurer. The insurer or reinsurer
may be a company or a Lloyd’s syndicate.

Central fund (Lloyd’s)


Specific to Lloyd’s, this is a contingency reserve built up from contributions by Lloyd’s
Names and held by Lloyd’s as a layer of protection for policyholders. It is used to
demonstrate overall solvency to the regulator. This capital is in addition to members’
capital resources held as Funds at Lloyd’s (FAL). See related Funds at Lloyd’s.

Chain ladder method (Basic chain ladder, CL, BCL)


A statistical method of estimating outstanding claims, whereby the weighted average of
past claim development is projected into the future. The projection is based on the ratios of
cumulative past claims, usually paid or incurred, for successive years of development. It
requires the earliest year of origin to be fully run-off or at least that the final outcome for
that year can be estimated with confidence. If appropriate, the method can be applied to
past claims data that have been explicitly adjusted for past inflation. Further variations on
this original theme are now in wide usage.

Claim
The word ‘claim’ has a variety of meanings. The most common ones are:

 as a noun: an assertion by a policyholder that an insurer is liable to make a payment


in accordance with the terms of a policy

 as a verb: to make a request for payment from an insurer.

Care is often needed to discover the precise meaning in a given context; for
example, whether a reference to ‘claims’ is to the number of claims or their cost.

Claim amount distribution


A statistical frequency distribution describing the total amount of claims.

Claim cohort
A group of claims with a common period of origin. The period is usually a month, a quarter
or a year. The origin varies but is usually defined by the incident date of a claim, the date of
reporting of a claim, the date of payment of a claim, or the date when the period of cover to
which a claim attaches commenced.

Claim cost inflation


The rate of increase in the cost of like-for-like claim payments.

Claim frequency
The number of claims in a period per unit of exposure, such as the number of claims per
vehicle year for a calendar year or the number of claims per policy over a period.

© IFE: 2019 Examinations The Actuarial Education Company


SP7: Glossary Page 9

Claim frequency distribution


A statistical frequency distribution for claim occurrence.

Claim ratio
The ratio of the cost of claims to the corresponding premiums, either gross or net of
reinsurance. An alternative term is loss ratio.

Claim size distribution


A statistical distribution describing the size of individual claims.

Claims handling expenses (CHE)


The expenses incurred in handling and settling claims are known in some countries,
including the UK, as claims handling expenses, the equivalent term in the US (and
increasingly elsewhere) being ‘loss adjustment expenses’.

In the US the terms allocated loss adjustment expenses (ALAE) and unallocated loss
adjustment expenses (ULAE) are used.

Claims incurred
See incurred claims.

Claims made policy


A policy that covers all claims reported to an insurer within the policy period irrespective of
when the incident occurred. The type of cover provided by such a policy is known as
claims made cover.

Claims reported
Claims incurred that have been reported to the insurer. The term is often used in relation to
those claims reported during the accounting period. It may refer to the number of claims
themselves or the cost of claims that have been reported.

Claims run-off analysis


A tabulation showing the speed of reporting or settlement for cohorts of claims. Also called
a delay table or, since it is usually triangular in form, a run-off triangle. The analysis may be
in terms of claim numbers or claim amounts. It is often presented as an intermediate step in
a chain ladder projection.

Clash cover
Excess of loss cover for liability business, limiting insurers’ exposure to the risk that one
event gives rise to claims on more than one policy.

The Actuarial Education Company © IFE: 2019 Examinations


Page 10 SP7: Glossary

Closed year
In the case of fund accounting a closed year is an underwriting year that is older than the
prescribed limit for the class in question. In the Lloyd’s market, a closed year is one that
has been closed by reinsurance to close (RITC).

Coinsurance
An arrangement whereby two or more insurers enter into a single contract with the insured
to cover a risk in agreed proportions at a specified premium. Each insurer is liable only for
its own proportion of the total risk. It is frequently applied to individual ‘slip’ business in
the London Market where a lead insurer takes a major share of the risk and manages the
outturn, while others subscribe on fixed terms. See related slip system.

The term is also used in direct insurance and reinsurance to describe an arrangement in
which the insured or cedant retains a proportion of their own risk.

Combined ratio or Combined operating ratio


The sum of the claim ratio and the expense ratio (and thus not a ratio itself, unless the two
separate ratios have the same denominator). Also called the operating ratio or underwriting
ratio. The fact that the denominators for the claim and expense ratio may be different can
give rise to anomalies.

Commercial lines
Classes of insurance for commercial and business policyholders. Those for individuals are
usually referred to as personal lines.

Committee of Lloyd’s
A committee that is responsible for administrative matters within Lloyd’s under delegation
from the Council of Lloyd’s. Prior to the establishment of the Council of Lloyd’s by the
Lloyd’s Act 1982, the Committee had sole responsibility for the overall direction of Lloyd’s.

Commutation
The process of prematurely terminating a reinsurance contract by agreeing an amount to
settle all current and future claims.

Commutation account
A register of the inflows and outflows to the treaty after the commutation has taken place.

Commutation clause
A clause in an insurance or reinsurance contract that allows the contract to be commuted
under certain conditions. The clause works in conjunction with commutation accounts,
which are used to calculate the relevant numbers.

Composite insurer
A single insurance company that writes both life and non-life business.

© IFE: 2019 Examinations The Actuarial Education Company


SP7: Glossary Page 11

Consequential loss insurance


See business interruption insurance.

Constructive total loss*


An expression defined by the Marine Insurance Act. Constructive total loss is where the
insured abandons the insured item because an ‘actual total loss’ is unavoidable or because
the costs of preventing a total loss exceed the value saved.

Co-reinsurance
Similar to coinsurance, but referring to reinsurance of a risk rather than insurance.

Council of Lloyd’s
The governing body responsible for the overall direction of Lloyd's. It was established as a
result of the Lloyd’s Act 1982 and consists of six working members, six external members
and six nominated members whose appointment must be confirmed by the Governor of the
Bank of England. One of the nominated members is the Chief Executive.

Cover note
A note issued by an insurance company to confirm the existence of insurance cover
pending the issue of formal policy documentation.

Credibility
A statistical measure of the weight to be given to a statistic.

CRESTA zones
The Global Catastrophe Risk Evaluating and Standardising Target Accumulations (CRESTA)
zone data set helps brokers and reinsurers assess and present risks, based on the zoning
system established by the world's leading reinsurers. Based primarily on the observed or
expected seismic activity (although drought, flood and wind storms are also considered)
within a country, CRESTA zones consider the distribution of insured values within a country
as well as administrative or political boundaries for easier assessment of risks.

Deductible
The amount which, in accordance with the terms of the policy, is deducted from the claim
amount that would otherwise have been payable and will therefore be borne by the
policyholder. See also excess.

Deep pocket syndrome


A situation where claims are made based on the ability of the defendant to pay rather than
on share of blame. An injured party may try to blame the party with the greatest wealth (that
is, deepest pocket) where there is more than one potential defendant.

The Actuarial Education Company © IFE: 2019 Examinations


Page 12 SP7: Glossary

Deferred acquisition costs (DAC)


Acquisition costs relating to unexpired periods of contracts in force at the balance sheet
date. They are carried forward as an asset from one accounting period to subsequent
accounting periods in the expectation that they will be recoverable out of future margins
within insurance contracts after providing for future liabilities.

Delay table
See claims run-off analysis.

Deposit premium
This occurs in cases where all relevant exposure or rating information is not known at the
start of the period of cover, or the premium to be paid is dependent on the claims
experience during the policy term. An initial ‘deposit’ premium is paid at the start of the
period of cover, followed by an adjustment at the end when the information required is
known.

Where this latter adjustment is stipulated at the outset as being upwards only, the term
‘Minimum and Deposit Premium’ applies.

Where it is found in cases relating to retrospective experience rating, the term ‘swing rated
premium’ is often applied.

Development factors
The factors emerging from a chain ladder calculation that are the ratios of claims in
successive development periods. Sometimes known as link ratios.

Direct business
This term has two meanings:

 Business acquired without the intervention of an intermediary.

 The cover provided by an insurer to an original policyholder, as opposed to any


reinsurance cover provided for the insurer.

The meaning intended is usually clear from the context in which the term is used.

Discovery period
A time limit, usually defined in the policy wording or through legislative precedent, placed
on the period within which claims must be reported. It generally applies to classes of
business where several years may elapse between the occurrence of the event or the
awareness of the condition that may give rise to a claim and the reporting of the claim to the
insurer, for example, employers’ liability or professional indemnity.

Dynamic financial analysis (DFA)


A phrase given to any form of actuarial modelling in financial services.

© IFE: 2019 Examinations The Actuarial Education Company


SP7: Glossary Page 13

Earned premiums
The total premiums attributable to the exposure to risk in an accounting period; they can be
gross or net of adjustment for acquisition expenses and gross or net of reinsurance.

Eighths method
A method of estimating the unearned premium reserve, based on the assumption that
annual policies are written evenly over each quarter and the risk is spread evenly over the
year.

Endorsement
Some change to the policy wording, usually following a change in the risk covered, which
takes effect during the original period of insurance and is usually, but not necessarily,
accompanied by an alteration in the original premium.

Escalation clause
A policy clause that permits the insurer to raise automatically the level of benefits or sum
insured (and therefore the premium) in line with some form of inflation index.

Estimated (or expected) maximum loss (EML)


The largest loss that is reasonably expected to arise from a single event in respect of an
insured property. This may well be less than either the market value or the replacement
value of the insured property and is used as an exposure measure in rating certain classes
of business.

See related probable (possible) maximum loss (PML).

Event
An occurrence that may lead to one or more claims.

Events not in data (ENIDs)


The balancing amount required to bring the best estimate claim liabilities before ENID up to
an amount allowing for all possible future outcomes. In other words, this is the additional
amount that needs to be included in the best estimate claim liabilities to ensure that the
best estimate is a true best estimate of all possible outcomes, as opposed to something
less, such as a best estimate of all reasonably foreseeable outcomes. In theory, this means
that one needs to allow for possibilities that one may not have experienced before either
within the (re)insurer or across the entire insurance industry.

Excess
The amount of a claim, specified in the policy, that the insured must bear before any liability
falls upon the insurer. See also deductibles.

The Actuarial Education Company © IFE: 2019 Examinations


Page 14 SP7: Glossary

Excess and surplus lines insurance


Excess and surplus lines insurance is a segment of the insurance market that allows
consumers to buy property and casualty insurance through the state-regulated insurance
market, where policyholders, agents, brokers and insurance companies all have the ability
to design specific insurance coverages and negotiate pricing based on the risks to be
secured.

Excess of loss (XL or XOL) reinsurance


A form of reinsurance whereby the reinsurer indemnifies the cedant for the amount of a loss
above a stated excess point, usually up to an upper limit. The excess point and upper limit
may be fixed, or indexed as specified in a stability clause. Usually this type of reinsurance
relates to individual losses, but it can be a form of aggregate excess of loss reinsurance
covering the total of all losses in a period and subject to a total aggregate claim limit. See
also risk excess of loss and aggregate excess of loss.

Exclusion
An event, peril or cause defined within the policy document as being beyond the scope of
the insurance cover.

Expected maximum loss (EML)


See estimated (or expected) maximum loss (EML).

Expense ratio
The ratio of management expenses plus commission to premium (usually calendar
accounted expenses to written premium, or sometimes to earned premium).

Experience account
Often a feature of multi-year financial engineering contracts, this is an account that tracks
the performance of the business reinsured by the treaty so that the profitability or otherwise
of the treaty can be determined.

Experience rating
A system by which the premium of each individual risk depends, at least in part, on the
actual claims experience of that risk (usually in an earlier period, but sometimes in the
period covered). The latter case is sometimes referred to as swing rated or loss sensitive,
and there are often upper and lower limits defining a ‘collar’ or ‘corridor’.

Experience rating also has a more general meaning; for example, in the context of London
Market rating. In this context, it is a rating based purely on the experience of the historic
risk presented, as opposed to ‘exposure rating’.

Expiry date
The date on which the insurance cover for a risk ceases.

© IFE: 2019 Examinations The Actuarial Education Company


SP7: Glossary Page 15

Exposure
This term can be used in three senses:

 the state of being subject to the possibility of loss

 a measure of extent of risk

 the possibility of loss to insured property caused by its surroundings.

Exposure statistics are usually shown in one of three common bases: written exposures,
earned exposures and in-force exposures.

Exposure rating
A method of calculating the premium that is based on external data or benchmarks. The
risk profile (exposure) of every insured from the product in question is examined.
Scenarios of losses of various sizes are analysed and the impact on the policies is
determined. The premium of each individual insured does not depend on the actual claims
experience of that insured. Instead, the amount of exposure that the insured brings to the
insurer and the experience for comparable risks is used to calculate a premium rate.

Exposure unit / measure


The basic unit used by the insurer to measure the amount of risk insured, usually over a
given period and usually used directly in rating, with premiums expressed as the rate per
exposure unit times the number of units for the risk.

Facultative reinsurance
A reinsurance arrangement covering a single risk as opposed to a treaty reinsurance
arrangement; commonly used for very large risks or portions of risks written by a single
insurer.

Facultative-obligatory reinsurance
A reinsurance facility with an obligation placed on the reinsurer to accept.

Fidelity guarantee insurance


Insurance covering the insured against financial losses caused by dishonest actions of its
employees (fraud or embezzlement).

Financial engineering
Financial engineering contracts can generally be characterised as ones that attempt to
improve a company's balance sheet but with little or no transfer of risk.

Financial risk reinsurance, finite risk insurance or reinsurance


This is a form of reinsurance (or insurance) involving less underwriting risk transfer and
more investment or timing risk transfer from the cedant than is customary in reinsurance.

The Actuarial Education Company © IFE: 2019 Examinations


Page 16 SP7: Glossary

Financial Services Compensation Scheme (FSCS)*


A scheme established by the Financial Services and Markets Act 2000 to compensate
policyholders of insolvent insurance companies in defined circumstances. It is funded by a
levy on insurance companies.

First loss
A form of insurance cover in which it is agreed that the sum insured is less than the full
value of the insured property and average will not be applied.

Fleet
A group of vehicles, ships or aircraft that are insured together under one policy.

Fleet rating
The process of determining premium rates for fleets.

Franchise
A minimum percentage or amount of loss that must be attained before insurers are liable to
meet a claim. Once it is attained the insurers must pay the full amount of the loss. This
feature distinguishes a franchise from a deductible or excess. Note that franchise is also a
term to describe the permission given to syndicates to operate within the Lloyd’s market.

Free reserves
The excess of the value of an insurer’s assets over its technical reserves and current
liabilities. Also known as the solvency margin and sometimes, in the case of a proprietary
insurer, referred to as shareholders’ funds or net asset value.

From the ground up


A statement of an original insurer’s experience of a class of business offered for
reinsurance is said to be from the ground up when it shows the number and distribution by
amount of all claims however small, even though reinsurance is required for large claims
only (above retention levels or other contract lower limits).

Fronting
Fronting occurs when an insurer, acting as a mere conduit, underwrites a risk and cedes all
(or nearly all) of the risk to another insurer which is technically acting as a reinsurer. The
ceding or ‘fronting’ insurer will typically receive a fee for its involvement to cover its
expenses and profit.

In insurance the term ‘fronting’ may also be used to describe the process whereby an
individual effects a policy for him/herself but tries to save money by putting the policy in
someone else’s name.

© IFE: 2019 Examinations The Actuarial Education Company


SP7: Glossary Page 17

Functional costing
A process used within an expense analysis to split the expenses of each line department
between the different classes of business covered by that department. The process usually
relies upon fixing relative unit costs for each of the processes carried out by the department
and counting the number of times that each of the processes is carried out over the period
in question.

Fund (or funded) accounting


Fund accounting is one of two commonly used statutory bases of accounting, the other
being annual accounting. A method of accounting whereby premiums, claims and
associated expenses are related to the underwriting year in which the policies start. The
recognition of any underwriting profit is deferred until a subsequent accounting period but
provision is made for losses as soon as they are foreseen. See related annual basis of
accounting.

Funds at Lloyd’s (FAL)


Each member of Lloyd's is required to provide capital as security to support their total
Lloyd’s underwriting business. This is known as Funds at Lloyd’s. The level of Funds at
Lloyd’s determines the amount of insurance business that a member can underwrite.

Generalised linear model (GLM)


In statistics, the generalised linear model (GLM) is a flexible generalisation of ordinary least
squares regression. The GLM generalises linear regression by allowing the linear model to
be related to the response variable via a link function and by allowing the magnitude of the
variance of each measurement to be a function of its predicted value.

Going-concern basis
The accounting basis normally required for an insurer’s published accounts, based on the
assumption that the insurer will continue to trade as normal for the long term future. See
also wind-up basis.

Grossing-up factor
A factor used to adjust an immature or incomplete figure to an ultimate or complete one.

Hard premium rates


High, profitable premium rates. See related soft premium rates.

Home foreign business*


General insurance business written in the UK relating to risks that originate or are located
outside the UK.

Hours clause
A clause within a catastrophe reinsurance treaty that specifies the limited period during
which claims can be aggregated for the purpose of one claim on the reinsurance contract.
Commonly 24 or 72 hours are used.

The Actuarial Education Company © IFE: 2019 Examinations


Page 18 SP7: Glossary

Hunger for bonus


See bonus hunger.

Inception date
This is the date from which the insurer assumes cover for a risk. This may or may not
coincide with the premium collection date.

Increased limit factors


These are factors which estimate the cost for a new limit as a multiple of the basic (original)
limit.

Incurred but not enough reported (IBNER) reserve


A reserve reflecting expected changes (increases and decreases) in estimates for reported
claims only (that is, excluding any ‘true’ or ‘pure’ IBNR claims). The abbreviation is
sometimes stated as ‘incurred but not enough reserved’. The two terms can be regarded as
identical in meaning.

Incurred but not reported (IBNR) reserve


A reserve to provide for claims in respect of claim events that have occurred before the
accounting date but still to be reported to the insurer by that date. In the case of a
reinsurer, the reserve also needs to provide for claims that, although already known to the
cedant, have not yet been reported to the reinsurer as being liable to involve the reinsurer.

Quite often, especially in reinsurance and in the London Market, IBNR provisions include
any IBNER provisions. Sometimes the provision for claims incurred on or before the
valuation date and reported after the valuation date is referred to as the True IBNR or the
Pure IBNR.

Incurred claims (or claims incurred)


Incurred claims refer to the total amount paid on a cohort of claims up to a specified
valuation date plus the total of all case estimates on these claims as at the valuation date.
In this sense it is used in contrast to paid claims, which refers only to the amounts paid up
to the valuation date. Incurred claims usually include ALAE (allocated loss adjustment
expenses).

The term is also used to refer to the estimate of ultimate claims in annual accounting, which
is defined as the total amount paid in the year plus the total claims reserve at the end of the
year less the total claims reserve at the start of the year.

This term can be used in different senses and it is essential to confirm the intended
meaning in every case.

© IFE: 2019 Examinations The Actuarial Education Company


SP7: Glossary Page 19

Indemnity, principle of
The principle whereby the insured is restored to the same financial position after a loss as
before the loss. This is typical of most types of insurance. This contrasts with the
new-for-old basis of settlement, often used in home contents insurance, under which the
insured is entitled to the full replacement value of the property without any deduction for
depreciation or wear and tear.

Insurance certificate
A certificate provided by an insurer to confirm that the policyholder has insurance cover.

Insurance cycle
The observed tendency of insurance prices and hence profitability to vary over a period of
several years.

Insured
The person, group or property for which an insurance policy is issued.

Integrated Lloyd’s Vehicles (ILVs)


Syndicates on which the entire participation has been bought by insurance groups.

Internal Model Approval Process (IMAP)


Process that insurers and reinsurers have to be approved through, under Solvency II, in
order to be able to use either a partial or full internal model. The approval or rejection is
given by their group supervisor.

Inwards reinsurance
Reinsurance business accepted or written by an insurer or reinsurer, as opposed to
outwards reinsurance which is ceded to a reinsurer.

Knock-for-knock agreement
An agreement between two insurers specifying how claims costs are shared between them
when vehicles insured by each of them are involved in the same accident. It specifies that
each insurer meets the cost of the damage to the vehicle it has insured without any
investigation or allocation of legal liability.

Lapse
When a policyholder, having been invited to renew the policy, does not do so, the policy is
said to lapse.

Lapse rate
Usually defined as the ratio of the number of lapses in a defined period to the
corresponding number of renewal invitations, but could be another ratio associated with
lapses.

The Actuarial Education Company © IFE: 2019 Examinations


Page 20 SP7: Glossary

Latent claims
Strictly, latent claims are those claims that result from perils or causes that the insurer is
unaware of at the time of writing a policy, and for which the potential for claims to be made
many years later has not been appreciated.

In common parlance, latent claims are also those that generally take many years to be
reported.

Lead underwriter
An underwriter who takes the lead in setting premium rates and agreeing policy conditions
under a system of coinsurance (for example, in the Lloyd’s market). A lead underwriter
may, or may not, be the lead claims handler depending on market practice and agreements
for the class of business.

Letter of credit
A financial guarantee issued by a bank that permits the party to which it is issued to draw
funds from the bank in the event of a valid unpaid claim against another party.

Liability insurance
Insurance against the risk of being held legally liable to pay compensation to a third party.

Line
Three different meanings arise  the context usually makes it clear which is intended:

 The ceding office’s retention under a surplus reinsurance treaty.

 The percentage allocated to an insurer under coinsurance arrangements.

 An alternative name for a class of business.

Line slip
A facility under which underwriters delegate authority to accept a predetermined share of
certain coinsured risks on their companies’ behalf. The authority may be exercised by the
leading underwriter on behalf of the following underwriters; or it may extend to the broker
or some other agent authorised to act for all the underwriters.

Link function
A link function provides the relationship between the linear predictor and the mean of the
distribution function.

Link ratios
See development factors.

© IFE: 2019 Examinations The Actuarial Education Company


SP7: Glossary Page 21

Lloyd’s of London (Lloyd’s)


A society, incorporated by the Lloyd’s Act 1871, that provides a market place and regulatory
framework within which individual and corporate members may participate in the
underwriting of insurance risks on their own account.

Lloyd’s broker
An agent approved by the Committee of Lloyd’s to place business with Lloyd’s
underwriters.

Lloyd’s deposit
Wholly owned, non-assigned assets that must be lodged in trust with the Committee of
Lloyd’s before a member can write any business. The amount of the Lloyd’s deposit, when
added to individual Names’ deposits or to incorporated Names’ capital, determines the
maximum limit of premium income that may be written on their behalf. See also Funds at
Lloyd’s.

Lloyd’s managing agent


A company appointed to manage the affairs of an underwriting syndicate, appoint an
underwriter, and provide technical and administrative services.

Also known as an underwriting agent.

Lloyd’s members' agent


A company that looks after the interests of individual Lloyd's Names. The members’ agent
will introduce Names to syndicates, and advise Names on how to spread their capital
between different syndicates. The members’ agent will also be responsible for the regular
audit of a Name’s wealth, and for submitting all financial statements to Lloyd’s.

Lloyd’s special reserve fund


A contingency reserve of limited size that may be built up by individual Lloyd’s Names out
of pre-tax income.

LMX on LMX
Excess of loss reinsurance provided for syndicates or companies operating in the London
Market in respect of LMX business written by them. This is a form of retrocession business.

LMX spiral
The concentration of risk that occurred prior to the mid-1990s when, through the writing of
retrocession business (particularly LMX on LMX business), insurers unwittingly ended up
reinsuring themselves.

London Market
The part of the insurance market in which insurance and reinsurance business is carried
out on a face-to-face basis in the City of London. Sometimes known as the London
Reinsurance Market although not all transacted business is reinsurance.

The Actuarial Education Company © IFE: 2019 Examinations


Page 22 SP7: Glossary

London Market excess of loss (LMX)


Excess of loss reinsurance provided for syndicates or companies operating in the London
Market.

Long-tailed business
Types of insurance in which a substantial number of claims take several years from the date
of exposure and/or occurrence to be notified and/or settled.

Loss
This may signify:

 the financial loss suffered by a policyholder, as distinct from the amount of any
insurance claim that may be payable in respect of that financial loss

 the amount of the insurance claim, as in the expression ‘loss reserve’ which means
the same as the reserve (or provision) for outstanding claims

 the opposite of ‘profit’ in relation to accounts. In this case, the word needs to be
appropriately qualified; for example, underwriting loss or operating loss.

Loss expense reserve


Another expression for any type of claims handling expense provision.

Loss of profits
See business interruption insurance.

Loss portfolio transfer


An arrangement whereby the total liabilities in respect of a specified book of business is
passed in its entirety from one insurance entity to another. Policyholders will be informed
of this ‘novation’.

Loss ratio
Another expression for claim ratio.

Loss reserve
Another name for claims reserve. The expression is also often used in association with the
reserve deposited by a reinsurer with the cedant to cover in part outstanding claims (exact
terms would indicate which party received the investment income on associated assets).

Loss sensitive
See experience rating.

Losses-occurring (losses occurring during or LOD) policy


A reinsurance policy providing cover for losses occurring in the defined period no matter
when they are reported, as opposed to a claims made policy or a risk attaching policy.

© IFE: 2019 Examinations The Actuarial Education Company


SP7: Glossary Page 23

Managing agent
See Lloyd’s managing agent.

Members' agent
See Lloyd’s members’ agent.

Minimum and deposit premiums


See deposit premium.

Model uncertainty
When modelling, the risk that an inappropriate model has been used is known as model
uncertainty. The quantification of model uncertainty is difficult to assess, but by using
alternative models the risk can be minimised and hence the level of uncertainty can be
assessed by comparing the outputs of alternative models.

Moral hazard
Moral hazard refers to the action of a party who behaves differently from the way that they
would behave if they were fully exposed to the circumstances of that action. The party
behaves inappropriately or less carefully than they would otherwise, leaving the
organisation to bear some of the consequences of the action. Moral hazard is related to
information asymmetry, with the party causing the action generally having more information
than the organisation that bears the consequences.

Motor Insurers Bureau (MIB)*


An association of motor insurers in the UK that meets the cost of claims under compulsory
motor insurance where the driver is uninsured or untraced.

Mutual insurer
An insurer owned by policyholders to whom all profits (ultimately) belong.

See related proprietary insurer.

Names (Lloyd’s)
The members of Lloyd’s who accept the liability for (and profits from) the risks underwritten
in their name. Names may be individuals or corporate entities.

Net premium
This can refer to the premium net of the cost of reinsurance, or net of premium tax, or net of
acquisition expenses and/or commission. Premium net of both reinsurance and acquisition
expenses is sometimes referred to as net net premium, this is common in the Lloyd’s
market.

New for old


See replacement; also indemnity, principle of.

The Actuarial Education Company © IFE: 2019 Examinations


Page 24 SP7: Glossary

Nil claim
A claim that results in no payment by the insurer, because, for example:

 the claim is found not to be valid

 the amount of the loss turns out to be no greater than the excess

 the policyholder has reported a claim in order to comply with the conditions of the
policy but has elected to meet the cost in order to preserve any entitlement to
no-claim discount.

Also known as zero claim.

No-claim discount (NCD)


A form of experience rating in which an individual policyholder may be granted a discount
from the relevant base premium depending on his or her claims experience. See related
protected NCD.

Non-proportional reinsurance
Reinsurance arrangements where the claims are not shared proportionately between the
cedant and reinsurer, for example an excess of loss contract.

Non-technical account
The non-technical account is a feature of accounts of insurance companies in the EU. It is
an account made up from the balance on the technical account plus the balance of the
investment income and gains not included in the technical account, plus profits on any
other activities less tax, dividends and any other charges.

Novation*
The transfer of the rights and obligations under a contract from one party to another.

Office premium
This is the total premium charged for the period of cover. This premium will contain the risk
premium, commission, an allowance to cover all other types of expenses, an allowance for
any premium tax and a profit loading.

Ogden tables
These are a set of tables used to help in the calculation of special damages and the present
value of loss of earnings or annual expenses in personal injury and fatal accident cases.
The tables provide multipliers which take account of life expectancies and a range of
discount rates and are prepared by the Government Actuaries’ Department. The discount
rate is set by the Government’s Lord Chancellor.

One-year (accident-year) accounting


A basis of accounting that presents, at the end of each year of account, the estimated
technical account for business exposed during the year. See also annual basis of
accounting.

© IFE: 2019 Examinations The Actuarial Education Company


SP7: Glossary Page 25

Open year
Under fund accounting an open year is one that has not yet reached the stipulated period
for closure. In the Lloyd’s market, an open year is one that has not yet been closed by
RITC. See related closed year.

Operating ratio
See combined ratio.

Original gross premium income (OGPI)


The gross premium income received by an insurer in relation to business that is covered by
a non-proportional reinsurance treaty. The reinsurance premium is calculated as a
percentage of this OGPI. Similar abbreviations, such as OGWP, OGEP, GWPI and GEPI are
also used (see the list of abbreviations).

Abbreviations are provided at the end of this Glossary

Outstanding claims reserve (OCR) or outstanding claims provision


The reserve set up in respect of the liability for all outstanding claims, whether reported or
not, including reserves for future payments on claims that are currently regarded as settled
but may be reopened.

or

The reserve set up in respect of the liability for all reported outstanding claims, including
reserves for future payments on claims that are currently regarded as settled but may be
reopened.

Outwards reinsurance
Reinsurance ceded by an insurer or reinsurer, as opposed to inwards reinsurance, which is
reinsurance accepted.

Overriding commission
Additional commission paid by a reinsurer to an insurer ceding proportional business, as a
contribution towards expenses and profit. The term is often used on primary business
written through agents or brokers and refers to any addition to basic commission rates
either for volume or for profitable business.

Parameter uncertainty
When a model is fitted based upon historic data, certain parameters are selected, for
example, development factors and associated tail distributions or average cost
assumptions. The goodness of fit represented by these parameters can be tested to
identify this element of uncertainty.

The Actuarial Education Company © IFE: 2019 Examinations


Page 26 SP7: Glossary

Partial payment
Partial claim settlement paid on account, before a claim is finalised or closed.

Any claim for less than the full sum insured.

Peril
A type of event that may cause a loss that may or may not be covered by an insurance
policy. An insured peril is one for which insurance cover is provided.

Period of unexpired risk


For a policy in force on an accounting date, the period from the accounting date to the
expiry date.

Persistency
A measure of the probability that a policy will remain in force at renewal, rather than lapse.

Personal lines
Types of insurance products offered to individuals, rather than to groups or business
entities. Products include private motor, domestic household, private medical, personal
accident, pet and travel insurance.

Points rating system


A system for calculating the office premium by relating it to points associated with each cell
within a rating factor; the higher the risk associated with the cell, the higher the points and
the higher the premium. For example, for a motor insurance policy a driver aged 20 would
be associated with many more points and, all other things being equal, a higher premium,
than a driver aged 40.

Pooling
Arrangements where parties agree to share premiums and losses for specific types of class
or cover in agreed proportions. To some extent all insurance is pooling but specific pooling
arrangements often apply particularly where the risks have very large unit size (for example,
atomic energy risks) or via mutual associations, such as P&I clubs, catering for an industry.

Portfolio claims*
Used in proportional and other forms of reinsurance. The outstanding claims that, together
with the portfolio premiums, make up the reinsurance premium required for a portfolio
transfer; usually used to transfer obligations from one year of account to the next and
hence enable a result for the year to be struck. Can also apply to the body of claims
transferred in a portfolio transfer.

Portfolio premiums*
The premiums that together with the portfolio claims make up the reinsurance premium
required for a portfolio transfer.

© IFE: 2019 Examinations The Actuarial Education Company


SP7: Glossary Page 27

Portfolio transfer*
The reinsurance of an entire portfolio at a premium relating to the estimated outstanding
claims (including IBNR) and unexpired risk under that portfolio. Usually used when an
insurer has decided to discontinue writing a particular class, or by a reinsurer wanting to
close a treaty year and pass on the liability to the following year for administrative reasons.

Possible maximum loss (PML)


See probable (possible) maximum loss (PML).

Premium income limit


The amount of premium that a Lloyd’s Name may write in a given year, determined by the
size of the Name’s wealth, the Name’s Lloyd’s deposit and whether or not incorporated.

Premiums trust fund (PTF)


A fund into which all premiums for a Lloyd’s syndicate in a given underwriting year are paid.
No moneys may be released from the fund other than any profit on closure and on-going
claims and expenses.

Primary insurer
An insurer providing cover directly to the insured policyholder, as distinct from a reinsurer.

Probable (possible) maximum loss (PML)


The term ‘probable maximum loss’ represents an attempt to quantify exposure, and is used
in rating or to judge requirements for outwards reinsurance. It may be used as another term
for estimated maximum loss, depending on the class of business. The term ‘possible
maximum loss’ implies the consideration of more remote scenarios than those for probable
or estimated maximum loss and therefore carries a higher value. The fact that the same
abbreviation, PML, may be used for both is a source of possible (and, indeed, probable)
confusion.

Process uncertainty
Process uncertainty is the risk inherent in writing business and settling claims in general
insurance. The modelling of the number and amount of claims will vary from the true value
owing to random variation.

Product costing
Product costing is the calculation of the theoretical office premium to be charged for a
particular class of business.

Product pricing
Product pricing is the determination of the actual office premium. This will take account of
current market conditions.

The Actuarial Education Company © IFE: 2019 Examinations


Page 28 SP7: Glossary

Profit commission
Commission paid by a reinsurer to a cedant under a proportional reinsurance treaty that is
dependent upon the profitability of the total business ceded during each accounting period.

Also, commission paid by an insurer to a broker or insured that is dependent upon the
profitability of the business written.

Profit testing
A term used for estimating the economic value of contracts using net present value
techniques; that is, proposed premium rates are tested by projecting possible levels of
future business, claims, expenses, investment experience and profit. The process may be
extended to include all business and so form a model office akin to those used in life
companies.

Proportional reinsurance
A reinsurance arrangement where the reinsurer and cedant share the claims proportionally.
Usually, premiums follow the same proportions but commission rates may differ. Two
types commonly arise: quota share and surplus.

Proprietary insurer
An insurance company owned by shareholders; that is, not a mutual insurer. See related
mutual insurer.

Protected NCD
A modification to an NCD system whereby a policyholder who has attained a high level of
NCD may elect to pay an extra premium in order to be able to make claims without losing
future entitlement to discount. There may be a specified limit to the number of claims that
can be made without affecting the discount, or the insurer may simply reserve the right to
withdraw the policyholder’s option to continue on protected NCD.

Protection and Indemnity (P & I) Clubs


Mutual insurers of marine risks.

Quota share (QS) reinsurance


A form of proportional reinsurance where the proportions used in apportioning claims and
premiums between the insurer and reinsurer are constant for all risks covered by the treaty.

Rate on line
For non-proportional reinsurance, the total premium charged (ignoring reinstatement
premiums) for the reinsurance divided by the width of the layer covered.

Rating
The process of arriving at a suitable premium for an insurance risk. The term is sometimes
synonymous with underwriting, though rating is strictly just one part of the underwriting
process.

© IFE: 2019 Examinations The Actuarial Education Company


SP7: Glossary Page 29

Rating basis
The collection of assumptions used to associate the risk premium with the characteristics
of the risk being insured.

Rating factor
A factor used to determine the premium rate for a policy, which is measurable in an
objective way and relates to the intensity of the risk. It must, therefore, be a risk factor or a
proxy for a risk factor or risk factors.

Realistic Disaster Scenarios (RDS)


Lloyd’s maintains a set of mandatory Realistic Disaster Scenarios to stress test both
individual syndicates and the market as a whole. The event scenarios are regularly
reviewed to ensure they represent material catastrophe risks.

There are three sets of Realistic Disaster Scenarios. First are the compulsory scenarios, for
which all syndicates report estimated losses to Lloyd’s; these represent events to which
most of the market would potentially be exposed, and for which Lloyd's monitors the total of
all syndicate losses. There is a set of more specialist scenarios, which need only be
reported if estimated losses exceed a threshold. Finally, there are two events which
syndicates must define for themselves as representing material potential losses not
captured in other scenarios.

Reciprocity
An arrangement between two insurers who agree to reinsure risks with each other.
Commonly used with quota share reinsurance to diversify the insurers’ overall portfolios.

Recoveries
Amounts received by insurers to offset directly part of the cost of a claim. Recoveries may
be made from several different sources, for example, reinsurers, other insurers, salvage and
liable third parties.

Reinstatement
The restoration of full cover following a claim.

Normally, the number of reinstatements, and the terms upon which they are made, will be
agreed at the outset. Once agreed, they are automatic and obligatory on both parties.

Reinsurance
An arrangement whereby one party (the reinsurer), in consideration for a premium, agrees
to indemnify another party (the cedant) against part or all of the liability assumed by the
cedant under one or more insurance policies, or under one or more reinsurance contracts.

The Actuarial Education Company © IFE: 2019 Examinations


Page 30 SP7: Glossary

Reinsurance to close (RITC)


An agreement under the Lloyd’s system of three-year accounting. Underwriting members
(the reinsured members) for one year of account (the closing year) of a syndicate agree with
another party (the reinsuring party) that the reinsuring party will assume responsibility for
handling and paying all known and unknown liabilities of the reinsured members arising out
of insurance business underwritten by the syndicate and allocated to the closing year.

The reinsuring party will usually be the subsequent open year of the same syndicate but
could also be a later open year, an open year of another syndicate or a reinsurer outside
Lloyd’s.

The term is also sometimes used to refer to the premium paid to the reinsuring party by the
reinsured members.

Reinsurer
An insurer providing reinsurance cover. Some reinsurers do not write any direct or primary
insurance business.

Reopened claim
A claim formerly deemed settled, but subsequently reopened because further payments
may be required.

Replacement
A basis of cover under which the insurer pays the cost of replacing the insured item with a
similar but new item. Also referred to as ‘replacement as new’ or ‘new for old’ and contrasts
with ‘the principle of indemnity’.

Reporting year
A reporting year grouping of claims will combine all the claims that are reported within a
given calendar year, irrespective of the date on which the relevant policy commenced,
irrespective of when the claims are actually paid and irrespective of the year in which the
incident actually arose. See related underwriting year, accident year.

Retention
The amount (or proportion) of risk retained by the cedant under a reinsurance arrangement
or the insured under an insurance arrangement.

Although, in the case of non-proportional reinsurance covering a band from R (retention) to


U (upper limit), the cedant may be said to retain not only the risk from 0 to R but also the
risk above U, it is R that would be termed the retention.

Retroactive date
Used for claims-made cover. It is the date after which claims must have occurred in order
to be covered.

© IFE: 2019 Examinations The Actuarial Education Company


SP7: Glossary Page 31

Retrocession
Reinsurance purchased by a reinsurer in relation to its inwards reinsurance liabilities (that
is, reinsurance of reinsurance).

Retrocessionaire
A reinsurer that accepts reinsurance from another reinsurer.

Return commission
Commission paid by a reinsurer to an insurer ceding proportional business, as a
contribution towards expenses and profit. Also called overriding commission.

Return period
A return period, also known as a recurrence interval, is an estimate of the interval of time
between events like an earthquake, flood or hurricane of a certain intensity or size. It is a
statistical measurement denoting the average recurrence interval over an extended period
of time, and is usually required for risk analysis (ie whether a project should be allowed to
go forward in a zone of a certain risk) and also to dimension structures so that they are
capable of withstanding an event of a certain return period (with its associated intensity).

Risk-attaching basis
A basis under which reinsurance is provided for claims arising from policies commencing
during the period to which the reinsurance relates.

Risk-based capital (RBC)


The assessment of the capital requirement for a general insurer by considering the risk
profile of the insurance business written and of any other operations. In the US, the
required minimum margins of solvency are determined after considering RBC requirements.

Risk excess of loss reinsurance


Excess of loss reinsurance that relates to individual losses affecting only one insured risk
at any one time.

Risk factor
A factor that is expected, possibly with the support of statistical evidence, to have an
influence on the intensity of risk in an insurance cover. See related rating factor.

Risk group
The rating cell or risk segment into which particular policies are categorised, within a type
of insurance cover. The objective is to achieve a group of policies or risks that have
homogeneous characteristics.

The Actuarial Education Company © IFE: 2019 Examinations


Page 32 SP7: Glossary

Risk premium
The amount of premium required to cover claims expected for a risk; that is, average claim
amount times average claim frequency. It may alternatively be expressed as a rate per unit
of exposure.

Road Traffic Act*


The UK legislation that, among other things, requires anyone using a motor vehicle on the
road to have insurance to cover their legal liabilities to third parties (including passengers)
in respect of personal injury and property damage.

Run-off basis
A valuation basis that assumes an insurer will cease to write new business, and continue in
operation purely to pay claims for previously written policies. Typically expenses and
reinsurance arrangements change after an insurer ceases to write new business.

Run-off triangle
See claims run-off analysis. The development or run-off triangle may be of paid or incurred
claims by amount or number, or of premiums.

Salvage
Amounts recovered by insurers from the sale of insured items that had become the property
of the insurer by virtue of the settling of a claim.

Self-insurance
The retention of risk by an individual or organisation, as distinct from obtaining insurance
cover.

Short-tailed business
Types of insurance in which most claims are usually notified and/or settled in a short period
from the date of exposure and/or occurrence.

Signing down
The process of reducing the proportion of risk that each coinsurer has accepted for a given
risk where the slip has been more than 100% subscribed.

Slip system
The face-to-face system used within the London Market to coinsure risks. Proposed risks
are described by a broker on a standard form (slip); terms and the premium rate are added
after negotiation with a lead underwriter (who also signs for a certain proportion of the risk),
before the slip is circulated by the broker amongst other underwriters who sign the slip to
confirm the proportion of risk that they will accept.

© IFE: 2019 Examinations The Actuarial Education Company


SP7: Glossary Page 33

Soft premium rates


Premium rates with significantly reduced margins due to the competitive state of the
market. See related hard premium rates.

Solvency margin
Another term for free reserves.

Solvency ratio
The free reserves divided by the net (of reinsurance) written premiums.

Stability clause
A clause that may be included in a non-proportional reinsurance treaty, providing for the
indexation of monetary limits (that is, the excess point and/or the upper limit) in line with a
specified index of inflation.

Stop loss reinsurance


An aggregate excess of loss reinsurance that provides protection based on the total claims,
from all perils, arising in a class or classes over a period. The excess point and the upper
limit are often expressed as a percentage of the cedant’s premium income rather than in
monetary terms; for example, cover might be for a claims ratio in excess of 110% up to a
limit of 140%.

Subrogation
The substitution of one party for another as creditor, with a transfer of rights and
responsibilities. It applies within insurance when an insurer accepts a claim by an insured,
thus assuming the responsibility for any liabilities or recoveries relating to the claim. For
example, the insurer will be responsible for defending legal disputes and will be entitled to
the proceeds from the sale of damaged or recovered property.

Sunset clause
Clause defining the time limit within which a claim must be notified, if it is to be valid.

Suretyship
Insurance to provide a guarantee of performance or for the financial commitments of the
insured. In the UK this is known as financial guarantee insurance.

Surplus lines insurance


Specialised property or liability coverage in the US provided by an unlicensed insurer in
instances where it is unavailable from insurers licensed in the state in question. See also
surplus reinsurance.

The Actuarial Education Company © IFE: 2019 Examinations


Page 34 SP7: Glossary

Surplus reinsurance
A form of proportional reinsurance where the proportions are determined by the cedant for
each individual risk covered by the treaty, subject to limits defined in the treaty.

Swing rated
See experience rating.

Syndicate (Lloyd’s)
A group of Lloyd’s Names who collectively coinsure risks. The syndicates often specialise
in particular types of insurance.

Tail Value-at-Risk (Tail VaR)


Tail VaR is the expected value of the loss in those cases where it exceeds the predefined
confidence level. It is sometimes also called conditional tail expectation, expected shortfall
or expected tail loss. Thus the Tail VaR is equal to the average loss a company will suffer in
case of (extreme) situations where losses exceed a predefined confidence level (eg 99.5%).

Technical Account – General Business*


The Technical Account – General Business is part of the Profit and Loss Account for
insurance companies within the EU. It is made up of earned premiums less incurred claims
(both adjusted for reinsurance as appropriate), less expenses (with an allowance for
deferred acquisition costs as appropriate), plus any part of the investment income that may
be allocated to the technical account.

Technical reserves (provisions) (TPs)


The accounting entries in the balance sheet that represent the insurer’s liabilities from the
business that has been written.

Under Solvency II, technical provisions comprise claims provision + premium provision +
risk margin, where:

 The claims provision is the discounted best estimate of all future cash flows (claim
payments, expenses and future premiums) relating to claim events before the
valuation date.

 The premium provision is the discounted best estimate of all future cash flows
(claim payments, expenses and future premiums due) relating to future exposure
arising from policies that the (re) insurer is obligated to at the valuation date.

 The risk margin is intended to be the balance that another (re)insurer taking on the
liabilities at the valuation date would require over and above the best estimate. It is
calculated using a cost of capital approach.

Three-year accounting
The usual form of funded accounting, in which the underwriting profits are first recognised
at the end of the third accounting year from the start of the underwriting year.

© IFE: 2019 Examinations The Actuarial Education Company


SP7: Glossary Page 35

Time and distance reinsurance


A type of financial reinsurance that had widespread use in the London Market and Lloyd’s,
whereby an insurer pays a single premium in return for a fixed schedule of future payments
matched to the estimated dates and amounts of the insurer’s claim outgo. The purpose of
such contracts was to achieve the effect of discounting in arriving at the reserves for
outstanding claims. Since Lloyd’s changed its rules so that the credit allowed for time and
distance policies in a syndicate’s accounts was limited to the present value, such policies
have become less popular.

Treaty reinsurance
Reinsurance that a reinsurer is obliged to accept, subject to conditions set out in a treaty.

Uberrima fides
Latin for ‘utmost good faith’. This honesty principle is assumed to be observed by the
parties to an insurance, or reinsurance, contract. An alternative form is uberrimae fidei: ‘of
the utmost good faith’.

UK Guarantee Fund
The UK guarantee fund is a fund operated by the Motor Insurers Bureau (MIB) to
compensate victims of negligent uninsured or untraced drivers who have no other source of
compensation.

Unallocated loss adjustment expenses (ULAE)


See claims handling expenses.

Underinsurance
When the sum insured is less than that required under the terms of the contract. Depending
on the policy conditions, where underinsurance is proved to exist, insurers may be able to
claim that the policy is null and void. Alternatively, average may be applied to claim
amounts.

Underwriter
An individual who assesses risks and decides the premiums, and the terms and conditions
under which they can be accepted by the insurer.

Underwriting
The process of consideration of an insurance risk. This includes assessing whether the
risk is acceptable and, if so, the appropriate premium together with the terms and
conditions of the cover. It may also include assessing the risk in the context of the other
risks in the portfolio. The more individual the risk (for example, most commercial lines), the
more detailed the consideration.

The term is also used to denote the acceptance of reinsurance and, by extension, the
transacting of insurance business.

The Actuarial Education Company © IFE: 2019 Examinations


Page 36 SP7: Glossary

Underwriting agent
An organisation at Lloyd’s providing management services for syndicates and/or advice for
Names. See Lloyd’s managing agent.

Underwriting factor
Any factor that is used to determine the premium, or terms and conditions for a policy. It
may be a rating factor or some other risk factor that is accounted for in a subjective manner
by the underwriter.

Underwriting ratio
See combined ratio.

Underwriting year
An underwriting year grouping of claims will combine all the claims relating to loss events
that can be attributed to all policies that commenced cover within a given year, irrespective
of when they are actually reported or paid and irrespective of the year in which the incident
actually arose. See related reporting year, accident year.

Unearned premium reserve (UPR) or provision for unearned premiums


The amount set aside from premiums written before the accounting date to cover risks
incurred after that date.

Unearned premiums
The portion of premium written in an accounting period that is deemed to relate to cover in
one or more subsequent accounting periods. It can be calculated in at least two ways:

 Net of deferred acquisition costs (DAC); that is, by deducting acquisition expenses
before proportioning the written premium.

 Gross of DAC; that is, by proportioning the full written premium without any
deduction for DAC.

The first approach is consistent with a going-concern basis, whilst the second is consistent
with a break-up basis. However, the second approach can also be used for a going-concern
basis by including DAC as an asset in the balance sheet.

A typical balance sheet includes values gross and net of reinsurance also.

Unexpired risks reserve (URR) or provision for unexpired risks


This term is often used in two ways:
(1) The reserve required to cover the claims and expenses that are expected to emerge
from an unexpired period of cover.
(2) The reserve required to cover the excess of (1) over the UPR. This is sometimes
known as the additional unexpired risk reserve (AURR).

© IFE: 2019 Examinations The Actuarial Education Company


SP7: Glossary Page 37

Value at Risk (VaR)


In financial mathematics and financial risk management, Value at Risk is a widely used
measure of the risk of loss. For a given probability and time horizon, VaR is defined as a
threshold value such that the probability that the loss on the portfolio over the given time
horizon exceeds this value is the given probability level.

Verticalisation
When different coinsurers have different terms (premium rates).

Wind-up basis
See break-up basis.

Working layer
A layer of excess of loss reinsurance at a level where there is likely to be a fairly regular
flow of claims.

Written premiums
The amount of premium, either gross or net of reinsurance, for which cover commenced in
an accounting period.

Zero claim
Another term for nil claim.

The Actuarial Education Company © IFE: 2019 Examinations


Page 38 SP7: Glossary

APPENDIX – List of abbreviations


ABI Association of British Insurers

ACPC Average cost per claim

AEP Aggregate exceedance probability

AIC Akaike information criteria

ALAE Allocated loss adjustment expenses

ALM Asset liability modelling or management

ALM Association of Lloyd’s members

ANOVA Analysis of variance

AP Adjustment premium

APH Asbestos, pollution, health hazard

APS Actuarial Profession Standards

ART Alternative risk transfer

ASTIN Actuarial STudies In Non-life insurance

ATAFs Age to age factors

ATUFs Age to ultimate factors

AURR Additional unexpired risk reserve

AV Annual venture (Lloyd’s)

BAU Business as usual

BCL Basic chain ladder

BF Bornhuetter-Ferguson

BHF Bornhuetter-Ferguson

BI Bodily injury

BI Business interruption

CAR Contractors’ all risks (insurance)

CAS Casualty Actuarial Society

CDF Cumulative density function

CGL Comprehensive general liability (policies)

CHE Claims handling expenses

CIL Coming into line (Lloyd’s)

© IFE: 2019 Examinations The Actuarial Education Company


SP7: Glossary Page 39

CL Chain ladder

CRTF Credit for reinsurance trust fund (Lloyd’s)

CUE Claims and underwriting exchange (in UK)

D&O Directors and officers (insurance)

DAC Deferred acquisition costs

DFA Dynamic financial analysis

E&O Errors and omissions (insurance)

E&S Excess and surplus (lines insurance)

ECA Economic capital assessment (Lloyd’s)

EIOPA European Insurance and Occupational Pensions Authority

EL Employers’ liability

ELC Expected loss cost

EML Estimated (or expected) maximum loss

ENIDs Events not in data

EPD Expected policyholder default

ERM Enterprise risk management

ESG Economic scenario generator

FAL Funds at Lloyd’s

FATF Financial Action Task Force

FCA Financial Conduct Authority

FGU From the ground up

FPD Franchise performance directorate (Lloyd’s)

FRC Financial Reporting Council

FSCS Financial Services Compensation Scheme

FSMA The Financial Services and Markets Act

GAAP Generally accepted accounting principles

GCR Group Capital Resources

GDP Gross domestic product

GEPI Gross earned premium income

GIRO General Insurance Research Organisation

GIS Geographical Information Systems

The Actuarial Education Company © IFE: 2019 Examinations


Page 40 SP7: Glossary

GLM Generalised linear model

GWPI Gross written premium income

HSE Healthy and Safety Executive

IAA International Actuarial Association

IAIS International Association of Insurance Supervisors

IASB International Accounting Standards Board

IBNER Incurred but not enough reported

IBNR Incurred but not reported

IFRS International Financial Reporting Standards

IID International Insurers Division

ILFs Increased limit factors

ILS Insurance-linked securities

ILVs Integrated Lloyd’s Vehicles

ILWs Industry loss warranties

IMAP Internal model approval process

IMF International Monetary Fund

IPO Initial public offering

IUA Insurance Underwriting Association

LDF Loss development factor

LDP Low discrepancy point

LMA Lloyd’s Market Association

LMX London Market excess of loss

LoC Letter of credit

LOD Losses occurring during

LPT Loss portfolio transfer

LTA Long term agreement

M&A Mergers and acquisitions

MAT Marine, aviation and transport

MCR Minimum capital requirement

MGA Managing general agent

MGF Minimum guarantee fund

© IFE: 2019 Examinations The Actuarial Education Company


SP7: Glossary Page 41

MGF Moment generating function

MIB Motor Insurers Bureau

MIG Mortgage indemnity guarantee (insurance)

MLE Maximum likelihood estimate or estimation

MoM Method of moments

MPL Maximum probable loss

MSARC Market supervision and review committee (Lloyd’s)

NAIC National Association of Insurance Commissioners

NCD No-claim discount

NCF New Central Fund (at Lloyd’s)

NYID New York Insurance Department

OCR Outstanding claims reserve

ODP Overdispersed Poisson

OEP Occurrence exceedance probability

OGEP Original gross earned premium

OGPI Original gross premium income

OGWP Original gross written premium

ONS Office for National Statistics (UK)

OPIL Overall premium income limit (Lloyd’s)

ORSA Own Risk and Solvency Assessment

P&I Protection & Indemnity

PCS Professional Conduct Standards

PD Property damage

PML Probable (possible) maximum loss

PRA Prudential Regulation Authority

PTF Premiums trust fund

QS Quota share

RBC Risk-based capital

RDS Realistic disaster scenarios

RITC Reinsurance to close

RMS Required margin of solvency

The Actuarial Education Company © IFE: 2019 Examinations


Page 42 SP7: Glossary

RMS Risk Management Solutions

ROC Reserving Oversight Committee

ROE Return on equity

RSR Regular Supervisory Reporting

SAO Statement of actuarial opinion

SCR Solvency Capital Requirement

SFCR Solvency and Financial Condition Report

SLTF Surplus lines trust fund (Lloyd’s)

SME Small to medium (sized) enterprises

SMR Solvency Margin Requirement

SORP Statement of recommended practice

TAS Technical Actuarial Standard

TPs Technical provisions

TPL Third party liability

TVaR Tail value at risk or Tail VaR

ULAE Unallocated loss adjustment expenses

ULR Ultimate loss ratio

UPR Unearned premium reserve

URR Unexpired risks reserve

VaR Value at risk

XL Excess of loss

XoL Excess of loss

YOA Year of account

© IFE: 2019 Examinations The Actuarial Education Company


Subject SP7: Assignment X1
2019 Examinations

Time allowed: 2¾ hours

Instructions to the candidate


1. Please:

– attempt all of the questions, as far as possible under exam conditions

– begin your answer to each question on a new page

– leave at least 2cm margin on all borders

– write in black ink using a medium-sized nib because we will be unable to mark
illegible scripts

– note that assignment marking is not included in the price of the course materials.
Please purchase Series Marking or a Marking Voucher before submitting your script.

– note that we only accept the current version of assignments for marking, ie you can
only submit this assignment in the sessions leading to the 2019 exams.

2. Please do not:

– use headed paper

– use highlighting in your script.

At the end of the assignment


If your script is being marked by ActEd, please follow
the instructions on the reverse of this page.

In addition to this paper, you should have available actuarial tables and an
electronic calculator.

The Actuarial Education Company © IFE: 2019 Examinations


Submission for marking

You should aim to submit this script for marking by the recommended submission date. The
recommended and deadline dates for submission of this assignment are listed on the summary page
at the back of this pack and on our website at www.ActEd.co.uk.

Scripts received after the deadline date will not be marked, unless you are using a Marking Voucher.
It is your responsibility to ensure that scripts reach ActEd in good time. If you are using Marking
Vouchers, then please make sure that your script reaches us by the Marking Voucher deadline date
to give us enough time to mark and return the script before the exam.

When submitting your script, please:

 complete the cover sheet, including the checklist


 scan your script, cover sheet (and Marking Voucher if applicable) and save as a pdf
document, then email it to: ActEdMarking@bpp.com
 do not submit a photograph of your script
 do not include the question paper in the scan.
In addition, please note the following:

 Please title the email to ensure that the subject and assignment are clear eg ‘SP7 Assignment
X1 No. 12345’, inserting your ActEd Student Number for 12345.
 The assignment should be scanned the right way up (so that it can be read normally without
rotation) and as a single document. We cannot accept individual files for each page.
 Please set the resolution so that the script is legible and the resulting PDF is less than 4 MB
in size.
 Do not protect the PDF in any way (otherwise the marker cannot return the script to ActEd,
which causes delays).
 Please include the ‘feedback from marker’ sheet when scanning.
 Before emailing to ActEd, please check that your scanned assignment includes all pages and
conforms to the above.

© IFE: 2019 Examinations The Actuarial Education Company


Subject SP7: Assignment X1
2019 Examinations
Please complete the following information:

Name:
Number of following pages: _______

Please put a tick in this box if you have solutions


and a cross if you do not:

Please tick here if you are allowed extra time or


ActEd Student Number (see Note below): other special conditions in the
profession’s exams (if you wish to
share this information):

Time to do assignment
(see Note below): _____ hrs _____ mins
Note: Your ActEd Student Number is printed on all
personal correspondence from ActEd. Quoting it will help Under exam conditions
us to process your scripts quickly. If you do not know (delete as applicable): yes / nearly / no
your ActEd Student Number, please email us at
Note: If you take more than 2¾ hours, you should
ActEd@bpp.com.
indicate how much you completed within this
Your ActEd Student Number is not the same as your exam time so that the marker can provide useful
IFoA Actuarial Reference Number or ARN. feedback on your progress.

Score and grade for this assignment (to be completed by marker):

Q1 Q2 Q3 Q4 Q5 Q6 Q7 Q8 Q9 Total

=_____%
10 12 11 9 7 8 7 10 6 80

Grade: A B C D E Marker’s initials: ________

Please tick the following checklist so that your script can be marked quickly. Have you:

[ ] Checked that you are using the latest version of the assignments, ie 2019 for the sessions leading
to the 2019 exams?
[ ] Written your full name in the box above?
[ ] Completed your ActEd Student Number in the box above?
[ ] Recorded your attempt conditions?
[ ] Numbered all pages of your script (excluding this cover sheet)?
[ ] Written the total number of pages (excluding the cover sheet) in the space above?
[ ] Included your Marking Voucher or ordered Series X Marking?

Please follow the instructions on the previous page when submitting your script for marking.

The Actuarial Education Company © IFE: 2019 Examinations


Feedback from marker

Notes on marker’s section

The main objective of marking is to provide specific advice on how to improve your chances of
success in the exam. The most useful aspect of the marking is the comments the marker makes
throughout the script, however you will also be given a percentage score and the band into which
that score falls. Each assignment tests only part of the course and hence does not give a complete
indication of your likely overall success in the exam. However it provides a good indicator of your
understanding of the material tested and the progress you are making with your studies:

A = Excellent progress B = Good progress C = Average progress


D = Below average progress E = Well below average progress

Please note that you can provide feedback on the marking of this assignment at:

www.ActEd.co.uk/marking

© IFE: 2019 Examinations The Actuarial Education Company


SP7: Assignment X1 Questions Page 1

X1.1 (i) State the cover provided under product liability insurance. [1]

(ii) Define the terms losses-occurring basis, and claims-made basis, and explain which is more
likely to be used for product liability insurance. [3]

(iii) State the common exposure measure, perils, risk factors and rating factors for product
liability insurance. [6]
[Total 10]

X1.2 A general insurance company that provides a wide range of commercial lines insurance business
in a developed country specialises in providing insurance cover for restaurants and cafés.

The government of the country is about to impose a ban on smoking in public places. This means
that smoking in restaurants and cafés will no longer be permitted.

Discuss the possible implications of this ban to the insurance company, and state the factors that
will influence the expected size of its impact. [12]

X1.3 A statutory body is considering introducing legislation that will impose minimum and maximum
premiums for companies writing general insurance business. Discuss the advantages and
disadvantages of this legislation. [11]

X1.4 A general insurance company is considering purchasing an adverse development cover.

(i) Describe what is meant by an adverse development cover. [2]

(ii) Describe the risks the insurer will transfer in purchasing an adverse development
cover. [2]

(iii) Suggest what risks are retained and what additional risks the insurer would accept in
purchasing an adverse development cover. [3]

(iv) Explain how these additional risks can be overcome. [2]


[Total 9]

X1.5 Your friend has recently established a pizza delivery company. List, with reasons, the types of
general insurance that they might need. [7]

X1.6 (i) List the ways in which personal lines and London Market business are sold. [3]

(ii) Describe the slip system used in the London Market. [5]
[Total 8]

The Actuarial Education Company © IFE: 2019 Examinations


Page 2 SP7: Assignment X1 Questions

X1.7 A general insurance company operates with a normal maximum retention of $1,000,000. It has
four surplus treaties, which have 3, 5, 10 and 20 lines respectively. It retains $1,000,000 on any
risk before any cession to the surplus treaties.

The following five risks were written by the insurance company, and during the coverage period
suffered the following claims:

Risk EML ($m) Claim ($m)


1 2 5
2 9 5
3 15 10
4 30 50
5 50 50

Calculate the claim to each of the four surplus treaties and the claim retained by the insurance
company, stating any assumptions you make. [7]

X1.8 (i) State the objectives of regulating the insurance industry. [3]

(ii) List the key activities of the IAIS. [1]

(iii) List the capital requirements that an insurance supervisor might impose on an insurer. [3]

(iv) Describe the disadvantages to insurers of complying with the IAIS Core Principles (ICPs).[3]
[Total 10]

X1.9 List the factors relating to the external environment that may affect claim frequencies or amounts
for motor insurance policies. [6]

END OF PAPER

© IFE: 2019 Examinations The Actuarial Education Company


Subject SP7: Assignment X2
2019 Examinations

Time allowed: 2¾ hours

Instructions to the candidate


1. Please:

– attempt all of the questions, as far as possible under exam conditions

– begin your answer to each question on a new page

– leave at least 2cm margin on all borders

– write in black ink using a medium-sized nib because we will be unable to mark
illegible scripts

– note that assignment marking is not included in the price of the course materials.
Please purchase Series Marking or a Marking Voucher before submitting your script.

– note that we only accept the current version of assignments for marking, ie you can
only submit this assignment in the sessions leading to the 2019 exams.

2. Please do not:

– use headed paper

– use highlighting in your script.

At the end of the assignment


If your script is being marked by ActEd, please follow
the instructions on the reverse of this page.

In addition to this paper, you should have available actuarial tables and an
electronic calculator.

The Actuarial Education Company © IFE: 2019 Examinations


Submission for marking

You should aim to submit this script for marking by the recommended submission date. The
recommended and deadline dates for submission of this assignment are listed on the summary page
at the back of this pack and on our website at www.ActEd.co.uk.

Scripts received after the deadline date will not be marked, unless you are using a Marking Voucher.
It is your responsibility to ensure that scripts reach ActEd in good time. If you are using Marking
Vouchers, then please make sure that your script reaches us by the Marking Voucher deadline date
to give us enough time to mark and return the script before the exam.

When submitting your script, please:

 complete the cover sheet, including the checklist


 scan your script, cover sheet (and Marking Voucher if applicable) and save as a pdf
document, then email it to: ActEdMarking@bpp.com
 do not submit a photograph of your script
 do not include the question paper in the scan.
In addition, please note the following:

 Please title the email to ensure that the subject and assignment are clear eg ‘SP7 Assignment
X2 No. 12345’, inserting your ActEd Student Number for 12345.
 The assignment should be scanned the right way up (so that it can be read normally without
rotation) and as a single document. We cannot accept individual files for each page.
 Please set the resolution so that the script is legible and the resulting PDF is less than 4 MB
in size.
 Do not protect the PDF in any way (otherwise the marker cannot return the script to ActEd,
which causes delays).
 Please include the ‘feedback from marker’ sheet when scanning.
 Before emailing to ActEd, please check that your scanned assignment includes all pages and
conforms to the above.

© IFE: 2019 Examinations The Actuarial Education Company


Subject SP7: Assignment X2
2019 Examinations
Please complete the following information:

Name:
Number of following pages: _______

Please put a tick in this box if you have solutions


and a cross if you do not:

Please tick here if you are allowed extra time or


ActEd Student Number (see Note below): other special conditions in the
profession’s exams (if you wish to
share this information):

Time to do assignment
(see Note below): _____ hrs _____ mins
Note: Your ActEd Student Number is printed on all
personal correspondence from ActEd. Quoting it will help Under exam conditions
us to process your scripts quickly. If you do not know (delete as applicable): yes / nearly / no
your ActEd Student Number, please email us at
Note: If you take more than 2¾ hours, you should
ActEd@bpp.com.
indicate how much you completed within this
Your ActEd Student Number is not the same as your exam time so that the marker can provide useful
IFoA Actuarial Reference Number or ARN. feedback on your progress.

Score and grade for this assignment (to be completed by marker):

Q1 Q2 Q3 Q4 Q5 Q6 Q7 Q8 Q9 Q10 Total

=_____%
4 13 6 3 4 18 10 7 8 7 80

Grade: A B C D E Marker’s initials: ________

Please tick the following checklist so that your script can be marked quickly. Have you:

[ ] Checked that you are using the latest version of the assignments, ie 2019 for the sessions leading
to the 2019 exams?
[ ] Written your full name in the box above?
[ ] Completed your ActEd Student Number in the box above?
[ ] Recorded your attempt conditions?
[ ] Numbered all pages of your script (excluding this cover sheet)?
[ ] Written the total number of pages (excluding the cover sheet) in the space above?
[ ] Included your Marking Voucher or ordered Series X Marking?
[ ] Rated your X1 marker at www.ActEd.co.uk/marking?

Please follow the instructions on the previous page when submitting your script for marking.

The Actuarial Education Company © IFE: 2019 Examinations


Feedback from marker

Notes on marker’s section

The main objective of marking is to provide specific advice on how to improve your chances of
success in the exam. The most useful aspect of the marking is the comments the marker makes
throughout the script, however you will also be given a percentage score and the band into which
that score falls. Each assignment tests only part of the course and hence does not give a complete
indication of your likely overall success in the exam. However it provides a good indicator of your
understanding of the material tested and the progress you are making with your studies:

A = Excellent progress B = Good progress C = Average progress


D = Below average progress E = Well below average progress

Please note that you can provide feedback on the marking of this assignment at:

www.ActEd.co.uk/marking

© IFE: 2019 Examinations The Actuarial Education Company


SP7: Assignment X2 Questions Page 1

X2.1 Explain the potential benefits and problems of using industry-wide data for reserving purposes. [4]

X2.2 (i) State the factors that have the biggest influence on the quality of data that is likely to be
available from different insurance companies. [4]

(ii) List possible reasons why data might be inadequate, and hence lead to uncertainty in
modelling. [9]
[Total 13]

X2.3 A rich friend of yours has just become a Lloyd’s Name. They have joined a syndicate that writes
only marine insurance. List sources of risk and uncertainty which will affect the return that they
make from their capital outlay. [6]

X2.4 List the reasons why off-shoring might increase expense uncertainty. [3]

X2.5 Describe how increased globalisation has resulted in increased uncertainty for insurers. [4]

X2.6 A new general insurance company is being set up to specialise in private motor insurance.

(i) Outline the factors that should be considered by the company when establishing a central
computer system. [10]

(ii) List the data you would expect to see held on the individual policy files, explaining why
the data would be maintained. [8]
[Total 18]

X2.7 List problems that might be faced when carrying out a claims analysis for a motor account. [10]

X2.8 Discuss the implications of using a very conservative valuation basis for:
 a financially strong company writing predominantly household and private motor business
 a rapidly expanding company writing mainly liability business
 a contracting company which writes a wide range of international business. [7]

X2.9 A general insurance company currently sells professional indemnity insurance through brokers. It
is considering starting to sell this business via the internet.

Explain the possible impact of this on the mix of professional indemnity business and suggest how
the experience from the two channels might differ. [8]

The Actuarial Education Company © IFE: 2019 Examinations


Page 2 ST7: Assignment X2 Questions

X2.10 You have just been recruited to work for a brand new general insurance company selling
household business.

Suggest reasons why claims experience might be worse than assumed for such a company. [7]

END OF PAPER

© IFE: 2019 Examinations The Actuarial Education Company


Subject SP7: Assignment X3
2019 Examinations

Time allowed: 2¾ hours

Instructions to the candidate


1. Please:

– attempt all of the questions, as far as possible under exam conditions

– begin your answer to each question on a new page

– leave at least 2cm margin on all borders

– write in black ink using a medium-sized nib because we will be unable to mark
illegible scripts

– note that assignment marking is not included in the price of the course materials.
Please purchase Series Marking or a Marking Voucher before submitting your script.

– note that we only accept the current version of assignments for marking, ie you can
only submit this assignment in the sessions leading to the 2019 exams.

2. Please do not:

– use headed paper

– use highlighting in your script.

At the end of the assignment


If your script is being marked by ActEd, please follow
the instructions on the reverse of this page.

In addition to this paper, you should have available actuarial tables and an
electronic calculator.

The Actuarial Education Company © IFE: 2019 Examinations


Submission for marking

You should aim to submit this script for marking by the recommended submission date. The
recommended and deadline dates for submission of this assignment are listed on the summary page
at the back of this pack and on our website at www.ActEd.co.uk.

Scripts received after the deadline date will not be marked, unless you are using a Marking Voucher.
It is your responsibility to ensure that scripts reach ActEd in good time. If you are using Marking
Vouchers, then please make sure that your script reaches us by the Marking Voucher deadline date
to give us enough time to mark and return the script before the exam.

When submitting your script, please:

 complete the cover sheet, including the checklist


 scan your script, cover sheet (and Marking Voucher if applicable) and save as a pdf
document, then email it to: ActEdMarking@bpp.com
 do not submit a photograph of your script
 do not include the question paper in the scan.
In addition, please note the following:

 Please title the email to ensure that the subject and assignment are clear eg ‘SP7 Assignment
X3 No. 12345’, inserting your ActEd Student Number for 12345.
 The assignment should be scanned the right way up (so that it can be read normally without
rotation) and as a single document. We cannot accept individual files for each page.
 Please set the resolution so that the script is legible and the resulting PDF is less than 4 MB
in size.
 Do not protect the PDF in any way (otherwise the marker cannot return the script to ActEd,
which causes delays).
 Please include the ‘feedback from marker’ sheet when scanning.
 Before emailing to ActEd, please check that your scanned assignment includes all pages and
conforms to the above.

© IFE: 2019 Examinations The Actuarial Education Company


Subject SP7: Assignment X3
2019 Examinations
Please complete the following information:

Name:
Number of following pages: _______

Please put a tick in this box if you have solutions


and a cross if you do not:

Please tick here if you are allowed extra time or


ActEd Student Number (see Note below): other special conditions in the
profession’s exams (if you wish to
share this information):

Time to do assignment
(see Note below): _____ hrs _____ mins
Note: Your ActEd Student Number is printed on all
personal correspondence from ActEd. Quoting it will help Under exam conditions
us to process your scripts quickly. If you do not know (delete as applicable): yes / nearly / no
your ActEd Student Number, please email us at
Note: If you take more than 2¾ hours, you should
ActEd@bpp.com.
indicate how much you completed within this
Your ActEd Student Number is not the same as your exam time so that the marker can provide useful
IFoA Actuarial Reference Number or ARN. feedback on your progress.

Score and grade for this assignment (to be completed by marker):

Q1 Q2 Q3 Q4 Q5 Q6 Q7 Q8 Q9 Total

=_____%
8 5 4 6 14 11 15 8 9 80

Grade: A B C D E Marker’s initials: ________

Please tick the following checklist so that your script can be marked quickly. Have you:

[ ] Checked that you are using the latest version of the assignments, ie 2019 for the sessions leading
to the 2019 exams?
[ ] Written your full name in the box above?
[ ] Completed your ActEd Student Number in the box above?
[ ] Recorded your attempt conditions?
[ ] Numbered all pages of your script (excluding this cover sheet)?
[ ] Written the total number of pages (excluding the cover sheet) in the space above?
[ ] Included your Marking Voucher or ordered Series X Marking?
[ ] Rated your X2 marker at www.ActEd.co.uk/marking?

Please follow the instructions on the previous page when submitting your script for marking.

The Actuarial Education Company © IFE: 2019 Examinations


Feedback from marker

Notes on marker’s section

The main objective of marking is to provide specific advice on how to improve your chances of
success in the exam. The most useful aspect of the marking is the comments the marker makes
throughout the script, however you will also be given a percentage score and the band into which
that score falls. Each assignment tests only part of the course and hence does not give a complete
indication of your likely overall success in the exam. However it provides a good indicator of your
understanding of the material tested and the progress you are making with your studies:

A = Excellent progress B = Good progress C = Average progress


D = Below average progress E = Well below average progress

Please note that you can provide feedback on the marking of this assignment at:

www.ActEd.co.uk/marking

© IFE: 2019 Examinations The Actuarial Education Company


SP7: Assignment X3 Questions Page 1

X3.1 As the actuary for a medium-sized general insurer, for each of the last three years you have
carried out chain-ladder calculations as part of the process for setting the company’s claims
reserves. Your chain ladder has been based on a triangle of claims paid, split by accident year and
year of payment.

Early in the year, you are given the new data by the claims department, ie claim payments in the
previous year, split by origin year. Describe the data checks you should carry out before
processing your chain ladder calculations. [8]

X3.2 Outline the advantages and disadvantages of discounting reserves. [5]

X3.3 General insurance company A is considering taking over general insurance company B. Outline
the key considerations in determining the appropriate reserving basis to use in the assessment of
the takeover. [4]

X3.4 The reserves for a general insurer’s household account at 31 December 2018 have been assessed
using a basic chain-ladder method based on a claims incurred cohort, with five years’ past claims
data. Discuss how the following events may lead to distortions in the results from the chain
ladder:

(i) A postal strike that started during November 2017 and lasted for two months. [5]

(ii) The number of recorded nil claims has been very erratic, due to changes in the settlement
and reporting procedures. [1]
[Total 6]

X3.5 An insurer estimates the IBNR reserve for its household portfolio using the following monthly
delay table in which lx and dx are indices of the number of IBNR claims at the beginning of month x
and the number of claims reported during month x respectively.

x lx dx
0 1,000 250
1 750 375
2 375 175
3 200 100
4 100 50
5 50 25
6 25 20
7 5 5
8 0

(i) Explain what is meant by a reserve for IBNR. [1]

The Actuarial Education Company © IFE: 2019 Examinations


Page 2 SP7: Assignment X3 Questions

(ii) Estimate the number of IBNR claims at 31 December 2018 based on the following
information:

Month of claim event No of claims reported by


(2018) 31st December 2018
January 624
February 723
March 782
April 941
May 771
June 848
July 921
August 1,186
September 1,318
October 1,280
November 787
December 190
[6]

(iii) Explain how the company would use the figures calculated in part (ii) to estimate the
reserve for IBNR at 31 December 2018. [3]

(iv) Explain why this delay table can only provide an approximate model of the delays that will
actually be experienced in the reporting of claims. [4]
[Total 14]

X3.6 A general insurer is calculating its outstanding claims reserves. It is considering whether to use
the basic chain ladder or the Bornhuetter-Ferguson method.

(i) Give the advantages of using each of these methods compared with the other. [4]

You have just calculated the year-end reserves for a class of business, using the
Bornhuetter-Ferguson method to be £2.6m. An independent assessor has estimated the reserves
to be £2.3m by looking at each one on a case by case basis.

(ii) State possible reasons for this difference. [7]


[Total 11]

© IFE: 2019 Examinations The Actuarial Education Company


SP7: Assignment X3 Questions Page 3

X3.7 The Government Actuary of a developing country has suggested that the balance sheet should
contain an item described as ’additional amount for unexpired risks’ in the statutory returns of
general insurers. This reserve should be calculated by a statutory formula, the formula being:

(CR)  (UP) – (UPR), where:


 (CR) is a claim ratio for the financial year of the return
 (UP) is the unearned premium at the year end, before any deduction for deferred
acquisition costs
 (UPR) is the provision for unearned premiums at the year end after deduction of
deferred acquisition costs.

(i) Explain the reasoning behind the formula, commenting where necessary on any further
definitions required. [3]

(ii) Comment on the advantages and disadvantages of this formula as a statutory valuation
basis. Suggest whether or not the proposal should be supported by the insurance
industry. [12]
[Total 15]

The Actuarial Education Company © IFE: 2019 Examinations


Page 4 SP7: Assignment X3 Questions

X3.8 You are given below the future run-off pattern assumed by an insurance company and data for
the five most recent accident years.

Proportion of payments made in development period (years)


1 2 3 4 5 after 5
10% 20% 25% 15% 10% 20%

Accident Premium Ultimate Cumulative paid claims at development year


year earned loss ratio
1 2 3 4 5

2014 6,500 125% 744 2,108 3,702 5,653 6,607


2015 7,000 140% 926 2,369 4,384 6,408
2016 8,500 120% 1,007 2,732 5,132
2017 8,000 130% 1,385 4,138
2018 9,200 160% 1,800

Note that the ultimate loss ratio is defined as the total claims incurred / earned premium.

Use an adjusted form of the Bornhuetter-Ferguson method and the above assumptions and data,
to calculate as at 31 December 2018:

(i) the total reserve required for outstanding claim payments from accidents occurring in
these five years [5]

(ii) separate revised expected ultimate loss ratios for each of the five years. [3]
[Total 8]

X3.9 Suggest, with reasons, an appropriate method of estimating outstanding claims for each of the
following classes of business:
(a) private motor
(b) mortgage indemnity guarantee
(c) employers’ liability.

In each case, you should identify any particular problems that may arise when estimating the
outstanding claims for that class of business. [9]

END OF PAPER

© IFE: 2019 Examinations The Actuarial Education Company


Subject SP7: Assignment X4
2019 Examinations

Time allowed: 3¼ hours

Instructions to the candidate


1. Please:

– attempt all of the questions, as far as possible under exam conditions

– begin your answer to each question on a new page

– leave at least 2cm margin on all borders

– write in black ink using a medium-sized nib because we will be unable to mark
illegible scripts

– note that assignment marking is not included in the price of the course materials.
Please purchase Series Marking or a Marking Voucher before submitting your script.

– note that we only accept the current version of assignments for marking, ie you can
only submit this assignment in the sessions leading to the 2019 exams.

2. Please do not:

– use headed paper

– use highlighting in your script.

At the end of the assignment


If your script is being marked by ActEd, please follow
the instructions on the reverse of this page.

In addition to this paper, you should have available actuarial tables and an
electronic calculator.

The Actuarial Education Company © IFE: 2019 Examinations


Submission for marking

You should aim to submit this script for marking by the recommended submission date. The
recommended and deadline dates for submission of this assignment are listed on the summary page
at the back of this pack and on our website at www.ActEd.co.uk.

Scripts received after the deadline date will not be marked, unless you are using a Marking Voucher.
It is your responsibility to ensure that scripts reach ActEd in good time. If you are using Marking
Vouchers, then please make sure that your script reaches us by the Marking Voucher deadline date
to give us enough time to mark and return the script before the exam.

When submitting your script, please:

 complete the cover sheet, including the checklist


 scan your script, cover sheet (and Marking Voucher if applicable) and save as a pdf
document, then email it to: ActEdMarking@bpp.com
 do not submit a photograph of your script
 do not include the question paper in the scan.
In addition, please note the following:

 Please title the email to ensure that the subject and assignment are clear eg ‘SP7 Assignment
X4 No. 12345’, inserting your ActEd Student Number for 12345.
 The assignment should be scanned the right way up (so that it can be read normally without
rotation) and as a single document. We cannot accept individual files for each page.
 Please set the resolution so that the script is legible and the resulting PDF is less than 4 MB
in size.
 Do not protect the PDF in any way (otherwise the marker cannot return the script to ActEd,
which causes delays).
 Please include the ‘feedback from marker’ sheet when scanning.
 Before emailing to ActEd, please check that your scanned assignment includes all pages and
conforms to the above.

© IFE: 2019 Examinations The Actuarial Education Company


Subject SP7: Assignment X4
2019 Examinations
Please complete the following information:

Name:
Number of following pages: _______

Please put a tick in this box if you have solutions


and a cross if you do not:

Please tick here if you are allowed extra time or


ActEd Student Number (see Note below): other special conditions in the
profession’s exams (if you wish to
share this information):

Time to do assignment
(see Note below): _____ hrs _____ mins
Note: Your ActEd Student Number is printed on all
personal correspondence from ActEd. Quoting it will help Under exam conditions
us to process your scripts quickly. If you do not know (delete as applicable): yes / nearly / no
your ActEd Student Number, please email us at
Note: If you take more than 3¼ hours, you should
ActEd@bpp.com.
indicate how much you completed within this
Your ActEd Student Number is not the same as your exam time so that the marker can provide useful
IFoA Actuarial Reference Number or ARN. feedback on your progress.

Score and grade for this assignment (to be completed by marker):

Q1 Q2 Q3 Q4 Q5 Q6 Q7 Q8 Q9 Q10 Total

=_____%
4 6 7 4 7 7 14 14 22 15 100

Grade: A B C D E Marker’s initials: ________

Please tick the following checklist so that your script can be marked quickly. Have you:

[ ] Checked that you are using the latest version of the assignments, ie 2019 for the sessions leading
to the 2019 exams?
[ ] Written your full name in the box above?
[ ] Completed your ActEd Student Number in the box above?
[ ] Recorded your attempt conditions?
[ ] Numbered all pages of your script (excluding this cover sheet)?
[ ] Written the total number of pages (excluding the cover sheet) in the space above?
[ ] Included your Marking Voucher or ordered Series X Marking?
[ ] Rated your X3 marker at www.ActEd.co.uk/marking?

Please follow the instructions on the previous page when submitting your script for marking.

The Actuarial Education Company © IFE: 2019 Examinations


Feedback from marker

Notes on marker’s section

The main objective of marking is to provide specific advice on how to improve your chances of
success in the exam. The most useful aspect of the marking is the comments the marker makes
throughout the script, however you will also be given a percentage score and the band into which
that score falls. Each assignment tests only part of the course and hence does not give a complete
indication of your likely overall success in the exam. However it provides a good indicator of your
understanding of the material tested and the progress you are making with your studies:

A = Excellent progress B = Good progress C = Average progress


D = Below average progress E = Well below average progress

Please note that you can provide feedback on the marking of this assignment at:

www.ActEd.co.uk/marking

© IFE: 2019 Examinations The Actuarial Education Company


SP7: Assignment X4 Questions Page 1

X4.1 Briefly outline the analyses that should be carried out and other factors an actuary should take
into account when analysing the results of a reserving exercise. [4]

X4.2 An insurer writes mortgage indemnity guarantee insurance throughout a number of countries. It
wishes to use scenario analyses to investigate the upper limit on the range of possible reserve
estimates.

(i) List three ways in which scenarios could be derived. [2]

(ii) Describe eight adverse scenarios that might be modelled by the company. [4]
[Total 6]

X4.3 Explain the meaning of the following terms and explain how each of these elements of
uncertainty can be reduced:
(i) model uncertainty [3]
(ii) parameter uncertainty [2]
(iii) process uncertainty. [2]
[Total 7]

X4.4 Give reasons for correlations between different lines of business and describe how allowance can
be made for correlations within a stochastic model. [4]

X4.5 Discuss the key points to consider when communicating a best estimate reserve. [7]

X4.6 Outline the rationale behind the Bornhuetter-Ferguson method for claims reserving, highlighting
the difference between the deterministic and the stochastic versions. [7]

X4.7 You are an actuary working in the investment department of a large general insurer.

(i) Describe how you would create and use an asset-liability model to decide upon the asset
strategy of the general insurer. [10]

The department manager has suggested that in order to minimise the chance of insolvency in the
future, the company should aim to invest a substantial proportion of its assets in medium-term
and long-dated loan stocks.

(ii) Discuss the potential problems of this suggestion. [4]


[Total 14]

The Actuarial Education Company © IFE: 2019 Examinations


Page 2 SP7: Assignment X4 Questions

X4.8 (i) List the criteria you would use to judge whether a method of determining reserves for a
general insurance portfolio makes appropriate allowance for reserve risk. [4]

(ii) The following methods have been suggested for adjusting reserves to allow for reserve
risk:
(a) Reserves are set equal to the best estimate plus 10%.
(b) Reserves are set equal to the 75th percentile amount.
(c) Reserves are set equal to the best estimate plus one standard deviation.

Describe the rationale behind each approach and outline briefly the advantages and
disadvantages of each method. [10]
[Total 14]

X4.9 (i) Define a diagnostic. [1]

(ii) Explain the value of using diagnostic tools to assess the suitability of reserving
results. [4]

(iii) Describe seven common diagnostics that might be used to examine claim amounts and
claim frequency. [13]

(iv) A range of diagnostic tests has been carried out on a particular class of business and a
number of unusually high figures have been noted. A student actuary has commented
that as a result we will need to amend the reserving assumptions. Comment on the
students view. [4]
[Total 22]

© IFE: 2019 Examinations The Actuarial Education Company


SP7: Assignment X4 Questions Page 3

X4.10 Four students are discussing the investment policy of a general insurance company that writes
motor, property and employers’ liability insurance only. Discuss each of the students’ comments
in turn.

(i) Student A states that all investments should be very long term. Their main reason is that
the company has been writing general insurance business for the past 30 years. Since
premium income has continued to grow and cashflow has always been positive, the
company need not have regard to the current liability profile. [4]

(ii) Student B states that the business should only be looked at on a break-up basis. They say
that all investments should be in short-term bonds and deposits to ensure that the
company can meet its liabilities if they become payable at the accounting year-end. [4]

(iii) Student C states that to maximise the dividends paid to shareholders all monies should be
invested in assets offering potential high returns, eg equities. [4]

(iv) Student D states that all funds should be invested to match the expected future cashflow
from existing claims liability ignoring any future premiums. The securities used should
include equities and index-linked bonds to make allowance for the possible effect of
inflation on current outstanding claims. [3]
[Total 15]

END OF PAPER

The Actuarial Education Company © IFE: 2019 Examinations


All study material produced by ActEd is copyright and is sold
for the exclusive use of the purchaser. The copyright is
owned by Institute and Faculty Education Limited, a
subsidiary of the Institute and Faculty of Actuaries.

Unless prior authority is granted by ActEd, you may not hire


out, lend, give out, sell, store or transmit electronically or
photocopy any part of the study material.

You must take care of your study material to ensure that it


is not used or copied by anybody else.

Legal action will be taken if these terms are infringed. In


addition, we may seek to take disciplinary action through
the profession or through your employer.

These conditions remain in force after you have finished


using the course.

The Actuarial Education Company © IFE: 2019 Examinations


Subject SP7: Assignment X5
2019 Examinations

Time allowed: 3¼ hours

Instructions to the candidate


1. Please:

– attempt all of the questions, as far as possible under exam conditions

– begin your answer to each question on a new page

– leave at least 2cm margin on all borders

– write in black ink using a medium-sized nib because we will be unable to mark
illegible scripts

– note that assignment marking is not included in the price of the course materials.
Please purchase Series Marking or a Marking Voucher before submitting your script.

– note that we only accept the current version of assignments for marking, ie you can
only submit this assignment in the sessions leading to the 2019 exams.

2. Please do not:

– use headed paper

– use highlighting in your script.

At the end of the assignment


If your script is being marked by ActEd, please follow
the instructions on the reverse of this page.

In addition to this paper, you should have available actuarial tables and an
electronic calculator.

The Actuarial Education Company © IFE: 2019 Examinations


Submission for marking

You should aim to submit this script for marking by the recommended submission date. The
recommended and deadline dates for submission of this assignment are listed on the summary page
at the back of this pack and on our website at www.ActEd.co.uk.

Scripts received after the deadline date will not be marked, unless you are using a Marking Voucher.
It is your responsibility to ensure that scripts reach ActEd in good time. If you are using Marking
Vouchers, then please make sure that your script reaches us by the Marking Voucher deadline date
to give us enough time to mark and return the script before the exam.

When submitting your script, please:

 complete the cover sheet, including the checklist


 scan your script, cover sheet (and Marking Voucher if applicable) and save as a pdf
document, then email it to: ActEdMarking@bpp.com
 do not submit a photograph of your script
 do not include the question paper in the scan.
In addition, please note the following:

 Please title the email to ensure that the subject and assignment are clear eg ‘SP7 Assignment
X5 No. 12345’, inserting your ActEd Student Number for 12345.
 The assignment should be scanned the right way up (so that it can be read normally without
rotation) and as a single document. We cannot accept individual files for each page.
 Please set the resolution so that the script is legible and the resulting PDF is less than 4 MB
in size.
 Do not protect the PDF in any way (otherwise the marker cannot return the script to ActEd,
which causes delays).
 Please include the ‘feedback from marker’ sheet when scanning.
 Before emailing to ActEd, please check that your scanned assignment includes all pages and
conforms to the above.

© IFE: 2019 Examinations The Actuarial Education Company


Subject SP7: Assignment X5
2019 Examinations
Please complete the following information:

Name:
Number of following pages: _______

Please put a tick in this box if you have solutions


and a cross if you do not:

Please tick here if you are allowed extra time or


ActEd Student Number (see Note below): other special conditions in the
profession’s exams (if you wish to
share this information):

Time to do assignment
(see Note below): _____ hrs _____ mins
Note: Your ActEd Student Number is printed on all
personal correspondence from ActEd. Quoting it will help Under exam conditions
us to process your scripts quickly. If you do not know (delete as applicable): yes / nearly / no
your ActEd Student Number, please email us at
Note: If you take more than 3¼ hours, you should
ActEd@bpp.com.
indicate how much you completed within this
Your ActEd Student Number is not the same as your exam time so that the marker can provide useful
IFoA Actuarial Reference Number or ARN. feedback on your progress.

Score and grade for this assignment (to be completed by marker):

Q1 Q2 Q3 Q4 Q5 Q6 Q7 Q8 Q9 Total

=_____%
7 5 9 11 11 11 18 12 16 100

Grade: A B C D E Marker’s initials: ________

Please tick the following checklist so that your script can be marked quickly. Have you:

[ ] Checked that you are using the latest version of the assignments, ie 2019 for the sessions leading
to the 2019 exams?
[ ] Written your full name in the box above?
[ ] Completed your ActEd Student Number in the box above?
[ ] Recorded your attempt conditions?
[ ] Numbered all pages of your script (excluding this cover sheet)?
[ ] Written the total number of pages (excluding the cover sheet) in the space above?
[ ] Included your Marking Voucher or ordered Series X Marking?
[ ] Rated your X4 marker at www.ActEd.co.uk/marking?

Please follow the instructions on the previous page when submitting your script for marking.

The Actuarial Education Company © IFE: 2019 Examinations


Feedback from marker

Notes on marker’s section

The main objective of marking is to provide specific advice on how to improve your chances of
success in the exam. The most useful aspect of the marking is the comments the marker makes
throughout the script, however you will also be given a percentage score and the band into which
that score falls. Each assignment tests only part of the course and hence does not give a complete
indication of your likely overall success in the exam. However it provides a good indicator of your
understanding of the material tested and the progress you are making with your studies:

A = Excellent progress B = Good progress C = Average progress


D = Below average progress E = Well below average progress

Please note that you can provide feedback on the marking of this assignment at:

www.ActEd.co.uk/marking

© IFE: 2019 Examinations The Actuarial Education Company


SP7: Assignment X5 Questions Page 1

X5.1 (i) List the requirements of a good model. [5]

(ii) List the additional requirements that apply when using stochastic models. [2]
[Total 7]

X5.2 You are using VaR as your risk statistic in a capital allocation exercise. Discuss briefly the methods
you could use to determine an appropriate allocation of capital to each of the sub-portfolios of
your business. [5]

X5.3 (i) State the reasons why two general insurance companies writing similar levels of gross
written premium might have different capital requirements. [5]

(ii) State the information you would require before you could give an opinion on the relative
riskiness of two general insurance companies’ portfolios. [4]
[Total 9]

X5.4 (i) State, with reasons, the parties that may be consulted in the design of a capital model. [8]

(ii) Outline the other parties who should be considered. [3]


[Total 11]

X5.5 You are an actuary working for a small general insurance company, writing only personal lines
household and motor insurance business. It sells this business through a variety of distribution
channels.

You are currently investigating the level of capital that the company will require. Part of this work
involves examining the risks to the company of various scenarios. The scenario being considered
is that of a major failure of the company’s entire IT systems, lasting several days.

(i) Outline the main risks to this company of the above scenario. [7]

(ii) In addition to an assessment of risks that the insurer is exposed to, outline other factors
that might influence the capital requirements for the company. [4]
[Total 11]

X5.6 A small general insurance subsidiary is part of a large, well-known multinational composite
insurer. As part of an expansion strategy, the subsidiary needs to raise a significant amount of
capital. One option is a loan from the parent company on better terms than would be available in
the market.

(i) Explain the main opportunities and risks to the subsidiary of this loan. [8]

(ii) List the additional group risks that the subsidiary is exposed to. [3]
[Total 11]

The Actuarial Education Company © IFE: 2019 Examinations


Page 2 SP7: Assignment X5 Questions

X5.7 Discuss the predictability of the following items of future cashflow within a capital model:
 premium income
 claims
 expenses
 investment income. [18]

X5.8 Outline the reinsurance risks that should be captured when modelling capital requirements and
suggest how they might be modelled. [12]

X5.9 You are a consulting actuary. A new general insurance company has asked you for advice on
determining the level of capital it would require in order to support its business.

(i) State, with reasons, the information you would require before giving advice. [6]

(ii) Outline the points that you would make when presenting the advice to the client. [10]
[Total 16]

END OF PAPER

© IFE: 2019 Examinations The Actuarial Education Company


Subject SP7: Assignment X6
2019 Examinations

Time allowed: 3¼ hours

Instructions to the candidate


1. Please:

– attempt all of the questions, as far as possible under exam conditions

– begin your answer to each question on a new page

– leave at least 2cm margin on all borders

– write in black ink using a medium-sized nib because we will be unable to mark
illegible scripts

– note that assignment marking is not included in the price of the course materials.
Please purchase Series Marking or a Marking Voucher before submitting your script.

– note that we only accept the current version of assignments for marking, ie you can
only submit this assignment in the sessions leading to the 2019 exams.

2. Please do not:

– use headed paper

– use highlighting in your script.

At the end of the assignment


If your script is being marked by ActEd, please follow
the instructions on the reverse of this page.

In addition to this paper, you should have available actuarial tables and an
electronic calculator.

The Actuarial Education Company © IFE: 2019 Examinations


Submission for marking

You should aim to submit this script for marking by the recommended submission date. The
recommended and deadline dates for submission of this assignment are listed on the summary page
at the back of this pack and on our website at www.ActEd.co.uk.

Scripts received after the deadline date will not be marked, unless you are using a Marking Voucher.
It is your responsibility to ensure that scripts reach ActEd in good time. If you are using Marking
Vouchers, then please make sure that your script reaches us by the Marking Voucher deadline date
to give us enough time to mark and return the script before the exam.

When submitting your script, please:

 complete the cover sheet, including the checklist


 scan your script, cover sheet (and Marking Voucher if applicable) and save as a pdf
document, then email it to: ActEdMarking@bpp.com
 do not submit a photograph of your script
 do not include the question paper in the scan.
In addition, please note the following:

 Please title the email to ensure that the subject and assignment are clear eg ‘SP7 Assignment
X6 No. 12345’, inserting your ActEd Student Number for 12345.
 The assignment should be scanned the right way up (so that it can be read normally without
rotation) and as a single document. We cannot accept individual files for each page.
 Please set the resolution so that the script is legible and the resulting PDF is less than 4 MB
in size.
 Do not protect the PDF in any way (otherwise the marker cannot return the script to ActEd,
which causes delays).
 Please include the ‘feedback from marker’ sheet when scanning.
 Before emailing to ActEd, please check that your scanned assignment includes all pages and
conforms to the above.

© IFE: 2019 Examinations The Actuarial Education Company


Subject SP7: Assignment X6
2019 Examinations
Please complete the following information:

Name:
Number of following pages: _______

Please put a tick in this box if you have solutions


and a cross if you do not:

Please tick here if you are allowed extra time or


ActEd Student Number (see Note below): other special conditions in the
profession’s exams (if you wish to
share this information):

Time to do assignment
(see Note below): _____ hrs _____ mins
Note: Your ActEd Student Number is printed on all
personal correspondence from ActEd. Quoting it will help Under exam conditions
us to process your scripts quickly. If you do not know (delete as applicable): yes / nearly / no
your ActEd Student Number, please email us at
Note: If you take more than 3¼ hours, you should
ActEd@bpp.com.
indicate how much you completed within this
Your ActEd Student Number is not the same as your exam time so that the marker can provide useful
IFoA Actuarial Reference Number or ARN. feedback on your progress.

Score and grade for this assignment (to be completed by marker):

Q1 Q2 Q3 Q4 Q5 Q6 Total

=_____%
10 10 14 17 25 24 100

Grade: A B C D E Marker’s initials: ________

Please tick the following checklist so that your script can be marked quickly. Have you:

[ ] Checked that you are using the latest version of the assignments, ie 2019 for the sessions leading
to the 2019 exams?
[ ] Written your full name in the box above?
[ ] Completed your ActEd Student Number in the box above?
[ ] Recorded your attempt conditions?
[ ] Numbered all pages of your script (excluding this cover sheet)?
[ ] Written the total number of pages (excluding the cover sheet) in the space above?
[ ] Included your Marking Voucher or ordered Series X Marking?
[ ] Rated your X5 marker at www.ActEd.co.uk/marking?

Please follow the instructions on the previous page when submitting your script for marking.

The Actuarial Education Company © IFE: 2019 Examinations


Feedback from marker

Notes on marker’s section

The main objective of marking is to provide specific advice on how to improve your chances of
success in the exam. The most useful aspect of the marking is the comments the marker makes
throughout the script, however you will also be given a percentage score and the band into which
that score falls. Each assignment tests only part of the course and hence does not give a complete
indication of your likely overall success in the exam. However it provides a good indicator of your
understanding of the material tested and the progress you are making with your studies:

A = Excellent progress B = Good progress C = Average progress


D = Below average progress E = Well below average progress

Please note that you can provide feedback on the marking of this assignment at:

www.ActEd.co.uk/marking

© IFE: 2019 Examinations The Actuarial Education Company


SP7: Assignment X6 Questions Page 1

X6.1 A general insurance company has been selling steady volumes of business for many years. The
company has the following reinsurance treaties in place:
 A quota share arrangement under which 60% of the business is ceded. The reinsurer pays
the insurance company an override commission of 7% of the reinsurance premium.
 A 10-line surplus treaty, with a maximum retention limit of £1m.
 An excess of loss treaty that pays 90% of all losses above £0.5m, the premium for which is
calculated as 3% of the insurer’s gross written premium.

The treaties operate in the order given.

(i) Give a formula that, for a surplus treaty, relates the expected maximum loss (EML) to the
retention (r) and number of lines (l). [1]

(ii) A company director has written asking if you can give them an indication of the maximum
amount that the company would have to pay out as a result of an individual claim. Set
out the points that you would make in reply. [4]

(iii) Comment on the suggestion: ‘The quota share treaty is better than the excess of loss
treaty since the quota share treaty pays commission whilst the excess of loss treaty costs
the insurer. This must also mean that the excess of loss reinsurer is making more profit
than the quota share reinsurer.’ [5]
[Total 10]

X6.2 You are the actuary with an insurer that has experience of writing motor insurance over a number
of years. The insurer is considering establishing a separate class for classic motor vehicles
(ie collectors’ items).

(i) By considering each risk factor, discuss how the risk arising from policies in this class
might differ to that arising from standard motor insurance. [8]

(ii) Describe how the company’s approach to new business acquisition may have to
change. [2]
[Total 10]

X6.3 A general insurance company, which writes only direct business, accounts for all its business on
an accident-year basis.

Comparison of the company’s solvency margin (excess of assets over liabilities) with the solvency
margin for other direct insurers shows that the company’s solvency margin has fallen, whereas
the margin for most other companies has risen.

Explain the extent to which comparison of the solvency margin of different insurance companies
can validly be made, and describe the factors that would need to be taken into account if the
purpose of the comparison were to establish the likelihood of future insolvency. [14]

The Actuarial Education Company © IFE: 2019 Examinations


Page 2 SP7: Assignment X6 Questions

X6.4 You are a consultant actuary. One of your clients is considering the purchase of a general insurer
that only sells motor business. Initially, you have been given two years’ accounts for the insurer,
which are as follows:

Year X Year X+1


Written premiums 15.0 16.0
Earned premiums 14.5 15.5

Claims paid 16.0 12.0


Increase in o/s claims reserves –5.5 1.5

Expenses paid 1.5 1.6


Increase in DAC 0.1 0.1

Investment income 9.0 4.0

Tax 2.9 1.1


Dividends 7.0 7.5

Retained profit 1.7 – 4.1

You are also given the balance sheets at the start of each year, which are:

Year X Year X+1 Year X+2


O/s claims reserves 50.0 44.5 46.0
Unearned premium reserve 7.0 7.5 8.0
Adjustment for DAC (0.5) (0.6) (0.7)
Shareholders’ funds 10.0 11.7 7.6

Total assets 66.5 63.1 60.9

The potential purchaser has asked you to prepare a short report commenting on the trends in the
accounting ratios and giving possible reasons for the trends.

Discuss the items that you would include in your report. [17]

© IFE: 2019 Examinations The Actuarial Education Company


SP7: Assignment X6 Questions Page 3

X6.5 A country’s legislation for the supervision of its insurance industry is being reviewed.

(i) State the main aspects of a general insurance company and its business that a supervisory
authority could control to ensure that the company would be able to fulfil its obligations
to policyholders. [8]

(ii) If such obligations could not be met by the company, outline the other steps that might
be taken to ensure that all valid claims on the company by policyholders are paid. [2]

The following suggestions have been made as part of the review of legislation:
(a) a minimum solvency margin should be proportional to technical reserves
(b) a minimum solvency margin should be proportional to the square root of premium
income
(c) there should be a statutory basis for the calculation of technical reserves
(d) all technical reserves plus the minimum solvency margin should be covered by
government securities
(e) all reinsurance should be placed with a state owned reinsurance company.

(iii) State possible reasons for each of the suggestions, and comment on their suitability or
otherwise. [15]
[Total 25]

X6.6 You are the reserving actuary for a large established reinsurer that specialises in providing private
motor third-party liability excess of loss reinsurance. A range of layers is sold, attaching at fixed
amounts upwards of £150,000.

Every quarter, each cedant notifies the reinsurer of all claims with incurred amounts higher than
£150,000.

(i) Describe the characteristics of the reinsurance recoveries that you would expect to
see. [5]

You are about to perform a reserving exercise for statutory purposes.

(ii) Describe, with reasons, how you would group and adjust the claims data, outlining any
problems you envisage. [14]

(iii) List, with reasons, the reserving methods you would consider using for your exercise. [5]
[Total 24]

END OF PAPER

The Actuarial Education Company © IFE: 2019 Examinations


All study material produced by ActEd is copyright and is sold
for the exclusive use of the purchaser. The copyright is
owned by Institute and Faculty Education Limited, a
subsidiary of the Institute and Faculty of Actuaries.

Unless prior authority is granted by ActEd, you may not hire


out, lend, give out, sell, store or transmit electronically or
photocopy any part of the study material.

You must take care of your study material to ensure that it


is not used or copied by anybody else.

Legal action will be taken if these terms are infringed. In


addition, we may seek to take disciplinary action through
the profession or through your employer.

These conditions remain in force after you have finished


using the course.

The Actuarial Education Company © IFE: 2019 Examinations


Assignment deadlines

For the session leading to the April 2019 exams – SP Subjects

Marking vouchers

Subjects Assignments Mocks

SP1, SP2, SP4, SP7, SP8 13 March 2019 20 March 2019

SP5, SP6, SP9 20 March 2019 27 March 2019

Series X Assignments

Recommended
Subjects Assignment Final deadline date
submission date

SP1, SP2, SP4, SP7, SP8 5 December 2018 9 January 2019


X1
SP5, SP6, SP9 12 December 2018 9 January 2019

SP1, SP2, SP4, SP7, SP8 12 December 2018 23 January 2019


X2
SP5, SP6, SP9 19 December 2018 23 January 2019

SP1, SP2, SP4, SP7, SP8 2 January 2019 6 February 2019


X3
SP5, SP6, SP9 9 January 2019 6 February 2019

SP1, SP2, SP4, SP7, SP8 16 January 2019 20 February 2019


X4
SP5, SP6, SP9 23 January 2019 20 February 2019

SP1, SP2, SP4, SP7, SP8 30 January 2019 6 March 2019


X5
SP5, SP6, SP9 6 February 2019 6 March 2019

SP1, SP2, SP4, SP7, SP8 13 February 2019 13 March 2019


X6
SP5, SP6, SP9 20 February 2019 20 March 2019

Mock Exams

Recommended
Subjects Final deadline date
submission date

SP1, SP2, SP4, SP7, SP8 6 March 2019 20 March 2019

SP5, SP6, SP9 13 March 2019 27 March 2019

We encourage you to work to the recommended submission dates where possible.

If you submit your mock on the final deadline date you are likely to receive your script back less than a week
before your exam.

The Actuarial Education Company © IFE: 2019 Examinations


Assignment deadlines

For the session leading to the September 2019 exams – SP Subjects

Marking vouchers

Subjects Assignments Mocks

SP2, SP8 21 August 2019 28 August 2019

SP1, SP4, SP5, SP6, SP7, SP9 28 August 2019 4 September 2019

Series X Assignments

Recommended
Subjects Assignment Final deadline date
submission date

SP2, SP8 5 June 2019 17 July 2019


X1
SP1, SP4, SP5, SP6, SP7, SP9 12 June 2019 24 July 2019

SP2, SP8 19 June 2019 24 July 2019


X2
SP1, SP4, SP5, SP6, SP7, SP9 26 June 2019 31 July 2019

SP2, SP8 3 July 2019 31 July 2019


X3
SP1, SP4, SP5, SP6, SP7, SP9 10 July 2019 7 August 2019

SP2, SP8 10 July 2019 7 August 2019


X4
SP1, SP4, SP5, SP6, SP7, SP9 17 July 2019 14 August 2019

SP2, SP8 24 July 2019 14 August 2019


X5
SP1, SP4, SP5, SP6, SP7, SP9 31 July 2019 21 August 2019

SP2, SP8 31 July 2019 21 August 2019


X6
SP1, SP4, SP5, SP6, SP7, SP9 7 August 2019 28 August 2019

Mock Exams

Recommended
Subjects Final deadline date
submission date

SP2, SP8 14 August 2019 28 August 2019

SP1, SP4, SP5, SP6, SP7, SP9 21 August 2019 4 September 2019

We encourage you to work to the recommended submission dates where possible.

If you submit your mock on the final deadline date you are likely to receive your script back less than a week
before your exam.

© IFE: 2019 Examinations The Actuarial Education Company


SP7: Assignment X1 Solutions Page 1

Assignment X1 Solutions

Solution X1.1

Product liability business is discussed in depth in Chapter 3, Insurance products – types.


Losses-occurring policies and claims-made polices are also discussed in Chapter 2, Insurance
products – background.

(i) Product liability

Product liability insurance indemnifies the insured against legal liability for the death of or bodily
injury to a third party, or for damage to property belonging to a third party, that results from a
product fault. [1]

The policy will usually also cover legal costs. [½]

Some policies will include the costs of recalling faulty products that have not actually caused
damage. [½]
[Maximum 1]

(ii) Basis of cover

A losses-occurring policy is a policy providing cover for losses occurring in the defined period no
matter when they are reported. [½]

A claims-made policy is one that covers all claims reported to an insurer within the policy period
irrespective of when they occurred. [½]

It would be inappropriate to write the business on a losses-occurring basis because the date of
the loss may not be clear. [½]

For example, the product may contain harmful substances which consumers may be exposed to
over long periods of time. [½]

Similarly, consumers may not remember when they first noticed the product fault. [½]

Therefore product liability business is likely to be written on a claims-made basis. [½]


[Total 3]

The Actuarial Education Company © IFE: 2019 Examinations


Page 2 SP7: Assignment X1 Solutions

(iii) Product liability

The normal measure of exposure is the annual turnover of the company. [½]

Perils

These depend on the nature of the product, but include:


 faulty design [½]
 faulty manufacture [½]
 faulty packaging [½]
 incorrect or misleading instructions. [½]

Risk and rating factors

 nature of the product [½]


 distribution channel [½]
 extent of US-exposure [½]
 usage [½]
 general trade of the policyholder [½]
 potentially dangerous components [½]
 the underwriter’s subjective assessment of the risk [½]
[Total 6]

Solution X1.2

This question is examining the material in Chapter 9, External environment.

Claims experience

We would expect the ban to lead to fewer fire-related claims caused by smoking on the premises,
eg by careless disposal of cigarettes. [½]

Hence, we would expect fewer claims on property damage insurance and any related business
interruption cover. [½]

Similarly, (public or employers’) liability claims that are as a result of fire on the premises are less
likely. [½]

This assumes there is no change in the level of fire safety measures taken in the premises
covered. [½]

However, the smoking ban might lead to staff having a more relaxed view on fire safety
(eg leaving fire doors open), which may partially offset the above point. [½]

Longer term, the change might result in there being less risk of liability claims against employers
for exposing staff to smoky conditions that may be detrimental to their health … [½]

© IFE: 2019 Examinations The Actuarial Education Company


SP7: Assignment X1 Solutions Page 3

… and fewer public liability claims for exposing other customers to these conditions. [½]

Usage of restaurants / cafés

The impact on the usage of restaurants (ie the number of people eating out) will be unknown. [½]

For example, the ban might deter smokers from eating out, or a ‘smoke-free’ atmosphere might
encourage more people to use restaurants. [½]

(We assume in the rest of our solution that the number of diners will reduce.)

This may impact on claims experience, for example we would expect fewer claims if there were
fewer customers. [½]

However, having fewer customers will adversely affect the profitability of the businesses taking
out the insurance, which may affect their ability to afford insurance. [½]

This may encourage them to seek more competitive products elsewhere, or not take out certain
types of (non-compulsory) insurance, such as commercial property. [½]

It might also reduce (say) the number of restaurants and cafés in the country, as some may go out
of business. [½]

These will adversely impact persistency and new business volumes, and thus the insurer’s
expected profits. [½]

There may be more attempts to make fraudulent or exaggerated claims from businesses that are
struggling to remain profitable, which would impact on claims experience and administration
costs. [1]

Other implications

The insurer may need to change premium levels to reflect changes in its expected experience
(eg claims levels or business volumes). [½]

Rating factors may also be reviewed. For example, the level of fire safety measures may not be as
significant an influence on claim frequency as it was before. [½]

The impact of the ban will be unknown, and so there will be greater uncertainty in reserving and
thus in assessing capital requirements. Higher margins may be required in reserving,
pricing, etc. [½]

Depending on the significance of the ban, it might lead to a change in reinsurance arrangements
or requirements, for example there may be less need for reinsurance of fire-related perils. [½]

The Actuarial Education Company © IFE: 2019 Examinations


Page 4 SP7: Assignment X1 Solutions

Dependencies

The impact of the ban will also depend on:


 current policy conditions, eg exclusions or excesses
 the size of the insurance classes affected relative to the insurer’s other business
 level of fire safety measures in the premises covered
 prevalence of smoking in the restaurants / cafés covered before the ban
 details of the proposed ban and any exceptions (eg for open-air cafés)
 how strictly the ban is expected to be enforced in practice
 attitudes to the ban, ie to what extent people will comply with it
 action taken externally to offset any adverse effects of the ban on business, such as
advertising by the restaurant trade.
[½ for each valid point, maximum 4]
[Maximum 12]

Solution X1.3

This question is examining the material in Chapter 7, General insurance markets, Chapter 8,
Regulation and Chapter 9, External environment.

Advantages

A maximum premium may protect policyholders from overpriced products. [½]

This will be particularly useful for:


 any compulsory classes of insurance, ie types of insurance that certain customers have to
have by law [½]
 types of risk that would otherwise be uninsurable, eg households in areas prone to
flooding. [½]

A minimum premium will help to ensure that the premiums charged are adequate, and hence
help maintain the financial strength of the insurer and the security of claims being met for the
policyholders. [1]

A minimum premium may also help protect the existence of small companies. Companies with
large financial backing may find it harder to price their competitors out of the market. [1]

This may lead to more stable premiums from year to year, and reduce the effect of the
underwriting cycle. [1]

Price will become a less important differentiating factor between companies. This may encourage
companies to offer new innovative products to the benefit of the policyholders. [1]

© IFE: 2019 Examinations The Actuarial Education Company


SP7: Assignment X1 Solutions Page 5

Disadvantages

A maximum premium may result in some companies struggling to break even and consequently
exiting the market for certain products or customers. This will then reduce competition
further. [1]

A maximum premium could result in more high-risk policyholders taking out cover (as it would
cap their premium). [½]

A minimum premium may be unaffordable for some policyholders. [½]

A minimum premium may restrict competition, which could be to the detriment of policyholders,
as average premiums would rise. [½]

The legislation may not therefore be popular with companies or policyholders. [½]

The statutory body will have to calculate and publish the premium limits. This will be costly, and
will need to be recouped from profits or higher premiums. [½]

The statutory body will have to decide whether to:


 impose minimum and maximum premiums for every possible combination of rating
factors for every class of business [½]
 apply limits to groups of rating cells. [½]

The former approach would be very time consuming and expensive. [½]

The latter approach would result in much wider premium bands and may not restrict the level of
competition quite as much. [½]

Extra time and money would be required to enforce and police the new legislation. [½]

It would be difficult to allow for varying levels of cover offered by different companies,
eg different excess levels and exclusions. [½]

It will also be difficult to set premium limits for classes of insurance that cover very
heterogeneous risks, eg commercial fire. [½]

Companies may try and use a form of retrospective experience rating to effectively avoid the
limits. This would have to be excluded. [1]
[plus up to two bonus half marks for relevant points, maximum 11]

The Actuarial Education Company © IFE: 2019 Examinations


Page 6 SP7: Assignment X1 Solutions

Solution X1.4

Adverse development covers are discussed in Chapter 6, Reinsurance products – types.

(i) Adverse development cover

An adverse development cover is a reinsurance arrangement whereby a reinsurer agrees, in


return for a premium, to cover the ultimate settled amount of a specified block of business above
a certain pre-agreed amount. [1]

Claims are usually still handled by the insurer. [½]

Reserves are maintained by the insurer and it receives all investment income generated from the
investments backing these reserves. [½]
[Total 2]

(ii) Risks transferred

The main risk transferred is the reserve development risk, ie the reserves being inadequate to
cover liabilities. [1]

It caps the liabilities … [½]

… and protects the balance sheet from future losses in respect of old business. [½]
[Total 2]

(iii) Risks retained and additional risks accepted

Risks retained

The insurer may not be able to transfer the entire risk of the reserves being inadequate. In
particular:
 it is usually only possible to reinsure a layer above a specified amount – this specified
amount may be in excess of the current level of reserves [½]
 there could be an upper limit to the cover provided by the reinsurer – if the ultimate cost
of losses is in excess of this, the insurer is liable for the excess [½]
 the reinsurer may also insist that the insurer retains a small participation in the layer. [½]

Since claims are likely to be handled by the insurer, the insurer is at risk that claims handling
expenses are higher than expected. [½]

Furthermore, since the insurer will maintain the reserves, it will be at risk that investment income
on the reserves is lower than expected. [½]

© IFE: 2019 Examinations The Actuarial Education Company


SP7: Assignment X1 Solutions Page 7

Additional risks accepted

The additional risks accepted include:


 the credit risk of the reinsurer, since legally the insurer remains liable to the insured
parties for all claims [½]
 that the premium paid for the adverse development cover is relatively high considering
the risks taken on (this depends on the risk appetite of the market). [½]
[Maximum 3]

(iv) Overcoming the additional risks

The insurer should assess the credit risk of the reinsurer by looking at its credit rating and past
history of writing adverse development covers. [½]

The insurer could request that the reinsurer deposits some reserves with the insurer, which will
be repaid if experience turns out to be better than expected. [½]

The insurer could consider an alternative that would transfer the legal liability to the reinsurer,
eg a loss portfolio transfer (LPT). [½]

The insurer could only take out an adverse development cover when market conditions are
favourable (ie when premium rates are low). [½]

In other (less favourable) conditions, the insurer could seek an alternative way of transferring the
run-off risk, eg it could securitise the business, although this may also be expensive. [½]
[Maximum 2]

Solution X1.5

This question requires you to apply the material covered in Chapter 3, Insurance products  types.

 Buildings insurance to protect against fire, storm, flood and other hazards. [½]
 Contents insurance to protect against theft, flood, storm and other hazards. [½]
 Goods in transit to protect against loss of or damage to pizzas while being delivered. [½]
 Business interruption cover to protect against loss of profits after damage to contents or
buildings. [½]
 Motor fleet cover for the delivery vehicles / mopeds, to cover theft, liability and other
hazards. [1]
 Employers’ liability to protect against claims from the staff. [½]
 Personal accident for the staff in case they suffer certain injuries at work. [½]
 Public liability against claims from the public in respect of damage / injury, that isn’t
covered by other insurance. [½]

The Actuarial Education Company © IFE: 2019 Examinations


Page 8 SP7: Assignment X1 Solutions

 Product liability against claims from the public in respect of damage / injury caused by
harmful pizzas. [½]
 Environmental liability against claims for unintentional pollution. [½]
 Fidelity guarantee to protect against fraudulent behaviour of employees, eg delivery
people over-charging customers and pocketing the extra. [½]
 Trade credit to protect against people ordering pizzas and then not paying for them. [½]
 Creditor insurance to cover any loans, eg a loan to start up the business. [½]
 Legal expenses cover to protect against legal expenses arising from policyholder
complaints, eg food poisoning. [½]
[Maximum 7]

Solution X1.6

Distribution channels are covered in Chapter 7, General insurance markets.

(i) Methods of sale

Personal lines business is sold via:


 insurance brokers [½]
 direct to the public by direct mail and advertising, internet or agents [1]
 specialist channels such as travel agents (travel insurance), shops (extended warranty)
and clubs, employers etc [1]
 banks and building societies (eg buildings insurance, and many other insurance policies
with financial aspects). [½]

London Market business is normally sold via specialist brokers. [½]


[Maximum 3]

(ii) The slip system

1. The party seeking cover approaches a London Market broker and the broker prepares a
slip that shows, in a standard format, the main features of the risk to be insured. [1]

2. The broker shows the slip to one or more quoting underwriters who, on the basis of the
slip and further information as appropriate, quote a premium. [½]

3. The cedant (with the broker’s advice) will then select a lead underwriter and a ‘firm order’
price for the broker with which to approach the market. This firm order price may be
below any of the quoted prices. [1]

4. The lead underwriter accepts a share of the risk by stamping and signing the slip. [½]

5. The broker then approaches other underwriters (the following market) to accept the risk
on the same terms. [½]

© IFE: 2019 Examinations The Actuarial Education Company


SP7: Assignment X1 Solutions Page 9

6. The follow underwriters indicate the share that they are willing to take by stamping and
signing the slip under the lead underwriter line. [½]

7. The broker continues until they have finished placing the risk (that is, received offers for
100% or more of the risk). [½]

8. If the written lines exceed 100% then, in agreement with the insured, they are then
reduced (or ‘signed down’) so that the signed lines total 100%. [½]

9. If it is not possible to find capacity to place 100% of the risk then an additional shortfall
cover may need to be placed at different terms. [½]
[Maximum 5]

Solution X1.7

Surplus reinsurance is covered in Chapter 6, Reinsurance products – types.

We will assume no further reinsurance cover on risk 5. [½]

Alternatively, we could state what reinsurance cover there is assumed to be and give the
appropriate numerical solution.

We also assume that all lines on the first surplus treaty have to be used before the second surplus
treaty is applied, and so on. [1]

First find the number of lines taken by each reinsurer for each risk, using the EML:

Risk Insurer Reinsurer 1 Reinsurer 2 Reinsurer 3 Reinsurer 4 EML ($m)


1 1 1 0 0 0 2
2 1 3 5 0 0 9
3 1 3 5 6 0 15
4 1 3 5 10 11 30
5 12 3 5 10 20 50
[3]

Now convert this into a proportion for each risk:

Risk Insurer Reinsurer 1 Reinsurer 2 Reinsurer 3 Reinsurer 4 Total


1 0.500 0.500 0.000 0.000 0.000 1.000
2 0.110 0.333 0.556 0.000 0.000 1.000
3 0.067 0.200 0.333 0.400 0.000 1.000
4 0.033 0.100 0.167 0.333 0.367 1.000
5 0.240 0.060 0.100 0.200 0.400 1.000
[2]

The Actuarial Education Company © IFE: 2019 Examinations


Page 10 SP7: Assignment X1 Solutions

Finally, multiply this proportion by the claim amount:

Risk Insurer Reinsurer 1 Reinsurer 2 Reinsurer 3 Reinsurer 4 Total


1 2.500 2.500 0.000 0.000 0.000 5.000
2 0.556 1.667 2.778 0.000 0.000 5.000
3 0.667 2.000 3.333 4.000 0.000 10.000
4 1.667 5.000 8.333 16.667 18.333 50.000
5 12.000 3.000 5.000 10.000 20.000 50.000
[2]
[Maximum 7]

Solution X1.8

This is a Core Reading question from Chapter 8, Regulation.

(i) Objectives of regulating the insurance industry

Key objectives of regulation and supervision are to promote efficient, fair, safe and stable
insurance markets and to benefit and protect policyholders. [1]

Further objectives include to:


 enhance overall efficiency of the financial system [½]
 reduce transaction costs [½]
 create liquidity [½]
 facilitate economies of scale [½]
 contribute to economic growth [½]
 allocate resources efficiently [½]
 manage risk [½]
 mobilise long-term savings. [½]
[Maximum 3]

(ii) Key activities of IAIS

The IAIS:
 issues global principles, standards and guidance papers [½]
 provides training and support on issues related to insurance supervision [½]
 organises meetings and seminars for insurance supervisors. [½]
[Maximum 1]

© IFE: 2019 Examinations The Actuarial Education Company


SP7: Assignment X1 Solutions Page 11

(iii) Capital requirements

Examples of regulatory capital requirements include:


 requirement to deposit assets to back claims reserves [½]
 requirement to maintain a minimum level of solvency [½]
 the use of prescribed bases to calculate premiums, asset values and liabilities to
demonstrate solvency [½]
 requirement to hold a claims equalisation reserve (eg outside the EU) [½]
 requirement for risk-based capital calculations and capital assessment analyses [½]
 requirements in respect of the capital model, eg the requirement to satisfy the ‘use test’,
so that the capital model and results should be used to help manage the business. [½]
[Total 3]

(iv) Disadvantages of complying with IAIS ICPs

Disadvantages include:
 valuations may be less prudent, eg where the regulation requires assumptions to be
based on actual experience, without including margins [½]
 a risk-based approach could increase the volatility of results, leading to: [½]
– more volatile distribution of profits [½]
– reduce the availability of capital [½]
 broader reporting requirements may mean that intra-group arrangements will have to be
disclosed [½]
– this may bring further disadvantages, eg tax implications for arrangements that
take place across different territories [½]
 increased costs of regulatory supervision, which may be passed onto insurers via
increased levies [½]
 increased costs of compliance [½]
 loss of market confidence if insurers are seen not to comply with recognised best
practice. [½]
[Maximum 3]

The Actuarial Education Company © IFE: 2019 Examinations


Page 12 SP7: Assignment X1 Solutions

Solution X1.9

This question is testing your ability to apply the ideas in Chapter 9, External environment.

Both claim frequencies and amounts

 a change in speed limits [½]


 a change in the use of speed cameras [½]
 a change in societal trends, eg a reduction in drink driving [½]
 an unusually severe winter [½]
 the introduction of speed-restricting technology in vehicles [½]

Claim frequencies

 an increase in crime rates [½]


 a change in the effectiveness of car security systems [½]
 a change in the stringency of driving tests [½]
 a change in attitudes to claiming, leading to an increase in the frequency of liability claims
[½]

Claim amounts

 tax on car parts [½]


 legislation increasing the amount of cover that must be provided [½]
 legislation requiring the use of car safety equipment (eg seat belts or children’s seats),
leading to a decrease in the cost of bodily injury claims [½]
 a legislative change affecting claims costs (eg a reduction in the Ogden discount rate) [½]
 publicity regarding a fault with a particular make of car (eg a batch of cars sold with faulty
brakes), leading to higher court awards for compensation [½]
 a shortage of mechanics, leading to an increased cost of labour [½]
 a change in procedures for building / repairing cars, leading to a change in the cost / ease
of repairing cars [½]
 a change in driving habits (eg an increase in motorway driving and a decrease in driving in
city centres) – this may be due to factors such as congestion charges [½]
 a change in economic growth (eg in a recession, fewer people may buy big, new,
expensive cars, so the average claim size may fall) [½]
 a change in the rate of price inflation [½]
 a change in the rate of court award inflation [½]
 a change in exchange rates, which may affect the cost of claims made overseas (ie people
driving their cars abroad) [½]
[Maximum 6]

© IFE: 2019 Examinations The Actuarial Education Company


SP7: Assignment X2 Solutions Page 1

Assignment X2 Solutions

Solution X2.1

The Core Reading for this question can be found in Chapter 12, Data.

Potential benefits

Industry-wide data provide a benchmark for insurers to assess their position compared to their
competitors ... [½]

… both at the overall level and at the level at which the data is classified. [½]

Industry-based development factors may be valuable for reserving … [½]

… especially for small insurers and insurers that have only been established a short-time. [½]

Potential problems

There is potential for distortions, particularly owing to heterogeneity. [½]

The data supplied by different companies may not be precisely comparable, eg: [½]
 companies operate in different geographical or socio-economic sections of the market [½]
 the policies sold by different companies may not be identical [½]
 companies will have different practices, eg underwriting, claims handling, reserving [½]
 the nature of the data stored will not always be the same [½]
 the coding used for risk factors may vary from company to company. [½]

The data will be much less detailed and less flexible than those available internally. [½]

External data is often much more out of date than internal data. [½]

The data quality will depend on the quality of the data systems of all its contributors. [½]

Not all companies contribute. [½]


[Maximum 4]

The Actuarial Education Company © IFE: 2019 Examinations


Page 2 SP7: Assignment X2 Solutions

Solution X2.2

This question is examining material in Chapter 12, Data.

(i) Biggest influences on data quality from different companies

 age of company [½]


 size of company [½]
 age of computer and information system [½]
 existence of legacy systems [½]
 integrity of systems, eg automated checks [½]
 quality of managers [½]
 quality and training of staff [½]
 nature of organisation, eg direct insurer vs reinsurer [½]
 sales outlet(s) used, eg direct vs broker [½]
 classes of insurance sold [½]
 type of insured, eg large commercial risks vs small personal risks [½]
 product design [½]
 rate of change in products [½]
 simplicity and ease of use of computer systems [½]
[Maximum 4]

(ii) Reasons for data being inadequate

 poor data due to: [½]


– inadequate data recording procedures [½]
– poor system design – not capturing all rating factors [½]
– poor system design – not retaining a history of risks [½]
 inconsistent or incorrect data due to problems in recording: [½]
– no validation / reasonableness checks on digits, dates, amounts etc [½]
– no spot checks [½]
– no reconciliations [½]
– no cross-checks between policy and claims info [½]
– no cross-checks over different policy years [½]
 incomplete, ie data fields missing, due to: [½]
– poor systems design [½]
– poor user processes [½]
– inadequate information collected at outset by the insurer [½]
– inadequate information collected at outset by third parties [½]

© IFE: 2019 Examinations The Actuarial Education Company


SP7: Assignment X2 Solutions Page 3

 nonexistent data for: [½]


– new class of business [½]
– new territory (may occur because of globalisation of markets) [½]
– unusual risks [½]
– infrequent claim events [½]
– certain rating groups (eg very old people, unusual classes) [½]
– extreme values (where data does not exist or is too volatile to be usable) [½]
 data not completely relevant due to changes in: [½]
– external factors (eg global weather patterns) [½]
– policy terms and conditions [½]
– underwriting philosophy [½]
– claims management processes [½]
[Maximum 9]

Solution X2.3

This question is testing Chapter 11, Risk and uncertainty.

Premiums may not be enough to cover the expected claim cost. [½]

This may be due to:


 the underwriting cycle leading to lower rates [½]
 anti-selection if the rating structure is incorrect [½]
 inadequate data upon which to price the risks. [½]

Premiums could be too high, leading to lower than expected volumes of business and a failure to
adequately cover fixed expenses. [½]

Claims experience may be higher than expected. [½]

This could be due to:


 catastrophes [½]
 latent claims [½]
 aggregations of risk [½]
 poor policy wording [½]
 higher than expected claims inflation [½]
 unfavourable judicial decisions. [½]

Higher than expected expenses, commission or expense inflation. [1]

Low investment income, falls in assets values or increased taxation of returns. [1]

The Actuarial Education Company © IFE: 2019 Examinations


Page 4 SP7: Assignment X2 Solutions

Other possibilities:
 adverse currency movements [½]
 failure of third parties, eg reinsurers [½]
 changes in legislation [½]
 mis-management of the syndicate. [½]
[Maximum 6]

Solution X2.4

The material for this question is covered in Chapter 11, Risk and uncertainty.

There may be uncertainty over:


 set-up costs in the overseas country (due to lack of knowledge) [½]
 currency movements [½]
 wage inflation in the overseas country [½]
 price inflation in the overseas country [½]
 the effect on volumes of business if customers boycott the insurer as a result of setting up
call centres overseas [½]
 changes in regulation in the overseas country [½]
 changes in tax in the overseas country [½]
 changes in accounting procedures in the overseas country. [½]
[Maximum 3]

Solution X2.5

The material for this question is covered in Chapter 11, Risk and uncertainty.

Globalisation of insurance markets has enabled insurers to write business in many different
territories, some of which they will have little experience in. [½]

This will lead to uncertainty in setting reserves and premium rates. [1]

It will also increase competition, leading to lower premiums and lower profits and extra volatility
in the insurance cycle. [1]

Writing business in different territories may also introduce currency risks. [½]

© IFE: 2019 Examinations The Actuarial Education Company


SP7: Assignment X2 Solutions Page 5

Globalisation of investment markets has led to:


 a wider range of types of assets the insurer can invest in (and it may have little experience
of dealing in these assets) [½]
 a wider range of currencies that it can invest in (which can increase currency risk if not
matched) [½]
 greater uncertainties in the investment markets themselves due to increased flows of
capital between markets [½]
 correlations with other markets. [½]
[Maximum 4]

Solution X2.6

The material for this question is covered in Chapter 12, Data.

(i) Factors considered in establishing central computer system

 Function of the system. Need input from all potential users:


– administration
– accounting
– actuarial / statistical
– management
– marketing
– other (underwriters, reinsurers, claims, investment, computing). [1]
 For private motor, there is potentially a lot of data to be processed. All of the above users
will rely heavily upon the system. Their input initially is important. [½]
 Structure or outline of the system. How the system should operate, eg the branches and
agents may have direct access to the system. For private motor, the company will
probably want to be able to give on-line premium quotes to brokers. [1]
 Type of hardware and software to be used. [½]
 Cost of implementation. [½]
 Security, eg how the system will be monitored and who will have access to it. Data
protection laws should be considered, particularly if the system is to be used by third
parties. [1]
 The data that will need to be held. For private motor, we should retain all the data for
individual policyholders on the system. [1]
 How files will be structured. For example, the policy and claims records might be held on
separate files. [½]
 How long the data will be stored. Statistical analyses will be necessary (in due course)
using the data that develops. Every effort should be made to store data indefinitely,
whilst complying with any regulations over data retention. [½]

The Actuarial Education Company © IFE: 2019 Examinations


Page 6 SP7: Assignment X2 Solutions

 Capability. The system will need to be able to capture claim features such as multiple
payments, nil claims and reopened claims. [½]
 The output from the system is important. Again, ask all potential users of the system.
Examples of required output are: premium quotations (linked to brokers’ systems), policy
documents, renewal notices, endorsement notices, management reports, marketing
statistics, run-off tables, statutory returns, accounting reports. [1½]
 Ease of use. Need to consider the computer literacy of the end users. [½]
 Implementation of the system. How the system will be developed, and by whom. How it
will be tested. [1]
 When the system will be ready. This needs to be linked to the proposed launch of the
new policies. [½]
 Error aversion, eg the steps that will be taken to ensure data accuracy. [½]
 Changes to the system. The system must be sufficiently robust so that future
developments can be incorporated. [½]
 New company. There should be few constraints (other than cost), so the system design
need not be too concerned with existing practices. [½]
[Maximum 10]

(ii) List of data (policy file) and explanation

Administration data

Policy number to define the policy uniquely and provide a link with related
files, eg to the claims file [½]

Name and address of policyholder for correspondence, etc [½]

Broker / branch / commission for accounting use, and also for management and
marketing information [½]

Dates on risk defines the exposure period, and when renewal notices are
needed
[½]

Original year of issue for statistical and management information [½]

Endorsement data indicates a change to the policy; prior data is needed so that
claim analyses are accurate [½]

Premiums (amounts / timing) needed for reserving, pricing and monitoring [½]

Commission to check that correct payments are made [½]

© IFE: 2019 Examinations The Actuarial Education Company


SP7: Assignment X2 Solutions Page 7

Rating data

The following data items will be needed so that premiums can be determined. This data will also
be necessary for subsequent claims experience or rating exercises. [½]

Cover comprehensive, third party fire and theft, third party only [½]

Type of use SDP or business or domestic [½]

Details on driver(s) name


date of birth
sex
accidents
convictions
other policies held by principal driver
occupation [½ each, maximum 2]

NCD for this policy number of years no claims’ discount, or protected NCD [½]

Excess compulsory, or voluntary [½]

Details on car make, model and engine capacity


vehicle rating group
year of manufacture / registration
value
garaged / off-street parking / street parking
modifications [½ each, maximum 2]

Claims data is not required here, but students should mention that they assume that the claims
data is stored in a separate file, with file references / links from the policy file. [1]
[Maximum 8]

Solution X2.7

Claims analyses are discussed in Chapter 13, Actuarial investigations and analyses.

Problems with claims analysis for motor:


 choice of a suitable base period
 any changes to policy conditions, such as the size of the excess
 if it is a small company, then they may not have enough data
 cleanliness of data (which can be difficult to achieve)
 how to handle large claims, eg definition and treatment
 different reserving methods should be used for large and attritional claims
 allowing correctly for the large claim threshold on older claims

The Actuarial Education Company © IFE: 2019 Examinations


Page 8 SP7: Assignment X2 Solutions

 any secular trends (eg more cars on roads) that will distort the analysis
 need to carry out an analyses by amount and frequency separately
 the number of rating factors to use – using all of them may give some empty cells
 data should be split by primary rating factors, type of cover, and also by type of claim –
however, there may not be enough liability type claims for a credible analysis
 need to identify the level of correlation between factors (eg NCD and age)
 need to consider the effect of inflation on claim amounts over the period
 need to make sufficient allowance for IBNR claims, particularly for most recent years
 allowance should be made for the seasonality of claims, eg more may be expected in
winter
 dealing with nil claims
 dealing with multiple claims
 dealing with accumulations of risk
 dealing with indemnity costs separately from expenses
 dealing with reopened claims
 dealing with partial payments
 dealing with new types of claim
 dealing with reinsurance and other recoveries
 accuracy of outstanding claims estimates [½ each, maximum 10]

Solution X2.8

Valuation bases are discussed in Chapter 13 (Actuarial investigations and analyses) and
Chapter 14 (Reserving bases).

Strong, predominantly short tail

 There will be few practical implications because the business is largely short tail. [½]
 A cautious reserving basis will probably have less proportionate impact on the short-tail
classes. [½]
 The company will look less strong than it really is, but not to a large extent. [½]
 If business volumes are relatively constant, the declared profitability may be a fair
reflection of the underlying profitability (provided the reserving basis does not change).[1]

© IFE: 2019 Examinations The Actuarial Education Company


SP7: Assignment X2 Solutions Page 9

Expanding, writing liability business

 Profits will appear low … [½]


 … since deferred profits for expanding business will understate actual profitability. [½]
 Published solvency may look very low. [½]
 With long-tail business, cautious reserving can have a big impact on the size of the free
reserves. [½]
 Rapid expansion of premiums will add to problems with the solvency ratio. [½]
 There might be implications for investment strategy (low risk, low volatility assets may be
required) and reinsurance (more may be needed). [1]

Contracting, international

 Profits will be overstated … [½]


 … as there will be large releases of reserves from previous years’ business. [½]
 The published results of a diversified company such as this may appear out of line with its
competitors. [½]
[Maximum 7]

Solution X2.9

This material is covered in Chapter 13, Actuarial investigations and analyses.

Mix of policyholders

Selling the business via the internet may attract a different class of customer. [½]

Larger businesses may be more likely to continue to buy insurance through brokers, who can seek
the best cover for their particular company. [½]

Similarly, specialist businesses may be more likely to continue to buy insurance through brokers.
[½]

Conversely, the internet may be an acceptable sales channel for common professions that require
standardised cover. [½]

However, even companies insuring standard professions may require tailor-made cover, in which
case they may be more likely to continue to use brokers. [½]

Therefore the internet sales channel is likely to attract small, standard businesses and individuals
requiring standard insurance cover. [½]

Brokers may start to lose business to internet sales, so might provide incentives to customers in
order to retain their business. This could affect the mix of business by attracting a different class
of policyholder. [½]

The Actuarial Education Company © IFE: 2019 Examinations


Page 10 SP7: Assignment X2 Solutions

Lower expenses (ie commission) could reduce the premium charged to internet customers, which
could attract a different class of policyholder. [½]

While broker sales may need to be competitive, brokers may also focus on other aspects, such as
security of the insurer and their propensity to pay claims, whereas internet sales may be more
dependent on price. Therefore business sold through the internet may be more competitively
priced (with lower profit margins) than business sold through brokers. This may also attract a
different class of policyholder. [1]

Claim and lapse experience

Selling business over the internet is likely to affect the rating structure for professional indemnity
business. In particular, products may have to be simpler, with fewer rating factors. [½]

This could lead to increased anti-selection by policyholders, and so worse claims experience. [½]

Claims made over the internet may experience smaller delays in settlement than those that are
made via a broker. [½]

The lack of a face-to-face sales process may increase customer non-disclosure, which could either
increase claim rates or increase claim disputes. [½]

Less face-to-face contact could also increase the number of fraudulent claims. [½]

If brokers take an active role in filtering out fraudulent or invalid claims (which might turn out to
be nil claims), then the number of claims from internet business might appear relatively high. [½]

On the other hand, brokers could actively encourage their clients to claim on their policies, which
would work in the opposite direction. [½]

With internet sales, the insurer has some direct contact with the policyholder, so it may be able to
provide advice to the policyholder in order to avoid claims. For example, the insurer could send
out an advice pack with the policy documents. [½]

Customers buying cover over the internet are likely to initiate the sale themselves, which may
lead to reduced lapse rates; however this is unlikely to be significant for professional indemnity.
[½]
[Maximum 8]

© IFE: 2019 Examinations The Actuarial Education Company


SP7: Assignment X2 Solutions Page 11

Solution X2.10

This question is testing your understanding of Chapter 11, Risk and uncertainty and Chapter 13,
Actuarial investigations and analyses.

Internal factors

 selection error  eg the use of an inappropriate benchmark upon which to base your
premium rates / determine your reserves [½]
 model error [½]
 parameter error  ie misinterpretation of the data available [½]
 process error  ie experience is volatile from year to year [½]
 poor control of claim payments [½]

Mix of business

 anti-selection, eg if you use number of bedrooms rather than sum insured as an exposure
measure, or don’t rate by location (and other insurers do) [½]
 under-insurance leading to higher than expected claims per unit of exposure [½]
 accumulations of risk, especially if the insurer is regionally based [½]
 poorer mix of business than expected, eg by source [½]

External factors

 worsening trend in crime [½]


 policyholders submitting fraudulent or inflated claims eg during a recession [½]
 an increase in the propensity to submit claims [½]
 unusual weather patterns leading to increased subsidence claims [½]
 a storm or flood leading to a larger number of property damage claims than expected [½]
 an unexpected catastrophe, such as an earthquake, or terrorist attack [½]
 a demand surge (eg on builders, following a catastrophe) that pushes up the cost of
claims [½]
 higher than expected claims inflation [½]
 a court award leading to high liability claims [½]
 a change in regulation, eg a requirement to provide unlimited cover on certain claims [½]
 failure of a reinsurer, so that we make lower than expected recoveries [½]
[Maximum 7]

The Actuarial Education Company © IFE: 2019 Examinations


All study material produced by ActEd is copyright and is sold
for the exclusive use of the purchaser. The copyright is
owned by Institute and Faculty Education Limited, a
subsidiary of the Institute and Faculty of Actuaries.

Unless prior authority is granted by ActEd, you may not hire


out, lend, give out, sell, store or transmit electronically or
photocopy any part of the study material.

You must take care of your study material to ensure that it


is not used or copied by anybody else.

Legal action will be taken if these terms are infringed. In


addition, we may seek to take disciplinary action through
the profession or through your employer.

These conditions remain in force after you have finished


using the course.

The Actuarial Education Company © IFE: 2019 Examinations


SP7: Assignment X3 Solutions Page 1

Assignment X3 Solutions

Solution X3.1

This question tests your understanding of the material in Chapter 15, Triangulation methods.

Check that the total claim payments from the claims department are the same as the total claim
payments from the accounts department. [1]

Check that the data is consistent with the data provided in prior years, eg: [½]
 claims expenses included [½]
 net or gross of reinsurance [½]
 similar treatment of reopened claims, etc. [½]

Carry out simple checks to see whether there are any obvious anomalies in the data, eg check
that the claim payments fit into the existing triangle in a regular manner. Consider the credibility
of the X/0 entry compared with the X–1/0 and X–2/0 entries. (Calculate each as a proportion of
earned premium for the year.) Investigate any anomalies. [1]

Investigate whether there have been any changes in claims handling processes, eg postal strikes,
staff shortages or new systems. [1]

Investigate (ie by requesting details on):


 large claim settlements (eg any claims over £X, where X depends on class) [½]
 catastrophe events [½]
 changes in policy conditions which impact on the speed of claim settlement [½]
 changes in admin procedures, internal and external strikes etc. [½]

Make sure that the triangle is fully run off, eg consider how old the oldest unsettled claims are
and the incidence of IBNR and re-opened claims at the longest durations. [1]
[Total 8]

Solution X3.2

The Core Reading for this question is covered in Chapter 14, Reserving bases.

Advantages of discounting:
 a more realistic picture of solvency is presented [½]
 if the reserves are not discounted it can lead to excessive prudence … [½]
… particularly for long-tail classes [½]
 it allows different classes of business to be compared more easily. [½]

The Actuarial Education Company © IFE: 2019 Examinations


Page 2 SP7: Assignment X3 Solutions

Disadvantages of discounting:
 discounting may be viewed as a sign of weakness, leading to a lack of confidence from
policyholders, brokers, reinsurers, rating agencies etc [1]
 all other things being equal, it is less prudent to discount reserves [½]
 the emergence of profit is accelerated so tax will be paid earlier [½]
 it has little effect for short-tail business [½]
 there may be regulatory limits on discounting [½]
 it can be difficult to determine an appropriate discount rate. [½]
[Maximum 5]

Solution X3.3

The question is examining your understanding of the material in Chapter 14, Reserving bases.

Company A will want to form a view of the adequacy of Company B’s booked reserves. [½]

In order to determine this Company A is likely to use a best estimate basis as a starting point … [½]

… and then carry out sensitivity testing by changing key assumptions. [½]

When formulating a bid, Company A will want to incorporate a degree of prudence into its
estimate of Company B’s liabilities. [½]

This will result in a lower value being placed on Company B. [½]

The starting point for the negotiations are likely to be the figures in Company B’s published
accounts. These are likely to be close to best estimate. [½]

How far the basis moves between Company A’s preference for a prudent basis and Company B’s
preference for an optimistic basis will depend upon who has the most bargaining power in the
negotiations. [1]
[Total 4]

© IFE: 2019 Examinations The Actuarial Education Company


SP7: Assignment X3 Solutions Page 3

Solution X3.4

The Core Reading for this question can be found in Chapter 15, Triangulation methods.

(i) Postal strike

The strike should have no effect on the occurrence dates, but will lead to some additional
reporting delays. [1]

Even if other forms of communication were used, the net effect would still be for claims reporting
being slower than if the postal strike had not happened. [1]

Overall, there might be a little delay of claim settlement from the 2017/0 cell to the 2017/1 cell of
the run off triangle. The key factors are: [½]
 few of the claims from November and December 2017 would be settled by
31 December 2017, irrespective of the strike [½]
 so the strike will not have much impact on the run off [½]
 the settlement process for other claims (eg from September and October 2017) may be
slower, and this may result in claims moving from 2017/0 to 2017/1. [½]

The effects between other cells would probably be quite marginal (ie deferrals from 2016/1 to
2016/2 should be relatively minor). [½]

For the reserving as at 31 December 2018, the main effect will be a small overstatement of the
01 development factor, and hence a slight overstatement of the reserves for the 2018 origin
year. [1]
[Maximum 5]

(ii) Nil claims

This should have no direct impact, provided the changes to settlement and reporting affect nil
claims only. [1]

The Actuarial Education Company © IFE: 2019 Examinations


Page 4 SP7: Assignment X3 Solutions

Solution X3.5

This question tests your understanding of Chapter 15, Triangulation methods.

(i) Incurred but not reported reserve

An amount of money held to meet claims for events that the insurer believes have already
happened but the insurer has not yet been notified of. [1]

(ii) Number of IBNR claims

Use the delay table method to estimate the number of IBNR claims expected, assuming that the
table given is appropriate for predicting future reporting patterns. [1]

Month of No of claims Percentage Ultimate no Number of


claim event reported by developed of claims IBNR claims
(2018) 31/12/2018 expected
January 624 1.000 624 0
February 723 1.000 723 0
March 782 1.000 782 0
April 941 1.000 941 0
May 771 1.000 771 0
June 848 0.995 852 4
July 921 0.975 945 24
August 1,186 0.950 1,248 62
September 1,318 0.900 1,464 146
October 1,280 0.800 1,600 320
November 787 0.625 1,259 472
December 190 0.250 760 570
[4]

Hence the estimated number of IBNR claims at 31 December 2018 is:


4  24  62  146  320  472  570  1,598. [1]
[Total 6]

© IFE: 2019 Examinations The Actuarial Education Company


SP7: Assignment X3 Solutions Page 5

(iii) Estimating the reserve for IBNR

Markers: Parts (iii) and (iv) overlap. Use some discretion, but only give credit once for each distinct
idea.

Part (ii) gives the number of IBNR claims as at the end of the year. We need to obtain the
estimates for the average cost per IBNR claim (ACPC). [½]

Multiplying the number of claims by the average cost per claim gives a basic figure for the IBNR
reserve. [½]

Consider reducing this to reflect the delay to settlement, during which investment income may be
earned. However, household has very short delays so this is unlikely to be significant. [1]

ACPC for IBNR claims are unlikely to be the same as for reported claims. Small, financially
insignificant claims may take longer to be reported, but big claims may take longer to emerge. [½]

There is also a need to allow for other factors that may affect either the speed at which claims are
reported or the ACPC, eg future inflation, other known trends, changes in the mix of
business. [1]
[Maximum 3]

(iv) Reasons why the reserve is only approximate

Markers: scale down the marks if the student gives only general comments not related to the
household business written.

The delay table method models random events. It is possible to attach a confidence interval to
the estimate, but it will only be exactly right by chance. [1]

It assumes that the past delay data will be a good guide to that in the future. [1]

There are seasonal factors, for example, Christmas may delay reporting of claims and low staff
numbers working over the holiday period may also delay recognition. [1]
Changes in administration or claims reporting procedures may alter the speed of reporting, eg a
new telephone helpline would speed up the pattern. [½]

Catastrophic events, eg storm damage (quite possible at this time of year), may have different
reporting patterns to other claim events. This should be adjusted for. [½]
[Total 4]

The Actuarial Education Company © IFE: 2019 Examinations


Page 6 SP7: Assignment X3 Solutions

Solution X3.6

This question is testing the material in Chapter 15, Triangulation methods.

(i) Advantages of the basic chain ladder

The results with BF will be distorted if the independently-estimated loss ratio is wrong. [½]

If the data is complete and stable, then the BCL will work well, without the need to estimate a loss
ratio. [½]

The method is simpler and easier. [½]

Advantages of Bornhuetter-Ferguson

The results are more stable from year to year. [½]

This is particularly useful if the data is scanty or there are significant changes from year to
year. [½]

The results are less sensitive to the tails of the BCL triangle. [½]

Full data is not required, so this is useful for quick approximate valuations, or for frequent
valuations. [½]

Unlike the BCL, the method can make allowance for what is known about the outstanding claims
(ie in the choice of loss ratio for the year). [½]
[Total 4]

(ii) Reasons for difference

The assumptions underlying the BF method may be inappropriate:


 future inflation is not a weighted average of past inflation [½]
 downward development within the triangle [½]
 inappropriate independent loss ratio [½]
 unstable development of claims eg: [½]
– changes in mix of business
– changes in reporting and settlement procedures
– distortions from catastrophic claims or other one off events
– changes in tax rates affecting claim amounts
– postal strike [½ each, maximum 2]
 the chain ladder is not fully run off or a tail factor cannot be estimated. [½]

The method / assumptions used by the independent assessor may be inappropriate:


 the method may have ignored IBNR [½]
 the assessor may be biased [½]

© IFE: 2019 Examinations The Actuarial Education Company


SP7: Assignment X3 Solutions Page 7

 there may be an explicit inflation assumption, which is not appropriate [½]


 the assessor may have mis-read or misinterpreted the file. [½]

Other possibilities include:


 insufficient or incorrect data [½]
 inappropriate data groupings [½]
 difference of opinion between the actuary and assessor on the outcome of a certain
category of claims, eg subject to a court ruling [½]
 estimators have more information and so can provide a more accurate answer. [½]
[Maximum 7]

Solution X3.7

UPR is discussed in Chapter 15, Triangulation methods.

(i) Reasoning behind the formula

The definition of the claim ratio, CR, is usually ‘incurred claims to earned premium’, for the latest
financial year. [½]

UP is the unearned premium at the last year end, ie the proportion of the previous year’s written
premium in respect of cover for this and future years. [½]

Thus, subject to the adequacy of the claim ratio for last year applying equally this year, CR x UP
will be the amount of claims incurred in future years in respect of premiums received last year. [½]

By subtracting the reserve already set up (the UPR) we are left with the additional amount for
unexpired risk (if greater than zero). [½]

This is subject to the UPR used above being the same as the unearned premium reserve in the
returns, and the denominator for the CR being consistent with the calculation of UP. [1]
[Total 3]

(ii) Advantages and disadvantages

An advantage of such a formula is that it is simple and, on the face of it, objective. These features
are desirable if the formula is to be used for statutory purposes. [½]

Arguably, the fact that there is no scope for judgement is also desirable. [½]

However, there is a fundamental dependence on the CR estimate being reasonable. This assumes
that the past is a good guide to the future. [½]

For example, a catastrophe may lead to an increase in outstanding reserves, and the formula
would assume that this will happen again. This may not be reasonable. [½]

The Actuarial Education Company © IFE: 2019 Examinations


Page 8 SP7: Assignment X3 Solutions

We also have taken the claim ratio to be incurred claims to earned premiums – which in some
cases will include changes in claims estimates for business written some years previously
(especially in liability and other long-tail business). The ratio will therefore be distorted by
movements in prior years’ reserves. [1]

The incurred claims for last year will also have a large component of estimate for outstanding
reported claims and IBNR for some classes of business. Thus, the estimate for additional amount
for unexpired risk is itself based on estimates. [1]

We must also be careful with a number of other factors:


 The AURR should have a minimum value of zero. [½]
 The figures should be net of reinsurance. [½]
 Care should be taken when allowing for expenses. The claim ratio may well be less than
100%, but the premiums may be inadequate (due to part of the premium being used to
cover expenses). [½]

We should specify how the claim ratio will be calculated, eg:


 The ratio may be calculated for the whole company or it may be split by class. We need
to decide how much cross-subsidy is allowable across classes. [½]
 The ratio may allow for industry experience or it may be based solely on each individual
company’s experience. [½]

Where estimates of outstanding reserves are high (ie conservative), the CR will be high, leading to
over-prudence in the estimate of additional unexpired risk reserve. [½]

More dangerously, the converse situation (under reserving) would lead to an inadequate
allowance for the additional unexpired risk reserve. [½]

If an insurer cuts its rates during the year the claim ratio may be acceptable, but the new rates
may be inadequate, in which case the formula would lead to under reserving. [1]

In the same way, the reserve will be unnecessarily large if the experience last year included a
large one-off claim. [½]

A further source of error might be a period of high inflation, with under provision for increasing
claims in the premium rates. [½]

If premium rates are altered, then previous claim ratios may no longer be appropriate. [½]

Other changes and distortions (eg mix of business, policy conditions, large claims, changes in
reinsurance, inflation, etc) will also tend to invalidate the appropriateness of the formula. [½]

Overall opinion, therefore, is that despite the intuitive correctness of this formula there are too
many other factors that cannot be overlooked. Blind use of this formula could lead to misleading
results, and should therefore be opposed by the industry. [1]

© IFE: 2019 Examinations The Actuarial Education Company


SP7: Assignment X3 Solutions Page 9

Another factor weakening the case for a statutory additional amount for unexpired risk is that it is
usually an insignificant amount, and where it would threaten solvency, other parts of the
statutory returns would show that there is a problem. [1]
[Maximum 12]

Solution X3.8

The material for this question is covered in Chapter 15, Triangulation methods.

Markers: Give full credit to any method giving exactly these answers. Several method marks
should be available to students attempting a valid approach, even if their final answers are wrong.
Be harsh, however, on wrong attempts where the logic is unclear.

(i) Total reserve

1 2 3 4 5 after 5
Proportion of claims
paid in development
10% 20% 25% 15% 10% 20%
year
Proportion of claims
outstanding at end of
90% 70% 45% 30% 20% 0
development year

Accident Expected Expected Expected


Premium
ultimate loss ultimate proportion Reserve
year earned
ratio claims outstanding
2014 6,500 125% 8,125 20% 1,625
2015 7,000 140% 9,800 30% 2,940
2016 8,500 120% 10,200 45% 4,590
2017 8,000 130% 10,400 70% 7,280
2018 9,200 160% 14,720 90% 13,248
Total 29,683
[5]

For example: 9,800 = 7,000  140%


2,940 = 9,800  30% (from table above)

The Actuarial Education Company © IFE: 2019 Examinations


Page 10 SP7: Assignment X3 Solutions

(ii) Revised loss ratios

Accident Premium Reserve Paid claims Revised Revised


year earned to date expected expected
ultimate ultimate
claims loss ratio
2014 6,500 1,625 6,607 8,232 127%
2015 7,000 2,940 6,408 9,348 134%
2016 8,500 4,590 5,132 9,722 114%
2017 8,000 7,280 4,138 11,418 143%
2018 9,200 13,248 1,800 15,048 164%
[3]

For example: 9,348 = 2,940 (from (i)) + 6,408 (from cumulative triangle)
134% = 9,348 / 7,000

Solution X3.9

This question combines the material in Chapter 15, Triangulation methods, with your knowledge
of insurance products, from Chapter 3, Insurance products  types.

(a) Private motor

This can be a large class with a high claim frequency. Statistical methods will be most suitable,
eg Bornhuetter-Ferguson or chain ladder techniques. [½]

Case estimates for property damage and small bodily injury claims would be costly and slow, and
would provide little, if any, extra accuracy. [½]

For the larger bodily injury claims, potential large claims and claims in the later development
years, case estimates may be appropriate. [1]

Problems include:
 practical issues of handling the large volume of data involved [½]
 the fact that claims need to be analysed into a number of different claim types to reduce
heterogeneity (eg own damage, third-party damage, bodily injury, windscreen only, zero
claims), this can reduce the credibility of data in each group. [1]

(b) Mortgage guarantee

The risks are dominated by economic factors and are therefore cyclical in nature. Triangulation
methods, such as the chain ladder method, are therefore inappropriate. [½]

A stochastic method is likely to be appropriate. [½]

© IFE: 2019 Examinations The Actuarial Education Company


SP7: Assignment X3 Solutions Page 11

Case estimates are limited by the lack of data ... [½]

… since the detailed data relating to individual borrowers will be held by the mortgage provider
rather than the insurer. [½]

Also, claims are not technically ‘incurred’ until a repossessed property is sold and the amount of
the loss (if any) is known. So, the definition of ‘claim’ is important in splitting reserves between
outstanding reported, IBNR, etc. [1]

(c) Employers’ liability

This is a large class of business, so the volume of data available justifies statistical methods for the
bulk of minor claims. [½]

This is a long-tail class with significant numbers of claims taking many years to settle. For
potential large claims, case estimates will be appropriate, since the extra specific data available
for these less predictable claims can be expected to lead to more accurate estimates. [1]

There can be extensive reporting delays in this class (eg it can take many years for industrial
illnesses such as mesothelioma to materialise and be diagnosed), so IBNR can be significant.
Statistical techniques are required for IBNR estimates. [½]

Problems include:
 The possibility of new, as yet unknown types of claims emerging, eg new industrial
disease claims. These cannot be accurately quantified. [1]
 Potential large claims need to be studied in detail to provide accurate case estimates and
case estimators will need to take into account recent trends in damage awards made by
the courts. [1]
[Maximum 9]

The Actuarial Education Company © IFE: 2019 Examinations


All study material produced by ActEd is copyright and is sold
for the exclusive use of the purchaser. The copyright is
owned by Institute and Faculty Education Limited, a
subsidiary of the Institute and Faculty of Actuaries.

Unless prior authority is granted by ActEd, you may not hire


out, lend, give out, sell, store or transmit electronically or
photocopy any part of the study material.

You must take care of your study material to ensure that it


is not used or copied by anybody else.

Legal action will be taken if these terms are infringed. In


addition, we may seek to take disciplinary action through
the profession or through your employer.

These conditions remain in force after you have finished


using the course.

The Actuarial Education Company © IFE: 2019 Examinations


SP7: Assignment X4 Solutions Page 1

Assignment X4 Solutions

Solution X4.1

The Core Reading for this question is covered in Chapter 17, Assessment of reserving results.

The actuary should consider applying a range of diagnostic tests to check for trends in claims
experience and the reasonableness of the data. [1]

For example, paid claims divided by incurred claims, for each year of account. [½]

These diagnostics should extend to considering the claims development pattern. [½]

The actuary should carry out an analysis of emerging experience. [½]

The actuary should carry out an overview of the whole exercise for reasonableness, checking
there are no gaps in the analysis. [½]

The actuary should bring their own judgement into play … [½]

… taking care if their own experience of this type of business is not extensive. [½]

Peer review may be used in this situation. [½]


[Maximum 4]

Solution X4.2

The Core Reading for this question is covered in Chapter 18, Reserving  use of ranges and best
estimates.

(i) Three ways of deriving scenarios

 based on historical events [1]


 using judgement to determine hypothetical events [½]
 from the results of a stochastic model [½]
[Total 2]

(ii) Eight scenarios that might be modelled

 single large claims such as the repossession of a property with a high sum insured
 an economic downturn leading to multiple large losses, eg an increase in business failures
leading to increased repossession rates on large properties
 poor attritional claims experience, eg very high interest rates or a crash in the housing
market leading to the repossession of many small properties
 failure of a reinsurer
 fall in interest rates (if discounting reserves)
 higher inflation levels than expected

The Actuarial Education Company © IFE: 2019 Examinations


Page 2 SP7: Assignment X4 Solutions

 higher expenses than expected, eg due to poor management controls


 adverse exchange rate movement, leading to a deterioration in experience when stated in
the domestic currency
 latent claims, eg the unintended omission in the terms and conditions of a cap on the
maximum loan to value ratio. This could lead to higher-risk lending on the part of the
insured, and a consequent deterioration in claims experience.
[Markers give credit for other sensible examples.]
[½ mark per example, maximum 4]

Solution X4.3

This question is testing the material in Chapter 18, Reserving  use of ranges and best estimates.
It also builds on the material in Chapter 11, Risk and uncertainty.

(i) Model uncertainty

This arises because the model is necessarily a simplification of the real life situation. [½]

Such simplification introduces unknown bias into the model and therefore uncertainty over the
validity of the results. [½]

This uncertainty can be reduced by exercising sound actuarial judgement when choosing a
model. [½]

This involves selecting the model that best reflects the key features of the underlying process. [½]

If there is sufficient data then model error can be reduced by dividing data into homogeneous
groups … [½]

… and by testing whether the selected model is an appropriate fit to the data … [½]

… but there is not always sufficient credible data to enable this. [½]
[Maximum 3]

(ii) Parameter uncertainty

This is the uncertainty that arises when choosing the parameters for the model. [½]

It is a result of the statistical variability in the past data used as a starting point for setting the
parameters … [½]

… for example due to the presence in the past data of an unusually large claim. [½]

Parameter uncertainty can be reduced by exercising actuarial judgement when selecting


parameters … [½]

… although this uncertainty can never be removed completely. [½]


[Maximum 2]

© IFE: 2019 Examinations The Actuarial Education Company


SP7: Assignment X4 Solutions Page 3

(iii) Process uncertainty

This is the uncertainty that arises because of the inherent randomness of the process being
modelled. [½]

It would exist even in the absence of model and parameter uncertainty. [½]

The process uncertainty is reduced if modelling large numbers of reasonably independent risks
… [½]

… but will be unavoidable when modelling small numbers of correlated risks. [½]
[Total 2]

Solution X4.4

This topic is covered in Chapter 16, Stochastic reserving models.

Correlations between lines of business

Lines of business can be correlated because:


 they are impacted by similar events, eg a windstorm could impact both household and
commercial property accounts
 legal changes can affect several lines of business, eg a change in the discount rate
required to calculate bodily injury awards would affect both employers’ liability and
motor classes
 the same claims team may handle claims from several lines of business and so changes to
claims handling processes may impact on more than one line
 problems with data may affect more than one line of business.
[Markers, award ½ each for any distinct reasons, up to a maximum of 2.]

Allowance for correlations in a stochastic model

A correlation matrix could be used ... [½]

... however this only allows for very simple dependencies between classes. [½]

A copula is a more flexible approach, because it allows for multiple dependencies. [½]

For example, the Gumbel copula and the t-copula allow for increased correlations in the tails of
the distribution. [½]

However, it can be difficult to parameterise dependencies from data, so judgement is important.


[½]

The Actuarial Education Company © IFE: 2019 Examinations


Page 4 SP7: Assignment X4 Solutions

The user must specify:


 underlying loss distributions for the classes of business [½]
 a two-way correlation matrix between all distributions [½]
 the form of the copula. [½]
[Maximum 4]

Solution X4.5

The Core Reading for this question is covered in Chapter 18, Reserving  use of ranges and best
estimates.

The key points to consider when communicating a best estimate reserve are:
 Who the recipients are, their level of knowledge and how they will use the information. [1]
 How we get across that the best estimate is just an estimate … [½]
 … that there are other possible reasonable estimates … [½]
 … that the actual result is likely to be different from the best estimate. [½]
 How we clarify that the best estimate is just the mean of the distribution ... [½]
 … it will not necessarily reflect the most likely outcome … [½]
 … particularly if the distribution is positively skewed. [½]
 The need to communicate the key assumptions made in deriving the best estimate. [1]
 The need to communicate the sensitivity of the results to changes in the key assumptions.
[½]
 The need to adhere to any professional guidance. [½]
 The need to comment on the main restrictions or shortcomings in the analysis, eg: [½]
– lack of data [½]
– restrictions in the scope of the work [½]
– lack of information on company operations. [½]
[Maximum 7]

Solution X4.6

The stochastic form of the Bornhuetter-Ferguson method is discussed in Chapter 16, Stochastic
reserving models. This question also builds on the material in Chapter 15, Triangulation methods.

The deterministic form of the Bornhuetter-Ferguson method combines two methods by


calculating reserves as a weighted average of the results from the chain ladder method and the
expected loss ratio method. [½]

The rationale is that, if the run-off pattern remains unchanged, a projection of the past claims
triangle should provide a good estimate of future claims. [½]

© IFE: 2019 Examinations The Actuarial Education Company


SP7: Assignment X4 Solutions Page 5

However, this estimate can be improved if we take into account an independent estimate of the
ultimate losses and apply it to the future development of losses. [½]

The independent estimate is usually calculated by applying an assumed loss ratio to the
premiums, which gives an estimate of the ultimate claim amount. [½]

Deterministic reserving methods assume that the observed claim amounts (in the triangle)
conform to a statistical model involving parameters that have fixed but unknown values. [½]

Once the data values have been established, the calculations could be carried out entirely
mechanically, requiring no subjective judgement (although, in practice, it is likely that some
adjustments would be made in the basic chain ladder method). [½]

The stochastic form of the Bornhuetter-Ferguson method is also a weighted average of the chain
ladder method and the expected loss ratio method, but it uses a Bayesian approach. [½]

With a Bayesian method, we assume that the parameters in the model do not have fixed values,
but conform to a certain prior distribution. [½]

To apply this method, we combine an assumed prior distribution for the parameters with a model
for the development of the claims to find the posterior distribution for the parameters. The
posterior distribution combines our initial beliefs about the parameter with the additional
information provided by the data. [½]

The posterior distribution can then be used (analytically) to calculate moments and percentiles for
the reserves. [½]

This method makes various assumptions, in particular, that the incremental claims are
independent and identically distributed with positive values and with variances proportional to
the mean. [½]

Determining the posterior distribution often requires a Monte Carlo approach. [½]

This method has a subjective element in that a suitable prior distribution must be selected. [½]

The Bayesian (stochastic) version can be considered to have two extremes, depending on the
prior distribution assumed. [½]

If a very ‘tight’ (low variance) prior distribution is assumed, the stochastic method makes very
little allowance for variability in the exposure measure. [½]

In this case, a very tight posterior distribution is obtained and the results are almost equivalent to
the deterministic Bornhuetter-Ferguson method. [½]

If a very ‘vague’ (high variance) prior distribution is assumed, the stochastic method largely
disregards the exposure measure. [½]

In this case, a very vague posterior distribution is obtained and the results (as determined by the
mean of the distribution) are almost equivalent to the deterministic method used for projecting
the claims triangle (which is usually the chain ladder method). [½]
[Maximum 7]

The Actuarial Education Company © IFE: 2019 Examinations


Page 6 SP7: Assignment X4 Solutions

Solution X4.7

This question tests your knowledge of the material in Chapter 19, Investment principles and ALM.

(i) Asset-liability model

Investment strategy

We will need an objective / target, for example: ‘to maximise return subject to a probability of not
meeting any statutory minimum solvency margin of no more than 1% in any of the next ten future
years’. [1]

Details of the current asset portfolio will be required, in order to check its suitability. This will be
the first asset mix to be run through the model. [½]

Investment rules / guidelines will need to be set, for example the company may impose a
maximum percentage holding in equity, under particular economic conditions. [½]

The model can be deterministic or stochastic … [½]

… if stochastic then an economic scenario generator may be used to produce interest rates,
inflation rates etc, which are then fed into the ALM. [½]

Assets

In order to model assets, we need to estimate parameters for:


 the mean and variance of returns, by asset class [½]
 income and capital growth, where appropriate [½]
 interactions, eg between inflation, investment return and dividend growth. [½]

Liability outgo

To model liabilities, we need to consider the claim settlement pattern for future years. So we
need:
 to look at the liabilities by class and claim type [½]
 a run-off pattern for each cohort of business mentioned above [½]
 an allowance for IBNR [½]
 an allowance for claims from business written but not yet earned [½]
 an allowance for other factors such as reopened claims. [½]

An allowance should also be made for expenses, split by fixed, variable, commission etc. [½]

Premiums and reinsurance recoveries should be offset from the liabilities. [½]

An understanding of the nature, currency and term of the liabilities is required. [1]

The interdependencies between correlated variables, both assets and liabilities needs to be
modelled. [½]

© IFE: 2019 Examinations The Actuarial Education Company


SP7: Assignment X4 Solutions Page 7

Future new business

A decision needs to be made as to whether to model expected future new business. If it is to be


included we need to consider future volumes of business by class. [½]

Output

Whether we use a deterministic or stochastic model, we will need to run the projection many
times to assess whether the asset distribution is robust to changing assumptions for example
different levels of inflation etc. [1]

The output will be summarised to derive information of key interest. [½]

Having run the model, if we find that the asset category is not robust, we will need to change the
distribution of assets between the main categories until we find a distribution that is appropriate.
[1]

This enables the analysis of optimal strategies. [½]


[Maximum 10]

(ii) Problems

For short-term classes, this gives a mismatch by term compared with the liabilities. [½]

For longer-term real liabilities, it may also be a poor match by nature if the loan stock is fixed
interest. [½]

Returns are expected to be lower over the long term compared with equity investment, so this is
a poor choice for shareholders’ funds. [½]

Market values can be volatile, particularly for longer-dated stock (although less so than equities,
for example), so there are still possible problems with statutory solvency if assets are valued at
market value. [½]

Possibly too much concentration in a single asset class. [½]

Lack of marketability if corporate loan stocks are used; this could be a problem if assets need to
be realised quickly. [½]

Lack of security if corporate loan stock is used. [½]

Lack of consistency with market practice. This may lead to a lack of confidence in the insurer,
which in turn may lead to rating downgrades, loss of business, expensive reinsurance etc. [½]
[Total 4]

The Actuarial Education Company © IFE: 2019 Examinations


Page 8 SP7: Assignment X4 Solutions

Solution X4.8

This material is discussed in Chapter 16 (Stochastic reserving models) and also draws on material
from Chapter 15 (Triangulation methods).

(i) Criteria

Alternative methods of allowing for reserve risk can be judged using the following criteria:
 the rationale underlying the method, and its validity
 the assumptions underlying the method, and their validity
 the shape of distribution of the loss distribution being modelled
 the shape of the tail of the distribution (ie large claims)
 the degree of security required by the insurer, shareholders and regulators
 how well it deals with different lines of business with different claims profiles
 how well it deals with portfolios of different sizes
 whether it makes suitable allowance for reinsurance arrangements
 whether it is acceptable to the insurer’s management, shareholders and policyholders
 whether it complies with insurance regulations.
[½ each, maximum 4]

(ii) Reserving methods

The following general points apply to these methods:


 they reduce the likelihood of holding inadequate reserves (relative to just holding the best
estimate)
 they look good to rating agencies, investors and regulators
 method (b) is more in line with international standards
 all defer profit and thus tax
 all provide some match to risk apart from (a)
 all help with other things, eg capital allocation.
[½ each]

(a) Best estimate plus 10%

The rationale here is that the extra 10% will provide a cushion in case claims exceed the best
estimate. [½]

The 10% appears to be an arbitrary figure that has been chosen because it is easy to understand
and calculate. [½]

No assumption about the shape of the loss distribution is required for this method. [½]

So this method can be considered to be less subjective than other methods. [½]

© IFE: 2019 Examinations The Actuarial Education Company


SP7: Assignment X4 Solutions Page 9

However, this method is indiscriminate in that it does not take into account:
 differences in the nature of the risks covered [½]
 the size of the portfolio [½]
 the levels of reinsurance being used. [½]

Theoretical arguments suggest that the size of the cushion should be linked to the standard
deviation or the percentiles of the distribution, not a fixed percentage. [½]

According to a Poisson model, the variation in the claims amount, as measured by the standard
deviation, should be proportional to the square root of the mean, rather than being a fixed
percentage. [½]

A margin of 10% is likely to be inadequate for a small portfolio where the variation will tend to be
large relative to the mean, but may be excessively cautious for a more stable portfolio. [1]

This method encourages better use of reinsurance. [½]

(b) 75th percentile

The rationale here is that a reserve based on the upper quartile, rather than the mean will provide
a cushion in case claims exceed the best estimate. [½]

This method is consistent with a value-at-risk approach. The reserve calculated using this method
can expect to be exceeded one year in four. [½]

Using the 75th percentile links the reserve amount directly to the loss distribution. [½]

It will therefore take into account differences in the nature of the risks covered and the size of the
portfolio. [½]

This method requires a good knowledge of, or an assumption to be made about, the form of the
loss distribution. [½]

The method can allow for reduction in claim volatility due to reinsurance by basing the reserve on
the net claims distribution. [½]

(c) Best estimate plus one standard deviation

The rationale here is that the extra standard deviation will provide a cushion in case claims exceed
the best estimate. [½]

Theoretical arguments suggest that the size of the cushion should be linked to the standard
deviation or the percentiles of the distribution. [½]

However, using one standard deviation (rather than two, say) appears to be an arbitrary choice
that has been made because it is easy to understand and describe. [½]

One standard deviation above the mean may correspond to very different percentiles, depending
on the actual shape of the underlying distribution. [½]

So the method does not, in general, conform directly to a value-at-risk approach. [½]

The Actuarial Education Company © IFE: 2019 Examinations


Page 10 SP7: Assignment X4 Solutions

However, if claims have a normal distribution, this method is consistent with a value-at-risk
approach corresponding to a confidence level of (1)  84% . [½]

This method requires a good knowledge of, or an assumption to be made about, the form of the
loss distribution. [½]

The method can allow for reduction in claim volatility due to reinsurance by basing the reserve on
the net claims distribution. [½]
[Maximum 10]

Solution X4.9

Diagnostics are discussed in Chapter 17, Assessment of reserving results.

(i) Diagnostic

A diagnostic is a measure used to help with the interpretation of data or results and to help test
and verify the underlying methodology and assumptions. [1]

(ii) Value of using diagnostic tools

Diagnostic tools indicate whether experience has been unusually high or low. [½]

The actuary needs then to consider whether the value of the diagnostic lies within an expected
reasonable range or not. [1]

The actuary can then consider whether any unusual experience is a one-off so that no adjustment
is required. [½]

If this is not a one-off then the diagnostic may be indicating that a change to the reserving
methodology and/or assumptions will be required to obtain a more reasonable reserving
estimate. [1]

The diagnostics provide a tool to:


 indicate which areas of the portfolio should be examined more closely [½]
 give the actuary greater confidence in the results presented [½]
 aid in the justification of the figures produced. [½]
[Maximum 4]

© IFE: 2019 Examinations The Actuarial Education Company


SP7: Assignment X4 Solutions Page 11

(iii) Seven common diagnostics

Loss ratios [½]

These can be based on some or all of paid, outstanding, IBNR, incurred and ultimate claims,
dependent upon the situation. [1]

They can be an indicator of changing:


 premium rate strength [½]
 claims uncertainty [½]
 inconsistencies in the model [½]
 errors in the reserving process. [½]

Paid to incurred claims or case estimates to incurred ratios [½]

Changes in these diagnostics over time may reflect:


 changes in the strength of case estimates [½]
 a change in the underlying business [½]
 a change in claim settlement patterns [½]
 a change in the establishment and settlement of outstanding claims [½]
 the presence or absence of a large loss. [½]

Average outstanding case estimate [½]

This can highlight changes in the strength of case reserves. [½]

Ratio of IBNR to case estimates [½]

This diagnostic is an indicator of the level of outstanding unreported claims and their uncertainty
relative to reported claims. It can help provide a feel for the reporting delays associated with the
business. [1]

Survival ratio [½]

This diagnostic looks at how long a reserve or IBNR estimate will last if current paid or incurred
claim development continues at a given rate. [1]

A change in this rate can be an indicator of under or over-reserving or a change in the underlying
business. [½]

Claim frequency and average cost per claim [½]

These diagnostics can be an indicator of inflationary trends in claim costs and also trends in claim
frequency per unit of exposure. [1]

The Actuarial Education Company © IFE: 2019 Examinations


Page 12 SP7: Assignment X4 Solutions

Reinsurance to gross ratios [½]

If the reinsurance program does not change then we would expect ratios of gross, reinsurance
and net estimates to be relatively constant. [1]

Changes may reflect:


 changes in the amount of business retained / ceded [½]
 changes in the type of reinsurance used (eg proportional versus non-proportional) [½]
 changes to the reinsurance terms, eg deductibles, limits and reinstatements [½]
 changes in the underlying claims experience of the insurer [½]
 inconsistencies in treatment of gross and net claim estimates. [½]
[Maximum 13]

(iv) Amending assumptions following diagnostic tests

It may be that the student’s comment is correct; however the most important thing is firstly to
understand the underlying reasons for the observed change in the diagnostics. [1]

This will dictate what action, if any, is needed. [½]

For example, no action may be required if:


 this is a one-off … [½]
… although if statistical projection methods are used, care should be taken to ensure that
this one-off event is not implicitly assumed to reoccur in the future [½]
 the impact is not material [½]
 the diagnostics, although higher than in the past, still fall in an expected reasonable range.
[½]

If action is required then it may require changes to the assumptions and/or the reserving
methodology. [1]
[Maximum 4]

© IFE: 2019 Examinations The Actuarial Education Company


SP7: Assignment X4 Solutions Page 13

Solution X4.10

This question requires you to apply the concepts described in Chapter 19, Investment principles
and ALM.

(i) Student A’s suggestion

Superficially this argument seems reasonable but it is flawed because: [½]


 deliberately mis-matching assets and liabilities is a dangerous policy … [½]
 … so is treating future premiums as assets [½]
 a reduction in the growth of premium income could cause liquidity problems [½]
 long-term assets are more volatile than short-term assets, so an unfavourable fluctuation
could result in technical insolvency even if the assets did not actually need to be realised
[1]
 a deterioration in the claims experience will increase the need for liquidity and could
cause difficulties. [1]
[plus up to 1 bonus mark for other valid points, maximum 4]

(ii) Student B’s suggestion

This approach would eliminate any liquidity problems. [1]

However:
 this is not a reasonable suggestion as the company is not about to break up [½]
 the assets and liabilities will not be matched by term  the liability claims could take many
years to settle [1]
 liability claims will increase in line with earnings and court award inflation, the investment
policy makes no attempt to match this [1]
 investment returns are likely to be poor in the long term, so the shareholders would
suffer [1]
 it is inappropriate for free reserves where a more aggressive strategy can be followed. [½]
[plus up to 1 bonus mark for other valid points, maximum 4]

(iii) Student C’s suggestion

The investment may give high returns. [½]

However:
 the assets are made up of both insurance funds and shareholders’ funds and the
investment policy should reflect this (ie the strategy should not be directed entirely
towards the shareholders’ funds) [1]
 there is no attempt to match the term of the liabilities [½]

The Actuarial Education Company © IFE: 2019 Examinations


Page 14 SP7: Assignment X4 Solutions

 the higher yielding assets are also likely to be high-risk assets – poor experience could
cause insolvency [1]
 there may be liquidity problems. [1]
[plus up to 1 bonus mark for other valid points, maximum 4]

(iv) Student D’s suggestion

This is a more balanced approach attempting to match inflation-related liabilities with real
assets. [1]

However:
 it does not make any suggestion as to how the free assets and shareholders’ funds should
be invested [1]
 ignoring future premiums may be excessively prudent [½]
 additional liquidity may be needed to meet any increase in cashflow requirements. [1]

Index-linked bonds will be less volatile than equities. [½]


[plus up to 1 bonus mark for other valid points, maximum 3]

© IFE: 2019 Examinations The Actuarial Education Company


SP7: Assignment X5 Solutions Page 1

Assignment X5 Solutions

Solution X5.1

This question examines the Core Reading in Chapter 20, Capital modelling – methodologies.

(i) Requirements of a good model

A model should ideally meet the following requirements:


 the model being used should be valid, complete and adequately documented [½]
 the model chosen should reflect adequately the risk profile of the classes of business
being modelled [½]
 the parameter values used should be accurate for the classes of business, and
investments being modelled [½]
 the outputs from the model and the degree of uncertainty surrounding them should be
capable of independent verification for reasonableness and should be readily
communicable to those to whom advice will be given [1]
 the model should not be overly complex so that the results become difficult to interpret
and communicate or the model becomes too long or expensive to run [1]
 the model should be sufficiently flexible [½]
 the model should have all parameters clearly identified and justified [½]
 the model should be structured and documented so that it can be understood by senior
management and board members who do not have actuarial expertise. [½]
[Total 5]

(ii) Additional requirements for stochastic models

A good stochastic model should, in addition to all the ‘good model’ points in part (i):
 be rigorous (ie strictly applying to constraints or principles) and self-consistent [½]
 be sufficiently detailed to deal adequately with the key risk areas and capture
homogeneous classes of business, but not excessively complex [½]
 be capable of being run with changed parameters for sensitivity testing [½]
 use sufficient simulations to reduce the simulation error that could arise from the
model – this is likely to be at least 50,000 [½]
 have a robust software platform. [½]
[Maximum 2]

The Actuarial Education Company © IFE: 2019 Examinations


Page 2 SP7: Assignment X5 Solutions

Solution X5.2

Capital allocation methodologies are discussed in Chapter 22, Capital modelling – allowance for
diversification.

Percentile method [½]

The percentile method allocates the aggregate diversified capital requirement down to the
sub-portfolios with reference to a lower percentile than was used to determine the overall
requirement, or with reference to various percentile-defined layers. [1]

This prevents over-allocation to catastrophe-type business. [½]

Proportional method [½]

The proportional method allocates the aggregate capital requirement across all the sub-portfolios
of the business in proportion to the value of the VaR for each sub-portfolio. [1]

It may be necessary to scale the resultant allocation to ensure that the sum of the individual
allocations is equal to the whole. [1]

The proportional method does not allow for levels of correlation between sub-portfolios. [½]

Marginal last-in method [½]

A marginal approach considers the capital requirement (measured using VaR) of each
sub-portfolio separately, assuming it is the last sub-portfolio to be added. [½]

The overall capital is then allocated in proportion to the incremental capital amounts for each
sub-portfolio. [½]

Under a marginal last-in method, highly correlated sub-portfolios are allocated a relatively high
amount of capital. [½]

There are several variants of the marginal last-in method, for example the Shapley method. [½]

The Shapley method allocates capital with reference to an average of the marginal capital
requirements, assuming that the class under consideration is added to the overall portfolio first,
second, third and so on. [1]

It can be used for a small number of classes, but in practice it quickly becomes onerous when
more classes are modelled. [½]
[Maximum 5]

© IFE: 2019 Examinations The Actuarial Education Company


SP7: Assignment X5 Solutions Page 3

Solution X5.3

Part (i) of this question is based on the material in Chapter 21, Capital modelling – assessment of
capital for various risk types. Part (ii) is based on Chapter 20, Capital modelling – methodologies.

(i) Reasons for having different capital requirements

 the classes of business written may be quite different – different classes of business can
have quite different capital requirements because of the uncertainty and variability of the
risks [1]
 different reinsurance arrangements – the net positions of the two could be quite
different … [½]
… or alternatively, they may have similar reinsurance arrangements but with different
reinsurers, therefore the credit risk charge could be quite different [½]
 one may write a single line of business, and the other a number of different classes, or in
different geographical areas and will therefore benefit from diversification [½]
 the investment strategies of the two companies may be quite different [½]
 one company may be part of a wider conglomerate and therefore subject to group risk [½]
 they may have different assessments of the acceptable probability of ruin or the term
under consideration – these would lead to different assessments of the capital
requirements of the business [½]
 different internal controls and processes – each company will therefore incur a different
operational risk charge [½]
 different growth prospects [½]
 different capital model structures / bases [½]
 different statutory requirements [½]
[Maximum 5]

(ii) Information required

 classes of business written [½]


 territories in which business is written [½]
 gross and net written premium [½]
 claims ratio [½]
 details of exceptional claims [½]
 claims accumulation [½]
 material changes in cover offered and policy wording [½]
 reserving policy [½]
 changes in reserving – a material change in reserving policy would not affect the mean
results but it would affect the volatility [½]

The Actuarial Education Company © IFE: 2019 Examinations


Page 4 SP7: Assignment X5 Solutions

 expense ratio [½]


 details of any exceptional expense items [½]
 changes in reinsurance [½]
 type of reinsurance [½]
 failure of reinsurer or other third parties [½]
 investment strategy regarding the investment of technical reserves [½]
 rating agencies’ assessment [½]
[Maximum 4]

Solution X5.4

This questions doesn’t examine any one area of Core Reading specifically.

Note to markers: award marks for mentioning parties in either part of the question, provided
appropriate reasons are given.

(i) Parties that should be consulted

Pricing and reserving actuaries [½]

The pricing and reserving actuaries should be able to provide information on:
 the risks covered [½]
 how the risks may have changed over time [½]
 how the risks have been allowed for in the pricing of each contract. [½]

Underwriters and marketing executives [½]

These parties should be consulted to ensure that the cover provided by the policies in the
portfolio(s) being modelled is fully understood. [½]

Claims managers [½]

Claims managers may also be consulted to ensure that the cover is understood. [½]

They may also be able to provide information on likely current and future claims through their
knowledge on: [½]
 future court precedents [½]
 possible legislative changes, eg changing Ogden rates [½]
 economic and social factors that may affect attitudes to claiming and the size of claims.[½]

Other department managers [½]

Other department managers around the company may be able to provide a qualitative
understanding of the nature and extent of different risks faced. [½]

© IFE: 2019 Examinations The Actuarial Education Company


SP7: Assignment X5 Solutions Page 5

Reinsurers [½]

Reinsurers may have expertise in modelling capital and so may be able to assist the insurer in
areas such as:
 the model used [½]
 the basis [½]
 setting assumptions, eg claims assumptions. [½]

Rating agencies [½]

Rating agencies will be concerned with the level of capital held by an insurance company. The
insurer may therefore request their input to ensure that its capital model is acceptable to the
rating agencies, in order to increase the likelihood of the insurance company obtaining the best
possible rating that it can. [½]
[Maximum 8]

(ii) Other parties that should be considered

Potential policyholders should be considered, since their motivations for purchasing insurance
should give an indication of the risks that will be taken on. [1]

The regulator should be considered, as changes in the regulatory environment can have a
significant impact on the capital requirements of a particular portfolio. Guidance issued by the
regulator will influence the design and parameterisation of the capital model. [1]

In addition, if the model is to be used to determine the company’s regulatory capital


requirements, then the regulator may need to approve the model first. [1]

The tax authority should be considered, since the level of capital held will affect the emergence of
profits, and hence the tax that is paid. [1]
[Maximum 3]

The Actuarial Education Company © IFE: 2019 Examinations


Page 6 SP7: Assignment X5 Solutions

Solution X5.5

Scenario tests are discussed in Chapter 20, Capital modelling – methodologies.

(i) Risks of IT system failure

The insurer’s internet site may be unavailable. This will result in the insurer being unable to
provide premium quotations for potential customers or accept new business sold through this
channel. [½]

Business sold through other channels is also likely to depend on IT systems to calculate premiums,
because household and motor products both tend to have many rating factors. The insurer will
therefore not be able to provide premium quotations. [1]

During the system failure period, the insurer may also have difficulty in processing new business
causing delays, eg in issuing policy literature. [½]

Similar issues may also affect business that is due to be renewed, meaning that policyholders may
lapse and take their business elsewhere instead. [½]

The above factors will increase the risk of insufficient levels of new business being sold, eg to
cover fixed costs. [½]

There will also be a potential loss of future business due to customer dissatisfaction and
potentially poor publicity. [½]

There may be delays in administering policies and in paying claims. This could cause problems if,
for example, policyholders need courtesy cars or temporary accommodation. [1]

In particular, the impact on claims is likely to be significant, because household and motor claims
are relatively frequent and many claims are short-tailed. [½]

In addition, the insurer’s brand awareness will suffer. For example, if the internet site is
completely closed, potential customers using this means of purchase will be less aware of the
insurer. [½]

Sales staff (eg brokers) and customers will be dissatisfied and may not feel as confident dealing
with this insurance company. [½]

Once the systems are up and running, there may be pressure on the insurer’s staff to catch up on
backlog of work, which may affect staff morale. [½]

If staff are over-worked, this will increase the risk of errors being made in processing the backlog.
[½]

© IFE: 2019 Examinations The Actuarial Education Company


SP7: Assignment X5 Solutions Page 7

The insurer is also likely to incur extra costs. In particular, to:


 fix the systems
 pay for extra staff to catch up on backlogs
 pay compensation to customers for poor standards. [½ each]

The system failure will distort the data used to manage the business. For example, premiums and
claims paid figures will be distorted, ie the claim development profile will show a lower than
expected claim development, and a subsequent higher than expected rise in notifications /
payments, as the backlog is processed onto the system. This will make trend analyses and
reserving more difficult. [1]

Functions of the insurer that rely on up-to-date data will not be able to do their jobs for a few
days, for example there may be delays in obtaining management information. [½]

Depending on when this information is required, this may make the company less able to respond
to market changes, such as reacting to competitors’ premium rate changes. [½]

Reinsurers may be less willing to do business and credit rating agencies may consider
downgrading the insurer. [½]
[Maximum 7]

(ii) Other factors influencing the capital requirements

 the extent and appropriateness of reinsurance in reducing the insurer’s uncertainty


 regulatory requirements, eg minimum solvency levels
 the strength of the reserving basis
 the level of diversification within the portfolio
 returns that could be made elsewhere (ie opportunity cost)
 influence of rating agencies
 management’s attitude to risk (eg desired ruin probability)
 levels of capital held by competitors
 expected future volumes from each type of business
 required profitability and dividend philosophy
[½ for each valid point, maximum 4]

The Actuarial Education Company © IFE: 2019 Examinations


Page 8 SP7: Assignment X5 Solutions

Solution X5.6

This question draws on the ideas discussed in Chapter 21, Capital modelling – assessment of
capital for various risk types.

(i) Main opportunities and risks to the subsidiary of the loan

Opportunities

The capital can be used to help the subsidiary to expand, eg:


 by meeting new business strain [½]
 by helping to develop new products [½]
 entering new markets by taking advantage of opportunities such as the acquisition of a
rival insurer. [½]

This loan is on better terms than would be available in the market, which would enable the
subsidiary to reduce its cost of capital and hence increase its return on capital for its
shareholders. [1]

Alternatively, it may enable the subsidiary to adopt a more aggressive pricing strategy in order to
gain market share … [½]

… or a more aggressive investment strategy to (further) increase expected returns. [½]

Risks

The main risk is that the loan is withdrawn by the parent company. [½]

This could happen if the group company has intra-group borrowing facilities or funding
arrangements in place. [½]

It also may be more likely for a multinational company, which may be exposed to significant
currency risks. [½]

In addition, if the multinational has exposure to emerging markets, then it may be exposed to
significant political risks. [½]

In this case, the subsidiary may be forced to find an alternative source of capital, which would
probably be on worse terms. [½]

The subsidiary may be forced to repay the loan at short notice. This could give rise to significant
liquidity risk if it had to realise investments in order to repay the loan. [½]

It would also restrict the expansion plan and could ultimately even threaten the subsidiary’s
solvency. [½]

Alternatively, the parent company could subsequently alter the terms of the loan. [½]

Again, this may lead to the subsidiary having to scale down its proposed expansion. [½]

Having a loan from the parent company now might restrict future parental loans later on. [½]

© IFE: 2019 Examinations The Actuarial Education Company


SP7: Assignment X5 Solutions Page 9

It may also restrict future borrowing from other lenders, in which case future sources of capital
may be restricted to equity. [½]

A loan on generous terms will be financially disadvantageous to the parent company, which could,
over time, threaten its solvency. This will have knock-on effects on the subsidiary. [½]

In particular, it is likely to damage the reputation of the subsidiary through association with the
(insolvent) parent company. [½]

The subsidiary may rely on the parent company in a number of ways eg through reinsurance
provided by the parent company, and the provision of centralised support functions. Therefore
accepting a loan may increase the concentration of risk. [1]
[Maximum 8]

(ii) Additional group risks

 reputation risk due to the association between the subsidiary and its parent company
(and other subsidiaries), which may be as a result of: [½]
– insolvency of another part of the group company [½]
– a credit downgrading of the parent company [½]
– bad press about the parent company [½]
 formal group reinsurance arrangements that rely on the existence of the parent
company [½]
 concentration of credit risk if the whole company is very dependent on one counterparty,
eg a reinsurer [½]
 the operational risk of centralised functions failing, eg marketing or IT [½]
 the subsidiary may have certain dependencies on the parent company, eg expertise of the
board [½]
 mismanagement at a group level, for example if risk is managed at an enterprise level,
then there may have been an over-allowance for diversification benefits (after all,
different insurance markets may be correlated to some extent) [½]
[Maximum 3]

The Actuarial Education Company © IFE: 2019 Examinations


Page 10 SP7: Assignment X5 Solutions

Solution X5.7

The components of a capital model are discussed in Chapter 20, Capital modelling 
methodologies.

Premium income

Future premium income will depend on both volumes of business and the level of premium rates.
[½]

In order to predict volumes of business, it may be necessary to consult with sales managers,
underwriters and senior management. [½]

While this may give some indication, volumes of business may be dependent on the future
environment – in particular, economic conditions and the insurance cycle, which are uncertain. [½]

They may also be affected by internal changes within the insurer, eg loss of underwriting staff
could impact the ability to meet planned premium income targets. [½]

Premium rates are predictable in the short term, and the insurer will have some control over
them in the long term. [½]

However, certain factors that affect premium rates will be outside the insurer’s control, eg the
insurance cycle. [½]

Both volumes of business and premium rates may be more predictable for certain lines of
business, and/or business sold through certain distribution channels. [½]

For example, volumes of personal lines motor insurance business in a country where such cover is
compulsory may be relatively stable and so fairly predictable. [½]

Claims

The predictability of future claims will vary depending on:


 the classes of business sold [½]
 types of claim, eg property damage, bodily injury [½]
 the timeframes over which claims are expected. [½]

Some business classes may have claims that are reasonably predictable. For example, personal
lines property-damage claims are likely to be more predictable than most liability claims. [½]

Claims may be split into:


 attritional claims – these may be reasonably predictable to some extent, based on an
adequate quantity of relevant, reliable data [½]
 large claims – these may be harder to predict, both due to their nature, and a lack of past
data on which to base them [½]

© IFE: 2019 Examinations The Actuarial Education Company


SP7: Assignment X5 Solutions Page 11

 catastrophe claims – these are extremely unpredictable, by the uncertain nature of most
catastrophes [½]
 latent claims – latent claims are claims that were not expected, so by definition, they are
very difficult to predict. [½]

At any point in time, there will be claims in different stages of development. In particular, claims
may:
 have been notified – these claims may develop from initial estimates, and are likely to be
reasonably predictable for certain classes of business (based on past experience) [½]
 have been incurred but not reported (IBNR) – less information is available on these claims
than claims that have been notified (in particular, the insurer doesn’t know that they have
occurred, or what event has occurred), so they are less predictable … [½]
… and this will also depend on the class of business and the volume of relevant data [½]
 be in respect of unexpired periods of risk – these claims have not yet occurred, so are
even less predictable than IBNR claims … [½]
… however the insurer will have information on exposure, so may be able to use loss
ratios to make some estimate of these claims [½]
 be in respect of future business – these claims will be hardest to predict and will be linked
to projections of future business volumes. [½]

In general, the further into the future the claims lie, the harder they are to predict. [½]

This is mainly due to uncertainty over the future external claims environment, eg in respect of:
 the economic environment, eg increased frequencies during recessions [½]
 trends in court awards, eg for liability claims [½]
 social trends, eg attitudes to claiming [½]
and changes within the insurance company, including changes in:
 policy terms and conditions [½]
 rating and underwriting [½]
 claims control processes. [½]

Expenses

Some changes are predictable as they are a function of other assumptions in the business plan,
eg commission is usually a percentage of written premium. [½]

Others are hard to predict, eg staff costs or rental, as they will depend on the projected business
growth and staffing plan. [½]

Short-term expenses are far more predictable than long-term expenses, both due to general
uncertainties, and expense inflation. [½]

The Actuarial Education Company © IFE: 2019 Examinations


Page 12 SP7: Assignment X5 Solutions

Investment income

Investment income is often hard to predict due to the typically uncertain nature of assets. [½]

This will depend on the assets held, eg the income on conventional government bonds is
relatively certain, whereas the return from equities may be very volatile. [½]

It will also depend on:


 investment prospects and expectations around the future economic environment [½]
 the current and projected future investment policy of the general insurer [½]
 expectations on premium and claims payment patterns, which impact the run-off of
reserves and investment assets. [½]

Investment income will be easier to predict in the short term because:


 current investments are known [½]
 the economic environment is more certain. [½]

Investment income is harder to predict in the long term because:


 the mix of business sold might change, leading to a different asset mix [½]
 the level of free assets might change, leading to a more / less aggressive investment
strategy [½]
 the investment policy of the insurer might change [½]
 the economic environment, including interest rates and inflation, is harder to predict [½]
 regulation on investment strategy might change. [½]
[Maximum 18]

© IFE: 2019 Examinations The Actuarial Education Company


SP7: Assignment X5 Solutions Page 13

Solution X5.8

The Core Reading for this question is covered in Chapter 21, Capital modelling  assessment of
capital for various risk types.

Credit risk

Counterparty credit risk is any risk to the liabilities through non-collection or failure of a
counterparty. The most common form of counterparty credit risk arises from reinsurance default.
[½]

Differentiation should be made between underwriting and reserving elements of reinsurance


credit risk:
 reserve credit risk is on a known asset, ie the risks have already been written and so are
known in this sense [½]
 underwriting credit risk is an uncertain security, ie the risks have not yet been written and
so the reinsurance recoveries (and consequent probability and size of default) are very
uncertain. [½]

There is likely to be a greater dependence between gross losses and credit risk for underwriting
risk than for reserving risk. [½]

Insurers should consider both the probability of default and the loss given default. [½]

They also need to consider the increased risk of reinsurance failure in extreme loss scenarios. [½]

In modelling terms this involves correlating reinsurance failure rates with large loss or catastrophe
scenarios. [½]

Where an insurer has a significant concentration of cover with an individual reinsurer it will need
to consider to what extent the Value-at-Risk measure adequately considers this concentration
risk. [½]

A suitable stress test to use for this risk might be the complete failure of a major reinsurer. [½]

Insurers should explicitly consider the duration of liabilities when considering the reinsurance
credit risk as there is a higher probability of default on a more distant recovery. Typically insurers
either apply a probability of default based on the expected mean term of the liabilities, or model
explicitly the rating for each reinsurer until each reinsurance recovery has been received. [1]

In addition counterparty credit risk should also consider:


 correlation between reinsurers … [½]
… a suitable stress test to use for this risk might be a one notch downgrade of all
reinsurers [½]
 non-recoveries due to reinsurance disputes, and the extent to which this is considered
within insurance and operational risk [½]
 any collateral held by the insurer in relation to specific counterparties – such as letters of
credit. [½]

The Actuarial Education Company © IFE: 2019 Examinations


Page 14 SP7: Assignment X5 Solutions

Insurance risk

To convert gross insurance risk to net insurance risk within a class, the reinsurance that can
contractually be recovered on large and catastrophe claims (and on attritional claims in the case
of proportional covers and working layers) should be deducted. [½]

Allowance should be made for disputes or exhaustion of cover, so it is preferable to calculate


reinsurance recoveries directly, rather than assume that historical net to gross ratios will remain
applicable. [½]

Depending on materiality, a firm may model future recoveries explicitly based on actual or
expected reinsurance covers, but use a net to gross ratio approach for prior claims. [½]

Where a reinsurance contract covers both future and prior business, we need to allow for any
reduction in cover due to prior reinsurance recoveries. [½]

Allowance should also be made for reinsurance of unexpired risk. [½]

If a firm has bought reinsurance on a losses-occurring basis rather than on a risks-attaching basis,
it should either:
 model the gross cost of risks unexpired at the end of the modelling period [½]
 make allowance for possible additional costs in renewing cover, especially in a post-loss
scenario. [½]

Liquidity risk

The impact of any loss may affect liquidity – particularly large or catastrophe claims. Following
such an extreme loss there may be delays in collecting reinsurance recoveries, which will increase
the liquidity risk to the insurer. [1]

Suitable stress tests that might be used include:


 an increase in attritional claims, which will increase the recoveries from proportional
reinsurance or working layer XL cover [½]
 a delay between a large loss and receipt of risk XL reinsurance recoveries [½]
 a delay between a catastrophe loss occurring and receipt of catastrophe XL reinsurance
recoveries [½]
 reduced level of new business, combined with longer than expected delays in recovering
claims from existing business and the associated impact on ability to pay claims. [½]

Operational risk

The operational risk is a possible lack of cover through mis-purchase of reinsurance. [½]

Operational risk is difficult to assess and significant judgement will be required. [½]

© IFE: 2019 Examinations The Actuarial Education Company


SP7: Assignment X5 Solutions Page 15

Group risk

A potentially significant area of group risk relates to formal group reinsurance arrangements. In
particular, there is a risk that the arrangements are not subject to the same contractual terms and
conditions of reinsurance agreements available in the market place. [½]

There may also be a risk of concentrations of credit risk. As above, this may be assessed by
assuming complete default of a key reinsurer. [½]

Strategic risk

A strategic risk may arise from an inadequate consideration of the appropriateness of the
reinsurance programme, considering the insurer’s business plan. [½]

This may be modelled stochastically, simulating results under varying types of reinsurance
programme. [½]

Alternatively, stress tests could be used, based on different reinsurance programmes. [½]

Political risk

An example of a political risk could be a reinsurance contract held with a foreign reinsurer, where
the government of that country subsequently takes control of the reinsurer and ceases to pay
claims to overseas companies. [½]

Political risk can be modelled by conducting a stress test to consider the appropriateness of the
reinsurance programme under different scenarios. [½]

Known examples should be used as a basis for any stress test. [½]
[Maximum 12]

The Actuarial Education Company © IFE: 2019 Examinations


Page 16 SP7: Assignment X5 Solutions

Solution X5.9

The communication of results is discussed in Chapter 23, Capital modelling - practical


considerations, as well as earlier in the course, in Chapter 18, Reserving – use of ranges and best
estimates.

(i) Information required

The information required will include:


 who the client is … [½]
… this will enable us to pitch our advice at the right level [½]
 the expertise of the insurer and the modelling software available to it … [½]
… this will affect the modelling methodology that is recommended [½]
 the purpose of the capital modelling exercise, eg whether it is determining its regulatory
capital requirement or its economic capital requirement … [½]
… this is likely to affect the recommended method and/or basis and the time horizon of
the exercise [½]
 the risk appetite of the insurer … [½]
… this will affect the strength of the basis recommended [½]
 the types / range of products the insurer will sell and the locations of these risks … [½]
… this will affect the methods used to determine capital requirements [½]
… eg business that is exposed to catastrophe risks may require the use of catastrophe
models [½]
 the volume of business the insurer expects to sell … [½]
… this, combined with the range of products it will sell, will affect the extent to which it
can allow for diversification effects … [½]
… and the extent of any offsetting capital requirement due to the cost of financing new
business volumes [½]
 whether the insurer will reinsure its business, and the extent of any reinsurance
arrangements … [½]
… this will affect advice on how reinsurance is allowed for in the capital requirements (if
at all). [½]
[Maximum 6]

© IFE: 2019 Examinations The Actuarial Education Company


SP7: Assignment X5 Solutions Page 17

(ii) Points to be made when presenting advice

It would be necessary to point out that the capital requirement will depend on a number of
things, including:
 the purpose of the exercise [½]
 the available data [½]
 the sensitivity of the result to the different assumptions including the correlation
assumptions. [½]

In particular, it is important to communicate that there is considerable uncertainty… [½]

… and that a range of results should be produced, based on different assumptions. [½]

The purpose of the exercise will affect the basis used. [½]

It may be useful to recommend a range of appropriate bases … [½]

… however it is important to point out that such bases will not necessarily be appropriate for
other uses. [½]

Since it is a new company, there will be limited data available. [½]

Any data used may lack relevance. It is important to highlight this, emphasising any specific
concerns over its quality and/or relevance … [½]

… and to point out that such issues may affect the validity of the result. [½]

The client should be made aware of any limitations of the advice given. [½]

Any actuarial judgement that may be needed should also be highlighted. [½]

It is important to emphasise that the final result is highly dependent on the assumptions made. [½]

The most significant assumptions will vary for different classes of business, however claim
frequency and claim severity are likely to be among the most important ones. [1]

Assumptions will be chosen based on the current economic, regulatory and tax environments,
however it is important to point out that if these change, then the assumptions will need to be
amended accordingly. [1]

It will be necessary to point out that correlation assumptions are also important, particularly in
future if the new insurer is planning to expand (either in terms of market share for existing classes
of business or into new business classes). [1]

Examples of possible correlations include:


 correlations within risk types, eg within any one class of business or between asset classes
when assessing market risk [½]
 correlations between insurance risk and market risk. [½]

The Actuarial Education Company © IFE: 2019 Examinations


Page 18 SP7: Assignment X5 Solutions

The advice should highlight the importance of continually monitoring and updating the
results … [½]

… and the need to comply with and monitor any regulation and/or guidance on setting capital
requirements. [½]
[Maximum 10]

© IFE: 2019 Examinations The Actuarial Education Company


SP7: Assignment X6 Solutions Page 1

Assignment X6 Solutions

Solution X6.1

This question tests your understanding of the material in Chapter 24, Determining appropriate
reinsurance, as well as building on your understanding of Surplus reinsurance, from Chapter 6,
Reinsurance products – types.

(i) Formula

EML  (1  l)  r [1]

(ii) Maximum loss

There is no maximum amount that the company could lose. Theoretically, losses are infinite. [½]

For anything above £0.5m, (net of quota share and surplus) the insurer only pays 10% of the
excess. There is reduced, but still unlimited, liability above this cut-off point. [½]

The size of the original, gross of reinsurance claim that would result in a net (of quota share and
surplus) claim of £0.5m is dependent upon whether the surplus treaty is used and the number of
lines used. [½]

A gross claim as small as £1.25m would result in a net claim of £0.5m if no surplus lines were
used, but a claim as high as £13.75m could also result in a net claim of £0.5m if the full 10 lines
were used. [1]

Assuming there is no inuring facultative reinsurance the largest risk that could be covered by the
surplus treaty is £27.5m ... [½]

... although it may be possible to cede larger risks through the surplus treaty and cover the
remaining amount (beyond the reinsurer’s maximum number of lines) ourselves. However, this
will depend on whether the terms of the surplus treaty allow us to do so. [½]

Although the maximum retention under the surplus treaty gives no guaranteed maximum that we
reasonably expect to lose, it does give a reasonable indication. [½]

In this case the maximum is £1m, net of the quota share treaty. [½]

If a claim for this ‘maximum’ of £1m did come in the insurer would pay £0.55m. [½]

All of these figures assume that the reinsurers stay solvent. If any of the three reinsurers default,
the amount could rise sharply. [½]
[Maximum 4]

The Actuarial Education Company © IFE: 2019 Examinations


Page 2 SP7: Assignment X6 Solutions

(iii) Comment on suggestion

The quota share arrangement does pay override commission … [½]

… but only to reimburse the insurer for the costs associated with setting up and servicing the
reinsured policies (eg underwriting, pricing, new business administration). [½]

So the ‘benefit’ of the commission is only to compensate for costs that the insurer has already
incurred. [½]

With excess of loss the insurer keeps all the profit that is loaded into its premium rates. [½]

However, the ‘cost’ of reinsurance is loaded into the premium paid. [½]

When deciding which is ‘best’ we must think about the benefits of reinsurance – not just the
costs. [½]

There could be other benefits, eg technical assistance, although this is more likely to come from
the broker rather than the reinsurer. [½]

The key benefit of excess of loss is to protect the company’s solvency (and stability of profits)
against large claims. [½]

Quota share would not be so effective in this respect, as claims are shared between the insurer
and the reinsurer, so you can’t really say that it is better. [½]

It is impossible to say which reinsurer is making the most profit. We should look at a
‘risk-adjusted’ profit. For example, the excess of loss reinsurer may be taking more risk. [1]
[Maximum 5]

Solution X6.2

The relevant material for this question is covered in Chapters 2 and 3, Insurance products and
Chapter 7, General insurance markets.

(i) Classic motor risk factors compared to standard motor

Mileage

Classic cars are not used to the same extent as a normal car. [½]

A classic car owner might own a standard car as well for commuting to work, shopping, etc. [½]

Hence risk will be lower. [½]

© IFE: 2019 Examinations The Actuarial Education Company


SP7: Assignment X6 Solutions Page 3

When car is driven

Standard cars tend to be used at various times of night, day and throughout the year. [½]

Classic cars are probably used more during the summer months (lower risk) and during daylight
hours at the weekend (lower risk). [1]

So classic cars have lower risk. [½]

Where car is driven

A classic car owner probably lives in an area of above average wealth (so lower risk), and drives in
more rural areas (also lower risk). [1]

So the risk is likely overall to be lower than standard. [½]

How fast car is driven

Classic cars are generally capable of lower speeds than standard cars. [½]

So they are likely to cause less damage to third party property than average. [½]

Hence lower risk. [½]

Safety features

Safety features of older cars will be less than modern ones, hence own damage may be worse for
an accident at a given speed (higher risk). [1]

Cost to replace car or parts

Some classic cars may be irreplaceable and others may need special parts to be made. This
compares to the ease of buying parts for current standard models. [½]

So, classic cars are probably more expensive to repair. [½]

Probability of theft

Classic cars may be more valuable than standard cars, making them higher risk. But they are
harder to sell on either whole or in parts than standard cars. So classic motors, are in general,
lower risk. [1]

Driver’s ability at avoiding accidents

Classic car owners put a lot of time and effort into their hobby. [½]

This is likely to make them far more careful of their car than the average standard motorist (lower
risk). [½]

Other drivers’ ability to avoid accidents

Difficult to tell, but the argument above may extend to other drivers of the vehicle. Also, the
owner is likely to be careful about who they authorise to drive the car. [½]

The Actuarial Education Company © IFE: 2019 Examinations


Page 4 SP7: Assignment X6 Solutions

Where kept overnight

Given the value, time and effort of restoration that often goes into classic cars, the majority of
them will be garaged when not in use. [½]

This compares to the general mix of garaged / on driveway / on the road, for standard motor
cover. [½]

Classic motors will be lower risk therefore. [½]

Cover level

Difficult to tell. Given the possible higher financial and emotional value of classic motors, this may
result in a higher proportion of fully comprehensive policies than the standard motor market. [½]
[Maximum 8]

(ii) New business strategy changes

For classic motors the approach will need to become much more specialised. The insurer needs
to find efficient ways of targeting this special group of motorists. [1]

For example, the insurer might advertise in classic car magazines, whereas a standard motor
insurer may well advertise in a daily newspaper. [½]

Other focused ways of selling policies are:


 making special arrangements with affinity groups such as an owners club [½]
 setting up a stall at classic car rallies [½]
 through specialist motor brokers [½]
 through other specialist motor organisations. [½]
[Maximum 2]

Solution X6.3

The Core Reading for this question is covered in Chapter 27, Interpreting accounts.

Comparison can validly be made between the solvency margins of two companies, but there are
many factors that will distort this comparison. These factors should be taken into account where
possible, ie comparison can only be made to a limited extent. [1]

Factors to take into account when comparing the solvency margins

In the text below, ‘less risk’ implies that there is a lower likelihood of insolvency.

Size of margin

Larger margin  stronger company [½]

Smaller margin  more risk [½]

© IFE: 2019 Examinations The Actuarial Education Company


SP7: Assignment X6 Solutions Page 5

Size of margin in relation to variation in outgo

More business  more risk (so bigger margin needed) [½]

Solvency margin is often quoted as a proportion of written premiums [½]

Required margin depends on classes of business (affecting the variability of outgo) [½]

Diversification

Smaller unit size of policies  less risk [½]

Fixed benefit classes  less risk [½]

Greater geographical spread  less risk [½]

Wider spread of business mix  less risk [½]

Suitable spread of investments  less risk [½]

More business usually means wider spread  less risk [½]

Asset values

If assets are valued on a mark-to-market basis, then a company with a bigger proportion of assets
that are temporarily undervalued by the market might be less risky than would appear from the
published returns. [1]

Valuation of liabilities

A company with a conservative basis for valuing liabilities is less risky than would appear.
Important aspects here are: [½]
 suitable provision for IBNR [½]
 suitable allowance for inflation of outstanding claims [½]
 allowance for any trends in claim settlement [½]
 extent of allowance for future investment income. [½]

Unrealised gains

A company may have a large element of unrealised gains, with a contingent tax liability for which
there is not an appropriate reserve. [½]

Reinsurance

The greater the use of reinsurance, the less the risk. [½]

This depends on having appropriate reinsurance. For example, catastrophe cover is particularly
important wherever there is the risk of an accumulation of claims from a single event, or excess of
loss for unlimited liability classes. [½]

The Actuarial Education Company © IFE: 2019 Examinations


Page 6 SP7: Assignment X6 Solutions

Policy conditions and history

A company may be more exposed to risk if policies written in previous years allow an indefinite
period for claiming, ie there is no sunset clause. [½]

Future business

The following additional aspects are all relevant in determining the likelihood of insolvency for a
particular company:
 adequacy of premium rates [½]
 suitability of rating bases (ie to discourage selection) [½]
 stringency of underwriting [½]
 quality of management: claims control, expense control [½]
 quality of management: investment policy, dividend / profit retention policy [½]
 quality of management: new business plans and growth rates. [½]
[Maximum 14]

Solution X6.4

The Core Reading for this question is covered in Chapter 27, Interpreting accounts.

General point

These ratios are based upon published accounts. So the figures will be distorted due to the
strength of the reserving basis and other accounting conventions, such as the treatment of
depreciation and DAC. [½]

For a sale and purchase we would need to consider accounts prepared on a realistic basis. [½]

Growth in premium income

Since earned premium for year X is 14.5, it is reasonable that written premium in year X–1
is 14.0. [½]

Therefore the growth in written premium is as follows:


X–1 to X: 7.1%
X to X+1: 6.7% [½]

This is a slight drop, but we should look at more years to assess trends. [½]

© IFE: 2019 Examinations The Actuarial Education Company


SP7: Assignment X6 Solutions Page 7

Possible reasons include:


 poor awareness [½]
 premiums too expensive. [½]

Claim ratio

This is defined as claims incurred divided by earned premium. We have assumed that all figures
are net of reinsurance. [½]

Year X: 72.4%
Year X+1: 87.1% [½]

On the face of it there is an upward trend, but we should consider:


 The impact of the insurance cycle. We must look at the company’s performance relative
to the market. [½]
 The figures from year X. In this year the claims paid look high but the increase in the
outstanding is negative. If there had been one massive claim that the insurer had been
over reserving for, then the claim ratio would be unreasonably low, since the payment
was less than the estimate. [½]
 Whether there has been a change (ie a deterioration) in underwriting standards. [½]
 How premium rates have changed. [½]
 The impact of large claims and any catastrophic claims. This might be an indication that
the reinsurance programme needs reviewing. [½]

Expense ratio

This is normally defined as expenses paid divided by written premium. [½]

Year X: 10.0%
Year X+1: 10.0% [½]

This is constant. [½]

Looking at incurred expenses divided by earned premium, ie a more realistic basis, we get, also
constant:
Year X: 9.7%
Year X+1: 9.7% [½]

This assumes that the level of DAC shown in the accounts is realistic. If the level of DAC is unduly
prudent (ie too low) then the ratio calculated above will be bigger than a realistic expense
ratio. [½]

The Actuarial Education Company © IFE: 2019 Examinations


Page 8 SP7: Assignment X6 Solutions

Investment return

This is defined as investment income/average assets. [½]

Year X: 13.9%
Year X+1: 6.5% [½]

We should investigate the extent to which capital gains have been allowed for, since generally
investment income is fairly stable but asset values are volatile. So the figures look reasonable if
capital gains and losses have been included. [½]

Taxation

The tax paid is a constant proportion of profits, ie 25% per annum. [½]

Dividends

These are rising slightly, despite the retained profits being negative. The impact of this upon
financial strength may be a concern to the purchaser. [½]

Solvency ratio

This is defined as shareholders’ funds divided by premiums written in the previous year. [½]

So at the start of each year, this figure will be:


Year X: 71.4%
Year X+1: 78.0%
Year X+2: 47.5% [½]

There is clearly a reduction in the latest year. The reason for this is that the premium income is
increasing and the shareholders’ funds are falling. [½]

The company is paying out more in dividends than its post-tax profit, so the shareholders’ funds
are falling. For an expanding company we would expect that dividends paid would be less than
the post-tax profit, so there is additional capital set aside in case the future is worse than
expected. [½]

This reduction in financial strength could eventually lead to:


 a greater chance of future insolvency
 reduced profits due to more reinsurance and a less risky investment policy
 a reduction in confidence by brokers and the stock market
 intervention by the regulators. [½ each]

Each of these factors will be of concern to the potential purchaser. [1]

© IFE: 2019 Examinations The Actuarial Education Company


SP7: Assignment X6 Solutions Page 9

Asset / liability ratio

This is defined as the assets divided by liabilities, net of DAC. At the start of each year these
figures are:
Year X: 17.7%
Year X+1: 22.8%
Year X+2: 14.3% [½]

This ratio increases from X to X+1, due to shareholders’ funds increasing (profitable business
written) and the reserves falling (maybe due to the large claim referred to earlier). [½]

It then falls, due to a reduction in the size of the shareholders’ funds and the reserves
increasing. [½]

Return on capital employed

This is defined as the post-tax profit divided by the capital employed at the start of the year. The
profit figure should include dividend payments, but exclude tax. [½]

Figures for the profit generated each year are:


Year X: 87.0%
Year X+1: 29.1% [½]

The return was very large but has dropped considerably. [½]

The reduction could be due to the volatility of the market value of assets. [½]

These figures will be very sensitive to the strength of the reserving basis, mentioned at the start.
On a realistic basis the return could be significantly higher. [½]

Indication of the length of tail

This is given by the ratio of claims paid to the outstanding claims reserve at the end of the year (or
the reciprocal). [½]

Figures for the profit generated each year are:


Year X: 36.0%
Year X+1: 26.1% [½]

The average length of the tail of the business appears to be about three years. [½]

As the ratio is decreasing it might be thought that the length of the tail is increasing, perhaps due
to a change in the business written. [½]
[Maximum 17]

The Actuarial Education Company © IFE: 2019 Examinations


Page 10 SP7: Assignment X6 Solutions

Solution X6.5

The Core Reading for this question is covered in Chapter 8, Regulation.

(i) Aspects that could be controlled by supervisory authority

Note: ensuring that a company can meet its obligations to policyholders may be very difficult
(impossible) to achieve. However, the following suggestions may help.

Companies must maintain their level of free reserves, including: [1]


 assets: (a) regulations on the assets that may be held (or counted) [½]
(b) place assets with the government or trustees [½]
(c) set bases for valuing assets [½]
 liabilities: restricting the bases for valuing (but might be difficult, or inappropriate in many
cases) [1]
 the requirement for risk-based capital calculations and capital assessment analyses. [½]

Other financial restrictions:


 minimum or prescribed premiums (this ensures sufficient funds coming in, but it is very
difficult to implement especially where individual underwriting is concerned) [½]
 restrictions on certain outgo items (dividends, expense levels) [½]
 statutory levels of reinsurance (possibly with the government) and restrictions on the type
of reinsurance that can be used [½]
 restrictions on discounting. [½]

Underwriting restrictions:
 restrictions on information that may be used in underwriting [½]
 restrictions on the countries where an insurer can write business [½]
 requirements and/or restrictions on policy terms and conditions. [½]

Companies must make information publicly available, such as:


 Prospective: regular business plans for approval by authority (and company can only
operate within limits agreed within the plan) [1]
 Retrospective: detailed reports on the business over the last accounting period(s), a
requirement for reports to be audited and a requirement to produce statements of
actuarial opinion. [1]

Approval required, for:


 types and amount of business written [½]
 new insurance companies, managers, directors and main agents [½]
 internal capital calculations. [½]
[Maximum 8]

© IFE: 2019 Examinations The Actuarial Education Company


SP7: Assignment X6 Solutions Page 11

(ii) Companies cannot meet all obligations

Need to pay legitimate claims from somewhere. Possible approaches:


 Set up a fund financed by companies and/or intermediaries to provide payments to
policyholders when companies fail. [1]
 Run a state insurance company which merges with any companies in danger of becoming
insolvent (bit impractical). [1]
 Run a state reinsurance company that reinsures business of a company in danger of
becoming insolvent. [1]
[Maximum 2]

(iii) Reasons for each suggestion

(a) MSM proportional to technical reserves

+ One of the purposes of the solvency margin is to provide a cushion for fluctuations in the
value of assets or liabilities (technical reserves). As fluctuations are generally proportional
to the amounts, it might be appropriate to have the margin proportional to the amount of
the technical reserves. [1]

(b) MSM proportional to square root of premium income

+ Consistent with statistical theory – the variation in the total claim amount, allowing for
variation in the number of claims, increases roughly in line with the square root of
premium income. [1]
+ Basis does give suitable MSM for new and growing companies. [1]

(c) Statutory basis for calculating technical reserves

+ Necessary / desirable as free reserves are defined as assets less technical reserves. This
basis prevents possibility of a specified MSM being satisfied through weakening the
reserving basis. [½]
+ Aims to provide consistency between companies. [½]
 But it may be very hard to establish appropriate statutory reserving bases, especially for
unusual risks. [½]

(d) Technical reserves + MSM invested in government securities

+ Ensures that the assets are secure (hopefully), so there is no danger of policyholder claims
not being met through investment failure. (But if these were only fixed interest
investments, monetary security would not protect against rampant inflation.) [½]
+ Provides the government with funding. [½]

The Actuarial Education Company © IFE: 2019 Examinations


Page 12 SP7: Assignment X6 Solutions

(e) State-owned reinsurance

+ Reinsurance provides protection for the insurer. With a state-owned reinsurer there
should be no question regarding the failure of reinsurers (which might happen from time
to time if they are not state owned). [1]

Suitability

The suitability or otherwise of each suggestion can be considered against the following criteria.
Note that the final outcome might be a combination of the suggestions.

(1) MSM might be met by company using a weak reserving basis: applies to all suggestions
except (c), and especially for (a).

(2) Some judgement might be appropriate within the reserving basis to allow for peculiar
circumstances and different classes of business (so (c) not ideal).

(3) A new or fast growing company needs a strong MSM; (b) is OK, but (a) is not.

(4) A contracting company (or one writing no new business) needs a margin in case
outstanding claims are greater than expected. So (a) is OK, but (b) is not.

(5) Credibility theory is based on a pure approach, and does not make full allowance for the
vagaries of the real world – so (b) may not be fully appropriate.

(6) Different classes of business may produce different levels of ruin probability – this is not
incorporated in any of the suggestions. For example, a company writing 10 industrial fire
policies may satisfy all of the criteria (a) to (e), but could go bust on a single claim from
any one of the policies.

(7) The solvency margin should make allowance for (i) the valuation basis of the assets, and
(ii) the quality of the assets. Only (d) makes any attempt to do this.

(8) The actual investments should reflect the type and term of the liabilities; (d) is restrictive
in this respect, and probably results in lower investment returns and hence higher
premiums.

(9) None of the bases enforce adequacy of premium rates so a company may have written
unprofitable business for two years before it is spotted that it is insolvent.

(10) The legislation may need to be tailored to suit the class(es) written – there is no mention
of this in (a) to (e).

(11) State-owned reinsurers introduce an element of ‘nationalising the insurance industry’


which may not be conducive to promoting an efficient market driven insurance industry.
Consideration would need to be given as to whether reinsurance would be compulsory,
and, if so, to what extent. Extension of this is state owned insurer (ie, cut out the
middleman).
[1 mark per point]
[Maximum 15]

© IFE: 2019 Examinations The Actuarial Education Company


SP7: Assignment X6 Solutions Page 13

Solution X6.6

This question tests your understanding of Chapter 25, Reinsurance reserving, but also draws on
ideas discussed in earlier parts of the course.

For example, for part (i) you could consider the claims characteristics described in Chapter 2,
Insurance products  background.

For part (ii) you could consider Chapter 12, Data or Chapter 15, Triangulation methods.

For part (iii) you could consider the appropriateness of reserving methods used for direct business
as well for reinsurance business. Therefore, you could look back at Chapter 15, Triangulation
methods and Chapter 16, Stochastic reserving models.

(i) Characteristics of reinsurance recoveries

Claims at this level will be relatively low frequency. [½]

Amounts could be of any size (once they reach the attachment point) and can run to several
millions of pounds. [½]

The claim severity distribution can be expected to be positively skewed, volatile and of uncertain
distribution. [½]

Claims at this level will virtually all be for bodily injury. [½]

Claims events are normally sudden and notified quickly, apart from perhaps whiplash claims
which can take slightly longer (although these would be unlikely to reach £150,000). [½]

Once reported, some claims can take a while to be settled, whilst liability is clarified in the courts,
but most claims are run off within a small number of years. [½]

Some claims may be settled as a series of regular payments as opposed to a lump sum. [½]

As the tail can be long, inflation is important. Most claims are related to salary inflation as court
awards are normally based on lost earnings. [½]

Claims can re-open as injuries develop. [½]

Accumulations can arise due to motorway pileups. [½]

Retrocession may be in place, which could change the characteristics of net claims payments. [½]
[Maximum 5]

(ii) Claims data

The key objective is to split the data into homogenous groups, subject to credibility within each
group. [1]

We would split the data into comprehensive and non-comprehensive, and further by other rating
factors and claim type. [½]

The Actuarial Education Company © IFE: 2019 Examinations


Page 14 SP7: Assignment X6 Solutions

The claims would be split by origin year (probably underwriting year or accident year), in order to
monitor trends. [½]

We would work in quarters if possible since we already have data of that frequency. [½]

We would, if possible, split the data by cedant, or at least by type of cedant (eg small or large). [1]

This is because each cedant could differ in terms of, for example:
 reserving basis
 terms and conditions
 mix of business changing from year to year. [½ each, maximum 1]

We would also want to analyse treaty business separately from facultative business, as there will
be very different experience between the two. [½]

We would look at catastrophe, aggregate and risk excess of loss reinsurance protections
separately. [½]

The actual cedants reinsured may differ from year to year, introducing extra heterogeneity. [½]

We would analyse each layer separately, as claims of different sizes may develop very differently.
[½]

We would look at claims paid triangles and claims incurred triangles separately. [½]

Claims paid at higher layers can often be zero in early development years and so may be
unreliable or very volatile for triangulations. [½]

However, one advantage of claims paid triangles are that they are not distorted by the insurer’s
reserving basis  they are more driven by the legal process and so may sometimes be more
reliable. [1]

We may have delays in claim notification due to brokers not passing on information swiftly, or
due to IT constraints. [½]

One major problem we have is that the census point of £150,000 is too high, because we will be
missing potentially valuable information regarding claims that have not reached the attachment
point. Ideally we would want a census point of at most £75,000. Many of these claims will
develop, with inflation or further information, to over £150,000. Cedants may not notify us if they
don’t believe a claim will ever exceed £150,000. [1½]

They may even under-reserve on purpose so that the number of claims hitting the layer look low
and thus results in a low reinsurance renewal premium. [½]

For reinsurance arrangements where there is an upper limit to the amount that cedants can
claim, we should bear in mind how close any large losses are to being total losses to the layer. An
early reported large loss which is just burning a layer is much more likely to develop adversely
than a large loss which is already close to a total burn. [1]

© IFE: 2019 Examinations The Actuarial Education Company


SP7: Assignment X6 Solutions Page 15

We should watch for clumping of claims at or just below the attachment point, which is the
natural thing to occur when cedants set case estimates. Once a claim exceeds the attachment
point, there is less incentive for the insurer to monitor it further. [1]

We should allow for any changes in attachment levels, reinstatement provisions or other changes
in terms and conditions. [½]

Where claims are infrequent at higher layers, we would also consider using external claims data.
For example: [½]
 inflation [½]
 trends in court awards [½]
 published indices for reinsurance development patterns [½]
 information from competitors, if available. [½]

However, industry benchmarks for reinsurance claims development are particularly rare for
non-proportional covers. [½]

We could instead use overseas benchmarks, but we should be aware of the need to allow for any
differences. [1]
[Maximum 14]

(iii) Reserving methods

We would want to use a number of methods and compare the results, our final estimate being
chosen after having seen and understood the differences. [1]

The chain ladder methods (basic and inflation-adjusted, paid and incurred) should be suitable
since we will have a large volume of data. [1]

Paid methods may be more reliable than incurred methods if cedants’ reserving methodologies
differ or if the cedants change from year to year. [1]

These could perhaps be extended to stochastic methods such as Mack’s method or bootstrapping
techniques. The advantage of this is that the variability and uncertainty we have described could
then be quantified explicitly. [1]

We could increase stability by using the BF method or the Cape Cod method – again based on
paid or incurred data  although the choice of the a priori estimated loss ratio will be important.
We could perhaps use the target loss ratio from the pricing department. [1]

Average cost per claim methods may be suitable if frequency and severity follow very different
trends. [½]

Curve fitting techniques may provide a further method, although their reliability depends on the
suitability of any curves used. [½]

We would want to exclude any major catastrophes from the data and reserve for these
separately. [½]
[Maximum 5]

The Actuarial Education Company © IFE: 2019 Examinations


All study material produced by ActEd is copyright and is sold
for the exclusive use of the purchaser. The copyright is
owned by Institute and Faculty Education Limited, a
subsidiary of the Institute and Faculty of Actuaries.

Unless prior authority is granted by ActEd, you may not hire


out, lend, give out, sell, store or transmit electronically or
photocopy any part of the study material.

You must take care of your study material to ensure that it


is not used or copied by anybody else.

Legal action will be taken if these terms are infringed. In


addition, we may seek to take disciplinary action through
the profession or through your employer.

These conditions remain in force after you have finished


using the course.

The Actuarial Education Company © IFE: 2019 Examinations

You might also like

pFad - Phonifier reborn

Pfad - The Proxy pFad of © 2024 Garber Painting. All rights reserved.

Note: This service is not intended for secure transactions such as banking, social media, email, or purchasing. Use at your own risk. We assume no liability whatsoever for broken pages.


Alternative Proxies:

Alternative Proxy

pFad Proxy

pFad v3 Proxy

pFad v4 Proxy